Você está na página 1de 668

THE LIFE AND TEACHINGS

OF JESUS

“Jesus saith unto him, I am the way, the truth, and the life: no man cometh unto the Father,
but by me.” John 14:6

http://breachrepairers.webs.com/ i
Table of Contents
THE LIFE AND TEACHINGS OF JESUS ........................................................................................................................................... i
Table of Contents................................................................................................................................................................. ii
History of Salvation in the Old Testament: Preparing the Way for Christ................................................... 1
Genesis ................................................................................................................................................................................ 1
Exodus ................................................................................................................................................................................. 7
Leviticus............................................................................................................................................................................ 11
Numbers ........................................................................................................................................................................... 13
Deuteronomy.................................................................................................................................................................. 15
Joshua ................................................................................................................................................................................ 17
Judges................................................................................................................................................................................. 19
Ruth .................................................................................................................................................................................... 20
1 Samuel ........................................................................................................................................................................... 21
2 Samuel ........................................................................................................................................................................... 23
1 Kings ............................................................................................................................................................................... 25
2 Kings ............................................................................................................................................................................... 28
1 Chronicles..................................................................................................................................................................... 30
2 Chronicles..................................................................................................................................................................... 34
Ezra ..................................................................................................................................................................................... 37
Nehemiah ......................................................................................................................................................................... 38
Esther................................................................................................................................................................................. 40
Job ....................................................................................................................................................................................... 41
Psalms................................................................................................................................................................................ 45
Proverbs ........................................................................................................................................................................... 60
Ecclesiastes...................................................................................................................................................................... 64
Song of Solomon ............................................................................................................................................................ 65
Isaiah .................................................................................................................................................................................. 66
Jeremiah............................................................................................................................................................................ 74
Lamentations .................................................................................................................................................................. 80
Ezekiel ............................................................................................................................................................................... 81
Daniel ................................................................................................................................................................................. 86
Hosea.................................................................................................................................................................................. 89
Joel ...................................................................................................................................................................................... 90
Amos................................................................................................................................................................................... 91
Obadiah ............................................................................................................................................................................. 92
Jonah .................................................................................................................................................................................. 92
Micah .................................................................................................................................................................................. 93
Nahum ............................................................................................................................................................................... 94
Habakkuk ......................................................................................................................................................................... 94
Zephaniah ........................................................................................................................................................................ 95
Haggai ................................................................................................................................................................................ 95
Zechariah .......................................................................................................................................................................... 95
Malachi .............................................................................................................................................................................. 97
Course Overview ............................................................................................................................................................... 98
Timeline of the Life of Jesus (2nd) ........................................................................................................................... 98
Historical Background (1st) ...................................................................................................................................... 98
Harmony of the Gospels ............................................................................................................................................. 98
The Life and teachings of Jesus (Birth to transfiguration) .......................................................................... 98
Reading the Gospels ......................................................................................................................................................... 98

http://breachrepairers.webs.com/ ii
Genre .................................................................................................................................................................................. 98
Perspectives .................................................................................................................................................................... 99
Distinctive of Matthew................................................................................................................................................ 99
Distinctive of Mark .................................................................................................................................................... 100
Distinctives of Luke................................................................................................................................................... 101
Distinctive of John ..................................................................................................................................................... 103
Distinctives of Acts .................................................................................................................................................... 104
The Four Gospels ............................................................................................................................................................ 105
Timeline of the Life of Jesus................................................................................................................................... 105
Summary of Chapter 1 ............................................................................................................................................. 107
Summary of Chapter 2 ............................................................................................................................................. 107
Summary of Chapter 3 ............................................................................................................................................. 108
Matthew.................................................................................................................................................................................. 110
Introduction ..................................................................................................................................................................... 110
Author and Title ......................................................................................................................................................... 110
Date.................................................................................................................................................................................. 110
Theme ............................................................................................................................................................................. 110
Key Themes .................................................................................................................................................................. 110
Purpose, Occasion, and Background .................................................................................................................. 111
History of Salvation Summary – Matthew & the Kingdom of God......................................................... 112
The Setting of Matthew ........................................................................................................................................... 120
Outline ............................................................................................................................................................................ 120
Chapter 1 – Genealogy of Jesus ................................................................................................................................. 123
The Genealogy of Jesus ............................................................................................................................................ 124
Chapter 2 ........................................................................................................................................................................... 126
Chapter 3 – Anointing | Baptism of Christ ........................................................................................................... 129
Chapter 4 – Temptations of Christ .......................................................................................................................... 131
Chapter 5 – Law of the Kingdom of God ............................................................................................................... 138
Reading .......................................................................................................................................................................... 139
Chapter 6 ........................................................................................................................................................................... 183
Chapter 7 ........................................................................................................................................................................... 192
Chapter 8 ........................................................................................................................................................................... 203
Reading .......................................................................................................................................................................... 203
Chapter 9 ........................................................................................................................................................................... 207
Reading .......................................................................................................................................................................... 207
Chapter 10 ......................................................................................................................................................................... 212
Reading .......................................................................................................................................................................... 212
Chapter 11 – Prophet’s of God Rejected ............................................................................................................... 216
Reading .......................................................................................................................................................................... 216
Chapter 12 – The Blind, Dumb Demoniac ............................................................................................................ 222
Reading .......................................................................................................................................................................... 222
Chapter 13 – Parables on the Kingdom................................................................................................................. 228
Chapter 14 – Feeding of the Five thousand ......................................................................................................... 247
Reading .......................................................................................................................................................................... 247
Chapter 15 ......................................................................................................................................................................... 252
Chapter 16 ......................................................................................................................................................................... 256
Chapter 17 ......................................................................................................................................................................... 259
Chapter 18 ......................................................................................................................................................................... 262
Child Training .............................................................................................................................................................. 262
Chapter 19 ......................................................................................................................................................................... 267

http://breachrepairers.webs.com/ iii
Chapter 20 ......................................................................................................................................................................... 270
Chapter 21 ......................................................................................................................................................................... 275
The Lord’s Vineyard (33-44) ................................................................................................................................ 279
Chapter 22 ......................................................................................................................................................................... 285
Chapter 23 – Warning to the Scribes and Pharisees........................................................................................ 291
Position of the Scribes and Pharisees (1-12) ................................................................................................. 291
Condition of the Scribes and Pharisees (13-31) ........................................................................................... 293
Solution, Warning, and Final Judgment (32-39) ........................................................................................... 294
Chapter 24 ......................................................................................................................................................................... 296
Chapter 25 ......................................................................................................................................................................... 305
Chapter 26 ......................................................................................................................................................................... 312
Chapter 27 ......................................................................................................................................................................... 321
Chapter 28 ......................................................................................................................................................................... 326
Mark - Jesus Christ as a Servant ................................................................................................................................... 327
Introduction ..................................................................................................................................................................... 327
Author and Title ......................................................................................................................................................... 327
Date and Location ...................................................................................................................................................... 327
Theme ............................................................................................................................................................................. 327
Key Themes .................................................................................................................................................................. 328
Purpose, Occasion, and Background .................................................................................................................. 328
Distinctive Features .................................................................................................................................................. 328
History of Salvation Summary.............................................................................................................................. 328
Outline ............................................................................................................................................................................ 328
Chapter 1 ........................................................................................................................................................................... 330
John prepares the way through Baptism (1-8).............................................................................................. 330
Baptism & Temptation of Jesus in the wilderness (9-13) ......................................................................... 331
Call of Peter, Andrew, James and John (14-20) ............................................................................................. 332
Christ's authority over all diseases and demons (21-34) ......................................................................... 332
Healing of the leper and Christ's fame spreads abroad (35-45) ............................................................ 333
Chapter 2 – Healing of Man with Palsy.................................................................................................................. 336
Healing of the man sick with palsy (1-12)....................................................................................................... 336
The call and feast of Levi Matthew (13-17) .................................................................................................... 338
The question on fasting (18-22) .......................................................................................................................... 339
Plucking corn on the Sabbath (23-28) .............................................................................................................. 339
Chapter 3 ........................................................................................................................................................................... 340
Healing of the withered hand on the Sabbath (1-6) .................................................................................... 340
Multitude from seven regions gathers at the sea of Capernaum (7-12) ............................................. 340
Ordination of the twelve disciples (13-19) ..................................................................................................... 340
Who are my brethren? & Warning against the sin of blasphemy (20-35) ......................................... 341
Chapter 4 – Disciples in the Storm .......................................................................................................................... 342
Parable of the four grounds (1-23) .................................................................................................................... 342
Parables of the full corn in the ear and the mustard seed (24-34) ....................................................... 343
Let us pass over to the other side (35-41) ...................................................................................................... 343
Chapter 5 - Demoniacs of the Gadarenes | Women with Issue of Blood ................................................. 347
Reading .......................................................................................................................................................................... 347
Christ casts out legion into the swine (1-20) ................................................................................................. 347
Resurrection of Jairus daughter and the Issue of Blood (21-43) ........................................................... 350
Chapter 6 ........................................................................................................................................................................... 356
Is this the carpenters Son? (1-6) ......................................................................................................................... 356
Disciples receive power over sicknesses and demons (7-13)................................................................. 356

http://breachrepairers.webs.com/ iv
Herod kills John the Baptist (14-29).................................................................................................................. 356
Feeding the 5,000 (30-44) ..................................................................................................................................... 357
Christ’s walks on water and heals in Gennsaret (45-56) .......................................................................... 358
Chapter 7 ........................................................................................................................................................................... 360
Commandents of God vs. Tradition of men (1-23) ...................................................................................... 360
Demon casts out of Syrophenecian woman (24-30) ................................................................................... 361
Ephatha (Blind and deaf man healed) (31-37).............................................................................................. 361
Chapter 8 – Blind Man near Bethsaida .................................................................................................................. 363
Feeding of the 4,000 (1-9)...................................................................................................................................... 363
Pharisees seek a sign (10-13) ............................................................................................................................... 363
Beware of the leaven of Herod and Pharisees (14-21) .............................................................................. 363
Healing of the blind man with saliva (22-26) ................................................................................................ 364
Who am I? (Confession of Peter) (27-38) ........................................................................................................ 366
Chapter 9 ........................................................................................................................................................................... 368
The coming Kingdom of God (Transfiguration of Christ) (1-13) ........................................................... 368
Cast out through Fasting and Prayer (14-29) ................................................................................................ 368
Who is the greatest? (30-37) ................................................................................................................................ 369
How to enter the kingdom of God (38-50) ...................................................................................................... 370
Summary ....................................................................................................................................................................... 370
Chapter 10 – Conditions of the Kingdom of God ............................................................................................... 371
Is it lawful to put away your wife? (1-12) ....................................................................................................... 371
Little children and the kingdom of God (13-16) ........................................................................................... 371
Rich young ruler (kingdom of God) (17-31)................................................................................................... 372
Let us sit on your right and left hand (32-45) ............................................................................................... 375
Blind Bartimaeus (Son of David) (46-52) ........................................................................................................ 375
Summary ....................................................................................................................................................................... 376
Chapter 11 ......................................................................................................................................................................... 377
Christ’s entry into Jerusalem (1-11) .................................................................................................................. 377
Curse of the fig tree (12-14) .................................................................................................................................. 377
Cleansing the temple (15-18) ............................................................................................................................... 378
Curse fig tree and have faith in God (19-26) .................................................................................................. 378
By what authority do you do these things? (27-33).................................................................................... 378
Chapter 12 ......................................................................................................................................................................... 380
Parable of the vineyard and the heir (1-12) ................................................................................................... 380
Render unto Caesar (Question by Pharisees and Herodians) (13-17) ................................................ 380
Question on marriage and resurrection (Sadducee's) (18-27) .............................................................. 381
Question on first commandment (Scribe) (28-34) ...................................................................................... 381
Son of David, Beware of scribes, Two mites (35-45) .................................................................................. 382
Chapter 13 ......................................................................................................................................................................... 383
Not one stone left for the temple (1-4) ............................................................................................................. 383
Signs of Christ's coming (5-13) ............................................................................................................................ 383
Abomination of Desolation (14-23) ................................................................................................................... 384
Time frame, manner, and purpose of Coming (24-31) .............................................................................. 384
Warning to watch and be ready for Coming (32-37) .................................................................................. 385
Chapter 14 ......................................................................................................................................................................... 386
Chapter 15 ......................................................................................................................................................................... 391
Chapter 16 ......................................................................................................................................................................... 394
Mary, Mary, and Salome tell Peter the Christ is risen (1-8) ..................................................................... 394
Disciples don't believe Mary and the two disciples (9-13) ...................................................................... 395
Gospel commission to believe and cast out demons (14-20) .................................................................. 397

http://breachrepairers.webs.com/ v
Luke - Humanity of Jesus Christ ................................................................................................................................... 398
Introduction ..................................................................................................................................................................... 398
Author............................................................................................................................................................................. 398
Date.................................................................................................................................................................................. 398
Theme ............................................................................................................................................................................. 399
Key Themes .................................................................................................................................................................. 399
Purpose, Occasion, and Background .................................................................................................................. 400
Location ......................................................................................................................................................................... 401
Distinctive Features .................................................................................................................................................. 401
History of Salvation Summary.............................................................................................................................. 401
Outline ............................................................................................................................................................................ 401
Chapter 1 - Gabriel's glad tidings of the birth of John & Jesus ..................................................................... 404
Ordered declaration of things believed among us (1-4) ........................................................................... 407
Gabriel sent to tell Zacharias that Elisabeth will have a son, John (5-25) .......................................... 408
Gabriel sent to tell Mary she will have a son, Jesus (26-38) .................................................................... 411
Elisabeth gives birth to John (57-66) ................................................................................................................ 412
Zachariah's prophecy concerning Jesus & John (67-80) ........................................................................... 413
Chapter 2 - Jesus' birth & visit to Jerusalem ....................................................................................................... 414
Jesus born in Bethlehem during the days of taxing (1-7) ......................................................................... 414
Angels and Shepard's tell good tidings of His birth to many (8-20)..................................................... 415
Jesus circumcised and seen by Simeon and Anna (21-39) ....................................................................... 418
Jesus' growth and visit to Jerusalem (39-52) ................................................................................................. 420
Chapter 3 - Jesus' Baptism & Genealogy ............................................................................................................... 424
John preaches the baptism of repentance | Ministry of John Baptist (1-20) .................................... 424
Jesus' baptism (21-23)............................................................................................................................................. 425
The biological genealogy of Jesus (23-38)....................................................................................................... 426
Chapter 4 - Jesus' temptation and ministry in Galilee .................................................................................... 428
The 3 temptations of Jesus (1-13) ...................................................................................................................... 428
Jesus returns to Galilee & Nazareth (14-30) .................................................................................................. 429
Jesus in Capernaum and departure to the rest of Galilee (31-44) ......................................................... 431
Chapter 5 - Jesus came to call sinners to repentance (Jesus' ministry) ................................................... 433
The call of Peter, James, and John by Gennesaret (1-11) .......................................................................... 433
Jesus heals the leper and His fame goes abroad (12-16) .......................................................................... 437
Jesus heals the man with palsy before Pharisees (17-26) ........................................................................ 437
Call and feast of Levi-Matthew & Parable of new wine and old bottles (27-39) ............................. 437
Chapter 6 ........................................................................................................................................................................... 439
Reading .......................................................................................................................................................................... 439
Disciples of Jesus eat corn on the Sabbath (1-5)........................................................................................... 439
Jesus heals on the Sabbath (Withered hand) (6-12) ................................................................................... 440
Jesus chooses 12 disciples (13-16)..................................................................................................................... 440
Sermon on the mount (Blessings, woes, parable) (17-49) ....................................................................... 442
Chapter 7 – The Centurion | Jesus a Prophet of God........................................................................................ 445
Centurion's faith (1-10) .......................................................................................................................................... 445
Resurrection of widow's son in Nain (11-17) ................................................................................................ 447
John questions Jesus (John more than a prophet) (18-35) ...................................................................... 448
Simon's feast (Mary Magdalene's faith) (36-50) .......................................................................................... 449
Chapter 8 ........................................................................................................................................................................... 450
Parable of the sower (1-21) .................................................................................................................................. 450
Jesus rebukes the sea and the legions of devils (22-40)............................................................................ 451
Jesus heals the issue of blood and raises Jairus' daughter (41-56)....................................................... 452

http://breachrepairers.webs.com/ vi
Chapter 9 - The Kingdom of God?............................................................................................................................ 454
Jesus sends 12 disciples to preach the kingdom of God (1-10) .............................................................. 455
Feeding the 5,000 (11-17) ..................................................................................................................................... 455
Jesus asks Peter who He is (Jesus explains His kingdom) (18-27) ....................................................... 456
Jesus is transfigured (28-36) ................................................................................................................................ 456
Casting out of devil/Jesus to be delivered up/Who is the greatest? (37-50) ................................... 457
Jesus goes to Jerusalem (through Samaria, others want to follow) (51-62) ..................................... 457
Chapter 10 – ..................................................................................................................................................................... 460
Jesus sends out 70 disciples (1-24) .................................................................................................................... 460
How to inherit eternal life (Good Samaritan) (25-37) ............................................................................... 461
Mary and Martha receive Jesus (38-42) ........................................................................................................... 466
Chapter 11 ......................................................................................................................................................................... 470
Pray that God's kingdom comes (Unity) (1-13) ............................................................................................ 470
Satan’s kingdom is divided (14-28) ................................................................................................................... 475
Sign of Jonas the Prophet & Judgment (29-36) ............................................................................................. 476
Woes to Pharisees, Scribes, and Lawyers (37-54) ....................................................................................... 476
Chapter 12 - Jesus as the Judge! ............................................................................................................................... 478
Who Jesus will confess before the angels (1-12) .......................................................................................... 478
Jesus as judge or divider (warning against coveting) (13-21)................................................................ 479
Father's pleasure to give the kingdom (22-40)............................................................................................. 481
Faithful and evil servant (portions divided) (41-53) ................................................................................. 482
Judge among yourselves what is right (54-59) ............................................................................................. 483
Chapter 13 ......................................................................................................................................................................... 484
Repent or perish (Pilate mingled blood-fig tree) (1-10) ........................................................................... 484
Woman healed who had infirmity 18 years (On Sabbath) (10-17) ...................................................... 486
Kingdom of God (Mustard Seed, Leaven, Strait gate) (18-30) ................................................................ 486
Jerusalem left desolate (31-35) ........................................................................................................................... 487
Chapter 14 – Jesus' Great Supper ............................................................................................................................ 488
Jesus heals man with dropsy on the Sabbath (1-6) ..................................................................................... 488
Parable of where to sit at the wedding feast (7-11) .................................................................................... 488
Who to invite to a supper (12-14) ...................................................................................................................... 488
The great supper (15-24) ....................................................................................................................................... 489
Who is worthy to be Christ disciple (25-35) .................................................................................................. 490
Chapter 15 – 3 Parables of the lost ......................................................................................................................... 492
Parable of one lost sheep (1-7) ............................................................................................................................ 493
Parable of the one lost coin (8-10) ..................................................................................................................... 494
Parable of the prodigal son (11-32) ................................................................................................................... 496
Chapter 16 ......................................................................................................................................................................... 500
Parable of the unjust steward (1-13) ................................................................................................................ 500
The law will not fail (14-18).................................................................................................................................. 504
Parable of the Rich man and Lazarus (19-31) ............................................................................................... 504
Chapter 17 – Salvation through faith not sight | Redemption through obedience ............................. 508
Woe to whom offences come (1-4)..................................................................................................................... 508
Increase our faith (5-10) ........................................................................................................................................ 508
Cleansing of the 10 lepers (Faith is increased by obeying) (11-19) ..................................................... 509
Signs of His return (20-37) .................................................................................................................................... 511
Chapter 18 – Prayer & Faith? .................................................................................................................................... 513
Pray continually (regarding your adversaries) (1-8) ................................................................................. 513
Prayer of justification (Pharisee and the Publican, be humble) (9-17) .............................................. 516
How to inherit eternal life (18-30) ..................................................................................................................... 519

http://breachrepairers.webs.com/ vii
Blind man asks for mercy (35-43) ...................................................................................................................... 520
Chapter 19 – Conditions of being a part of God’s house ................................................................................ 524
Story of Zacchaeus (a child of Abraham) (1-10) ........................................................................................... 524
Parable of the Nobleman receiving a kingdom (occupy till I come) (11-27) .................................... 526
Jesus' entry into Jerusalem (28-44) ................................................................................................................... 527
Jesus cleanses the temple the second time (45-48) .................................................................................... 528
Chapter 20 ......................................................................................................................................................................... 529
Pharisees ask what authority (parable of vineyard given to others) (1-19) .................................... 529
Spies question Tribute to Caesar (19-26)........................................................................................................ 530
Sadducees question resurrection and marriage (27-39) .......................................................................... 530
How is Christ David's Son, and beware of scribes (40-47)....................................................................... 531
Chapter 21 - The Second Coming of Jesus ............................................................................................................ 532
Applications ................................................................................................................................................................. 532
Widow with the two mites (1-4) ......................................................................................................................... 532
Persecution & Betrayal of God's people (5-19) ............................................................................................. 532
Days of Vengeance on Jerusalem (20-24) ........................................................................................................ 534
The Second Coming (signs and seasons) (25-33) ........................................................................................ 534
Preparation to be ready for the Second Coming (34-38) .......................................................................... 535
Chapter 22 - Betrayal and Denial of the Son of Man ........................................................................................ 536
Judas makes a covenant to betray Jesus (1-6) ............................................................................................... 537
The Passover supper & who is the greatest (7-30) ..................................................................................... 537
Peter pledges not to deny Christ (31-38) ........................................................................................................ 538
Gethsemane (39-53)................................................................................................................................................. 538
Trial of Jesus and the denial of Peter (54-71) ................................................................................................ 539
Chapter 23 - Jesus before Pilate and Herod ........................................................................................................ 541
Jesus before Pilate the first time (1-7) .............................................................................................................. 541
Jesus before Herod (8-12) ...................................................................................................................................... 542
Jesus before Pilate the second time (13-26)................................................................................................... 542
Jesus goes to Calvary (27-34) ............................................................................................................................... 543
Death and burial of Jesus (35-56) ....................................................................................................................... 544
Chapter 24 - The Resurrection of Jesus................................................................................................................. 548
Mary and others tell disciples Jesus has risen (1-12) ................................................................................. 548
Walk to Emmaus (13-35) ....................................................................................................................................... 548
Jesus in the upper room with His disciples (36-49) .................................................................................... 552
Jesus returns to heaven (50-53) .......................................................................................................................... 552
John........................................................................................................................................................................................... 553
Introduction ..................................................................................................................................................................... 553
Author and Title ......................................................................................................................................................... 553
Date and Place of Writing ....................................................................................................................................... 554
Theme ............................................................................................................................................................................. 554
Purpose, Occasion, and Background .................................................................................................................. 554
Key Themes .................................................................................................................................................................. 554
Distinctive Features .................................................................................................................................................. 555
History of Salvation Summary.............................................................................................................................. 555
Outline ............................................................................................................................................................................ 556
Chapter 1 – Identity of the Prophet of God .......................................................................................................... 557
Reading .......................................................................................................................................................................... 557
The Word, Life and Light (1-5)............................................................................................................................. 557
John the Baptist to bear witness (6-36) ........................................................................................................... 559
John's two disciples abide with Jesus (37-42) ............................................................................................... 562

http://breachrepairers.webs.com/ viii
The calling of Philip (43-45) ................................................................................................................................. 563
Nathanael (46-51) ..................................................................................................................................................... 563
Chapter 2 – The Miracle at Cana | Cleansing of the Temple ......................................................................... 564
Jesus’ first miracle – the marriage at Cana (1-11)........................................................................................ 564
Jesus, His mother, brethren and disciples go down to Capernaum (12-13) ..................................... 567
The beginning of Jesus’ ministry (14-17) ........................................................................................................ 567
The Jews asks for a sign and Jesus speaks of the temple of His body (18-22) ................................. 568
Jesus knows what is in man (23-25).................................................................................................................. 569
Chapter 3 – Nicodemus | Ministry of John (Conditions of Salvation-In reach) .................................... 570
Jesus’ interview with Nicodemus (1-21).......................................................................................................... 570
Jesus and His disciples go into the land of Judea and baptize (22) ....................................................... 577
John the Baptist is baptizing in Aenon (23-24) ............................................................................................. 577
John testifies of Jesus again – He must increase, I must decrease (25-36) ........................................ 577
Chapter 4 – Women at the Well | the Nobleman (Message to the Gentiles-Outreach)...................... 579
Jesus departs because of the issue of baptism between Himself and John (1-3) ............................ 579
Jesus and the Samaritan woman (4-30) ........................................................................................................... 579
The meat of Jesus – to do the will of the Father and to finish His work (31-38) ............................. 583
Jesus abides two days with the Samaritans and many more believe (39-42) .................................. 584
Jesus’ second miracle in Galilee – He heals the son of a noble man by speaking (43-54) ........... 585
Chapter 5 – Healing at Bethesda .............................................................................................................................. 588
Jesus heals the cripple man at Bethesda (1-9) .............................................................................................. 588
The cripple man knows not who healed him so he cannot tell the Jews (10-13) ........................... 590
The Jews find out it is Jesus and seek to persecute and slay Him for healing on the Sabbath (14-
16) .................................................................................................................................................................................... 591
Jews seek to kill Jesus more because He calls God His Father (17-18) ............................................... 591
Jesus explains His relationship between Himself and the Father (19-47)......................................... 591
Chapter 6 – The True Bread ....................................................................................................................................... 594
Reading .......................................................................................................................................................................... 594
Jesus feeds the multitude and they testify that He is a prophet (1-14)............................................... 594
Jesus walks on the sea and brings the disciples boat over to Capernaum (15-21) ........................ 595
Jesus exhorts the multitude to labour for the meat that does not perish (22-29) ......................... 595
Jesus introduces Himself as the bread of life, the living bread (30-52) .............................................. 596
Jesus exhorts them to eat his flesh and drink his blood (53-65) ........................................................... 597
Separation of true and false disciples (66-71) .............................................................................................. 598
Chapter 7 ........................................................................................................................................................................... 600
Chapter 8 – Women taken in Adultery .................................................................................................................. 604
Chapter 9 – The Man Born Blind .............................................................................................................................. 611
Chapter 10 – The true Shepard ................................................................................................................................ 617
Chapter 11 – The Ressurection and the Life ....................................................................................................... 624
Chapter 12 - ...................................................................................................................................................................... 627
Chapter 13 ......................................................................................................................................................................... 633
Chapter 14 ......................................................................................................................................................................... 638
Chapter 15 – The true Vine ........................................................................................................................................ 641
Chapter 16 – Coming of the Comforter ................................................................................................................. 644
Chapter 17 – Prayer of Unity ..................................................................................................................................... 646
Chapter 18 ......................................................................................................................................................................... 648
Chapter 19 ......................................................................................................................................................................... 652
Chapter 20 - Resurrection of Jesus ......................................................................................................................... 655
Chapter 21 – Peter at the Sea of Tiberias ............................................................................................................. 657

http://breachrepairers.webs.com/ ix
History of Salvation in the Old Testament: Preparing the Way
for Christ
(The notes in this feature are identified by single verses only, for easy cross-reference with the
main study notes on Bible-text pages. However, many of these notes apply to more than just the one
verse by which they are identified. Directions for the reader to “see note on” another verse refer
only to notes within this feature, not to the main study notes on Bible-text pages.)

Genesis
After God creates a world of fruitfulness and blessing, Adam's fall disrupts the harmony. God
purposes to renew fruitfulness and blessing through the offspring of the woman (3:15). Christ is the
ultimate offspring (Gal. 3:16) who brings climactic victory (Heb. 2:14–15). Genesis traces the
beginning of a line of godly offspring, through Seth, Enoch, Noah, and then God's choice of Abraham
and his offspring (Gen. 12:2–3, 7; 13:14–17; 15:4–5; 17:1–14; 18:18; 22:16–18; 26:2–5; 28:13–15).

1:1 God's act of creation is the foundation for the entire biblical history. A considerable number of
passages refer back to creation (e.g., Psalms 8; 104; 148; John 1:1–3; 1 Cor. 8:6; Col. 1:15–17; Heb.
1:2; 11:3; 1 John 1:5–7). All the rest of the Bible depends indirectly on it.

1:3 God speaks, and it is done. The centrality of the word of God in the acts of creation anticipates
the deeper truth given in John 1:1, that the second person of the Trinity is the Word.

1:3 God created physical light. The Bible also says that God is light in a moral and spiritual sense (1
John 1:5). By God's design, the physical aspects of creation can serve as vehicles for developing
themes about God and his salvation. Jesus is “the light of the world” (John 8:12).

1:26 The divine Son is “the image of the invisible God” (Col. 1:15). Man was created in a way that
reflects the imaging relation among the Persons of the Trinity. The redemption of man from the fall
and sin includes re-creation (2 Cor. 5:17), his being “created after the likeness of God in true
righteousness and holiness,” in the image of Christ (Eph. 4:24).

1:28 God created a permanent order of creation. But he also intended a development in which man
would play a central role. Because Adam failed and fell into sin, Christ came as the last Adam to
achieve dominion (see 1 Cor. 15:22, 45–49; Eph. 1:21–22).

1:31 Sin is a later intrusion into an originally good creation. It is not inherent in the world, and so it
can be completely removed when God achieves his purposes in the consummation (Rev. 22:3–5).

2:2 God rested from his works of creation. But he continues to work in providence and (after sin
enters) in redemption. See John 5:17. As human beings we look forward to entering into God's rest
(Heb. 4:4, 9–11).

2:3 Man imitates the pattern of God's work and rest in the sabbath cycle of days (Ex. 20:8–11) and
years (Leviticus 25).

2:7 God has life in himself and imparts life to his creatures. The impartation of physical life
anticipates the impartation of spiritual life (John 1:4; see 1 Cor. 15:45). Life is in the Son (John 5:21,
26; 1 John 5:12) and comes to us through the Spirit (John 3:5).

http://breachrepairers.webs.com/ 1
2:8 The garden of Eden and paradise reminds us of what we have lost (Joel 2:3) but also of what
will yet be renewed in the world to come (Isa. 51:3; Rev. 22:1–3).

2:9 After the fall, the tree of life was barred to man (3:24). But God promises fruitfulness to those
who know him (Ps. 1:3) and to those who obtain wisdom (Prov. 3:18). Eternal life is obtained in
Christ (John 5:24), and free access to the tree of life reappears in the consummation (Rev. 22:2).

2:24 Divorce is a deviation from God's design in creation (Matt. 19:4). The marriage relationship
anticipates the relation of Christ to the church (Eph. 5:22–33). See Overview of the Bible,
concerning Christ as the last Adam.

3:1 Later Scripture indicates that Satan worked through the serpent (Isa. 27:1; Rev. 12:9). He was
defeated by Christ's work on the cross (Heb. 2:14–15), and will be utterly destroyed in the events
leading to the consummation (Rev. 20:7–10).

3:4 Throughout history Satan is engaged in deceiving (2 Thess. 2:9–12; Rev. 12:9) and casting
doubt on the word of God. When tempted by Satan, Christ rejected his lies (Matt. 4:1–11). In spite of
Satan's attacks, the word of God will stand forever (Ps. 119:89; Matt. 24:35).

3:8 God appears and judges Adam and Eve, anticipating the final day of judgment in Christ (John
5:22). Because of the sacrificial work of Christ, judgment can be tempered with mercy on those who
belong to Christ.

3:15 The offspring of the woman who inflicts decisive defeat on the serpent is Christ (Heb. 2:14).
But earlier in time, within the OT, there are partial defeats through people who prefigure Christ and
foreshadow the final conflict.

3:24 When Christ opens the way to eternal life, the barring of the way to life is removed (John 14:6;
Heb. 10:19–22; Rev. 22:2).

4:26 The line of Seth appears to be a more godly line, corresponding to the promise of the offspring
of the woman (3:15), while Cain and his descendants correspond more to the offspring of the
serpent. The line of Seth ultimately leads to Christ (Luke 3:38).

5:5 Death is a repeated, grim occurrence, reminding us of the reality of the curse (2:17; 3:19) and
the need for God in mercy to provide a final remedy for death through Christ (John 11:25–26; Rev.
1:18; 21:4).

5:24 Enoch's walk with God makes him an early example of faith (Heb. 11:5–6), and his being taken
by God without dying anticipates the eternal resurrection life that Christ gives (Rom. 8:11).

6:9 For Noah's faith, see Heb. 11:7. Noah by his righteousness saved not only himself but his family,
prefiguring the righteousness of Christ by which Christ saved his spiritual family.

6:18 God promises in a covenant (see Overview of the Bible) to save Noah, prefiguring the new
covenant in Christ by which we receive eternal salvation (1 Cor. 11:25; Heb. 10:15–18).

7:23 The flood brought a whole world to an end (2 Pet. 2:5; 3:6). It prefigures the final judgment,
which ends the present heavens and earth and brings a new world (Rev. 21:1). God preserves those

http://breachrepairers.webs.com/ 2
who belong to Christ, the final Noah.

8:13 The emerging of a new world prefigures the creation of the new heaven and the new earth
(Rev. 21:1–4; see 2 Pet. 3:5–7).

9:7 God repeats the command given to man in 1:28. Noah is a new head or representative for
humanity, prefiguring Christ, who will be the final head of the new humanity (1 Cor. 15:45–48). All
those descending from Noah are privileged for his sake.

9:11 In a covenant God guarantees to all mankind blessings that come through Noah. He shows
mercy, based on sacrifice (8:21), pointing forward ultimately to the mercy that comes through the
sacrifice of Christ (Heb. 10:12).

10:32 All the nations of the world are encompassed in the plan of God. He chooses Abram alone
(12:1–3), but eventually “all the nations of the earth shall be blessed in him” (18:18; see 12:3; Rev.
5:9).

11:4 Babel, and later Babylon (Revelation 17–18), is the quintessential worldly city, where man
tries to exalt himself to the position of a god. It contrasts with the holy city of God's people, whose
name is made great not through their prideful self-exaltation but by the power of God (Gen. 12:2;
Rev. 21:2).

12:1 God will give Abram a great name, in contrast to the self-exalting desire in Babel (11:4). The
choice of Abram narrows down the line of the offspring of the woman (3:15) to Abram's offspring.
Ultimately, Abraham is great as a progenitor of Christ (Rom. 9:5).

12:2 God's promise is reiterated and expanded as time passes (13:14–17; 15:4–5; 17:1–14; 18:18;
22:16–18; 26:2–5; 28:13–15; 35:10–12).

12:3 The inclusion of all the families of the earth anticipates the spread of the gospel and salvation
in Christ to the ends of the earth (Matt. 28:18–20; Acts 1:8; Gal. 3:8).

12:7 God's promise has a short-range fulfillment when the nation of Israel conquers Canaan under
Joshua (Josh. 21:43; see 1 Kings 4:21). Ultimately the offspring narrows down to Christ (Gal. 3:16),
whose dominion extends not only over the land of Canaan but over all the world (Matt. 28:18). The
land of Canaan prefigures the eternal inheritance of the world in Christ (Heb. 4:1–11; 11:10, 13–
16). In Christ believers are the offspring of Abraham (Gal. 3:7, 29).

13:15 God confirms and expands his promise to Abram (see notes on 12:1; 12:2; and 12:7).

14:18 Melchizedek, a priest and king, prefigures Christ's priesthood (Heb. 7:1–8:6).

15:6 Abram's trust in God is the model for Christians' trust in God's promises in Christ (Gal. 3:6–9).
Righteousness is “counted” or reckoned, not on the basis of our achievement, but because in faith
we look to God who supplies righteousness in Christ (Rom. 4:5–9; 2 Cor. 5:21; Gal. 3:6).

15:17 The flame, symbolizing God, passes between the pieces, symbolizing that God himself will
bear the penalty if the promise is broken. Ultimately, Christ bears the penalty for our disobedience.

http://breachrepairers.webs.com/ 3
16:10 Because of the line of chosen offspring, leading to Christ (Gal. 3:16), some blessings overflow
and extend even to collateral descendants like Ishmael.

16:13 Hagar perceives that the Lord has spoken to her, which implies that “the angel of the Lord” is
divine. Some think that this is a pre-incarnate appearance of Christ. Christ is the final, divine
messenger of the covenant (Mal. 3:1) who is anticipated in this scene.

17:4 The multiplication of the nation of Israel represents the proximate fulfillment of God's
promise (Ex. 1:7). Those who place their trust in Christ, the offspring of Abraham (Gal. 3:16), now
become sons of Abraham (Gal. 3:6–9), so that ultimately all the multitude of the saved (Rev. 5:9)
have Abraham as father (Rom. 4:17–18).

17:10 Circumcision symbolizes the covenant relation to God, which demands holiness. It is fulfilled
in Christ's purification of believers (Col. 2:11).

18:2 Two of the “men” turn out to be angels (19:1), while the third is the Lord (18:22). The
appearance of God in human form anticipates the incarnation of the Son (John 1:1–18).

18:10 The miraculous birth of a son according to the power of God's word anticipates later
instances where God's word overcomes a “dead” womb and brings new life: 25:21; 30:22; 1 Sam.
1:20; Isa. 54:1. The pattern culminates in the virgin birth of Christ (Luke 1:35), and has relevance
for understanding God's sovereignty in election (Rom. 9:8–9).

18:24 Abraham's limited intercession fails to spare Sodom. Christ's perfect intercession always
succeeds (Heb. 7:23–25).

19:16 Though Lot is a mixed character who makes compromises, God saves him and his family,
prefiguring his mercy in eternal salvation (2 Pet. 2:7–9).

19:24 The destruction of Sodom and Gomorrah prefigures eternal judgment (2 Pet. 2:6, 9–10; Rev.
14:10–11).

20:6 Even though Abraham misuses her, God in mercy preserves Sarah, who embodies the line of
holy offspring leading to Christ.

21:2 The miraculous birth of Isaac, the special offspring of promise, prefigures the coming birth of
Christ, in accordance with all the promises of God.

21:4 Circumcision represents purification and holiness, anticipating the purity of Christ (Luke 2:21;
3:22; Col. 2:11; see Gen. 17:10).

21:10 The distinction between the miraculous son of promise and the son from human planning
prefigures the distinction between the church and natural descendants of Abraham (Gal. 4:30).

22:3 Abraham demonstrates the reality of his faith in action, serving as a model for how our good
works demonstrate our faith (James 2:18–24).

22:8 Isaac comes near to being sacrificed, but God provides a substitute. Ultimately God will
sacrifice his only Son, who dies in our place (Gal. 3:13, 16). The ram prefigures the sacrifice of

http://breachrepairers.webs.com/ 4
Christ.

22:16 Abraham's willingness to sacrifice his son leads to great blessing to his offspring. God's
sacrifice of his only Son leads to even greater blessings to Christ's spiritual offspring (Rom. 5:8–11;
Heb. 6:13–14).

23:19 Abraham takes care about Sarah's burial, expressing thereby his faith in God's promise that
he will possess the land. The fact that the land is not theirs during Sarah's or Abraham's earthly life
points forward to the resurrection of the dead (Heb. 11:13–16).

24:4 The marriage of Isaac is important, because he is the offspring of promise through whose
offspring the world will be blessed. The special provision of a wife for Isaac prefigures God's
offspring of promise, Christ, receiving a bride, the church (Rev. 19:7).

25:23 Jacob the chosen one and Esau the one not chosen prefigure the age-long struggle between
the chosen people and their adversaries (Mal. 1:2–3; Rom. 9:10–13). The principle applies in the OT
to Israel and in the NT to the church.

26:28 Abimelech's respect for Isaac prefigures the salvation of the nations through Abraham's
offspring in Christ (18:18).

27:35 God carries out his sovereign purpose of confirming Jacob as the chosen line of the offspring
of Abraham (12:7; 25:23), in spite of Isaac's intent to bless Esau and in spite of the sinfulness in
Jacob's deceit.

28:12 The opening of access to heaven anticipates Christ, who opens access permanently (John
1:51; Heb. 10:19–20).

29:25 Even in the midst of trickery God sovereignly works to give Jacob wives, through whom he
will fulfill the promise to multiply Abraham's offspring (15:5).

30:1 In the midst of sordid competition between Leah and Rachel, God sovereignly fulfills the first
stage of his promise to multiply Abraham's offspring (12:2; 15:5; 17:5; 26:4; 28:14).

31:24 God protects Jacob, fulfilling his earlier promise (28:13–15) and protecting the line of chosen
offspring leading to Christ (Gal. 3:16).

32:24 God appears in human form, anticipating the incarnation of Christ.

33:4 God delivers Jacob and his family from a feared attack by Esau, fulfilling his promise to Jacob
and his offspring (28:14–15) and protecting the offspring leading to Christ.

34:9 Though Simeon and Levi are later criticized for their deceit and violence (49:5–7), God uses
them in preserving the line of holy offspring from intermarriage (see Deut. 7:3), thus protecting the
line until the coming of Christ the final offspring (Gal. 3:16).

35:10 God confirms earlier promises to Abraham and his offspring (see note on 12:2).

http://breachrepairers.webs.com/ 5
36:1 The record of collateral, rejected offspring (25:23) is given before continuing with the record
of the line leading to Christ (Gal. 3:16).

37:7 Prophetic dreams concerning God's plan for the offspring of promise foreshadow the final
prophetic unveiling of God's purposes through Christ.

37:20 Joseph, who is to be the key deliverer of God's people, has a scrape with death, and is finally
glorified (41:41), foreshadowing the suffering and glorification of Christ the final deliverer.

38:29 In spite of unrighteous sexual behavior by several males, God brings about his own purpose
of continuing the offspring leading to Christ (Matt. 1:3).

39:9 Joseph, in contrast to Adam and Eve, firmly rejected temptation, anticipating Christ's rejection
of temptation (Matt. 4:1–11; 16:23).

40:23 The trials of Joseph, testing his faith, anticipate the trials that come to Christ as man (Matt.
4:1–11), and that come to disciples of Christ (Acts 14:22; 1 Thess. 3:4).

41:36 Through prophetic gifts given by God, Joseph is able to save from famine not only Jacob and
his family, but Egypt. He foreshadows Christ, whose prophetic teaching and suffering bring eternal
salvation both to Jews and to Gentiles. (See 18:18.)

42:9 God works according to his plan, which was already revealed in Joseph's dreams (37:5–9). God
cares for the line of offspring leading to Christ (3:15; Gal. 3:16).

43:9 Judah offers himself as a substitute, prefiguring the substitution of Christ the offspring of
Judah.

44:33 See note on 43:9.

44:29 Salvation through Joseph includes not only rescue from famine, but a change of heart in the
brothers, compared to their earlier envy and violence toward Joseph. The change prefigures the
change of heart that Christ works through the Spirit (John 3:3–8).

45:15 Reconciliation springs from forgiveness, prefiguring God's reconciliation and forgiveness in
Christ.

46:4 God delivers the entire family from famine and promises permanent care, anticipating both
the exodus from Egypt (Exodus 1–14) and the subsequent generations leading to Christ.

47:6 Through Joseph's deliverance abundant blessings come to his family, prefiguring the blessings
of deliverance in Christ.

48:5 The transformation of one tribe (Joseph) into two further illustrates the fruitfulness of
blessing to the line of offspring that God has chosen and blessed.

49:10 At this early point God already reveals that through Judah will come a line of kings, leading
finally to Christ the great, eternal king (Matt. 1:1–16).

http://breachrepairers.webs.com/ 6
50:20 God uses even evil to work out his good purposes, foreshadowing the time when he will
bring the supreme good, namely, eternal salvation, out of the wicked actions of the men who
condemned and crucified Jesus (Acts 2:23; 4:25–28).

50:24 God's promises stand firm through generations (12:7; 15:13–14). His faithfulness is
expressed climactically in Christ (2 Cor. 1:20).

Exodus
Through Moses God redeems his people from slavery in Egypt, prefiguring Christ's eternal
redemption of his people from slavery to sin.

1:7 The multiplication of the people fulfills God's promise to multiply Abraham's descendants (Gen.
15:5) and to bless the world through them (Gen. 18:18), specifically through Christ (Gal. 3:8).

1:13 Bitter suffering precedes release, symbolizing that suffering under sin precedes the
deliverance from sin in Christ.

2:10 Moses, the special agent for God's deliverance, has his life preserved, anticipating the rescue of
baby Jesus from Herod's murders (Matt. 2:13).

2:15 God brings deliverance through his power and in his way, through the weakness of the cross,
not through merely human impulses for justice (1 Cor. 1:25).

3:5 The overwhelming holiness of the presence of God anticipates the presence of God in Christ's
incarnation.

3:12 The commissioning of Moses by God's word and God's power prefigures the commissioning of
Christ for his work (Matt. 3:17).

3:14 The name “I am” anticipates the “I am” sayings of Jesus (see John 8:58), which show his deity.

4:13 Moses' reluctance points forward ultimately to the need for a divine deliverer, Jesus Christ.

5:2 Pharaoh's refusal to recognize the true God prefigures the resistance of people to Christ's
claims, even though miracles supported his claims.

6:8 The mention of the patriarchs (see Gen. 12:7) shows the faithfulness of God and the continuity
of his purposes over time. This faithfulness comes to ultimate fruition with the sending of the Son.

7:17 The plagues on Egypt foreshadow the plagues preceding the second coming (Rev. 11:6).

9:16 God uses even those who resist his will, prefiguring his use of Herod and Pilate (Acts 2:23).

10:4 The locusts prefigure the judgments associated with the day of the Lord (Joel 1–2; Rev. 9:1–
11).

11:5 The plague of death reminds us that the wages of sin is death (Rom. 6:23). Only through the

http://breachrepairers.webs.com/ 7
death of God's Son are we delivered.

12:6 Deliverance through the blood of a lamb prefigures the coming of the Lamb of God to obtain
final salvation through his death (John 1:29).

12:46 Because Jesus is the fulfillment of the Passover lamb (1 Cor. 5:7), it is fitting that none of
Jesus' bones were broken (John 19:36).

13:3 We now look back to the final Passover in which Christ brought eternal salvation from sin (1
Cor. 5:7), and we remember it in the Lord's Supper (1 Cor. 11:23–26).

14:19 God's special presence in the cloud prefigures his presence in Christ, who is our protection
and refuge against all the attacks of Satan.

14:22 The people go down symbolically into death and come up alive, prefiguring the reception of
resurrection life through Christ (see Rom. 6:4; 1 Cor. 10:2).

15:2 Praise for God's salvation anticipates the songs of praise for Christ's final work of salvation
(Rev. 5:9–14; 15:3).

15:17 The conquest of Canaan prefigures the entrance into the final sanctuary of God's presence,
mediated by Christ (Heb. 10:19–20; Rev. 21:22).

16:4 Manna prefigures Christ the bread of heaven, who gives eternal life (John 6:31–35).

16:18 The sufficiency of the manna prefigures the sufficiency of Christ to meet every need of his
people (Phil. 4:19).

17:6 God providing water after striking the rock prefigures Christ, who is stricken to provide the
water of eternal life (John 4:14; 19:34).

18:18 The limitations of Moses prefigure the need for Christ, the divine judge, and Christ's
appointment of shepherds under him (elders) to carry out his will (1 Pet. 5:1–4).

19:6 The privileges of Israel prefigure the higher privileges of the NT church (1 Pet. 2:9–10), won
through Christ's redemption (Heb. 10:10).

19:12 The threat of death illustrates the impossibility of sinful people approaching a holy God. The
impossibility is overcome only through the sacrifice and mediation of Christ (Heb. 10:19–20).

20:2 Christians now obey God's commandments because he has brought us out of sin and death
(Rom. 13:9; Col. 1:13; Rev. 1:5–6).

20:11 The celebration of the Sabbath looks back to creation (see notes on Gen. 2:2 and 2:3), back to
redemption from Egyptian slavery (Deut. 5:15), and forward to final rest through faith in Christ
(Heb. 4:1–11).

20:13 The Ten Commandments are deepened through Jesus' teaching (Matt. 5:17–48) and fulfilled

http://breachrepairers.webs.com/ 8
in Jesus' perfect righteousness (Heb. 4:15; 5:9).

21:2 The ordinances concerning slavery anticipate our being freed from slavery to sin and
becoming slaves to Christ (Rom. 6:20–22; 1 Cor. 7:22).

21:12 The principles of retribution and restitution, though they hedge in sin and give partial
remedies, do not bring a perfect kingdom, but look forward to the perfection of the kingdom of
Christ (Isa. 9:6–7; Matt. 5:38–48).

23:1 The truthfulness of God, coming to its climax in Christ, is to be reflected in truthfulness
displayed to fellow human beings, and the compassion and justice of God is to be reflected in
treatment of fellow humans.

24:8 Consecration through blood prefigures consecration through the blood of Christ (Heb. 9:18–
26).

24:11 Fellowship with God prefigures our seeing God in the face of Jesus Christ (John 14:9).
Christians enjoy fellowship with God in Christ, who is the food of eternal life (John 6:53–58),
symbolized in the Lord's Supper and consummated in the final feast (Rev. 19:9; 22:4).

25:8 The making of a dwelling place anticipates Solomon's temple (1 Kings 6) and prefigures God's
dwelling with humanity in Christ (Matt. 1:23; John 2:19–21; Rev. 21:22), in the church (1 Cor. 3:16;
Eph. 2:19–22), in the individual Christian (1 Cor. 6:19), and in the consummation (Rev. 21:3, 22–
27). The actual construction of the tabernacle is described in Exodus 36–39.

25:22 God's meeting with and speaking to his people prefigures his intimacy and communion with
believers in Christ (John 15:4).

25:30 Bread expressing fellowship with God prefigures Jesus feeding us as the bread of life (John
6:35, 52–58).

25:37 The provision of light in the presence of God prefigures Jesus as the light of the world (John
1:4–9; 3:19–21; 8:12; 9:5).

25:40 The tabernacle is a shadow or copy of the heavenly, final dwelling of God, as indicated in Heb.
8:5. The symbolism in the tabernacle therefore consistently prefigures Christ and the church (see
note on Ex. 25:8).

26:33 The curtain bars access to all except the specially qualified high priest (Leviticus 16),
prefiguring that only Christ can open the way to God (Heb. 9:7–14; 10:20).

27:1 Access to God is only through sacrifice on the altar (Lev. 4:10), prefiguring the necessity of the
sacrifice of Christ (Heb. 9:12–14).

27:9 The hangings of the court erect one more barrier to approaching God, thereby emphasizing his
holiness. See note on 26:33.

28:2 The external holiness and beauty of the priest prefigures the perfect holiness of Christ (Heb.
7:23–8:6).

http://breachrepairers.webs.com/ 9
29:1 The priests, being sinful, need atoning sacrifice for themselves, contrasting with the perfection
of Christ's priesthood (Heb. 7:26–28).

30:1 Burning incense represents intercessory prayer (Rev. 5:8), prefiguring Christ's intercession
(Heb. 7:25).

30:16 Atonement money prefigures Christ's buying us at the price of his own blood (1 Pet. 1:18–
19).

30:20 Washing prefigures cleansing from sin in Christ (Zech. 13:1; 1 Cor. 6:11).

31:3 The giving of the Spirit prefigures Christ's building the church through the Spirit (Matt. 16:18;
1 Cor. 14:12; Eph. 2:20–22). The building of the church is based on Christ's resurrection through
the Spirit (John 2:19–21; Rom. 8:11). See note on 1 Kings 7:14.

32:12 Moses' intercession prefigures the intercessory prayers of Christ (Heb. 7:25).

32:32 Moses offers himself as a substitute, prefiguring Christ's substitutionary death (Heb. 10:10).

33:19 God as sovereign works his will in election (Rom. 9:15).

33:22 Moses as sinful must be shielded from the full weight of God's holiness, prefiguring Christ's
shielding us from the wrath of God (Rom. 5:9–11).

34:9 God's mercy prefigures the mercy given in Christ (Rom. 4:8).

35:21 The willingness of the people prefigures the willingness of Christ's self-giving sacrifice (John
10:18), and then the willingness that he works in us to be used by God (Rom. 12:1; 2 Cor. 8:9–15;
9:7, 13–15).

36:10 The construction exactly according to God's design (26:1–6; see 39:42) prefigures the
construction of the church according to God's design (Eph. 4:11–16) and the construction of the
new world (Rev. 21:2).

37:1 The construction matches 25:10–22. See note on 25:22.

37:10 The construction matches 25:23–30. See note on 25:30.

37:17 The construction matches 25:31–39. See note on 25:37.

37:25 The construction matches 30:1–10. See note on 30:1.

38:1 The construction matches 27:1–8. See note on 27:1.

38:8 The construction matches 30:17–21. See note on 30:20.

38:9 The construction matches 27:9–19. See note on 27:9.

http://breachrepairers.webs.com/ 10
39:1 The garments match 28:1–43. See note on 28:2.

40:34 See the parallel in 1 Kings 8:10–11. The filling of the tent with God's glory prefigures the
fullness of the Spirit in Christ (Matt. 3:16–17; John 1:14; 3:34–35) and in the church (Acts 2:3–4; 1
Cor. 3:16).

Leviticus
The requirement of holiness points to the holiness of Christ (Heb. 7:26–28). The sacrifices prefigure
the sacrifice of Christ (Heb. 10:1–10).

1:9 The offering of the whole sacrifice to God prefigures Christ's giving of his whole self (Heb. 10:5–
10). The whole sacrifice ascends in smoke, prefiguring the ascension of Christ (Heb. 9:24).

2:1 The offering of the fruitfulness of the land prefigures the honor given to God through the
fruitfulness of Christ (John 13:31–32; 1 Cor. 15:23).

3:1 Most of the peace offering is eaten by the worshiper (7:15–16), signifying fellowship with and
blessing from God. It is fulfilled in Christ's reconciliation and giving himself as food (John 6:52–57;
Rom. 5:9–11).

4:2 The promise of forgiveness is fulfilled in Christ's giving himself as a sacrifice for sin (Rom. 8:3;
Heb. 10:1–10).

4:12 The position outside the camp prefigures Christ's crucifixion outside Jerusalem (Heb. 13:11–
14).

5:1 Sins of falsehood and sins against holiness are forgiven in anticipation of Christ's work in
holiness (Heb. 9:23–26; 10:11–20).

6:13 The continuation of the altar fire indicates the insufficiency of repeated sacrifices (Heb. 10:1–
4), in contrast to the sufficiency of Christ's sacrifice (Heb. 10:10) and intercession (Heb. 7:25).

7:20 Fellowship with God and with the things of God requires holiness, prefiguring the holiness of
Christ purifying us (Heb. 10:10; 12:14).

8:1 For the instructions for consecration, see Exodus 29.

8:30 Consecration through oil and blood prefigures purification from sin through the Spirit and the
blood of Christ (Heb. 9:19–26; 1 Pet. 1:2).

9:24 God's acceptance of the offering prefigures his acceptance of the sacrifice of Christ (Heb. 9:13–
14).

10:2 The rejection of human inventions prefigures the fact that Christ is the only way to God (John
14:6; Acts 4:12).

11:45 Separation from uncleanness symbolizes separation from sin in order to be intimate with

http://breachrepairers.webs.com/ 11
God. It prefigures Christ's work bringing holiness (Heb. 7:26; 10:10).

12:7 Human birth is contaminated with sin ever since Adam. The remedy is in new birth (John 3:3–
8) through Christ (Rom. 5:15–21).

14:2 Cleansing prefigures Christ's work of cleansing from sin (Luke 5:12–14; Heb. 9:9–14).

15:1 Disorders of the body symbolize the disorder of sin, to be cleansed by Christ (Heb. 9:9–14).

16:16 Symbolical atonement prefigures Christ's final atonement (Heb. 9:7–14).

17:11 The blood symbolizing life prefigures the blood of Christ, whose poured-out life brings
atonement for sin (Rom. 3:25; Heb. 9:12–14, 18–26).

17:14 In the superior blessing of the new covenant we partake of the blood of Christ as the source
of spiritual life (John 6:53–56).

18:3 Separation from pagan practices is part of holiness with God, prefiguring the holiness of Christ
(Heb. 7:26) and his people (2 Cor. 6:14–18).

18:5 Ultimately, the holiness of God requires perfect obedience, which is found in Christ (2 Cor.
5:21). Sinful man cannot keep the law (Rom. 10:5; Gal. 3:12–14).

19:2 Loyalty to God requires a life of holiness (1 Pet. 1:15–22).

19:18 The love commandment finds fulfillment in Christ and in those who are his (Matt. 22:39;
Rom. 13:9; Gal. 5:14; James 2:8; 1 John 3:11–18; 4:7–21).

20:2 Sin has consequences in curse and death, prefiguring both the death of Christ as sin-bearer (1
Pet. 2:24) and eternal death in hell (Rev. 20:14–15).

21:1 Holiness requires separation from death, which symbolizes sin. The priests prefigure the
priesthood of Christ (Heb. 7:26–28) and of his redeemed people (1 Pet. 2:5, 9; Rev. 1:6; 5:10).

22:3 Sin, symbolized by uncleanness, disqualifies us from heavenly things and must be cleansed by
Christ (Heb. 9:8–13).

23:5 See Deut. 16:1–8. The Passover prefigures the Last Supper and Christ's death (Matt. 26:19, 26–
28; 1 Cor. 5:7).

23:16 See Deut. 16:9–12. This is the feast of “Pentecost,” fulfilled in Acts when the firstfruits from
the nations are gathered into the church (Acts 2:1–11).

23:28 The day of atonement, an annual day described in chapter 16, prefigures the once-for-all
atonement of Christ (Heb. 9:7–14; 10:3–5).

24:2 Continual light prefigures Jesus as the light of the world (John 1:4–9; 3:19–21; 8:12; 9:5).

http://breachrepairers.webs.com/ 12
24:8 Continual bread prefigures Jesus as the bread of life (John 6:35, 48–51).

25:4 The rest given to the land prefigures the final rest given in the consummation (Heb. 4:9–11;
Rev. 21:1–22:5). See notes on Gen. 2:2 and 2:3.

26:14 Sin leads to a curse, anticipating Christ's sin-bearing (Gal. 3:13–14), and sin ultimately leads
to hell (Rev. 20:14–15).

27:10 The permanence of holiness prefigures the permanence of redemption (John 10:28–29) and
of the new world (Rev. 22:5).

Numbers
The journey through the wilderness prefigures the Christian journey through this world to the new
world (1 Cor. 10:1–11; Heb. 4:3–10).

1:3 Readiness for war prefigures spiritual war (Eph. 6:13).

2:17 The people of God are to be organized with God at the center (Eph. 4:4–6).

3:12 The Levites as a holy substitute prefigure Christ as priest, representative, and substitute (Heb.
7:23–28).

4:15 The penalty of death for approaching God's holiness indicates the need for perfect mediation
through Christ (Heb. 9:23–26).

5:20 The need for faithfulness in marriage prefigures the faithfulness of the church to Christ (2 Cor.
11:2–4; Eph. 5:25–27).

6:5 The special holiness of the Nazirite prefigures the holiness of Christ (Heb. 7:26).

7:5 Holy service prefigures the service of Christ (Heb. 7:23–8:2) and his people (Rom. 12:1–2).

8:16 Christ substitutes for us and represents us before God (Heb. 7:23–28).

9:10 Being clean for the Passover prefigures moral purity in the church (1 Cor. 5:7–8).

10:2 Summoning prefigures God's instruction to the church (Eph. 4:1; 1 Thess. 4:1–3).

11:17 The distribution of the Spirit foreshadows the wider distribution at Pentecost (11:29; Joel
2:28; Acts 2:4, 16–18).

12:8 Rejection of Moses prefigures the seriousness of rejecting Christ's unique prophetic ministry
(John 3:32–36; 5:23).

13:31 The unbelief of Israel contrasts both with the faithfulness of Christ (Matt. 4:1–10) and the
faith of Christians (Heb. 3:7–4:3).

http://breachrepairers.webs.com/ 13
14:35 Death indicates judgment on unbelief (Heb. 3:16–19).

15:30 Cutting off prefigures apostasy from Christ (Heb. 10:26–31).

16:2 Rebellion prefigures false teaching in the church (Jude 10–13).

17:5 The choice of Aaron alone prefigures Christ as the one way (John 14:6).

18:5 The priests turn away wrath, prefiguring Christ's propitiation (Rom. 3:23–25).

19:9 Purification prefigures the purification of Christ's work (Heb. 9:13–14).

20:24 The failures in the priests point to the need for the greater priesthood of Christ (Heb. 7:23–
25).

21:9 Looking at the serpent prefigures faith in Christ who is lifted up (John 3:14–16).

22:12 God overrules all plots against his purposes (Acts 2:23; Eph. 1:11–12).

24:17 Partial fulfillments in David's and Solomon's rule anticipate Christ's rule over his enemies (1
Cor. 15:24–27; Eph. 1:20–22).

25:3 Idolatry leads to chastisement and death (1 Cor. 10:20; Rev. 14:9–11).

27:4 Inheritance of the land anticipates eternal inheritance of the new world (Heb. 11:13–16).

28:3 Repeated, scheduled offerings anticipate one final offering by Christ (Heb. 10:1–10).

30:3 The authority of a man anticipates the authority of Christ over the church (Eph. 5:21–24).

31:16 The war prefigures holy war against sin (Eph. 6:11; 1 Pet. 2:11).

32:17 The 2 1/2 tribes receive their inheritance in Josh. 13:8–33. The tribes' commitment to the
whole nation prefigures cooperative work in the church (1 Corinthians 12).

33:2 The names of the locations record God's faithfulness to his promise to bring his people to the
land (Gen. 12:7; Ex. 6:4), prefiguring his faithfulness to believers in Christ (2 Cor. 1:20).

34:13 The inheritance is distributed in Joshua 14–19. The allotment of this land prefigures
allotment to each of Christ's people of an eternal inheritance (Eph. 1:11; Col. 1:12).

35:11 See Joshua 20. Deliverance from death prefigures Christ becoming a refuge from death for
his people (John 8:51; Heb. 2:14; 6:18).

36:2 See note on 27:4.

http://breachrepairers.webs.com/ 14
Deuteronomy
The righteousness and wisdom of the law of God prefigure the righteousness of Christ, which is
given to his people. The anticipation of entering the Promised Land prefigures Christians' hope for
the new heaven and the new earth (Rev. 21:1–22:5).

1:32 The people's unbelief (see Numbers 14) contrasts with faith for entering God's rest (Heb. 3:7–
4:11).

2:24 God, not human strength, gives victory (3:22), prefiguring victory in Christ (Heb. 2:14–15).

3:12 Moses recalls Numbers 32; see note on Num. 32:17.

3:26 The insufficiency of Moses contrasts with the sufficiency of Christ, who has entered the eternal
inheritance on our behalf (Heb. 9:23–26; 10:19–22).

4:6 Israel by obeying would have been a light to the nations. Christ in his obedience is the light that
Israel failed to be (Isa. 42:6; John 1:4–9).

5:2 The covenant at Horeb (Sinai) anticipates the new covenant, where obedience will spring from
the heart (Heb. 8:8–13), because of Christ's purification (Heb. 10:14).

6:5 Love for God is the greatest commandment (Matt. 22:37–38). One's relation to God himself is
central to life, and true love for God and reconciliation to God are possible only in Christ (John 14:6;
Rom. 5:1–10).

6:14 Holiness before God avoids compromise with evil, prefiguring the holiness of Christ (Heb.
7:26) and his people (1 Pet. 1:15–16; 2:11).

8:18 Gratitude rather than pride characterizes the people of God (1 Cor. 1:28–31; 2 Cor. 9:15).

9:19 Moses' intercession prefigures Christ's intercession (Heb. 7:23–25).

10:16 Circumcision of the heart comes from renewal through the Spirit of Christ (Rom. 8:9–13; Col.
2:11; Heb. 8:8–13).

11:9 Obedience is the basis for life, prefiguring Christ's resurrection life as the reward for his
obedience (Phil. 2:8–11).

12:5 Access to God at a single location (Jerusalem, 1 Kings 8:16; Ps. 122:4) prefigures access
through Christ alone (John 14:6).

13:2 False prophets prefigure the danger of false teachings drawing people away from serving God
through Christ (2 Pet. 2:1).

14:2 Refraining from unclean foods symbolizes separation from sin (2 Cor. 6:17).

15:2 Release of debtors anticipates the great release from sin through Christ (Luke 4:18–19).

http://breachrepairers.webs.com/ 15
16:1 The great feasts (see Leviticus 23) prefigure the celebration of Christ's deliverance (1 Cor.
5:7).

17:7 The purging of evil prefigures the purging of evil from the church (1 Cor. 5:13) and from the
consummation (Rev. 21:8).

17:15 Kings prefigure the righteousness of Christ the perfect king (Isa. 9:6–7; Matt. 27:37; Rev.
19:16).

18:18 Prophets anticipate Christ, the final prophet (Acts 3:22–26).

19:4 The provision for justice prefigures the justice of Christ's rule (Isa. 9:6–7).

20:4 God fights in anticipation of Christ's fight against evil during his earthly life (Matt. 12:28–29),
in his death (Heb. 2:14–15), and in his second coming (Rev. 19:15–21).

21:9 Provisions for purity and justice anticipate final purification and justice in Christ (Heb. 9:23–
28).

21:23 The curse anticipates Christ bearing the curse of God on our behalf when he is crucified
(“hanged on a tree”) (Gal. 3:13).

22:22 Provisions for sexual purity anticipate the purity of the church as Christ's bride (Eph. 5:25–
27; Rev. 19:7–8).

23:9 God's presence in the camp for war (20:4) requires holiness, prefiguring holy war in Christ
(Rev. 19:14–16).

24:1 Provisions for divorce are due to hardness of heart and are inferior to God's design (Matt.
19:3–9), which is to be fulfilled in Christ (Eph. 5:22–33).

25:4 Provision for the ox is an illustration of a larger principle of provision for labor in the church
(1 Cor. 9:9–11; 1 Tim. 5:18).

25:5 Provision for a continuing name and inheritance prefigures God's promise and provision for
our name (Rev. 2:17) and our inheritance (Eph. 1:13–14; 1 Pet. 1:4–5). It also prefigures Christ, who
as younger “brother” to Adam raises up spiritually alive children (Heb. 2:13).

26:8 Thanksgiving for redemption prefigures Christian thanksgiving for redemption in Christ (Heb.
13:15–16).

27:26 All are subject to the curse, and can escape only through Christ's taking the curse on himself
(Gal. 3:10–14).

28:1 Eternal blessings of salvation come in Christ (Gal. 3:14), who removed the curse we deserved
(Gal. 3:13).

29:4 Renewal of the heart is to come in Christ (Rom. 11:8; Heb. 8:8–13).

http://breachrepairers.webs.com/ 16
30:12 Christ brings power to obey God from the heart (Rom. 10:6–8).

31:26 God makes provision for the preservation of the law for future generations, including us
(Rom. 15:4; 1 Cor. 10:11).

32:5 Israel's rebellion contrasts with the faithfulness that is to characterize God's children (Phil.
2:15).

32:6 God's care for Israel prefigures his care for Christ's people (Rom. 8:15–17).

32:21 The apostasy of Israel anticipates the rejection of the gospel (Rom. 10:19).

34:10 The uniqueness of Moses anticipates the uniqueness of Christ (Acts 3:22–26).

Joshua
The conquest through Joshua prefigures Christ conquering his enemies, both Satan (Heb. 2:14–15)
and rebellious human beings. The conquest takes place both through the gospel (Matt. 28:18–20)
and in the destruction at the second coming (Rev. 19:11–21).

1:6 Joshua's role prefigures Jesus empowering his disciples (Matt. 28:18–20; Acts 1:8).

2:9 Rahab in her faith anticipates the salvation of Gentiles through faith (Gal. 3:6–9; Heb. 11:31;
James 2:25).

3:11 God's presence brings the people through the waters of death into the land, prefiguring Christ
leading us to eternal life (John 11:25–26).

4:6 Memorials of God's faithfulness look forward to the message of Christ's salvation.

5:14 The divine commander anticipates Christ, who is the commander in climactic spiritual war
(Matt. 28:18; Heb. 2:14–15; Rev. 17:14; 19:11–21).

6:2 The fall of Jericho prefigures the fall of Babylon and the end of the world (Rev. 18:2).

7:11 Israel's suffering because of unholiness prefigures the need for holiness in the church (1 Cor.
5:1–13).

8:32 A permanent record and a recital of the covenant fulfill the instructions given under Moses
(Deut. 27:2–8). Intimacy with God through the covenant looks forward to the new covenant in
Christ (Heb. 8:8–13).

9:3 Though Israel fails in not consulting the Lord (9:14), the result prefigures the time when
through the gospel people from many nations will come to recognize the God of Israel (Luke 24:47;
Acts 1:8; Rev. 5:9–10).

10:14 The great display of God's power on behalf of his people prefigures the power of Christ's
resurrection and God's commitment to save those who belong to Christ (Eph. 1:19–23).

http://breachrepairers.webs.com/ 17
11:23 The whole conquest takes place according to the plan and promise of God (Deuteronomy 7,
etc.), illustrating God's commitment to Israel in love and anticipating his commitment to believers
in Christ (Eph. 1:3–14).

12:1 The list of defeated kings prefigures the triumph of Christ over all nations (Eph. 1:22; Rev.
5:9–10; 19:11–21; 20:8–9).

13:8 Inheritance takes place according to plan (Numbers 32), prefiguring God's faithfulness with
respect to the eternal inheritance in the new heaven and the new earth (Eph. 1:11, 14; 2:18; 1 Pet.
1:4; 2 Pet. 3:13).

14:2 See Numbers 32–35, especially 32:33; 33:54; 34:17; 35:2. Inheritance takes place according to
the plan of God, anticipating eternal inheritance.

14:6 See Num. 14:6–8. Caleb is a special example showing that inheritance comes to those who
have faith in God and his promises. He prefigures eternal inheritance by faith (Rom. 4:13–16; Gal.
3:7, 18).

15:1 Detailed specification of boundaries underlines for future generations their participation in
the promise. It prefigures the detailed care and provision that God makes for each of us,
anticipating the full inheritance in the new heaven and the new earth (1 Pet. 1:4; 2 Pet. 3:13; Rev.
21:1).

16:1 Each of the tribes is provided for (Num. 33:54), and with it each of the members of the tribes,
prefiguring God's provision for each follower of Christ (John 10:3, 14; see also John 6:35).

18:4 The situation is reminiscent of the spying of the land in Numbers 13. But this time the result is
more favorable, prefiguring the even greater blessings that God has in store through the new
covenant (Heb. 8:8–13).

19:1 See note on 15:1.

20:1 The selection of cities of refuge fulfills the instructions through Moses (Num. 35:9–29; Deut.
19:1–13). It makes provision for refuge from death, prefiguring the coming of Christ as final refuge
and solution to death (Heb. 2:14–15; Rev. 1:18).

21:2 The distribution of the Levites among the tribes fulfills Gen. 49:7 and Num. 35:1–8, and
provides all the tribes with people to teach the law (Lev. 10:11; Mal. 2:4–9). Their teaching
prefigures the knowledge of God from the heart in the new covenant (Heb. 8:8–13).

22:26 The altar confirming participation in God's promises prefigures the Holy Spirit sealing
participation in Christ (2 Cor. 1:22; Eph. 1:13).

23:6 The call to loyalty to the Mosaic covenant prefigures the call to faith in Christ (Matt. 28:18–20;
Heb. 3:12–14).

24:15 God must be served with exclusive loyalty (Deut. 5:7), prefiguring the exclusivity of
commitment to Christ as the one way of salvation (Matt. 6:24; 10:34–39; John 14:6; Acts 4:12; 1 Cor.
10:21–22).

http://breachrepairers.webs.com/ 18
Judges
The judges save Israel, thus prefiguring Christ. But the judges have flaws and failures, and Israel
repeatedly slips back into idolatry (2:19), spiraling downward to chaos. They need a king (21:25),
and not only a king but a perfect king, the Messiah (Isa. 9:6–7).

1:2 The leading role of Judah anticipates the rise of kings from the line of Judah (Gen. 49:10),
beginning with King David and culminating in Jesus Christ (Matt. 1:1–16).

2:18 God raises judges to save the people, prefiguring the sending of Christ (Matt. 1:21). But the
judges' help is only temporary (Judg. 2:19).

3:20 The surprise prefigures the surprising character of salvation in Christ, which seems to the
world to be weakness (1 Cor. 1:25).

4:9 The glory goes ultimately to God, not to human strength or courage, prefiguring the divine glory
through human weakness in the cross of Christ (1 Cor. 1:25).

5:4 God's power and glory at Seir (Deut. 33:2) prefigure his present and future triumphs (Rev.
19:6).

6:15 God again chooses to save Israel through a weak and timid person (cf. 4:9), prefiguring the
triumph of divine glory through human weakness in Christ (1 Cor. 1:25; 2 Cor. 13:4).

7:3 God reduces the number of troops, prefiguring his work of eternal salvation through a single
person, Jesus Christ.

8:16 Those who despise the work of God through a small number prefigure those who despise the
work of God in Christ (1 Cor. 1:18–31).

9:56 The horrors due to Abimelech give evidence for the need for a king, thus looking forward to
the coming of David and his descendants, above all Jesus Christ the son of David and final king.

10:6 Disobedience and idolatry further multiply (see 2:19), giving further evidence for the need of
permanent salvation through the coming line of King David.

11:2 Jephthah is a flawed judge because of his ancestry, because of his appointment by the elders
rather than a direct call from God, and because of his foolish vow. He makes evident the need for
permanent salvation through the coming line of King David.

12:4 The fighting among the Israelites shows the need for a king in the coming line of David who
will bring unity to the people.

13:5 Samson is to be a Nazirite (see Numbers 6) and especially holy. He shows great promise as a
savior of Israel prefiguring Christ.

13:8 The “man of God,” “the angel of the Lord” (v. 15) is God himself (v. 22), anticipating the
incarnation of Christ.

http://breachrepairers.webs.com/ 19
14:3 Israel is told not to intermarry with the Canaanites (Deut. 7:3). In Samson's case the Lord uses
it for good (Judg. 14:4), but it ultimately becomes Samson's downfall (ch. 16), indicating the need
for a perfect savior to deliver people from their spiritual “marriage” to idolatry.

15:14 Samson triumphs after being delivered as a captive over to the enemies, prefiguring Christ's
victory after being delivered to his enemies.

16:30 Samson, though sinful, delivers Israel through his death, prefiguring Christ the sinless one
delivering his people.

17:2 Sin is compounded, in stealing, making an idol, partly backing down from a vow (v. 4), and
making a false priesthood (v. 5). This shows further descent into sinfulness and the need for the
coming king in the line of David.

18:19 The multiplication of sin shows the need for salvation through the coming king in the line of
David.

19:30 Gibeah has become like Sodom (Genesis 19), showing the depths of sin and the need for
salvation.

20:14 Division and war, rather than unity in righteousness, show the need for salvation through the
coming king in the line of David.

21:10 The tribe of Benjamin is saved from utter annihilation, but only through further disunity,
slaughter, and disorder. The disaster shows the need for permanent salvation through the king.

Ruth
The line of offspring leading to Christ goes through Judah to Boaz to David (4:18–22; Matt. 1:5–6).
Boaz the redeemer (Ruth 2:20), prefiguring Christ, enables Naomi's disgrace to be removed and
Ruth, a foreigner, to be included in God's people (prefiguring the inclusion of the Gentiles, Gal. 3:7–
9, 14–18, 29).

1:16 Ruth expresses faith in the God of Israel, as well as love for Naomi, anticipating the role of faith
when Christ comes to bring salvation.

1:20 Naomi's transition from bitterness to blessedness prefigures the participation of God's people
in Christ's death and resurrection (Phil. 3:10).

2:20 The kindness and protection of Boaz, the kinsman-redeemer, prefigure the work of Christ the
redeemer.

3:9 Christ spreads his protection over the church, his bride (2 Cor. 11:2; Eph. 5:25–27).

4:11 The blessing of fruitfulness has a near fulfillment in the birth of Obed (v. 13), but points
ultimately to Christ and his fruitfulness (Heb. 2:10).

http://breachrepairers.webs.com/ 20
1 Samuel
David, the king after God's heart (16:7; Acts 13:22), prefigures Christ, in contrast to Saul, who is the
kind of king that the people want (1 Sam. 8:5, 19–20). Saul's persecution of David prefigures
worldly people's persecution of Christ and of Christ's people.

1:11 By his power to bring life out of barrenness God raises up Samuel as his representative,
prefiguring the virgin birth of Christ (Matt. 1:25).

2:7 The raising of the downtrodden that Hannah experiences prefigures the reversal in Christ (Luke
1:48–53).

3:19 Samuel's calling at an early age prefigures the intimacy with God that Christ as the Son enjoys
with the Father from all eternity.

4:11 The capture of the ark, which symbolizes God himself, and the death of the priests is a kind of
“humiliation” of God's name, prefiguring the humiliation of Christ in his crucifixion. But it all takes
place in accordance with God's sovereign purpose (2:34–35; Acts 2:23; 4:25–28).

5:4 God executes judgment on Dagon, prefiguring the judgment in Christ against all idols and idol
worship (Rev. 2:20).

6:12 By miraculous power God delivers the ark, the symbol of his name, prefiguring the miraculous
deliverance of Christ from death.

7:8 Samuel acts as a faithful judge (v. 15; cf. Judg. 13:5), prophet (1 Sam. 3:19–20), and priest (7:8–
9), prefiguring the work of Christ as king, prophet, and priest (Heb. 1:1–3).

8:5 A king like the nations contrasts with God's kingship (v. 7). God intends Israel to have a king
(Deut. 17:14–20), but the people's desires and the kings themselves fall short. Saul's failure
contrasts with David's success. But eventually David too fails (2 Samuel 11). The failure of merely
human kings points to the need for the perfect king, Christ, who will be divine and human (Isa. 9:6–
7).

8:7 The people's rejection of God's ways prefigures the rejection of Christ (Acts 3:13–15; 7:51–53).

9:16 God indicates his sovereignty over the appointment of kings, prefiguring the appointment of
Christ as king over all (Ps. 2:6; Eph. 1:20–22; Phil. 2:9–11).

10:1 The oil prefigures the coming of the Holy Spirit to empower. Saul's later failures show that he
is only a shadow of the greater anointing that comes to David (16:13) and climactically to Christ
(Luke 4:18; John 3:34), and then to those who belong to Christ (2 Cor. 1:21–22).

11:15 Saul is initially successful, receiving the benefits of God's favor. This temporary favor
contrasts with the lasting favor on David and his offspring, supremely on Christ (Matt. 3:17).

12:14 As the king goes, so go the people. Their failures show the need for the coming of Christ the
perfect king, who is able to change the hearts of his people.

http://breachrepairers.webs.com/ 21
13:12 Saul knew that sacrifice was supposed to be offered only by the priests (Num. 18:7). Saul's
sins lead to his replacement by David (1 Sam. 13:14; 16:7), prefiguring the need for Christ the
perfect king.

14:6 The Lord saved Israel through Jonathan that day (v. 23). Ultimate salvation comes through one
man, Christ Jesus (1 Tim. 2:5).

15:22 Sinners replace real obedience with outward tokens (see Mic. 6:6–8). Full obedience from
the heart is found in Christ (Heb. 10:5–10).

16:7 The choice of David contrasts with people's looking on outward appearance (10:23–24). The
contrast prefigures people's rejection of Christ's humiliation and suffering (Isa. 53:3; 1 Cor. 1:18–
31).

17:47 God's working national deliverance through David prefigures international salvation through
Christ, who defeats Satan (Heb. 2:14–15).

18:3 Despite Saul's antagonism, Saul's son Jonathan and daughter Michal go over to David's side.
David prefigures the spiritual attraction of Jesus Christ, who is the final David (Matt. 4:18–22; 8:9–
13).

19:10 Saul's repeated persecution of David in his innocence prefigures the repeated persecution of
Christ (John 8:44–47).

20:33 The conflict with Jonathan prefigures the conflict within households over loyalty to Christ
(Matt. 10:34–39).

21:5 The exception made for David as God's anointed prefigures the role of Christ, God's anointed,
in relation to the law (Matt. 12:3–4, 8).

22:16 As Saul continues to pursue David, Saul's sins multiply, prefiguring the progressive
enslavement to sin on the part of those who refuse to come to Christ.

23:2 Directions from God repeatedly help David to choose a path forward, prefiguring the direction
from God through Christ to the road to eternal life (Matt. 7:24–27; John 5:24).

24:6 David respects Saul's position as God's anointed king, unlike Pilate, who failed to recognize
Jesus' position as God's anointed King (John 19:10).

24:17 David shows mercy to Saul, prefiguring the mercy of Christ even toward those who have
opposed him (1 Tim. 1:13–16).

25:24 Abigail offers herself as a guilt-bearer for her worthless husband, prefiguring the gracious
guilt-bearing of Christ (1 Pet. 2:23–25).

25:29 Vengeance belongs to the Lord (Rom. 12:19). In recalling this, David prefigures Christ's
willingness to leave vengeance in God's hands (1 Pet. 2:23).

http://breachrepairers.webs.com/ 22
26:9 See the note on 24:6.

27:1 Though David loses heart, God continues to protect David in fulfillment of his purpose to make
David king (16:1). God's faithfulness even to an imperfect man magnifies his faithfulness in the case
of Christ, the perfect king.

28:7 By consulting a medium, Saul makes a further step into wickedness, further contrasting his life
with the righteousness of David, and the climactic righteousness of the Messiah.

29:11 God continues faithfulness to David by removing him from involvement in the death of Saul
and Jonathan (31:2) and enabling him to return to Ziklag in time to rescue the wives and children
(30:1–31). See note on 27:1.

30:6 David through the strength of God acts as deliverer, prefiguring Christ the deliverer of
captives (Luke 4:18–19).

31:6 God fulfills his word against Saul (28:19), showing that sin in a ruler brings suffering and
death not only on himself but on others under his care. The failure of Saul shows the need for a
perfect ruler in the line of David (Isa. 9:6–7).

2 Samuel
David as a model king brings blessing to the nation until he falls into sin with Bathsheba (ch. 11).
Though he repents, the remainder of his reign is flawed, pointing to the need for the coming of
Christ the perfect messianic king.

1:23 David mentions nothing of Saul's failures and sins, prefiguring the grace and forgiveness of
Christ.

2:10 Judah and Israel are eventually united under David and Solomon (5:1–5; 1 Kings 4:20), but
division reappears under Rehoboam and his successors (1 Kings 11:11–13; 12:16–24). The strife
points to the need for permanent union, which will be achieved only through Christ the king.

3:37 David's graciousness and respect for Abner, in contrast to Joab's vengeance, display the
qualities of a godly king, prefiguring the graciousness of Christ.

4:11 David's respect for Ish-bosheth, like his respect for Abner, shows the desire for reconciliation
and forgiveness, prefiguring Christ's reconciliation.

5:2 David unites Israel and Judah under one head, fulfilling God's prophetic purpose (1 Sam. 16:1)
and prefiguring the greater unity of God's people to be accomplished in Christ (1 Corinthians 12;
Eph. 4:1–16).

6:7 Only the Levites were to carry the ark, touching only its poles (Ex. 25:14; Num. 4:15). God in his
holiness destroys sinners who approach him unauthorized, but his presence can also bring blessing
(2 Sam. 6:12). The tension is resolved only when the way to approach God is opened through
Christ's work of purification (Heb. 10:19–22).

http://breachrepairers.webs.com/ 23
7:12 God's covenant with David has a proximate fulfillment with Solomon (1 Kings 1:46; 8:15–21).
But Solomon fails (1 Kings 11:1–10). God preserves the line of offspring (1 Kings 11:12, 36; 15:4; 2
Kings 8:19) until Christ the everlasting king comes (Matt. 1:1–16).

7:14 God promises David that he will be a father to Solomon. As God's son, Solomon prefigures
Christ the eternal Son (Heb. 1:5).

8:15 David as model king subdues enemies and brings justice, prefiguring the work of Christ the
king (Isa. 9:6–7).

9:1 David's graciousness toward the house of Saul fulfills his earlier promise to Saul (1 Sam. 24:21–
22) and Jonathan (1 Sam. 20:15–17), and it prefigures the graciousness of Christ the king.

10:2 Willingly or unwillingly Ammon comes to acknowledge David's rule, prefiguring the willing or
unwilling submission of all nations to Christ's rule (Psalm 2).

11:4 David later repents (12:13). But David and his house and his rule over the whole nation suffer
various consequences for the rest of his life. The devastation from one sin points to the need for
Christ the perfect, sinless king (Isa. 42:1–4).

12:13 God is gracious to forgive, ultimately for the sake of Christ (1 John 1:9). But sin still brings
consequences (2 Sam. 12:10–12, 14). See note on 11:4.

13:22 The sin of Amnon, in its similarity to David's sin (11:4), begins a series of devastating
consequences for David's house (12:10–12), including not only Absalom's actions but David's
neglect of discipline and justice toward Amnon and Absalom. See note on 11:4.

14:1 David's love for Absalom prefigures Christ's love for sinners. But David falls short of Christ by
neglecting justice: murder deserves death (Num. 35:31–34).

15:1 Absalom's betrayal of his father prefigures Judas's betrayal of Jesus (John 13:18), and more
broadly the treachery of all who rebel against God the Father and Christ.

15:30 David's sorrow prefigures the sorrow of Christ as he leaves Jerusalem and prays in
Gethsemane (Matt. 26:30, 36–46).

16:12 David leaves vengeance to God, prefiguring the patience of Christ before his enemies (1 Pet.
2:23).

16:22 Absalom's sordid behavior fulfills God's prophecy in 12:11–12, further illustrating the
devastation of sin and the need for a perfect redeemer king.

17:5 Through Hushai and other circumstances, God shows mercy to David and answers David's
need expressed in 15:31–37. The turning back of the effects of sin, and David's rescue from death,
look forward to final redemption in Christ.

18:33 David's grief, though flawed (19:2, 5–7), prefigures the willingness of the Son of God to die in
place of sinners (Rom. 5:8).

http://breachrepairers.webs.com/ 24
19:22 Forgiveness under the reestablished kingship prefigures forgiveness for former rebels under
Christ's kingship (1 Tim. 1:12–16).

20:1 Divisiveness continues to rear its head after Absalom's death, partly because of David's
preference for Judah in 19:11–15, leading to the anger in 19:43. The kingdom continues to suffer
indirect consequences from David's sin with Bathsheba, underlining the need for Christ the perfect
king. See note on 11:4.

20:10 Though David is reconciled to Amasa (v. 4), Joab kills him, probably because of his role in
Absalom's rebellion (17:25). See note on 20:1.

21:3 Atonement and blessing are needed, but David's solution (v. 6) does not give ultimate
satisfaction (Deut. 24:16). Full resolution of justice requires Christ the divine king with infinite
wisdom, and the coming of resurrection from the dead (Rev. 20:11–15).

22:1 This song is included in the Psalter in Psalm 18, indicating that it is to be sung by the people of
God as well as David. See note on 1 Chron. 15:16.

22:50 The spread of praise among the nations anticipates the spread of the gospel (Acts 1:8; Rom.
15:9).

22:51 God's salvation for David prefigures his salvation through Christ the king.

23:8 The list of mighty men prefigures the might in the army of God under Christ the king (Rev.
19:11–14).

24:1 Out of the need for atonement comes the designation of the site for the temple of Solomon (1
Chron. 21:28–22:1), which prefigures Christ as the final temple where atonement is accomplished
(John 2:19–21). See note on 1 Chron. 22:1.

24:17 The suffering of the sheep for the sin of their king is reversed when Christ suffers for the sins
of the sheep (John 10:15). Christ's suffering answers David's request that God's hand would be
against “my father's house,” the line leading to Christ.

1 Kings
The reign of Solomon fulfills the first stage of God's promise to David to establish the kingdom of his
offspring (2 Sam. 7:12). Solomon in some ways is a model king, prefiguring Christ. But his decline
into sin (1 Kings 11), the sins of his offspring, the division and strife between Israel and Judah, and
the continual problems with false worship indicate the need for a perfect king and an everlasting
kingdom (Isa. 9:6–7) surpassing the entire period of the monarchy. Many passages in 1 Kings have
parallels in 2 Chronicles.

1:13 David's purpose prefigures the purpose of God to establish Christ as king, when many prefer
alternatives (Psalm 2; Acts 13:33).

2:6 Solomon's wisdom is tested in dealing with unfinished business from the reign of David.
Solomon's wisdom prefigures the wisdom of Christ (Matt. 12:42; Col. 2:3). The combination of

http://breachrepairers.webs.com/ 25
mercy and justice characterizes David and Solomon in anticipation of Christ.

3:9 See note on 2:6. God promises wisdom in 3:12, and fulfillment is seen in 3:28 and 4:29–34.

4:1 The blessings of order, peace, justice, and prosperity in Solomon's reign prefigure the blessings
of Christ's reign.

4:34 The attraction of Solomon's wisdom prefigures all nations hearing the wisdom of Christ (Acts
1:8).

5:5 Solomon's building of the temple fulfills God's promise in 2 Sam. 7:13 (cf. 1 Chron. 17:12) and
prefigures the building of an everlasting temple. Christ's resurrection body is an everlasting temple
(John 2:19–22), and then Christ builds the church as a temple (Matt. 16:18; 1 Cor. 3:16).

5:8 The aid in building from Hiram, a Gentile, prefigures the inclusion of the Gentiles in the building
of the church as a temple (Eph. 2:19–22).

6:2 The temple is like the tabernacle of Moses (Exodus 25–27; see note on Ex. 25:8), but it is larger
and more magnificent, symbolizing an expansion and a further stage in God's purpose to dwell with
his people. Still further development takes place with Ezekiel's vision of a new temple (Ezekiel 40–
43), with the church (Eph. 2:19–22), and with the new Jerusalem in the consummation (Rev. 21:3,
10–22:5).

7:14 See note on 5:8. Hiram's God-given wisdom is like that of Bezalel and Oholiab, who supervised
the construction of the tabernacle (Ex. 31:1–6). It prefigures the wisdom of Christ and of his
servants in the building of the church (Eph. 2:19–22).

7:23 The sea greatly enlarges the basin for washing that was in the tabernacle (Ex. 30:17–21). See
note on Ex. 30:20.

7:27 The stands with their basins (v. 38) represent small, mobile versions of the sea (vv. 23–26),
further underlining the abundance of water (see note on v. 23). The multiplication of water,
compared with the single basin for washing in Ex. 30:17–21, anticipates the even greater
abundance when the water provided by God becomes a river of life (Ezek. 47:1–12; John 4:10–14;
19:34; Rev. 22:1–2).

8:11 See Ex. 40:34–35. The glory of the Lord later departs, because of the apostasy of the people
(Ezekiel 10). The coming of God's presence prefigures the fullness of the Spirit in Christ (Matt.
3:16–17; John 3:34–35; 1:14) and within the church (Acts 2:3–4; 1 Cor. 3:16).

8:24 The promise to David is in 2 Sam. 7:13. The temple anticipates the greater fulfillment in the
dwelling of God with man through Christ. See notes on 1 Kings 5:5 and 6:2.

8:30 The key role of the temple in prayer prefigures the role of Christ, through whose name we
have access to God (John 14:13–14; Heb. 10:19–22).

9:8 The desolation comes to pass in 2 Kings 25:9–11, indicating the need for true obedience and a
greater temple that is to come in Christ (John 2:19–21).

http://breachrepairers.webs.com/ 26
10:1 The queen of Sheba's coming to hear wisdom, mentioned also in Matt. 12:42, prefigures the
coming of the nations to Christ (Acts 1:8; Col. 2:3).

11:2 Solomon's disobedience leads to disastrous judgment (vv. 9–11), anticipating the judgments
on later idolatries among God's people. Solomon's failure indicates the need for Christ the perfect
king in the line of David (Matt. 1:1–16).

12:15 God's prophecy in 11:29–39 begins to be fulfilled, and God's people split into two kingdoms.
Both Rehoboam's failure and the resulting disunity and strife among God's people show the need
for Christ the perfect king as the unifier of his people (1 Corinthians 12; Eph. 4:1–6).

13:2 A striking prophecy, fulfilled in 2 Kings 23:15–17, shows the power of God's word even in the
midst of sin, corruption of worship, and chaos. The power of the prophetic word prefigures the
power of Christ, the final prophet (Acts 3:22–26; Heb. 1:1–2).

13:34 See the description of Jeroboam's sin in 12:26–33. Judgment for sin is prophesied in 14:9–12,
and falls in 14:17–18, 15:29–30. Jeroboam's sin continues with his successors (15:34; 16:2, 7, 19,
26; 22:53; 2 Kings 3:3; 10:29, 31; 13:2, 11; 14:24; 15:9, 18, 24, 28), ultimately leading to the exile of
the northern kingdom (2 Kings 17:21–23). The judgments on false worship show the need for true
worship, prefiguring Christ as the one way to God (John 14:6).

14:10 See note on 13:34. The power of God's word is seen when the judgment falls in 14:17–18 and
15:29–30.

14:22 Just as in the northern kingdom (v. 9), false worship in the southern kingdom eventually
leads to exile (2 Kings 23:26–27; 25:1–21; see note on 1 Kings 13:34).

15:4 In spite of sin God is faithful to the promise to David (2 Sam. 7:5–17), and maintains the line of
David (1 Kings 11:12, 32, 34, 36; 2 Kings 8:19; 19:34) down through a list of kings of Judah leading
to Christ (Matt. 1:1–16).

15:18 In contrast to the kings of Israel (vv. 26, 34), Asa is a good king (v. 11), prefiguring the
righteousness of Christ his descendant. Yet in this case he fails to rely on God (see 2 Chron. 16:7–
12), underlining the need for perfect righteousness in the king.

15:29–30 The killing fulfills the prophecy in 14:9–11 (see note on 13:34). The wiping out of the
king's line of descent contrasts with God's faithfulness in maintaining the line of David leading to
Christ (see note on 15:4).

16:3 See 15:29–30. Judgments on the northern kingdom show the consistency of God's word and
his holiness (see note on 13:34).

17:1 The power of the prophetic word prefigures the power of Christ's word (Heb. 1:1–3).

17:14 The miraculous supply of food through the power of God's word prefigures the power of
Christ to multiply bread (Matt. 14:13–21; Mark 8:1–9) and to himself be the bread of heaven (John
6:26–51).

17:21 Impartation of life prefigures Christ's resurrection of Jairus's daughter (Matt. 9:18–25), his

http://breachrepairers.webs.com/ 27
resurrection of Lazarus (John 11:38–44), his own resurrection (John 10:18), and his role as “the
resurrection and the life” (John 11:25–26) who gives spiritual life to us in anticipation of the
resurrection of the body (John 5:28–29).

18:39 Miraculous power anticipates the resurrection of Christ, which displays the power of God
and draws the nations to acknowledge him (John 12:32).

19:2 Jezebel's opposition undermines Elijah's previous work, seeming to lead to failure (v. 4). But
God's purpose through his prophetic word stands (vv. 12, 15–18), prefiguring the victory when
Christ fulfills prophecy.

19:16 See v. 19. Elijah is not the end, but one of a succession of prophets leading to Christ, the final
prophet (Heb. 1:1–2).

19:18 The 7,000 illustrate the concept of a remnant, to be fulfilled by the Jews who believe in Christ
(Rom. 11:3–10; see note on Isa. 6:13).

20:28 God's desire to magnify his glory enables Ahab to defeat Ben-hadad twice (see vv. 19–21).
The victory in battle prefigures the final victory of Christ and his army (Rev. 19:11–21).

20:42 Ahab's failure contrasts with the complete elimination of enemies in the final battle led by
Christ (Rev. 19:11–21).

21:19 The prophecy is fulfilled in 2 Kings 9:25–26, 36–37; 10:10–11, 17, showing the power of
God's word in judgment. This power prefigures the power of Christ's word (Heb. 1:1–2; 4:12–13;
Rev. 19:15, 21).

22:19 The superiority of God to all earthly thrones is shown when Micaiah's prophecy (vv. 23, 28)
is fulfilled (vv. 34–36). The power of God and of his word anticipates the power shown in the
resurrection of Christ (Eph. 1:20–22) and in the spread of the gospel, which confounds worldly
authorities (1 Cor. 2:6–9).

2 Kings
Following the history in 1 Kings, Israel and Judah continue to decline through their false worship
and disobedience, leading to exile (2 Kings 17; 25). Some good kings (notably Hezekiah and Josiah,
chs. 18–20; 22:1–23:30) prefigure the need for Christ the perfect king, while Elisha prefigures the
need for Christ the final prophet (Heb. 1:1–3). Many passages in 2 Kings have parallels in 2
Chronicles.

1:4 The prophecy is fulfilled in v. 17. The triumph of God's word over all opposition prefigures the
triumph of Christ and of the gospel.

2:11 Elijah's ascent prefigures the triumph of Christ over death and his ascension (Luke 24:51; Acts
1:9).

2:14 The dividing of the waters, reminiscent of Moses at the Red Sea (Ex. 14:21–22), Joshua at the
Jordan (Josh. 3:7–17), and Elijah at the Jordan (2 Kings 2:8), confirms that Elisha has received the

http://breachrepairers.webs.com/ 28
prophetic succession from Elijah (v. 9). The power over the waters (which are a symbol of death
and chaos) prefigures the resurrection of Christ.

3:17 The provision of water, like the provision under Moses (Ex. 17:6; 20:8–11), prefigures Christ
as the giver of the water of eternal life (John 4:10, 13–14; Rev. 22:1).

4:34 The giving of life, like the instance with Elijah (1 Kings 17:17–24), prefigures the resurrection
of Christ and the life he gives to us through union with him (Rom. 6:4, 8–11; 8:10–11; Col. 3:1–4).

5:14 Cleansing from leprosy (Leviticus 14) prefigures cleansing from sin through the power of
Christ (Luke 5:12–14). The inclusion of Naaman, a Syrian, prefigures the inclusion of the Gentiles in
God's salvation (Luke 24:47).

6:17 The vision of God's angelic army indicates dimensions of spiritual warfare. It anticipates the
spiritual war with the coming of Christ (Matt. 12:28–29; Luke 10:18–19; John 12:31; Rev. 19:11–
21).

7:1 The provision of food in spite of unbelief (see Ex. 16:1–21) prefigures Christ giving himself as
the bread of heaven (John 6:35, 47–51).

8:15 Hazael's fulfillment of earlier prophetic words (1 Kings 19:15; 2 Kings 8:10) shows the power
of God's word in judgment. (See 10:32.) This power anticipates the power of Christ's words (John
12:48; Heb. 1:1–2; 4:12–13; Rev. 1:16).

9:25 This fulfillment of earlier prophecy (1 Kings 19:16–17; 21:19–24) emphasizes the power of
God's word in bringing judgment. See notes on 1 Kings 21:19 and 2 Kings 8:15.

10:10 Jehu fulfills God's prophetic words of judgment against Ahab's house and wipes out the
worship of Baal introduced by Jezebel (1 Kings 16:31–33), showing God's power in judgment and
anticipating the day of judgment (Rev. 20:11–15). See note on 1 Kings 21:19.

11:2 The rescue of Joash prefigures the rescue of Jesus from Herod (Matt. 2:13–15). God preserves
the line of David for the sake of his promise (2 Sam. 7:16) and to carry out his purpose of salvation
through the work of Christ (Rev. 12:4–5).

12:9 The attention to the temple prefigures the importance of building the church (Matt. 16:18; 1
Cor. 14:12; Eph. 2:20–22).

13:23 God's compassion even toward a sinful people prefigures his compassion in Christ toward
sinners (Matt. 9:13; Luke 5:32).

14:10 A single act of pride from Amaziah brings disaster on the people, indicating the need for
Christ as the perfect, humble king (Zech. 9:9).

15:9 See note on 1 Kings 13:34. The northern kingdom goes downhill toward eventual exile in 2
Kings 17:6–23. The degeneration points to the need for perfect kingship and redemption from the
heart, both of which await the coming of Christ.

16:3 Under Ahaz the southern kingdom also suffers serious spiritual degeneration, pointing to the

http://breachrepairers.webs.com/ 29
need for perfect kingship in Christ.

17:7 The exile is God's judgment on sin (see note on 1 Kings 13:34), prefiguring the judgment on
sin that Christ bore as a substitute (1 Pet. 2:21–24) and the final judgment at the consummation
(Rev. 20:11–15).

18:5 Hezekiah as a faithful king prefigures the faithfulness and righteousness of Christ (Isa. 9:6–7;
42:1–4) and its fruits in the lives of Christ's people. See the parallel passages in 2 Chronicles 32 and
Isaiah 36–38.

18:30 Rabshakeh symbolizes the voice of Satan, who deceives and attacks the faith of God's people
(Gen. 3:4–5; Matt. 4:1–10; Eph. 6:16; Rev. 12:9).

19:22 God vindicates his name against all slanders, prefiguring the vindication of his name in the
resurrection of Christ (John 12:28).

20:5 God mercifully hears prayer, anticipating his mercy in Christ, through whom he hears our
prayers (John 14:13–14; 15:16; 16:26–27).

21:8 Manasseh directly affronts God's command and his holiness, which leads to a prophecy of
judgment (vv. 12–15) and illustrates the pattern of rebellion leading to exile (24:2–4). By contrast,
Manasseh's evil points to the need for Christ as the perfect king.

22:2 Josiah as a righteous king prefigures Christ.

22:13 Words of prophecy, not only from Elijah and Elisha but from Moses (Deut. 11:26–28), show
that God judges in accordance with his purpose and his righteousness. This righteousness is
supremely manifested in Christ, both when in his innocence he bears sin (2 Cor. 5:21) and when he
comes to judge the world (Acts 17:31).

22:20 See 23:30. Because of his righteousness and humility, Josiah receives a blessing. But unlike
Christ (Gal. 3:13–14), he is unable to reverse the impending curse and punishment that will come to
his people (see 2 Kings 23:26–27).

24:2 See notes on 21:8 and 22:13.

25:9 God's righteous judgment falls because of accumulated sins (23:26–27; 24:2–4). The judgment
also destroys God's own house, prefiguring the judgment that will fall on Christ, whose body is the
temple (John 2:19–21; Gal. 3:13–14).

25:27 The provision for the king of Judah, in the line of David, indicates that God still remembers
his promise to David (2 Sam. 7:16) and anticipates the eventual coming of Jesus the Messiah
through the line of Jehoiachin (also called Jeconiah, 1 Chron. 3:16; Matt. 1:11–12).

1 Chronicles
David as the righteous leader and king prefigures Christ the king, not only in his rule over the
people of God but in his role in preparing to build the temple. First Chronicles looks back on the

http://breachrepairers.webs.com/ 30
faithfulness of God to his people in the entire period from Adam (1:1) to David (3:1) and even
beyond (3:10–24; 9:1–34), indicating the steadfastness of God's purpose in preparing for the
coming of the Messiah as the offspring of Adam (1:1; Gen. 3:15; Luke 3:38), offspring of Abraham (1
Chron. 1:28; Gal. 3:16), and offspring of David (1 Chron. 3:1; 17:11, 14; Luke 3:23–38; Acts 13:23).

1:1 God promises victory over Satan by the offspring of the woman (Gen. 3:15) and of Abraham
(Gen. 17:7; see notes on Gen. 3:15 and 12:1). The line of chosen offspring goes from Adam through
Seth and Noah (1 Chron. 1:4) to Abraham (vv. 27–28), Isaac (v. 34), and Israel (v. 34; 2:1), earlier
called Jacob (Gen. 32:27–28). It will culminate in Christ (Matt. 1:1–16; Gal. 3:16).

2:1 The line of chosen offspring goes from Israel to David and includes the blessing of
multiplication of offspring in the form of the 12 tribes (see Gen. 13:16; 15:5). See note on 1 Chron.
1:1.

3:1 The line of the Messiah comes through King David (2 Sam. 7:16; Matt. 1:1, 6; see note on 1
Chron. 1:1).

3:10 Solomon and his offspring are a stage in the fulfillment of the promise to David for his
offspring (2 Sam. 7:16). The offspring ultimately lead to Christ (Matt. 1:1–16; see note on 1 Chron.
1:1).

4:1 After recording the Messianic line of David, which will lead to Christ (see note on 3:10),
Chronicles gives the record for Judah, the tribe of David. The recording of individual names and
families underlines their inclusion in the promise to Abraham concerning blessing, land, and
fellowship with God (Gen. 17:4–8). It prefigures the blessing (Gal. 3:14), land (Rom. 4:13; Heb.
11:16; 12:22; Rev. 21:1), and fellowship with God (Rom. 5:1; Gal. 3:26–29) that come from union
with Christ the greater David. God has enrolled our names in his book of life (Rev. 13:8; 17:8; 20:15;
see John 10:3, 14; Eph. 1:4).

5:1 The record of Reuben, Gad (v. 11), and Manasseh (v. 23) indicates their continued inclusion
among God's people as offspring of Abraham and Israel (2:1–2). It answers doubts that might arise
because of the location of their land east of the Jordan (Numbers 32; Josh. 13:8–32; 22:24–29). The
reassurance prefigures the guarantee give to Christians (2 Cor. 1:22; Eph. 1:13–14). See note on 1
Chron. 4:1.

6:49 The special list for Aaron the priest and for the tribe of Levi, which indicates some of their
priestly privileges before God, prefigures the priestly privileges given to Christians through Christ
the final high priest (Heb. 7:23–8:2; 10:19–22).

7:1 Other tribes descending from Israel (2:1–2) are briefly listed. See note on 4:1.

8:33 Special focus is given to Saul, because he was king of Israel (10:14; 1 Sam. 10:1). But he was
superseded by David (1 Sam. 16:1, 12; 2 Sam. 7:15; 1 Chron. 10:13–14; 17:13), whose line of kings
leads forward to Christ the king (Matt. 1:6–16).

9:2 The enrollment of names of returned exiles indicates God's continued faithfulness to the
offspring of Israel. It prefigures God's enrollment and faithfulness to those who belong to Christ the
Israelite (Gal. 3:14, 16, 28–29; see note on 1 Chron. 4:1).

http://breachrepairers.webs.com/ 31
10:14 The movement of kingship to David is the beginning of the line of kingly offspring leading to
Christ (17:11, 14; Matt. 1:6–16).

11:3 David is established as king in fulfillment of God's purpose (v. 2), prefiguring the
establishment of Christ the son of David as the final king (Ps. 2:6–12; Acts 13:33; Eph. 1:20–22).

12:23 The unification of God's people under David, and their strength for war, prefigures the
unification and spiritual strength under Christ the king (Eph. 4:1–16; 6:10–20).

13:10 See note on 2 Sam. 6:7. When the Levites take the appropriate role (Ex. 25:14; Num. 4:15; 1
Chron. 15:2, 13–15), the ark is brought up safely (1 Chron. 15:26).

13:12 The supreme holiness of God and his reaction to the approach of sinners, produces fear. The
resolution comes through Christ's propitiation, which permanently answers God's wrath (Rom.
3:20–26; 5:1).

14:15 God fights with David against Israel's enemies, prefiguring Christ defeating Satan and his
hosts (Matt. 12:28–29; Luke 10:18–19; John 12:31; Rev. 19:11–21; 20:7–10).

15:2 Unlike Uzzah (13:10), the Levites bring up the ark safely, because they are following God's
instructions (Ex. 25:14; Num. 4:15). The importance of following God's way prefigures the one way
to God opened through Christ (John 14:6; Heb. 10:19–22).

15:16 David and the singers are involved in writing and singing many of the Psalms (see 1 Chron.
16:8–36 and parallels in the Psalms: Ps. 96:1–13; 105:1–15; 106:47–48). They prefigure the role of
Christ in leading his people in singing praise to God for climactic salvation (Heb. 2:12; 13:15; Rev.
19:6–8).

16:4 See note on 15:16.

16:8 See Ps. 105:1–15. Songs of praise are to be sung repeatedly, not only to give praise to God, but
to remind people of his excellence and to anticipate the surpassing display of his excellence when
Christ comes. See note on 1 Chron. 15:16.

16:23 See Ps. 96:1–13 and note on 1 Chron. 16:8.

16:35 See Ps. 106:47–48 and note on 1 Chron. 16:8.

17:4 To underline the importance of Davidic kingship as leading to Christ, Chronicles records the
all-important covenant with David given in 2 Sam. 7:5–16. See note on 2 Sam. 7:12.

17:16 David's marveling over God's grace prefigures the marveling over the grace that has come in
Christ (John 1:16; Eph. 2:7–9).

18:6 The subduing of Israel's enemies prefigures Christ winning victory over Satan and his hosts
(see note on 14:15).

18:14 The coming of justice prefigures the justice of the Messiah (Isa. 9:6–7; 42:1–4; 2 Cor. 5:10;

http://breachrepairers.webs.com/ 32
Rev. 20:11–15).

19:2 See note on 2 Sam. 10:2.

20:1 Chronicles, unlike the parallel in 2 Sam. 11, omits mention of David's sin with Bathsheba,
which more effectively highlights ways in which David's kingship points positively forward to the
triumphs of Christ as final king.

20:8 David's victory over Goliath in 1 Samuel 17 is one of a series of victories that destroy terrifying
enemies of God's people. The victories prefigure the victory of Christ and his people (Matt. 12:28–
29; Luke 10:18–19; John 12:31; Rev. 2:7, 11, 17, 26; 12:11; 19:11–21; 20:7–10).

21:7 See note on 2 Sam. 24:1.

21:17 See note on 2 Sam. 24:17.

22:1 The selection of the site for Solomon's temple takes place according to God's word through
Gad the prophet (21:18). Once the temple is built, it will be the exclusive place for atonement and
approach to God (Deuteronomy 12), prefiguring Christ as the final one who brings atonement and
opens the way to God (John 14:6; Heb. 10:19–22).

22:9 Solomon prefigures Christ as prince of peace, who opens the way to peace with God (Rom.
5:1–10).

23:26 See Num. 4:5–15. God inspires David to make a change in the duties of the Levites,
corresponding to the change in the house of God. The service of the Levites prefigures the service of
Christ as high priest to God (Heb. 7:23–8:6) and subordinately the service of Christians (Rom. 12:1;
Eph. 4:1–16; Heb. 13:15).

24:7 The priests are a special group within the tribe of Levi, chosen to minister in the sanctuary
(Numbers 18). The priesthood prefigures Christ the great high priest (Heb. 7:23–8:6). The duties
rotate to the different divisions (see Luke 1:5, 8), indicating that no one priest is permanent, until
the coming of Christ the everlasting priest (Heb. 7:23–24).

25:1 See note on 15:16. The attention to arrangements for singing prefigures the ordering of the
church's worship through the power of the Holy Spirit (1 Corinthians 12; Eph. 2:22; 5:18–21).

26:1 The gatekeepers protect access to the presence of God in the temple (Num. 18:7, 22),
prefiguring the one way of access to God through Christ (John 10:7; 14:6). Church discipline,
exercised under the authority of Christ (1 Cor. 5:4–5), warns the unrepentant of their danger.

26:20 The care for God's gifts prefigures the guarantee of the inheritance of eternal life in Christ (1
Pet. 1:4–5) and the advice to lay up treasure in heaven (Matt. 6:19–34; see 2 Cor. 9:6–15). Money
given for the needs of God's people is to be carefully handled (2 Cor. 8:20–21).

27:1 Arrangements for the military prefigure the spiritual war fought under Christ's command
(Eph. 6:10–20; see note on 1 Chron. 14:15).

28:6 See the promise to David in 17:11–14, now being fulfilled. See note on 2 Sam. 7:12.

http://breachrepairers.webs.com/ 33
28:19 The temple is built in accordance with God's instructions, just as the tabernacle was (see
note on Ex. 36:10).

28:20 The empowering of God is essential, prefiguring the centrality of God's power in building the
church, the new temple (1 Cor. 3:16; Eph. 2:20–22).

28:21 The previous arrangements of various divisions of the Levites and the people (chs. 23–27)
have all been for the purpose of aiding in the service of the house of God. They prefigure God's
planning for the building of the church as temple (1 Cor. 3:16; Eph. 2:20–22) and the new Jerusalem
as final temple (Rev. 21:22–27).

29:6 The generous offering is like that for the tabernacle (Ex. 35:4–36:7). It prefigures the
generosity of Christ (see note on Ex. 35:21).

29:18 Wholehearted commitment comes ultimately with the perfection of Christ (Heb. 10:7–10)
and the change of the heart that he works in us in the new covenant (Heb. 10:16–17).

2 Chronicles
Solomon as a wise king and temple builder prefigures Christ the king and temple builder. After
Solomon the line of Davidic kings continues, leading forward to Christ the great descendant of
David (Matt. 1:6–16). But many of the later kings go astray from God, and they and the people suffer
for it, showing the need for Christ as the perfect king. Hezekiah (2 Chronicles 29–32) and Josiah
(chs. 34–35) as righteous kings prefigure Christ. Second Chronicles has parallels in 1–2 Kings but
focuses on the southern kingdom (Judah) and the line of David, and it shows focused concern for
the temple and its worship, anticipating the fulfillment of temple and worship with the coming of
Christ (John 2:19–21; 4:20–26; Eph. 2:20–22; Rev. 21:22–22:5).

1:10 See note on 1 Kings 3:9. Wisdom is needed to build the temple (1 Chron. 29:1; 2 Chron. 2:6,
12).

2:3 See note on 1 Kings 5:8.

2:13 See note on 1 Kings 7:14.

3:1 See note on 1 Kings 6:2. The location for the temple was appointed in 1 Chron. 22:1 (see note on
1 Chron. 22:1).

4:1 The altar is twice as large as the one for the tabernacle (Ex. 27:1–8), indicating the more
abundant provision for atonement. See note on Ex. 27:1.

4:7 There are ten lampstands instead of the one in the tabernacle (Ex. 25:31–39), indicating the
more abundant provision of light. See notes on Ex. 25:37 and 1 Kings 6:2.

5:14 See note on 1 Kings 8:11.

6:6 The selection of Jerusalem fulfills the plan given through Moses in Deuteronomy 12. It
prefigures the appointment of Christ as the one way of salvation (John 14:6; Heb. 5:5–10).

http://breachrepairers.webs.com/ 34
6:15 See note on 1 Kings 8:24.

6:21 See note on 1 Kings 8:30.

7:1 The miraculous approval by God is like what happens with Elijah in 1 Kings 18:39 (see note).

7:2 The glory of the Lord signifies the magnificence of his presence, prefiguring Christ's presence.
See 5:14 and note on 1 Kings 8:11.

7:20 See note on 1 Kings 9:8.

8:5 Solomon takes care to provide security against foreign enemies, performing one of the
important duties of ancient kings and prefiguring the spiritual security given through Christ the
king (John 10:28–29; see Rev. 21:24–27; 22:3).

8:14 David's instructions are found in 1 Chronicles 23–27. See the note on 1 Chron. 28:21.

9:1 See note on 1 Kings 10:1.

9:22 Solomon's riches and wisdom prefigure the riches and wisdom of Christ the king (Eph. 1:18;
Col. 2:3; 1 Cor. 1:30).

10:15 See note on 1 Kings 12:15.

11:14 The Levites were distributed among the tribes (Joshua 20–21; see note on Josh. 21:2). But
Jeroboam's false worship (see 1 Kings 12:25–13:5) forces them and others who follow God to join
Judah. The conflict over worship prefigures the conflict over the exclusive claims of Christ (see note
on 1 Kings 13:34).

12:6 Rebellion against the Lord leads to disaster, but repentance brings relief. The pattern
anticipates God's final judgment on rebellion (Rev. 20:11–15) and relief through repentance and
faith in Christ (John 5:24; Rev. 20:15).

13:9 For Jeroboam's promotion of false worship, see 1 Kings 12:25–33 and note on 1 Kings 13:34.
The blessing on true worshipers prefigures the blessing on worship in spirit and truth that Christ
brings (John 4:20–24).

14:7 Blessings come from following God's way, prefiguring the blessings through Christ the final
way (John 14:6; Eph. 1:3–14).

15:8 Asa continues to work for true worship according to the law (Ex. 27:1–8; Deut. 11:28; 12:1),
prefiguring Christ's establishment of true worship (Matt. 21:12–16; John 4:20–24).

16:9 God's judgment takes place within history, as well as at the consummation (Rev. 20:11–15).
Judgment comes climactically when Christ as a substitute takes judgment on himself, and then in
his resurrection receives the reward for his blamelessness (Phil. 2:10–11). See note on 1 Kings
15:18.

http://breachrepairers.webs.com/ 35
17:5 See note on 14:7.

18:18 See note on 1 Kings 22:19.

19:7 Mosaic instructions for judgeship are in Ex. 23:8; Deut. 16:18–20. Promoting justice is one of
the duties of the king, prefiguring the justice of Christ the king (Isa. 9:6–7; 42:1–4).

20:22 God honors those who trust in him, anticipating the giving of honor to Christ in his
resurrection (Phil. 2:10–11) and the blessing to Christians who trust in Christ (Galatians 3).

21:7 The line of David is nearly, but not quite, wiped out, prefiguring the attack by Herod (Matt.
2:13–18) and God's faithfulness to Christ the offspring of David. See note on 1 Kings 15:4.

22:11 See note on 2 Kings 11:2.

23:11 The establishment of the true king, in spite of all opposition, prefigures the establishment of
Christ as king (Ps. 2:7–12; Acts 13:33).

24:4 See note on 2 Kings 12:9.

24:20 See note on 12:6.

25:16 Prophetic warning gives opportunity for repentance, but Amaziah hardens himself instead.
Amaziah's failure points to the need for a perfect king (Matt. 21:5). The call to repentance
prefigures the call to repentance and faith in the NT. See note on 2 Chron. 12:6.

25:19 See note on 2 Kings 14:10.

26:16 Uzziah's sin and its consequences point to the need for a perfect king (Matt. 21:5).

27:6 See note on 14:7.

28:3 See note on 2 Kings 16:3.

28:15 The unusual kindness shows God's mercy (v. 9) and anticipates the love that Jesus embodies
(Matt. 8:14–17; Luke 7:21–22; 1 John 3:16; 4:7–12), that he teaches (Luke 10:25–37), and that he
creates in his followers (John 13:34–35; 1 John 4:17–21).

29:8 Judgments against false worship (predicted in Deut. 11:28) are reversed by Hezekiah,
prefiguring Christ the king coming to remove the curse on sin (Gal. 3:13–14).

30:9 The theme of mercy and repentance looks forward to God's mercy in Christ to those who
repent and turn to him (Luke 18:13). See notes on 2 Chron. 12:6 and 25:16.

30:19 The desire of the heart is of greater importance than mere external conformity (1 Sam.
15:22; Hos. 6:6; Mic. 6:6–8; Matt. 9:13; 25:25–28), anticipating the centrality of renewal of the heart
in Christ's work (Heb. 8:10).

http://breachrepairers.webs.com/ 36
30:26 The contrast between Hezekiah and the past shows the difference that a good leader can
make, prefiguring the climactic renewal with the coming of Christ (Heb. 8:8–12).

31:2 Hezekiah restores the temple service as specified by Moses (Numbers 18) and David (1
Chronicles 23–26). His obedience prefigures Christ's obedience and the obedience of those who
follow Christ (Eph. 4:1–16). See note on 2 Kings 18:5.

32:8 Trusting the Lord to fight prefigures trust in Christ as the victor against the kingdom of evil
(Col. 1:13; 2:15; Heb. 2:14–15).

32:15 See note on 2 Kings 18:30.

32:17 See note on 2 Kings 19:22.

33:7 See note on 2 Kings 21:8.

33:12 See note on 12:6.

34:2 Josiah as a righteous king prefigures Christ.

34:21 See note on 2 Kings 22:13.

35:1 The keeping of the Passover is another high point in serving God (see note on 30:26).

35:4 See note on 31:2.

36:16 God shows his righteous judgment against sin, prefiguring the even greater manifestations of
righteousness in the death and resurrection of Christ and in the final judgment (Rev. 20:11–15). See
notes on 2 Chron. 12:6; 16:9; and 2 Kings 25:9.

36:21 The judgment confirms God's faithfulness to his word, anticipating his faithfulness in Christ.
It also gives the land rest in accordance with Leviticus 25, prefiguring final rest (see note on Lev.
25:4).

36:23 Cyrus's proclamation, prophesied in Isa. 44:28 and recorded in Ezra 1:1–4, shows that God
has not forgotten his people (Rom. 11:1). His continued faithfulness and repeated acts of mercy and
salvation look forward to the coming of Christ as the climax of faithfulness and mercy.

Ezra
The restoration and rebuilding after the exile, in fulfillment of prophecy (1:1), prefigure Christ's
salvation (Col. 1:13) and the building of the church (Matt. 16:18; Eph. 2:20–22). They also look
forward to the consummation of salvation in the new heaven and new earth (Rev. 21:1).

1:1 God's raising of Cyrus prefigures his raising of Christ, who in the gospel sends out the
proclamation to build the new people of God (Isa. 44:28–45:1).

1:5 It is God who empowers the restoration in the people as well as in Cyrus, prefiguring the

http://breachrepairers.webs.com/ 37
empowering of his people through the Spirit (Acts 1:8; 2:1–4; Rom. 8:10–11).

2:1 The detailed record of people shows God's knowledge of individuals and families, symbolizing
his detailed knowledge of those chosen for salvation (Eph. 1:4; Rev. 13:8; 17:8; see note on 1 Chron.
4:1).

3:2 Restoration of true worship of God is central to the restoration as a whole. Sacrificial worship
prefigures the sacrifice of Christ (Heb. 10:1–10).

3:10 Temple building, analogous to what Solomon did (2 Chronicles 3), prefigures Christ's body as
a temple (John 2:19–21), the church as a temple (Matt. 16:18; 1 Cor. 3:16; Eph. 2:20–22), and the
new Jerusalem as a temple (Rev. 21:9–22:5). See Haggai and Zechariah for prophecy relating to the
restoration.

3:11 The singing, using the refrain of 1 Chron. 16:34 and Psalm 136, follows the pattern in 1
Chronicles 25 and looks forward to the praise offered by Christ (Heb. 2:12) and his people (Heb.
13:15).

4:1 The adversaries, incited ultimately by Satan, symbolize opposition to God's purposes for his
people and prefigure opposition to Christ and his people (Matt. 4:1–11; Rev. 12:3–4, 7–17).

5:1 Directives both from prophets and from Cyrus (1:1–4) have a key role in the restoration,
prefiguring the role of God's word in building the church (Eph. 2:20–22; 4:6–16).

6:6 God reverses the plans of the opponents and uses Darius to favor the restoration, prefiguring
God's work in blessing the church (Rom. 8:28; Acts 4:29–31; 8:4).

7:27 Through Ezra and Artaxerxes, God shows his providential blessing on the restoration,
prefiguring his willingness to supply our needs (2 Cor. 9:6–12).

8:31 God provides protection, prefiguring his protection to those in Christ (John 10:27–29).

9:1 Intermarriage was forbidden in Deut. 7:3–4 because it led to idolatry (see Ezra 9:11–14).
Separation prefigures the need for uncompromising allegiance to Christ (Matt. 10:34–39; Luke
14:26–33; 2 Cor. 6:14–7:1).

10:2 See note on 2 Chron. 12:6.

10:3 Families are put away for the sake of holiness, to eliminate compromise with idolatry (Deut.
7:3–4; see note on Ezra 9:1). The superior power of Christ's holiness is such that, in the NT, a
Christian may remain in an unbelieving family with the hope that others may come to know Christ
(1 Cor. 7:12–16).

Nehemiah
The restoration and rebuilding after the exile prefigure Christ's salvation (Col. 1:13) and the
building of the church (Matt. 16:18; Eph. 2:20–22).

http://breachrepairers.webs.com/ 38
1:11 Nehemiah's intercession for the people prefigures Christ's intercession for us before God the
Father (Heb. 7:25).

2:18 Rebuilding Jerusalem prefigures building the church (Matt. 16:18; 1 Cor. 14:4–5, 12; Gal. 4:26;
Eph. 2:20–22).

3:1 God records the names of the builders, indicating his knowledge of each contribution. The
division of labor prefigures the cooperation in the body of Christ (Rom. 12:3–8; 1 Corinthians 12;
Eph. 4:1–16).

4:1 Opposition to building prefigures opposition to the church and to Christians (John 15:18–20).

5:7 God's law through Moses forbids exacting interest from a fellow Israelite (Ex. 22:25; Lev.
25:36). The help to the poor anticipates the church's helping the poor (Acts 2:44–45; 4:32–37; 2
Cor. 9:6–15) on the basis of God's generosity in Christ (2 Cor. 8:9; 9:15).

6:2 Opposition includes deceit as well as mocking and threats (see note on 4:1). This deceit
manifests the deceitfulness of Satan the great enemy (John 8:44; 2 Thess. 2:9–10; Rev. 12:9; 20:3).

7:6 See note on Ezra 2:1.

8:3 Instruction from God's Word plays a key role in building up the people of God. It prefigures the
role of Christ as the Word of God (John 1:1; Rev. 19:13), the role of the gospel (Rom. 1:16–17; 1
Thess. 2:13; 1 Pet. 1:23), and the role of Scripture (1 Tim. 3:13; 2 Tim. 3:16–17; see Psalm 119).

9:8 God's faithfulness is displayed in fulfilling the promise to Abraham (Gen. 12:1–3, 7; 13:14–17;
15:4, 13–21; 17:1–14). His faithfulness to his promises is supremely manifested in Christ (2 Cor.
1:20–22), who has brought everlasting blessings to God's people (Eph. 1:3–14).

9:38 The names indicate the personal commitment of individuals and families, prefiguring personal
commitment to Christ (Acts 2:38–41; see note on Ezra 2:1).

10:29 Obedience to the law anticipates the obligation of disciples of Christ to follow him in
everything (Matt. 10:37–39; Luke 14:25–33; John 14:15, 23). Christ alone is perfectly obedient to
God (Heb. 4:15).

11:1 Jerusalem has a key role as the holy city. In the NT all of God's people are citizens in the
heavenly Jerusalem (Gal. 4:26–28; Phil. 3:20; Heb. 12:22–24).

11:4 The list of names and numbers indicates God's knowledge of the details of individuals and
families. See note on Ezra 2:1.

12:27 The Levites' role in singing was established in 1 Chronicles 25. The celebration anticipates
the celebration and praise to God for the resurrection of Christ (Eph. 5:19–20; Heb. 13:15) and for
the consummation (Rev. 19:1–8).

13:3 See note on Ezra 9:1.

http://breachrepairers.webs.com/ 39
13:15 The people promised to keep the Sabbath in 10:31. The Sabbath is a sign of the covenant
with God (Ex. 20:8–11; 31:12–17), celebrating creation (Ex. 20:11) and redemption (Deut. 5:15). It
points forward to Christ, who is Creator (Col. 1:15–16) and Redeemer (Col. 1:18–20), and who has
prepared our place of rest (John 14:2–3). See notes on Gen. 2:2 and 2:3.

13:23 See note on Ezra 9:1.

Esther
God providentially brings deliverance to his people through Esther, prefiguring final deliverance
through Christ.

1:12 The rejection of Vashti is one step in God's providential acts to deliver the Jews (see note on
2:15). It introduces the key theme of rejection and selection, by which God prepares the way for
salvation.

2:15 God causes Esther the Jew to be chosen, which will later play a key role in delivering the Jews.
Esther in her beauty prefigures the church as the bride of Christ (2 Cor. 11:2; Eph. 5:26–27; Rev.
19:7–8; see note on Est. 1:12).

2:22 God's hand of providence leads to key action from Mordecai, which will later prove important
(6:2). God's providential control illustrates his continual care for his people (John 10:27–29; Rom.
8:28; Eph. 1:22).

3:1 The conflict between Mordecai and Haman is explained 1 Sam. 15:2–3, 32–33. Haman is an
Agagite, an Amalekite, an opponent of Israel and a descendant of the people whom Saul should have
wiped out.

3:6 Haman exemplifies all who oppose God's people, and especially Satan (see Rev. 12:10–12).

4:16 Esther is willing to sacrifice her own life, prefiguring the willingness of Christ to die for us
(Rom. 5:6–11).

5:2 The king's favor toward Esther prefigures the favor resting on Christ as the obedient son of God
who redeems us (Matt. 3:17; 2 Pet. 1:17). It is the turning point in the story, prefiguring the
resurrection as the turning point in redemption.

5:11 Pride goes before destruction (Prov. 16:18). Haman typifies the false confidence of those
belonging to the kingdom of Satan.

6:1 A number of seemingly “chance” events show God's providential control and his power to act
secretly on behalf of his people (see note on 2:22).

7:10 Fitting retribution comes as Haman receives what he would have done to Mordecai (Obad.
15). The retribution prefigures the justice of God's final judgment (Rev. 20:11–15) and the
elimination of the enemies of God's people (Rev. 20:7–10; 21:8, 27).

8:8 The effects of victory now extend to all the Jews, prefiguring the extending of Christ's victory to

http://breachrepairers.webs.com/ 40
those who are his (Rom. 8:10–11; 1 Cor. 15:54–57; Col. 3:1–4).

9:1 The reversal anticipates the reversal of positions with Christ's coming (Luke 1:48–53; 14:11;
18:14) and the justice of God's final judgment (see note on Est. 7:10).

10:3 The blessings to the Jews through Esther and Mordecai prefigure the blessings that come to us
through Christ (Eph. 1:3–14; see note on Est. 8:8).

Job
Job's suffering and relief prefigure the suffering and glory of Christ.

1:1 Job, though not sinlessly perfect, is upright, prefiguring the righteousness of Christ (Heb. 4:16).

1:11 Satan is an accuser of God's people (Rev. 12:10). Redemption in Christ includes giving a final
answer to Satan's accusations, both by justifying the ungodly (Rom. 4:5) and by making the ungodly
into godly people (Rom. 6:4, 15–19; Rev. 19:8; 21:27).

1:21 Job trusts God even though he does not know about Satan's accusation. He exemplifies all who
walk by faith and not by sight (2 Cor. 5:7). Christ as man trusted in God perfectly (Heb. 2:13; 5:7–
10).

2:6 God uses even the works of Satan for his own glory and for the sanctification of his people. God
forbids Satan to take Job's life. But when Christ comes, he is allowed to die at the hands of sinful
men (Acts 2:23). It is the supreme act of trust and of vindication of the name of God, as well as
victory over Satan (John 12:31).

3:3 Intense suffering negates all the meaning of life, underlining the fact that both suffering and
death are horrible effects from the fall (Gen. 3:19). An answer comes only with the meaningful
sufferings of Christ (Phil. 3:10) and his resurrection from the dead, which is the beginning of the
end to all suffering (Rev. 21:4).

4:7 Eliphaz speaks as if God's protection to the righteous were a universal rule. But the mystery of
the death of Christ the innocent one shows the superficiality of his reasoning.

4:15 Eliphaz does not realize that he may have seen an evil spirit who, like Satan, accuses God's
people (see note on 1:11).

4:17 Yes, a man can be pure, as is demonstrated by the purity of Christ. Moreover, Christ gives his
righteousness to his people through justification (Rom. 5:1; 2 Cor. 5:21).

5:13 God catches the wise with the foolishness of the cross, according to 1 Cor. 3:19. Ironically,
Eliphaz, who claims to be wise, is himself caught in his speeches (Job 42:7), because he does not
know the wisdom of the cross, and its meaning for the suffering of the innocent.

5:18 The statement parallels Hos. 6:1. Eliphaz correctly describes God's discipline to sinful people.
But he does not see that God may discipline the innocent for more mysterious purposes (Job 1:12; 2
Cor. 5:21; see note on Job 4:7).

http://breachrepairers.webs.com/ 41
6:15 Job's misery is increased by his friends. It anticipates Christ's betrayal by Judas (John 13:18)
and abandonment by the disciples (Matt. 26:31).

7:17 Note similarities with Ps. 8:4 and Heb. 2:6. God has set his heart on man and brought suffering
with a view to redemption in Christ, but Job cannot see the full picture yet.

8:3 God is just, but his justice is deeper than straightforward rewards and punishments in this life.
The issue of justice points forward to the achievement of justice in the work of Christ (Rom. 3:23–
26) and in the final judgment (Rev. 20:11–15).

9:2 See note on 4:17.

9:14 Job sees the need for an intercessor, anticipating the intercession of Christ (Heb. 7:25).

9:24 The frustration over injustices finds resolution only in the future, with the coming of final
salvation (Rev. 20:11–22:5). In the meantime, the righteous may suffer and the wicked prosper,
anticipating the human injustice in the crucifixion of Christ.

9:30 Isaiah 1:18 gives hope that God will himself makes us white as snow, which he accomplishes
in Christ (Rom. 8:1).

9:33 Christ is both God and man, and will stand in between (1 Tim. 2:5–6; see note on Job 9:14).

10:4 Doubts about whether God sympathizes with man are resolved with Christ's manifestation of
sympathy (Heb. 4:15).

10:11 God's creation of Job shows care and intimacy (see Ps. 139:13–16), anticipating the love
displayed in the incarnation of Christ (John 1:14).

11:17 The life of the righteous will end in bright day (Prov. 4:18), ultimately the day of
consummation (Rev. 21:23–22:5). But Zophar underestimates the complexity. The mysteries of
God's providence lead to consummation only through the sufferings of Christ (1 Pet. 2:21–25) and
his people (Phil. 2:10–11).

12:3 Job's anguish is increased by what he knows concerning God's wisdom and power, because it
seems inconsistent with his sufferings. God's wisdom and power are climactically manifested in the
suffering of Christ (1 Cor. 1:18–25).

13:3 See note on 9:14.

13:15 Job's continued hope anticipates Christ's trust even to the point of death (Matt. 26:38–39).

14:14 Job sees that resurrection is needed to solve the mystery of suffering. He thereby anticipates
the resurrection of Christ (Rom. 4:25) and of Christ's people (John 5:24–25, 29; 1 Thess. 4:13–18).

14:17 Job anticipates forgiveness, which has now been accomplished in Christ (Rom. 4:7–8; 8:1).

15:9 See note on 12:3.

http://breachrepairers.webs.com/ 42
15:14 See note on 4:17.

16:11 Job's abandonment prefigures the abandonment of Christ (Matt. 20:18–19).

16:17 See the parallel in the sufferings of Christ in Isa. 53:9.

16:19 Job anticipates the intercession of Christ, who pleads our cause (Rom. 8:34).

16:21 See note on 9:14.

17:6 The despising of Job anticipates the despising of Christ (Ps. 69:11; Isa. 50:6; Matt. 27:30).

18:21 God will judge the wicked (Rev. 20:11–15). But justice is delayed for the sake of salvation
(Ps. 73:3; 2 Pet. 3:9).

19:7 See the parallel in Hab. 1:2–4. Faith is necessary in waiting for the justice of Christ.

19:19 Job's abandonment by friends anticipates the abandonment of Christ on the cross (Ps. 55:13;
John 13:18).

19:25 Job anticipates both the vindication of Christ's justification (Rom. 4:25) and the open
manifestation of righteousness at the last judgment (2 Cor. 5:10; Rev. 20:11–15).

19:26 Seeing God takes place through seeing Christ, both now (John 14:9) and in the
consummation (Rev. 22:4). See note on Ex. 33:22.

20:29 See note on 18:21.

21:7 A similar struggle is found in Ps. 73:3. See notes on Job 18:21 and 19:7.

22:8 False accusations imitate those of Satan (1:11; 2:5) and anticipate the false accusations against
Christ (Matt. 26:59–60; 27:13; Luke 23:10, 14) and against his people (Rev. 12:10).

23:7 Job's desire for God and for acquittal anticipates the justification that is found in Christ (Rom.
4:25–5:1; 8:1).

24:12 See Ps. 50:21 and note on Job 9:24.

25:4 See note on 4:17.

26:13 God's victory over the serpent anticipates the final victory over Satan through Christ (John
12:31; Rev. 20:7–10). Job knows that God's ways are mysterious, but he continues to hope.

27:5 Job's holding fast to the right anticipates Christ's steadfastness toward God and our privilege
of holding fast to his righteousness (2 Cor. 5:21).

28:12 Job cannot fathom God's ways, but wisdom is found ultimately in Christ (1 Cor. 1:30; Col.
2:3).

http://breachrepairers.webs.com/ 43
28:27 Wisdom was with God even in creation, as in Prov. 8:22–31. The association of wisdom with
creation anticipates the revelation that Christ (the wisdom of God) was with God in the beginning
and was mediator of creation (John 1:1–3; Col. 1:15–17).

28:28 See Prov. 1:7.

29:3 Job's time of blessing anticipated the blessings that come through Christ (John 8:12).

30:10 See note on 17:6.

30:20 The unanswered cries anticipate the abandonment of Christ on the cross (Ps. 22:1–2; Matt.
27:46).

31:1 Job's commitment to God anticipates the integrity of Christ (Heb. 4:15).

32:12 God has put in us a desire for wisdom and understanding that will be satisfied only in Christ
(1 Cor. 1:30; Col. 2:3; see notes on Job 28:12 and 28:27).

33:23 The desire for a mediator anticipates the exclusive mediation of Christ (1 Tim. 2:5–6; see
notes on Job 9:14 and 9:33).

34:11 God's reward or punishment according to justice is a regular theme (e.g., Ps. 62:12; Prov.
24:12; Rev. 2:23; 20:12–13). But final payment awaits the working out of justice and mercy in
Christ (see notes on Job 8:3 and 11:17). God's justice does not endorse a superficial conclusion
about Job's situation.

35:2 See notes on 34:11 and 8:3.

37:5 The wisdom of God is inaccessible, except through Christ (Col. 2:3; 1 Cor. 1:30; see note on Job
28:12).

37:24 The danger of man-centered wisdom is real (as in Prov. 3:7; Rom. 11:25; 12:16) and holds
people back from humbly seeking God and his wisdom in Christ (1 Cor. 1:18–31).

38:4 See note on 28:27.

38:17 Only God has power over death, anticipating the victory of Christ over death (Heb. 2:14–15;
Rev. 1:18).

39:9 Both wisdom and power belong to God but not to man (see note on 12:3).

40:8 Man has a God-given sense of justice, but it is inadequate in the face of the depths of God. The
depths of God's justice and mercy and wisdom are to be revealed in Christ (1 Cor. 1:30; see notes on
Job 12:3 and 28:12).

40:14 Job confronts not only the issue of wisdom and justice, but salvation. Salvation ultimately is
worked out in Christ (1 Cor. 1:30).

http://breachrepairers.webs.com/ 44
41:1 God has power even over the most untamable creature, and ultimately even over Satan, who is
named Leviathan (Isa. 27:1). Christ's victory over Satan (John 12:31) will ultimately answer all the
human frustrations of suffering and injustice (Rev. 21:4).

42:3 Job finds satisfaction in knowing God and his wisdom. Final satisfaction is to be found in Christ
(John 16:33; 17:3; Col. 2:3; Rev. 21:4).

42:10 Job's vindication after his sufferings anticipates the vindication of Christ after his sufferings.

Psalms
By expressing the emotional heights and depths in human response to God, the Psalms provide a
permanent treasure for God's people to use to express their needs and their praises, both
corporately and individually. Christ as representative man experienced our human condition, yet
without sin, and so the Psalms become his prayers to God (see esp. Heb. 2:12; cf. Matt. 27:46 with
Ps. 22:1). The Psalms are thus to be seen as his words, and through our union with him they
become ours.

1:1 God's commitment to bless the righteous is supremely shown when he blesses Christ, the
perfectly righteous man, by raising him from the dead and enthroning him (Phil. 2:10–11).

2:1 The rebellion of the peoples anticipates the rebellion against the message of Christ (Acts 4:25–
27).

2:6 God uses David and other Israelite kings to protect his people against enemies. These kings
prefigure Christ, who is enthroned after his resurrection (Acts 13:33) and now rules on behalf of his
people (Eph. 1:20–22).

2:8 Christ rules over all nations (Matt. 28:18; Eph. 1:21).

2:12 Salvation or damnation depends on one's relation to the Son (John 3:36).

3:1 Protection from earthly enemies prefigures protection from the ultimate evils of Satan, sin, and
death (Heb. 2:14–15). God the Father delivered Christ from his enemies in his resurrection (Acts
3:13–15), and that is the basis for our deliverance (Rom. 4:25).

3:5 Being preserved through the night anticipates the hope of resurrection after the “sleep” of
death (13:3; 1 Thess. 4:13–18).

4:7 The joy of knowing God anticipates the joy and peace that Christ promises (John 15:11; 16:33).

5:4 Sinners cannot stand before God's holiness. Christ's perfection allows us to come into God's
presence and for our prayers for deliverance to be heard (Heb. 10:19–22).

5:9 See Rom. 3:13 and note on Ps. 14:1.

5:12 See note on 1:1.

http://breachrepairers.webs.com/ 45
6:2 Sufferings of God's people ultimately turn out to be analogous, on a lesser level, to the sufferings
of Christ (Ps. 22:14; Phil. 3:10).

7:8 God's justice gives hope for vindication when we are in the right. But in the matter of eternal
salvation, no one is in the right except Christ alone, and in him we take refuge (Rom. 3:23–26).

8:2 Praise from infants anticipates children's praise of Christ (Matt. 21:16).

8:5 God gave Adam a distinguished role (Gen. 1:28–30). But because of the disobedience of Adam
and his posterity (Rom. 5:12–21), it is Christ who fulfills the role and receives glory and honor in his
resurrection and ascension (Heb. 2:5–9).

8:6 Dominion is finally achieved through Christ's reign (1 Cor. 15:25–28; Eph. 1:22; Heb. 2:5–9).

9:13 Deliverance from death anticipates the resurrection of Christ, and through him the
resurrection of his people (1 Cor. 15:42–49; Col. 3:1–4).

10:1 The lack of immediate answers from God frustrates our desire for justice. This frustration
finds its climax in the death of Christ, which from a human point of view was supremely unjust
(Luke 23:14–16). But God answers in the resurrection (Acts 3:13–16), and therefore we hope for
further answers, culminating in the consummation (Rev. 21:4).

10:7 The treachery of man contrasts with the righteousness to be found in Christ alone (Rom. 3:14–
26; see note on Ps. 14:1).

11:4 The Lord's holiness and power, which are supremely revealed in Christ, guarantee an answer
to the distress of his people.

12:6 In the midst of lies from man, God's word is supremely true, anticipating the truthfulness of
Christ (John 14:6), who is able to deliver us from lies (John 8:44–47).

13:1 See note on 10:1.

13:3 See note on 3:1.

13:5 Salvation includes both the deliverance of Christ himself from death in his resurrection (Heb.
5:7) and the deliverance of believers through Christ (Col. 1:13).

14:1 In ultimate terms, none is righteous except Christ, through whom we may be part of the
generation of the righteous (Rom. 3:10–12).

15:2 Fellowship with God in his holiness ultimately requires perfection, which we receive through
the mediation of Christ the final high priest (Heb. 10:19–22).

16:8 God's mercies to David look forward to the climactic answer when Christ does not remain in
the grave but is raised (Acts 2:25–33).

17:2 See note on 7:8.

http://breachrepairers.webs.com/ 46
17:7 Christ above all others waited for God to deliver him from his adversaries (Matt. 26:53; 27:43;
1 Pet. 2:23).

17:15 Awaking may mean awaking from sleep, but it looks forward ultimately to the new life of the
resurrection and seeing God face to face (Rev. 22:4; see note on Ps. 3:5).

18:1 David's song from 1 Samuel 22 has been included in the book of Psalms, indicating its
relevance to the people of God as a whole.

18:4 See note on 9:13.

18:17 Christ's resurrection is the ultimate case of deliverance from enemies.

18:20 See note on 7:8.

18:34 God gives the king effectiveness in war for the sake of defending his people from their
enemies in other nations. OT war prefigures Christ's conquest of all enemies (Matt. 28:18–20; Eph.
1:20–22; Rev. 19:11–21).

18:49 See note on 2 Sam. 22:50.

18:50 Victory to David's offspring ultimately points to the victory of Christ in his resurrection
(Rom. 6:8–10).

19:1 Revelation of God through nature leaves man with no excuse (Rom. 1:18–23).

19:7 The close relation between God's instruction through creation (vv. 1–6) and through his law
(vv. 7–14) anticipates the role of Christ as mediator in creation and redemption (Col. 1:15–20).

20:6 The key to salvation to all the people is salvation to the anointed king. Christ's deliverance in
his resurrection is the foundation for our salvation (1 Cor. 15:17–22).

21:4 The blessing of long life to the king in the line of David anticipates the blessing of eternal
resurrection life that Christ possesses as he sits at the right hand of God (John 11:25; Rev. 1:18).

21:8 See note on 18:34.

22:1 The suffering and abandonment of the psalmist prefigure the suffering of Christ (Matt. 27:46).

22:8 The bystanders mock Christ's trust (Matt. 27:43).

22:18 The soldiers around the cross divide Christ's garments (Matt. 27:35 and John 19:23–24).

22:22 Public praise prefigures Christ praising God to his people for the salvation that God has
accomplished in him (Heb. 2:12).

22:27 The Abrahamic promise of salvation to all nations (Gen. 12:3) will be fulfilled as the message
of Christ's resurrection spreads (Matt. 28:18–20; Luke 24:47; Gal. 3:14).

http://breachrepairers.webs.com/ 47
23:1 Jesus is the good shepherd (John 10:11–18, 27–29) who embodies God's care for his people.

23:4 See note on 9:13.

23:6 Dwelling in the presence of God is fulfilled for Christ personally in his ascension (John 16:10;
Acts 1:9–11) and for believers in the consummation (Rev. 22:4).

24:4 See note on 15:2.

24:7 Heaven is opened to receive Christ in his ascension (Luke 24:51; Heb. 9:24).

25:2 See note on 3:1.

25:4 Christ perfectly followed the path of the Lord (John 5:36; 14:31). Through Christ and his
instruction and through the teaching of the Spirit of Christ believers learn to be disciples and follow
his path (John 14:6; 16:13).

26:1 The ultimate vindication takes place in Christ (1 Tim. 3:16), who perfectly trusted in the Lord
without wavering. In him his people find vindication (Rom. 4:25).

26:12 See note on 22:22.

27:1 Christ is the light of the world (John 8:12).

27:4 Enjoyment of fellowship with God in his presence anticipates the joy of knowing God through
Christ (John 15:11; 16:24; 17:3; Rev. 22:4). Christ opens the way into the heavenly sanctuary (Heb.
10:19–22).

27:11 See note on 25:4.

28:8 Salvation to God's people and salvation to the anointed king go together. Both are fulfilled in
Christ the anointed One (Luke 4:18).

29:3 God's word is powerful to save and to destroy, anticipating the power of Christ the Word (John
1:1) and the power of the gospel (Rom. 1:16; 2 Cor. 2:15–17).

30:2 God's healing from physical sickness anticipates rescue from death (v. 3) and eternal salvation
through the resurrection of Christ (John 5:24; 11:25).

31:5 Trust in God for deliverance anticipates Christ's trust as he dies (Luke 23:46).

32:1 Forgiveness of sins anticipates the sacrifice of Christ as the ultimate basis for forgiveness
(Rom. 4:7–8).

33:6 God's power and wisdom displayed in creation and in providence encourage praise and
encourage hope in his salvation. Instances of temporal salvation look forward to eternal salvation in
Christ (see 33:22; Matt. 1:21; Luke 2:30).

http://breachrepairers.webs.com/ 48
34:8 Experiencing God's goodness anticipates the experience of goodness in Christ (1 Pet. 2:3).

34:12 Christians now imitate Christ the Righteous One (Acts 3:14) in walking in the way of
righteousness (1 Pet. 3:10–12).

34:20 The OT deliverances of the righteous prefigure the deliverance of Christ (John 19:36).

35:3 Small acts of salvation prefigure the climactic salvation in Christ—that Christ is raised from
the dead and that through him we are rescued from sin and Satan (Col. 1:13–14).

35:4 See note on 3:1.

35:18 See note on 22:22.

35:19 Hatred for the righteous prefigures hatred against Christ (John 15:25).

36:1 See Rom. 3:18 and note on Ps. 14:1.

36:8 Joy in God's presence anticipates the joy that Christ gives (John 15:11), which is to be fulfilled
in the consummation (Rev. 19:6–9).

36:11 See note on 3:1.

37:9 In the consummation ultimate blessing will come to God's people and ultimate overthrow to
his enemies (Rev. 20:11–21:8). The first stage of this goal occurs in Christ's resurrection, where he
as our representative inherits the earth (Matt. 28:18) and triumphs over his enemies (Col. 2:15).

38:1 Deliverance from God's wrath comes ultimately through Christ (John 3:36; Rom. 5:1).

38:4 See note on 32:1.

39:4 The threat of death hangs over all human existence and finds relief ultimately only through
the resurrection of Christ (1 Cor. 15:12–26, 35–58).

40:7 The psalmist's eagerness to serve God prefigures the perfection of Christ's willingness and the
perfection of his sacrifice (Heb. 10:5–10).

40:9 See note on 22:22.

41:9 The treachery against the psalmist prefigures Judas's treachery against Christ (John 13:18).

41:12 The eternal enjoyment of God's presence anticipates the resurrection of Christ (Heb. 9:24).

42:7 The waters of suffering threaten death (see Jonah 2:3). Such suffering according to God's will
anticipates the suffering and death of Christ, and the hope for deliverance anticipates his
resurrection.

43:1 See note on 26:1.

http://breachrepairers.webs.com/ 49
43:3 Coming into the presence of God prefigures Christ as our representative coming into heaven
(Heb. 9:12).

44:22 Victory based on Christ's resurrection sustains God's people in the midst of oppression
(Rom. 8:36).

45:6 The kings in the line of David prefigure the reign of God the king through the reign of the
divine Son (Heb. 1:8–9).

45:11 The marriage of the Davidic king prefigures the marriage of Christ to the church (Eph. 5:25–
27).

46:5 The dwelling of God with his people anticipates his coming to dwell with us in Christ (John
1:14; 2:19–21; Eph. 2:20–22).

47:9 The promise of God's subduing the nations is fulfilled in Christ (Matt. 28:18–20; Luke 24:47;
Eph. 1:20–22; Rev. 5:9–10).

48:1 Jerusalem as the holy city prefigures the heavenly Jerusalem (Gal. 4:26; Heb. 12:22–24; Rev.
21:2, 9–10), both as a present reality in Christ and as a future hope.

49:7 Reliance on God is the only solution to death. Such reliance anticipates faith in Christ's
resurrection (Rom. 10:9) and the hope for our future resurrection (1 Cor. 15:42–57; 1 Thess. 4:13–
18).

50:4 God acts to judge, both in preliminary ways and climactically in the final judgment (Rev.
20:11–15).

50:15 True reliance on God is fulfilled both in Christ's trust in God (see note on 31:5) and in our
faith in Christ (Rom. 10:9).

51:1 See note on 32:1.

51:7 Hyssop alludes to cleansing ceremonies (Lev. 14:4; Num. 19:18) that point forward to the final
cleansing from sin through the work of Christ (Heb. 9:19–28).

52:5 See note on 3:1.

52:8 Enjoyment of the house of God in the OT prefigures eternal enjoyment of the presence of God
in Christ, both in this life (John 15:11–16) and in the consummation (Rev. 22:2–4).

53:1 This psalm is very similar to Psalm 14. See note on 14:1.

54:1 The role of the name of God in salvation anticipates the fact that salvation is in the name of
Christ alone (Acts 4:12).

54:4 God's upholding of life prefigures the giving of eternal life in the resurrection of Christ (1 Cor.
15:42–57; Col. 3:1–4).

http://breachrepairers.webs.com/ 50
54:5 See note on 3:1.

55:3 See note on 3:1.

55:13 The treachery of friends anticipates Judas's betrayal of Christ (John 13:18).

56:1 See note on 3:1.

56:3 The psalmist's trust in God anticipates both Christ's trust in the Father during his earthly life
(Heb. 2:13; see note on Ps. 31:5) and Christians' trust in Christ (Acts 16:31).

56:13 Deliverance from death anticipates the resurrection (see note on 9:13).

57:2 God's acts of salvation work out his plan and purpose from all eternity (Eph. 1:3–4, 11).

57:9 The spread of the message of salvation among the nations anticipates the spread of the gospel
message (Luke 24:47; see note on Ps. 22:27).

58:2 Distress over injustice will be satisfied when God brings righteous judgment (58:11). The
longing for justice anticipates the justice accomplished in the resurrection of Christ (Rom. 4:25) and
in the last judgment (Rev. 20:11–21:8). See note on Ps. 10:1.

59:1 See note on 3:1.

59:8 As in 2:4, God will triumph over the rebellious nations through his anointed, the Messiah (2:6–
7; Acts 13:33).

60:12 Earthly foes prefigure the ultimate foes of sin, death and Satan, which are subdued by Christ
(1 Cor. 15:25–28; Eph. 1:20–22; Heb. 2:14–15; see note on Ps. 3:1).

61:7 Blessing to the king is a key to the salvation of God's people as a whole. The king in the line of
David anticipates Christ the king (Matt. 1:1–16).

62:1 Salvation comes from God, not man, anticipating the fact that Christ who brings salvation is
God incarnate (John 1:14; 10:30).

63:2 True satisfaction is to be found in God alone, anticipating the satisfaction and blessing in
Christ (John 15:11; Eph. 1:3–14; Rev. 22:3–5).

63:11 See note on 61:7.

64:2 Wickedness can be all the more dangerous when it is secret and deceitful. The deceit
anticipates Satan's deceitfulness (Rev. 12:9). See note on Ps. 3:1.

65:4 Salvation means enjoying the presence of God. It is accomplished through Christ, the unique
one whom God chooses to come near as our representative (Luke 9:35; Heb. 10:19–22) and
through whom we can come near and be blessed (Eph. 1:3–14).

http://breachrepairers.webs.com/ 51
65:9 The prosperity of the land, which is a blessing to its people, anticipates the prosperity of the
consummation (Rev. 22:1–5).

66:6 God's salvation in the exodus produces hope for further acts of salvation, culminating in
salvation in Christ (Col. 1:13).

67:2 Salvation is to be made known among the nations, anticipating the spread of the gospel to the
nations (Luke 24:47).

68:1 God's arising against his enemies anticipates the resurrection of Christ as a triumph over
demonic enemies (Col. 2:15; Heb. 2:14–15).

68:18 God's ascending to reign anticipates Christ's resurrection and ascension, through which his
enemies are subdued and his people delivered (Eph. 4:8–16).

68:26 Praise is the appropriate response to God's salvation (Eph. 5:19–20; Heb. 13:15; see note on
Ps. 22:22).

69:2 See note on 42:7.

69:9 The zeal of the psalmist prefigures the zeal of Christ for the honor of God's name and God's
house (John 2:17; Rom. 15:3).

69:21 The mercilessness of enemies prefigures the action of the enemies of Christ when he is on
the cross (Matt. 27:48).

69:22 The desire for judgment on God's enemies finds fulfillment in Rom. 11:9–10.

69:25 Retribution for the wicked has an notable fulfillment in the fate of Judas (Acts 1:20).

70:4 Praise and admiration for God's salvation anticipates the praise for the salvation in Christ
(Eph. 1:3–14; 5:19–20).

71:6 The psalmist's trust in God prefigures Christ's trust in the Father (22:8–9) and is also a model
for our trust in Christ (see note on 56:3).

71:11 The enemies prefigure Christ's enemies, who imagine that they have won when Christ is on
the cross.

71:14 See notes on 22:22 and 68:26.

72:1 The king in the line of David has a key role in bringing justice. Justice is climactically achieved
through Christ the king (Matt. 1:1–16; Rom. 3:24–26; 4:25).

72:8 Dominion for the Davidic king is fulfilled in the universal reign of Christ (Isa. 9:6–7; 1 Cor.
15:24–28; Eph. 1:20–21).

72:19 The filling of the earth with God's glory will be fulfilled in the consummation (Rev. 21:22–

http://breachrepairers.webs.com/ 52
27).

73:3 See note on 10:1.

73:17 In the presence of God in the sanctuary one finds an answer to frustration. His presence
anticipates God's presence in Christ (John 1:14; 2:19–21; 14:9–10).

74:3 The destruction of the sanctuary, the place of God's presence, prefigures the destruction of
Christ in death. But God answers and fulfills his promises in Christ's resurrection (2 Cor. 1:20). In
union with Christ we participate in his death and resurrection (2 Cor. 4:7–15; Phil. 3:10–11).

74:10 See note on 10:1.

74:13 God's dividing the sea in the exodus symbolizes his power over chaos and his power to
deliver his people from death. His victory in the exodus anticipates Christ's victory over death and
Satan (Heb. 2:14–15).

75:7 God's providential control of rulers and his preliminary judgments within history give us hope
for climactic judgment. And the climactic judgment began when God lifted up Christ from death to
the highest position (1 Cor. 15:20–28; Phil. 2:10–11).

75:8 See note on 3:1.

76:3 The establishment of peace in God's dwelling place prefigures the peace that Christ brings
(John 16:33), first in reconciling us to God (Rom. 5:1–10), but also in reconciliation with one
another (Matt. 18:15–20; 1 Corinthians 12).

76:9 See note on 50:4.

77:11 Remembrance of God's past acts of salvation, like the exodus (v. 19), strengthen the hope for
present and future salvation. Now we look back on the climactic salvation in the death and
resurrection of Christ (Acts 2:29–41; Rom. 4:25).

78:2 The expounding of the deeper meaning of God's past acts of salvation anticipates the role of
Christ in expounding the meaning of God's ways (Matt. 13:34–35).

78:4 See note on 77:11.

78:17 The rebellious hearts in Israel are ultimately overcome only through the renewal in the heart
that takes place in the new covenant in Christ (Heb. 8:8–13).

78:72 The rebellion in Israel points to the need for a shepherd-king who will guide them. David is a
preliminary fulfillment (v. 70) pointing forward to Christ as the final shepherd (Ezek. 34:23–24;
John 10:11, 14).

79:1 See note on 74:3.

79:9 Ultimate salvation and the glorification of God's name come through Christ (John 13:31–32;

http://breachrepairers.webs.com/ 53
17:1–5).

80:1 Christ is the true shepherd (John 10:11, 14).

80:17 The “son of man,” the key representative for the people of God, is ultimately Christ (Matt.
26:64; see note on Ps. 61:7).

81:1 Praise is the appropriate response to God's salvation (see note on 68:26).

81:13 See note on 78:17.

82:2 The failure of judges to bring justice points to the need for God's ultimate judgment. He has
brought justice in Christ (Rom. 4:25) and will bring ultimate judgment in the consummation (Rev.
20:11–21:8).

82:6 Judges reflecting God's authority (Rom. 13:1) foreshadow Christ, who is the exact image of
God (Heb. 1:3) and is God himself (John 10:34–36).

83:1 See note on 10:1.

83:9 The destruction of Israel's enemies prefigures the destruction of the ultimate enemies—sin,
death, and Satan (Heb. 2:14–15; Rev. 21:4; see note on Ps. 3:1).

84:1 God's dwelling place in the OT prefigures Christ as the dwelling place of God (John 1:14; 2:19–
21), the church as dwelling place through the Spirit (1 Cor. 3:16; Eph. 2:20–22), and the new
Jerusalem as final dwelling place (Rev. 21:2–3, 21:22–22:5). See notes on Ps. 23:6 and 27:4.

85:4 The forgiveness of Israel in the OT anticipates the permanent forgiveness in Christ (Col. 1:13–
14).

86:2 See note on 35:3.

86:9 The coming of the nations to worship is fulfilled in Christ (Luke 24:47; see note on Ps. 57:9).

86:11 See note on 25:4.

87:4 The incorporation of other nations into the holy city is fulfilled as the nations come to Christ
(Luke 24:47; Rev. 5:9–10; 21:24–26).

88:3 The miseries of the psalmist prefigure the sufferings of Christ (Luke 24:26–27; see note on Ps.
22:1).

89:4 The promise concerning offspring is ultimately fulfilled in Christ (Matt. 1:1–16). But victory is
preceded by suffering, abandonment, and apparent failure of the promise, all anticipating the
sufferings of Christ.

89:48 In the resurrection of Christ is the ultimate answer to death (1 Cor. 15:50–57; Heb. 2:14–15).

http://breachrepairers.webs.com/ 54
90:3 See note on 89:48.

90:17 Despite the reality of death, Christ's resurrection guarantees victory and demonstrates that
work has eternal meaning (1 Cor. 15:58).

91:1 God is our ultimate dwelling place and protection, prefiguring Christ as dwelling place and
protection (John 1:14; 10:27–30).

92:1 See note on 68:26.

92:13 Fruitfulness is found in the presence of God (see 1:3). Fruitfulness prefigures the fruitfulness
of Christ (Isa. 53:10) and of his people (John 15:1–16).

93:1 See note on 11:4.

93:4 The Lord's power is greater than the threat of overwhelming waters. The power over waters
threatening death prefigures the power in Christ's resurrection (Eph. 1:19–22; see note on Ps.
42:7).

94:2 See notes on 50:4 and 58:2.

94:3 See note on 10:1.

94:11 The limitations of human thinking contrast with the wisdom of God, which is to be found in
Christ (1 Cor. 3:20; Col. 2:3).

94:15 Final justice, accomplished in Christ, will have benefits for all who are his (1 Cor. 15:42–49).

95:1 See note on 68:26.

95:8 Israel's rebellion (Numbers 14; Deut. 32:5) serves as a negative example for all time (Heb.
4:7–12). Faith in God, culminating in faith in Christ, is the proper response to God (Heb. 4:2).

96:1 See note on 68:26.

96:3 The declaration to the nations anticipates the spread of the gospel (Luke 24:47; Acts 1:8; see
note on Ps. 22:27).

97:2 See note on 7:8.

97:8 God's people can rejoice in judgment, ultimately because Christ has taken away the negative
judgment against their sins and they may receive blessing in him (2 Cor. 5:21).

98:1 See Psalm 96 and note on 68:26.

98:7 Ultimate salvation in Christ includes blessing to all nations (see note on 22:27) and renewal of
the world itself (2 Pet. 3:13; Rev. 21:1).

http://breachrepairers.webs.com/ 55
99:3 See note on 11:4.

99:4 The experience of the benefits of justice make us long for ultimate justice, which is to be found
in Christ and his justification (Rom. 3:23–26; 4:25–5:1). Justice includes both the vindication of
God's people and the removal of enemies. The ultimate enemies are sin, death, and Satan (see note
on Ps. 3:1).

100:4 Entering the presence of God has been made possible through Christ who opened the way
(John 14:6; Heb. 10:19–22).

101:5 The zeal of the Davidic king to remove wickedness prefigures the power of Christ in
triumphing over all evil and making people new (John 13:10; Eph. 4:20–24).

102:3 See note on 6:2.

102:15 See note on 22:27.

102:16 God appears in his glory climactically in Christ (John 1:14; 13:31–32; 17:1–5).

102:26 Through Christ the abiding character of God benefits us (Heb. 1:10–12).

103:4 Earlier redemptions look forward to the climactic redemption in Christ.

104:2 God's people are to praise God for his works of creation and providence, seeing in them
displays of God's power and goodness. His power and goodness and blessing are supremely
manifested in Christ (John 1:14; Eph. 1:3–14).

105:5 The faithfulness of God in past generations encourages Israel to respond in faithfulness.
Christians look back not only on God's acts of salvation in the OT, but on the climactic salvation in
Christ, which gives the ultimate basis for our trust.

106:6 The unfaithfulness of Israel in response to God is answered by Christ's obedience, and then
by the obedience of God's people who follow Christ (John 14:15; Eph. 2:10).

107:2 God's acts of redemption in the OT prefigure final redemption in Christ (Col. 1:13–14).

108:6 See note on 35:3.

108:7 God is committed to subduing his enemies, and this commitment is fulfilled climactically in
Christ, both in his resurrection (Heb. 2:14–15) and in his second coming (Rev. 19:11–21).

109:8 Judas is a chief example of the enemies whom God judges (Acts 1:20; see note on Ps. 69:25).

109:31 Christ, having been himself saved from death in his resurrection, is able to save us from
death (John 11:25; Heb. 2:14–15; Rev. 1:18).

110:1 The Messiah is superior even to David and exercises universal rule (Matt. 22:44–45; Acts
2:34–36; 1 Cor. 15:25–28; Eph. 1:22; Heb. 1:13).

http://breachrepairers.webs.com/ 56
110:4 The Messiah has an eternal priesthood superior to Aaron's (Heb. 5:6; 7:21–8:2).

111:1 See note on 22:22.

111:9 Final redemption and final fulfillment of God's covenant is accomplished in Christ (2 Cor.
1:20; Heb. 7:25; 8:6–13).

112:1 Christ is the supremely righteous man (Acts 3:14), and in him we too receive the reward for
righteousness (Eph. 1:3–14). See note on Ps. 1:1.

112:9 The principle of generosity continues in the NT (2 Cor. 9:9).

113:7 Attentiveness to the needy is supremely manifested in Christ (Luke 1:48–55; 6:20).

114:3 The crossing of the Red Sea (Exodus 14–15) and of the Jordan River (Joshua 3) are acts of
salvation and symbolic triumphs over death that anticipate the triumph of Christ (John 10:18;
11:25; Rev. 1:18; 21:4).

115:1 God is supremely glorified and his faithfulness manifested in Christ's work (John 13:31–32;
17:1–5).

116:3 See notes on 9:13 and 13:5.

116:13 See note on 68:26.

116:15 God continues to care for his saints even after death, hinting at the hope for the
resurrection (John 11:25; 1 Thess. 4:13–18).

117:1 All nations will come to praise God as a result of his salvation in Christ (Rom. 15:11),
fulfilling the promise to Abraham (Gen. 12:3; see note on Ps. 22:27).

118:5 See note on 35:3.

118:6 God has expressed his commitment in Christ, giving us all the more reason to trust him (Heb.
13:6).

118:22 The Lord's exaltation of the one rejected by man is fulfilled in the exaltation of Christ (Matt.
21:42; Luke 20:17; Acts 4:11–12; Eph. 2:20–22; 1 Pet. 2:4–7).

118:26 Israel ought to recognize Jesus as one who brings the salvation of God (Matt. 23:39).

119:1 People with renewed hearts delight to obey God and learn from his word, which guides
them. Christ was perfectly obedient to God (Heb. 10:7–10), and through his Spirit we are
transformed into his image (Rom. 8:9–17; 2 Cor. 3:18) and become obedient servants of God.
Delight in God's word anticipates delight in Christ, who is the Word of God (John 1:1).

119:11 Having God's word in the heart anticipates the new covenant (Heb. 8:10–13; 10:16–18).

http://breachrepairers.webs.com/ 57
120:1 See note on 35:3.

120:2 Deliverance from deceit anticipates the purity of God's word and God's work of deliverance
from Satanic deceit through Christ (Rev. 12:9; see note on Ps. 64:2).

121:2 Salvation comes from God alone, anticipating the fact that Christ is the divine Savior.

122:1 Joy in experiencing the presence of God in his house anticipates the joy of the presence of
God in Christ (John 1:14; 15:11; see note on Ps. 27:4).

122:6 Jerusalem as the city of God prefigures the heavenly Jerusalem (Gal. 4:26–28; Heb. 12:22–24)
of which we are citizens (Phil. 3:20). Christ has given peace to his people (John 16:33; Eph. 4:3; Col.
3:15).

123:2 Mercy is received ultimately through Christ (Eph. 2:4; see note on Ps. 121:2).

124:4 See note on 42:7.

125:1 Trust in the Lord anticipates trust in Christ (Acts 16:31), who has supremely manifested
God's faithfulness.

126:1 Relief from misfortune prefigures the great salvation in Christ (John 16:20–22).

127:1 The necessity of the Lord's power for temporal achievements anticipates the necessity for
God, and him alone, to accomplish eternal salvation through Christ (John 15:4–5; Acts 4:12).

128:1 See note on 112:1.

128:2 Temporal blessings prefigure the eternal blessings in Christ (Eph. 1:3–14; Rev. 21:1–4).

129:1 See note on 6:2.

129:5 See note on 60:12.

130:4 Forgiveness is ultimately accomplished in Christ (Col. 1:13–14; see note on Ps. 32:1).

131:1 The psalmist's humble trust anticipates the humble trust of Christ in the Father (Matt. 11:29;
Heb. 5:7–10) and the trust that Christians are to have in Christ (Acts 16:31).

132:12 The promise to David culminates in Christ the offspring of David (Matt. 1:1–16), who is
both king in the line of David and priest in God's heavenly dwelling (Ps. 110:2, 4; Heb. 8:1–2).

133:1 Unity among God's people is produced in Christ and in his Spirit (Eph. 4:1–6).

134:1 Praise of God looks forward to the praises offered by Christ (Heb. 2:12), the praises of God's
NT people (Eph. 5:19–20; Heb. 13:15), and the praises of the consummation (Rev. 19:1–10).

135:4 God's acts of grace and salvation to his people in the OT anticipate the climactic salvation

http://breachrepairers.webs.com/ 58
accomplished in Christ (Luke 2:30–32; Acts 4:12).

136:4 God's works of creation, providence, and merciful deliverance show the steadfast love that
has now been climactically revealed through salvation in Christ (John 1:14).

137:6 Devastation to God's holy city makes people long for future blessing and destruction to God's
enemies. God's ultimate answer is found in salvation in Christ and in the last judgment (Rev. 20:11–
21:8). Jerusalem prefigures the heavenly Jerusalem (Gal. 4:26–27; Heb. 12:22–24).

138:3 See note on 35:3.

138:4 See note on 22:27.

138:6 Mercy to the lowly comes in Christ (Luke 1:48–55).

139:1 Detailed knowledge and care for the psalmist anticipates God's care for us (John 10:14–16).

140:1 Deliverance from enemies prefigures Christ's deliverance from his enemies, both human and
demonic (Matt. 26:46; Col. 2:15); it also prefigures our deliverance in Christ from sin, death, and
Satan (Heb. 2:14–15).

140:3 See Rom. 3:13 and note on Ps. 14:1.

141:3 The need for wise speech, in prayer as well as in other circumstances, anticipates the purity
of Christ's speech (John 8:43–47) and the purity that we receive from Christ (John 17:17–19). Our
prayers are heard because of him (John 14:13–14; 1 John 5:14–15).

142:4 See notes on 6:2 and 22:1.

142:6 Deliverance from persecutors anticipates the deliverance of Christ from his persecutors,
after he was brought low in his crucifixion and death.

143:2 Perfect righteousness is found only in Christ, who provides righteousness for those who are
his (2 Cor. 5:21; see notes on Ps. 7:8 and 14:1).

144:1 See note on 18:34.

144:10 Deliverance for David prefigures final deliverance given to Christ the offspring of David. See
notes on 2:6 and 18:50.

145:1 See note on 68:26.

145:8 The Lord's grace and mercy is climactically poured out in the salvation in Christ (Rom. 8:32).

146:3 Mere man cannot save, pointing to the need for Christ to be God as well as man (John 1:14).

147:5 God's greatness and goodness, in both providence and redemption, motivates praise and
trust. God's goodness has now been supremely manifested in Christ (Rom. 8:32).

http://breachrepairers.webs.com/ 59
148:3 The created world declares the character of its maker (19:1–6), anticipating the final, even
more glorious praise in the consummation (Rev. 21:1–4). The creation reflects the glory of the Son,
who is mediator of creation (John 1:1–3; Col. 1:15–17).

149:4 See note on 68:26.

149:7 At Christ's second coming rebellious nations will be subdued (Rev. 19:11–21). In the
meantime, gracious subduing comes through the power of the gospel (Matt. 28:18–20).

150:2 See note on 68:26. Praise, not a cry of distress, has the final position in the Psalms,
anticipating the victory of Christ (Eph. 4:8) and the final abolition of suffering (Rev. 21:4).

Proverbs
Wisdom ultimately comes from God and his instruction, which anticipates the fact that Christ is the
wisdom of God (1 Cor. 1:30; Col. 2:3) and that in him and his instruction we find the way of life and
righteousness (John 14:6, 23–24). Through the Spirit we may walk in the right way (Gal. 5:16–26).

1:1 Solomon's wisdom prefigures the wisdom of his greater descendant, Jesus Christ (1 Cor. 1:30;
see note on 1 Kings 2:6).

1:7 Wisdom is to be sought from God, anticipating that we seek wisdom from Christ, the incarnate
God (John 1:14; Col. 2:3).

1:8 Listening to parents is one aspect of honoring them, which is an abiding principle (Ex. 20:12;
Eph. 6:1–3). Within the church we are now to have specifically Christian instruction of children
(Eph. 6:4). The archetype for this obedient listening is found in the relation of the Son of God to the
Father (John 8:28–29).

1:18 The principle of just retribution is broad (Obad. 15) and is to be fulfilled ultimately in the
consummation (Rev. 20:12–14).

1:19 Sin leads to death (Rom. 6:23), but in Christ there is life (John 14:6; 1 John 5:12).

1:20 The call of wisdom prefigures the call of the gospel, which contains the wisdom of God (1 Cor.
1:18–25; 2:6–10).

2:4 The diligent seeking for wisdom prefigures the need to seek the kingdom of God (Matt. 13:44).

2:13 The path of righteousness is ultimately that of Jesus Christ, the perfectly Righteous One (John
14:6). All other ways lead to destruction (Matt. 7:13–14; Acts 4:12).

2:16 Wisdom involves the avoidance both of literal adultery and of the spiritual adultery of idolatry
(Ex. 34:16; Hos. 1:2; 2:1–5; 3:1–3; 2 Cor. 11:3).

2:21 Temporal blessings prefigure the blessings of eternal salvation (Eph. 1:3–14).

3:2 Length of days prefigures eternal life that comes through fellowship with Christ, who is the

http://breachrepairers.webs.com/ 60
wisdom of God (1 Cor. 1:30).

3:5 Trust in the Lord anticipates trust in Christ, who is the Lord's salvation (Acts 16:31).

3:11 Christians as sons of God receive the Lord's discipline (Heb. 12:5–6).

3:18 The possession of the tree of life anticipates the final inheritance in the consummation (Rev.
2:7; 22:1–2).

3:34 The call for humility anticipates the role of humility in the NT (Matt. 11:29; James 4:6; 1 Pet.
5:5).

4:13 Instruction for the path of life anticipates the instruction of Christ, who is the way and the
truth and the life (John 14:6).

5:3 See note on 2:16.

5:5 Ultimately Christ delivers us from death (John 11:25–26), and as one aspect of deliverance he
gives wisdom and integrity of heart (1 Cor. 1:30).

6:6 Diligent work now has as its deepest motivation the hope of final satisfaction in Christ (1 Cor.
15:58).

6:24 See note on 2:16.

7:21 Smooth, deceitful talk is linked ultimately to the deceit of Satan (John 8:44–47; Rev. 12:9).

8:1 See note on 1:20.

8:22 The eternality of wisdom with God anticipates the eternality of the second person of the
Trinity, who is the Word of God and who mediated creation (John 1:1–3).

8:35 Life is obtained ultimately from Christ, who is the life (John 14:6) and the wisdom of God (1
Cor. 1:30).

9:2 The invitation to feasting anticipates the spiritual food of Christ (John 6:52–58) and the future
marriage supper of the Lamb (Rev. 19:9).

9:18 See note on 1:19.

10:1 On Solomon, see notes on 1:1 and 1 Kings 2:6.

10:2 Blessings on the righteous anticipate the blessings on Christ, the perfectly righteous man, and
the blessings that come to those in Christ (Eph. 1:3–14).

10:12 Wisdom transforms relations with others, anticipating the NT transformation through love
(John 13:34–35; 1 John 3:16–18; 4:7–21).

http://breachrepairers.webs.com/ 61
10:21 The blessing to others anticipates the blessing of gracious words in the church (Eph. 4:14–
16; Col. 3:16; 4:6).

11:2 The value of humility anticipates the humility of Christ (Matt. 11:29) and of his people (Luke
14:11; Eph. 4:2; see note on Prov. 3:34).

11:3 See note on 2:13.

11:4 Temporary avoidance of death prefigures the promise of eternal life, based on the
righteousness of Christ (John 5:24; Rom. 4:25; see note on Prov. 2:13).

12:18 The blessing of wise words anticipates the blessings of the words of Christ (John 6:63) and of
his followers (Eph. 4:29; Col. 4:6).

13:4 See note on 6:6.

13:14 Christ the supremely wise One has the words of eternal life (John 6:68–69).

13:24 Christians are to train their children in Christ (Eph. 6:1–4; see note on Prov. 1:8).

14:2 True trust in Christ manifests itself in obedience (Gal. 5:13–26; James 2:14–26).

15:1 Gentle words anticipate the gentleness of Christ (Matt. 11:29). Gentleness is also to
characterize his people (Gal. 5:23; Eph. 4:2, 25–29).

16:3 Only through union with Christ can we bear fruit (John 15:1–11).

16:12 The duty of kings to bring justice anticipates Christ, who is the great king and the one who
brings perfect justice (Rom. 3:26; Rev. 19:11).

17:3 The Lord's discernment is perfect (Heb. 4:12–13), implying the need for purification (Heb.
9:9–14).

18:3 Temporal judgments on wickedness prefigure final judgment (Rev. 20:11–15), underlining the
need for repentance.

19:1 We must be discerning about real value and seek first the kingdom of God (Matt. 6:33).

19:5 See note on 18:3.

19:11 Readiness to forgive anticipates the forgiveness of Christ (Col. 1:14) and the practice of
forgiving among his people (Col. 3:13; James 1:19).

20:8 Authorities have an obligation to punish evildoing (Deut. 16:18–20; Rom. 13:1–4). In this they
anticipate the final judgment of God (Rev. 20:11–15).

20:22 Vengeance belongs to God (Rom. 12:17–21). Christ himself waited patiently for vindication
(1 Pet. 2:21–23).

http://breachrepairers.webs.com/ 62
21:3 See 1 Sam. 15:22–23 and Mic. 6:6–8. The requirement for real obedience, and ultimately for
perfect obedience, is fulfilled in Christ (Heb. 10:5–10).

22:4 See note on 2:21.

23:4 Counsel against lust for money anticipates Jesus' counsel about true riches (Luke 12:22–40;
16:10–13; Eph. 5:5).

23:13 See note on 13:24.

23:19 The way of righteousness is found ultimately in Christ (John 14:6). See note on Prov. 1:8.

23:30 The warning against drunkenness is repeated in the NT, and it is complemented by a positive
command to be filled with the Spirit (Eph. 5:18).

24:3 Human use of wisdom imitates God's use of wisdom (8:22–31) and anticipates Christ, who is
the wisdom of God (1 Cor. 1:30) and who builds the church (Matt. 16:18).

24:19 See note on Ps. 10:1.

24:30 See note on 6:6.

25:7 The principle of humility is further developed in Christ's teaching and example (Luke 14:7–
11).

25:11 See notes on 12:18 and 15:1.

25:21 The principle of doing good to enemies is further developed in Christ's example and his
teaching (Matt. 5:43–48; Rom. 12:20–21).

26:3 The answer to folly and its disasters is found in seeking the wisdom of Christ (1 Cor. 1:30; Col.
2:3).

26:11 It is folly to turn back from following Christ (2 Pet. 2:22).

26:13 See note on 6:6.

26:20 The answer to words of strife is found in Christ's peace and his empowering of his people to
be at peace with one another (Col. 3:13–15).

27:3 See note on 26:20.

27:11 See note on 13:24.

28:1 The boldness of the righteous anticipates the boldness of followers of Christ (2 Cor. 3:12; Phil.
1:28–30).

28:2 See note on 16:12.

http://breachrepairers.webs.com/ 63
28:9 God desires righteousness and obedience, which are fulfilled in Christ (2 Cor. 5:21; see note on
Prov. 21:3).

29:2 See note on 16:12.

29:25 Trust in the Lord anticipates trust in Christ (see 3:5).

30:4 The inaccessibility of wisdom to man points to the need for Christ, who comes down from
heaven (John 3:12–15; 6:33, 50–51).

31:3 See note on 2:16.

31:10 The excellent wife prefigures the excellence of the church, the bride of Christ (Eph. 5:25–27;
Rev. 19:7–8).

Ecclesiastes
The meaninglessness, frustrations, and injustices of life “under the sun” call out for a solution from
God. Christ through his suffering and resurrection provides the first installment (1 Cor. 15:22–23)
of meaning, fulfillment, and new life (John 10:10), to be enjoyed fully in the consummation (Rev.
21:1–4).

1:14 The crumbling of human works makes life pointless, unless there is relief in God. Knocking
down false ambitions creates a longing for the relief that will come in Christ (Matt. 11:28–30).

2:10 The fading pleasures in this life contrast with the eternal pleasures in God's presence (Ps.
16:11; John 15:11; Rev. 21:4).

2:14 Wisdom in this world contrasts with the wisdom in Christ that will last forever (1 Cor. 1:30).

2:16 What is needed is a remedy for death, and this remedy comes through Christ (1 Cor. 15:54–
58).

3:11 Now in the light of revelation we can know that God's purpose is to unite all things in Christ (1
Cor. 2:9–10; Eph. 1:10).

3:12 Man need not understand everything but can live a life of joy as a servant of Christ (John
15:11), trusting that God's plans are good (Rom. 8:28).

3:17 God will execute final judgment (Rev. 20:11–21:8). But in the meantime we must endure much
injustice (John 16:33).

3:20 See note on 2:16.

4:1 See note on 3:17.

4:9 The virtue of cooperation anticipates the mutual help in the body of Christ (1 Corinthians 12).

http://breachrepairers.webs.com/ 64
5:8 See note on 3:17.

5:10 The fleeting character of riches implies that we should invest in God's kingdom (Matt. 6:33;
Luke 12:22–34).

7:2 See notes on 2:10 and 2:16.

7:15 See note on 3:17.

7:18 In the midst of much confusion and frustration about outward circumstances, we must hold
fast to God. God brings ultimate salvation from vanity in Christ (Rev. 21:1–4).

8:14 See note on 3:17.

8:15 See note on 3:12.

9:5 See note on 2:16.

9:7 See note on 3:12.

9:16 The seeking for wisdom ultimately culminates in Christ, who is the wisdom of God (Matt.
12:42; 1 Cor. 1:30).

10:17 Good rulers make a notable difference in the character of a nation. The final, perfect ruler is
Christ himself, who brings the kingdom of God and everlasting righteousness (Matt. 12:28; Rev.
21:1–4).

11:1 Work done for Christ will be rewarded (Col. 3:22–25).

12:1 See note on 1:14.

12:7 Reckoning with death leads to abandoning a focus on selfish achievement and pleasure and
seeking God (see note on 2:16).

12:14 Reckoning with the final judgment (Rev. 20:11–21:8) changes the orientation of life. We are
to follow Christ who delivers us from condemnation (Rom. 8:1) and death (John 11:25–26) and
gives meaning to work in fellowship with him (1 Cor. 15:58).

Song of Solomon
The Song of Solomon depicts marital love. But after the fall merely human love is always short of
God's ideal, and so we look for God's remedy in the perfect love of Christ (Eph. 5:22–33; 1 John
3:16; 4:9–10). The connection with Solomon (Song 1:1; 3:7, 9, 11; 8:11) invites us to think
especially of the marriage of the king in the line of David (Ps. 45:10–15), and the kings point
forward to Christ the great king, who has the church as his bride (Rev. 19:7–9, 21:9).

1:1 The marriage of the Davidic king points forward to Christ (Ps. 45:10–15; cf. Ps. 45:6–7 with
Heb. 1:8–9).

http://breachrepairers.webs.com/ 65
1:2 Perfect love has been demonstrated in Christ (1 John 4:9–10).

1:4 Longing for intimacy prefigures the longing for intimacy with the love of Christ (1 John 4:7–21).

1:15 The beauty of the lovers anticipates the beauty of Christ and his bride (Eph. 5:26–27; Rev.
19:7–8).

2:3 Delight in love prefigures the joy in Christ (John 15:11).

2:16 The possession of the loved one prefigures the possession of Christ and the church.

3:1 See note on 1:4.

3:11 The wedding of Solomon prefigures the wedding of the Messiah (Ps. 45:10–15).

4:1 See note on 1:15.

4:13 Edenic abundance in the “garden” anticipates the abundance and satisfaction and fulfillment
of the final consummation (Rev. 22:1–5).

5:1 Satisfaction with the loved one contrasts with God's dissatisfaction with the disobedience and
disloyalty of Israel (Isa. 5:1–4), who was supposed to be married to the Lord (Ezek. 16:8–15). The
remedy is found in Christ's salvation (Eph. 5:25–27).

5:8 See note on 1:4.

5:10 See note on 1:15.

6:9 The focus on the beloved anticipates the uniqueness of God's love for the church.

7:1 See note on 1:15.

7:6 Delight in the loved one prefigures Christ's delight in the church (Eph. 5:26–27; Rev. 19:8).

8:6 The abiding character of commitment in love prefigures the abiding character of the new
covenant (John 10:27–29; Phil. 1:6; Heb. 8:8–13).

Isaiah
Isaiah prophesies exile because of Israel's unfaithfulness. But then God will bring Israel back from
exile; this restoration prefigures the climactic salvation in Christ. Christ as Messiah and “servant” of
the Lord will cleanse his people from sin, fill them with glory, and extend blessing to the nations.
Christ fulfills prophecy in both his first coming and his second coming.

1:1 God gives the prophecies during the time covered in 2 Kings 15–20 and 2 Chronicles 26–32.

1:4 The failures of Israel precipitate the exile, and indicate the need for the messianic servant of the
Lord, who will faithfully obey the Lord (42:1–4; 49:1–12).

http://breachrepairers.webs.com/ 66
1:9 The Lord preserves a few, a remnant for Israel. The theme of the remnant is fulfilled in Christ,
who is the ultimate remnant of one, and then the remnant is expanded to include Christ's people
(see Rom. 11:5 and note on Isa. 6:13).

1:18 Ultimate cleansing comes through Christ's sacrifice (Heb. 10:1–10).

2:2 Christ himself is the ultimate “house” or dwelling place of God (John 1:14; 2:19–21). Through
him the church becomes a temple (1 Cor. 3:16; Eph. 2:20–22), and through Christ's exaltation the
nations are drawn to him (Luke 24:47; John 12:32).

2:6 See note on 1:4.

2:11 The humbling of human pride takes place in Christ (Matt. 20:25–28; Luke 1:48–53; 1 Cor.
1:31).

3:2 The lack of adequate leaders shows the need for the Messiah as the final, perfect leader (9:6–7).

4:4 Cleansing looks forward to the forgiveness and purification in Christ (Col. 1:13–14; Heb. 10:10–
14).

5:7 The lack of fruit from Israel contrasts with the fruitfulness of Christ and those in him (John
15:1–6; see also Matt. 21:33–44).

6:1 Isaiah's vision of the glory of God anticipates the glory of God in Christ (John 1:14; 12:41; Rev.
4:2–10).

6:9 The resistance of Israel to Isaiah's message anticipates resistance to the gospel (Matt. 13:11–17;
Acts 28:24–28; Rom. 11:7–8).

6:13 The holy seed, the remnant, are those in Israel who remain faithful to God. Ultimately none is
completely faithful except Christ, who is the final remnant (11:1; Gal. 3:16; see note on Isa. 1:9).

7:14 The prophecy concerning Immanuel (see also Gen. 3:15) is fulfilled in Jesus Christ (Matt. 1:20–
23). It is related to the larger OT theme in which God brings new life and offspring to barren women
(see note on Gen. 18:10).

8:13 Treating the Lord as holy culminates in the holiness of Christ (Acts 2:27) and our obligation to
holiness (1 Pet. 1:15–16; 3:15).

8:14 The nation of Israel being offended by the Lord Almighty prefigures their rejection of Christ
(Matt. 21:43–44; Rom. 9:31–33; 1 Pet. 2:6–8).

9:1 Jesus brings light by preaching in Galilee (Matt. 4:12–17). He is the light of the world (John 1:5,
8–9; 8:12; 9:5).

9:6 The Messiah is both human (from the line of David) and divine (see John 1:14; Col. 2:9).

9:7 The Messiah establishes his rule in justice (Rom. 3:26; Eph. 1:20–22) and peace (John 16:33).

http://breachrepairers.webs.com/ 67
10:22 In NT times, the remnant consists of those who believe in Christ (Rom. 11:1–10; see note on
Isa. 1:9).

11:1 The Messiah is from the line of Jesse, the father of David (1 Sam. 16:1). He is filled with the
Spirit (Matt. 3:16; Luke 4:18), with wisdom (Col. 2:3), and with justice (Rev. 19:11).

11:10 Christ draws the nations to himself (John 12:32; Rom. 15:12; see note on Isa. 2:2).

12:1 The song of praise for God's salvation anticipates the praise for God's salvation in Christ (Eph.
5:19–20; Heb. 2:12; 13:15; Rev. 19:1–8).

13:6 The day of the Lord is a day of judgment. Judgments within history, such as the judgment of
the exile of Israel, anticipate the final judgment (1 Thess. 5:2–11; 2 Pet. 3:10–13; Rev. 20:11–21:8).
Because of Christ's salvation, the day is a day for which Christians hope (Titus 2:13).

13:9 All sinners will be swept away in the ultimate judgment. We must take refuge in Christ (2 Cor.
5:21).

13:10 The darkening is a symbol of judgment, prefiguring judgment at the crucifixion (Matt. 27:45)
and at the second coming (Matt. 24:29; Rev. 6:12–13; see Rev. 8:12).

14:4 The fall of Babylon to the Medes and Persians (Dan. 5:28) prefigures the final fall of Babylon
the Great (Rev. 17:15–19:3) and the defeat of Satan (Luke 10:15; Rev. 12:7–9; 20:10), as well as
looking back on the fall of Babel (Gen. 11:1–9).

15:1 Moab, one of the traditional enemies of Israel (Num. 22:1–6), is defeated, prefiguring final
judgment on God's enemies (Rev. 20:11–15) and fulfilling Num. 21:29.

16:5 Despite her record of enmity, Moab (like other hostile nations) can find refuge in the Messiah.
Christ's mercy extends to all nations (Acts 1:8; Rev. 5:9–10).

17:6 See note on 1:9.

17:7 God the Maker is seen in Christ (John 14:9) and will be seen face to face by the pure in heart
(Matt. 5:8; Rev. 22:4).

18:7 The coming of the nations takes place as Christ draws them (Matt. 28:18–20; John 12:32; Acts
1:8; see note on Isa. 2:2).

19:18 Egypt, traditionally an enemy to God's people, will come to submit to God. Christ calls the
nations to himself (Acts 2:10; see notes on Isa. 2:2 and 18:7).

20:6 The failure of human hopes highlights the need to hope in God through the way that he has
provided in Christ (Ps. 146:3–4; John 14:6).

21:9 The fall of Babylon prefigures the defeat of all evil and the victory of Christ over evil (Col. 2:15;
Rev. 14:8; 18:2; see note on Isa. 14:4).

http://breachrepairers.webs.com/ 68
22:11 A basic temptation is to trust in man rather than in God (Acts 4:12; 16:31; see note on Isa.
20:6).

22:13 Abandonment of hope would be appropriate only if God did not provide salvation in Christ
(1 Cor. 15:19, 32).

22:22 Kingly authority in the right hands provides security. But even Eliakim (v. 20) is ultimately
not up to the task (v. 25). Only the Messiah in the line of David can bear the full weight of
responsibility that will bring final salvation (Matt. 1:21; see Rev. 3:7).

23:9 After destroying human pride, the Lord brings about blessing and glory to himself (v. 18). The
reversal of human ambitions takes place preeminently in the death and resurrection of Christ (Phil.
2:6–11; see note on Isa. 2:11).

24:6 In fulfillment of the curse from the fall of Adam, all the earth will ultimately be judged (2 Pet.
3:10; Rev. 20:11–15). But through the work of Christ blessing comes to the godly (Isa. 24:15; Rev.
21:3–4).

25:8 God's overwhelming victory, resulting in blessing, will come at the consummation (1 Cor.
15:54; Rev. 7:17; 21:4).

26:4 Trusting in God anticipates trusting in Christ, who has accomplished climactic salvation (Phil.
4:7).

26:5 See notes on 2:11 and 23:9.

26:19 The hope for reversal of death is fulfilled in Christ's resurrection (John 11:25–26; 1 Cor.
15:46–57; Eph. 5:14).

27:1 Satan will be completely defeated (John 12:31; Rev. 20:10).

27:6 Fruitfulness is found ultimately in Christ (John 15:1–17).

28:1 See note on 2:11.

28:11 The foreign tongue is analogous to speaking in tongues in the NT (1 Cor. 14:21).

28:16 Christ is the stone, both providing a foundation to those who trust in him (Eph. 2:20–22; 1
Cor. 3:11; 1 Pet. 2:4) and becoming a cause of stumbling to those who reject him (Matt. 21:42–44;
Rom. 9:31–33; 1 Pet. 2:6–8; see Ps. 118:22).

29:10 Spiritual hardness comes to part of Israel in Rom. 11:7–8 (see note on Isa. 6:9).

29:13 The stubbornness and disobedience of God's people comes to a climax with the opposition to
and rejection of Jesus (Matt. 15:8–9; see Col. 2:22).

29:14 Human wisdom is confounded by the gospel (1 Cor. 1:18–25).

http://breachrepairers.webs.com/ 69
29:18 Jesus' healing of the blind and the deaf symbolizes the giving of spiritual light (John 9:39–
41).

30:2 See note on 22:11.

30:20 Christ is the ultimate teacher who instructs us in the way of the Lord (Matt. 23:10) through
the Spirit (John 16:12–15).

31:1 See note on 22:11.

31:5 The protection of Jerusalem prefigures God's protection of his people in Christ (John 10:27–
29; see Isa. 40:11).

32:3 See note on 29:18.

32:15 The blessings of salvation in Christ come in two stages, in his first coming (Acts 1:8; Eph. 1:3–
14) and his second coming (Rev. 21:1–22:5).

33:6 See note on 32:15.

33:14 Only perfect righteousness will remedy sin. Such righteousness is found in Christ (Rom.
3:21–26; 2 Cor. 5:21; Heb. 10:1–14; see Heb. 12:29).

34:2 God's judgment against sin and evil anticipates the final judgment (Rev. 20:11–15; see notes
on Isa. 13:6; 13:9; and 15:1).

34:4 The skies disappear at the second coming (Rev. 6:13–14; 20:11).

35:3 The call for strengthening occurs in responding to God's NT discipline (Heb. 12:12).

35:5 Christ gives sight and hearing, symbolizing the giving of spiritual sight and hearing (Luke
7:20–22; Acts 26:18; see note on Isa. 29:18).

35:10 The return to Palestine in the restoration prefigures the coming to heavenly Jerusalem and
the presence of God in heaven (Heb. 10:19–23; 12:22–24; Rev. 21:4).

36:1 The threat from Assyria anticipates the later threat from Babylon (39:6), which in turn
illustrates all the attacks of Satan on God's people. God answers with redemption that prefigures
redemption in Christ. See the parallels in 2 Kings 18:13–20:19 and 2 Chronicles 32.

36:15 See note on 2 Kings 18:30.

37:1 God is our refuge in time of distress (Ps. 46:1). We now look to Christ for salvation (Acts 4:12).

37:23 See note on 2 Kings 19:22.

38:5 See note on 2 Kings 20:5.

http://breachrepairers.webs.com/ 70
38:10 The ultimate remedy for death is found in the resurrection of Christ (see note on 26:19).

39:6 God's judgment of exile, which comes on account of sin (2 Kings 23:26–27; 2 Chron. 36:15–
16), prefigures final judgment (Rev. 20:11–15). But through Christ we may escape condemnation
(Rom. 8:1).

40:1 Comfort to Jerusalem prefigures the comfort that is found in Christ (2 Cor. 1:3–7).

40:3 John the Baptist uses these words to announce the coming of the Lord (Matt. 3:3; John 1:23) in
the person of Christ (John 10:30; 14:9).

40:5 The glory of the Lord is revealed in Christ (Luke 2:32; John 1:14; 13:31–32; 17:1–5).

40:6 The fading of human life contrasts with eternal salvation in Christ (1 Pet. 1:24–25; see James
1:10–11).

40:11 Jesus is the good shepherd (John 10:11, 14).

41:17 God's mercy to the poor is manifest in Christ (Luke 4:18–19; 7:22).

42:1 The servant, the Messianic king (9:6–7), rules with justice and mercy (Matt. 12:17–21; see
Matt. 3:17).

42:6 Christ the light of the world (John 8:12; 9:5) brings light to the nations (John 12:32; Acts
26:18, 23), fulfilling the promise to Abraham of blessing to the nations (see note on Gen. 12:3).

43:25 Forgiveness is found ultimately in Christ (Mark 2:7; Col. 1:14; Heb. 10:1–18).

44:3 See note on 32:15.

44:28 The restoration under Cyrus (Ezra 1) prefigures the eternal salvation in Christ in the
heavenly Jerusalem (Heb. 12:22–24).

45:1 Cyrus as anointed by God prefigures the Messiah and his salvation (Luke 4:18–19).

45:23 The submission of the nations is accomplished in Christ (Phil. 2:10–11; Rev. 15:4).

46:1 The worthlessness of idols is expressive of the principle that only Christ, the one way of
salvation that God has established (John 14:6; Acts 4:12), is worthy of trust.

47:3 The oppressor of God's people will be judged (see note on 14:4).

47:8 See Rev. 18:7 and note on Isa. 14:4.

48:20 Rescue from Babylon prefigures rescue from sin and death (Col. 1:13–14; Rev. 18:4).

49:2 The word of God is like a sharp sword (Eph. 6:17; Heb. 4:12; Rev. 1:16; 2:12, 16; 19:15).

http://breachrepairers.webs.com/ 71
49:6 See Acts 13:47 and 26:23, and note on Isa. 42:6.

49:8 Now, subsequent to Christ's resurrection, is the time of salvation (2 Cor. 6:2).

49:10 Protection and blessing ultimately come through the Lamb (Rev. 7:16–17).

50:6 The Messianic servant undergoes suffering and humiliation for the sake of accomplishing
salvation (Matt. 27:26–31).

51:10 God's redemption in the exodus is analogous to his redemption of his people from
Babylonian exile, and both look forward to his climactic redemption in Christ.

51:11 See note on 35:10.

51:17 Wrath is followed by exaltation, prefiguring the movement from the wrath of Christ's
crucifixion to the exaltation of his resurrection and ascension. On the cup of wrath, see note on Jer.
25:15.

52:7 The gospel is the good news of salvation (Rom. 10:15).

52:10 The inclusion of the nations fulfills the promise to Abraham concerning blessing to all
nations (Gen. 12:3; Luke 2:30–31; see note on Isa. 42:6).

52:11 The Israelites' departure from pagan Babylon prefigures the departure of believers from the
contamination of the world (2 Cor. 6:14–7:1).

52:13 Exaltation of the servant, the Messiah, follows his suffering (v. 14; 53:3–9; see note on
51:17).

52:15 Many who have not heard of Christ will be awed (v. 14) by his suffering sacrifice. Paul
spreads the message to those who have not heard (Rom. 15:14–21).

53:1 The message of salvation in Christ often meets an unbelieving response (John 12:37–43; Rom.
10:16).

53:5 The messianic servant undergoes substitutionary suffering (Rom. 4:25; 2 Cor. 5:21; 1 Pet.
2:24–25).

53:9 Christ was put to death between two robbers (Matt. 27:38) and buried in the tomb of a rich
man, Joseph of Arimathea (Matt. 27:57–60).

53:11 Christ's death and resurrection results in our justification (Rom. 3:23–26; 4:25; 5:19).

54:1 The return of Jerusalem's inhabitants from exile prefigures the multiplication of children of
the promise (Rom. 9:8) who will return to God through Christ (Gal. 4:27).

54:7 See note on 51:17.

http://breachrepairers.webs.com/ 72
54:10 The new covenant results in permanent peace with God (Rom. 5:1) and is secure forever
(Heb. 9:12).

55:2 God's offer of food is fulfilled in Christ, who is the food and drink of eternal life (John 6:52–58).

56:7 The extension of salvation to the nations takes place in Christ (Matt. 21:13; Acts 1:8; Rev. 5:9).

57:3 See notes on 1:4; 12:6; and 34:2.

57:19 God gives the invitation of salvation to all, anticipating the spread of the gospel (Acts 2:39;
Eph. 2:17).

58:1 See note on 1:4.

58:2 Israel's hypocrisy anticipates the hypocrisy and externalism that Christ will confront (Matt.
15:1–10).

59:2 See note on 1:4.

59:7 Paul uses these words (Rom. 3:15–17) to show that Jews and Gentiles alike are guilty of sin.
See note on Ps. 14:1.

59:17 God's battle anticipates (1) the coming of righteousness and salvation at Christ's first coming
(Rom. 3:23–26), (2) Christians' battle against evil (Eph. 6:10–20; 1 Thess. 5:8), and (3) the war at
Christ's second coming (Rev. 19:11–21).

59:20 The Redeemer is Christ, who saves both Jews and Gentiles (Rom. 11:25–32).

60:1 God's glory is seen in Christ (John 1:14).

60:3 Nations come to Christ through the gospel (Luke 24:47; John 1:32; Acts 1:8; Rev. 21:24–25;
see notes on Isa. 2:2 and 11:10).

60:6 The wise men, representing the nations, bring gold and frankincense and myrrh (Matt. 2:11).

60:19 God is the sole light in the new Jerusalem (Rev. 21:22–24).

61:1 Christ applies these words to himself and his work of salvation (Luke 4:18).

61:10 The church as Christ's bride is given beautiful clothing (Rev. 19:8; see Eph. 5:25–27).

62:1 Righteousness and salvation come in Christ (see note on 9:7).

62:4 God's restoration of Israel prefigures Christ as husband to the church (2 Cor. 11:2; Eph. 5:25–
27; Rev. 19:7–9).

63:3 The execution of punishment anticipates final punishment (Rev. 14:20; 19:15).

http://breachrepairers.webs.com/ 73
63:4 See notes on 13:6 and 13:9.

63:12 Past acts of salvation foreshadow the great future salvation (see note on 51:10).

64:1 God comes from heaven both at the first and second coming of Christ (John 6:33, 38, 50; Rev.
19:11).

64:11 See note on 51:17.

65:1 The resistance and rebellion of Israel fits into the plan of God to extend salvation to all nations
(Rom. 10:20–21; 11:11–32).

65:9 See note on 1:9.

65:17 Ultimate blessing to God's people comes in the consummation (2 Pet. 3:13; Rev. 21:1). The
new creation has come in its beginnings already in Christ (2 Cor. 5:17).

66:1 The inadequacy of a house of stone indicates by contrast the fact that God's purpose to dwell
with man is fulfilled in Christ (Matt. 1:23; John 1:14; 2:19–21; Acts 7:48–50; 17:24).

66:8 The restoration of inhabitants to Jerusalem prefigures the multiplication of children of God in
the church, the heavenly Jerusalem (Isa. 54:1; Gal. 4:26–27).

66:18 On the gathering of the nations, see notes on 2:2 and 11:10.

66:24 The picture of unending judgment anticipates the NT teaching about Gehenna, the lake of fire
(Mark 9:48; Rev. 20:15; 21:8).

Jeremiah
Jeremiah's prophetic indictment of Israel is largely rejected, prefiguring the rejection of Christ's
prophetic message to Israel (Luke 11:49–51). God's judgment on Israel for apostasy prefigures the
judgment that Christ bears as substitute for the apostasy of mankind (1 John 2:2). It also prefigures
final judgment (Rev. 20:11–15). Restoration from exile prefigures final restoration to God through
Christ (Heb. 10:19–22).

1:2 God gives the prophecies during the time covered in 2 Kings 22–25 and 2 Chron. 34:1–36:20.

1:5 God's care from the womb prefigures the Father's relation to the Son in the incarnation (Luke
1:35) and also the calling of the apostle Paul (Gal. 1:15).

1:8 God delivers Paul from plots at Corinth (Acts 18:9–11) and elsewhere.

1:9 The firmness of the prophet amid opposition prefigures the firmness of Christ's teaching amid
opposition.

1:16 God's judgment on evil and apostasy (see 2 Chron. 36:15–16) anticipates the final judgment
(Rev. 20:11–15). Christ in the crucifixion bears judgment for our apostasy (1 Pet. 2:24; 1 John 2:2).

http://breachrepairers.webs.com/ 74
2:2 In forsaking the Lord, Israel is like an adulteress. Her unfaithfulness contrasts with the
faithfulness and purity that will be worked out in the church (2 Cor. 11:2; Eph. 5:25–27; Rev. 19:7–
8).

2:11 The folly of apostasy prefigures the folly of rejecting Christ, who opens the way of salvation
(John 14:6; Acts 4:12).

2:13 Living water is found in Christ (John 4:10–14).

2:21 See Isa. 5:1–4.

3:1 See note on 2:2 and the connection with Deut. 24:1–4.

3:10 The pretense in Judah illustrates the hypocrisy that can infect religion (Matt. 23:13–36; see
note on Isa. 58:2).

3:13 Forgiveness comes to those who acknowledge guilt, but not to those who continue to think
they are righteous (Luke 18:9–14).

3:17 The gathering to Jerusalem anticipates the NT gathering to heavenly Jerusalem (Heb. 12:22–
24) and the future gathering to the new Jerusalem (Rev. 21:24–26).

4:4 Circumcision of the heart comes in Christ (Col. 2:11; Heb. 8:8–13).

5:1 The lack of a righteous man is finally remedied in Christ (Acts 3:14).

5:9 See note on 1:16.

5:14 See note on 1:9.

6:1 See note on 1:16.

6:14 True peace with God can come only through the definitive overcoming of sin in Christ (John
16:33; Rom. 5:1).

7:11 Israel's hypocrisy anticipates the hypocrisy and corrupt worship that Christ confronts (Matt.
21:13).

7:14 The destruction of the temple anticipates the later destruction of Herod's temple that Christ
predicts (Matt. 24:2).

8:3 The remedy for death and for sin that leads to death is the resurrection of Christ (1 Cor. 15:42–
57).

8:11 See note on 6:14.

9:1 Apostasy is a deep grief both to Jeremiah and to Christ, the final prophet (Luke 19:41–44).

http://breachrepairers.webs.com/ 75
9:23–24 The principle of boasting in God alone is fulfilled in 1 Cor. 1:29–31 (see 2 Cor. 10:17).

10:5 Only God, not idols, offers salvation. The uniqueness of God and his way anticipates the unique
role of Christ as the way to God (John 14:6; Heb. 10:19–22; see note on Isa. 46:1).

11:8 Through Moses God predicts that disasters and exile will result from disobedience
(Deuteronomy 28).

11:19 The hostility to Jeremiah prefigures the hostility to Christ as prophet (Isa. 53:7; Matt. 27:1;
Luke 6:11).

12:7 God forsakes his house and his people on account of their sin. This anticipates later judgments
on sin, including the forsaking of Christ when he is the sin-bearer (Matt. 27:46).

13:9 The pride of God's people contrasts with the need for people who truly serve him. The need is
answered in the new covenant (31:31–34) in Christ (Heb. 8:8–13; 10:15–25).

14:3 Drought fulfills the curse in Deut. 28:22 that must come when Israel forsakes the Lord. It
contrasts with the blessing of living water in Christ (John 4:14; 6:35).

14:14 The conflict between true and false prophets anticipates the conflict between Jesus and his
opponents, and between true and false teaching in the church (2 Pet. 2:1–3).

15:2 Judgments fulfill the prophetic curses in Deut. 28:15–68 (see Rev. 6:8). God's wrath against sin
anticipates the final judgment (Rev. 20:11–15; see note on Jer. 1:16).

15:10 Jeremiah as a rejected prophet prefigures the rejection of Christ's prophetic ministry (Luke
11:49–51).

16:15 Restoration, prophesied in Deut. 30:1–5, prefigures final salvation in Christ (Isa. 40:1–11).

17:8 The blessing promised to the righteous man (Ps. 1:3) is fulfilled in Christ the perfectly
righteous man (Acts 3:14) and in those who are righteous in him (2 Cor. 5:21).

18:6 God's power as creator can bring salvation even to the wayward (Rom. 9:20–24; Eph. 2:4–10).

18:11 The call to repent anticipates the call to repent from John the Baptist (Matt. 3:2) and in
gospel proclamation (Luke 24:47; Acts 2:38).

19:3 See note on 1:16.

19:9 The specific horror of eating human flesh was prophesied in Deut. 28:53–55. Horror upon
horror shows the results of the degradation of sin, and prefigures the final judgment (Rev. 20:11–
15; see note on Jer. 1:16).

20:2 Opposition to Jeremiah the prophet prefigures opposition to Christ the final prophet (Luke
11:49–51). Those who oppose Christ will experience judgment (Rev. 20:11–15).

http://breachrepairers.webs.com/ 76
21:8 Even in the midst of the greatest disaster God in mercy holds open a way of escape. The escape
prefigures the escape from sin, death, and destruction through the salvation in Christ (John 11:25–
26; 14:6).

22:3 The demand for justice from the king fails to be answered. The answer finally comes in Christ
the king (Isa. 9:6–7; Rev. 19:11).

23:1 The false shepherds contrast with Jesus the true shepherd (John 10:11, 14).

23:5 The “Branch,” alluding to Isa. 11:1, is the Messiah in the line of David (see Zech. 6:12; John
15:1–17).

23:8 The restoration, which is parallel to the exodus (Ex. 12:33–38), prefigures rescue from sin and
the kingdom of Satan (Col. 1:13–14).

23:16 See note on 14:14.

24:5 The exiles are the remnant to whom God gives favor, illustrating the remnant theme (see
notes on 1 Kings 19:18; Isa. 1:9; and 6:13).

24:7 The renewal of the heart, already prophesied in Deut. 30:6, is further explained in the promise
of the new covenant (Jer. 31:31–34; Heb. 8:8–13; 10:15–25).

25:11 Daniel relies on the prophecy of 70 years when he prays for restoration (Dan. 9:2). The 70
years are years of sabbath rest for the land (2 Chron. 36:21). The restoration looks forward to final
rest in the consummation (Heb. 4:9–10).

25:15 The cup of wrath prefigures the wrath of God in final judgment (Rev. 14:10; 16:1, 19). Christ
on the cross drank the cup of wrath as our substitute (Matt. 26:39, 42).

26:6 See note on 7:14.

26:8 The desire for death illustrates a pattern of opposing the prophets, a pattern that culminates
in the death of Christ (Matt. 21:33–41; Luke 11:49–51; see note on Jer. 20:2).

27:9 See note on 14:14.

27:11 To those who listen the service to Babylon becomes a judgment tempered with mercy,
prefiguring the mercy in Christ (Heb. 12:5–11; see note on Jer. 21:8).

28:9 Peace with God does not come without first dealing with the issue of sin. The answer is to be
found in Christ (Col. 1:13–14; see note on Jer. 6:14).

28:15 See note on 14:14.

29:8 See note on 14:14.

29:10 See note on 25:11.

http://breachrepairers.webs.com/ 77
29:13 See notes on 24:7 and 31:31.

29:14 Restoration from exile prefigures the reconciliation with God by which we may be gathered
into the presence of God in heaven (Rom. 5:1–10; Gal. 4:26–28; Heb. 10:19–22; 12:22–24; see notes
on Jer. 3:17 and 16:15).

30:18 The rebuilding of cities prefigures the heavenly Jerusalem (Gal. 4:26–28; Heb. 12:22–24; Rev.
21:9–14).

31:1 The promise, “I will be . . . God, and they shall be my people,” is a repeated refrain in Jeremiah
(11:4; 24:7; 30:22; 31:33; 32:38). It builds on the promise to Abraham (Gen. 17:7) and to Israel
through Moses (Ex. 19:5–6). It is fulfilled in the new covenant in Christ (Jer. 31:33; Heb. 8:10; see
note on Jer. 31:31).

31:15 Past devastations to Israel anticipate the devastation when Herod kills the children (Matt.
2:16–18).

31:31 The new covenant is fulfilled in the covenant that Christ makes at the Last Supper (Matt.
26:28; 1 Cor. 11:25; 2 Cor. 3:6; Heb. 8:8–13; 10:15–25) and includes Gentiles as well as Jews
through union with Christ (Gal. 3:9, 14, 27–29).

32:20 A long history of God's demonstrations of faithfulness is linked to his faithfulness in


Jeremiah's time and in the climactic salvation in Christ (Rom. 3:3–4).

32:40 See notes on 24:7 and 31:31.

33:8 Forgiveness is foreshadowed in the restoration from exile, but it is fully accomplished in
Christ (Col. 1:13–14).

33:15 See note on 23:5.

34:11 The faithlessness of the people concerning the law in Ex. 21:2 and Deut. 15:12 contrasts with
the faithfulness of Christ, who brings full and permanent liberty from sin (Luke 4:18–19).

35:10 The obedience of the Rechabites contrasts with the disobedience of Israel and prefigures the
obedience of the Christ the Son to his Father (John 8:29).

36:2 The triumph of the word of God over opposition prefigures Christ (John 1:10–11) and his
triumph over opposition.

37:18 The innocent suffering of Jeremiah prefigures the innocent suffering of Christ (Matt. 27:24).

37:19 See note on 14:14.

38:6 Jeremiah's brush with death prefigures Christ's being put to death (see notes on 11:19 and
15:10).

38:17 See note on 21:8.

http://breachrepairers.webs.com/ 78
39:1 The prophecies of disaster from Jeremiah and from other prophets (2 Chron. 36:15–16; Jer.
25:4–11) now come to pass, confirming the faithfulness of God in judgment. Judgments in history
prefigure the final judgment (see note on 1:16).

39:18 See note on 21:8.

40:4 God shows mercy to Jeremiah and to those who are left, anticipating the mercy he will show in
Christ (Rom. 6:23).

41:14 God shows mercy to the captives, anticipating the mercy he will show in Christ (Rom. 6:23;
Col. 1:13–14).

42:6 After all of the rebellion in previous times, the people finally resolve to obey the Lord. But then
they prefer their own judgment (43:1–7). Their stubbornness shows the need for renewal of the
heart that will come in Christ (31:31–34; see notes on 4:4 and 31:31).

44:16 The persistence in rebellion shows the justice of God's judgment but also the need for a
radical renewal of heart, promised in the new covenant (see note on 31:31).

45:5 Even the righteous suffer as a result of the sins of the people. The righteous suffering
prefigures the suffering of Christ as the sin-bearer (2 Cor. 5:21; 1 Pet. 2:22–24).

46:2 God as judge of the whole world executes judgment on the nations as well as on his own
people, prefiguring final judgment (Rev. 20:11–15; see note on Jer. 1:16).

46:10 See note on Isa. 13:6.

47:1 The destruction of the Philistines, one of the long-time enemies of Israel, prefigures final
judgment (Rev. 20:11–15; see note on Jer. 1:16; cf. the note on Isa. 15:1).

48:7 Chemosh, the patron god of Moab, is shown to be worthless. Destruction of false hopes and the
punishment for idolatry looks forward both to the final judgment (Rev. 20:11–15) and to the gospel
as a call to worship God in truth (John 4:23; 14:6).

48:47 See note on Isa. 16:5.

49:2 Deliverance for God's people includes judgment on their oppressors. Judgments within history
look forward to final judgment (Rev. 20:11–15). Sin, Satan, and death, as ultimate oppressors, have
already been defeated in Christ (Heb. 2:14–15; see note on Jer. 1:16).

49:9 Some verses about Edom are similar to Obadiah (Obad. 5).

49:12 See note on 25:15.

50:1 See note on Isa. 14:4.

50:8 The command to flee prefigures the command to flee the final Babylon, the city of sin (Rev.
18:4).

http://breachrepairers.webs.com/ 79
50:20 Forgiveness of sins is found ultimately in Christ (Col. 1:14; Heb. 10:1–14). On the remnant,
see note on Isa. 6:13.

51:9 Judgment on Babylon prefigures final judgment against God's enemies (Rev. 18:5; see note on
Jer. 1:16).

51:11 Judgment through the Medes is predicted also in Isa. 13:17 and comes to pass in Dan. 5:31.

52:1 See the parallel in 2 Kings 24:18–25:21.

52:3 See note on 2 Chron. 36:16.

52:7 Jeremiah's earlier prophecies about destruction (e.g., 7:14; 34:2–4) are here fulfilled,
underlining the faithfulness of God and the power of his word. The words of judgment foreshadow
Christ's prophecies about the destruction of Jerusalem (Matt. 24:2; Luke 19:43–44) and the
prophecies of final judgment (Rev. 20:11–15). See note on 2 Chron. 36:21.

52:13 See note on 2 Kings 25:9.

52:31 See note on 2 Kings 25:27.

Lamentations
The lament over Jerusalem anticipates Christ's lamenting over the future fall of Jerusalem (Luke
19:41–44). In both cases, Jerusalem suffers for her own sins. But suffering for sin finds a remedy
when Christ suffers as a substitute for the sins of his people (2 Cor. 5:21; 1 Pet. 2:22–24).

1:5 God shows his justice in judging the sins of Jerusalem. He prefigures the manifestation of justice
in the last judgment (Rev. 20:11–15) and in the work of Christ.

2:14 On false prophets, see note on Jer. 14:14.

2:17 God's fulfillment of prophecy underlines his faithfulness and the power of his word. His
faithfulness is supremely manifested in the suffering and vindication of Christ (see note on Ps.
105:5).

3:14 The sufferings of the prophet prefigure the sufferings of Christ (Matt. 27:27–31, 39–44).

3:26 Even in the midst of disaster and pain there is final hope for the salvation of the Lord. This
hope has come to fruition in the salvation that Christ has accomplished (Matt. 1:21), and we now
wait for its consummation (Rom. 8:18–25).

4:11 The pouring out of God's wrath on Jerusalem prefigures the wrath poured out on Christ as sin-
bearer (Gal. 3:13), the wrath on Jerusalem in its second destruction (Luke 21:22–24), and the wrath
in the final judgment (Rev. 20:11–15).

5:21 Restoration is promised to Jerusalem after 70 years of exile (Jer. 25:11–12; 29:10–14). The
restoration prefigures final salvation in Christ (Col. 1:13–14; see note on Jer. 29:14).

http://breachrepairers.webs.com/ 80
Ezekiel
God judges Israel's apostasy through the exile. Israel suffers for her own sin, and in so doing
anticipates God's final judgment against sin (Rev. 20:11–15). But the suffering also anticipates the
suffering of Christ for the sins of others. The subsequent blessing in restoration prefigures the
blessings of eternal salvation in Christ (Eph. 1:3–14).

1:26 God appears in human form, anticipating the incarnation of Christ and his glory (John 1:14;
Rev. 1:12–16).

2:3 The resistance to Ezekiel as a prophet prefigures the resistance to Christ as final prophet (Luke
11:49–51; see note on Jer. 1:9).

2:8 The picture of eating, symbolizing an appropriation of the words of God, anticipates Rev. 10:9–
11.

3:8 See note on Jer. 1:9.

3:12 The empowering by the Spirit prefigures the role of the Spirit in Christ's prophetic ministry
(Luke 4:18), and then his empowering of gospel proclamation (Acts 1:8).

3:17 Ezekiel has a responsibility for faithfulness analogous to the responsibility in gospel
proclamation (2 Cor. 2:14–17; 3:5; 4:2).

4:4 Ezekiel's identification with the punishment of the people prefigures Christ's bearing the sins of
his people (2 Cor. 5:21; 1 Pet. 2:22–24).

5:1 The prophet's own body becomes a symbol for the sinful people. It prefigures Christ's
identification with the sins of his people (2 Cor. 5:21).

5:2 The casting off of much of the hair leaves a remnant, anticipating the remnant in the NT (Rom.
9:27; 11:5; see note on Isa. 6:13).

5:8 Judgment against sin prefigures the final judgment (Rev. 20:11–15).

5:13 Knowing that “I am the Lord”—an important theme in Ezekiel—anticipates the deeper
knowledge of God given in Christ (John 14:9; 17:1–5).

6:4 God's judgment makes plain the worthlessness of idols. God destroys false hopes to make plain
that Christ is the one, God-ordained way of salvation (John 14:6; Acts 4:12; see notes on Isa. 46:1;
Jer. 48:7).

7:2 See note on 5:8.

8:2 See note on 1:26.

8:3 God judges idolatry, making plain that the true God alone is the source of salvation (see note on
6:4).

http://breachrepairers.webs.com/ 81
9:4 Mercy comes to those who follow God's ways, prefiguring the mercy in Christ. The mark
prefigures the seal of the Holy Spirit and of the name of God, guaranteeing our salvation (2 Cor.
1:22; Eph. 1:13; Rev. 7:2–8; 14:1–3).

9:8 On the remnant, see notes on Isa. 1:9 and 6:13.

10:18 The departure of God's presence from the temple is one aspect of judgment. It contrasts with
the dwelling of God in the temple, which prefigures the coming of God to dwell with us in Christ
(Matt. 1:23).

11:13 On the remnant, see notes on Isa. 1:9 and 6:13.

11:19 The promise of a new heart, reiterated in 36:25, is connected to the new covenant that will
come in Christ (Jer. 31:31–34; Heb. 8:8–13; 10:16–18; see note on Jer. 31:31).

12:11 On the exile as judgment, see note on Isa. 39:6.

13:2 The false prophets prefigure Christ's opponents and false teachers in the church (2 Pet. 2:1–3;
see note on Jer. 14:14).

13:10 The religious leaders opposing Jesus are like whitewashed tombs (Matt. 23:27). On false
peace, see note on Jer. 6:14.

14:3 God does not reveal himself to the rebellious. The lack of understanding anticipates the lack of
understanding of Jesus' teaching (Matt. 13:10–17).

14:6 On repentance, see note on Jer. 18:11.

14:9 The deception that falls on the rebellious anticipates the deception on those who refuse the
truth of the gospel (2 Thess. 2:10–12).

15:2 Israel is a vine without fruit. See note on Isa. 5:7.

16:8 The faithlessness of Israel contrasts with the faithfulness of the bride of Christ (Eph. 5:25–27;
Rev. 19:7–8). The church also is tempted to go astray from her covenant with Christ (2 Cor. 11:2–3).
See note on Jer. 2:2.

17:13 The penalty for breaking a covenant with a human king shows by analogy the seriousness of
breaking the covenant with God (Heb. 10:29–31).

17:22 After destruction comes a new beginning, symbolizing the kingdom of Christ and its growth
to fill the nations (see Isa. 11:1).

18:4 God will execute justice. The judgments within history look forward to the final judgment,
when perfect justice will come (Rev. 20:11–21:8).

18:9 The granting of life to the righteous in the short run prefigures the granting of eternal life. The
gift of eternal life comes only through perfect righteousness, the righteousness of Jesus Christ (Rom.

http://breachrepairers.webs.com/ 82
3:23–26; 6:23).

19:9 On the exile as judgment, see note on Isa. 39:6.

20:3 See note on 14:3.

20:8 The repeated rebellion of Israel calls for judgment. God must also be faithful to his name in
rescuing them. Judgment and mercy are finally both achieved in Christ (Rom. 3:25–26).

20:11 See note on Lev. 18:5.

21:31 Fire and wrath anticipate the final judgment (Rev. 20:11–15). The sword (Ezek. 21:28)
anticipates the sword of Christ in final judgment (Rev. 19:15; see Heb. 4:12–13).

22:15 On the exile, see note on Isa. 39:6.

22:20 The melting process prefigures the coming of the Messiah as refiner (Mal. 3:3).

22:30 No man is adequate to the task of redemption except Jesus Christ (1 Tim. 2:5–6).

23:3 See note on 16:8.

23:22 Any lover other than the true God will be found to be treacherous, resulting in judgment. The
failure of other gods points to the one way of salvation through the true God (John 14:6; Acts 4:12).

23:32 On the cup of wrath, see note on Jer. 25:15.

24:8 The coming of God's wrath prefigures his wrath in the final judgment (Rev. 20:11–15).

24:21 The destruction of the temple destroys false pride and confidence. By contrast it points to
confidence in God alone for salvation (John 2:19–21; Acts 4:12).

25:2 God's judgment against Israel's traditional enemies prefigures future judgments against
enemies, including the final judgment (Rev. 20:11–15).

26:4 The completeness of destruction indicates God's zeal to remove evil completely. His zeal is
manifested both in the death of Christ and in the last judgment (Rev. 20:11–15).

27:9 The fall of Tyre with its riches prefigures the fall of Babylon the prostitute (Rev. 18:19).

28:13 The fall of Tyre is reminiscent of the fall of Adam (Gen. 3:1–19), and some think it is also
reminiscent of the fall of Satan. The proud beauty of Tyre also prefigures the beauty of Babylon
(Rev. 17:4), in contrast with the true beauty of the new Jerusalem (Rev. 21:18–21).

29:3 Egypt, another traditional enemy of God and God's people, is judged by God, prefiguring the
last judgment. By depicting Egypt as a dragon, Ezekiel makes the connection between her and the
defeat of Satan the dragon (Rev. 12:3–17).

http://breachrepairers.webs.com/ 83
29:13 God shows mercy to Egypt, in analogy with the mercy shown to Israel in bringing them back
from exile. This mercy anticipates the mercy in Christ (Rom. 5:6–11).

30:3 On the day of the Lord, see note on Isa. 13:6.

31:14 Human pride is put down (see note on Isa. 2:11), anticipating the humbling of pride through
salvation in Christ (1 Cor. 1:26–31).

32:2 On Egypt as a dragon, see note on 29:3.

32:7 The darkening of light prefigures the darkening at the second coming of Christ (Matt. 24:29–
31; Rev. 6:12–13).

32:21 An answer to the powerlessness and humiliation of death is found only in Christ and his
resurrection (John 11:25–26; 1 Cor. 15:42–58).

33:2 On the watchman, see note on 3:17.

33:11 The invitation to repent anticipates the gospel invitation (2 Pet. 3:9; Acts 2:38–41).

33:16 See note on 18:9.

33:17 See note on 18:4.

33:31 Hypocrisy among the people anticipates the hypocrisy that Christ confronts (Matt. 23:13–36;
see Jer. 7:11).

34:2 The false shepherds in Israel contrast with God, who is the true shepherd through Christ (vv.
11–31; Isa. 40:11; Luke 15:1–7; John 10:11, 14).

34:23 God is shepherd in connection with David, prefiguring the fact that Christ is both God and
man, and that as man he is the king in the line of David (Matt. 1:1–16).

35:5 See note on 25:2.

35:6 The principle of retribution manifests God's justice and anticipates the final judgment (Rev.
20:11–15; see note on Prov. 1:18).

36:10 The return from exile prefigures God's climactic redemption from sin through Christ (Col.
1:13–14; see note on Isa. 39:6).

36:22 The Lord vindicates his name in Christ when he shows holiness and justice in punishing sins
and mercy in saving the sinner (Rom. 3:23–26).

36:25 Cleansing from sins is accomplished in Christ (Col. 1:14; Heb. 9:23–28).

36:27 The promise of the Holy Spirit is fulfilled at Pentecost (Acts 2:1–21) and in the giving of the
Spirit to those who believe in Christ (Rom. 8:9–17).

http://breachrepairers.webs.com/ 84
37:5 The vision of new life through the Spirit has a partial fulfillment in the return from exile
(37:12). It prefigures the giving of resurrection life through the Spirit of Christ (John 11:25–26;
Rom. 8:9–17; Col. 3:1–4).

37:24 See note on 34:23.

38:2 Gog and Magog attack, pointing to the final war between God and his enemies in Rev. 20:8–10.

38:22 Fire comes from heaven in Rev. 20:9.

39:17 The sacrificial feast is depicted in Rev. 19:17–21.

39:29 On the pouring out of the Spirit, see notes on 36:27 and 37:5.

40:2 The vision of a new temple builds on the earlier passages about the tabernacle of Moses
(Exodus 25–40) and the temple of Solomon (1 Kings 5–8). Ezekiel's temple is even more glorious,
pointing forward to several realities: (1) the glory in which God dwells with man in Christ (John
1:14); (2) Christ's body that is the temple (John 2:19–21); (3) the church as a temple (1 Cor. 3:16;
Eph. 2:20–22; 1 Pet. 2:5); (4) the body of the individual believer (1 Cor. 6:19); and (5) the heavenly
Jerusalem (Rev. 21:9–22:5).

40:6 The gateways give access to the presence of God from all four directions. This access
prefigures the access to God through Christ, an access extending to all nations (Heb. 10:19–22; Rev.
21:12–13, 24–26).

40:38 The burnt offering, described in Lev. 1:1–17; 6:8–13, prefigures the sacrifice of Christ (Eph.
5:2; Heb. 10:5–10; and note on Lev. 1:9). God gives the vision to Israel (Ezek. 43:10–11) using the
symbolism belonging to the Mosaic covenant, but all the symbolism finds its culmination and
fulfillment in Christ (Heb. 8:8–13).

40:45 The priesthood descending from Aaron is described in Leviticus 9–10; 21–22; Numbers 3–4;
8; 17–18; and other passages. This priesthood is a shadow and a symbol, to be fulfilled in the
eternal priesthood of Christ (Heb. 7:23–8:6).

41:2 The spaciousness prefigures free access to God through Christ (see notes on 40:2 and 40:6).

42:13 The eating of the holy food prefigures spiritual food in Christ (John 6:53–58; see note on
Ezek. 40:45).

43:3 The coming of the glory of the Lord, as described in chapter 1, indicates the blessing of his
presence, giving a remedy for God's departure in chapter 10. The presence of God comes to the
church as a temple through the Holy Spirit (Acts 2:2–4; 1 Cor. 3:16).

43:18 On the burnt offering, see note on 40:38.

44:4 See note on 43:3.

44:15 The holiness required to serve God prefigures the holiness of Christ (Heb. 7:23–8:6; 9:11–28;

http://breachrepairers.webs.com/ 85
see note on Ezek. 40:45).

45:1 The expansion of holy area prefigures the holiness of the church, which is an international
community (Rev. 5:9–10), and the holiness of the new heaven and the new earth (Rev. 21:1–22:5).

45:4 On the priests, see note on 40:45.

45:7 The princes as leaders of Israel belong to the symbolism of the Mosaic order that God uses in
this vision (see notes on 40:2 and 40:38). They point forward to the leaders in the church (Eph.
4:11; 1 Pet. 5:1–5) and in the new heaven and new earth. Christ is the supreme Lord over all (Eph.
1:19–23). Fulfillment in Christ transforms the nature of worship and so displaces the forms of
worship belonging to the shadows of the Mosaic order (Heb. 8:1–9:14).

45:18 Permanent purification has now been accomplished through the offering of Jesus Christ once
and for all (Heb. 10:1–14).

45:21 Christ is our Passover lamb (1 Cor. 5:7; see notes on Ezek. 40:38 and 40:45).

46:2 On the prince and the priests, see notes on 40:45 and 45:7.

47:1 Refreshing water from the presence of God (see Ps. 46:4) prefigures the living water that
Christ offers through the Spirit (John 4:10, 13–14; 6:35; 7:37–39; Rev. 22:1–2).

47:12 The trees prefigure the tree of life as a symbol of abundant blessing from God (Rev. 22:2).

47:13 The inheritance of the land with its boundaries picks up the theme from Numbers 34, Joshua
14–19, and other passages. The land prefigures the new heaven and the new earth (Heb. 11:13–16;
Rev. 21:1).

47:22 The inclusion of foreigners prefigures the inclusion of the Gentiles in the blessing of the
gospel and the inheritance from Abraham (Gal. 3:9, 14, 26–29; 4:28–31).

48:1 See note on 47:13.

48:21 God dwells consummately in the midst of his people in Rev. 21:1–22:5. See notes on Ezek.
40:2 and 40:38.

48:31 The gates are found in Rev. 21:12–13 (see notes on Ezek. 40:2 and 40:6).

Daniel
Daniel and his friends exemplify the conflict between the kingdom of God and the kingdoms of this
world, a conflict that will come to its climax in Christ, in both his first coming and his second
coming.

1:5 Daniel and his friends resist the temptation to assimilate to the idolatrous culture in which they
are immersed. Christ was in this world but did not yield to temptation (Matt. 4:1–11; Heb. 4:15),
and we are called to follow in his steps (John 17:14–19; 1 Pet. 2:21).

http://breachrepairers.webs.com/ 86
1:17 Daniel is like Joseph (Gen. 40:8; 41:39) and prefigures the wisdom of Christ (1 Cor. 1:30; Col.
2:3).

2:11 God by answering Daniel shows that he is the true God, and anticipates the time when God's
dwelling will be in the flesh (John 1:14).

2:24 Daniel also saves the lives of others, prefiguring Christ who saves us (Heb. 2:14–15).

2:44 In the days of the fourth kingdom, the Roman Empire, the kingdom of God is established
through Christ (Matt. 3:2), especially through his resurrection (Matt. 28:18; Eph. 1:19–23).

3:6 The forcing of false worship anticipates the situation with the beast (Rev. 13:12–15) and the
persecution against the church (Acts 8:1–3).

3:18 The willingness to die for the faith anticipates Christ's willingness to die (John 10:17–18) and
the willingness of Christians to be martyrs (Acts 7:55–60; Rev. 6:9; 12:11).

3:25 The one like “a son of the gods” is the preincarnate Christ (cf. Rev. 1:12–16). Christ identifies
with the persecution of the Jews and in his power protects them.

3:29 The resurrection-like deliverance from death results in the spread of the message about the
true God. The message prefigures the message of the gospel announcing the resurrection of Christ.

4:9 Daniel's wisdom and ability to interpret dreams is like that of Joseph (Gen. 41:38). Daniel serves
to mediate divine wisdom to Nebuchadnezzar, and so prefigures the unique mediation of Christ,
who is the wisdom of God (1 Cor. 1:30; Col. 2:3).

4:30 Nebuchadnezzar is humbled by God's judgment. The putting down of human pride anticipates
the putting down of pride when God brings salvation in Christ (1 Cor. 1:26–31; see Dan. 4:37).

5:4 The judgment on idolatry anticipates the last judgment (Rev. 20:11–15) and demonstrates the
sovereignty of God.

5:11 See note on 4:9.

5:20 See note on 4:30.

6:7 See note on 3:6.

6:23 The resurrection-like deliverance of Daniel prefigures the resurrection of Christ.

6:26 The message concerning the true God is spread, prefiguring the spread of the gospel, which
announces the resurrection of Christ (see note on 3:29).

7:3 The four beasts are four kingdoms (v. 17), corresponding to the four kingdoms of 2:36–40.
Features of the four beasts are combined in the beast of Rev. 13:1–8, which represents a final
opponent of God's people.

http://breachrepairers.webs.com/ 87
7:9 Features of this appearance of God reappear in Christ (Rev. 1:12–16), who is God in the flesh
(John 1:14).

7:13 Jesus is the Son of Man (Matt. 24:30; 26:64).

7:14 The dominion of Christ is associated with his resurrection and ascension (Matt. 28:18; Acts
2:33–35; Eph. 1:20–22) and continues until the consummation (Rev. 22:1).

7:21 The war against the saints is described in Rev. 11:7; 13:7–10.

7:25 The period of “a time, times, and half a time” (also in 12:7) is echoed in the half week in 9:27
and is the time of persecution of the church in Rev. 11:2, 3, 11; 12:6, 14. See also Dan. 8:14 and
12:11, 12 for possible further echoes.

8:10 The little horn, Antiochus Epiphanes, persecuted the faithful Jews and profaned the temple
(168 b.c.; see 8:23). He prefigures the man of lawlessness, the final Antichrist, the great opponent of
God's people (2 Thess. 2:3–4, 7–12; Rev. 12:4).

9:2 See 2 Chron. 36:21; Jer. 25:11–12; and 29:10.

9:9 Definitive forgiveness comes only in Christ (Rom. 4:6–8; Col. 1:14).

9:24 Atonement comes in Christ (Heb. 7:23–8:6; 10:1–14). Everlasting righteousness comes both
with Christ the perfectly Righteous Savior (Acts 3:14) and with the righteousness that he gives to
his people in justification (Rom. 3:23–26; 2 Cor. 5:21).

10:6 The glorious appearance, reflecting the glory of God, prefigures the glory of Christ in Rev.
1:12–16.

10:12 Daniel's intercession for Israel prefigures the intercession of Christ the great high priest
(Heb. 7:25).

10:13 The angelic war prefigures the spiritual war in Revelation (Rev. 12:7–9).

11:2 Tumults and wars on earth continue until the end (Matt. 24:6–7; Rev. 6:2–4) and remind us of
spiritual war, part of which is invisible (Rev. 12:7–9). In the midst of tumult, Christ alone provides
true peace (John 16:33; cf. Phil. 4:6–7; 1 Thess. 3:4).

11:31 See note on 8:10.

11:35 The refining process looks forward to God's refining of the church (Rom. 5:3–5; Heb. 12:3–
11; 1 Pet. 1:6–7).

11:36 The king is either the man of lawlessness of 2 Thess. 2:3–4 or a foreshadowing of him.

12:1 The book is identified as the book of life of the Lamb that was slain (Eph. 1:4; Rev. 13:8; 17:8),
guaranteeing the salvation of those who belong to Christ.

http://breachrepairers.webs.com/ 88
12:2 Life and final judgment are controlled by the power of Christ (John 5:27–29).

12:3 The brightness looks forward to the brightness in the new Jerusalem (Rev. 21:22–27; 22:5).

Hosea
The unfaithfulness of Israel calls for a permanent remedy, which will come in the faithfulness of
Christ to the Father and the faithfulness that Christ then works through the Spirit in his people.
God's love for Israel foreshadows Christ's love for the church (Eph. 5:25–27).

1:1 God gives the prophecies during the time covered in 2 Kings 15–20 and 2 Chronicles 26–32.

1:2 Israel's spiritual adultery, indicated also in Jeremiah (see note on Jer. 2:2), is a shocking
rebellion that must lead to judgment on God's part (Hos. 1:4). Yet God will eventually bring a
remedy in Christ (1:10; Rom. 9:26). Christ prepares the church as a faithful bride (Eph. 5:25–27).

1:10 In faithfulness to the promise to Abraham (Gen. 13:16; 22:17) God will remember Israel. The
remembrance takes surprising form in that it includes Gentiles (Rom. 9:25–26) as well as Jews
(Rom. 11:25–32).

2:3 God in justice brings judgment on unfaithfulness. His justice is climactically manifested in
Christ, through whom we escape condemnation (Rom. 3:23–26; 8:1), and is manifested in the final
judgment (Rev. 20:11–21:8).

2:14 Punishment and restoration for Israel prefigure the punishment and resurrection of Christ,
the true Israel (Rom. 4:25).

2:23 See note on 1:10.

3:1 God's love for the wayward prefigures his love for sinners in Christ (Rom. 5:6–11).

4:5 On false prophets, see note on Jer. 14:14.

4:10 False gods are not able to satisfy. Their failure shows the folly of false worship and points by
contrast to the one true God, and ultimately to his way of salvation in Christ (John 14:6).

5:4 The lack of knowledge of God points by contrast to true knowledge, which is to be found
ultimately in Christ (John 14:7; 17:3).

5:14 See note on 2:3.

6:2 The invitation to come to the Lord prefigures the invitation of the gospel (Acts 16:31; 17:30–
31). The granting of life on the third day prefigures the resurrection of Christ as the source of life to
his people (Col. 3:1–4).

6:3 God is known truly in Christ (Matt. 11:27; John 14:6; 17:3).

6:6 Jesus teaches the centrality of steadfast love (Matt. 9:13; 12:7).

http://breachrepairers.webs.com/ 89
7:5 The king and princes participate in sin with the people, pointing to the need for a faithful king.
Christ is the faithful king in the line of David (Matt. 1:1–16).

8:13 God in his justice punishes. Since the people have broken his covenant, he reverses the
deliverance from Egypt that was an aspect of covenantal redemption. A greater redemption is
needed, which is to be found in Christ (Matt. 2:15; Heb. 8:8–13).

9:10 Israel's present apostasy repeats the old apostasy at Baal-peor (Num. 25:1–5), pointing to the
need for a permanent remedy and a permanent change of heart, which will come in Christ (Heb.
8:8–13).

10:6 See note on 4:10.

10:8 Fear of God's wrath prefigures the fearful character of the final judgment (Luke 23:30; Rev.
6:16).

10:12 Full righteousness comes in Christ (Rom. 3:23–26; 8:1–4).

11:1 Israel, labeled God's “son” in Ex. 4:22 (see Deut. 8:5), came out of Egypt in the exodus (Exodus
14). The movement of Israel prefigures the movement of Christ (Matt. 2:15), who is the faithful Son
(Matt. 3:17), whereas Israel as son repeatedly failed (Hos. 11:2).

11:11 On the restoration from exile, see note on Isa. 35:10.

12:2 God's punishments are the product of his justice, prefiguring the justice of final judgment
(Rev. 20:11–15). Such demonstrations of justice make plain the need for pardon through the
propitiation of Christ (1 John 2:1–2).

13:14 The threat of death as punishment for sin (Rom. 6:23) is finally answered through the
resurrection of Christ (John 11:25–26; 1 Cor. 15:55–57; Heb. 2:14–15).

14:1 The command to repent anticipates the command to repent in the gospel (Acts 2:38).

14:5 The promise of blessing prefigures the blessings of salvation in Christ (Eph. 1:3–14; see note
on Isa. 27:6).

Joel
The day of the Lord, the day of God's coming (see note on Isa. 13:6), brings judgment on sin but also
may include blessing. Both aspects are fulfilled in both the first coming and the second coming of
Christ.

1:4 God sent a locust plague on the Egyptians during the time of Moses (Ex. 10:1–20). But the
plague in Joel's day comes on God's own people because of their sins (see Deut. 28:38). It shows the
desperate need for forgiveness in Christ and prefigures the locust plague preceding the judgment of
the second coming (Rev. 9:1–11).

1:13 See note on 2:12.

http://breachrepairers.webs.com/ 90
1:15 The day of the Lord, the day when God appears, is a day of judgment (see note on Isa. 13:6).

2:12 The call to repent anticipates the gospel call to repent (Acts 2:38).

2:18 Christ welcomes repentant sinners (Luke 5:32; 15:7).

2:28 The climactic blessing is the pouring out of the Spirit, accomplished at Pentecost (Acts 2:16–
21).

2:32 NT preaching invites listeners to be saved by the name of Christ (Acts 2:38–41; Rom. 10:13;
see Acts 4:12).

3:13 At the second coming God executes judgment (Rev. 14:14–20).

3:15 The light is darkened at the second coming as part of God's judgment (Matt. 24:29–31; Rev.
6:12; see Rev. 8:12). The darkening at the crucifixion also indicates judgment (Matt. 27:45).

3:17 The holiness of Jerusalem is perfected in the consummation (Rev. 21:27).

Amos
God comes to Israel with both judgment for sin and promises of restoration. The judgment and
restoration anticipate the crucifixion and resurrection of Christ, as well as the final judgment (Rev.
20:11–15). The demand for righteousness is fulfilled in the righteousness of Christ (Rom. 8:1–4).

1:1 God gives the prophecies during the time covered in 2 Kings 14:23–15:7.

1:2 The power of God's word in judgment anticipates the power of Christ's word, both in his first
coming and in his second coming (John 12:48–50; Rev. 19:15, 21).

1:3 Damascus, the capital of Syria, is judged, prefiguring final judgment on God's enemies (Rev.
20:11–15). On the display of God's justice in judgment, see notes on Lam. 1:5 and Ezek. 18:4.

2:4 God does not overlook the sins of his own people, but calls them to account just as he did the
other nations (1:3–2:3). He shows his impartiality (Rom. 2:11–16). All are subject to curse for their
disobedience, and escape is found only in Christ (Gal. 3:13–14; Rom. 3:9–31).

3:2 Those with greater privileges are liable to greater punishment (Luke 12:48). The principle is
shown in the guilt that comes to those Jews who reject Christ (Matt. 11:20–24; John 15:22–25).

3:8 See note on 1:2.

3:10 The demand for righteousness is an integral part of God's law. Righteousness is to be fulfilled
in the righteousness of Christ (Rom. 1:17; 2 Cor. 5:21) and in his followers (Rom. 8:1–4).

3:12 On the remnant, see note on Isa. 1:9 and 6:13.

4:6 Stubbornness, like the stubbornness of Pharaoh in the exodus, increases guilt. Stubbornness

http://breachrepairers.webs.com/ 91
characterizes Israel's history, and comes to a climax in the rejection of Christ (Acts 7:51–53; see
note on Isa. 29:13). The judgments on Israel were prophesied in Deut. 28:15–68.

5:18 People hoped that the day of the Lord would mean judgment against Israel's enemies. But it
involves judgment on sinners universally, including Israel. See notes on 2:4; Isa. 13:6; and 13:9.

5:27 The exile of the northern kingdom to lands beyond Damascus and then the southern kingdom
to Babylon prefigures final judgment.

6:1 Pride and self-confidence are judged by God, anticipating the judgment against human pride in
the gospel (1 Cor. 1:26–31).

7:3 The Lord in mercy does not simply destroy, but refines his people. His mercy anticipates the
mercy to be manifested in Christ (Matt. 9:27).

8:9 The darkening is a symbol of judgment, prefiguring the judgment at the crucifixion (Matt.
27:45) and at the second coming (Matt. 24:29–31; Rev. 6:12; see Joel 3:15 and Rev. 8:12).

9:1 The lack of escape prefigures the universality of the last judgment (Rev. 20:11–15).

9:8 On the remnant, see 3:12 and notes on Isa. 1:9 and 6:13.

9:11 The house of David is raised up when Christ is raised.

9:12 When Christ is raised, the nations (Gentiles) become included in God's blessings, in fulfillment
of the promise to Abraham (Gen. 12:3).

Obadiah
The judgment against Edom, a traditional enemy of Israel, contributes to the blessing of God's
people. The judgment and vindication prefigure the vindication of Christ and the judgments against
his enemies, both in his first coming and in his second coming.

3 God judges human pride, anticipating the gospel's judgment on pride (1 Cor. 1:26–31).

10 Those who attack God's people will ultimately be destroyed in the last judgment (Rev. 20:11–
15).

15 On the day of the Lord, see note on Isa. 13:6. On the principle of just retribution, see note on
Prov. 1:18.

Jonah
Jonah's rescue from death prefigures the resurrection of Christ (Matt. 12:39–40). The repentance of
the Ninevites prefigures the repentance of Gentiles who respond to the gospel (Matt. 28:18–20;
Luke 24:47).

1:15 The saving of mariners through the sacrifice of Jonah prefigures the salvation of all nations

http://breachrepairers.webs.com/ 92
through the death of Christ (1 John 2:2).

1:17 Jonah is under the sea, symbolizing the realm of death. His state prefigures the death of Christ
(Matt. 12:40).

2:6 Jonah's rescue from death prefigures the resurrection of Christ from the dead (Matt. 12:40).

3:5 Gentiles repent in response to the preaching of Jonah, who figuratively has been raised from the
“death” of the belly of the fish. Gentiles repent in response to the preaching of the resurrection of
Christ (Matt. 28:18–20).

3:10 The repentance of Gentiles contrasts with the repeated lack of repentance on the part of Israel
(Matt. 12:41; 21:43).

4:11 God's mercy is shown abundantly in the gospel and in the salvation of Gentiles who deserve
nothing (Rom. 9:30–31; 11:30).

Micah
God pronounces judgment on Israel, prefiguring final judgment (Rev. 20:11–15) and the judgment
that fell on Christ (Gal. 3:13). He promises blessing through the Messiah, anticipating the blessings
of salvation in Christ (Eph. 1:3–14).

1:1 God gives the prophecies during the time covered in 2 Kings 16–20 and 2 Chronicles 27–32.

1:5 God in justice cannot overlook the sins of his people. Punishment prefigures the punishment of
the last judgment (Rev. 20:11–15) and the substitutionary punishment that Christ bore for his
people (1 Pet. 2:24).

2:3 On the judgment of human pride, see notes on Isa. 2:11; Ezek. 31:14; and Amos 6:1.

2:12 On the remnant, see notes on Isa. 1:9 and 6:13.

3:5 On false prophets, see note on Jer. 14:14.

3:12 On the destruction of the holy city, see note on Ps. 74:3.

4:1 The exaltation of the name of God is accomplished in Christ (see note on Isa. 2:2).

5:2 The Messiah is to be born in Bethlehem (Matt. 2:1–6).

5:8 On the remnant, see 2:12 and notes on Isa. 1:9 and 6:13.

6:2 Israel does not escape judgment for her sins. This judgment prefigures the justice and
thoroughness of final judgment (Rev. 20:11–15).

6:8 Sacrifices cannot replace the need for justice and kindness. The focus on real righteousness
anticipates Jesus' teaching (Matt. 5:23–24; 9:13; 15:10–20) and is fulfilled in Jesus' own

http://breachrepairers.webs.com/ 93
righteousness (Acts 3:14; Rom. 8:1–4).

7:6 The family treachery in Israel anticipates the family treachery from resistance to Christ (Matt.
10:35–36).

7:18 Pardon is accomplished in Christ (Rom. 3:23–26; 1 John 2:2). On the remnant, see notes on Isa.
1:9 and 6:13.

Nahum
Judgment on Nineveh, a traditional enemy of God's people, prefigures final judgment and final
release from oppression (Rev. 20:11–21:8).

1:15 The good news of deliverance from the oppression of Nineveh prefigures the good news of
eternal deliverance from sin and death in the gospel (Isa. 52:7; Mark 1:1; Rom. 1:1).

2:3 The attack and destruction of Nineveh prefigures God's war through Christ against his ultimate
enemies (Matt. 12:29; Luke 10:17–19; John 12:31; Rev. 19:11–21; 20:7–10).

3:4 Nineveh's punishment prefigures the punishment for the idolatrous seduction of Babylon the
prostitute (Rev. 17:1–6; 18:1–3).

Habakkuk
God's use of a wicked nation to accomplish his righteousness foreshadows the use of wicked
opponents to accomplish his purpose in the crucifixion of Christ.

1:4 The perversion of justice in the triumph of the wicked prefigures the temporary triumph of the
wicked in the crucifixion of Christ.

1:5 The unbelievability of God's use of a wicked people, the Chaldeans, prefigures the
unbelievability of the way in which the injustice of the crucifixion of Christ is used by God for
salvation.

1:13 In the crucifixion of Christ the wicked leaders swallowed up Christ the righteous one.

2:4 The righteous person trusts in God; he believes that God's promises are true and that he will
bring to pass his righteous purposes. This trust anticipates trust in Christ (Rom. 1:17; Gal. 3:11;
Heb. 10:37–38), in whom the promises of God are fulfilled (2 Cor. 1:20).

2:8 On the principle of just retribution, see note on Prov. 1:18.

2:16 On the cup of God's wrath, see note on Jer. 25:15.

3:13 God appears to bring salvation to his people and to the anointed king. Salvation comes when
God appears in Christ (John 1:14; 14:9), when Christ the anointed king is saved from death in his
resurrection, and when his people are saved through Christ.

http://breachrepairers.webs.com/ 94
Zephaniah
Judgments on evil people anticipate the final judgment (Rev. 20:11–15) and indicate the necessity
of Christ's work and sin-bearing in order to save us from judgment (see note on Isa. 13:9).

1:1 God gives the prophecies during the time covered in 2 Kings 22–23 and 2 Chronicles 34–35.

1:2 God in his holiness is zealous to eliminate all evil. His commitment anticipates the final
judgment and renewal of the consummation (2 Pet. 3:10–13; Rev. 21:1).

1:7 On the day of the Lord, see note on Isa. 13:6.

2:3 The call for humility prefigures the gospel call to repent and turn to the Lord (Acts 2:38), and
the call to avoid the coming wrath (Acts 17:30–31).

2:9 On the remnant, see notes on Isa. 1:9 and 6:13.

2:10 On the punishment of pride, see note on Ezek. 31:14.

3:15 The removal of judgments and curse comes with Christ (Rom. 8:1; Gal. 3:13–14). Christ is the
Lord in our midst (Matt. 1:23; John 1:14) and now indwells the church through the Spirit (John
14:20; Rom. 8:9–10).

Haggai
The rebuilding of the temple prefigures the building of NT temples: the church (1 Cor. 3:16; Eph.
2:20–22) and the new Jerusalem (Rev. 21:9–22:5).

1:1 God gives the prophecies during the time covered in Ezra 5–6 (see Ezra 5:1 and 6:14).

1:2 The house of the Lord symbolizes his presence and looks forward to Christ as temple (John
1:14; 2:19–21), the church as temple (1 Cor. 3:16; Eph. 2:20–22), and the dwelling of God in the
new Jerusalem (Rev. 21:3; 21:22–22:5).

1:13 The promise to be with the people anticipates God being with his people in Christ (Matt. 1:23,
“Immanuel”) and through the Spirit of Christ (Rom. 8:9–10; 1 Cor. 3:16).

2:4 Our work is not in vain in the Lord (1 Cor. 15:58; Phil. 2:12–13).

2:6 God shakes the old order, showing that we should put our hope in his unshakable kingdom in
Christ (Heb. 12:26–28).

2:7 The ultimate glory of God is found in Jesus Christ (John 1:14; Rev. 21:22–23).

Zechariah
The rebuilding in the time of the restoration from exile prefigures the eternal salvation that comes

http://breachrepairers.webs.com/ 95
in Christ.

1:1 God gives the prophecies during the time covered in Ezra 5–6 (see Ezra 5:1 and 6:14).

1:3 The call to return prefigures the gospel call to repent and come to God (Acts 2:38; 17:30–31).

1:16 Mercy on Jerusalem prefigures the mercy on sinners in Christ (Luke 5:32).

2:5 The glory of God is manifest in Jesus Christ (John 1:14; 17:1–5; Rev. 21:22–27).

2:11 On the coming of the nations, see notes on Isa. 2:2; 11:10; and 42:6.

3:4 The removal of iniquity symbolizes justification in Christ (Rom. 3:23–26; 5:1).

3:8 The Branch is the Messiah (based on Isa. 11:1).

4:6 The Spirit of Christ gives a permanent supply of power and light (John 16:13–15; Rom. 8:9–13).

5:3 We can escape the curse for wrongdoing through Christ, who bore the curse for us (Gal. 3:13–
14).

6:12 On the Branch, see notes on 3:8 and Isa. 11:1.

6:13 Christ builds the temple, the church (Matt. 16:18; 1 Cor. 3:10–16).

7:9 The central importance of righteousness, rather than sacrifice, appears in 1 Sam. 15:22–23,
Amos 5:21–27; Mic. 6:7–8, and Matt. 9:13, and underlines the superiority of the righteousness of
Christ over all animal sacrifices (Heb. 10:1–14).

8:3 On God's dwelling, see note on Hag. 1:2.

8:11 On the remnant, see notes on Isa. 1:9 and 6:13.

8:22 On the coming of the nations, see notes on Isa. 2:2 and 11:10.

9:9 Jesus the king comes to Jerusalem on a donkey (Matt. 21:1–9).

10:9 The restoration from exile prefigures final salvation and life in Christ (John 6:35; 14:6).

11:10 Faithlessness leads to annulling the covenant, indicating the need for a new covenant (Heb.
8:8–13).

11:12 Thirty pieces of silver is the payoff connected with repudiating the Lord as true shepherd. It
anticipates the payoff for Judas (Matt. 26:15; 27:9–10).

12:10 Repentance involves looking on the crucified Messiah (John 3:14–15; 19:37).

13:1 Cleansing from sin comes in Christ (1 John 2:1–2).

http://breachrepairers.webs.com/ 96
13:7 The disciples are scattered at the time when Christ the shepherd is crucified (Matt. 26:31).

14:8 The living waters are found in Christ (John 4:10; Rev. 22:1; see note on Ezek. 47:1.

14:20 Holiness is found in Christ (Acts 2:27; Heb. 7:26) and in the new Jerusalem that he
establishes (Rev. 21:22–22:5).

Malachi
Disobedience and compromise are eliminated with the coming of Christ and his purification.

1:2 God's sovereign love for Jacob prefigures the sovereignty of his love for the elect (Rom. 9:1–29).

1:7 The danger of despising God continues in the church at the Lord's Supper (1 Cor. 10:21).

2:8 The corruption of the covenant shows the need for a new covenant (Heb. 8:8–13) and a perfect
priest (Heb. 7:11–8:6).

3:1 John the Baptist is the messenger preparing the way for Christ, who is the Lord, the messenger
of the covenant (Matt. 11:10–11).

3:7 On the call to repentance, see note on Jer. 18:11.

4:1 On the day of the Lord, see note on Isa. 13:6.

4:6 John the Baptist prepares hearts for the coming of the Lord in the person of Christ (Luke 1:17).

http://breachrepairers.webs.com/ 97
Course Overview
Timeline of the Life of Jesus (2nd)
I will show the complete timeline from His birth to resurrection

Historical Background (1st)


This will cover the history of the Jewish dispersion and differences between the Hellenist Jews
(Jews of the West) and Jews of the East. It will also cover the different translations of the scriptures
and which one influenced them during the time of Christ.

DA 2-3 “Chosen People” & “The Fullness of Time.” All my information supplements this chapter

Harmony of the Gospels


I will cover from Christ’s Birth to His transfiguration. Showing the chronology of events, and how
they relate to each other in the gospels.

The Life and teachings of Jesus (Birth to transfiguration)


This will focus on the theme of each Gospel; portions of scripture covering L&T1, Matt 1-18, Mark 1-
9, Lk 1-9:50; 11:14-32; 13:18-21, Jn 1-7:1. Jesus’ parables, Jesus’ sermons, Jesus’ questions, Jesus’
miracles.

Reading the Gospels


The Gospels and were designed to be read as full accounts, each in their own right, even as they
seek to tell about Jesus and his followers. The main obstacle in the Gospels continues into Acts:
many in Israel have rejected a message and promise originally intended for them. A key to
understanding these accounts is to trace the negative reaction and what it teaches about how
people respond to God, and how God still moves to draw people to himself.

Genre
The Gospels have a genre parallel in the ancient world that was called the bios. This was ancient
biography. Rather than focusing on physical description and tracing psychological thinking and
personal development like modern biographies, a bios highlighted the key events that surrounded a
person and his teaching. That is very much what the Gospels do. The key characters are Jesus and
God, as Jesus carries out the plan of the Father.

Acts belongs to a different kind of genre. It is a legitimization document: its goal is to explain and
legitimate the early church and its roots. This was necessary because in the ancient world what
counted in religion was its age and time-tested quality. Since Christianity was new, it needed to
explain how it could be new and still be of merit. The answer was that, although the form of
Christianity was new, the faith itself was old, rooted in promises and commitments made to Israel.
In fact, the new movement did not seek to make itself into a new entity but was moved in a new
direction only when official Judaism rejected it and expelled it from the synagogue, with the result
that (in accord with God's plan, as Acts clarifies) the gospel was taken to the Gentiles also. Acts tells
this story as it presents how the promise of God expanded as far as Rome. Though the Gospels are
historical writings, they are not always presented in a strict chronology, since some of their scenes
are organized topically. For example, Mark 2:1–3:6 reports five controversies in a row that Matthew
spreads out over chapters 8–12.

http://breachrepairers.webs.com/ 98
Perspectives
Even though the Gospels each offer varying accounts, they all share the view that Jesus is the
promised Messiah, uniquely related to God to bring his promise and salvation. Three of the Gospels
(called the Synoptics because they overlap at many places) tell the story of Jesus “from the earth
up,” gradually depicting how one can see his unique relationship to the Father. Mark starts with
John the Baptist, while Matthew and Luke start with Jesus' unique birth. John, however, tells the
story very much “from heaven down.” He starts with the preincarnate Word becoming flesh. His
presentation of Jesus as Son of God is more direct and explicit. The Synoptics allow the reader to
gradually see this idea, much in the manner people come to realize gradually who Jesus is. This
difference in how the story unfolds does not represent a conflicting account of Jesus, but simply a
distinct perspective on how to highlight who he is and what he has done.

Acts chronicles the expansion of Jesus' newly formed community from Jerusalem to Rome. Here
God and Jesus are the key figures, directing the action through the Spirit, with the key human
figures being Peter, Stephen, Philip, and Paul. Acts is not a defense of Paul, as many argue, but is a
defense of what Paul's ministry to the Gentiles represents: the realization of God's promise to
reconcile all people groups to himself and to one another through Jesus.

Distinctive of Matthew
Matthew's major concerns include Jesus' relationship to Israel and explaining Israel's rejection of
him. Those who were Christians did not seek a break with Judaism but had separated from Judaism
because the nation rejected the completion of the divine and scriptural promise Jesus brought and
offered. However, that rejection did not stop the arrival of the promise; it raised the stakes of
discipleship and led to the creation of a new entity, the church. The message was not limited to
Israel but included the whole world. Five discourse units consisting of six discourses (long sections
of teaching by Jesus) are the backbone of the book (chs. 5–7; 10; 13; 18; 24–25 [eschatological
discourse followed by a parables section]). As with all the Gospels, there is an interaction and
interchange between Jesus' word and deeds. Jesus' actions support what he preaches. Jesus' death
was an act of the divine plan that led to his vindication and mission. Disciples are those who come
to Jesus in personal relationship and trust, seeking forgiveness and the righteousness that God so
graciously offers.

A brief listing of major Matthean themes shows the variety of his interests. (Italics identify the key
themes, which in some cases overlap with other Gospels and in other cases are unique.) Matthew's
Christology presents a royal, messianic understanding of Jesus, who as Son of God comes to be seen
as the revealer of God's will and the bearer of divine authority. As the promised King of the Jews,
Jesus heals, teaches the real meaning of the OT in all its dimensions, calls for a practical
righteousness, inaugurates the kingdom, and teaches about the mystery elements of God's promise.
Matthew associates all of this with a program he calls the kingdom of heaven. This kingdom is both
present and yet to come (12:28; 13:1–52; 24:1–25:46). Jesus proclaims its hope throughout the
nation to the lost sheep of Israel. He calls on them to repent, challenges their current practices,
expresses his authority over sin and the Sabbath, and calls them to read the law with mercy. Most of
Israel rejects the message, but the mystery is that the promise comes despite that rejection. One day
that kingdom will encompass the entire world (cf. the parables of ch. 13). At the consummation, the
authority of Jesus in that kingdom will be evident to all in a judgment rendered on the entire
creation (chs. 24–25). Thus, for Matthew the kingdom program, eschatology, and salvation history
are all bound together.

http://breachrepairers.webs.com/ 99
 He was a publican (tax collector): Matt 9:9, 10:3. They bought their position in the
government which meant to betray their people and working for the oppressive Roman
government.
 He’s another name is Levi: Mark 2:14, Luke 5:27.
 He was a disciple: he was among those who were classed as the worst people, as chief of
sinners. Yet, Jesus chose Matthew. One day Jesus came and “he saith unto him, Follow me.”
Matt 9:9. As the writer of this book, all he said was, “and he(Matthew) arose, and followed
him.” Jesus picked disciples from the people that we consider the worst.
 He was a Jew.
 His audience is Jews: begins Christ’s genealogy with Abraham, father of Israel.
 He is trying to show that Jesus fulfilled the prophecies which point to the Messiah: As it
was written by the prophet Jeremiah, Zechariah, Isaiah…He’s trying to say, “Look,
everything that Jesus did, this points exactly to what the prophets of old said.

Distinctive of Mark
Mark is generally regarded today as the first Gospel to have been written, although a minority of
scholars regard Matthew as first. Thus, Mark's outline of Jesus' ministry has become the basic
structure through which his life has been traced, even though sections of it are probably given in
topical rather than chronological arrangement (e.g., the conflicts of chs. 2–3). The first major
section of this Gospel (1:16–8:26) cycles through a consistent structure in each of its three parts.
There is a story about disciples at the start (1:16–20; 3:13–19; 6:7–13) and a note about rejection
or a summary at the end (3:7–12; 6:1–6; 8:22–26). The turning point of the Gospel is the confession
in 8:27–31 that Jesus is the Christ. Half of the Gospel treats the movement toward the final week of
Jesus' ministry, while a full quarter of it is on the last week alone. For Mark, the events of the final
week are central to the story.

The key themes are also evident in how the account proceeds. It begins with a note that what is
being told is the gospel. Though to a lesser degree than Matthew or Luke, Mark also traces the
kingdom of God as a theme. For Mark, it has elements that indicate its initial presence, while the
bulk of the emphasis is that it will come in fullness one day in the future. The mystery of the
kingdom is that it starts out small but will accomplish all that God has called it to be. It will grow
into a full harvest.

Mark is more a Gospel of action than of teaching. Things happen immediately, one of Mark's favorite
expressions. Mark has only two discourses, the parables of the kingdom (4:1–33) and the
eschatological discourse (13:1–37). Miracles abound. Mark has 20 miracle accounts. Combined with
healing summaries, these units comprise a third of the Gospel and are nearly one-half of the first 10
chapters. These pictures of Jesus' authority are important to Mark, as he presents Jesus as one who
teaches with authority. The authority underscores that Jesus is the Christ, the Son of God (1:1; 8:29;
15:39). Mark's Christology presents Jesus as this promised figure. His claims of authority over sin,
human relationships, and practices tied to purity, Sabbath, and temple get him into trouble with the
Jewish leaders, who early on determine they must stop him. This conflict raised by Jesus' claims is
also a central feature of the Gospel.

However, Jesus' authority is not one of raw power. In terms of proportion, Mark highlights Jesus as
the suffering Son of Man and suffering Servant more than the other Gospels. His mission is to come
and give his life as a ransom for many (10:45). The importance of understanding the suffering role

http://breachrepairers.webs.com/ 100
probably explains the commands for silence given to those, including demons, who confess Jesus as
Messiah (1:44; 5:43; 9:9). Without an appreciation of his suffering, Jesus' messianic calling is not
understood. It is here that the pastoral demands of discipleship appear as well (10:35–45; cf. 8:31–
38; 9:33–37). Mark is like Matthew here. After the suffering come glory and vindication. The same
Son of Man will return one day to render judgment, as the eschatological discourse reveals (ch. 13).
The need for discipleship and really listening to Jesus is clear as Mark notes without hesitation the
failures of the disciples. Their instincts will not take them in the right direction. Instead, they must
trust in God and his ways. In addition, Mark notes the emotions of Jesus and the disciples more than
any of the other Gospels.

 He was also a contemporary: Mark appears in Acts, and some of Paul’s writings.
 He was not a disciple
 He got the information by talking with Peter
 He has information which no other gospel writer has
 It was his grandma’s house where the Last Supper was held at.
 The man in chapter 14 who ran away naked must be Mark: all the disciples were gone
including Peter, so there’s no way Peter would know about this and tell Mark.
 His account is very short
 We believe that it was the first gospel written: about 32 years after Jesus died – 65 AD.
Why would it take so long for them to write it? They were busy evangelizing, until Mark was
moved by the Holy Spirit to sit down and write this book.
Mark comes from a devote Christian family. Christians gathered for prayer in her mother
Mary’s house. He’s sername is John. (Acts 12:12)

Mary is Barnabas sister. Mark is nephew of Barnabas. (Col 4:10)

Paul and Barnabas took John Mark with them in their mission trip. (Acts 12:25).

John served as minister in the trip. (Acts 13:5)

After severe trials, John left Paul and returned home (Acts 13:13)

Barnabas wanted to take John again in another mission trip and Paul fiercely opposed
remembering his unfaithfulness in the previous trip. There was a great contention and Paul
not wanting to take John went separate way. Acts 15:37-39

Worked with Peter and considered as son by Peter (1Pet 5:13)

Mark was ministering to Paul when he was in the prison. (Col 4:10)

Paul calls Mark “my fellowlabourer.” (Philemon 23-24)

Paul states that Mark is profitable to him for the ministry. (2Tim 4:11)

Distinctives of Luke
The third Gospel is the longest. It has a mix of teaching, miracles, and parables. Luke gives more

http://breachrepairers.webs.com/ 101
parables than any other Gospel. Whereas Matthew presents teaching in discourse blocks, Luke
scatters his teaching throughout his Gospel, usually in smaller units. Many key discourses happen in
meal scenes (7:36–50; 11:37–52; 14:1–24; 22:1–38; 24:36–49).

Key themes center on God's plan. Things “must be” (Gk. dei) in Luke (2:49; 4:43; 9:22; 24:7, 26, 44–
47). God has designed a plan to reach and deliver the poor, the oppressed, and those caught in
Satan's oppressive grip (4:16–18; 11:14–23). The plan reflects a promise and fulfillment structure,
where key figures express scriptural realization of the plan (7:28; 16:16). The opening infancy
section does this through the use of hymns decorated in scriptural language, underscoring the note
of joy that works through the Gospel. Things also happen with an immediacy, as many texts speak of
what is happening “today” (2:11; 4:21; 5:26; 19:9; 22:34; 23:43). The gospel marches forward, as is
indicated by the geographic progression in the story from Galilee to Jerusalem (9:52–19:44).

Jesus appears as the Messiah-Servant-Lord. The basic category is messianic (1:31–35; 3:21–22;
4:16–30; 9:18–20), but as the story proceeds it is clear that this role is one of great authority that
can be summarized by the image of the judging Son of Man or by the concept of Lord (5:24; 20:41–
44; 21:27; 22:69). All of these connections reflect what Scripture has said about the plan. Jesus also
functions as a prophet like Moses, a leader-deliverer-prophet who is to be heard (4:20–30; 9:35).
Jesus' miracles provide evidence for the inaugurated presence of the kingdom. Ultimately the
kingdom brings with its deliverance the defeat of Satan (11:14–23; 17:20–21). Yet there also is a
future to that kingdom, which will see Jesus return to reign over both Israel and the nations, visibly
expressing the sovereignty he now claims (ch. 21). Thus Jesus' deliverance looks to the realization
of covenantal promises made to Abraham, David, and the nation (1:45–54).

The national leadership is steadfast in its rejection of the message. Nevertheless, the plan proceeds.
Israel will experience judgment for her unfaithfulness (19:41–44; 21:20–24). Her city will be
destroyed as a picture of what final judgment is like and as an assurance that God's program is
taking place. Efforts to call Israel to faithfulness continue despite her refusal to embrace God's care
and Promised One.

In the meantime, Jesus forms a new community (called “the Way” in the book of Acts). This
community is made up of those who turn to embrace Jesus' message and follow in faith. Acts is
really the second half of Luke's story, telling how God led the gospel into the heart of the Roman
Empire, despite stiff opposition, through the boldness of exemplary witnesses drawing on God's
Spirit.

 He got the information from other disciples who were around, and from other things
that were written. Luke 1:2, 3: EGW’s writing.
 He wrote for Theophilus: “Theo” means “God”; “Philus” means “friend.” So Theophilus
means “the friend of God.” This book is for friends of God—you and I.
 He was a Gentile, a Greek
 He was trying to reach the Gentile mind: Begins his genealogy from Adam;
 Tries to anchor the life of Jesus into secular history (Luke 3:1). He’s trying to correlate
what Jesus was doing on earth, with what was going on in the world at that time. If it
weren’t Luke, we would have difficult determining exactly when Jesus was baptized.

http://breachrepairers.webs.com/ 102
Distinctive of John
The fourth Gospel's account emphasizes Jesus as the Sent One from God, who acts in unity with the
Father. John highlights Jesus' uniqueness from the declaration of the incarnation, through a
narration of seven signs, to the use of multiple discourse-dialogues. This Gospel's explicit portrayal
of Jesus gives it its literary power.

John's themes focus on Christology. Unlike the Synoptics, he speaks little of the kingdom. Rather, it
is eternal life that is the key theme to express what the Synoptics call the kingdom promise. The
emphasis in the term “eternal life” is not only the duration of the life (eternal) but also its quality
(i.e., real, unending life). Thus, to know the Father and Jesus Christ whom the Father sent is eternal
life (17:3). This life is available now (5:24–26). Along with the opportunity is also the prospect of
judgment for those who refuse it (3:16–21, 36).

The promise is brought by the Word/Logos sent from God in the form of human flesh. The “I Am”
sayings convey various ways in which Jesus represents the way of God. Each image (light of the
world, the resurrection and the life, the good shepherd, the bread of life, the vine) specifies some
central role that belongs to Jesus. As Son, Jesus only does that which the Father shows him. It is the
unity with the Father in mission that John highlights. Jesus is the hoped-for Messiah, as well as the
Son of Man who ascends and descends between earth and heaven. In this role, he will judge (5:27),
be lifted up (3:14), and serve in mediating salvation (3:13; 6:27). Even when Jesus is seen as a
prophet, it is as a leader-prophet like Moses (6:14; 7:40).

Seven signs dominate the first two-thirds of the Gospel. The response to them covers the range from
rejection (12:37–39) to openness (9:25). Interestingly, unlike the Synoptics, there is no casting out
of demons in John. He focuses on acts of healing, restoration, and provision. What these signs
especially highlight is Jesus' superiority to Jewish institutions (1:17; 2:19–21; 7:37–39; 9:38; 10:1–
18). Most of the miracles take place in a setting of Jewish celebrations and underscore how Jesus
provides what the feasts celebrate. At the end of the Gospel, blessing comes to those who have faith
without the need for such signs (20:29).

Jesus is seen as the revelator of God. He makes the Father and his way known, functioning as light
(1:14–18). Jesus' death shows the love of the Father for his own people and is an example to
disciples of how they should love (13:1, 11–17). Jesus' death also serves to gather God's people
together (10:1–18) and is a means by which the Son and Father are glorified as life is made
available though him (3:14–16).

Also of great importance to John is the Spirit, also called the Helper (Gk. paraklētos; see John 14:16–
18, 26; 15:26; 16:7–14; 20:22), the one Jesus sends after his death, a point Acts also highlights. This
encourager-enabler leads the disciples into the truth, empowers them for ministry and mission, and
convicts the world of sin, righteousness, and judgment (John 14:25–31; 16:8–11). He is the one who
sustains life (4:8–10; 7:37–39).

 He was a disciple: the beloved


 He wrote 5 books – John, 1,2,3 John and Revelation
 He wrote the book of John after he wrote the book of Revelation: If the Lord’s day was
Sunday, in the book of John he would have recorded something. He never, ever mentioned
Sunday as sacred day, but always referred to the Sabbath.

http://breachrepairers.webs.com/ 103
 John is not part of the synoptic gospels: “syn” means “together”; “optic” means “eye.”
“Synoptic” means “to see together.” Matthew, Mark and Luke almost have the same content
– they are called the Synoptic gospel. But John is unique.
 Purpose of John’s gospel (20:31) is to prove that Jesus is divine & to experience salvation:
he is the clearest on the issue of conversion and salvation. It’s experiential.

Distinctives of Acts
Acts teaches that the new community is rooted in old promises. It does this by telling how God
directed the inclusion of Gentiles and took the message from Jerusalem to Rome. The central figures
in the book are Peter (chs. 1–5; 10–12); evangelists from the Hellenistic believing community, such
as Stephen and Philip (chs. 6–8); and Paul (chs. 9; 13–28). Discourses are important to the book,
whether they be missionary speeches to call people to belief or defense speeches where the Christian
mission is explained. In the end, the book makes it clear how an originally Jewish movement came
to include Gentiles. The gospel can go to all the world because (1) Jesus is Lord and (2) God directed
that the gospel go into all the world. The book ends on a note of triumph as the gospel comes to
Rome, even though believers suffered in terms of injustice and physical persecution in an effort to
get the gospel there.

http://breachrepairers.webs.com/ 104
The Four Gospels
There are 4 gospel accounts because they all emphasize different characteristics of Jesus just like
the sanctuary.

Because sin took place, and because Jesus came to this earth, something happened that would never
have happened. What was it? God is doing something for us that is greater that would not have
happened had we not sinned. He brought us back and brought us higher through the plan of
salvation.

Luke gives us an account on the life of John the Baptist that you will not find anywhere else in the
Bible.
Luke 1:
John 1:15
Mark 1:1-3

Why did they all begin with introducing John the Baptist? Because before there is ever a coming of
Jesus Christ, there will always be one announcing His coming, to prepare the way for Him.

DA 101 In preparing the way for Christ's first advent, he was a representative of those who are to
prepare a people for our Lord's second coming.

His life parallel’s with the lives of the SDA’s. He was predicted by prophecy, he was a reformer in
diet, he was a reformer in dress, and he has a message of the Sanctuary.

The gospels are not to cover every aspects of Jesus’ life.

Jn 21:25 And there are also many other things which Jesus did, the which, if they should be written
every one, I suppose that even the world itself could not contain the books that should be written.
Amen.

These writers are trying to help us each see a different aspects of Jesus. But most importantly, to
point us to Him as the source of salvation. There are 4 gospel accounts because they all emphasize
different characteristics of Jesus just like the sanctuary:

 Matthew: King Lion


 Mark: Servant Cow
 Luke: Humanity Man
 John: Divinity Eagle

Timeline of the Life of Jesus


This is my own personal edited timeline. I have chosen random events to give an overall timeline.

1. The Birth of Jesus


2. Angels & Shepherds
3. Visit of Magi (Pic Ava) (Matt 2)
4. Flight of Jesus to Egypt (Matt 2)
5. First Passover visit at the age of 12
6. The Baptism of Jesus (Matt 3, Mark 4, Luke 4)

http://breachrepairers.webs.com/ 105
7. The temptation of Jesus (Matt 4, Mark 4, Luke 4)
8. First Passover, A.D. 28 – Second Passover A.D. 29
a. Nicodemus (Pic Ava) (John 3)
b. Samaritan Woman (John 4)
c. Nobleman’s Son
d. John Imprisoned
9. Second Passover, A.D. 29 – Third Passover A.D. 30
a. Call by the sea
b. Plucking Grain on the Sabbath (Matt 12)
c. Sermon on the mount (Matt 5-8)
d. Sermon by the sea (Matt 13)
i. The tares
ii. The Sower and 4 different grounds
e. Death of John the Baptist (Matt 14)
10. Third Passover A.D. 30 – Fourth Passover A.D. 31
a. Feeding the 5,000
b. Jesus walks on the Lake (Peter sinks)
c. Feeding the 4,000
d. The Blind man near Bethsaida
e. The transfiguration (Matt 17)
f. The Adulteress (John 8)
g. Mission of the Seventy
h. The good Samaritan (Luke 10)
i. Lost Sheep (Luke 15)
j. Lost Coin (Luke 15)
k. Prodigal Son (Luke 15)
l. The Raising of Lazarus (John 11)
m. The ten lepers
n. The rich young ruler (Matt 18)
o. Betrayal Plot
11. Passion Week (Fourth Passover A.D. 31)
a. Triumphal Entry (Matt 21-Monday)
b. Second Cleansing of the temple
c. Mt. Olives (Signs of Christ Return) (Tuesday)
d. The Lord’s Supper (Thursday)
e. Gethsemane (Friday)
f. Judas confession and suicide (Friday)
g. First trial before Pilate (Friday)
h. Hearing before Herod Antipas (Friday)
i. Second trial before Pilate
j. The crucifixion (6th Hour-12:00pm)
k. The Burial
l. Resurrection
12. Resurrection to Ascension (Spring, A.D. 31)
a. Walk to Emmaus
b. First appearance in the upper room
c. Appearance on a mountain in Galilee
d. The Ascension

http://breachrepairers.webs.com/ 106
Summary of Chapter 1
Because we sinned…

Because sin took place, and because Jesus came to this earth, something happened that would never
have happened. What was it?

God is doing something for us that is greater that would not have happened had we not sinned. He
brought us back and brought us higher through the plan of salvation.

Let’s say you wrecked a Honda and take it to the body shop, and now they bring you out a Mercedes
Benz. It wasn’t restored exactly the same as before, it’s actually better now. God actually is doing
something for us that is greater than what would have happened had we not sinned. It shows you
the extent of the work of redemption. God didn’t just brought us back, but He brought us back and
higher.

DA 25 By His life and His death, Christ has achieved even more than recovery from the ruin wrought
through sin. It was Satan's purpose to bring about an eternal separation between God and man; but
in Christ we become more closely united to God than if we had never fallen. In taking our nature, the
Saviour has bound Himself to humanity by a tie that is never to be broken. Through the eternal ages
He is linked with us.

We can now more closely united to God, because God became a man.

This is the theme of this whole book called, “Desire of Ages”—Jesus becoming a man, and how
through becoming a man, He is going to restore the image of God in us, and He’s going to brings us
to become something we’ve never been before, perfectly united with Christ and He is going to be
our elder brother. We will become closer to God than if we’ve never sinned.

Our world as a lesson book

DA 19 Our little world is the lesson book of the universe. God's wonderful purpose of grace, the
myste ry of redeeming love, is the theme into which "angels desire to look," and it will be their study
throughout endless ages.

Others are watching us even as we speak right now. They’re watching our planet to see the
resolution of the issue in the Great Controversy.

Summary of Chapter 2
Chapter 2, what was God’s purpose for the nation of Israel? They were to be the examples for the
whole earth for what God wanted to do His people.

DA 27 But the Israelites fixed their hopes upon worldly greatness.

Your theology can be right, but if your philosophy is wrong, your theology will end up being wrong
too. Israelites had the truth, they understood what was right. But the problem was that all the truth
they had was eclipsed by the idea of worldly greatness. Because of that, they started to go wrong.

DA 27 It was in vain that God sent them warning by His prophets. In vain they suffered the
chastisement of heathen oppression. Every reformation was followed by deeper apostasy.

http://breachrepairers.webs.com/ 107
Because they had their hopes fixed upon worldly greatness, all these reformations were followed by
deeper apostasy.

Listen, young people, here you are studying to be a missionary. This school is not to train you for
success in the business world, nor political world. We are training you for service. But there may be
a philosophy in your mind, “I want to be somebody great. I want to be famous!” If you are still
seeking for a worldly greatness, you may come out of school with the greatest truth, but you will
always be striving to exalt self. In that sense, ultimately with that kind of philosophy, God can’t bless
you, and you will continue to go down and down until the thought changes and even your theology
will end up being wrong too. Listen, young people, as you learn the truth and gain experiences, you
may caught up in the mood of “I’m somebody now.” When a small group of people ask you to come
and speak for them you wonder if you should even go there. In other words, you are saying, “I’m too
good for them!”

DA 30 At the time of the birth of Christ the nation was chafing under the rule of her foreign masters,
and racked with internal strife. The Jews had been permitted to maintain the form of a separate
government; but nothing could disguise the fact that they were under the Roman yoke, or reconcile
them to the restriction of their power.
The Romans did control Palestine and Judea, but they still allowed a puppet government to exist. So
there were certain things they did allow and they didn’t. That sets the stage for the closing scenes of
Christ. What was taking place right then was a fulfillment of the prophecy. Jacob, before he died,
prophecied:

Gen 49:10 The sceptre shall not depart from Judah, nor a lawgiver from between his feet, until Shiloh
come; and unto him shall the gathering of the people be.

After Jesus died, Israel lost all their privileges and Jerusalem was destroyed.

DA30 Hatred of the Romans, and national and spiritual pride, led the Jews still to adhere rigorously
to their forms of worship…They had studied the prophecies, but without spiritual insight.

With that wrong mentality, desire of human worldly greatness, these people have gotten
themselves to the point where they were so far down and ruled by someone else, and having
limited powers.

Summary of Chapter 3
DA 32 When the fullness of the time was come, God sent forth His Son." Providence had directed the
movements of nations, and the tide of human impulse and influence, until the world was ripe for the
coming of the Deliverer. The nations were united under one government. One language was widely
spoken, and was everywhere recognized as the language of literature. From all lands the Jews of the
dispersion gathered to Jerusalem to the annual feasts. As these returned to the places of their
sojourn, they could spread throughout the world the tidings of the Messiah's coming.

This parallels the time right before Christ’s second coming.

One government:

You can freely go any part of the world. In 1888, there was no country where missionary couldn’t
go. God sent the message to carry throughout the whole world. If that message had been taken, SOP
tells us that in two years Jesus would have come. Though the whole world wasn’t under one

http://breachrepairers.webs.com/ 108
government, the message of the gospel could go anywhere. In these days, you can go almost any
country you want.

One language:

English is widely used in our globe.

Jews returning to different parts of the world: Communication


Internet, Radio, satellite, TV.

The world right now is very similar to the conditions that existed in the first coming of Jesus.

DA 32 At this time the systems of heathenism were losing their hold upon the people. Men were
weary of pageant and fable. They longed for a religion that could satisfy the heart. While the light of
truth seemed to have departed from among men, there were souls who were looking for light, and
who were filled with perplexity and sorrow. They were thirsting for a knowledge of the living
God, for some assurance of a life beyond the grave.

http://breachrepairers.webs.com/ 109
Matthew
Introduction
Matthew GK 3156 = “gift of Jehovah”

Author and Title


Since none of the four Gospels includes the names of their authors in the original manuscripts, they
are all technically anonymous. This is not surprising, since the authors likely compiled their Gospel
accounts for members of their own churches, to whom they were already well known. However,
historical documents from early church history provide significant insight into the Gospels'
authorship. The earliest traditions of the church are unanimous in attributing the first Gospel to
Matthew, the former tax collector who followed Jesus and became one of his 12 disciples. The
earliest and most important of these traditions comes from the second century in the writings of
Papias, bishop of Hierapolis in Asia Minor (c. a.d. 135), and Irenaeus, bishop of Lyons in Gaul (c.
175). Because these early church leaders had either direct or indirect contact with the apostolic
community, they would have been very familiar with the Gospels' origins. Moreover, no competing
traditions now exist (if they ever did) attributing Matthew's Gospel to any other author. If Matthew
did not write the book, it is hard to see why the false ascription would bear the name of a relatively
obscure apostle when more well-known and popular figures could have been chosen (e.g., Philip,
Thomas, or James).

When Jesus called him, Matthew was sitting in the tax collector's booth (9:9), collecting taxes for
Herod Antipas, and this may have been along a commercial trading route about 4 miles (6.4 km)
from Capernaum. However, since the narrative surrounding Matthew's call is set in Capernaum
(9:1, 7, 10; cf. 4:13), the tax booth may have been on the Sea of Galilee at Capernaum, since Herod
also taxed fishermen. At his calling in the first Gospel he is referred to as “Matthew” (9:9), while
Mark's and Luke's Gospels describe him as “Levi the son of Alphaeus” (Mark 2:14) and “Levi” (Luke
5:27). The reason for the variation in names has elicited much discussion, but most scholars believe
that the tax collector had two names, Matthew Levi, which he either possessed from birth or took
on following his conversion. His occupation as a tax collector implies that he had training in scribal
techniques and was thus able to write, while his identity as a Galilean Jewish Christian suggests his
ability to interpret the words and actions of Jesus in light of OT messianic expectations.

Date
The precise date of the writing of Matthew's Gospel is not known. Some people argue for a date
earlier than the destruction of Jerusalem in a.d. 70, since Jesus alludes to this event in 24:1–28.

Theme
This is the story of Jesus of Nazareth, recorded by the apostle Matthew as a compelling witness that
Jesus is the long-anticipated Messiah, who brought the kingdom of God to earth and is the
prophesied fulfillment of God's promise of true peace and deliverance for both Jew and Gentile.

Key Themes
1. Portrait of Jesus. Jesus is the true Messiah, Immanuel (God incarnate with 1:1, 23; 2:2;
his people), Son of God, King of Israel, and Lord of the church. 14:33;
16:16;

http://breachrepairers.webs.com/ 110
18:20; 21:5–
9
2. The bridge between Old and New Testaments. Jesus fulfills the hopes and 1:1–17, 22–
promises of the OT through his messianic genealogy, fulfillment of OT 23; 2:4–5, 15,
prophecies, and fulfillment of the OT law. These bridging qualities may have 17, 23; 5:17–
been one reason Matthew was chosen to begin the NT canon. Another 20
possible reason is that many in the early church thought that Matthew was
the first Gospel written, and another is that it was personally written by an
apostle, in contrast to Mark and Luke.
3. Salvation-historical “particularism” and “universalism.” Matthew's Gospel 10:5–6; 28:19
traces God's continuing work of salvation within Israel (“particularism”) and
extends this saving work to all the peoples of the earth (“universalism”),
through the person and work of Christ.
4. The new community of faith. The early church included both Jewish and 11:28; 16:18–
Gentile Christians. Matthew's Gospel would have encouraged them to 19; 28:19
transcend ethnic and cultural barriers to find unity in service to Jesus the
Messiah as members of his universal church.
5. The church is built and maintained by Jesus' continuing presence. God's 16:18; 18:15–
saving work in the present age is carried out chiefly by and through the 20; 22:10;
church, which Jesus continues to build and inhabit. Anyone who responds to 28:20
Jesus' call—whether Jew or Gentile, male or female, rich or poor, slave or
free—is brought into the fellowship of his church to enjoy him and
participate in the community of his kingdom.
6. A “great commission” for evangelism and mission. Jesus' command to “make 28:19
disciples of all nations” is found only in Matthew and has motivated countless
believers to reach out to the lost with the good news of the gospel. As Jesus
made disciples in his earthly ministry, he commissions his church to follow
his example.
7. Jesus' five discourses recorded in Matthew can be viewed as a manual on chs. 5–7; 10;
discipleship. The presentation of five of Jesus' major discourses, addressed at 13; 18–20;
least in part to his disciples, forms the most comprehensive collection of 24–25
Jesus' instructional ministry found anywhere in Scripture. They paint a
holistic picture of life lived in obedience to Christ, and the church has used
them to instruct disciples through the ages.

Purpose, Occasion, and Background


Matthew crafted his account to demonstrate Jesus' messianic identity, his inheritance of the Davidic
kingship over Israel, and his fulfillment of the promise made to his ancestor Abraham (Matt. 1:1) to
be a blessing to all the nations (Gen. 12:1–3). Thus in large part Matthew's Gospel is an evangelistic
tool aimed at his fellow Jews, persuading them to recognize Jesus as their long-awaited Messiah. At
the same time, the Gospel reveals clearly to Gentiles that salvation through Jesus the Messiah is
available to all nations. For Jewish Christians, Matthew's Gospel provides encouragement to stand
steadfast amid opposition from their own countrymen, as well as Gentile pagans, secure in the
knowledge of their citizenship in God's kingdom.

Against the backdrop of such opposition to Jesus' message, Matthew establishes the identity of
Christ's church as the true people of God, who now find their unity in service to Jesus despite
previous racial, class, and religious barriers. His Gospel provides necessary instruction for all future
disciples, Jew and Gentile, who form a new community centered upon devotion and obedience to

http://breachrepairers.webs.com/ 111
Jesus the Messiah amid significant opposition.

Many scholars have suggested that the prominent church in Antioch of Syria, whose members
included both Jewish and Gentile Christians (cf. Acts 11:19–26; 13:1–3), was the intended audience
of Matthew's Gospel. They point to the Gospel's influence on Ignatius, an early bishop of Antioch. At
the same time, Matthew's message spoke to all of the fledgling churches of his day, and the Gospel
appears to have circulated rapidly and widely.

History of Salvation Summary – Matthew & the Kingdom of God


Jesus comes as the messianic King in the line of David to fulfill the OT, especially its promises of
everlasting salvation. The ultimate fulfillment comes with his crucifixion and resurrection. This
section covers topical studies from the book of Matthew

Second Coming

Matt 10:23 But when they persecute you in this city, flee ye into another: for verily I say unto you, Ye
shall not have gone over the cities of Israel, till the Son of man be come.

First mention of the Second Coming

Matt 16:27 For the Son of man shall come in the glory of his Father with his angels; and then he shall
reward every man according to his works.

He comes to reward according to our works. This verse shows by implication the investigative
judgment

Matt 13:40-43 [40] As therefore the tares are gathered and burned in the fire; so shall it be in the
end of this world. [41] The Son of man shall send forth his angels, and they shall gather out of his
kingdom all things that offend, and them which do iniquity; [42] And shall cast them into a furnace of
fire: there shall be wailing and gnashing of teeth. [43]Then shall the righteous shine forth as the sun
in the kingdom of their Father. Who hath ears to hear, let him hear.

Shows when He comes and what the rewards are. Note: V.41 Calls it the kingdom of Christ; V.43
calls it the kingdom of the Father (Study).

Matt 19:28 And Jesus said unto them, Verily I say unto you, That ye which have followed me, in the
regeneration when the Son of man shall sit in the throne of his glory, ye also shall sit upon twelve
thrones, judging the twelve tribes of Israel.

We will judge Israel after His coming. Note: Then the kingdom of Glory will come. See Ps 102:16

Matt 8:11 And I say unto you, That many shall come from the east and west, and shall sit down with
Abraham, and Isaac, and Jacob, in the kingdom of heaven.

Abraham, Isaac, Jacob all sitting together with many in the kingdom of heaven..= Note: If you
connect this with chapter 19:28 they will be judging.

Luke 22:30 That ye may eat and drink at my table in my kingdom, and sit on thrones judging the
twelve tribes of Israel.

Shows that we will sit down and eat and drink together.

http://breachrepairers.webs.com/ 112
Matt 26:29 But I say unto you, I will not drink henceforth of this fruit of the vine, until that day when
I drink it new with you in my Father's kingdom.

Shows that we will be drinking grape juice together Note: Christ calls it His Father’s kingdom; Mark 14:25
Christ calls it the Kingdom of God, therefore God is the Father; Luke 22:18 He says He won’t drink until the
Kingdom of God shall come. That means the kingdom of the Father.

1 Corin 15:24-28 [24] Then cometh the end, when he shall have delivered up the kingdom to God,
even the Father; when he shall have put down all rule and all authority and power. [25] For he must
reign, till he hath put all enemies under his feet. [26] The last enemy that shall be destroyed is death.
[27] For he hath put all things under his feet. But when he saith all things are put under him, it is
manifest that he is excepted which did put all things under him. [28] And when all things shall be
subdued unto him, then shall the Son also himself be subject unto him that put all things under him,
that God may be all in all.

This must be the best passage showing the transition between the kingdom of Christ and kingdom of God

Matt 23:39 For I say unto you, Ye shall not see me henceforth, till ye shall say, Blessed is he that
cometh in the name of the Lord.

There will be a special resurrection at the Second Coming. Note: this could also apply to after the Millennium.

Matt 26:64 Jesus saith unto him, Thou hast said: nevertheless I say unto you, Hereafter shall ye see
the Son of man sitting on the right hand of power, and coming in the clouds of heaven.

Caiaphus will be among the special resurrection of the wicked.

Kingdom of God & Christ

Matt 28:18 And Jesus came and spake unto them, saying, All power is given unto me in heaven and in
earth.

(Relationship of Father and Son), Before you can completely understand the Kingdom of God and
the Kingdom of Christ, you must understand the relationship that exists between God (the Father)
and the Son (Jesus). They are equal in the sense of both are God. But when it comes to the roles that
they each have in relationship to their created beings they are different. They have these roles so
that the created beings can understand and comprehend who God is to them. Now after the fall of
man, they take different roles in the plan of salvation, God the Son, and the Holy Spirit all work
together for the salvation of man. You also must understand their roles in the light of the great
controversy Satan has accused the Father and the Son and therefore God reveals certain details to
help us understand how God has met the accusations of Satan.

“All power is given unto me in heaven and in earth” – When we read this verse a natural
questions should come to mind. What? Does this mean Jesus didn’t have all power in heaven and
earth before? It also shows that the Father is Supreme Sovereign. If He gives all power to His Son, it
means that the Father has all power.

Matt 11:27 All things are delivered unto me of my Father: and no man knoweth the Son, but the
Father; neither knoweth any man the Father, save the Son, and [he] to whomsoever the Son will
reveal [him].

“All things are delivered unto me of my Father” – This shows that everything the Son has in His

http://breachrepairers.webs.com/ 113
possession comes from the Father. He receives everything from the Father. Note: It also shows their
relationship. The Son reveals the Father

Jn 3:35 The Father loveth the Son, and hath given all things into his hand.

Jn 13:3 Jesus knowing that the Father had given all things into his hands, and that he was come from
God, and went to God.

Jn 14:28 I go unto the Father: for my Father is greater than I.

This is not speaking in reference to Divinity in the sense of the Father being a greater God. But the
Son as a man willing places Himself in subjection to His Father as an example of how you and I are
suppose to place ourselves in subjection to Christ. Christ says…

Jn 14:31 …as the Father gave me commandment, even so I do.

This shows that as a man in fallen nature Jesus obeyed. Again we now it is not talking about divinity
because Jesus says

Jn 10:30 I and [my] Father are one…

Phil 2:6 Who, being in the form of God, thought it not robbery to be equal with God:

Both of these verses show that Jesus is equal with God! Therefore, when Christ says the Father is
greater or all power is given to me, it is in reference to their roles in the government of God. All
government must have order and a supreme head. And others must fulfill other roles for the
government to operate efficiently and this is what I see.

1 Corin 15:27 For he hath put all things under his feet. But when he saith all things are put under
[him, it is] manifest that he is excepted, which did put all things under him.

Again, this re-emphasizes the fact that the Father is greater than the Son in reference to the
government of God. So when Matthew says ‘All power is given’ what does it mean? When did it
happen? From Matt 28 we can see that it happened after His resurrection. This power refers to all
ecclesiastical and political power. Both being given at His resurrection and both power being
wielded in different ways. Did the Son have this power before? I would have to say yes, but He had
that power in His divine form. Was the Son a King before? Yes, but a co-ruler. Was He revealed as a
King? (In heaven I am not sure, but at some point after the creation of this earth He was) Note:
There is much that is revealed to us about the Kingdom of God and Christ after 31 A.D. before that
we have glimpses, but it is not as clear. More study is needed

Is 6:1-4 I saw also the Lord sitting upon a throne, high and lifted up.

Jn 12:41 These things said Esaias, when he saw his glory, and spake of him.

It was Christ on the throne, as a king, but again it was the throne of the Father.

Phil 2:6-7 [6] Who, being in the form of God, thought it not robbery to be equal with God: [7] But
m:ade himself of no reputation, and took upon him the form of a servant, and was made in the
likeness of men.

Christ was equal with God. That means that He was God. But the bible says He ‘made himself of no

http://breachrepairers.webs.com/ 114
reputation’. That means that He clothed His divinity with humanity, hiding His glory which would
have made it manifest that He was God. Previously He was in the “form of God” now in the “form of
a servant”. So when you look at Him He just looked like an ordinary man.

Phil 2:8 And being found in fashion as a man, he humbled himself, and became obedient unto death,
even the death of the cross.

Note: ‘found’ Gk 2147 “2 to find by enquiry, thought, examination, scrutiny, observation, to find out
by practice and experience. 2A to see, learn, discover, understand. 2B to be found i.e. to be seen, be
present. 2C to be discovered, recognised, detected, to show one’s self out, of one’s character or state
as found out by others (men, God, or both)1 ”

Phil 2:9 Wherefore God also hath highly exalted him, and given him a name which is above every
name:

“Wherefore” – meaning because He went from being in the form of God to the form of a servant
and being obedient unto death.

“hath highly exalted him” – exalting the Son in the ‘form of a servant….in the likeness of men’. So
He is exalting the Son as a Man. And the name Jesus is above all names.

Phil 2:10 That at the name of Jesus every knee should bow, of [things] in heaven, and [things] in
earth, and [things] under the earth;

Phil 2:11 And [that] every tongue should confess that Jesus Christ [is] Lord, to the glory of God the
Father.

Now all of heaven and earth acknowledge Him as Lord in Human form. (He was Lord before in
Divine form Is 6:1-4) Now He is Lord as Jesus a man, but also Christ, God. This further explains Matt
28:18 “All power is given unto me (as God in human form) in heaven and earth” every knee in
heaven and earth shall bow. Political power

“to the glory of God the Father” – This is the great controversy theme. When we all acknowledge
Christ as Lord. This vindicates the Father who has exalted Him. He was not being arbitrary towards
Satan. He exalted His Son because His Son was God.

Eph 1:20-21 [20] Which he wrought in Christ, when he raised him from the dead, and set [him] at his
own right hand in the heavenly [places] [21] Far above all principality, and power, and might, and
dominion, and every name that is named, not only in this world, but also in that which is to come:”

This is clear that Jesus will be exalted over all power and dominion and every name that is named.
But because He will sit on the right hand of God. It means that Christ is above everything except for
the Father. He is equal with the Father. The Father is above all things and the Son is above all things,
but the Father is above Christ. In this world and the world to come.

Eph 1:22 And hath put all [things] under his feet, and gave him [to be] the head over all [things] to
the church.

Notice He is the head over all things to the church. That is complete ecclesiastical power. Note: The
papacy desires to be over all things to the church. This shows he is the antichrist.

1 Strong, J. 1996. The exhaustive concordance of the Bible

http://breachrepairers.webs.com/ 115
Eph 1:23 Which is his body, the fulness of him that filleth all in all.

This shows that the church also will be above all things. But Christ will be above the church. The
hierarchy will be. The Father, The Son, The Church, and all principalities and powers, etc. Extra

Col 1:18 And he is the head of the body, the church: who is the beginning, the firstborn from the
dead; that in all [things] he might have the preeminence.

This explains the same concept. The Son has preeminence over everything except the Father. Note:
All these passages show Jesus being exalted and having preeminence after the resurrection

Eph 5:23-24 [23] …Christ is the head of the church… [24] the church is subject unto Christ.

This means that the church is not subject to anything else. They are above everything except for
Christ and the Father. And Christ is above everything except the Father. This is further proven by…

1 Corin 15:28 And when all things shall be subdued unto him, then shall the Son also himself be
subject unto him that put all things under him, that God may be all in all.

“And when all things shall be subdued unto him”

Ps 18:39 For thou hast girded me with strength unto the battle: thou hast subdued under me those
that rose up against me.

This describes the battle that takes place at the end of the millennium. It gives a time frame to when
all things will be subdued to Christ.

Rev 20:7-9 [7] And when the thousand years are expired, Satan shall be loosed out of his prison, And
shall go out to deceive the nations which are in the four quarters of the earth, Gog and Magog, to
gather them together to battle: the number of whom [is] as the sand of the sea. And they went up on
the breadth of the earth, and compassed the camp of the saints about, and the beloved city: and fire
came down from God out of heaven, and devoured them.

Ps 18:47 [It is] God that avengeth me, and subdueth the people under me. He delivereth me from
mine enemies: yea, thou liftest me up above those that rise up against me… Great deliverance giveth
he to his king; and showeth mercy to his anointed, to David, and to his seed for evermore (His seed is
Christ)

“then shall the Son also himself be subject unto him that put all things under him”

1 Corin 11:3 …the head of Christ is God.

Note: Same as 1 Corin 3:23 “Christ is God’s”

Phil 3:20-21 [20] For our conversation is in heaven; from whence also we look for the Saviour, the
Lord Jesus Christ: [21] Who shall change our vile body, that it may be fashioned like unto his glorious
body, according to the working whereby he is able even to subdue all things unto himself

And because the Father subdued all things under Christ. He is able to change our bodies and give us
immortality. The Father put all enemies under His feet. And according to that power that was given
Christ by the Father He gives us immortality. This verse can also be understood as according the

http://breachrepairers.webs.com/ 116
working of the Father Christ is able to subdue all things to Himself. The Father said sit on my right
hand until I make thy enemies thy footstool. So the Father makes His enemies His footstool. The last
enemy is death. Now Christ can exercise power to destroy death.

1 Corin 15:24-28 [24] Then cometh the end, when he shall have delivered up the kingdom to God,
even the Father; when he shall have put down all rule and all authority and power. [25] For he must
reign, till he hath put all enemies under his feet. [26] The last enemy that shall be destroyed is death.
[27] For he hath put all things under his feet. But when he saith all things are put under him, it is
manifest that he is excepted which did put all things under him. [28] And when all things shall be
subdued unto him, then shall the Son also himself be subject unto him that put all things under him,
that God may be all in all.

These verses give the most comprehensive understanding of the Kingdom of God and Christ. The
theme of 1 Corin 15 is the resurrection. It also includes the second coming and the kingdoms of God
and Christ. This is because they are all related. The citizens of the kingdom are those who died in
Christ. Therefore Christ must return and resurrect them and then finally Christ and His people will
be a part of the Father’s kingdom (Still in study)

1 Corin 15:24 Then [cometh] the end, when he shall have delivered up the kingdom to God, even the
Father; when he shall have put down all rule and all authority and power.

“End” – End of the plan of Salvation; or most likely the end of the reign of Christ. (this is hard to
understand because other passages say that Christ kingdom will last forever. Therefore it has to be
one of two main ideas. 1.) Forever means duration or a set amount of time in the same sense of
Samuel being given to the Lord forever as a child, or David’s kingdom lasting forever. If I maintain
this position it would mean Christ kingdom of glory would last from the close of probation until the
end of the millennium. 2.) Forever means eternally and even though the kingdom will be given to
the Father Christ reigns jointly with His Father. From this verse we can see Christ was given all rule,
authority, and power from the Father as related to Christ’s Kingdom. Paul gives the end picture
right here. Then he will explain what leads up to this event.

1 Corin 15:25 For he must reign, till he hath put all enemies under his feet.

“reign” – This word explains all rule, authority and power. Note: Matt 28:18 Christ said all power is
given unto me in Heaven and in Earth. Once all enemies are subdued under his feet He no longer
will reign or have all rule, authority and power. The question is when did He begin to reign? Notice
the OT passages that teach about the reign of Christ

Ps 2:6 Yet have I set my king upon my holy hill of Zion.

It sounds like present tense, but when did the Father set His king upon His holy hill? Note: This
verse makes it clear also that the Father was the sole King over His own Kingdom

Ps 2:7 I will declare the decree: the LORD hath said unto me, Thou [art] my Son; this day have I
begotten thee.

Decree: Means commandment or law; this verse shows future tense. When will the Father sent His
king of His holy hill? When He declares thou art my Son; this day have I begotten thee. Note: This
verse implies several things. First this doesn’t necessary mean that the Son wasn’t a king prior to
this even, but that He wasn’t a King as the begotten Son of God. When was Christ begotten? At his
resurrection

http://breachrepairers.webs.com/ 117
Acts 13:33 God hath fulfilled the same unto us their children, in that he hath raised up Jesus again; as
it is also written in the second psalm, Thou art my Son, this day have I begotten thee.

Ps 2:8 Ask of me, and I shall give [thee] the heathen [for] thine inheritance, and the uttermost parts
of the earth [for] thy possession.

“shall give” shows future tense again. So in the future the Father will give the whole earth for His
possession (or kingdom).

Ps 2:9 Thou shalt break them with a rod of iron; thou shalt dash them in pieces like a potter's vessel.”

Future tense again. Another promise

Ps 110:1 The LORD said unto my Lord, Sit thou at my right hand, until I make thine enemies thy
footstool.

Now when did this take place? This could not be prior to Him coming to this earth. The enemies of
Christ were cast from heaven. This took place at His resurrection.

Summary: At His resurrection the Father set on His holy hill. At His resurrection He was begotten as
a Son of God in human form. At His resurrection He was made a High Priest after the order of
Melchizedeck to rule as king and priest.

Heb 10:12 But this man, after he had offered one sacrifice for sins for ever, sat down on the right
hand of God; From henceforth expecting till his enemies be made his footstool.

This makes it crystal clear that it was after His death and resurrection. That means that Christ has
been expecting, waiting, for the Father to make His enemies His footstool. The Father will
accomplish this through the work of Christ in the Holy Place and Most Holy Place.

Acts 2:32-36 [32] This Jesus hath God raised up…[33] Therefore being by the right hand of God
exalted…[34] For David is not ascended into the heavens: but he saith himself, The Lord said unto my
Lord, Sit thou on my right hand, [35] Until I make thy foes thy footstool. [36] Therefore let all the
house of Israel know assuredly, that God hath made that same Jesus, whom ye have crucified, both
Lord and Christ

“Lord” – Master or King

“Christ” – Anointed Priest

This began in 31 A.D. At this time Christ is ruling in the midst of His enemies Ps 110:2, but He is
sharing the throne of His Father as a King in Human form. When Jesus went to heaven in 31 A.D. He
was called the King of glory. Why? Not just king of heaven, but it was in anticipation that His
kingdom would be established (Luke 22:29).

Ps 24:3 Who shall ascend into the hill of the LORD? or who shall stand in his holy place?

This is the same hill of Ps 2 that the Father says He will set His king on His holy hill.

Ps 24:4 He that hath clean hands, and a pure heart; who hath not lifted up his soul unto vanity, nor
sworn deceitfully.

http://breachrepairers.webs.com/ 118
Christ has clean hands, a pure heart was not proud and did not swear deceitfully.

Ps 24:7 Lift up your heads, O ye gates; and be ye lift up, ye everlasting doors; and the King of glory
shall come in.

This is in anticipation that His kingdom of glory would be established. So the kingdom of grace
prepares the way for the kingdom of glory.

“enemies” – This is a long study. The heathen or kings of this earth are his enemies, sin, death, the
grave, etc (study out later).

1 Corin 15:26, 55 [26] The last enemy [that] shall be destroyed [is] death. [55] O death, where [is]
thy sting? O grave, where [is] thy victory?

Hos 13:5 I will ransom them from the power of the grave; I will redeem them from death: O death, I
will be thy plagues; O grave, I will be thy destruction: repentance shall be hid from mine eyes.

“ransom…power of the grave” – 1st Resurrection

“grave…thy destruction” – After the millennium. Note: Grave is also likened to a prison. Who has
the keys? The ward of the prison. Who has the keys to hell and death? Jesus Rev 1. When is death
destroyed?

Rev 20:14 And death and hell were cast into the lake of fire. This is the second death

1 Corin 15:27 For he hath put all things under his feet. But when he saith all things are put under
[him, it is] manifest that he is excepted, which did put all things under him.

Kingdom of God & Christ (We receive a Kingdom like Christ)

Matt 19:28 And Jesus said unto them, Verily I say unto you, That ye which have followed me, in the
regeneration when the Son of man shall sit in the throne of his glory, ye also shall sit upon twelve
thrones, judging the twelve tribes of Israel.

We judge in the kingdom of Christ

Luke 22:29 And I appoint unto you a kingdom, as my Father hath appointed unto me;

Christ appoints us a kingdom as the Father appointed Him a kingdom

Matt 25:34 Then shall the King say unto them on his right hand, Come, ye blessed of my Father,
inherit the kingdom prepared for you from the foundation of the world:

Inherit kingdom from the foundation of the world

Luke 12:32 Fear not, little flock; for it is your Father's good pleasure to give you the kingdom.

Fathers pleasure to give us a kingdom (Margin: see the references to the OT)

Jam 2:5 Hearken, my beloved brethren, Hath not God chosen the poor of this world rich in faith, and
heirs of the kingdom which he hath promised to them that love him?

http://breachrepairers.webs.com/ 119
Heirs of the kingdom

Note: Jam 1:12 says we receive a crown of life

Ps 103:19 The LORD hath prepared his throne in the heavens; and his kingdom ruleth over all.

The Lord prepared His throne Note: Ps 102:19 the throne is in the sanctuary. Ps 11:4 the Lord’s throne is in
heaven.

Matt 24:47 Verily I say unto you, That he shall make him ruler over all his goods.

We will be made rulers of Christ’s goods

Ps 45:6 Thy throne, O God, is for ever and ever: the sceptre of thy kingdom is a right sceptre.

Throne of God is forever. This goes with Heb 1:8-9

Ps 93:2 Throne of old…

The Setting of Matthew


The events in the book of Matthew take place almost entirely
within the vicinity of Palestine, an area extending roughly from
Caesarea Philippi in the north to Beersheba in the south. During
this time it was ruled by the Roman Empire. The opening chapters
describe events surrounding Jesus' birth in Judea, where Herod had
been appointed king by the Romans. The closing chapters end with
Jesus' death, resurrection, and ascension during the rule of Pontius
Pilate and the tetrarchs Antipas and Philip.

Outline
1. The Arrival in History of Jesus the Messiah (1:1–2:23)
a. The genealogy of Jesus the Messiah (1:1–17)
b. The angelic announcement of the conception of Jesus the Messiah (1:18–25)
c. Magi report the star-sign of the birth of “the King of the Jews” (2:1–12)
d. OT prophecies are fulfilled in Jesus the Messiah (2:13–23)
2. John the Baptist Prepares for the Appearance of the Messianic Kingdom (3:1–17)
3. Jesus the Messiah Begins to Advance the Messianic Kingdom (4:1–25)
a. Temptations of the Messiah (4:1–11)
b. Jesus the Messiah begins his Galilean ministry (4:12–25)
4. The Authoritative Message of the Messiah: Kingdom Life for His Disciples (5:1–7:29) (First
Discourse)
a. Setting, Beatitudes, and witness of the kingdom of heaven (5:1–16)
b. The messianic kingdom in relation to the law (5:17–48)
c. The development of kingdom life in the real world (6:1–7:12)
d. Warning! With Jesus or against him? (7:13–29)
5. The Authoritative Power of the Messiah: Kingdom Power Demonstrated (8:1–9:38)
a. Healings, discipleship, and overpowering Satan's strongholds (8:1–9:8)
b. Unexpected discipleship, miracles, and workers (9:9–38)
6. The Authoritative Mission of the Messiah's Messengers (10:1–42) (Second Discourse)

http://breachrepairers.webs.com/ 120
a. Commissioning and instructions for the short-term mission to Israel (10:1–15)
b. Instructions for the long-term mission to the world (10:16–23)
c. Characteristics of missionary disciples (10:24–42)
7. Opposition to the Messiah Emerges (11:1–12:50)
a. Jesus, John the Baptist, and ministry in Galilee (11:1–30)
b. Confrontations with the Pharisees (12:1–45)
c. Jesus' disciples are his true family (12:46–50)
8. Mysteries of the Messianic Kingdom Revealed in Parables (13:1–53) (Third Discourse)
a. The opening of the Parabolic Discourse (13:1–23)
b. Further parables told to the crowds (13:24–35)
c. Explanations and parables told to the disciples (13:36–53)
9. The Identity of the Messiah Revealed (13:54–16:20)
a. Prophet(s) without honor (13:54–14:12)
b. Compassionate healer and supplier for Israel (14:13–21)
c. The Son of God worshiped (14:22–36)
d. Teacher of the Word of God and compassionate healer (15:1–39)
e. Peter confesses Jesus as the Christ, the Son of the living God (16:1–20)
10. The Suffering of the Messiah Revealed (16:21–17:27)
a. The suffering sacrifice (16:21–28)
b. The beloved, transfigured Son (17:1–13)
c. Sons of the kingdom (17:14–27)
11. The Community of the Messiah Revealed (18:1–20:34) (Fourth Discourse)
a. Characteristics of life in the kingdom community (18:1–35)
b. Valuing the kingdom community (19:1–20:34)
12. The Messiah Asserts His Authority over Jerusalem (21:1–23:39)
a. The triumphal entry into Jerusalem: Jesus' authority as Messiah (21:1–11)
b. The temple actions: Jesus' pronouncement on the temple establishment (21:12–17)
c. Cursing the fig tree: Jesus' judgment of the nation (21:18–22)
d. Controversies in the temple court over Jesus' authority (21:23–22:46)
e. Warnings against the teachers of the law and the Pharisees (23:1–12)
f. Woes of judgment against the teachers of the law and the Pharisees (23:13–36)
g. Lament over Jerusalem (23:37–39)
13. The Delay, Return, and Judgment of Messiah (24:1–25:46) (Fifth [Olivet] Discourse)
a. The beginning of birth pains (24:1–14)
b. “Great tribulation” and the coming of the Son of Man (24:15–31)
c. The nearness and time of Jesus' coming (24:32–41)
d. Parabolic exhortations to watch and be prepared for the coming of the Son of Man
(24:42–25:30)
e. Judgment at the end (25:31–46)
14. The Crucified Messiah (26:1–27:66)
a. Plot, anointing, and betrayal to the religious leaders (26:1–16)
b. The Passover and the Lord's Supper (26:17–35)
c. Gethsemane: Jesus' agonizing prayers (26:36–46)
d. Jesus arrested (26:47–56)
e. The Jewish trial of Jesus (26:57–27:10)
f. The Roman trial of Jesus (27:11–26)
g. Jesus the Messiah crucified (27:27–44)
h. The death of Jesus the Messiah (27:45–50)
i. Testimonies, women followers, and burial (27:51–66)
15. The Resurrection and Commission of the Messiah (28:1–20)

http://breachrepairers.webs.com/ 121
a. An empty tomb and the risen Jesus (28:1–10)
b. The conspiracy to deny the truth of Jesus' resurrection (28:11–15)
c. The risen Jesus' Great Commission (28:16–20)

http://breachrepairers.webs.com/ 122
Chapter 1 – Genealogy of Jesus
Matt 1:1 The book of the generation of Jesus Christ, the son of David, the son of Abraham.

“Generation” – GMK 1078 = Genesis; translates as “generation” once, “natural” once, and “nature”
once.

Matt 1:17 So all the generations from Abraham to David are fourteen generations; and from David
until the carrying away into Babylon are fourteen generations; and from the carrying away into
Babylon unto Christ are fourteen generations.

The word means a succession or series of persons from the same stock.

Matt 24:34 Verily I say unto you, This generation shall not pass, till all these things be fulfilled.

“This generation” = the persons then living contemporary with Christ. The Hebrews seem to have
reckoned time by the generation. In the time of Abraham a generation was an hundred years, thus:
Gen. 15:16, “In the fourth generation” = in four hundred years (comp. verse 13 and Ex. 12:40). In
Deut. 1:35 and 2:14 a generation is a period of thirty-eight years

Isa 53:8 Who shall declare his generation?

His manner of life who shall declare? or rather = His race, posterity, shall be so numerous that no
one shall be able to declare it. 2

Conclusion = Generation represents the lineage of Christ as well as those living contemporary with
Christ. It may also allude to the spiritual descendants of Christ. (More study)

Matt 3:7 But when he saw many of the Pharisees and Sadducees come to his baptism, he said unto
them, O generation of vipers, who hath warned you to flee from the wrath to come?

Matt 11:16-17 [16] But whereunto shall I liken this generation? It is like unto children sitting in the
markets, and calling unto their fellows, [17] And saying, We have piped unto you, and ye have not
danced; we have mourned unto you, and ye have not lamented

Matt 12:34 O generation of vipers, how can ye, being evil, speak good things?

Matt 12:39 But he answered and said unto them, An evil and adulterous generation seeketh after a
sign

Matt 12:41 The men of Nineveh shall rise in judgment with this generation, and shall condemn it:
because they repented at the preaching of Jonas; and, behold, a greater than Jonas [is] here.

Matt 12:42 The queen of the south shall rise up in the judgment with this generation, and shall
condemn it: for she came from the uttermost parts of the earth to hear the wisdom of Solomon; and,
behold, a greater than Solomon [is] here.

Matt 12:45 Then goeth he, and taketh with himself seven other spirits more wicked than himself,
and they enter in and dwell there: and the last [state] of that man is worse than the first. Even so shall

http://breachrepairers.webs.com/ 123
it be also unto this wicked generation.

Matt 16:4 A wicked and adulterous generation seeketh after a sign; and there shall no sign be given
unto it, but the sign of the prophet Jonas

Matt 17:17 Then Jesus answered and said, O faithless and perverse generation, how long shall I be
with you? how long shall I suffer you? bring him hither to me

Matt 23:33 [Ye] serpents, [ye] generation of vipers, how can ye escape the damnation of hell?

Matt 23:35-36 [35] That upon you may come all the righteous blood shed upon the earth, from the
blood of righteous Abel unto the blood of Zacharias son of Barachias, whom ye slew between the
temple and the altar Verily I say unto you, All these things shall come upon this generation.

Matt 24:34 Verily I say unto you, This generation shall not pass, till all these things be fulfilled

1:1 The book of the generation of Jesus Christ, the son of David, the son of Abraham.

The Genealogy of Jesus


Matthew 1
 He was a Jew and was trying to prove that Jesus was the Messiah of the Genealogy only goes
up to Abraham
 This is the genealogy of Joseph
Luke 3
 This is the genealogy of Mary
Jesus was the partaker of the law of heredity – He came like as we were.

1:2 Abraham begat Isaac; and Isaac begat Jacob; and Jacob begat Judas and his brethren;

1:3 And Judas begat Phares and Zara of Thamar; and Phares begat Esrom; and Esrom begat Aram;

1:4 And Aram begat Aminadab; and Aminadab begat Naasson; and Naasson begat Salmon;

1:5 And Salmon begat Booz of Rachab; and Booz begat Obed of Ruth; and Obed begat Jesse;

1:6 And Jesse begat David the king; and David the king begat Solomon of her that had been the wife of
Urias;

1:7 And Solomon begat Roboam; and Roboam begat Abia; and Abia begat Asa;

1:8 And Asa begat Josaphat; and Josaphat begat Joram; and Joram begat Ozias;

1:9 And Ozias begat Joatham; and Joatham begat Achaz; and Achaz begat Ezekias;

1:10 And Ezekias begat Manasses; and Manasses begat Amon; and Amon begat Josias;

1:11 And Josias begat Jechonias and his brethren, about the time they were carried away to Babylon:

1:12 And after they were brought to Babylon, Jechonias begat Salathiel; and Salathiel begat Zorobabel;

1:13 And Zorobabel begat Abiud; and Abiud begat Eliakim; and Eliakim begat Azor;

http://breachrepairers.webs.com/ 124
1:14 And Azor begat Sadoc; and Sadoc begat Achim; and Achim begat Eliud;

1:15 And Eliud begat Eleazar; and Eleazar begat Matthan; and Matthan begat Jacob;

1:16 And Jacob begat Joseph the husband of Mary, of whom was born Jesus, who is called Christ.

1:17 So all the generations from Abraham to David are fourteen generations; and from David until the
carrying away into Babylon are fourteen generations; and from the carrying away into Babylon unto
Christ are fourteen generations.

1:18 ¶ Now the birth of Jesus Christ was on this wise: When as his mother Mary was espoused to Joseph,
before they came together, she was found with child of the Holy Ghost.

1:19 Then Joseph her husband, being a just man, and not willing to make her a publick example, was
minded to put her away privily.

1:20 But while he thought on these things, behold, the angel of the Lord appeared unto him in a dream,
saying, Joseph, thou son of David, fear not to take unto thee Mary thy wife: for that which is conceived in
her is of the Holy Ghost.

1:21 And she shall bring forth a son, and thou shalt call his name JESUS: for he shall save his people from
their sins.

1:22 Now all this was done, that it might be fulfilled which was spoken of the Lord by the prophet, saying,

1:23 Behold, a virgin shall be with child, and shall bring forth a son, and they shall call his name
Emmanuel, which being interpreted is, God with us.

1:24 Then Joseph being raised from sleep did as the angel of the Lord had bidden him, and took unto him
his wife:

1:25 And knew her not till she had brought forth her firstborn son: and he called his name JESUS.

http://breachrepairers.webs.com/ 125
Chapter 2
2:1 Now when Jesus was born in Bethlehem of Judaea in the days of Herod the king, behold, there came
wise men from the east to Jerusalem,

The Life of Jesus

Matt 2:1-2 [1] Now when Jesus was born in Bethlehem of Judaea in the days of Herod the king,
behold, there came wise men from the east to Jerusalem, [2] Saying, Where is he that is born King of
the Jews? for we have seen his star in the east, and are come to worship him.

Matt 2:15 And was there until the death of Herod: that it might be fulfilled which was spoken of the
Lord by the prophet, saying, Out of Egypt have I called my son.

Hosea 11:1 is the text quoted but in Hosea it is referring to the children of Israel. The similarities is
after they were baptized, one went into wilderness for 40 years and Jesus went into 40 days into
wilderness. Jesus was the one that Hosea was referring to. Israel is a type of Jesus.

Luke 2:4 And Joseph also went up from Galilee, out of the city of Nazareth, into Judæa, unto the city
of David, which is called Bethlehem; (because he was of the house and lineage of David:)

It was prophesized that Jesus was to be born in Bethlehem. The reason why Joseph went back to
Bethlehem because everyone was to be taxed.

Luke 2:25-32, 36-38, 40 [25] And, behold, there was a man in Jerusalem, whose name was Simeon;
and the same man was just and devout, waiting for the consolation of Israel: and the Holy Ghost was
upon him. [26] And it was revealed unto him by the Holy Ghost, that he should not see death, before
he had seen the Lord's Christ. [27] And he came by the Spirit into the temple: and when the parents
brought in the child Jesus, to do for him after the custom of the law, [28] Then took he him up in his
arms, and blessed God, and said, [29] Lord, now lettest thou thy servant depart in peace, according to
thy word: [30] For mine eyes have seen thy salvation, [31] Which thou hast prepared before the face
of all people; [32] A light to lighten the Gentiles, and the glory of thy people Israel. [36] And there was
one Anna, a prophetess, the daughter of Phanuel, of the tribe of Aser: she was of a great age, and had
lived with an husband seven years from her virginity; [37] And she was a widow of about fourscore
and four years, which departed not from the temple, but served God with fastings and prayers night
and day. [38] And she coming in that instant gave thanks likewise unto the Lord, and spake of him to
all them that looked for redemption in Jerusalem. [40] And the child grew, and waxed strong in spirit,
filled with wisdom: and the grace of God was upon him.

Simeon was indicating to the parents that God had given to them the Messiah. This is important
because EGW mentioned that Mary wandered sometimes if Jesus was the Messiah or not – but she
always remember the words that had been spoken to her.

DA 73 He expected much; therefore He attempted much.

DA 74 Yet Jesus shunned display. During all the years of His stay in Nazareth, He made no exhibition
of His miraculous power. He sought no high position and assumed no titles. His quiet and simple life,
and even the silence of the Scriptures concerning His early years, teach an important lesson. The
more quiet and simple the life of the child,--the more free from artificial excitement, and the more in
harmony with nature,--the more favorable is it to physical and mental vigor and to spiritual strength.

DA 83 If Joseph and Mary had stayed their minds upon God by meditation and prayer, they would
have realized the sacredness of their trust, and would not have lost sight of Jesus. By one day's

http://breachrepairers.webs.com/ 126
neglect they lost the Saviour; but it cost them three days of anxious search to find Him.

How do we know that John the Baptist ministered for 6 months?

Luke 1:36 And, behold, thy cousin Elisabeth, she hath also conceived a son in her old age: and this is
the sixth month with her, who was called barren.

We can see here that Jesus and John the Baptist were cousins, we know that they were six months
apart.

Luke 3:1-2 [1] Now in the fifteenth year of the reign of Tiberius Cæsar, Pontius Pilate being governor
of Judæa, and Herod being tetrarch of Galilee, and his brother Philip tetrarch of Ituraea and of the
region of Trachonitis, and Lysanias the tetrarch of Abilene, [2] Annas and Caiaphas being the high
priests, the word of God came unto John the son of Zacharias in the wilderness.

John the Baptist was in the wilderness until the time came. The word of God came to him in the 15th
year of the reign of Tiberius Caesar

2:2 Saying, Where is he that is born King of the Jews? for we have seen his star in the east, and are come
to worship him.

2:3 When Herod the king had heard these things, he was troubled, and all Jerusalem with him.

2:4 And when he had gathered all the chief priests and scribes of the people together, he demanded of
them where Christ should be born.

2:5 And they said unto him, In Bethlehem of Judaea: for thus it is written by the prophet,

2:6 And thou Bethlehem, in the land of Juda, art not the least among the princes of Juda: for out of thee
shall come a Governor, that shall rule my people Israel.

2:7 Then Herod, when he had privily called the wise men, enquired of them diligently what time the star
appeared.

2:8 And he sent them to Bethlehem, and said, Go and search diligently for the young child; and when ye
have found him, bring me word again, that I may come and worship him also.

2:9 When they had heard the king, they departed; and, lo, the star, which they saw in the east, went
before them, till it came and stood over where the young child was.

2:10 When they saw the star, they rejoiced with exceeding great joy.

2:11 ¶ And when they were come into the house, they saw the young child with Mary his mother, and fell
down, and worshipped him: and when they had opened their treasures, they presented unto him gifts;
gold, and frankincense, and myrrh.

2:12 And being warned of God in a dream that they should not return to Herod, they departed into their
own country another way.

2:13 And when they were departed, behold, the angel of the Lord appeareth to Joseph in a dream, saying,
Arise, and take the young child and his mother, and flee into Egypt, and be thou there until I bring thee
word: for Herod will seek the young child to destroy him.

2:14 When he arose, he took the young child and his mother by night, and departed into Egypt:

http://breachrepairers.webs.com/ 127
2:15 And was there until the death of Herod: that it might be fulfilled which was spoken of the Lord by
the prophet, saying, Out of Egypt have I called my son.

2:16 ¶ Then Herod, when he saw that he was mocked of the wise men, was exceeding wroth, and sent
forth, and slew all the children that were in Bethlehem, and in all the coasts thereof, from two years old
and under, according to the time which he had diligently enquired of the wise men.

2:17 Then was fulfilled that which was spoken by Jeremy the prophet, saying,

2:18 In Rama was there a voice heard, lamentation, and weeping, and great mourning, Rachel weeping
for her children, and would not be comforted, because they are not.

2:19 ¶ But when Herod was dead, behold, an angel of the Lord appeareth in a dream to Joseph in Egypt,

2:20 Saying, Arise, and take the young child and his mother, and go into the land of Israel: for they are
dead which sought the young child's life.

2:21 And he arose, and took the young child and his mother, and came into the land of Israel.

2:22 But when he heard that Archelaus did reign in Judaea in the room of his father Herod, he was afraid
to go thither: notwithstanding, being warned of God in a dream, he turned aside into the parts of Galilee:

2:23 And he came and dwelt in a city called Nazareth: that it might be fulfilled which was spoken by the
prophets, He shall be called a Nazarene.

http://breachrepairers.webs.com/ 128
Chapter 3 – Anointing | Baptism of Christ
3:1 In those days came John the Baptist, preaching in the wilderness of Judaea,

3:2 And saying, Repent ye: for the kingdom of heaven is at hand.

3:3 For this is he that was spoken of by the prophet Esaias, saying, The voice of one crying in the
wilderness, Prepare ye the way of the Lord, make his paths straight.

3:4 And the same John had his raiment of camel's hair, and a leathern girdle about his loins; and his
meat was locusts and wild honey.

3:5 Then went out to him Jerusalem, and all Judaea, and all the region round about Jordan,

3:6 And were baptized of him in Jordan, confessing their sins.

3:7 But when he saw many of the Pharisees and Sadducees come to his baptism, he said unto them, O
generation of vipers, who hath warned you to flee from the wrath to come?

Two generations in the book of Matthew: The generation of Christ and the generation of vipers,
Another word for viper is serpent. The generation of serpents.

Gen 3:15 And I will put enmity between thee and the woman, and between thy seed and her seed

Here in Matthew we see the descendents of the woman and the descendents of the serpent. Both
are the descendents of Abraham, but through Christ the descendents are not limited to the blood
line. (PO Note: Perhaps the Pharisees and Sadducees came from the tribe of Dan.)

Gen 49:17 Dan shall be a serpent by the way, an adder in the path, that biteth the horse heels, so that
his rider shall fall backward.

The Pharisees and Sadducees were used by Satan

Rev 12:9 And the great dragon was cast out, that old serpent, called the Devil, and Satan, which
deceiveth the whole world…

Satan deceived the land of Palestine through the Pharisees and Sadducees. The Jews were suppose
to be the remnant. Matt 22, but they were deceived.

3:7 But when he saw many of the Pharisees and Sadducees come to his baptism, he said unto them, O
generation of vipers, who hath warned you to flee from the wrath to come?

3:8 Bring forth therefore fruits meet for repentance:

3:9 And think not to say within yourselves, We have Abraham to our father: for I say unto you, that God is
able of these stones to raise up children unto Abraham.

3:10 And now also the axe is laid unto the root of the trees: therefore every tree which bringeth not forth
good fruit is hewn down, and cast into the fire.

3:11 I indeed baptize you with water unto repentance: but he that cometh after me is mightier than I,
whose shoes I am not worthy to bear: he shall baptize you with the Holy Ghost, and with fire:

http://breachrepairers.webs.com/ 129
“he shall baptize you with the Holy Ghost, and with fire” – This was fulfilled on the Day of
Pentecost. John describes four phases of Christ’s work.
 His baptism
 “baptize with…fire” – Resurrection
 “gather wheat into the garner” – Second coming. This also lays the foundation for Matt 13,
the wheat and the tares
 “burn up the chaff with unquenchable fire” – third coming. Note that this verse describes
the doctrine of Hell’s fire. It says ‘burn up’ meaning it will not longer be. It will be
consumed. It does not say burn forever with unquenchable fire.

3:12 Whose fan is in his hand, and he will throughly purge his floor, and gather his wheat into the
garner; but he will burn up the chaff with unquenchable fire.

3:13 Then cometh Jesus from Galilee to Jordan unto John, to be baptized of him.

3:14 But John forbad him, saying, I have need to be baptized of thee, and comest thou to me?

3:15 And Jesus answering said unto him, Suffer it to be so now: for thus it becometh us to fulfil all
righteousness. Then he suffered him.

3:16 And Jesus, when he was baptized, went up straightway out of the water: and, lo, the heavens were
opened unto him, and he saw the Spirit of God descending like a dove, and lighting upon him:

3:17 And lo a voice from heaven, saying, This is my beloved Son, in whom I am well pleased.

http://breachrepairers.webs.com/ 130
Chapter 4 – Temptations of Christ
4:1 Then was Jesus led up of the Spirit into the wilderness to be tempted of the devil.

4 :2 And when he had fasted forty days and forty nights, he was afterward an hungred.

How long had Jesus been fasting for before the devil came and tempted Him? 40 days.

How did he feel after he had been fasting that long? He was hungry. How would you feel if you
hadn’t eaten for a long time?

4:3 And when the tempter came to him, he said, If thou be the Son of God, command that these stones be
made bread.

The Temptation of Jesus – Why did the Spirit drive Him into the wilderness?

DA 114 He went to the wilderness to be alone, to contemplate His mission and work. By fasting and
prayer He was to brace Himself for the bloodstained path He must travel. But Satan knew that the
Saviour had gone into the wilderness, and he thought this the best time to approach Him –

Mark 1:12-13 [12] And immediately the Spirit driveth him into the wilderness. [13] And he was
there in the wilderness forty days, tempted of Satan; and was with the wild beasts; and the angels
ministered unto him.

It is covered very quickly. In Luke 4 and in Matthew 4.

The First temptation

Matt 4:2-3 [2] And when he had fasted forty days and forty nights, he was afterward an hungred. [3]
And when the tempter came to him, he said, If thou be the Son of God, command that these stones be
made bread.

Satan came to Jesus as if he was a messenger from Heaven

DA 118 There came to the Saviour, as if in answer to His prayers, one in the guise of an angel from
heaven. He claimed to have a commission from God to declare that Christ's fast was at an end.
Though he appears as an angel of light, these first words betray his character. "If Thou be the Son of
God."

There is no evidence in the scriptures that this happened.

What was the first temptation that the devil tempted Jesus on? It was on appetite. Do you think
that this would have been something difficult for Jesus? Of course. Remember, how long had he
been fasting?

Matt 4:3-4 [3] And when the tempter came to him, he said, If thou be the Son of God, command that
these stones be made bread. [4] But he answered and said, It is written, Man shall not live by bread
alone, but by every word that proceedeth out of the mouth of God.

Was there anything wrong with turning stones into bread? Why would that have been “bad”
for Jesus to do? 1 Cor. 15:22 – Christ had to pick up where Adam failed. Gen. 3:1-6 – where was it
that Adam and Eve failed? It was first upon the point of appetite. So Jesus had to first overcome on

http://breachrepairers.webs.com/ 131
appetite because He had to pick up where humanity fell. For Jesus to use His power to turn the
stone into bread would be to use His own power for selfish purposes. That was not the reason why
He came. He HAD to overcome on this point. Ever since the fall of man, the greatest weakness has
always been on the point of appetite. Many sins can be lead back to the sin of intemperance.

1 Cor 15:22 For as in Adam all die, even so in Christ shall all be made alive.

Gen 3:1-6 [1] Now the serpent was more subtil than any beast of the field which the LORD God had
made. And he said unto the woman, Yea, hath God said, Ye shall not eat of every tree of the garden?
[2] And the woman said unto the serpent, We may eat of the fruit of the trees of the garden: [3] But of
the fruit of the tree which is in the midst of the garden, God hath said, Ye shall not eat of it, neither
shall ye touch it, lest ye die. [4] And the serpent said unto the woman, Ye shall not surely die: [5] For
God doth know that in the day ye eat thereof, then your eyes shall be opened, and ye shall be as gods,
knowing good and evil. [6] And when the woman saw that the tree was good for food, and that it was
pleasant to the eyes, and a tree to be desired to make one wise, she took of the fruit thereof, and did
eat, and gave also unto her husband with her; and he did eat.

What did the phrase “If thou be the Son of God” insinuate?

“If Thou be the Son of God” – Answer: It insinuated distrust in God and asking for proof that He
was the Son of God. The “if” showed that if Jesus had listened to Satan, it would have proved that He
didn’t believe that voice that He had heard when Jesus was baptized when God the Father said form
heaven “This is my beloved Son” (Matt. 3:17). Satan was insinuating that Jesus may not be the Son
of God – it was because of the way that he looked:

DA 118 Weak and emaciated from hunger, worn and haggard with mental agony, "His visage was so
marred more than any man, and His form more than the sons of men." Isa. 52:14. Now was Satan's
opportunity. Now he supposed that he could overcome Christ.

DA 119 When Satan and the Son of God first met in conflict, Christ was the commander of the
heavenly hosts; and Satan, the leader of revolt in heaven, was cast out. Now their condition is
apparently reversed, and Satan makes the most of his supposed advantage. One of the most powerful
of the angels, he says, has been banished from heaven. The appearance of Jesus indicates that He is
that fallen angel, forsaken by God, and deserted by man. A divine being would be able to sustain his
claim by working a miracle; "if Thou be the Son of God, command this stone that it be made bread."
Such an act of creative power, urges the tempter, would be conclusive evidence of divinity. It would
bring the controversy to an end.

The first temptation deals with doubt. Not only was it to do with appetite, but it was to also prove
that He was the Son of God.

1 John 2:16 For all that is in the world, the lust of the flesh, and the lust of the eyes, and the pride of
life, is not of the Father, but is of the world.

All the sin that is in the world:


1. Lust of the flesh
2. Lust of the eyes
3. Pride of life

DA 120 It was in the time of greatest weakness that Christ was assailed by the fiercest temptations.

http://breachrepairers.webs.com/ 132
Satan tries to cause us to sin at our weakest point. Jesus overcame by saying – “It is written”. Jesus
showed us how we can overcome by quoting the Bible.

DA 121 Perhaps it appears that obedience to some plain requirement of God will cut off his means of
support. Satan would make him believe that he must sacrifice his conscientious convictions.

It is so easy to think that God will not supply our needs if we follow Him 100% of the time.

DA 122 Of all the lessons to be learned from our Lord's first great temptation none is more
important than that bearing upon the control of the appetites and passions. In all ages, temptations
appealing to the physical nature have been most effectual in corrupting and degrading mankind.
Through intemperance, Satan works to destroy the mental and moral powers that God gave to man
as a priceless endowment.

Matt 4:6 And saith unto him, If thou be the Son of God, cast thyself down: for it is written, He shall
give his angels charge concerning thee: and in their hands they shall bear thee up, lest at any time
thou dash thy foot against a stone.

Satan didn’t cast Him down but asked Him to cast Himself down. What is the difference between the
first and second temptation? The first was to show His power. The second was to show if God was
really with Him or not. Satan can never force us to sin. It is always our own choice.

4:4 But he answered and said, It is written, Man shall not live by bread alone, but by every word that
proceedeth out of the mouth of God.

4:5 Then the devil taketh him up into the holy city, and setteth him on a pinnacle of the temple,

The Second Temptation – What was the second temptation that the devil tempted Jesus on?

Matt 4:5-6 [5] Then the devil taketh him up into the holy city, and setteth him on a pinnacle of the
temple, [6] And saith unto him, If thou be the Son of God, cast thyself down: for it is written, He shall
give his angels charge concerning thee: and in their hands they shall bear thee up, lest at any time
thou dash thy foot against a stone.

Stimulate discussion by asking why the devil asked Jesus to jump off? What does that represent?
Surely there was something bad behind the devil asking Jesus to do that or else he wouldn’t have
tried to tempt him. This temptation represented presumption. How did this temptation represent
presumption? The devil quoted scripture but we shouldn’t quote scripture for our own benefit. The
devil tried to get Jesus to take action on a scripture that the devil quoted.

What was different about this temptation as compared to the other two that Satan
presented? The devil quoted scripture. Compare the scripture that Satan quoted to what is actually
written in the Bible. What is the difference between the two and discuss the importance of what
was omitted. What important lessons can we learn from this?

Psa 91:11-12 [11] For he shall give his angels charge over thee, to keep thee in all thy ways. [12]
They shall bear thee up in their hands, lest thou dash thy foot against a stone.

He left out “to keep thee in all thy ways” which meant all the ways of God’s choosing. Why was it so
important? Because it omitted the need to follow the way of God. What important lesson can we
learn from this? If we read scripture, we must make sure we don’t misquote it otherwise we may
act presumptuously on something that God didn’t promise. That is why it is so important to know

http://breachrepairers.webs.com/ 133
how to study the Bible properly. Eve misquoted God in the beginning and she believed a lie because
of that (Gen. 3:3). Obedience to the word of God safe guards us from Satan. This is true faith. Faith in
the word of God brings forth fruit in obedience. Whereas, presumption claims the promise to
excuse transgression.

Matt 4:5-6 [5] Then the devil taketh him up into the holy city, and setteth him on a pinnacle of the
temple, [6] And saith unto him, If thou be the Son of God, cast thyself down: for it is written, He shall
give his angels charge concerning thee: and in their hands they shall bear thee up, lest at any time
thou dash thy foot against a stone.

Con 47 But Satan was not willing to cease his efforts until he had tried every means to obtain victory
over the world's Redeemer. He knew that with himself all was at stake, whether he or Christ should
be victor in the contest. And in order to awe Christ with his superior strength he carried Him to
Jerusalem and set Him on a pinnacle of the Temple, and continued to beset Him with temptations.

Satan quotes from Psalms 91 but leaves out “to keep thee in all thy ways” – in God’s way. God only
gives us protection when we are obedient to His ways. Angels do not attend the theatres with us. So
can we expect God’s protection over us if we attend theatres? Just like driving. If we pray for
protection on the road and then speed, that is presumption. God does also expect us to obey the
speed limit on the roads. Needlessly showing that God is with us, the love of display that would lead
to presumption, would not accomplish anything. Satan was trying to tell Jesus to show that God was
with Him. Jesus quoted from Deut 6:16

Ex 17:2, 7 [2] Wherefore the people did chide with Moses, and said, Give us water that we may drink.
And Moses said unto them, Why chide ye with me? wherefore do ye tempt the LORD? [7] And he
called the name of the place Massah, and Meribah, because of the chiding of the children of Israel, and
because they tempted the LORD, saying, Is the LORD among us, or not?

Jesus quoted the text in the right context because they were asking if God was with them or not. We
need to claim the promises that are relevant for that situation.

4:6 And saith unto him, If thou be the Son of God, cast thyself down: for it is written, He shall give his
angels charge concerning thee: and in their hands they shall bear thee up, lest at any time thou dash thy
foot against a stone.

4:7 Jesus said unto him, It is written again, Thou shalt not tempt the Lord thy God.

4:8 Again, the devil taketh him up into an exceeding high mountain, and sheweth him all the kingdoms
of the world, and the glory of them;

What was the third temptation that the devil tempted Jesus on? He was trying to tempt Jesus to
take the easy way out. He was trying to tempt Jesus with power, and a path less sacrificing to get the
whole world back. He was trying to get Jesus to worship him – something that he had been trying to
do from the very beginning before he was thrown out of heaven

Matt 4:8-9 [8] Again, the devil taketh him up into an exceeding high mountain, and sheweth him all
the kingdoms of the world, and the glory of them; [9] And saith unto him, All these things will I give
thee, if thou wilt fall down and worship me.

How did this temptation differ from the other two? What lesson can we learn from its
difference? Satan came outright in trying to get Jesus to disobey the commandments of God. Every
time we resist temptation, it comes out stronger and more bold in its demands to go against God’s

http://breachrepairers.webs.com/ 134
commandments

What is our only safeguard when Satan comes to tempt us? How did Jesus respond each
time? We must learn to use the words “It is written.” How does this become practical for us to
apply today? We need to make sure we don’t misquote the Bible, otherwise we deceive ourselves
thinking that God had said that when He didn’t and hence we believe a lie. We should learn to
memorize scripture. Jesus didn’t have the Bible with Him on the mount of temptation. He had it
memorized and could say “It is written.” The Bible is our safe guard against temptation. If we want
victory in life, we need to behold the Bible so much that we can say “It is written.” Suggest to them
to maybe memorize a book of the Bible together. Start with something small – Philemon or Jude.

4:9 And saith unto him, All these things will I give thee, if thou wilt fall down and worship me.

4:10 Then saith Jesus unto him, Get thee hence, Satan: for it is written, Thou shalt worship the Lord thy
God, and him only shalt thou serve.

4:11 Then the devil leaveth him, and, behold, angels came and ministered unto him.

4:12 Now when Jesus had heard that John was cast into prison, he departed into Galilee;

4:13 And leaving Nazareth, he came and dwelt in Capernaum, which is upon the sea coast, in the borders
of Zabulon and Nephthalim:

4:14 That it might be fulfilled which was spoken by Esaias the prophet, saying,

4:15 The land of Zabulon, and the land of Nephthalim, by the way of the sea, beyond Jordan, Galilee of the
Gentiles;

4:16 The people which sat in darkness saw great light; and to them which sat in the region and shadow
of death light is sprung up.

4:17 From that time Jesus began to preach, and to say, Repent: for the kingdom of heaven is at hand.

4:18 And Jesus, walking by the sea of Galilee, saw two brethren, Simon called Peter, and Andrew his
brother, casting a net into the sea: for they were fishers.

Fishers of Men – Aim = A purpose or intention toward which one's efforts are directed. Motto = A
maxim adopted as a guide to one's conduct.Class Aim = To render Efficient service as Fishers of
men; Motto = Together with God

Calling of the fisher men

Matt 4:18-22 [18] ¶ And Jesus, walking by the sea of Galilee, saw two brethren, Simon called Peter,
and Andrew his brother, casting a net into the sea: for they were fishers. [19] And he saith unto them,
Follow me, and I will make you fishers of men. [20] And they straightway left their nets, and followed
him. [21] And going on from thence, he saw other two brethren, James the son of Zebedee, and John
his brother, in a ship with Zebedee their father, mending their nets; and he called them. [22] And they
immediately left the ship and their father, and followed him.

I will make you fishers of men (Why did He choose them?). App: You must be called, you can’t make
yourself a fisher men. Note: Fish symbolize men; Net symbolizes the tool necessary to gather the
men.

http://breachrepairers.webs.com/ 135
Ezek 22:23, 26, 30 [23] ¶ And the word of the LORD came unto me, saying, [26] Her priests have
violated my law, and have profaned mine holy things: they have put no difference between the holy
and profane, neither have they shewed difference between the unclean and the clean, and have hid
their eyes from my sabbaths, and I am profaned among them. [30] And I sought for a man among
them, that should make up the hedge, and stand in the gap before me for the land, that I should not
destroy it: but I found none.

Called to stand in the gap. Note: Ps 106:23 – Moses stood in gap to prevent destruction of God’s
people.

Jud 3:9, 15 [9] And when the children of Israel cried unto the LORD, the LORD raised up a deliverer
to the children of Israel, who delivered them, even Othniel the son of Kenaz, Caleb's younger brother.
[15] But when the children of Israel cried unto the LORD, the LORD raised them up a deliverer, Ehud
the son of Gera, a Benjamite, a man lefthanded: and by him the children of Israel sent a present unto
Eglon the king of Moab.

Jud 4:3-4 [3] And the children of Israel cried unto the LORD: for he had nine hundred chariots of
iron; and twenty years he mightily oppressed the children of Israel. [4] ¶ And Deborah, a prophetess,
the wife of Lapidoth, she judged Israel at that time.

Called as an answer to praying saints. Training of the fisher men. 3 ½ years of training, Note: DA 30
“He ordained twelve”; 25 “The Call by the sea.” After 3 ½ years you have a Judas and a Peter;
Betrayer and one unconverted.

Commission of the fisher men

Matt 13:47-50 [47] ¶ Again, the kingdom of heaven is like unto a net, that was cast into the sea, and
gathered of every kind: [48] Which, when it was full, they drew to shore, and sat down, and gathered
the good into vessels, but cast the bad away. [49] So shall it be at the end of the world: the angels
shall come forth, and sever the wicked from among the just, [50] And shall cast them into the furnace
of fire: there shall be wailing and gnashing of teeth.

Commissioned to gather men for the judgment

PA: 1844-after millennium (Par Rev 14 3 angels, second coming, winepress after the millennium).
Note: Parables many times are prophetic in nature: Matt 22 Wedding Feast (Invest Judgment); Matt
21 Husbandman (crucifixion, destruction of Jerusalem); Matt 25 10 Virgins (Millerite Advent
Movement, Midnight cry) .

Appeal/Con

Matt 4:19 …Follow me, and I will make you fishers of men…

EGW Quotes

Passed by wise men DA 249-250 “But He passed by the wise men of His time, because they were so
self-confident that they could not sympathize with suffering humanity, and become colaborers with
the Man of Nazareth. In their bigotry they scorned to be taught by Christ…. Jesus chose unlearned
fishermen because they had not been schooled in the traditions and erroneous customs of their time.
They were men of native ability, and they were humble and teachable,--men whom He could educate
for His work.”

http://breachrepairers.webs.com/ 136
Priests appointed by Rome – DA 30 The Romans claimed the right of appointing and removing the
high priest, and the office was often secured by fraud, bribery, and even murder. Thus the priesthood
became more and more corrupt.

Disciples training -- DA 250 When the disciples came forth from the Saviour's training, they were no
longer ignorant and uncultured. They had become like Him in mind and character, and men took
knowledge of them that they had been with Jesus.

Most import work given -- Jesus had called His disciples that He might send them forth as His
witnesses, to declare to the world what they had seen and heard of Him. Their office was the most
important to which human beings had ever been called, and was second only to that of Christ
Himself. They were to be workers together with God for the saving of the world. DA 291

4:19 And he saith unto them, Follow me, and I will make you fishers of men.

4:20 And they straightway left their nets, and followed him.

4:21 And going on from thence, he saw other two brethren, James the son of Zebedee, and John his
brother, in a ship with Zebedee their father, mending their nets; and he called them.

4:22 And they immediately left the ship and their father, and followed him.

4:23 And Jesus went about all Galilee, teaching in their synagogues, and preaching the gospel of the
kingdom, and healing all manner of sickness and all manner of disease among the people.

4:24 And his fame went throughout all Syria: and they brought unto him all sick people that were taken
with divers diseases and torments, and those which were possessed with devils, and those which were
lunatick, and those that had the palsy; and he healed them.

4:25 And there followed him great multitudes of people from Galilee, and from Decapolis, and from
Jerusalem, and from Judaea, and from beyond Jordan.

http://breachrepairers.webs.com/ 137
Chapter 5 – Law of the Kingdom of God
The Beatitudes can be summed up simply has how to enter into the Kingdom of heaven. Jesus
concludes His sermon saying Matt 7:21 “Not every one that saith unto me, Lord, Lord, shall enter
into the kingdom of heaven; but he that doeth the will of my Father which is in heaven.” If you are a
partaker of the kingdom of grace, you will also be a part of the kingdom of glory. How do you enter
into heaven? By fulfilling or keeping the law and the prophets.

The rest of Matthew builds upon this sermon. The principles constantly unfold while at the same
time maintaining its overall theme of Jesus being the King, the Messiah, the seed of David.

Why did Jesus preach the sermon on the mount? People were looking to this man to set up this
kingdom.

DA 299 In the Sermon on the Mount He sought to undo the work that had been wrought by false
education, and to give His hearers a right conception of His kingdom and of His own character. –

Jesus was addressing a people that wanted an earthly kingdom. Jesus also deals with the law of God.
Verses 1 to 12 I believe that some how these verses lay the foundation for the sermon on the mount
& the rest of Matthew.

Matt 5:5 …blessed are the meek…

Chapter 6 explains how to be meek; humble acts of devotion in alms, praying, and fasting.

Matt 5:6 Blessed [are] they which do hunger and thirst after righteousness: for they shall be
filled.

Matt 5:20 except your righteousness shall exceed [the righteousness] of the scribes and Pharisees,
ye shall in no case enter into the kingdom of heaven.

Matt 6:33 But seek ye first the kingdom of God, and his righteousness; and all these things shall be
added unto you.

Matt 5:7 Blessed [are] the merciful: for they shall obtain mercy.

Matt 7:12 Therefore all things whatsoever ye would that men should do to you, do ye even so to
them: for this is the law and the prophets.

Matt 9:13 But go ye and learn what that meaneth, I will have mercy, and not sacrifice: for I am not
come to call the righteous, but sinners to repentance.

Matt 9:27 And when Jesus departed thence, two blind men followed him, crying, and saying, Thou
Son of David, have mercy on us.

Matt 12:7 But if ye had known what this meaneth, I will have mercy, and not sacrifice, ye would not
have condemned the guiltless.

Matt 15:22 And, behold, a woman of Canaan came out of the same coasts, and cried unto him, saying,
Have mercy on me, O Lord, thou Son of David; my daughter is grievously vexed with a devil.

Matt 17:15 Lord, have mercy on my son: for he is lunatick, and sore vexed: for ofttimes he falleth

http://breachrepairers.webs.com/ 138
into the fire, and oft into the water.

Matt 20:30-31 [30] And, behold, two blind men sitting by the way side, when they heard that Jesus
passed by, cried out, saying, Have mercy on us, O Lord, thou Son of David. [31] And the multitude
rebuked them, because they should hold their peace: but they cried the more, saying, Have mercy on
us, O Lord, thou Son of David.

Matt 23:23 Woe unto you, scribes and Pharisees, hypocrites! for ye pay tithe of mint and anise and
cummin, and have omitted the weightier matters of the law, judgment, mercy, and faith: these ought
ye to have done, and not to leave the other undone.

Matt 5:8 Blessed [are] the pure in heart: for they shall see God.

Matt 5:28 But I say unto you, That whosoever looketh on a woman to lust after her hath committed
adultery with her already in his heart.

Matt 6:21 For where your treasure is, there will your heart be also.

Matt 11:29 Take my yoke upon you, and learn of me; for I am meek and lowly in heart: and ye shall
find rest unto your souls.

Matt 5:9 Blessed [are] the peacemakers: for they shall be called the children of God.

Matt 5:44 But I say unto you, Love your enemies, bless them that curse you, do good to them that
hate you, and pray for them which despitefully use you, and persecute you. (See 5:38-45)

Reading
 The Sermon on the mount

5:1 And seeing the multitudes, he went up into a mountain: and when he was set, his disciples came unto
him:

The disciples were the closest to Him. The beatitudes are steps in conversion.

5:2 And he opened his mouth, and taught them, saying,

5:3 Blessed are the poor in spirit: for theirs is the kingdom of heaven.

Blessed means “happy.” So it is happy are they who are poor in spirit meaning in humility Jesus was
showing something far different than what they were expecting. There are 2 kinds of poverty.
Relative poverty, which is us relative to Bill Gates is poor then there is teal poverty. We are talking
about people who are lacking – they are “poor in spirit.” These people will inherit the kingdom.

“blessed are the poor in spirit” –The meaning of the word “Blessed” means happy or
extremely blessed. According to the Bible how can we be happy?

Deut 11:26-28 [26] ¶ Behold, I set before you this day a blessing and a curse; [27] A blessing, if ye
obey the commandments of the LORD your God, which I command you this day: [28] And a curse, if
ye will not obey the commandments of the LORD your God, but turn aside out of the way which I
command you this day, to go after other gods, which ye have not known.

Prov 29:18 Where there is no vision, the people perish: but he that keepeth the law, happy is he.

http://breachrepairers.webs.com/ 139
We can be happy if we obey the commandments of God. Many people feel miserable when they
think about keeping the commandments of God, but we are going to find that in the Bible true
happiness comes from following God’s words and God’s commandments. According to the first
beatitude, what is the condition of the people in this world today? The condition of the people
in this world today is that they are poor.

Rev 3:17 Because thou sayest, I am rich, and increased with goods, and have need of nothing; and
knowest not that thou art wretched, and miserable, and poor, and blind, and naked:

But not just that. They are poor but they don’t realize it. What does it mean to be poor in spirit?

Is 66:2 For all those things hath mine hand made, and all those things have been, saith the LORD: but
to this man will I look, even to him that is poor and of a contrite spirit, and trembleth at my word.

To be poor in spirit means to be contrite in spirit. What is the meaning of contrite? Get people to go
around and discuss. Meaning from the strongs concordance – feeling regret and sorrow for one’s
sins or offences; penitent. But remember, you cannot feel sorry for your sins or offences if you don’t
know what you’ve done. That is why the first step in being poor in spirit is to recognize that you are
poor in spirit. We are told that being poor in spirit is bad. But what is the purpose of the Holy Spirit?

Ezek 36:26-27 [26] A new heart also will I give you, and a new spirit will I put within you: and I will
take away the stony heart out of your flesh, and I will give you an heart of flesh. [27And I will put my
spirit within you, and cause you to walk in my statutes, and ye shall keep my judgments, and do them.

The Holy Spirit is there to help us to walk in God’s statutes (commandments) and to do them. So
how can we have the spirit?

John 3:3, 5 [3] Jesus answered and said unto him, Verily, verily, I say unto thee, Except a man be
born again, he can not see the kingdom of God. [5] Jesus answered, Verily, verily, I say unto thee,
Except a man be born of water and of the Spirit, he can not enter into the kingdom of God.

We need to be born of the water and also of the spirit. A lot of people are poor in spirit because they
are not born of the spirit. What happens when a person is born again?

1 John 3:9 Whosoever is born of God doth not commit sin; for his seed remaineth in him: and he
cannot sin, because he is born of God.

When a person is born again, it means that they do not sin anymore. The fact a person sins shows
that we are poor in spirit. What is the role of the Holy Spirit other than to help us to obey the
commandments?

John 16:7-8 [7] Nevertheless I tell you the truth; It is expedient for you that I go away: for if I go not
away, the Comforter will not come unto you; but if I depart, I will send him unto you. [8] And when he
is come, he will reprove the world of sin, and of righteousness, and of judgment:

Spirit reproves us

John 16:13 Howbeit when he, the Spirit of truth, is come, he will guide you into all truth: for he shall
not speak of himself; but whatsoever he shall hear, that shall he speak: and he will shew you things to
come.

Spirit is to guide us

http://breachrepairers.webs.com/ 140
How will the Spirit reprove and guide us?

Eph 6:17 And take the helmet of salvation, and the sword of the Spirit, which is the word of God:

The word of God is like a sword which is called the Sword of the spirit

Heb 4:12 For the word of God is quick, and powerful, and sharper than any twoedged sword,
piercing even to the dividing asunder of soul and spirit, and of the joints and marrow, and is a
discerner of the thoughts and intents of the heart.

It will reprove us because it knows our thoughts and goes against the sin that we like to do. It
discerns our thoughts and intents. It knows our motives and reproves the evil thoughts that we
have.

Ps 119:105 Thy word is a lamp unto my feet, and a light unto my path.

The word of God is like a lamp to guide our feet to show us the way that we ought to walk.
Basically, the Holy Spirit works through the Bible. So how can we be born of the Spirit today that
we may have victory over sin?

1 Pet 1:22, 23 [22] Seeing ye have purified your souls in obeying the truth through the Spirit unto
unfeigned love of the brethren, see that ye love one another with a pure heart fervently: [23] Being
born again, not of corruptible seed, but of incorruptible, by the word of God, which liveth and abideth
for ever.

We must be born of the word of God which is really the same as being born of the spirit since the
Holy Spirit works through the Bible.

Psa 119:9-11 [9] Wherewithal shall a young man cleanse his way? by taking heed thereto according
to thy word. [10] With my whole heart have I sought thee: O let me not wander from thy
commandments. [11]Thy word have I hid in mine heart, that I might not sin against thee.

We need to hide the word of God in our hearts today

John 1:29 The next day John seeth Jesus coming unto him, and saith, Behold the Lamb of God, which
taketh away the sin of the world.

How to have our sins taken away? We need to behold the Lamb

John 5:39 Search the scriptures; for in them ye think ye have eternal life: and they are they which
testify of me.

How do we behold the lamb? We have to behold the scriptures because it tells us about Jesus

Acts 5:32 And we are his witnesses of these things; and so is also the Holy Ghost, whom God hath
given to them that obey him.

We must obey. How is all this applicable to us today? We must first realize that we are sinners.
Solution – we must be born again of the Spirit, born of the Bible and learn to hide it in our heart. But
not just that. We must learn to apply the things that we learn in the Bible as the Holy Spirit is only
given to those that obey. May God give us true insight to our own wretchedness today and give us

http://breachrepairers.webs.com/ 141
the strength to apply the things that we have learnt in beholding the Bible. Maybe we should pray
that God would give us a love for His word.

5:4 Blessed are they that mourn: for they shall be comforted.

“mourn” – sorrow for sin. If we compare ourselves to God, then we really realize our
unrighteousness.

There are two types of mourning in the Bible. What are they?

1 Thes 4:13-18 [13] But I would not have you to be ignorant, brethren, concerning them which are
asleep, that ye sorrow not, even as others which have no hope. [14] For if we believe that Jesus died
and rose again, even so them also which sleep in Jesus will God bring with him. [15] For this we say
unto you by the word of the Lord, that we which are alive and remain unto the coming of the Lord
shall not prevent them which are asleep. [16] For the Lord himself shall descend from heaven with a
shout, with the voice of the archangel, and with the trump of God: and the dead in Christ shall rise
first: [17] Then we which are alive and remain shall be caught up together with them in the clouds, to
meet the Lord in the air: and so shall we ever be with the Lord. [18] Wherefore comfort one another
with these words.

Sorrow for trials and bereavement.

Psa 38:18 For I will declare mine iniquity; I will be sorry for my sin.

Being sorry for sin.

Matt. 11:28 says that we are heavy laden. But what does this heavy burden represent?

Heb 12:1 Wherefore seeing we also are compassed about with so great a cloud of witnesses, let us
lay aside every weight, and the sin which doth so easily beset us, and let us run with patience the race
that is set before us,

We are heavy laden with sin. We are not going to go to Jesus until we realize that we are heavy
laden with sin. See how important the first step is in recognizing your need as a sinner?

What happens when we go to Jesus? What does He do?

Acts 5:31 Him hath God exalted with his right hand to be a Prince and a Saviour, for to give
repentance to Israel, and forgiveness of sins.

Jesus actually gives us repentance and forgiveness. Repentance is not something that actually takes
place within ourselves, Jesus actually gives us that repentant heart.

How do we get repentance from Jesus?

Rom 2:4 Or despisest thou the riches of his goodness and forbearance and longsuffering; not
knowing that the goodness of God leadeth thee to repentance?

We repent when we understand God’s goodness.

Rom 5:6-8 [6] For when we were yet without strength, in due time Christ died for the ungodly. [7]
For scarcely for a righteous man will one die: yet peradventure for a good man some would even

http://breachrepairers.webs.com/ 142
dare to die. [8] But God commendeth his love toward us, in that, while we were yet sinners, Christ
died for us.

We will repent when we understand what Christ did for us on the cross and His great love.
The most ungrateful people are those that do not realize the sacrifice the others have made on their
behalf.

In your own words, describe what the meaning of repentance is? It means to change one’s
mind and purpose as a result of after knowledge. That means when we come to comprehend that
we are sinners and when we understand the great love of Christ in dying for our sins, then that will
lead us to repentance which Christ gives us.

How do we come to a knowledge that we are sinners?

Rom 3:20 Therefore by the deeds of the law there shall no flesh be justified in his sight: for by the
law is the knowledge of sin.

It is through the law that we understand that we are sinners.

Jas 1:23-25 [23] For if any be a hearer of the word, and not a doer, he is like unto a man beholding
his natural face in a glass: [24] For he beholdeth himself, and goeth his way, and straightway
forgetteth what manner of man he was. [25] But whoso looketh into the perfect law of liberty, and
continueth therein, he being not a forgetful hearer, but a doer of the work, this man shall be blessed
in his deed.

It is continually looking into the law that we understand that we are sinners and that we never
forget what God has saved us from. Example – what does it mean to look continually? After every
time you wash your face, you always look up into the mirror to check to see if your face is still dirty
or not. Everyday, we must look into that mirror (not to be vain) but to check to see if we are clean
or not.

1 John 1:9 If we confess our sins, he is faithful and just to forgive us our sins, and to cleanse us from
all unrighteousness.

We are cleansed when we come and confess our sins. Note that we will not confess unless we
realize that we have done something wrong. But confession is our part.

John 15:3 Now ye are clean through the word which I have spoken unto you.

We can be clean through hearing the word being spoken to us.

Eph 5:26 That he might sanctify and cleanse it with the washing of water by the word,

We are cleansed by the word.

Psa 119:9-11 [9] Wherewithal shall a young man cleanse his way? by taking heed thereto according
to thy word. [10] With my whole heart have I sought thee: O let me not wander from thy
commandments. [11] Thy word have I hid in mine heart, that I might not sin against thee.

We not only need to hear the word but we need to pay heed to it. And some people will never have
victory over sin until they memorize scripture and hide it in their heart. There are two types of

http://breachrepairers.webs.com/ 143
sorrow for sin in the Bible. What is the differences between the two?

2 Cor 7:10-11 [10] For godly sorrow worketh repentance to salvation not to be repented of: but the
sorrow of the world worketh death. [11] For behold this selfsame thing, that ye sorrowed after a
godly sort, what carefulness it wrought in you, yea, what clearing of yourselves, yea, what
indignation, yea, what fear, yea, what vehement desire, yea, what zeal, yea, what revenge! In all things
ye have approved yourselves to be clear in this matter.

Sorrow of the world is just being sorry because you were caught. Godly sorrow is true repentance,
making sure that you are clear in the very matter of what was done. How do we know if a person
has truly repented or not?

Prov 28:13 He that covereth his sins shall not prosper: but whoso confesseth and forsaketh them
shall have mercy.

The person will prove through their actions that they are a different person. They will forsake their
old acts. This proves that their confession is genuine. Give an example of a person who keeps saying
sorry but yet keeps doing the same act over and over again. Do you think they are really sorry? True
repentance and confession is a U-turn. The promise of mourning is that we will be comforted. What
is the purpose of this comfort that God gives us?

2 Cor 1:3-5 [3] Blessed be God, even the Father of our Lord Jesus Christ, the Father of mercies, and
the God of all comfort; [4] Who comforteth us in all our tribulation, that we may be able to comfort
them which are in any trouble, by the comfort wherewith we ourselves are comforted of God. [5] For
as the sufferings of Christ abound in us, so our consolation also aboundeth by Christ.

Is 40:1-2 [1] Comfort ye, comfort ye my people, saith your God. [2] Speak ye comfortably to
Jerusalem, and cry unto her, that her warfare is accomplished, that her iniquity is pardoned: for she
hath received of the LORD'S hand double for all her sins.

The purpose of this comfort is so that we will comfort others who are going through the same trials
as us and may be even committing the same sins we committed in the past.This will help us to not
be so critical when a person falls into the same sin as us, because we remember how we were like
that in the past. A quote you can use if it is all Adventists or just say it is your favorite author:

MCP 431 Helping Others Helps Oneself.--Many are in obscurity. They have lost their bearings. They
know not what course to pursue. Let the perplexed ones search out others who are in perplexity and
speak to them words of hope and encouragement. When they begin to do this work, the light of
heaven will reveal to them the path that they should follow. By their words of consolation to the
afflicted they themselves will be consoled. By helping others, they themselves will be helped out of
their difficulties. Joy takes the place of sadness and gloom. The heart, filled with the Spirit of God,
glows with warmth toward every fellow being. Every such a one is no longer in darkness; for his
"darkness" is "as the noon day.

5:5 Blessed are the meek: for they shall inherit the earth.

“blessed are the meek” – When we are meek, we are really happy. When you don’t have to prove
that you are best and are just content with where you are, then you don’t have stress. When you
don’t have pride, then you are happy. We must first realize that we are heavy laden with sin. This is
one of the key ingredients that will help to keep us meek. Or else we would become proud thinking
that we don’t need help from Christ. Because the wages of sin is death, how meek do you think
someone would be who is waiting for a death sentence?

http://breachrepairers.webs.com/ 144
We must also yoke ourselves together with Christ and learn of Him.

What does it mean to be yoked together with Christ? What does the word “yoke” mean?
Explain how when two oxen are yoked together. They have to walk the same path. It is usually a
younger oxen with an older oxen. The younger provides the strength and the older provides the
guidance. This is how the younger oxen learns, by being yoked up and learns through experience.
When we are yoked with Christ, it means that we will walk the same path that He walks, and we will
follow the same way that we went. How do we learn of Christ in His school that we may be His
followers?

1 Pet 2:21 For even hereunto were ye called: because Christ also suffered for us, leaving us an
example, that ye should follow his steps:

In the school of Christ we learn through suffering, not just through books. It’s easy to talk religion,
but to live it is another story.

Matt 16:24 Then said Jesus unto his disciples, If any man will come after me, let him deny himself,
and take up his cross, and follow me.

Before we can follow Jesus, we must deny ourselves and take up the cross.

Gal 2:20 I am crucified with Christ: nevertheless I live; yet not I, but Christ liveth in me: and the life
which I now live in the flesh I live by the faith of the Son of God, who loved me, and gave himself for
me.

We must crucify ourselves so that Christ can live through us. We must expect to suffer when we
follow Christ, because that is what He went through.

What type of suffering is this relating to? What type of suffering did Jesus have to go
through? This is not relating to physical suffering. It was the suffering that we go through in the
mind. Yes Jesus went through physical suffering, but other type of suffering did He go through? He
had the power to destroy all the sinners, He had the power to retaliate and show others that He was
God, but on the cross the greatest suffering He went through, apart of dying from a broken heart,
was to withhold Himself from showing His divinity. Just like us when we have the power to
retaliate, the greatest suffering is to withhold yourself from showing that you can beat them or be
better than them, etc.According to Matt. 5:5 the meek shall inherit the earth. But in Ps. 37:9-11, who
are those that will inherit the earth?

Psa 37:9-11 [9] For evildoers shall be cut off: but those that wait upon the LORD, they shall inherit
the earth. [10] For yet a little while, and the wicked shall not be: yea, thou shalt diligently consider
his place, and it shall not be. [11] But the meek shall inherit the earth; and shall delight themselves in
the abundance of peace.

They that wait upon the Lord = the meek. Both these groups shall inherit the earth.

What does it mean to wait upon the Lord?

Prov 20:22 Say not thou, I will recompense evil; but wait on the LORD, and he shall save thee.

To wait upon the Lord means that we will wait for His salvation and will not recompense evil for
evil.

http://breachrepairers.webs.com/ 145
Rom 12:19 Dearly beloved, avenge not yourselves, but rather give place unto wrath: for it is written,
Vengeance is mine; I will repay, saith the Lord.

Waiting on the Lord means to overcome evil with good because we realize that vengeance is God’s,
not ours. To wait upon the Lord means that we need to be patient:

Rev 14:12 Here is the patience of the saints: here are they that keep the commandments of God, and
the faith of Jesus.

There will be a group of people called saints that have patience. How do we develop patience?

Jas 1:2-3 [2] My brethren, count it all joy when ye fall into divers temptations; [3] Knowing this, that
the trying of your faith worketh patience.

We develop patience through trials. You have to suffer even more when you have to love your
enemy. It isn’t easy. The suffering is inside… willing to be humble and kind and loving and gentle to
the one that hates you or that hurt you.

What is meek also equivalent to according to the Bible?

Ps 25:9-13 [9] The meek will he guide in judgment: and the meek will he teach his way. [10] All the
paths of the LORD are mercy and truth unto such as keep his covenant and his testimonies. [11] For
thy name's sake, O LORD, pardon mine iniquity; for it is great. [12] What man is he that feareth the
LORD? him shall he teach in the way that he shall choose. 13His soul shall dwell at ease; and his seed
shall inherit the earth.

The meek He shall teach His way and he that fears God shall also He teach His way. So being meek
means to fear God. Because the meek will inherit the earth, so also will those that fear God. How
does fearing God relate to being meek?

Ex 20:20 And Moses said unto the people, Fear not: for God is come to prove you, and that his fear
may be before your faces, that ye sin not.

Fearing God means that we don’t sin.

Deut 6:2 That thou mightest fear the LORD thy God, to keep all his statutes and his commandments,
which I command thee, thou, and thy son, and thy son's son, all the days of thy life; and that thy days
may be prolonged.

Eccl 12:13, 14 [13] ¶ Let us hear the conclusion of the whole matter: Fear God, and keep his
commandments: for this is the whole duty of man. [14] For God shall bring every work into
judgment, with every secret thing, whether it be good, or whether it be evil.

Fearing God means that we keep the commandments meaning that we don’t sin because sin is the
transgression of the law (1 John 3:4).

Ps 149:4 For the LORD taketh pleasure in his people: he will beautify the meek with salvation.

The meek will receive salvation. Salvation means victory. We will be given victory over what? Over
sin. Therefore, if we want the promise of inheriting the earth to come true, then we must learn to
have victory over sin.

http://breachrepairers.webs.com/ 146
In the context of what we studied, how is it that we may inherit the earth? TEACHERS,
basically this is to recap the whole lesson. If we realize our condition of being a sinner, being heavy
laden. If we learn to be patient through trials and suffering – not recompensing evil for evil. If we
have victory over sin. Somehow, we can never have victory over sin or be perfect if we are not
patient.

5:6 Blessed are they which do hunger and thirst after righteousness: for they shall be filled.

“Blessed [are] they which do hunger and thirst after righteousness” – Each drink of the living
water will make the drinker want to drink more.

What is the definition of righteousness?

Rom 3:10-12 [10] As it is written, There is none righteous, no, not one: [11] There is none that
understandeth, there is none that seeketh after God. [12] They are all gone out of the way, they are
together become unprofitable; there is none that doeth good, no, not one.

It means to seek after God and to do good.

Who is unrighteous?

1 John 5:17 All unrighteousness is sin: and there is a sin not unto death.

All unrighteousness is sin.

Rom 3:23 For all have sinned, and come short of the glory of God;

All have sinned. Therefore, everyone is unrighteous. There is not one person here on this earth that
is righteous.

Who then is righteous?

1 John 2:1 My little children, these things write I unto you, that ye sin not. And if any man sin, we
have an advocate with the Father, Jesus Christ the righteous:

Jesus Christ is righteous. According to what we have studied so far, if we are hungering and
thirsting after righteousness, what does that mean? What are we really hungering and
thirsting after? It means that we are hungering and thirsting after Jesus Christ because He is
righteous.

When does one literally feel hungry or thirsty? When your stomach is empty or when your fluids in
your body is low. So it is when we are to hunger and thirst for Christ. We must feel that we are
empty inside or else we won’t be hungry or thirsty for Him. Many feel like they are ok, so they don’t
hunger and thirst after Him. That is why the first step is so important, to realize we are sinners, or
else we will never feel like we need Christ. Because when we realize we are sinners, we realize that
we are empty and nothing without Christ. Of course we must never just continue to bash ourselves
over the head saying we are sinners, but at least we must ever keep this before us, or else we will
come to this point one day thinking that we don’t need Christ anymore. So there is a fine balance
between the two.

http://breachrepairers.webs.com/ 147
What does it mean to hunger and thirst after Christ?

John 6:35 And Jesus said unto them, I am the bread of life: he that cometh to me shall never hunger;
and he that believeth on me shall never thirst.

To eat of Christ, to eat the bread. To go to Him and believe on Him. So we must eat of the living
bread, the Bible (Matt. 4:4).

John 6:53-57

To eat Jesus’ flesh and drink His blood. Really it means to abide in Christ and He in us (verse 56)
and also to live by Christ (verse 57).What does it mean to abide in Christ?

John 15:4-10

To abide in Christ means to have His words abide in us (verse 7). How can we have His words abide
in us? By keeping his commandments (verse 10).

1 John 2:3-6

To abide in Christ means to walk as He walked. How did He walk? He kept the commandments. And
so contextually speaking, how do we know if we are in Him or not, by keeping His word, which is
referring to the commandments. So if we want to abide in Christ, we need to keep His words in us,
we need to keep His commandments.

What do we understand about the commandments according to the Bible and how is this
applicable to us?

Psa 119:172

All the commandments are righteousness. So if we are hungering and thirsting after righteousness,
we are really hungering and thirsting after Christ, we are abiding in Him. And if we are abiding in
Him, it’s because we are keeping the commandments, which are righteousness. Therefore, we
should be hungering and thirsting after the commandments of God which is righteousness.

How can we abide in Christ that we may keep His commandments?

John 14:15

If we love Him we will keep His commandments.

Rom 13:10

Love is the fulfilling of the law. We will abide in Christ because we love Him.

Where does this love come from?

1 John 4:19

We will only love Him when we realize the love that He first showed us.

http://breachrepairers.webs.com/ 148
How did He show His love to us?

John 3:16

The love of God was demonstrated at the cross.

Luke 7:47

To him that was forgiven much shall also love much. But when we don’t realize that we are heavy
laden and burdened with sin, we won’t feel that we need forgiveness. And as a result, we won’t love
much. That is why the first step is so important.

In the context of what we have studied today, how can we be filled today? If we want to be full,
then we must hunger and thirst after righteousness. Righteousness = Jesus Christ. Righteousness =
commandments. To hunger and thirst after Christ means to abide in Christ = keeping His
commandments. And we will keep His commandments if we love Him, and we will love Him when
we realize how much He has forgiven us. And we will seek and remember His forgiveness when
realize that we are heavy laden with sin. THIS IS A NATURAL PROGRESSION. EMPHASIZE THAT.
AND ALSO REMIND THEM THAT THIS IS THE STEPS TO HAPPINESS (Blessed means to be happy)
End with this text: Prov. 29:18 – happy is he that keepeth the law.

5:7 Blessed are the merciful: for they shall obtain mercy.

One of the things that we are told about a condition for our forgiveness, is that we would be willing
to forgive others. The ones that love us the most, hurt us the most. They are the hardest to forgive.

In order to obtain mercy, what must we do?

Matt 6:12

If we want mercy, we must show mercy, we must be merciful. If we want to follow the Lord’s
prayer, we must pray that our debts be forgiven only as we forgive others.

In your own words, what is the definition of mercy? Mercy – to have compassion (by word or
deed, specially by divine grace)s, have pity on.

What is mercy connected to?

Luke 6:36

We are to be merciful as our Father in heaven is merciful.

Matt 5:48

A parallel verse, we are to love as Jesus loved. So really, the Bible’s definition of merciful is to love
as Jesus loved. We will never show the same mercy upon others if we don’t love like Jesus.

According to the Bible, what is another word for mercy?

Gen 19:19

http://breachrepairers.webs.com/ 149
Heb 4:16

Grace = Mercy

How is God’s mercy or grace displayed?

Eph 1:7

Riches of his grace is needed for forgiveness of sins.

Exo 34:7

Mercy needed for forgiveness of iniquity.

Psa 86:5

Mercy of God is to forgive. God’s mercy is mostly connected with forgiveness. If we want to have
the same mercy as Jesus, we must have the same love. What was that love?

John 3:16 ….for God so loved the world that He gave….

Somehow forgiveness is tied in with something that we must give. God gave His Son for the sins of
the world. This also was His ultimate act of love. How did God love? He had to forgive.

Rom 5:8 ….while we were yet sinners, Christ died for us…

Dying for the ones that hate us is really much harder than we think it is. Also, forgiving someone
that really deserves punishment or death is also much harder than we think it is. But that is God’s
love. To forgive your enemies and those that deserve the punishment that they are getting. BUT
keep in mind, it required us to give up something.

How do we get this love of God, that we may also forgive and be merciful like our Father?

Luke 7:36-47

Those that are forgiven much will love much. Those that are forgiven little love little. So those that
are unmerciful and unforgiving, it is because they have forgotten how much Christ has forgiven
them. If they remember that, their love would flow out to others just like God’s love flowed out to
us.

Matt 18:21-35

Similar story to show that God has forgiven us so much, that we ought to forgive others, whose debt
to us is so little compared to our debt to God the Father.

Rom 3:23 …all have sinned…

Rom 6:23

There are no degrees of sin. We have all sinned. We all deserve death. The problem is that many
think they are worthy of eternal life because their sins are not as bad as others. We should not

http://breachrepairers.webs.com/ 150
compare a sinner to a sinner. We should all compare ourselves to Jesus, and then we will realize
how far short we have really fallen.

What is God’s ultimate act of mercy towards us?

Heb 8:12

God’s ultimate act of mercy is to not remember our sins anymore.

Is 43:25

Not remembering our sins is to blot them out. A Christian has no right to hold grudges. When we
forgive someone, we should not even remember it. We should act as if they never did it. God will not
bring our sins back to remembrance in heaven if we ever get there.

What is our part to play that God may blot out our sins?

Heb 10:16-17

We must allow Him to write the law in our hearts and our minds. We must remember His
commandments.

Deut 7:9

Keeps His covenant and mercy to them that love Him and keep His commandments.

Rev 14:5, 12

These people are without fault and they also keep the commandments of God. Let us not be fooled
that there is not part for us to play in the plan of salvation. In order for God to blot out our sins, we
need to remember His commandments and also show that same mercy to others as well. Love is not
enough, we must also show it (John 14:15).

5:8 Blessed are the pure in heart: for they shall see God.

“Pure in heart” right now:

DA 302 The merciful shall find mercy, and the pure in heart shall see God. Every impure thought
defiles the soul, impairs the moral sense, and tends to obliterate the impressions of the Holy Spirit. It
dims the spiritual vision, so that men cannot behold God.

Spiritual things are spiritually discerned. We will not be able to see God in the scriptures.

“Pure in heart” when Jesus comes we can actually see God.

According to the Bible what is pure?

Ps 19:8

The statutes and commandments are pure.

http://breachrepairers.webs.com/ 151
Psa 12:6

Prov 30:5

The words of the Lord are pure. So basically, it is referring to the Bible as the one that is pure.

What are we to put in our heart?

Psa 40:8

The law is to go into the heart.

Psa 119:11

The word of God is to go into the heart so that we don’t sin. So we see that the word of God and the
law of God (basically the Bible) have a lot to do with having a pure heart.

What are we to be pure from?

1 Tim 5:22

To be pure means to be pure from sin.

Prov 20:9

To be pure (or have a clean heart) means you are pure from sin. So when the Bible says blessed are
the pure in heart, it means that these people are pure or free from sin. Contextually speaking: We
cannot be free from sin if we don’t realize we are sinners (blessed are they that are poor in spirit).
We cannot be free from sin if we don’t mourn for our sins (repentance). We cannot be free from sin
if we don’t hunger and thirst after righteousness (seeking after the Bible and the law).

According to the previous beatitudes, how can we have a pure heart?

Matt 5:3-7

Poor in spirit – to realize our need of Jesus. Mourn – to mourn and be sorry for our sins. Meek – to
yoke ourselves up with Christ and follow where He goes. Hunger and thirst after righteousness –
hungering and thirsting after Jesus and the Bible. Merciful – forgiving and giving grace because we
are forgiven and have the love of Jesus in us.

What is our part to play that we may have a pure heart? We saw earlier that we need to hide the
word of God in our hearts (Ps. 119:11).

Eph 5:26

Use the word to clean our hearts and minds.

Phil 4:8

We need to train our mind to dwell on more eternal things and not let the mind wander. We need to
learn to use our imagination to reflect on pure and holy things.

http://breachrepairers.webs.com/ 152
1 Pet 1:22

Psa 119:9

Not only do we have to study the word of God and think/dwell on eternal things, but we also have
to obey what the Bible says if we want to have a pure heart. How can we obey?

Rom 6:12-13

We must learn to yield ourselves to God. What does that word “yield” mean? It means to surrender.
We must learn to surrender ourselves by God by placing our will in His hands. Then the result will
be found in the next verse.

Phil 2:13

God will work in us to will and do of His good pleasure. How do we know if our heart is pure or
not?

Prov 20:11

We will be known by our works whether they are pure or not.

Prov 21:8

What does the Bible warn about those that are striving to be pure? What must we be careful
of?

Prov 30:12

There is a generation of people that think they are pure. We must be careful when striving to be
pure that we don’t think that we are pure when we really aren’t. The litmus test is our works. There
are those that say and think they are pure, but their very works deny their profession. What are
they called? Hypocrites. We must remember though that we are not saved by works. Our works
only show what type of person we are.

Prov 16:2

We, in our own eyes, may think that we are pure. But we need to allow the Lord to judge us on that.
How does He do that? But using His word as the standard. So we must compare our lives to God’s
standard – the Bible. What does the Bible call those people that think they are pure but really
aren’t?

Rev 3:17

They don’t know that they are wretched, miserable, poor, blind and naked. The Bible calls them
Laodicean. They think they are alright when they really aren’t. Why? Because they haven’t
compared their lives to the standard of God’s word. Who will see God when He comes again?

Rev 1:7

http://breachrepairers.webs.com/ 153
Everyone will see him. What will be the difference between those that have pure heart and
those that have filthy heart when God comes again?

Rev 6:14-16

The wicked will run and try and hide from Him and ask the mountains and rocks to fall on them.

Isa 25:9

They will rejoice to see Him for they have been waiting for Him. One group will run away from Him,
and the other will run towards Him. What is this group of people called that are able to see
God?

1 John 3:2, 3

They are called the sons of God. Keep in mind that a son resembles the Father in some way. Do
people see God’s character or resemblance in you today? How can we be a son of God today?

John 1:12

We have to believe and also receive Him. If you believe in Him and receive Him today, you are called
the son of God. It’s that simple, believe in Him.

Gen 15:6

It says that Abraham believed God and it was counted to Him for righteousness. Many of us don’t
believe. We say we believe, but our acts show the opposite. Let us begin to really live like a child of
God today. Believe that we have a pure heart and start living like God has cleansed us from our sins.
Believe that we are holy and that we don’t participate in the sins of the world anymore. Believe!
Believe on Him and He will save you!

5:9 Blessed are the peacemakers: for they shall be called the children of God.

The carnal heart is enmity of God. But if we make peace, we shall be called the children of God.

In your own words, what does the word peacemaker mean? Let them share and then try and
confirm their answer from the Strong’s Concordance. To reconcile between two parties. From the
Strong’s Concordance: It means to pacify, to appease. To pacify means to soothe or calm. It has
much to do with trying to quieting people who are upset, excited or disturbed. Appease means that
you have given into someone’s demands or made concessions in order to please them (eg. She said
she would visit her mother just to please her).

Before we can become peacemakers, what must take place first?

Rom 5:1

We need to be justified by faith first. We need to make peace between God and us (the sinner) first.
We need to realize this peace so that we can show it to others.

How are we justified?

http://breachrepairers.webs.com/ 154
Rom 5:9

We are justified by the blood of Jesus.

Job 25:4

Justify = clean

Jer 33:8

Clean = pardon

Num 14:19

Pardon = forgiveness

1 John 1:9

In order for us to be forgiven we need to confess our sins. Jesus dying on the cross for us was not
enough to clean us. We need to want to be clean in order for His blood to justify us. So we are
justified by two things: By the blood of Jesus. By confessing our sins.

How can we become a peacemaker?

1 John 3:2-3

Those that are peacemakers are the children of God. Notice that the sons (children) of God are also
pure, meaning they are pure in heart. Therefore, the beatitudes give us a logical progression in how
we may be able to become peacemakers. Remember, the beatitudes are steps.

NOTE: Go through it quickly just to refresh it with them. Blessed are the poor in spirit: for theirs
is the kingdom of heaven. Realizing your need of Jesus. Blessed are they that mourn: for they shall
be comforted. Being sorry for your sins – repentance. Blessed are the meek: for they shall inherit
the earth. Following after Jesus to learn of Him (Matt. 11:28). Blessed are they which do hunger and
thirst after righteousness: for they shall be filled. Seeking after Jesus and the Bible. Blessed are the
merciful: for they shall obtain mercy. Forgiving and giving grace because we are forgiven and have
the love of Jesus in us. Blessed are the pure in heart: for they shall see God. Having the law written
in our hearts and our works showing our pureness so that we will see Jesus face to face when He
comes again for a second time.

What else does the Bible talk about in how we can become peacemakers?

Psa 119:165

Great peace have they which love thy law. If we love the law of the Lord, it will give us peace and
nothing shall offend us.

Prov 3:1-2

Forget not the law, but let the heart keep the commandments. This is going to be important for a
future question.

http://breachrepairers.webs.com/ 155
What are the characteristics of a peacemaker or a child of God?

Rom 12:18-21

Living peaceably with all men. How? It first comes with a knowledge that vengeance (vindication)
belongs to the Lord, not us. If we think we have the right to take revenge on those that have hurt us
then we have forgotten then words of the Lord. Be nice to those that hurt you. This is called the
perfect revenge. This is a peacemaker.

Matt 5:43-48

To be a peacemaker that we may be called children of God is to love your enemies.

1 John 3:10

Isa 32:17

They do righteousness. Their outward acts show that they are a child of God.

Phil 2:14-15

They are blameless and harmless. How? They do all things without murmuring and disputing
(debate). Note: Remember that these characteristics are all important because these are needed to
lead a soul to Christ. Sin separates us from Christ. If we are a peacemaker, it is because we are in
peace with God. We have confessed our sins. And so we need to help others make peace with God as
well. Help them to renounce their sins and yield their lives to God.

Why is it so important for us to learn the lesson of being peacemakers?

Matt 24:9

There is coming a time where people will persecute us because of the name of Jesus. It becomes so
important to understand what we believe now and have the law written in our hearts, because in
the future when people will persecute us for following the Lord at least we will know what we
believe. And we are not believing it because of anyone but because we love Him.

Matt 5:11-12

Same concept brought out about being persecuted in the future. If we aren’t willing to stand up for
the Lord now in times of peace, we will definitely not want to be peaceable with those who will
persecute us in the future. It is going to be in our retaliation, how we react, that will show whether
we are children of God or not.

Have you been through trying circumstances in life that you can see now that the Lord is
trying to teach you to be a peacemaker or to be more long suffering? This is an open-ended
question for discussion. Summary: Remember that God allows calculated trials to test our faith and
patience that He may prepare us for the future struggles ahead.

John 1:12-13

http://breachrepairers.webs.com/ 156
5:10 Blessed are they which are persecuted for righteousness' sake: for theirs is the kingdom of heaven.

When you are trying to do what is right and are persecuted, then we have the promise that we will
have the kingdom of Heaven.

What is the difference between this blessing and all the others before it? This is the one
beatitude that we don’t have a choice over. People are persecuting us. That is not a choice. It
involves another party. But we have a choice to mourn, to hunger and thirst, to be a peacemaker. All
the others involve choice except this one. However, we do have one choice in the midst of this,
whether we want to be happy or not.

According to the verses that we read, why are we persecuted? We are persecuted for
righteousness sake. Note: There are some that are persecuted for their own foolishness sake, for
their own actions or words and they deserved to receive that persecution, because of their own
sins. But this is referring to those who are righteous, not unrighteous.

What does the word righteousness mean? Discuss. The word righteousness simply means “right
doing.” So therefore, in these verses it is describing the fact that we are being persecuted because of
doing what is right. Have you ever been persecuted or punished for doing the right thing? Teachers,
please make sure they share. And think of examples to share of your own experience. Note: the
beatitudes are basically steps on how we may be righteous.

What is righteousness according to the Bible?

Psa 119:172

Law is righteous

Rom 7:12

The law is holy, just and good. So righteousness is being holy and being just and being good. This is
a transcript of God’s character. So really, being righteous means to reflect God’s character.

2 Tim 3:12

Being righteous means to live Godly lives. How do we know? Because living Godly life means you
will be persecuted, and living righteous life means to you will be persecuted also. Therefore,
righteousness = living Godly life.

Who will persecute those that live Godly lives in 2 Tim. 3:12?

2 Tim 3:1-5

This is describing those that have a form of Godliness by deny the very power thereof. They are
Christian’s, and claim to be Christian’s but live the very opposite to what a Christian should be.
Those sorts of people you need to be afraid of the most. They will be the greatest persecutors of the
righteous in the last days, not the heathen’s, but those Christian’s in the church who are
unconverted.

Gal 4:29

http://breachrepairers.webs.com/ 157
They haven’t died to self. They still have a carnal mind and haven’t been born again and are not .
These will be the greatest persecutors.

Why will those mentioned in 2 Tim. 3:12 be the greatest persecutors?

John 15:20-21, 25

Because they didn’t really know who Jesus was. It is possible to go to church every week and still
don’t know who Jesus is.

John 3:19-20

They love darkness rather than light. And so when a truly righteous person comes along, they hate
him because by his life their own evil deeds are reproved.

Rev 3:17

They don’t even realize they are sinners. They haven’t even taken the first step in the beatitudes in
realizing that you need Jesus because you are a sinner. This Laodicean condition is very dangerous.
Remember, they are not cold or hot. They are lukewarm. They are in the church but don’t have the
power of the true Christian. We have much to fear from within.

Why will they be the greatest persecutors? They will look at those who really are living righteous
and Godly lives and will start to ridicule them saying they are too strait laced, or too legalistic, or get
angry at them because they know their own lives aren’t like those who are righteous.

What examples are we given in the Bible about those that will be persecuted for righteousness
sake?

1 John 3:11-13

Why did Cain kill Abel? Because his works were more righteous. Recap the Cain and Abel story.
Teacher’s please read up on Gen. 4:3-8. Notice, Cain was a professed worshipper of the true God,
but yet it was he that slew his own brother. And this is what will happen in the last days as well.

According to what we have studied so far, why is there so little persecution in the world
today?
Because there are little or few righteous people in the world. There is little godliness in this world.

How should we react to this persecution?

Matt 5:11-12

We should rejoice. Notice that they are even saying evil things against you. That means they are
lying about what you are doing and who you are. And how should we respond? REJOICE!

Rom 12:14, 18-21

We need to bless them that persecute us. We must remind ourselves that vengeance belongs to God,
not us.

http://breachrepairers.webs.com/ 158
Matt 27:12-14

When Jesus was accused of evil deeds, He didn’t answer even a word. How often we try to vindicate
ourselves in what we do but speaking. But the way that Jesus handled it was by remaining quiet.
Are going through you persecution? Learn to be silent. Remember that God will avenge, we don’t
need to do it here on this earth. What must we remind ourselves of in these times that we live?

Rev 12:17

It is the great dragon who is Satan (Rev. 12:9) that really wants to destroy those that are righteous,
who follow God and His commandments.

Matt 5:12

We must remind ourselves that our reward is in heaven. RECAPPING LESSONS LEARNT: Learning
to be quiet in the face of persecution. You may be going through persecution in the home with
family, learn to be quiet and not retaliate. Remembering that vengeance is God’s, not ours.
Remembering that our reward is in heaven, not on this earth.

5:11 Blessed are ye, when men shall revile you, and persecute you, and shall say all manner of evil
against you falsely, for my sake.

5:12 Rejoice, and be exceeding glad: for great is your reward in heaven: for so persecuted they the
prophets which were before you.

Luke 6:23

It tells us to leap for joy. When studying John the Baptist, the greatest gift that he has bestowed
upon man: Fellowship of Christ and His suffering, Jesus knew that John the Baptist could endure it.

5:13 Ye are the salt of the earth: but if the salt have lost his savour, wherewith shall it be salted? it is
thenceforth good for nothing, but to be cast out, and to be trodden under foot of men.

“Salt” – It is a preserving agent for food. Spiritual Application: Salt makes you thirsty – thirsty for
the living water. So a well ordered family can make someone thirsty. But how can we be salt here? If
we have true joy, love, happiness, patience, etc, people will want that. It melts ice. Spiritual
Application: As Christians, we are the ones that should be breaking through the prejudice, anger,
hatred, etc. Salt that has no flavour, is useless. We can only obtain our savor by studying God’s
Word. In order to have this potency, is to study the Word of God. They used salt in the older times
for remedies:

Eze 16:4

It seems that when the baby was born, they would put the baby in salt. According to Matt. 5:13,
what was salt to be used for? It was to be used as a flavouring, as a savour. What was being a
savour also connected to in the Bible?

Gen 8:20, 21

Noah made a sacrifice and it came up as a sweet savour.

http://breachrepairers.webs.com/ 159
Exo 29:25

Sweet savour is connected with burnt offering.

Num 15:13

Sweet savour connected to offering made by fire.

Ezra 6:10

Sweet savour is connected to sacrifices again. If you look up the word “savour” in the Bible, you find
that it is connected a lot with sacrifices and offerings.

What was salt used for in the Bible?

Lev 2:13

Salt was to be put with all the offerings and sacrifices. So we see here that salt is connected
with sacrifice, and salt gives savour which is also connected with sacrifices.

What does the Bible call us to be?

Rom 12:1

We are called to be living sacrifices. The word “living sacrifice” seems like an oxymoron. Living
means to be alive, but sacrifice seems like we are to die. How can we be a living sacrifice?
How can we be alive and dead at the same time?

Rom 6:3-11

We are to be dead to sin. Baptism is a symbol of that death to sin. But we aren’t to remain dead, but
we are to rise in newness of life. That is why Jesus’ resurrection was so important. Verse 10 – we
are to live unto God. That means, our life is going to be lived in such a way that will glorify God, but
we must die first. Many Christians are not dead to self and sin yet. They are professing to live a life
of a Christian, but sin still reigns in their lives.

Rom 12:2

To be a living sacrifice in this text means to be NOT conformed to this world. By being not
conformed to the world means that we are not like the world. We don’t enjoy the same things as the
world does, we don’t do the same things they do. Our pleasures and enjoyments are found in other
things.

NOTE: Oxymoron means two words which means the opposite but cannot be put in the same
phrase. Eg. Cruel kindness, make haste slowly, deafening silence.

How do we know if we have the experience of being salt of the earth?

Mark 9:49-50

http://breachrepairers.webs.com/ 160
If we have salt in ourselves, we will have peace with one another. That means we will be
peacemakers. That means we will have the experience in the beatitudes of being a
peacemaker, which means we will have gone through the steps of the beatitudes.
How can we have peace with one another?

Rom 12:18-21

It means that we won’t retaliate to other people’s unkindness and their actions of being
unjust. We will overcome their wicked actions with kindness. It doesn’t mean people won’t
hate us. Christ had peace with everyone but people still crucified him. But we must in our
minds have peace with them.

How else do we know if we have the experience of being salt of the earth?

Col 4:6

By our speech people will know whether we are of God or not. Our speech will be seasoned with
grace. Our words will heal and not destroy. How was Jesus salt of the earth when he was on the
earth? And how can we follow His example?

Eph 5:2

Christ became a sweetsmelling savour by being a sacrifice for us. And in the similar manner, we are
to walk in the same way as He did. We are to walk in love. Our actions are to show to others of His
love towards us. We are to be a sacrifice for others.

John 15:13

Our greatest way of being salt of the earth is to lay down our lives for our friends. That is to walk in
His love. How else can walk in His love that we may give a sweet savour to others?

1 John 3:16-18

To love in deed. Helping our brother in need. Our works don’t get us into heaven, but it is by our
works that it will show whether we are salt with savour or salt that is good for nothing. What type
of salt are you today? How does your life stand with Christ and others? Is it salt that has a good
flavour? Or are you salt that just looks like salt but has not flavour? Are you just a Christian by name
or are you really a true Christian which by word and deed shows the love of Christ to others?

5:14 Ye are the light of the world. A city that is set on an hill cannot be hid.

What is the relationship between “ye” and the “city.” Zion is God’s people. There are many
references that show that Zion is a symbol of God’s people.

How can we become lights of the world?

2 Cor 4:6

In the same way that Jesus shines light we are to shine the light. But before we can even shine light,
we must have that light shining in our heart first because we have no light in our hearts of our own
self. What was the condition of our heart before God shone light into it?

http://breachrepairers.webs.com/ 161
Gen 1:1-2

The earth was without form and void, and darkness was upon the face of the deep. “Without form”
means a desolation, worthless, empty place, confusion, waste, wilderness. “Void” means an
undistinguishable ruin, emptiness. “Dark” means misery, destruction, death, ignorance, sorrow,
wickedness. This sounds like the earth that we live in today. Why? Because this is the condition of
many of our hearts before God shines light into it. How did God shine light into our hearts?

Gen 1:3

God spoke and the light was created. So in the same way that God spoke light into the dark earth, He
spoke that light into our hearts. So how can He shine light into our hearts? We must hear Him speak
and as a result, the light will be created. What does light represent in the Bible?

1 John 1:1, 4

Light represents the Word who was Jesus (John 1:14 – only say this text if necessary).

John 8:12

Jesus is the light of the world.

Psa 119:105

Light represents the Word of God.

2 Cor 4:4

Light represents the gospel. So we must hear about the Bible which speaks about gospel of Jesus
Christ. This is the light that must shine into our hearts to fix the problems of darkness and void and
without form.

According to what we have studied so far, how can we be a light of the world? We must learn
to behold Jesus who IS the light of the world. And by beholding we will become changed. So we
need to study the Bible which tells us about Him. We also need to share this same light. How do we
share the word of God? Preaching. Bible study, and Evangelism.

How else can we be lights of the world? What else does light represent in the Bible?

2 Cor 4:6

Light of the knowledge of the glory of God. Therefore, light = glory. What does glory represent?

Exo 33:18, 19

It represents character. It is by our character. Why does the Bible state in Matthew 5:14 that a
city that is set on a hill cannot be hid? How does that relate to our previous question?

Matt 5:14

http://breachrepairers.webs.com/ 162
Just as a city that is on a hill can be easily seen from a distance, so our characters are also seen from
a distance. People will naturally know us for who we really are eventually. We may be able to hide it
for a time because they don’t know us, but soon enough our true character will come out. So if we
want to naturally shine Jesus’ character, we must behold the Bible that we may be changed into the
same likeness.

What is the emphasis of Matthew 5:15? Why do you think Jesus gave this illustration? What
was His point?

Matt 5:15

Naturally people do not light a candle and then hide it. The purpose of the candle is to give light.
And so naturally they would want to set it in a place where the most light could be given – on a
candlestick. Jesus is trying to emphasize more His point about a city being set on a hill and not being
able to hide. As a Christian comes in contact with Christ and his/her life begins to change, they
cannot help but shine the light of Christ. It is not something that is forced. It is natural. It we are
lights of the world, we will naturally shine the character and love of Christ to this world. We cannot
force ourselves to be lights. It is something that will come naturally.

What does a candlestick represent in the Bible and what application can we draw from that?

Rev 1:20

A candlestick represents the church. Application: God’s appointed way of shining the Bible and His
character to the world has always been through the church. Not just an individual, but also as a
corporate body are we to shine into this dark world. According to Matt. 5:16, what does the light
represent and what important application can we draw from this?

Matt 5:16

Light represents our works.

Phil 2:14-15

What type of works? – doing all things without murmuring or disputing. No complaining or fighting.
Not only are we to shine the character of Jesus and preach, but we are to show through our works,
our actions, that we are the children of the light. Many people think that all we need to do is have
faith and believe. But our works will tell a lot as to what sort of person we truly are and whether we
have the character of Jesus or not.

According to Matt. 5:16, what should be the result of shining our lights?

Matt 5:16

People should give glory back to God. Glory represents character – we studied it earlier in the
lesson. So naturally, when people see our good works, they should naturally shine the character of
God to others as well. End with these questions: Stop and think about the people around you. Are
they more blessed and drawn towards God when they hang around you? Or do you bring out the
worst in them? What is your influence like? Are you shining the light of Christ and helping them
reflect the character of Christ? Or are you shining darkness and making them worse? Let us be
lights in this world today.

http://breachrepairers.webs.com/ 163
5:15 Neither do men light a candle, and put it under a bushel, but on a candlestick; and it giveth light
unto all that are in the house.

There is one thing that could hinder your life from shining – a bushel. The term “bushel” is just a
measuring instrument. It is used in connection with agricultural harvest so it can be used as a
symbol of prosperity. Prosperity can cause people to forget about God.

Luke 8:16

What does a vessel represent? – a person

2 Cor 4,

Matt 25

How can a person keep your light from shining? Looking at the faults of others or looking at man.
People’s faults sometimes keep us from doing our own Christian duty. We can not let our light shine
sometimes because we see the good qualities in others. We believe that there is someone else who
can do it. But we must just do our thing. What does a bed represent? Laziness

Prov 6:9

“Light” is a symbol good works. The way we prepare for the coming of Jesus is by good works. But
what is the source of the light? It was the oil and that is a symbol of the Holy Spirit (Zach 4, Gal 4).
The source of all good works is from the Holy Spirit. But do not let the legalistic idea come in. None
of our good works will merit us any favor with God. We cannot be saved with our works, but we
cannot be saved without them either. In true salvation, we are doing these good works because of
the Holy Spirit within us.

5:16 Let your light so shine before men, that they may see your good works, and glorify your Father
which is in heaven.

5:17 Think not that I am come to destroy the law, or the prophets: I am not come to destroy, but to fulfil.

It is indicated that the word “fulfill” is the opposite to destroy. Some say that because Jesus fulfilled
the law, they don’t have to obey it anymore. But the opposite to fulfill is destroy, so indicating that it
was not taken away. Fulfill meant to exalt, to uphold.

Mat 5:17-20 [17] Think not that I am come to destroy the law, or the prophets: I am not come to
destroy, but to fulfil. [18] For verily I say unto you, Till heaven and earth pass, one jot or one tittle
shall in no wise pass from the law, till all be fulfilled. [19] Whosoever therefore shall break one of
these least commandments, and shall teach men so, he shall be called the least in the kingdom of
heaven: but whosoever shall do and teach them, the same shall be called great in the kingdom of
heaven. [20] For I say unto you, That except your righteousness shall exceed the righteousness of the
scribes and Pharisees, ye shall in no case enter into the kingdom of heaven.

The rest of His sermon is an explanation on how He will not ‘destroy the law, or the prophets’ but
‘fulfill’ them. This is also a fulfillment of Is 42:21 how Christ will magnify the law and make it
honorable. The law refers to the law of Moses, but keep it mind that law of Moses includes the 10
commandments. The law of God can be split into two parts. The first 4 commandments describing
our love to God and the last 6 describing our love to our brothers and sisters. Of the last 6 Jesus

http://breachrepairers.webs.com/ 164
points out two. Someone these two commandments are at the root of hatred toward each other, and
these two commandments will keep many out of the kingdom of God.

When Jesus mentions “think not,” what does this tell us about the mindset of the hearers and
what they were thinking? There were some there that were thinking that the law and the
prophets had been done away with at the cross. When Jesus mentions the law and the prophets,
what or who is He referring to?

Matt 11:12-13

If all the law and prophets prophesied until John the Baptist, then it must be referring to things
written before that – talking about the Old Testament.

John 1:45

Philip was talking about the Old Testament because he mentioned Moses who is found in the Old
Testament. So when Jesus talks about the law and the prophets, He is referring to the Old
Testament. How did Jesus fulfil the law and the prophets?

Matt 1:21-23

He fulfilled it by being born.

Matt 2:23

He fulfilled it by living in Nazareth.

Matt 8:17

He fulfilled it by healing people.

Matt 13:34-35

He fulfilled it by speaking in parables.

Matt 27:35

He fulfilled it when they parted His garments and cast lots at the cross.

Luke 24:44

Jesus fulfilled all the prophecies of the law and the prophets (the Old Testament) in His life.

According to Matthew 5:18, when will everything be fulfilled? Everything will be fulfilled when
heaven and earth passes away.

When will heaven and earth pass away?

2 Pet 3:10

Heaven and earth will pass away at the day of the Lord as it will come as a thief in the night.

http://breachrepairers.webs.com/ 165
When is the day of the Lord that He will come as a thief?

Isa 13:9

The day of the Lord is when He shall lay the land desolate and destroy the sinners out of it.

Joel 2:31

The sun shall turn into darkness and the moon into blood just before the day of the Lord.

Rev 6:12-14

The sun is turned into darkness and the moon into blood when the heaven is rolled together as a
scroll and every mountain and island is moved out of their places.

Luke 12:39-40

When Jesus comes as a thief, it is referring to His second coming. When will the heaven and the
earth totally pass away?

Rev 21:1-2

The old heaven and earth will totally pass away when God recreates a new heaven and earth. This
will be at the third coming. So the second coming is only a partial fulfillment of the heaven’s and
earth passing away. What is easier to pass – heaven and earth? Or one jot or tittle from the
law? What important application can we draw from this?

Matt 5:18

Luke 16:17

Heaven and earth will pass before even a jot or tittle fails from the law. We have already seen when
heaven and earth will pass – at the second and third coming. Therefore, the law and the prophets
will remain until then.

App: This is strong evidence to show that the Ten Commandments given in the Old Testament are
still valid today. Many people say that Jesus nailed the commandments to the cross when He died,
but we can see from this that that is not true.

What was the law and prophets referring to again? The Old Testament. Therefore, the Old
Testament will still be valid to us at least until the second and third coming of Jesus. Many people
say that the Old Testament is not relevant or valid to us anymore. But that is not what Jesus said. He
said that not even one jot (an iota) or tittle (the least particle of an Hebrew letter) will pass before
heaven or earth does which just shows us that today the Old Testament is still relevant and that
Jesus didn’t do away with it when He came. When He said He fulfilled it, He was saying that He
fulfilled the prophecies of it, but not that He was doing away with the Old Testament.

What is our role in Christ’s fulfillment of the law today?

Gal 5:14

http://breachrepairers.webs.com/ 166
All the law is fulfilled in one word – love your neighbor as yourself.

Rom 3:31

We establish the law through faith in Christ.

John 14:15

If we love Him we will keep the law/commandments. What is more important? They both are
important. You cannot love without expression of keeping the law, and it is impossible to keep
God’s law if we don’t love Him.

5:18 For verily I say unto you, Till heaven and earth pass, one jot or one tittle shall in no wise pass from
the law, till all be fulfilled.

As long as we are here on earth, the law will still exist because the earth is still in existence.

5:19 Whosoever therefore shall break one of these least commandments, and shall teach men so, he shall
be called the least in the kingdom of heaven: but whosoever shall do and teach them, the same shall be
called great in the kingdom of heaven.

“do and teach them” – Jesus said about the Pharisees, do as they say, but not as they do. Jesus is
commending them to do and teach them.

5:20 For I say unto you, That except your righteousness shall exceed the righteousness of the scribes and
Pharisees, ye shall in no case enter into the kingdom of heaven.

The Pharisees only had a form of outward appearance. Not true righteousness.

According to Matt. 5:19, when are we called the least in the kingdom of heaven? When we
break one of the least commandments and teaching others to do so as well. That means when we
break one commandment that we think is so small or so irrelevant we are called the least. Teachers
– take everyone back to the 10 commandments and ask them which one they consider to be the
least. Lying is pretty small in some people’s eyes. Taking God’s name in vain is very little in people’s
eyes.

When we break one, what does the Bible say that we have done? How is that applicable to us
today?

James 2:10

When we break one, we have broken them all. Sometimes we like to compare our sins to
others. And just because we lied but haven’t committed murder or adultery, we think we are
better than them. But in the eyes of God, we are just as great a sinner as the person that
killed someone or that committed adultery.

When will we be considered great in the kingdom of heaven? We will be considered great in the
kingdom of heaven when we do and teach them. We aren’t considered great based upon how much
money we give to the church. We are considered great in the eyes of heaven when we learn to do
and teach the commandments of God.

http://breachrepairers.webs.com/ 167
What is so important about the order that Jesus listed about first doing and then teaching the
commandments? Discuss. It is easy to teach and tell others to keep the commandments. But Jesus
wanted us to do them first and then teach them. If we only teach them and not do them, what are
these people called? Hypocrites.

Why did Jesus have to talk about the law and the prophets before He mentioned the
commandments? What important application can we learn from that?

Matt 5:17-18

Jesus had to talk about the law and the prophets because He had to establish that the Old Testament
was still valid even though He is fulfilling the prophecies of it. The reason He has to establish that is
because when you take away the Old Testament, you have to take away the 10 commandments. So
Jesus was establishing the Old Testament because that was the foundation which the 10
commandments was to stand upon. Today many people are doing away with the Old Testament
saying that it is not relevant or it is old fashioned, and as a result they are doing away with the 10
commandments.

What is the condition upon how we can enter the kingdom of heaven?

Matt 5:20

Our righteousness must exceed the righteousness of the scribes and Pharisees. What is the
righteousness of the scribes and the Pharisees?

Matt 23:1-7

They say and don’t do. They like to be praised by men for their works – meaning, when they do
good deeds they make sure others see them so they can be praised.

Mark 7:5-9

They honor God with their lips but their heart is far from Him. They hold tradition above the
commandments of God.

Luke 15:2

The fact that the Pharisees and scribes pointed out that Jesus ate with sinners means that they
never ate with them. So they only ate with those that they thought were righteous. What does the
Bible define as righteousness that will allow us to enter the kingdom of heaven?

Matt 3:2

We must repent.

Matt 7:21

We must do the will of the Father.

Matt 18:3

http://breachrepairers.webs.com/ 168
We must be converted.

John 3:5

We must be born of the water and of the spirit.

Matt 5:3, 10

We must go through the steps of the beatitudes. So if we want to enter the kingdom of heaven, we
must have all of these characteristics. If we want to be righteous we must repent, do the will of the
Father, be converted, be born of the water and of the spirit. The beatitudes simply outline the steps
in how we can be righteous. What else is righteousness connected to?

Ps 119:172

Righteousness is connected to the commandments.

How can we fulfill this righteousness in our lives today that it may exceed the righteousness
of the Pharisees?

Rom 13:10

Love will fulfill the law.

Gal 5:14

The law is fulfilled in this word – Love. The one main thing that was lacking in the scribes and
Pharisees was a love for God and a love for others. Because they lacked this, they lacked the very
thing needed to be a righteous person. Do you have love in your hearts today for God and for
others?

5:21 Ye have heard that it was said by them of old time, Thou shalt not kill; and whosoever shall kill shall
be in danger of the judgment:

This is the sixth commandment.

Ex 20:13

Verses 22-26 are an explanation and expansion of the sixth commandment.

What does Jesus put on the same level as killing someone?

Matt 5:21-22

Hating your brother without a cause. Both have the same consequences – they shall be in danger of
judgment.

1 John 3:15

Whoever hates his brother is a murderer. So if we hate our brother, we are a murderer. What is the

http://breachrepairers.webs.com/ 169
difference between hating someone and killing someone? Is there a difference? Why does
the Bible put it on the same level? Discuss. The difference between someone who hates and
someone who murders is that the person who hates lacks opportunity or is too scared to perform
the act. There is no difference between the two. One just lacks the opportunity. But the heart is the
same. The Bible puts it on the same level because Jesus is not dealing with just our acts, He is
dealing with the root of the problem – our heart.

If you hate someone, who do you belong to?

John 8:44

You are a child of the devil if you are a murderer because he was a murderer from the beginning.
What else is put on the same level as murdering and hating your brother? Why do you think
they are categorized together?

Matt 5:22

Calling your brother Raca (which means worthless or vain fellow) or a fool is categorized in the
category with hating your brother. In some respects, calling your brother a fool is much worse,
because the consequence is worse – being in danger of hell fire. Leading question for the second
part: Why is hating your brother, calling your brother Raca (worthless) or calling him a fool all put
together on the same level? It affects our perception of how we view our brother or sister. Makes us
to think bad of them. And no one is worthless or a fool in the eyes of God. And God doesn’t hate
anyone in His heart either. God is trying to deal with our heart, how we think and view others.

According to Matt. 5:23-24, who should do the reconciling? Compare this to how we operate
today in this world. The person that should do the reconciling is the one that realizes that his
brother has something against him. Today in this world, when someone has a problem with us, we
don’t usually approach him. Because it is not our problem so we usually wait for them to approach
us. But that is not the way a Christian should deal with those that hate us or have something against
us.

What does the Bible define as reconciliation?

Eph 2:16

To be in one body again, to unify or unite.

Col 1:20-21

To make peace between two parties, to not be aliens or enemies anymore. How were we enemies
of Christ or separated from Him?

Jam 4:4

We were enemies of Christ because we are friends with the world.

Isa 59:1-2

We were separated from Christ by our sins.

http://breachrepairers.webs.com/ 170
How did Christ show us an example of this reconciliation? And why did He do what He did?

Rom 5:7-10

We are hesitant to even die for a righteous man, never mind a sinner. Christ showed us this example
of reconciliation by dying for us while we were yet sinners, while we were enemies of Him. So who
had the problem? We did. We were sinners. We hated Him. We crucified Him. Who reconciled?
Christ did. Why did He do what He did? Because He loved us.

According to what we have studied, if our neighbour hates us, how far should we go to be
reconciled to them?

1 John 3:13-16

We ought to lay down our lives for those that hate us as well. Are you willing to die for your enemy?

How can we practically apply dying for our enemies today that we may truly be reconciled to
those that hate us?

1 John 3:17-18

Rom 12:19-21

When we see them hungry, feed then. When we see them thirsty, give them drink. This is
how we may be reconciled back to them, but sacrificing for them even though they hate us.
We think this is easy, but it isn’t. Why? Because it’s not so much dealing with just actions but
our motives and our heart.

When Jesus said at the beginning, “Ye have heard that it was said by them of old time,” was
He really teaching anything new?

Lev 19:17-18

Do not hate your brother in your heart, but love your neighbour as yourself. No, He wasn’t teaching
anything new. He was just reiterating what was taught in the past. It’s just that many people had
forgotten about the wickedness of their own heart.

How can our righteousness exceed the righteousness of the Pharisees based upon the study
that we have learnt today? Discuss. The Pharisees kept the letter of the law but not the spirit of
the law, which is love. Many of us don’t outwardly kill people, but in our hearts we do. Also, in order
for our righteousness to exceed that of the Pharisees, we need to learn to love those that hate us
and be reconciled back to them by dying for them. Loving the unlovable. So when we keep the law
of God we need to keep it from our hearts which will be automatically be expressed in our words
and actions. So in this teaching, Jesus was dealing with both sides of the coin: Those that hate
without any good reason or holding grudges. And those that are hated by others and how we should
deal with them in reconciliation.

5:22 But I say unto you, That whosoever is angry with his brother without a cause shall be in danger of
the judgment: and whosoever shall say to his brother, Raca, shall be in danger of the council: but
whosoever shall say, Thou fool, shall be in danger of hell fire.

http://breachrepairers.webs.com/ 171
“angry with his brother without a cause” this shows that the sixth commandment condemns us
being angry with each other, and it is the first time Christ introduces the topic of ‘the judgment’ and
mentions ‘hell fire’.

“whosoever is angry with his brother without a cause shall be in danger of the judgment” –
There is a time to be angry. Mistreatment and genocide. Can we be angry without violating His
principles? Yes. We can be jealous for His cause. “Raca” and “fool” are derogatory expressions.

5:23 Therefore if thou bring thy gift to the altar, and there rememberest that thy brother hath ought
against thee;

Verses 23 and 24 teach reconciliation; once you reconcile you are in harmony with the sixth
commandment

5:24 Leave there thy gift before the altar, and go thy way; first be reconciled to thy brother, and then
come and offer thy gift.

We are being taught to reconcile with our brother even though they have some occasion against us.
If we know that, we HAVE to go to them and ask them.

5:25 Agree with thine adversary quickly, whiles thou art in the way with him; lest at any time the
adversary deliver thee to the judge, and the judge deliver thee to the officer, and thou be cast into prison.

Verses 25 and 26 provide practical guidance on how to deal with someone who is angry with you
and avoid prison. Many will go to prison because they will not agree with their ‘adversary’ and
adversary brings accusations. And if you are guilty and don’t admit it, then you go before a ‘judge’
and the judge sees that the accusations are correct and you go to the officer and then to prison.

5:26 Verily I say unto thee, Thou shalt by no means come out thence, till thou hast paid the uttermost
farthing.

5:27 Ye have heard that it was said by them of old time, Thou shalt not commit adultery:

Mat 5:27-32 [27] Ye have heard that it was said by them of old time, Thou shalt not commit adultery:
[28] But I say unto you, That whosoever looketh on a woman to lust after her hath committed
adultery with her already in his heart. [29] And if thy right eye offend thee, pluck it out, and cast it
from thee: for it is profitable for thee that one of thy members should perish, and not that thy whole
body should be cast into hell. [30] And if thy right hand offend thee, cut it off, and cast it from thee:
for it is profitable for thee that one of thy members should perish, and not that thy whole body
should be cast into hell. [31] It hath been said, Whosoever shall put away his wife, let him give her a
writing of divorcement: [32] But I say unto you, That whosoever shall put away his wife, saving for
the cause of fornication, causeth her to commit adultery: and whosoever shall marry her that is
divorced committeth adultery.

This is the seventh commandment. Verses 28-32 are an expansion of the seventh commandment.
These verses are about not committing adultery through: Divorce or lust. When does a thought
become a sin? When you cherish the thought and it turns into something more. When you choose to
think about it again and again and again. Spiritual Application: Adultery is the unholy union
between the two parties. Love not the world

John 2:15

It is literal when it says to “pluck it out” – your eye? Could you have one eye and still lust? – Of

http://breachrepairers.webs.com/ 172
course. You can still do wrong actions. What do the eye and the hand represent? The eyes is
connected to our thoughts and the hand is connected to actions, If you are having trouble with
thoughts and actions, then what should we do? Whatever the root of the temptation might be – get
rid of it.

There is only one Bible reason for divorce. It is for adultery – when one party has been unfaithful.
How about abuse? It is not a grounds for divorce according to the Bible.

What does Jesus equate committing adultery to?

Matt 5:27-28

Jesus equates committing adultery to a man lusting after a woman in his own heart. What is the
difference between lusting and a passing thought or look? A passing thought or look is
something that you pass by and your eyes behold, and if it is evil you cast your eyes away from it
(ie. All those horrible bill board that you may pass in your car, or advertisements that appear on
TV). To lust means to set your heart upon it, to long for, to covet or desire.

Is there a difference between actually committing adultery and lusting after a woman?
Discuss. There really is no difference other than one had the action and the other didn’t. But for the
person that was lusting after that woman, if he had the opportunity, he would have. This is exactly
the same as the killing and hating your brother (Matt. 5:21-24). One lacked the opportunity for it to
come to pass, or they were too scared to put it into action.

What is Jesus really trying to address here about adultery?

Jer 17:9

The heart is deceitful and desperately wicked.

Matt 12:34

Out of the abundance of the heart, the mouth speaks. Jesus is trying to address the issues of the
heart, the fountain and root of all the problems of our actions. If our heart was right, our actions
would be right too.

Prov 23:7

As we think in our heart, so are we. Our thoughts ultimately lead us to sin. Many people think that
just because they don’t commit adultery or kill they are fine. But they don’t realize that they are the
same condition as those that do commit such acts. What is Jesus’ solution to our sinning ways of
our eyes and hands?

Matt 5:29-30

If we can’t stop sinning with our eyes of hands Jesus says we should pluck it out or cut it off.

Why does Jesus mention about the eyes and hands?

Matt 6:22-23

http://breachrepairers.webs.com/ 173
The light of the body is the eye. It is our eyes that dictate to a great degree how our body reacts. And
it is also our eyes that give light or darkness to our minds, which is the very thing that controls our
bodies. Jesus mentions eyes, because it is the very thing that will affect our mind and ultimate our
actions.

Psa 26:10

Our hands are full of mischief.

Prov 6:17

Our hands shed innocent blood. Our hands represent our filthy actions. Actions that lead us to sin.

What did Jesus really mean when he said to pluck out our eyes and cut off our hands? Are we
to do that today literally?

Rom 6:6, 11-13, 19

What happens to a body part after it is cut off? It dies. Therefore Jesus was talking about spiritually
killing the old man. The body part that causes you to sin is part of the old man. It must be destroyed.
What are the two key action words that we read in the Bible text other than killing? Reckon and
yield. We must learn to surrender our will to God. Our eyes and our hands are necessary parts of
our bodies. Because we are all sinners, if we did that, all of us would be disabled today. So Jesus isn’t
meaning that in a literal sense. But our hands and our eyes represent in a way those things that are
very necessary to us in our lives. Eg. If our computer leads us to sin, then we should throw that
away or put it in a inconvenient place in the house so that you don’t sin. If there are certain things in
our lives that lead us to sin, that maybe are almost essential, then we should learn to do without. Eg.
If taking the train leads us to sin, then we should drive the car even though it is more expensive.
What is more important? Saving the money on train ticket or your eternal salvation. TEACHERS ask
this question – are there things in your life that cause you to sin that you know you should throw
away or get rid of? Maybe it is expensive – big screen TV? Etc… Jesus wants us to weigh up
everything on this earth in the light of heaven. He doesn’t want us to put things of this earth over
things in heaven. Sometimes it is almost like cutting off our hands or plucking out our eyes because
it is a big sacrifice to us.

How often should we cut off our sinful body parts?

1 Cor 15:31

We should die daily. Everyday in our devotional life we should ask God to kill our sinful body parts.
Therefore, it is NOT when the temptation comes, but on a daily basis before we start the day, we
must acknowledge our sinful body parts and ask God to help us reckon and yield because we know
that Satan will surely bring the temptation.

Even though we are not married or do not lust after a woman, how else are we adulterers
according to the Bible?

Jas 4:4

http://breachrepairers.webs.com/ 174
By being a friend of the world, through looking and lusting and touching the things of the world.

Instead of looking at the things of the world, what should we behold?

John 1:29 ….behold the Lamb.

Heb 12:2 …looking unto Jesus.

2 Cor 3:18 …by beholding Jesus we will be changed into the same likeness.

SUMMARY – things we should do daily: Cut off our body parts by reckoning ourselves dead and
yielding through surrender. Behold and look unto Christ.

5:28 But I say unto you, That whosoever looketh on a woman to lust after her hath committed adultery
with her already in his heart.

5:29 And if thy right eye offend thee, pluck it out, and cast it from thee: for it is profitable for thee that
one of thy members should perish, and not that thy whole body should be cast into hell.

This verse is especially related to adultery. If your right eye offend you by looking on a woman to
lust after her being a married man is adultery. If you are single it is fornication. Verse 29-30 gives
the solution. Pluck it out. Jesus give problem, then solution.

“whole body should be cast into hell” this shows two reasons why people will go to hell.

5:30 And if thy right hand offend thee, cut it off, and cast it from thee: for it is profitable for thee that one
of thy members should perish, and not that thy whole body should be cast into hell.

5:31 It hath been said, Whosoever shall put away his wife, let him give her a writing of divorcement:

5:32 But I say unto you, That whosoever shall put away his wife, saving for the cause of fornication,
causeth her to commit adultery: and whosoever shall marry her that is divorced committeth adultery.

5:33 Again, ye have heard that it hath been said by them of old time, Thou shalt not forswear thyself, but
shalt perform unto the Lord thine oaths:

What did Jesus mean when He said “thou shalt not forswear thyself?” What does the word
forswear mean? God does not make your keep your oaths because He saved you out of something.
He does not work like that. This is an expansion of the third commandment. One way to take the
name of the Lord God in vain is by swearing in His name

Ex 20:7 Thou shalt not take the name of the LORD thy God in vain; for the LORD will not hold him
guiltless that taketh his name in vain.

Lev 19:12 And ye shall not swear by my name falsely, neither shalt thou profane the name of thy
God: I [am] the LORD.

We should make promises to each other and leave the Lord’s name, dwelling place, His creation, out
of it. Otherwise it would lead to evil. Verses 34-37 expand this thought. This is the text that Jesus
was referring to and He was talking about swearing falsely, or taking an oath falsely. That was
equivalent to profaning the name of God. Forswear in the concordance means to commit perjury
which means to give a false testimony or false witness.

http://breachrepairers.webs.com/ 175
What did Moses teach about swearing or taking an oath? Was it OK to swear?

Num 30:2

Deut 6:13

Deut 10:20

Moses taught the Israelites that it was OK to swear by God’s name. How is this different to what
Moses was teaching the Israelites and what Jesus was dealing with? Moses was teaching the
Israelites to tell the truth under oath. However, Jesus was dealing with the issue of lying under oath
– making promises that they did not keep. This is already taking place in the Old Testament:

Isa 48:1

They swear by the name of God but not in the truth or in righteousness. Moreover, He is
dealing with people that swear by God or objects that they are telling the truth when they
are not.

Was Jesus trying to destroy oath taking altogether, or is there still room for the judicial oath?

Matt 26:63-64

If Jesus was condemning the judicial oath, He would have reproved the high priest for making such
a statement instead of giving him an answer. So there is still room for the judicial oath in which God
is solemnly called to witness that what is said is truth and only the truth. What did Jesus mean
that our communication should be yea, yea; nay, nay?

Jas 5:12

Let your yea be yea, and your nay be nay. Our communication should be clear and precise and we
should mean what we say and not halt between two opinions. Many people say yes but they really
mean no or vice versa. They don’t mean what they say.

2 Cor 1:17-18

Our word toward you was not yea and nay. The ministering to the unbelievers, Paul told them that
his communication was just as he said it. He didn’t mix a yea with a nay. He didn’t mix truth with
error. He didn’t say yes and then no. It was clear and precise. What does the Bible say about how
we should communicate with our speech?

Col 4:6

Our speech should always be with grace and seasoned with salt

Matt 5:13

How can we be salts of the earth? By what we say. Our communication can be a great blessing to
people and it can also be a great curse to people. RELFECTION QUESTION: Does your speech always
show that you are serving the true God? Can people tell that you are a Christian just by the way you
speak?

http://breachrepairers.webs.com/ 176
Eph 4:29

Let no corrupt communication proceed out of your mouth Nothing vile or profane should come out
of our mouths. God desires that we should use our speech to edify others and minister grace to
those that hear us. When Jesus talks about our communication, what is He really addressing?

Matt 12:34

Out of the abundance of the heart the mouth speaks. What we say is just a fruit of the condition of
our heart. So once again Jesus is dealing with the heart issue. He is trying to tell us that it is not just
in outward forms and a mere keeping of the commandments that we should obey, but that we
should obey from the heart. What has Jesus been addressing in the previous passages after the
beatitudes and what is the common theme that runs through them all?

Matt 5:17-20

The law

Matt 5:21-26

The law – thou shalt not kill and being angry at your brother (a heart issue)

Matt 5:27-32

The law – thou shalt not commit adultery in the heart

Matt 5:33-37

The law – thou shalt not lie and our communication which stems back into out heart. The common
theme that runs through it all is the law and how Jesus is trying to deal with our hearts. The
spirituality of the law. The people such as the Pharisees could keep the law in an outward form, but
they lacked the true spirit of keeping the law. What does Jesus desire to do with the law today?

Heb 10:16

Jesus desires to write the law in our hearts today that the keeping of the law will not just be an
outward form but that we will truly keep it form the heart.

Psa 40:8

When we have the law in our heart, we will delight in it. We will be happy in obeying it. Today are
you keeping the law because it is in your heart, or are you keeping the law out of obligation or
tradition? Jesus desires us to be happy. You can be happy today as well if you have the law within
your heart.

5:34 But I say unto you, Swear not at all; neither by heaven; for it is God's throne:

5:35 Nor by the earth; for it is his footstool: neither by Jerusalem; for it is the city of the great King.

5:36 Neither shalt thou swear by thy head, because thou canst not make one hair white or black.

http://breachrepairers.webs.com/ 177
5:37 But let your communication be, Yea, yea; Nay, nay: for whatsoever is more than these cometh of evil.

In our dealings with people, we should make it very simple. A “yes” or a “no”

5:38 Ye have heard that it hath been said, An eye for an eye, and a tooth for a tooth:

Verses 39-42 expand on this verse. It is an expansion of the law of Moses. In our modern day
language, what does it mean to give an eye for an eye, and a tooth for a tooth? It means to exact
justice on people for what they’ve done to you. To give back to them exactly what they did to you.
Equality in justice.

When Moses gave this law, in what context was he giving it for?

Lev 24:17-22

Moses was giving the law to establish order and laws for the country so that no matter who entered,
stranger or citizen, all were to abide by the law of that land. It was civil law. Specifically Lev. 24:22
says that any person, even a stranger was to abide by such laws. So this is the laws of the
land/country.

What offences are these texts in Leviticus applied to specifically? Was Jesus dealing with the
same issues that were mentioned there? The offences that were mentioned in Leviticus were
referring to such that killed a person or that permanently destroyed a person’s goods or that caused
a blemish on the person. Jesus was dealing with something totally different. He was dealing with
someone that slapped them on the cheek, or took their coat, or told them to run a mile.

According to Jesus’ comments, how do you think the people were applying it in their day?
They were using the civil law that Moses instituted to exact revenge on every petty thing that
people did to them. If they were hit on the cheek, they would retaliate and hit back on the cheek. If
they had their coat taken away, they would try to retaliate by taking away their enemies coat.

Instead of exacting justice on our enemies and those that deserve it, what does Jesus advise
that we should do? Jesus’ advice is to go beyond what they ask. Go the extra mile. If they slap you
on one side, instead of retaliating give them your other cheek. If they take your coat, give them your
cloak also. If they ask you to run a mile, run two.

What is the pattern that you can find according to the severity of the actions done?

Matt 5:39 …smite…

Matt 5:40 …sue…

Matt 5:41 …compel…

Matt 5:42 ….ask…

It is going from most severe to simply asking. What is the pattern that you can find in how we
should respond in our actions?

Matt 5:39 …turn….

http://breachrepairers.webs.com/ 178
Matt 5:40 …let…

Matt 5:41 …Go…

Matt 5:42 …Give…

It is going from a simple action such as turning, to having to invoke a deep response as to whether
we will give or not. If you try to summarize these verbs of our responses how would you
categorize them? They would be categorized at selflessness and submission. If we wanted to
protect ourselves we would do the opposite.

Compare the severity of the action done to a person to the response that we should give to
that action. What important lesson can you learn from that?

Matt 5:39 …smite…turn…

Matt 5:40 …sue…let…

Matt 5:41 ….compel…go….

Matt 5:42 …ask…give…

Which is the most severe action? Smite (That is without a doubt). Which is the hardest to do in
response? Turn or give? I would say give. Yes when someone smites you it is hard to turn. But on an
individual basis it is easy to turn. But it’s even harder to give not knowing whether you will get it
back or not. It requires a pro-active approach to give, but it almost seems like turning is very
passive. So the more the severe the action done, the less proactive we should be in responding. Yet
when someone asks, we should give. The important lesson that we can learn from that is, if we don’t
learn to give first when someone asks, we will never be able to turn when someone smite’s us.
Turning is easier than giving, yet smiting is more severe than asking. We need to learn the lesson of
benevolence first.

So how do we not resist evil today when we are treated unfairly?

1 Pet 2:23

When he was reviled, Jesus didn’t retaliate. But He committed himself to him that judgeth
righteously. What does it mean that we commit ourselves to him that judges righteously? When we
commit ourselves, the word commit means to surrender, to yield, to intrust ourselves to that
person. But we must understand who we are committing ourselves to. The Bible didn’t mention
Jesus or God or Saviour. It is to the person that judges righteously, or that judges fairly. Was Jesus
really teaching anything new? What did the Old Testament say about how we should treat
those that hurt us?

Prov 20:22

Don’t recompense evil for evil.

Prov 24:29

http://breachrepairers.webs.com/ 179
Don’t say you will do to him as he has done to you.

Prov 25:21-22

If your enemy is hungry give him bread to eat.

Rom 12:18-21

A repeat of Prov. 25 but it tells us that vengeance is God, He will repay, we do not need to avenge for
ourselves. Basically, do not treat them as they have treated you. Jesus wasn’t teaching anything
new. Have there been people in your life that have treated you unfairly? How have you responded?
Have you committed yourself to God knowing that He will judge righteously? Or have you retaliated
thinking that God will not avenge? Let us commit ourselves to God today that we may use our
submission and spirit of non-retaliation to reveal the character of Christ to them and to win them
over to the Saviour.

5:39 But I say unto you, That ye resist not evil: but whosoever shall smite thee on thy right cheek, turn to
him the other also.

5:40 And if any man will sue thee at the law, and take away thy coat, let him have thy cloke also.

5:41 And whosoever shall compel thee to go a mile, go with him twain.

5:42 Give to him that asketh thee, and from him that would borrow of thee turn not thou away.

5:43 Ye have heard that it hath been said, Thou shalt love thy neighbour, and hate thine enemy.

What do you have to do in order to earn God’s love? God’s love is unconditional. In order to be
perfect, we need to have this love that is described.

BE YE THEREFORE PERFECT

Read Matt. 5:43-48 and answer the following questions.

According to this passage, what qualifies us to be called the children of our Father in heaven?
When we love our enemies. Blessing them that curse us. Doing good to those that hate us. Praying
for those that despitefully use us and persecute us.

If God expects those qualities of His children, what does it imply about the Father?

1 John 3:2

When we see God, we shall be like Him, we shall see Him as He is. God doesn’t demand something of us or
expect something from us that He Himself hasn’t first done. So if God expects us to love our enemies, He must
have demonstrated it somewhere first.

How did God demonstrate His love towards enemies?

Rom 5:6-8

God sent His Son to die for the ungodly, for sinners. We already find it difficult to die for a righteous
person, how much more difficult would it be to die for your enemy? Someone that hates you?

http://breachrepairers.webs.com/ 180
Luke 23:34

While Jesus was on the cross, He prayed for those that reviled Him and persecuted Him.

Why does Jesus bring in the illustration of how God sends the sunshine and the rain? What
characteristic is He trying to imply about God? Jesus is trying to emphasize the fact that Jesus is
not impartial to anyone. He doesn’t favor a righteous person more than an unrighteous person.
Both wicked and evil He gives the blessings of rain and sunshine. So we should not be impartial to
anyone either.

What does James say about being partial and favoring one person over another?

Jas 2:1-4, 9

James is saying that if we are partial, we are committing sin. Why does Jesus use the publicans as
an example? Who was listening to this sermon and how did they regard publicans?

Matt 4:25

Basically it was all the Jewish people, including the Pharisees and scribes. How did the Jews regard
the publicans?

Luke 15:1-2

They classified them in the same category as the sinners.

Luke 19:2, 5-7

They out rightly called Zacchaeus a publican and a sinner. So not just in the same category but they
called them sinners.

Matt 18:17

They were considered Heathen’s. People that didn’t believe in God. So why does Jesus use the
publican’s as an example? Basically He’s saying that if we treat those around us nice that treat us
nice, we are no better than a sinner and a heathen person. Even the worldly person knows how to
do that. We are no better than them if we do that.

Therefore, when Matthew tells us to be perfect, contextually what does that mean? Discuss.
To be perfect means to be perfect in love. But not just to love those that love you, but to love your
enemies and those that hate you and curse you.

Why is love so important? Why does Jesus emphasize this so strongly at the end of this
portion of His sermon?

Rom 13:10

Love is the fulfilling of the law. If we love Jesus, all that He talked about in Matthew 5, going beyond
the mere letter of the law, will be easily fulfilled. We will exceed the righteousness of the Pharisees.

http://breachrepairers.webs.com/ 181
Why? Because they did it as a mere form, not form the heart.

1 Cor 13:1-3, 13

Paul makes it clear that love is the most important of everything. And in everything we do, if our
motivation is not because we love then our motivation is wrong. We may do the right things, but
some of us do good things to be praised by others, or to come across in a certain way when we
really aren’t. What is your motivation for doing good works? To be praise of men? Because if you
don’t learn to love your enemy, your love for God is not totally complete. As the saying goes, you
love God as much as the person you love the least. And if you don’t love that person, you hate them.
And in Matthew it says that if we hate our brother or are angry at them without a cause, we have
committed murder (Matt. 5:22).

What has Jesus really been trying to deal with in this portion of His sermon so far?

Matt 5:17

At the beginning He told that He did not come to destroy the law and the prophets. And in the rest
of the sermon He addresses the law and how they have been obeying it wrongfully. They didn’t kill,
but they hated their brother. They didn’t outwardly commit adultery, but they lusted after women
in their hearts. Jesus was dealing with heart issues.

Matt 22:36-40

Jesus was dealing with the law – loving God and loving your neighbour. It’s all about love. How do
we get this love according to everything that we have studied in Matthew 5? We need to go
through the steps of the beatitudes at the very beginning. Because it ends with telling us to rejoice
when we are persecuted. They are steps to happiness. But they are also steps to teach us how to
truly love God and love our neighbour.

5:44 But I say unto you, Love your enemies, bless them that curse you, do good to them that hate you, and
pray for them which despitefully use you, and persecute you;

5:45 That ye may be the children of your Father which is in heaven: for he maketh his sun to rise on the
evil and on the good, and sendeth rain on the just and on the unjust.

5:46 For if ye love them which love you, what reward have ye? do not even the publicans the same?

5:47 And if ye salute your brethren only, what do ye more than others? do not even the publicans so?

5:48 Be ye therefore perfect, even as your Father which is in heaven is perfect.

http://breachrepairers.webs.com/ 182
Chapter 6
6:1 Take heed that ye do not your alms before men, to be seen of them: otherwise ye have no reward of
your Father which is in heaven.

What does alms mean? (Check the middle of your Bible). Alms = righteousness. From STRONGS;
compassionateness, that is, (as exercised towards the poor) beneficence, or (concretely) a
benefaction: - alms (-deeds). Basically alms = good works.

Which particular group of people was Jesus particularly referring to as hypocrites? Who do
they represent today according to Mark 7?

Matt 6:2

Matt 23:13-15

Mark 7:5-6

Christ is referring to the Scribes and Pharisees. From Mark 7:5-6, they particularly represent those
who worship with their mouths but not with their hearts, which is far from God. Application: What
does it mean to honor God with our lips and not our hearts? Ask the group for examples. One
example = come to church on Sabbath to worship, but in our hearts be thinking about the sports
results or talking with our friends about the latest fashion, etc.

With the above definition of a hypocrite in mind, contrast what it means to be a hypocrite vs
Christ’s true follower in doing alms?

Matt 6:1-4

Hypocrite: will try to do their alms (good works) in public. True follower of Christ: does alms in
secret. Application: What does it mean to do alms in secret? Are we talking about hiding our good
works?

What is the difference between the intentions of good works in Matt 5:16 and Matt 6:2? What is
Jesus emphasizing when he asks us to do alms in secret?

Matt 5:16

Matt 6:2

Matt 5:16 tells us to do our good works for the glory of God, but in Matt 6:2, Jesus is saying the
Pharisees or hypocrites are doing good works for their own glory. What is Jesus emphasizing?
Doing alms in secret = doing things for God’s glory.

In the context of doing alms, what does it mean to give glory to God?

Matt 6:4

What does it mean to do things for God’s glory? To seek HIS reward and recognition, and NOT
man’s. CONCLUSION: Jesus is pointing to the MOTIVE in our hearts. Who do we really want
recognition from, man OR God? What is usually our motive for good works? (ask group). Our own

http://breachrepairers.webs.com/ 183
glory. So that is why we tend to do things so that everyone can see. Eg. We get so hung up because
we didn’t get an opportunity to go up on the pulpit and be recognized for our “alms”, and that we
have to settle for the little things behind the scenes. According to this passage, only hypocrites will
get upset if they are left with the “invisible” jobs. Why are they upset? Because their motive was
wrong in the first place.

What did the hypocrites do during Jesus’ time with their prayers?

Matt 6:5, 7

1. Love to pray in public. 2. Vain repetition. What does vain repetition mean? Eg. Heathen prayers
(Buddhist, Hindu) generally repeat the same words over and over again. They are also very
monotone. We have to careful that our prayers are not the same. Why? Because our God is a living
God, an intelligent God. Example: Just like talking to a friend, can you imagine using his name over
and over again, or repeating the same phrase to them. They will think you are insulting their
intelligence.

What was the problem with the public prayers of the Pharisees in Jesus’ time?

Luke 18:10-13

The Pharisee was more interested in self praise and self glory. Can you imagine shouting these
words from the corners of the streets? CONCLUSION: When we use vain repetition, or self praise to
God in public, or repeat private things in public, we are not giving glory to God, but to man.

Was Jesus discounting public prayer?

Acts 1:14-15

Acts 4:31-32

No. The disciples continued to pray together in public. Acts 1:14-15 says there were 120 people.
If public prayer was not ordained by Christ, the Holy Spirit would not have been poured out.
What is the difference? There is a place for secret prayer and a place for public prayer. However, the
Pharisees were using public prayer as an occasion for self praise and self glory.

What does praying to the Father in secret imply? Who needs to be involved?

Matt 6:6

It simply means there is no one else around. Therefore, Jesus is saying there is no need for a human
mediator between God and man. All prayer whether secret or public is to be directly to God the
Father and does not need to go through anybody else. That means we need only confess our sins to
God, NOT man. Some religions ask us to confess our sins to priests or holy men.

Contrast what a hypocrite and a true follower does in respect to fasting? What does it mean
to fast in secret?

Matt 6:16-18

Hypocrite – puts on sad countenance and disfigured face. What does this mean? Show how painful

http://breachrepairers.webs.com/ 184
and torturous it is when fasting. True follower – anoint thine head and wash thy face, appear not
unto men to fast. What does this mean? You can’t tell when a true follower is fasting. He should be
just as cheerful and not complaining about being hungry. What does it mean to anoint thine head
and wash they face? Basically to look fresh, healthy, strengthened. Not go out of the way to look
weak, weary and hungry.

What is the phrase that is being constantly repeated in regards to alms, prayer and fasting?
What was Christ emphasizing?

Matt 6:4, 6, 18

Do good works, prayer and fasting in secret, because the Father seeth in secret and shall reward
openly. What does it mean to do things in secret? Our motive should always be to look for God’s
glory and not our own. Therefore, we should not need to seek public recognition for these 3 actions
(good works, prayers, fasting). Bottom line, God is interested in our attitude and motives for
service. This will come out in how we do good works, how we pray and how we fast. These are
indicative of the condition of our heart and its desires. Remember, in God’s time, He will reward us
openly.

6:2 Therefore when thou doest thine alms, do not sound a trumpet before thee, as the hypocrites do in
the synagogues and in the streets, that they may have glory of men. Verily I say unto you, They have their
reward.

6:3 But when thou doest alms, let not thy left hand know what thy right hand doeth:

When we do something good, naturally people want to receive acknowledgement. We should do


our works in humility.

6:4 That thine alms may be in secret: and thy Father which seeth in secret himself shall reward thee
openly.

6:5 And when thou prayest, thou shalt not be as the hypocrites are: for they love to pray standing in the
synagogues and in the corners of the streets, that they may be seen of men. Verily I say unto you, They
have their reward.

When you pray out loud, it helps your mind not to wander. Vain repetitions make a prayer or song
have no meaning sometimes.

6:6 But thou, when thou prayest, enter into thy closet, and when thou hast shut thy door, pray to thy
Father which is in secret; and thy Father which seeth in secret shall reward thee openly.

6:7 But when ye pray, use not vain repetitions, as the heathen do: for they think that they shall be heard
for their much speaking.

6:8 Be not ye therefore like unto them: for your Father knoweth what things ye have need of, before ye
ask him.

6:9 After this manner therefore pray ye: Our Father which art in heaven, Hallowed be thy name.

This prayer highlights that we must live day by day. We must ask for strength day by day. God
works with us one day at a time.

http://breachrepairers.webs.com/ 185
Which person of the Godhead should we address when we start our prayers? We should
address the Father, and not Jesus or the Holy Spirit. It is God the Father that answers our prayers,
but it is Jesus that pleads on our behalf.

Just as we understand our parents love towards us, Jesus also wanted us to know that when we
pray, we pray to a Father, someone who cares deeply for us. Not as one that approaches a King or a
God or a Creator that we must kneel and beg.

When we call God our Father, what does that imply in our relation to Jesus and also to each
other?

Heb 2:11

We are all brothers and sisters, including Christ. We are all members of one family and so we pray
for our neighbours as we pray for ourselves.

What is the first thing that Jesus instructs us to pray for? How would that change our
perspective in this life by praying that? He asks us to pray for the coming of His kingdom. That
would change our perspective in a way that by saying that it makes us personally responsible for
the fulfillment of those words. That we would go throughout the day to hasten His kingdom.

What is the one way that the Lord’s prayer suggests that we can hasten the coming of His
kingdom?
The way to hasten His kingdom to come is that His will be done on earth as it is in Heaven. If the
will of God is done in earth as in Heaven, then there will be no difference between heaven and earth.
Then Jesus can come again! What is the will of God?

1 Thes 4:3

The will of God is our sanctification, to live a holy life, to be separate from the world. Therefore, one
way that we can hasten the coming of His kingdom is simply living a holy life here on this earth so
that we are ready for Him to come again.

Why is it important for us to ask God to give us our daily bread? How does that change our
perspective on our own lives? By asking God to give us our daily bread we are recognizing that it
is God that puts the food on our tables and fills our stomachs with food. By praying this we are
giving thanks to the Lord for feeding us. This will help us not to become self sufficient thinking that
we are the ones that provided.

What is the important spiritual application behind asking for the daily bread?

Matt 4:4

We must have the daily bread from heaven, the word of God, which is tied in with doing the will of
God in earth as in heaven. Spending time in the word of God will also stir up in our hearts the need
to hasten His kingdom to come.

To what extent should we be asking that the Lord forgive us? How many of us really ask the
Lord for forgiveness in this way? That He forgives us as much as we forgive others. Maybe we are
scared to pray such a prayer because we know that we have not forgiven our brother or sister out

http://breachrepairers.webs.com/ 186
there. Is there anyone you know that you need to forgive today?

Why does Jesus make forgiving others a condition for Him to forgive our sins?

Luke 7:47

It will help us to understand and appreciate the mercy and love of God in forgiving our sins and as a
result, love and mercy should spring up from within us. It is easy for a sinless God to forgive our
sins, but how much easier should it be for us, who are sinners, to forgive those that have done just
as much wrong as we have. Forgiveness breeds love. And those that don’t forgive others show that
they don’t love them much because they forgot their past sins and how they have been forgiven.
May God help us to be merciful to our brother’s and sisters today.

Contextually speaking, why do we end saying “Thine is the kingdom, and the power, and the
glory?”

Matt 6:13

Jude 24, 25

We must recognize that the power comes from God and is available to us now and forever to keep
us from falling, to keep us from temptation and to deliver us from evil.

How should we end our prayers and why is it important to end this way?

John 14:13-14

John 15:16

Acts 4:12
We must learn to ask in the name of Jesus, and not simply end with the words Amen. Why? Because
it is through this name that we are saved, and it is through Jesus Christ that we are able to present
our petitions before our Heavenly Father.

Overall, what was the main emphasis that Jesus was trying to bring out about prayer at the
end? He recapped in verse 14 and 15 a very important point that He mentioned in the prayer itself,
and that was forgiveness. That we are to recognize that God would only forgive our sins as much as
we forgave others. If there was anything that He wanted His disciples to remember about the
prayer it was about forgiving each other. That was the very last thing that Jesus prayer for when He
was hanging on the cross, for forgiveness of His enemies.

6:10 Thy kingdom come. Thy will be done in earth, as it is in heaven.

6:11 Give us this day our daily bread.

6:12 And forgive us our debts, as we forgive our debtors.

6:13 And lead us not into temptation, but deliver us from evil: For thine is the kingdom, and the power,
and the glory, for ever. Amen.

6:14 For if ye forgive men their trespasses, your heavenly Father will also forgive you:

http://breachrepairers.webs.com/ 187
6:15 But if ye forgive not men their trespasses, neither will your Father forgive your trespasses.

We cannot be forgiven unless we forgive others.

6:16 Moreover when ye fast, be not, as the hypocrites, of a sad countenance: for they disfigure their faces,
that they may appear unto men to fast. Verily I say unto you, They have their reward.

6:17 But thou, when thou fastest, anoint thine head, and wash thy face;

6:18 That thou appear not unto men to fast, but unto thy Father which is in secret: and thy Father, which
seeth in secret, shall reward thee openly.

6:19 Lay not up for yourselves treasures upon earth, where moth and rust doth corrupt, and where
thieves break through and steal:

These three things described cover everything single thing on earth. Moth, rust, and thieves.

What is the comparison that Jesus makes between verse 19 and verse 20? One is earthly, the
other is heavenly. One is temporary, the other is eternal.

What does the Bible say about those that lay up treasures for themselves on earth?

Jas 5:1-4

They will howl and weep for the miseries that will come upon them. These are the people that have
garments that are moth eaten, their treasures are rusted and thieves have broken in and stolen
their possessions.

What day is that referring to when the rich men will weep and howl?

Ezek 30:1-3

People will howl at the coming of the day of the Lord.

Zeph 1:14-15

The day of the Lord is compared to the day of wrath. When is that?

Rev 6:12-17

The day of wrath is equated to the second coming of the Lord. So those that lay up treasures for
themselves will not be ready at the second coming. How can we lay up for ourselves treasures in
heaven? What is the first step that Jesus gave?

Matt 6:21

Our hearts need to be fixed on the treasures of heaven first. How can we fix our hearts on the
treasures of heaven?

Matt 6:22-23

Prov 23:26

http://breachrepairers.webs.com/ 188
What our eyes behold will also change our mind, our heart. So the first step to fixing our heart on
heaven is to fix our eyes on heaven. Our eyes need to behold light.

What eyes is this referring to and what does the light represent?

2 Cor 5:7

We walk by faith, not by sight. So this is referring to the eye of faith.

Psa 119:105

The light represents the word.

Rom 10:17

Faith is built upon the word of God. So our first step is to build our faith upon the word of God.
What is the next step that Jesus gave to lay for ourselves treasures in heaven?

Matt 6:25-32

To take no thought for our life, what we shall eat, drink or wear. What are these categorized as
according to the previous verses? The treasures of earth. When we start to worry so much about
our own life, food, drink and clothes we tend to lose sight of heavenly things. We get into the rat
race.

Does that mean it doesn’t matter what we eat or drink or wear at all? That we shouldn’t even
think about our next meal? No, but we are to commit our lives to God and allow Him to lead us
each day. The words “Take no thought” mean “not to be anxious.” Some of us worry so much about
our temporal things that we work so hard to secure our treasures here and forget about heaven.
Note: This is not approving laziness, not having to work for our food and raiment. But we are not to
be anxious and stressed out about our temporal things.

What is the final step that Jesus gave that we may lay up treasures in heaven?

Matt 6:33

We should seek first the kingdom of Heaven and his righteousness and then all the other things will
be added to us. How does it work together? Because as we study the Bible, it will instill in us
principles of hard work, and we will not be lazy. It will instill in us principles of honesty and
integrity which we can show to the world as we work for our daily bread.

How can we seek for the kingdom of God and His righteousness?

John 5:39

Jer 29:13

We need to search the scriptures, and if we diligently search, we will find Him, if we search with all
of our hearts.

http://breachrepairers.webs.com/ 189
What is the fruits of a person that is seeking after the kingdom of God and His righteousness?

Matt 5:20

Their righteousness exceeds the righteousness of the Pharisees. How can we exceed that? By not
hating our brother in our heart. By not committing adultery in our heart. Loving our enemies. Then
will we know that our focus is on heaven and His righteousness.

Rom 14:17

When we seek after the kingdom of God, we experience joy and peace and righteousness.

6:20 But lay up for yourselves treasures in heaven, where neither moth nor rust doth corrupt, and where
thieves do not break through nor steal:

6:21 For where your treasure is, there will your heart be also.

6:22 The light of the body is the eye: if therefore thine eye be single, thy whole body shall be full of light.

Body represent the church. If the eye is single and not double minded, it will be full of light and
truth. But if the eye is evil, then the people will be full of darkness – there is a great deal that rests
on the leadership of the church.

6:23 But if thine eye be evil, thy whole body shall be full of darkness. If therefore the light that is in thee
be darkness, how great is that darkness!

6:24 No man can serve two masters: for either he will hate the one, and love the other; or else he will
hold to the one, and despise the other. Ye cannot serve God and mammon.

6:25 Therefore I say unto you, Take no thought for your life, what ye shall eat, or what ye shall drink; nor
yet for your body, what ye shall put on. Is not the life more than meat, and the body than raiment?

6:26 Behold the fowls of the air: for they sow not, neither do they reap, nor gather into barns; yet your
heavenly Father feedeth them. Are ye not much better than they?

6:27 Which of you by taking thought can add one cubit unto his stature?

6:28 And why take ye thought for raiment? Consider the lilies of the field, how they grow; they toil not,
neither do they spin:

6:29 And yet I say unto you, That even Solomon in all his glory was not arrayed like one of these.

6:30 Wherefore, if God so clothe the grass of the field, which to day is, and to morrow is cast into the
oven, shall he not much more clothe you, O ye of little faith?

6:31 Therefore take no thought, saying, What shall we eat? or, What shall we drink? or, Wherewithal
shall we be clothed?

The things listed are the basic necessities of life. We are admonished to seek ye first the kingdom of
God, otherwise we are just like Gentiles.

6:32 (For after all these things do the Gentiles seek:) for your heavenly Father knoweth that ye have need

http://breachrepairers.webs.com/ 190
of all these things.

6:33 But seek ye first the kingdom of God, and his righteousness; and all these things shall be added unto
you.

6:34 Take therefore no thought for the morrow: for the morrow shall take thought for the things of itself.
Sufficient unto the day is the evil thereof.

Each day has its own problems.

http://breachrepairers.webs.com/ 191
Chapter 7
7:1 Judge not, that ye be not judged.

Is there a time when we are called to judge? Yes – judge righteous judgment. What we are dealing
with is that there is a type of judgment which stems from self-righteousness.

In what context does Jesus say that we should judge, or not judge, others?

Luke 6:37-38

We should judge others the same as we wished to be judged. Same as condemning others and also
forgiving others and also giving to others. We should treat others the way we wish to be treated.

Does that mean we should do away with the Judicial law? What does the Bible mean about
not judging?

Jas 4:11

When we speak evil of one another, the Bible says that we are also judging each other. When people
tend to speak evil behind each others backs, that is judging their motive or character. Unless the
fruit is there for proof, we should not be judging one another’s motives if we don’t know what they
are doing.

1 Cor 6:3

We ought to judge those things that pertain to this life.

Exo 18:13

Moses was a judge for the people to settle disputes between each other. And later on in that chapter
he also set people over each other as well.

What is the difference between a mote and a beam? And who is the one that has the mote and
who is the one that has the beam? A mote is a straw or a twig. A beam is a stick of timber. The one
that is being judged has the mote, and the one that has the beam is the one that jus judging (the
hypocrite).

What does the Bible say is the condition of a person who has a beam in their eye?

Luke 6:39

The person who has a beam in their eye is called a blind man by Jesus. What is a blind man
according to the Bible?

2 Pet 1:9

A blind man is a person who has forgotten that they were purged of their past sins.

According to what we have studied so far, why is it that we pronounce such harsh judgment

http://breachrepairers.webs.com/ 192
upon others and are so ready to condemn others for their sins? Because we forget that we have
been purged of our past sins and forget the mercy of God in our lives, and as a result we become
harsh and exacting of other people’s sins which we once did.

What is the definition of a hypocrite according to Matt. 7:5? They try to cast out the mote from
their brother’s eye before they cast out the beam from their own. Basically, it’s the pot calling the
kettle black.

Why does the hypocrite judge people and try to cast out the mote from their brother’s eye?
Teachers note that this is a different perspective from a blind person. The blind person forgot their
past sins. But the hypocrite is different. This is showing another aspect of why others are so harsh
and exacting.

Luke 6:31

Because they have a bigger mote in their own eyes, it’s a beam. But they have forgotten that they
have that in their eyes. They perceive not!

Rom 2:1

They judge because they do the same thing. So often when people are critical and judgmental, it’s
because the sin in their own lives is much bigger than the actual sin that they are seeing in other
people’s eyes. So the very thing that they are doing, they become harsh on others.

How does the Bible advise that we should act when we find a person in sin?

Gal 6:1

We should seek to restore that person to God in meekness. Meekness meaning with in humbleness
and sincerity, without condemning or judging them. The Bible also says that we should consider
ourselves too incase we are tempted. How can we consider ourselves? How can we make that
practical? It may not always be appropriate for a person to help another individual. Example, a
married man consoling a single lady. That is dangerous and tempting to both parties. We should
consider how we help each other if that may cause us to stumble or not in the long run.

1 Cor 10:12

We should take heed, incase we think we shall stand we will fall. How about if we are being
condemned or judged? How should we act?

1 Pet 2:23

We should commit ourselves to God incase we react in such a way which will prove us right but yet
destroy the character of God in us. For example, sometimes when we know we are right and being
judged unfairly, we tend to come out with a vengeful spirit or a angry spirit which is not of Christ.
Christ was treated unfairly, but He didn’t revile against those that reviled against Him, even though
they found no fault in Him. And He didn’t threaten them – He didn’t say “You just wait till I get out
of this and you will be in trouble.” He didn’t look for an opportunity to exact revenge upon them.

7:2 For with what judgment ye judge, ye shall be judged: and with what measure ye mete, it shall be

http://breachrepairers.webs.com/ 193
measured to you again.

7:3 And why beholdest thou the mote that is in thy brother's eye, but considerest not the beam that is in
thine own eye?

It is talking about a level of care and compassion for someone.

MB 128 Not until you feel that you could sacrifice your own self-dignity, and even lay down your life
in order to save an erring brother, have you cast the beam out of your own eye so that you are
prepared to help your brother.

7:4 Or how wilt thou say to thy brother, Let me pull out the mote out of thine eye; and, behold, a beam is
in thine own eye?

7:5 Thou hypocrite, first cast out the beam out of thine own eye; and then shalt thou see clearly to cast
out the mote out of thy brother's eye.

7:6 Give not that which is holy unto the dogs, neither cast ye your pearls before swine, lest they trample
them under their feet, and turn again and rend you.

Both of these animals are unclean which can be applied to the Gentiles. There are some that you
can’t preach the gospel to them as we know it because it may turn them off from it. Emphasizing the
need to read the Bible or proving that the Bible is trustworthy.

What is the condition that Jesus specifies if we want to receive or find or have something
opened to us?

Luke 6:37-38

We must ask, seek or knock.

How should we ask that it may receive?

Matt 21:22

We should ask in prayer and also believe. Does that mean that if we pray and really believe that we
will get a BMW that God will give it?

John 14:13

We should ask in the name of Jesus that the Father may be glorified and not for the glory of
ourselves, otherwise that becomes a selfish prayer. Example – praying for a wife – you must ask
yourselves questions like why do you want a wife? Just so you won’t be lonely? Or just to fulfill your
lusts? Or so that you may look good in public? Sometimes we may pray selfishly.

What are the conditions for asking?

John 15:7

We have to abide in Christ and His words abide in us, then whatever we ask it will be done unto us.
It is important for Christ’s words to abide in us and us in Him that we may know His will. Maybe
that is why our prayers are not answered sometimes, because we aren’t abiding in Christ.

http://breachrepairers.webs.com/ 194
Psa 66:18

If we regard iniquity in our heart the Lord will not hear. We must make sure that we confess and
forsake our sins (Prov. 28:13) that the prayers we pray may ascend up before God. However, bear
in mind that when we ask for forgiveness, the Lord will hear us because we cannot cleanse
ourselves (1 John 1:9). So do not wait for yourself to be clean before you come to the Lord – that
will never happen. It is by His grace that we are made clean.

By asking, what does that imply about that person’s mindset? Why is that so important?
Further add to the question by asking “What brings a person to ask for help? Or when does a person
ask?” For example, when would you ask your parents for money? Or when do you ask your friends
for help in some homework? The asking implies that we realize our necessity, that we need
something or need help because we cannot do it alone. This is so important because it will keep us
humble not self sufficient, to help us realize that God will be our help and our strength in time of
need. Those that never ask for help will never receive help because they think they can do it all
alone.

What should we seek after that we may find?

Luke 12:29-31

We shouldn’t seek after temporal things like food or water. We should seek first the kingdom and
all those things shall be added to us. It’s almost as if Jesus gave us a step of assurance how we can be
guaranteed our bread and water. We should seek after the kingdom of God.

How should we seek that we may find?

Jer 29:13

We are promised that we will find if we seek with all of our heart. That means it must not be a half-
hearted searching. Many people do not receive blessings even though they do the same things as
those that do sometimes because their heart really isn’t in it. Example – a doctor may be happy and
content ministering to patients, but yet another doctor may be very dissatisfied because his heart is
not in that job. What does mean with all of our heart? That we put all of our efforts into what we do.

What is the assurance that Jesus gives to us that our asking, seeking or knocking will be
answered?

Matt 7:8-11

First Jesus puts a triple emphasis that God is able to do all things and will answer if we ask, seek or
knock. Secondly, he gives an analogy of how a father would give bread to his son instead of a stone
or a fish instead of a serpent. And HOW MUCH MORE will our Heavenly Father would give us so
much more. The question is “HOW MUCH MORE?” Contrasting God the Father to our earthly Father.

Isa 49:14-16

A woman may forget her suckling child but God has given us the assurance that He will not forget
us. He will answer us when we ask, seek and knock.

http://breachrepairers.webs.com/ 195
What is that good thing that Jesus promised that He would give us if we ask?

Luke 11:13

The Holy Spirit is compared to a good thing which Jesus promised He would give if we ask.

What should be out attitude so that we can receive the Holy Spirit? We need to ask, seek, and
knock. This involves our faculties: Ask = lifting our voice in prayer for the Holy Spirit. Seeking =
searching the Scriptures, how do we do this by using our eyes. Knocking = involves the sense of
touch, involving doing, action. In order to knock we must walk to a door and knock. All our being,
voice, eyes, and our actions/hand should show that we are earnestly desiring the good gift of the
Holy Spirit. It's not a haphazard request to God.

Why is it so important, or good, that we ask for the Holy Spirit?

John 14:26

The Holy Spirit will teach us all things.

John 16:13

The Holy Spirit will guide us into all truth.

1 Pet 1:2

The Holy Spirit is the one that sanctifies us and enables us to live a holy life.

Apart from asking, what are the conditions for receiving the Holy Spirit?

Acts 5:32

The Holy Spirit is given to those that obey. It is not enough just to ask, presuming that we
will receive the Holy Spirit. We must learn to obey those things that God has outlined in the
Bible which is one of the conditions to receiving Him.

7:7 Ask, and it shall be given you; seek, and ye shall find; knock, and it shall be opened unto you:

One of the greatest promises in the Bible.

7:8 For every one that asketh receiveth; and he that seeketh findeth; and to him that knocketh it shall be
opened.

7:9 Or what man is there of you, whom if his son ask bread, will he give him a stone?

7:10 Or if he ask a fish, will he give him a serpent?

7:11 If ye then, being evil, know how to give good gifts unto your children, how much more shall your
Father which is in heaven give good things to them that ask him?

“how much more” – The gift that He desires to give to us is the Holy Spirit.

http://breachrepairers.webs.com/ 196
Luke 11:13

Why is the Holy Spirit such a great gift? Because the fruit of the Spirit is all these other things.
Without the Holy Spirit, we could never be convicted of sin.

7:12 Therefore all things whatsoever ye would that men should do to you, do ye even so to them: for this
is the law and the prophets.

It is the most famous verse in the Gentile world – it is called the golden rule. So the idea is that we
should treat others as we want to be treated. What we do to others will always return back to us.

7:13 Enter ye in at the strait gate: for wide is the gate, and broad is the way, that leadeth to destruction,
and many there be which go in thereat:

The platform of truth is like a path coming narrower. We start of with broad topics and then zoom
into more specific topics. There will always be very few people who want to do right. The road to
destruction is wide and easy.

What is the original meaning of the word “strait” in the Bible? In the concordance it means
“narrow.” Strait does not mean straight as in a straight line, but narrow or difficult.

More than just being exhorted to enter the strait gate, what are we advised to do?

Luke 13:24

We are exhorted to STRIVE to enter in at the strait gate. That word strive means to struggle, or
fight, or contend.

According to the Bible, what does it mean to strive, or fight or contend?

Rom 15:30

We can strive by praying.

Jude 1:3

We should fight for the faith.

Heb 12:4

By striving to enter in the strait gate, we need to strive against sin.

According to Matt. 7:15, what will stop us from striving to enter into the strait gate?

Matt 7:15

False prophets will stop us from striving to enter into the strait gate. How may we know who is a
false prophet?

Matt 7:16-20

We may know who is a false prophets by the fruits that they bear. What are the fruits that will

http://breachrepairers.webs.com/ 197
show who is and who is not a false prophet?

Matt 12:33-34

The words that they speak will show what their heart is like. We may hide for a while who we really
are, but just by how we speak it will reveal the deep thoughts and intents of the heart.

Gal 5:19-23

The fruits of the Spirit is love, joy, peace, etc. In our character it will be revealed what sort of person
we are. How we live our life it will show if we are a false prophet or not.

Prov 11:30

The fruit of the righteous is a tree of life and that fruit is shown in winning souls. A true prophet will
win many souls to God. But a false prophet does not win any to the kingdom of God. A person is
known by the company that they keep. It will be easy to determine a false prophet to look at the
people that he/she has ministered to, to see what type of character they are because they are most
likely reflecting the type of person that he is.

According to Matt. 7:21-23, what should we do if we want to strive to enter into the strait
gate?

Matt 7:21-23

We must do the will of God. Notice that doing the will of God is not the same as performing miracles,
or praying or preaching or casting out devils. Doing the will of God is opposite of trying to work our
way into heaven.

What does it mean to do the will of God?

1 Thes 4:3

To do the will of God means that we are to be sanctified. What does the word sanctification mean? It
means to be set apart for a holy use. To be made holy.

How can we be sanctified?

John 17:17

We are sanctified by the word of God. So according to what we have studied so far, how else can we
strive to enter into the strait gate? We must be sure not to follow false prophets. We will know them
by their fruits. But if we want to detect false prophets and their false teachings what should we
study? THE BIBLE! And by studying the Bible we are? SANCTIFIED! APPLICATION QUESTION: Have
you studied your Bible today? Are you doing the will of God and striving to enter into the strait gate
by studying the Word?

How can we help and encourage each other today to strive to enter into the strait gate?
Discuss? Encourage them to share with each other some thoughts on this question. Suggest things
like: Pick and partner not of your own family and share with each other during the week your

http://breachrepairers.webs.com/ 198
devotions on what we read in the Bible. Prayer group chain, praying for each other over the phone.

7:14 Because strait is the gate, and narrow is the way, which leadeth unto life, and few there be that find
it.

“few there be that find it” – we have to look, we have to search for it.

7:15 Beware of false prophets, which come to you in sheep's clothing, but inwardly they are ravening
wolves.

An index to the heart is mostly what you can see outside. This is one of the tests of a true prophet.
What are the results of their work? Their fruits? What can we learn about conversion here with the
context of good and bad fruits? It is talking about false prophets but can also talk about people. The
issue here is the heart because a good branch can easily be grafted on.

7:16 Ye shall know them by their fruits. Do men gather grapes of thorns, or figs of thistles?

7:17 Even so every good tree bringeth forth good fruit; but a corrupt tree bringeth forth evil fruit.

7:18 A good tree cannot bring forth evil fruit, neither can a corrupt tree bring forth good fruit.

7:19 Every tree that bringeth not forth good fruit is hewn down, and cast into the fire.

7:20 Wherefore by their fruits ye shall know them.

7:21 Not every one that saith unto me, Lord, Lord, shall enter into the kingdom of heaven; but he that
doeth the will of my Father which is in heaven.

7:22 Many will say to me in that day, Lord, Lord, have we not prophesied in thy name? and in thy name
have cast out devils? and in thy name done many wonderful works?

7:23 And then will I profess unto them, I never knew you: depart from me, ye that work iniquity.

Not all Christians will enter into Heaven. Miracles are not a test that a person is right with God.

“iniquity” in verse 23 means lawlessness.

7:24 Therefore whosoever heareth these sayings of mine, and doeth them, I will liken him unto a wise
man, which built his house upon a rock:

What is the difference between the wise man and the foolish man in this short parable? The
wise man hears and does Jesus’ sayings but the foolish man hears but doesn’t do it. The wise man
builds his house upon the rock and the foolish man builds his house upon the sand.

What does the rock represent that we are to build our foundation upon?

Luke 6:48

We are to build our foundation upon the rock.

2 Sam 22:47

The Lord God is our rock.

http://breachrepairers.webs.com/ 199
1 Cor 3:11

Which God is that, it is Jesus Christ who is our foundation. What elements beat against the house
to try to make it to fall?

Matt 7:25

The elements that beast against the house was the rain, the flood and the winds. What did these
elements try to do to the house that was built upon the rock?

Luke 6:48

It tried to shake the house but could not.

What does the shaking represent?

Isa 13:13

The shaking represents the time when God will arise to shake the heavens and the earth.

2 Thes 2:2

The shaking represents those who will be shaken in mind and troubled with the things that are
taking place upon the earth.

What is the purpose of the shaking?

Heb 12:26-27

To shake everything that can be shaken so that those things which remain are those which were
founded upon the rock.

Neh 5:13

To shake every man which does not do according to the promise (the word) of the Lord. It is a
testing time which God allows every person to go through to test what sort of foundation they were
building upon.

It is the elements that cause the shaking. So what does the rain, the floods and the winds
represent?
Deut 32:2

Doctrine will fall as rain. So it is the doctrine of God that will cause a shaking to test what
foundation we have been building upon. How does that cause a shaking? Because when people hear
the pure truth, some will not accept it because it goes against what they are doing. That is exactly
what happened with the scribe and Pharisees when they heard Jesus speaking? That is why they
plotted to kill Him.

The floods

http://breachrepairers.webs.com/ 200
Isa 59:19

The flood represents your enemies, those that hate you and persecute you.

Psa 69:1-4

The flood represents those that hate you without a cause. They will come to persecute you as David
was persecuted by Saul in the wilderness. So the flood represents persecution. But what does the
Bible tell us to do in times of persecution?

Matt 5:11, 12

The winds:

Eph 4:14

The winds represents the winds of doctrines, of the false doctrines, that will lead men astray and
deceive them. So it is the straight testimony of truth, persecution or false doctrines that will cause
people to be shaken in the last days.

The foolish man built his house upon the sand. What does the sand represent and what does
it mean to build on sand?

Gen 32:12

The sand represented the people that could not be counted. Sand represents people. So to build
upon sand means to build upon people and not the word of God.

How do we build upon people today?

Matt 15:9, 6

We listen more to the preacher than to the Word. We obey the commandments of man rather than
the commandments of God. What is the commandments of men? Verse 6 – men’s traditions.

More than just Jesus Christ, what is that foundation that we must build upon today that will
stand forever?

Isa 40:8

The word of the Lord will stand forever.

Matt 24:35

Heaven and earth shall pass away, but My words shall not pass away. It is the word of the Lord (the
Bible) that we must learn to build our sure foundation.

7:25 And the rain descended, and the floods came, and the winds blew, and beat upon that house; and it
fell not: for it was founded upon a rock.

http://breachrepairers.webs.com/ 201
7:26 And every one that heareth these sayings of mine, and doeth them not, shall be likened unto a
foolish man, which built his house upon the sand:

7:27 And the rain descended, and the floods came, and the winds blew, and beat upon that house; and it
fell: and great was the fall of it.

Both had a house. They both heard Him. A rock represents Christ in the Bible

1 Cor 10:4

What does a house represent? The house represents a church or God’s people

Heb 3:6

Rain represents doctrine.

Deut 32:2

Wind represents winds of doctrine.

Eph 4:14

The sand represents a multitude of people.

Joshua 11:4

Floods represents ungodly men –

Ps 18:4

In the Bible, Babylon also has a great fall. So what is the structure of the Sermon on the Mount.
Conversion, true obedience, then Jesus makes an appeal – not only to hear but also do.

7:28 And it came to pass, when Jesus had ended these sayings, the people were astonished at his
doctrine:

7:29 For he taught them as one having authority, and not as the scribes.

http://breachrepairers.webs.com/ 202
Chapter 8
Reading
 Desire of Ages – Chapter 27 – Thou Canst Make Me Clean
 Desire of Ages – Chapter The Centurion
 Desire of Ages – Chapter Peace be still

8:1 When he was come down from the mountain, great multitudes followed him.

8:2 And, behold, there came a leper and worshipped him, saying, Lord, if thou wilt, thou canst make me
clean.

Leprosy in the Bible is a symbol of sin. Leprosy was contagious. Leprosy would often cause death
likewise, sin will ultimately will produce death.

DA 263 Some try to prevent him from approaching Jesus, but in vain.

Many that want to approach Jesus, Satan makes it very hard for them to make it able. Satan makes it
hard for them to come to Jesus for repentance.

DA 263 Pressing to Jesus, he casts himself at His feet with the cry, "Lord, if Thou wilt, Thou canst
make me clean." Jesus replied, "I will; be thou made clean," and laid His hand upon him. –

8:3 And Jesus put forth his hand, and touched him, saying, I will; be thou clean. And immediately his
leprosy was cleansed.

As soon as the leper asked, he was healed straight away. When we pray, is it always answered
straight away?

DA 266 In some instances of healing, Jesus did not at once grant the blessing sought. But in the case
of leprosy, no sooner was the appeal made than it was granted. When we pray for earthly blessings,
the answer to our prayer may be delayed, or God may give us something other than we ask, but not
so when we ask for deliverance from sin. It is His will to cleanse us from sin, to make us His children,
and to enable us to live a holy life.

Whenever we pray for deliverance from sin, God answers that prayer right away.

8:4 And Jesus saith unto him, See thou tell no man; but go thy way, shew thyself to the priest, and offer
the gift that Moses commanded, for a testimony unto them.

Why did He tell him to tell no one? Too many lepers would come – if He was known as a leper
healer, then no one else would come because it was contagious.

DA 264 Notwithstanding the caution of Jesus, the man made no further effort to conceal the fact of
his cure. It would indeed have been impossible to conceal it, but the leper published the matter
abroad. Conceiving that it was only the modesty of Jesus which laid this restriction upon him, he
went about proclaiming the power of this Great Healer. He did not understand that every such
manifestation made the priests and elders more determined to destroy Jesus.

There is a danger to adding more to what Jesus actually said.

DA 267 Yet it was not physical restoration he desired so much as relief from the burden of sin. If he

http://breachrepairers.webs.com/ 203
could see Jesus, and receive the assurance of forgiveness and peace with Heaven, he would be
content to live or die, according to God's will.

When people sin, the guilt of sin can cause you to go insane. The results of many physical illnesses is
the guilt of sin that rests heavy on a persons mind.

8:5 And when Jesus was entered into Capernaum, there came unto him a centurion, beseeching him,

8:6 And saying, Lord, my servant lieth at home sick of the palsy, grievously tormented.

8:7 And Jesus saith unto him, I will come and heal him.

8:8 The centurion answered and said, Lord, I am not worthy that thou shouldest come under my roof: but
speak the word only, and my servant shall be healed.

Faith always manifests itself in works. Love is the same as righteousness.

Luke 7:4

In that instance they were seeking for righteousness by works. True Faith is paired with true
humility.

He who stood beside the sorrowing mother at the gate of Nain, watches with every mourning one
beside the bier. He is touched with sympathy for our grief. His heart, that loved and pitied, is a heart
of unchangeable tenderness. His word, that called the dead to life, is no less efficacious now than
when spoken to the young man of Nain. He says, "All power is given unto Me in heaven and in earth."
Matt. 28:18. That power is not diminished by the lapse of years, nor exhausted by the ceaseless
activity of His overflowing grace. To all who believe on Him He is still a living Savior.

8:9 For I am a man under authority, having soldiers under me: and I say to this man, Go, and he goeth;
and to another, Come, and he cometh; and to my servant, Do this, and he doeth it.

8:10 When Jesus heard it, he marvelled, and said to them that followed, Verily I say unto you, I have not
found so great faith, no, not in Israel.

8:11 And I say unto you, That many shall come from the east and west, and shall sit down with Abraham,
and Isaac, and Jacob, in the kingdom of heaven.

8:12 But the children of the kingdom shall be cast out into outer darkness: there shall be weeping and
gnashing of teeth.

8:13 And Jesus said unto the centurion, Go thy way; and as thou hast believed, so be it done unto thee.
And his servant was healed in the selfsame hour.

8:14 And when Jesus was come into Peter's house, he saw his wife's mother laid, and sick of a fever.

8:15 And he touched her hand, and the fever left her: and she arose, and ministered unto them.

8:16 When the even was come, they brought unto him many that were possessed with devils: and he cast
out the spirits with his word, and healed all that were sick:

8:17 That it might be fulfilled which was spoken by Esaias the prophet, saying, Himself took our
infirmities, and bare our sicknesses.

http://breachrepairers.webs.com/ 204
8:18 Now when Jesus saw great multitudes about him, he gave commandment to depart unto the other
side.

8:19 And a certain scribe came, and said unto him, Master, I will follow thee whithersoever thou goest.

8:20 And Jesus saith unto him, The foxes have holes, and the birds of the air have nests; but the Son of
man hath not where to lay his head.

8:21 And another of his disciples said unto him, Lord, suffer me first to go and bury my father.

8:22 But Jesus said unto him, Follow me; and let the dead bury their dead.

8:23 And when he was entered into a ship, his disciples followed him.

DA 334 The sun had set, and the blackness of night settled down upon the stormy sea. The waves,
lashed into fury by the howling winds, dashed fiercely over the disciples' boat, and threatened to
engulf it. Those hardy fishermen had spent their lives upon the lake, and had guided their craft safely
through many a storm; but now their strength and skill availed nothing. They were helpless in the
grasp of the tempest, and hope failed them as they saw that their boat was filling.

All their past experiences were helpless. They were at this point where they knew they could not
help themselves.

8:24 And, behold, there arose a great tempest in the sea, insomuch that the ship was covered with the
waves: but he was asleep.

Jesus never feared – He was asleep.

Rev 21:8

Those who are fearful are those that will be lost. God hath not given us a spirit of fear, but of love
and a sound mind. Fear is not from God.

8:25 And his disciples came to him, and awoke him, saying, Lord, save us: we perish.

8:26 And he saith unto them, Why are ye fearful, O ye of little faith? Then he arose, and rebuked the
winds and the sea; and there was a great calm.

DA 335 Never did a soul utter that cry unheeded. As the disciples grasp their oars to make a last
effort, Jesus rises. He stands in the midst of His disciples, while the tempest rages, the waves break
over them, and the lightning illuminates His countenance. He lifts His hand, so often employed in
deeds of mercy, and says to the angry sea, "Peace, be still."

8:27 But the men marvelled, saying, What manner of man is this, that even the winds and the sea obey
him!

The same power that He possesses as Creator, He possesses here on the sea.

8:28 And when he was come to the other side into the country of the Gergesenes, there met him two
possessed with devils, coming out of the tombs, exceeding fierce, so that no man might pass by that way.

8:29 And, behold, they cried out, saying, What have we to do with thee, Jesus, thou Son of God? art thou
come hither to torment us before the time?

http://breachrepairers.webs.com/ 205
8:30 And there was a good way off from them an herd of many swine feeding.

8:31 So the devils besought him, saying, If thou cast us out, suffer us to go away into the herd of swine.

8:32 And he said unto them, Go. And when they were come out, they went into the herd of swine: and,
behold, the whole herd of swine ran violently down a steep place into the sea, and perished in the waters.

8:33 And they that kept them fled, and went their ways into the city, and told every thing, and what was
befallen to the possessed of the devils.

8:34 And, behold, the whole city came out to meet Jesus: and when they saw him, they besought him that
he would depart out of their coasts.

http://breachrepairers.webs.com/ 206
Chapter 9
Chapter 9 is unique in that it shows Christ authority as King to forgive sins. He is a religious
spiritual king. This means that His kingdom consists of those whose sins He has forgiven. There
were many examples of Christ healing before in the book of Matthew, but this is the first time it
mentions healing in relationship to Him forgiving sins. That is Matthew’s main point.

Chapter 9 can be connected with Matthew 1:21 “and thou shalt call his name JESUS: for he shall
save his people from their sins.” This chapter shows how He will save His people from their sins,
and the latter chapters show Him dying on the cross

Reading
 Desire of Ages – Chapter 28 – Levi-Matthew
 Desire of Ages – Chapter The Touch of Faith

9:1 And he entered into a ship, and passed over, and came into his own city.

“into a ship, and passed over” – this shows that Jesus was separated from this city by water.
There was a separation between Him and the people in Capernaum.

9:2 And, behold, they brought to him a man sick of the palsy, lying on a bed: and Jesus seeing their faith
said unto the sick of the palsy; Son, be of good cheer; thy sins be forgiven thee.

“they brought to him a man sick of the palsy” – This shows the attitude of the men who were in
Capernaum. They were waiting, longing, and anticipating the day when Jesus would be in close
proximity to them so they could bring the man. This shows the attitude of the man who had palsy.
The man had to first trust Christ. He did this by faith. Before this Jesus was out of their reach. Where
is He? In another city? Where is He? On the Mt. of Olives. But once He came to their city they could
reach Him. But what Jesus did was made Himself available. He came near to where they were to
incite within their heart the desire to come to Him. He says “Come unto me all ye that labor and are
heavy laden”. Two things took place, Jesus came near to them, and then the men had to make the
effort to then come to Him.

App: the first thing we need to do is long to see Jesus. We need to anticipate Him. Coming, but
before we can go to Him we must first know where He is. The men found Him in the city of
Capernaum in a home. Where is He now? In another home, His heavenly home. And that Home has
two compartments, the holy place, and the most holy place. Where is He? In the MHP. But before
they were separated by a body of water, and Jesus made the effort to come. We may feel like we are
separated by a great body of water, something keeping us from Him, but He will part that obstacle
so He can come near to you.

“Jesus seeing their faith” – After we find Him, we realize where He is we must exercise faith. When
we exercise faith Jesus sees it. That means recognizing where He is should cause us to have faith.
They had faith all along, but now that faith was revealed to all. The faith of the men was also
instrumental in the healing of this man. When there are others who are sick, we need to go get them
and bring them to Jesus, and this will help them to have greater faith to receive healing.

9:3 And, behold, certain of the scribes said within themselves, This man blasphemeth.

The scribe missed the whole point. He should have been paying attention to the attitude of the three

http://breachrepairers.webs.com/ 207
men. Jesus was trying to help the scribe exercise faith. But He said Jesus committed blasphemy.

Application: We do the same thing. Instead of being happy that people exercise faith for healing, and
learning a lesson in faith. We murmur about the people who perform the healing. I understand
there are many ways of practicing the healing art and God only approves one. But that doesn’t mean
that God only heals based on your understanding of healing. You should have paid attention to the
lesson God was trying to teach instead of criticizing.

9:4 And Jesus knowing their thoughts said, Wherefore think ye evil in your hearts?

The scribe needed to be forgiven. But his sin was hid from everyone else’s eyes.

9:5 For whether is easier, to say, Thy sins be forgiven thee; or to say, Arise, and walk?

Arise and walk is synonymous with thy sins be forgiven thee. That means every time Christ healed
it was the same as Him forgiving sin. But there was a condition they had to have faith in Him.

9:6 But that ye may know that the Son of man hath power on earth to forgive sins, (then saith he to the
sick of the palsy,) Arise, take up thy bed, and go unto thine house.

“ye may know” – especially the leaders, the scribes, not just the multitudes. Jesus was trying to
forgive them. When the man trusted Christ, exercised faith, they exercised His will by choosing to
obey the power was given for Him to arise and walk. There was a co-operation.

9:7 And he arose, and departed to his house.

9:8 But when the multitudes saw it, they marvelled, and glorified God, which had given such power unto
men.

We glorify God by acknowledging His power to forgive sins on this earth because He came as the
Messiah, the King! Jesus as King has power to forgive.

9:9 And as Jesus passed forth from thence, he saw a man, named Matthew, sitting at the receipt of
custom: and he saith unto him, Follow me. And he arose, and followed him.

To types of arising in chapter 9. Having faith, and arising to have your sins forgiven and arising to
follow Jesus. That means Jesus wants to give our sins so that we can follow Him

What was Matthew like? He was a tax collector and a Jew. Often times, publicans were
dishonest and greedy

DA 273 To Matthew in his wealth, and to Andrew and Peter in their poverty, the same test was
brought; the same consecration was made by each. At the moment of success, when the nets were
filled with fish, and the impulses of the old life were strongest, Jesus asked the disciples at the sea to
leave all for the work of the gospel. So every soul is tested as to whether the desire for temporal good
or for fellowship with Christ is strongest.

When success comes, it is then that Christ calls you. You have to choose between worldly success
and a life of sacrifice.

9:10 And it came to pass, as Jesus sat at meat in the house, behold, many publicans and sinners came
and sat down with him and his disciples.

http://breachrepairers.webs.com/ 208
First time the people are referred to as sinners. Your mind should go back to Matt 1:21

9:11 And when the Pharisees saw it, they said unto his disciples, Why eateth your Master with publicans
and sinners?

DA 273 The calling of Matthew to be one of Christ's disciples excited great indignation. For a
religious teacher to choose a publican as one of his immediate attendants was an offense against the
religious, social, and national customs. By appealing to the prejudices of the people the Pharisees
hoped to turn the current of popular feeling against Jesus.

Sometimes it is the despised of the community that are the most earnest. To know!!

DA 280 Man must be emptied of self before he can be, in the fullest sense, a believer in Jesus. When
self is renounced, then the Lord can make man a new creature. New bottles can contain the new
wine. The love of Christ will animate the believer with new life. In him who looks unto the Author
and Finisher of our faith the character of Christ will be manifest.

Wine represents Christ’s Words. If a heart is unconverted and the Words touch the heart, without
the change of heart, it may harden it more. We need a new heart first.

9:12 But when Jesus heard that, he said unto them, They that be whole need not a physician, but they
that are sick.

“sick” – from the previous verses it means that they had sins that were not forgiven. But in order to
be forgiven they needed faith. So who are the whole? Those whose sins are forgiven.

9:13 But go ye and learn what that meaneth, I will have mercy, and not sacrifice: for I am not come to
call the righteous, but sinners to repentance.

“righteous” = Whole - those who are whole and there sins are forgiven

“Sinners” = Sick – Jesus wants to forgive their sins

Note: This also shows that Jesus calls or draws us. He drew the three men in the previous verses.

“I will have mercy” what is the mercy of God? Forgiven our sins. Note: Mercy seat in the sanctuary
is where forgiveness is complete. The Pharisees and Scribes should have been merciful to the
publicans and sinners. But they were more concerned about the ceremonial law, when this is what
the ceremonial law taught. Jesus said “Blessed [are] the merciful: for they shall obtain mercy.” Matt
5:7 What is that mercy? Forgiveness of sin. Therefore, the scribes and Pharisees were not eligible to
have their sins forgiven. And the act of being merciful is an alms deed, or deed of good work. That
should remind you of Matt 6.

9:14 Then came to him the disciples of John, saying, Why do we and the Pharisees fast oft, but thy
disciples fast not?

9:15 And Jesus said unto them, Can the children of the bridechamber mourn, as long as the bridegroom
is with them? but the days will come, when the bridegroom shall be taken from them, and then shall they
fast.

9:16 No man putteth a piece of new cloth unto an old garment, for that which is put in to fill it up taketh
from the garment, and the rent is made worse.

http://breachrepairers.webs.com/ 209
9:17 Neither do men put new wine into old bottles: else the bottles break, and the wine runneth out, and
the bottles perish: but they put new wine into new bottles, and both are preserved.

9:18 While he spake these things unto them, behold, there came a certain ruler, and worshipped him,
saying, My daughter is even now dead: but come and lay thy hand upon her, and she shall live.

9:19 And Jesus arose, and followed him, and so did his disciples.

9:20 And, behold, a woman, which was diseased with an issue of blood twelve years, came behind him,
and touched the hem of his garment:

Why is the hem significant? It represented obedience. Power comes to us when we are willing to be
obedient to God.

Mark 5:26

This particular woman, she basically spent all her money, and at the end of this, she came to Jesus.
Many people are like that – when we have lost all hope, then we go to Jesus.

Mark 5:31-34

Why did Jesus have to ask who touched Him? She had to acknowledge that it was her who touched
Him.

DA 347After healing the woman, Jesus desired her to acknowledge the blessing she had received.
The gifts which the gospel offers are not to be secured by stealth or enjoyed in secret. So the Lord
calls upon us for confession of His goodness. "Ye are My witnesses, saith the Lord, that I am God." Isa.
43:12. –

DA 347 Our confession of His faithfulness is Heaven's chosen agency for revealing Christ to the
world. We are to acknowledge His grace as made known through the holy men of old; but that which
will be most effectual is the testimony of our own experience. –

DA 348 It is for our own benefit to keep every gift of God fresh in our memory. Thus faith is
strengthened to claim and to receive more and more. There is greater encouragement for us in the
least blessing we ourselves receive from God than in all the accounts we can read of the faith and
experience of others. The soul that responds to the grace of God shall be like a watered garden. His
health shall spring forth speedily; his light shall rise in obscurity, and the glory of the Lord shall be
seen upon him. Let us then remember the loving-kindness of the Lord, and the multitude of His
tender mercies.

9:21 For she said within herself, If I may but touch his garment, I shall be whole.

9:22 But Jesus turned him about, and when he saw her, he said, Daughter, be of good comfort; thy faith
hath made thee whole. And the woman was made whole from that hour.

9:23 And when Jesus came into the ruler's house, and saw the minstrels and the people making a noise,

9:24 He said unto them, Give place: for the maid is not dead, but sleepeth. And they laughed him to scorn.

9:25 But when the people were put forth, he went in, and took her by the hand, and the maid arose.

9:26 And the fame hereof went abroad into all that land.

http://breachrepairers.webs.com/ 210
9:27 And when Jesus departed thence, two blind men followed him, crying, and saying, Thou Son of
David, have mercy on us.

9:28 And when he was come into the house, the blind men came to him: and Jesus saith unto them,
Believe ye that I am able to do this? They said unto him, Yea, Lord.

9:29 Then touched he their eyes, saying, According to your faith be it unto you.

9:30 And their eyes were opened; and Jesus straitly charged them, saying, See that no man know it.

9:31 But they, when they were departed, spread abroad his fame in all that country.

9:32 As they went out, behold, they brought to him a dumb man possessed with a devil.

9:33 And when the devil was cast out, the dumb spake: and the multitudes marvelled, saying, It was
never so seen in Israel.

9:34 But the Pharisees said, He casteth out devils through the prince of the devils.

9:35 And Jesus went about all the cities and villages, teaching in their synagogues, and preaching the
gospel of the kingdom, and healing every sickness and every disease among the people.

9:36 But when he saw the multitudes, he was moved with compassion on them, because they fainted, and
were scattered abroad, as sheep having no shepherd.

9:37 Then saith he unto his disciples, The harvest truly is plenteous, but the labourers are few;

9:38 Pray ye therefore the Lord of the harvest, that he will send forth labourers into his harvest.

http://breachrepairers.webs.com/ 211
Chapter 10
Reading
 Desire of Ages – Chapter The First Evangelists

10:1 And when he had called unto him his twelve disciples, he gave them power against unclean spirits,
to cast them out, and to heal all manner of sickness and all manner of disease.

10:2 Now the names of the twelve apostles are these; The first, Simon, who is called Peter, and Andrew
his brother; James the son of Zebedee, and John his brother;

10:3 Philip, and Bartholomew; Thomas, and Matthew the publican; James the son of Alphaeus, and
Lebbaeus, whose surname was Thaddaeus;

10:4 Simon the Canaanite, and Judas Iscariot, who also betrayed him.

10:5 These twelve Jesus sent forth, and commanded them, saying, Go not into the way of the Gentiles, and
into any city of the Samaritans enter ye not:

10:6 But go rather to the lost sheep of the house of Israel.

Why did Jesus choose this first method of working with the Jews? Jesus also knew that if He were to
win the Jews, the strength of that would help the gospel to go to the whole world. The 70 weeks was
fast coming to a close.

10:7 And as ye go, preach, saying, The kingdom of heaven is at hand.

The kingdom of God was here.

10:8 Heal the sick, cleanse the lepers, raise the dead, cast out devils: freely ye have received, freely give.

10:9 Provide neither gold, nor silver, nor brass in your purses,

10:10 Nor scrip for your journey, neither two coats, neither shoes, nor yet staves: for the workman is
worthy of his meat.

What does this indicate? It indicated that Paul had a trade. What is the principle about gospel work?
When people give you food, money, etc, we deserve it because we are working full time in the
ministry.

10:11 And into whatsoever city or town ye shall enter, enquire who in it is worthy; and there abide till ye
go thence.

10:12 And when ye come into an house, salute it.

10:13 And if the house be worthy, let your peace come upon it: but if it be not worthy, let your peace
return to you.

By song and prayer, you can bless the house. By your testimony, etc. That is how we are to bless
them.

10:14 And whosoever shall not receive you, nor hear your words, when ye depart out of that house or

http://breachrepairers.webs.com/ 212
city, shake off the dust of your feet.

10:15 Verily I say unto you, It shall be more tolerable for the land of Sodom and Gomorrha in the day of
judgment, than for that city.

10:16 Behold, I send you forth as sheep in the midst of wolves: be ye therefore wise as serpents, and
harmless as doves.

Where do have any mention of the serpent in the Bible? Garden of Eden. Jesus did not say be as
deceptive as serpent. Why harmless as doves? They do not have any defense. It’s only defense is to
climb higher and higher and higher.

10:17 But beware of men: for they will deliver you up to the councils, and they will scourge you in their
synagogues;

We should never trust man. Cursed is he that trusteth in the arm of flesh.

10:18 And ye shall be brought before governors and kings for my sake, for a testimony against them and
the Gentiles.

10:19 But when they deliver you up, take no thought how or what ye shall speak: for it shall be given you
in that same hour what ye shall speak.

10:20 For it is not ye that speak, but the Spirit of your Father which speaketh in you.

Parallel passage is found in Mark 13:11. That passage indicates that there will be the gift of
prophecy – it is God speaking through men.

DA 349 He had set before them the truths of Scripture in contrast with tradition. Thus He had
strengthened their confidence in God's word, and in a great measure had set them free from their
fear of the rabbis and their bondage to tradition.

DA 350 None were sent forth alone, but brother was associated with brother, friend with friend. –

When God sends out people, He sends out people together.

DA 351 If they had now preached the gospel to the Gentiles or the Samaritans, they would have lost
their influence with the Jews. By exciting the prejudice of the Pharisees they would have involved
themselves in controversy which would have discouraged them at the outset of their labors. Even the
apostles were slow to understand that the gospel was to be carried to all nations.

We must start with the church and deal with avoiding controversy.

DA 350 During His ministry Jesus devoted more time to healing the sick than to preaching.

Health will open doors which preaching cannot open.

10:21 And the brother shall deliver up the brother to death, and the father the child: and the children
shall rise up against their parents, and cause them to be put to death.

10:22 And ye shall be hated of all men for my name's sake: but he that endureth to the end shall be
saved.

http://breachrepairers.webs.com/ 213
10:23 But when they persecute you in this city, flee ye into another: for verily I say unto you, Ye shall not
have gone over the cities of Israel, till the Son of man be come.

10:24 The disciple is not above his master, nor the servant above his lord.

10:25 It is enough for the disciple that he be as his master, and the servant as his lord. If they have called
the master of the house Beelzebub, how much more shall they call them of his household?

10:26 Fear them not therefore: for there is nothing covered, that shall not be revealed; and hid, that
shall not be known.

10:27 What I tell you in darkness, that speak ye in light: and what ye hear in the ear, that preach ye upon
the housetops.

10:28 And fear not them which kill the body, but are not able to kill the soul: but rather fear him which is
able to destroy both soul and body in hell.

Jesus told us that we shouldn’t fear man.

10:29 Are not two sparrows sold for a farthing? and one of them shall not fall on the ground without
your Father.

10:30 But the very hairs of your head are all numbered.

10:31 Fear ye not therefore, ye are of more value than many sparrows.

10:32 Whosoever therefore shall confess me before men, him will I confess also before my Father which
is in heaven.

10:33 But whosoever shall deny me before men, him will I also deny before my Father which is in heaven.

10:34 Think not that I am come to send peace on earth: I came not to send peace, but a sword.

“sword” – it represents the Word of God. The message that we bear has to do the cutting, not our
attitude or character.

10:35 For I am come to set a man at variance against his father, and the daughter against her mother,
and the daughter in law against her mother in law.

10:36 And a man's foes shall be they of his own household.

10:37 He that loveth father or mother more than me is not worthy of me: and he that loveth son or
daughter more than me is not worthy of me.

Jesus at His death, made sure His mother was taken care of. As long as the law is concerned, we
should obey our parents, but when it makes us at variance with the law, then we must obey Jesus.

10:38 And he that taketh not his cross, and followeth after me, is not worthy of me.

10:39 He that findeth his life shall lose it: and he that loseth his life for my sake shall find it.

10:40 He that receiveth you receiveth me, and he that receiveth me receiveth him that sent me.

10:41 He that receiveth a prophet in the name of a prophet shall receive a prophet's reward; and he that

http://breachrepairers.webs.com/ 214
receiveth a righteous man in the name of a righteous man shall receive a righteous man's reward.

10:42 And whosoever shall give to drink unto one of these little ones a cup of cold water only in the name
of a disciple, verily I say unto you, he shall in no wise lose his reward.

http://breachrepairers.webs.com/ 215
Chapter 11 – Prophet’s of God Rejected
Reading
 Imprisonment of John the Baptist
 Desire of Ages – Chapter The Invitation

11:1 And it came to pass, when Jesus had made an end of commanding his twelve disciples, he departed
thence to teach and to preach in their cities.

11:2 Now when John had heard in the prison the works of Christ, he sent two of his disciples,

DA 214 For a time Herod feebly sought to break the chain of lust that bound him; but Herodias
fastened him the more firmly in her toils, and found revenge upon the Baptist by inducing Herod to
cast him into prison.

The hold that a woman can have on a man

DA 215 These questions were not without effect. Doubts which otherwise would never have arisen
were suggested to John. Satan rejoiced to hear the words of these disciples, and to see how they
bruised the soul of the Lord's messenger. Oh, how often those who think themselves the friends of a
good man, and who are eager to show their fidelity to him, prove to be his most dangerous enemies!
How often, instead of strengthening his faith, their words depress and dishearten!

John was alone. Sometimes the closeness of a friend can prove to be a downfall.

DA 216 There were hours when the whisperings of demons tortured his spirit, and the shadow of a
terrible fear crept over him…For the success of this mission his whole life had been sacrificed. Had it
been in vain?

The devil cannot force us to sin, but can induce thoughts which could lead to sin. Satan could
possibly have said to John “have you made a mistake?” Things to tempt him to doubt Jesus.The
parallel passage of Matt 11:2-11 is Luke 7:18-28.

Where was John the Baptist during this encounter? In prison.Who had imprisoned him? Why?

Mark 6:17-18

Herod. John had rebuked Herod for marrying his brother’s wife. What question did John the
Baptist ask Jesus through his disciples? He asked if Jesus was the one, meaning the Messiah. How
do we know? Matt 3:3 – John knew he was the forerunner for the Lord, the Messiah.

What had been confirmed in John the Baptist heart earlier?

John 1:29-34

John the Baptist testifies that he bore witness to baptism of Christ and was convinced that He was
the Son of God. He calls Jesus the Lamb that takes away the sin of the world, that is the Messiah.

Was John the Baptist really doubting Christ? What was he really trying to ask Jesus?

Mark 1:15

http://breachrepairers.webs.com/ 216
Remember, John the Baptist was now in prison. He was hearing all the works that Jesus was doing.
He now heard that Jesus was preaching that now is the Kingdom of God at hand. If you were in
prison, and you heard that Christ was about to set up His Kingdom, what would you think if you
were John the Baptist?

Answer – why didn’t Jesus overthrow the Romans? More importantly, why didn’t Jesus save John
the Baptist if He was setting up His kingdom now? Was Jesus really the Messiah? Therefore, when
John the Baptist asked Jesus if “we look for another”, he wasn’t doubting Jesus, but he was asking
WHY? Why don’t you come to save me out of my persecution.

How does John the Baptist represent us? What personal application can we draw? John the
Baptist represents us when we feel like Christ has abandoned us. John the Baptist was expecting a
miracle from Christ for himself. Why hadn’t Christ overthrown Herod or the Romans and saved His
servant? PERSONAL APPLICATION QUESTION – don’t we often feel like this? Have you had such an
experience where you are being persecuted and you ask Christ, should I look for another? Not that
you are doubting Christ’s divinity, but you wonder whether he really cares about what you are
going through. (Ask for personal testimony)

11:3 And said unto him, Art thou he that should come, or do we look for another?

11:4 Jesus answered and said unto them, Go and shew John again those things which ye do hear and see:

Jesus didn’t prove to them that He was the Messiah. He let the disciple watch Him as He began to
comfort and heal. That was the testimony that they were to bear back to John

What does Jesus ask John’s disciples to do? What application can we learn from this?

Hear and see. Basically, Jesus trusted that whatever John’s disciples saw Christ do, it would
encourage John the Baptist. Application: In our own Christian experience, its important for us to
hear and see. Hear what? Rom 10:17 – Faith comes by hearing, hearing by the word of God. So its
important for us to hear God’s voice through His word. This will build faith. See what? John 14:6-9
– Once hear God’s word, its important for us to see Jesus. By seeing Christ, we see the Father.

Spiritual Application: This is also the problem of the Laodicean church – Rev 3:18. They are
counseled to buy eyesalve, because they are blind. What will help them see? 1 Sam 9:9 – prophets
were/are called seers. Therefore the writings of the prophets are to help God’s people see.

What were John’s disciples to tell him that they had seen?

Matt 11:5

Blind can see, lame can walk, lepers cleansed, deaf hear, dead raised, gospel preached to poor. How
was this a comfort to John the Baptist? The words reminded John of the prophecies of Isaiah
regarding Christ.

Luke 4:18

Jesus said that the Spirit of the Lord was upon Him and His mission was to those things that John
was hearing. This is a quotation from Isa 61:1

http://breachrepairers.webs.com/ 217
Isa 61:1

The word anointed. What does it mean? Messiah.

Isa 42:6-8

Verse 6 especially tells John that Jesus was to be a light to the Gentiles too.

Summary: Jesus gave John more insight into His mission, that as the Messiah, He was there as the
anointed one, to heal the blind, lame, lepers, deaf, dead, and bring light to the Gentiles.

What was Jesus gentle rebuke to John?

Matt 11:6

Blessed is he who shall not be offended in me. John the Baptist had felt he was unfairly treated,
because Christ had not helped to release him. Jesus gently rebuked him for his doubts and asked
him to have faith and accept his situation.

What did Jesus call John the Baptist after his disciples had left? Why?

Luke 7:27-28.

The greatest of all prophets. Why? Because he prepared the way for Christ. Application – How can
we be greater than John the Baptist? Preparing the way for Christ’s second coming. We too, as
Seventh-day Adventists, are a type of John the Baptist.

What type of character and appearance did John have according to Jesus? How does it apply
to us?

Luke 7:24-25

“Not to be a reed blown in the wind” = not blown around by every wind of doctrine (Eph 4:14)

Luke 7:25

“Not clothed in soft raiment” – not dressed like kings and queens. Application – How does this
apply to us who are a type of John the Baptist? In these last days, as we preach the coming of Christ,
we also should not be easily persuaded by every wind of doctrine, nor should we portray ourselves
as wealthy. John the Baptist lived a simple lifestyle and dressed simply, not according to the
fashions of his time.

How did Jesus prove His Messiahship to John's disciples? What important lesson can we
learn from that? Jesus didn't use His sermon or testimony. He used His works of healing.
Important Lesson: The greatest evidence that we are bearing a message from Christ is not in what
we preach, but in what we do. It is through the fruits of our labor that people will see whether we
are from God or not.

Conclusion: Sometimes we are in discouragement because we only have partial understand of God's

http://breachrepairers.webs.com/ 218
will. Like John, he knew about the ministry of Jesus in righteousness yet John was hoping for the
earthly kingdom like others during his time. His misunderstanding of Jesus' spiritual kingdom made
him doubt the Messiahship of Jesus in the time of trials. Therefore in order for us to go through
trials in life, we need to understand God's word as whole and not partially.

11:5 The blind receive their sight, and the lame walk, the lepers are cleansed, and the deaf hear, the
dead are raised up, and the poor have the gospel preached to them.

11:6 And blessed is he, whosoever shall not be offended in me.

This was the message to John. The rest was the testimony. Jesus was telling John not to be offended
in Jesus. Many people run well at the start, but fall right at the end. John made one mistake at the
end, he doubted Jesus.

DA 218 The Saviour's words, "Blessed is he, whosoever shall find none occasion of stumbling in Me,"
were a gentle reproof to John. It was not lost upon him. Understanding more clearly now the nature
of Christ's mission, he yielded himself to God for life or for death, as should best serve the interests of
the cause he loved.

As soon as John heard, he realized that he should live for the Lord. He was content at where he was.

11:7 And as they departed, Jesus began to say unto the multitudes concerning John, What went ye out
into the wilderness to see? A reed shaken with the wind?

Jesus was not saying that John was a reed shaken in the wind, or else people would not have gone
out into the wilderness to see him. Pharisees who were swayed by the demands of other people –
that is what EGW says of a reed shaken in the wind. God does not like two camps, He likes only one.
There is only one truth – not multiple versions of them. If you have an adherence to one truth, you
will find that it will attract people – if you are known for believing one thing.

11:8 But what went ye out for to see? A man clothed in soft raiment? behold, they that wear soft clothing
are in kings' houses.

11:9 But what went ye out for to see? A prophet? yea, I say unto you, and more than a prophet.

People didn’t go out to see his clothing or anything of that sort, but because that he was a prophet.
He was preaching the truth. People wanted to hear him because he was so singular – he was
peculiar. If you are different, that is interesting.

Why was he more than a prophet? Because he saw Jesus. There were 3 people that were called
more than a prophet Moses, John the Baptist, and Ellen G White. If John was more than a prophet to
prepare the way for the first coming, what sort of people should be preparing the way for the Lord

11:10 For this is he, of whom it is written, Behold, I send my messenger before thy face, which shall
prepare thy way before thee.

11:11 Verily I say unto you, Among them that are born of women there hath not risen a greater than
John the Baptist: notwithstanding he that is least in the kingdom of heaven is greater than he.

11:12 And from the days of John the Baptist until now the kingdom of heaven suffereth violence, and the
violent take it by force.

http://breachrepairers.webs.com/ 219
From the time John the Baptist started preaching until now, the kingdom of Heaven suffereth
violence

“if ye will receive it” – This primary application if before the second coming. The problem of their
day, they wanted to see the leaders move to whatever they wanted. They called Jesus a
“winebibber” because he associated himself with the publicans and sinners. But that didn’t deter
Him.

Matt 14:1-2

Herod was scared that it was John the Baptist resurrected. When you have done nothing wrong, you
have nothing to fear. You only have fear only when you have some guilt in you, when you have done
something wrong.

Matt 14:3-10

We find this similar situation in Daniel. The king didn’t want to enforce it. But it was forced upon by
the temptation of the daughter in request of the mother. This brings us to Rev 17. But the mother
doesn’t tempt the father. So in application: The state doesn’t want to enforce it, but the children of
the mother of Harlots enforces it – the image to the beast (apostate Protestantism) on the wish of
the papacy. We see here that the king was also drunk with wine. Wine representing false doctrines.

11:13 For all the prophets and the law prophesied until John.

11:14 And if ye will receive it, this is Elias, which was for to come.

11:15 He that hath ears to hear, let him hear.

11:16 But whereunto shall I liken this generation? It is like unto children sitting in the markets, and
calling unto their fellows,

11:17 And saying, We have piped unto you, and ye have not danced; we have mourned unto you, and ye
have not lamented.

11:18 For John came neither eating nor drinking, and they say, He hath a devil.

11:19 The Son of man came eating and drinking, and they say, Behold a man gluttonous, and a
winebibber, a friend of publicans and sinners. But wisdom is justified of her children.

11:20 Then began he to upbraid the cities wherein most of his mighty works were done, because they
repented not:

11:21 Woe unto thee, Chorazin! woe unto thee, Bethsaida! for if the mighty works, which were done in
you, had been done in Tyre and Sidon, they would have repented long ago in sackcloth and ashes.

11:22 But I say unto you, It shall be more tolerable for Tyre and Sidon at the day of judgment, than for
you.

11:23 And thou, Capernaum, which art exalted unto heaven, shalt be brought down to hell: for if the
mighty works, which have been done in thee, had been done in Sodom, it would have remained until this
day.

11:24 But I say unto you, That it shall be more tolerable for the land of Sodom in the day of judgment,

http://breachrepairers.webs.com/ 220
than for thee.

11:25 At that time Jesus answered and said, I thank thee, O Father, Lord of heaven and earth, because
thou hast hid these things from the wise and prudent, and hast revealed them unto babes.

11:26 Even so, Father: for so it seemed good in thy sight.

11:27 All things are delivered unto me of my Father: and no man knoweth the Son, but the Father;
neither knoweth any man the Father, save the Son, and he to whomsoever the Son will reveal him.

11:28 Come unto me, all ye that labour and are heavy laden, and I will give you rest.

Psa 38:4

The burden that we are bearing is the burden of sin and that burden is on our shoulders

DA 330 The yoke that binds to service is the law of God.

If we take Christ’s yoke upon us, we will go with Christ.

Ps 40:8

Doing God’s will and having His law written in our hearts is one and the same thing.

11:29 Take my yoke upon you, and learn of me; for I am meek and lowly in heart: and ye shall find rest
unto your souls.

When we learn from the school of Christ, we are to learn meekness and humility and then we shall
find rest for our souls.

Principles of true recreation:

DA 361 The rest which Christ and His disciples took was not self-indulgent rest. The time they spent
in retirement was not devoted to pleasure seeking. They talked together regarding the work of God,
and the possibility of bringing greater efficiency to the work.

11:30 For my yoke is easy, and my burden is light.

The Christian is called to sacrifice.

http://breachrepairers.webs.com/ 221
Chapter 12 – The Blind, Dumb Demoniac
Reading
 Desire of Ages – Chapter Come Rest a While
 Desire of Ages – Chapter Who Are My Brethren?

12:1 At that time Jesus went on the sabbath day through the corn; and his disciples were an hungred,
and began to pluck the ears of corn, and to eat.

12:2 But when the Pharisees saw it, they said unto him, Behold, thy disciples do that which is not lawful
to do upon the sabbath day.

12:3 But he said unto them, Have ye not read what David did, when he was an hungred, and they that
were with him;

12:4 How he entered into the house of God, and did eat the shewbread, which was not lawful for him to
eat, neither for them which were with him, but only for the priests?

12:5 Or have ye not read in the law, how that on the sabbath days the priests in the temple profane the
sabbath, and are blameless?

12:6 But I say unto you, That in this place is one greater than the temple.

12:7 But if ye had known what this meaneth, I will have mercy, and not sacrifice, ye would not have
condemned the guiltless.

12:8 For the Son of man is Lord even of the sabbath day.

12:9 And when he was departed thence, he went into their synagogue:

12:10 And, behold, there was a man which had his hand withered. And they asked him, saying, Is it
lawful to heal on the sabbath days? that they might accuse him.

12:11 And he said unto them, What man shall there be among you, that shall have one sheep, and if it fall
into a pit on the sabbath day, will he not lay hold on it, and lift it out?

12:12 How much then is a man better than a sheep? Wherefore it is lawful to do well on the sabbath
days.

12:13 Then saith he to the man, Stretch forth thine hand. And he stretched it forth; and it was restored
whole, like as the other.

12:14 Then the Pharisees went out, and held a council against him, how they might destroy him.

12:15 But when Jesus knew it, he withdrew himself from thence: and great multitudes followed him, and
he healed them all;

12:16 And charged them that they should not make him known:

12:17 That it might be fulfilled which was spoken by Esaias the prophet, saying,

12:18 Behold my servant, whom I have chosen; my beloved, in whom my soul is well pleased: I will put
my spirit upon him, and he shall shew judgment to the Gentiles.

http://breachrepairers.webs.com/ 222
12:19 He shall not strive, nor cry; neither shall any man hear his voice in the streets.

12:20 A bruised reed shall he not break, and smoking flax shall he not quench, till he send forth
judgment unto victory.

12:21 And in his name shall the Gentiles trust.

12:22 Then was brought unto him one possessed with a devil, blind, and dumb: and he healed him,
insomuch that the blind and dumb both spake and saw.

Who was healed? One possessed with a devil, blind and dumb.

12:23 And all the people were amazed, and said, Is not this the son of David?

12:24 But when the Pharisees heard it, they said, This fellow doth not cast out devils, but by Beelzebub
the prince of the devils.

Contrast the different reactions from the audience?

Matt 12:23-24

The common people were amazed and convicted enough to call Jesus the son of David. What did it
mean for the Jews to call Jesus the son of David?

John 7:42

Christ, the Messiah was to come from the seed of David. The Pharisees (the leaders) accused Jesus
of casting out demons by Beelzebub. NOTE: it was the common people in their simplicity who
believed that Jesus was the Messiah. Those who knew the prophecies condemned him.

12:25 And Jesus knew their thoughts, and said unto them, Every kingdom divided against itself is
brought to desolation; and every city or house divided against itself shall not stand:

12:26 And if Satan cast out Satan, he is divided against himself; how shall then his kingdom stand?

12:27 And if I by Beelzebub cast out devils, by whom do your children cast them out? therefore they shall
be your judges.

How does Jesus discredit their thoughts?

Matt 12:25-27

It does not make sense for Satan to use his own name to cast himself out. He may use God’s name in
mockery, but he certainly will not use his own name to do it. If Jesus casts out devils in Beelzebub,
then how about their children, other Jews? They were willing to accept it from other Jews, but not
Jesus.

“They shall be your judges” – the common people who recognized Jesus as son of David were a
rebuke and judgment against the Pharisees. They should greater faith.

12:28 But if I cast out devils by the Spirit of God, then the kingdom of God is come unto you.

http://breachrepairers.webs.com/ 223
What is another name for the Spirit of God? Why do you think He is given this name?

Luke 11:20

Finger of God. Why? Because He acts as our conscience. Pointing out our sins so that we may repent
(like woman caught in adultery). Also in this case, casting out devils.

12:29 Or else how can one enter into a strong man's house, and spoil his goods, except he first bind the
strong man? and then he will spoil his house.

12:30 He that is not with me is against me; and he that gathereth not with me scattereth abroad.

What will be the actions of those for Jesus vs those against? For Jesus = gathers with Christ,
meaning participates in the gospel commission. Against Jesus = scattereth abroad. Those who do
not participate in the gospel work will eventually participate in pushing others away from Christ.

What sin cannot be forgiven? What is this related to in this encounter?

Blasphemy against the Holy Spirit = speaking against the Holy Spirit. It is related to saying Christ,
the Son of God was of Satan, even though the Pharisees felt the conviction of the Holy Spirit (how do
we know? The common people were convicted that Jesus was the Son of David).

Application: How can we commit the unpardonable sin? One way is to ignore the voice of the Holy
Spirit when He is convicting us of sin and we are convinced. Another way is to ignore the prompting
of the Holy Spirit and accuse the servants of God of being from Beelzebub, or constantly put down
God’s servants, Eg. Continual putting down Ellen White and her writings can lead us to committing
the unpardonable sin if we have been convicted that they are true.

12:31 Wherefore I say unto you, All manner of sin and blasphemy shall be forgiven unto men: but the
blasphemy against the Holy Ghost shall not be forgiven unto men.

12:32 And whosoever speaketh a word against the Son of man, it shall be forgiven him: but whosoever
speaketh against the Holy Ghost, it shall not be forgiven him, neither in this world, neither in the world
to come.

12:33 Either make the tree good, and his fruit good; or else make the tree corrupt, and his fruit corrupt:
for the tree is known by his fruit.

How can we tell those who are for Christ vs. those against? By their fruit.

What is fruit being related to?

Matt 12:34-37

The words from our mouth, which according to v34 come from the heart.

How important is our tongue? Please explain what the verses are trying to say.

Jam 3:3-6

The tongue is compared to a helm of a ship that controls its direction (v4). It has the ability to defile
the whole body just by what it says (v6).

http://breachrepairers.webs.com/ 224
Application: Our tongue, the words we speak, help to steer the direction of our heart. If we speak a
lot of vulgar words, our heart will naturally gravitate towards vulgar things. If we are sarcastic with
our words, our heart will naturally criticize people all the time. Many times we think our heart
governs our tongue, but it seems from James 3, that our tongue can influence our heart too. Are
your words pure?

12:34 O generation of vipers, how can ye, being evil, speak good things? for out of the abundance of the
heart the mouth speaketh.

12:35 A good man out of the good treasure of the heart bringeth forth good things: and an evil man out
of the evil treasure bringeth forth evil things.

12:36 But I say unto you, That every idle word that men shall speak, they shall give account thereof in
the day of judgment.

12:37 For by thy words thou shalt be justified, and by thy words thou shalt be condemned.

12:38 Then certain of the scribes and of the Pharisees answered, saying, Master, we would see a sign
from thee.

What do the Pharisees then ask Jesus to do?

Show a sign. They wanted a miracle. What story does this remind you of? Remember the nobleman
who was also looking for outward proof.

12:39 But he answered and said unto them, An evil and adulterous generation seeketh after a sign; and
there shall no sign be given to it, but the sign of the prophet Jonas:

12:40 For as Jonas was three days and three nights in the whale's belly; so shall the Son of man be three
days and three nights in the heart of the earth.

12:41 The men of Nineveh shall rise in judgment with this generation, and shall condemn it: because
they repented at the preaching of Jonas; and, behold, a greater than Jonas is here.

What was the sign of Jonah? Compare this with Christ’s signs?

Matt 12:39-42

Jesus rebukes the Pharisees for being an evil and adulterous generation. Compare Jonah vs Christ.
Sign of Jonah = his preaching. This is what lead to the Ninevites repentance and conversion. What
was Christ’s sign to the Jews throughout his life? Healing the sick, casting out devils, preaching. And
yet what did the Jews do? They crucified Jesus.

Conclusion: As a result, the Ninevites and the queen of the south will condemn the Jews because
they simply believed on the servant of the Lord’s words. BUT the Jews were sent someone far
greater – The Son of God. And even with signs, wonders and preaching they would not believe.

12:42 The queen of the south shall rise up in the judgment with this generation, and shall condemn it:
for she came from the uttermost parts of the earth to hear the wisdom of Solomon; and, behold, a
greater than Solomon is here.

http://breachrepairers.webs.com/ 225
12:43 When the unclean spirit is gone out of a man, he walketh through dry places, seeking rest, and
findeth none.

12:44 Then he saith, I will return into my house from whence I came out; and when he is come, he findeth
it empty, swept, and garnished.

12:45 Then goeth he, and taketh with himself seven other spirits more wicked than himself, and they
enter in and dwell there: and the last state of that man is worse than the first. Even so shall it be also
unto this wicked generation.

How do we prevent our house from being empty so that it will not be in a worse state than
before?

Matt 12:43-45

Luke 11:24-28

Blessed are they that hear the word and keep it. We must keep the word. What does it mean to
“keep” the Word?

James 1:22-27

To keep the word = To be a doer of the word. What does it mean to be a doer of the word? 1. Bridle
the tongue. Why? We have already read in James 3. 2. Visit the fatherless and widows. Why?
Because true love for God is manifested in love for others and this translates into acts of love.
Fatherless and the widows are the 2 most challenging groups to care for. They need the most love.
A doer of the word puts his belief into action. 3. Keep himself unspotted from the world.

Why is this compared with filling our house? Because after conversion, God expects us to fill our
house with new habits. What kind of habits? Habits of good works listed in James 1:22-26. This will
prevent the demons from coming back. What kind of demons? Demons of our past wicked lives –
addiction of any kind – drugs, partying, alcohol, bad music, pornography. All these are overcome by
believing in Christ and then asking for His help to fill our lives with good habits, which are good
works stated in the Bible. This will keep us busy for the Lord and stop us from missing the ‘old’ life
we have been saved from.

12:46 While he yet talked to the people, behold, his mother and his brethren stood without, desiring to
speak with him.

12:47 Then one said unto him, Behold, thy mother and thy brethren stand without, desiring to speak
with thee.

His brothers heard of this, and also of the charge brought by the Pharisees that He cast out devils
through the power of Satan. They felt keenly the reproach that came upon them through their
relation to Jesus. They knew what a tumult His words and works created, and were not only
alarmed at His bold statements, but indignant at His denunciation of the scribes and Pharisees.
They decided that He must be persuaded or constrained to cease this manner of labor, and they
induced Mary to unite with them, thinking that through His love for her they might prevail upon
Him to be more prudent.

http://breachrepairers.webs.com/ 226
It seems that Jesus’ brothers were older than Him. So this could be Joseph’s second marriage.

Matt 12:31

Blasphemy against the Holy Ghost is rejecting the promptings of the Holy Spirit unto repentance.

Matt 12:36

It’s very easy to say something you didn’t mean, but you start believing it anyways.

12:48 But he answered and said unto him that told him, Who is my mother? and who are my brethren?

12:49 And he stretched forth his hand toward his disciples, and said, Behold my mother and my
brethren!

12:50 For whosoever shall do the will of my Father which is in heaven, the same is my brother, and
sister, and mother.

http://breachrepairers.webs.com/ 227
Chapter 13 – Parables on the Kingdom
13:1 The same day went Jesus out of the house, and sat by the sea side.

13:2 And great multitudes were gathered together unto him, so that he went into a ship, and sat; and the
whole multitude stood on the shore.

As Jesus was sitting in the ship He saw all types of people: lawyers, doctors, partisans, merchants,
politicians, carpenters, farmers, tax collectors, fathers, mothers, etc. And He isn’t moved by the
external display. As He looks at this vast multitude of people He has one question in mind. “How do
they receive my words?” How do they respond to the Word of God? And because He is more than
just a man and has the ability to read hearts, He separates the multitude into four categories.

Four different types of hearts

Matt 13:2 …great multitudes…whole multitude.

Matt 13:3-9 – Multitude divided into 4 grounds: Wayside, stony, thorny, and good ground. App:
Every time the word of God goes forth there are four categories. Each of us is in one of these
categories. We want to believe I am a good ground hearer, but we would be naïve to believe that.

Matt 13:10-11, 13, 15 …mysteries of the kingdom…

This parable describes the mysterious working of God upon different types of hearts, and how come
they receive or reject the truth. In other words, I am speaking this parable to them because of the
condition of their hearts. And Jesus is warning them.

Prophetic in nature

Matt 13:13-14 ...in them is fulfilled the prophecy…

Who are them? The multitudes.

“prophecy” – The parable is prophetic in nature. At that time the multitude was fulfilling prophecy.
App: Many times we talk about what is about to take place in the future, but do you know we are
fulfilling prophecy now! The way you respond to the Word of God every time you hear it
determines how prophecy is being fulfilled.

Acts 13:26-27 …they have fulfilled [them] in condemning [him]…

Note: Parable also represents the condition of the church just prior to the Second Coming of Jesus.
Which Prophecy?

Isa 6:8-11

Prophesy until the land be desolate

Isa 6:9 …hear ye indeed…

[margin without ceasing] they are continually hearing. How long?

http://breachrepairers.webs.com/ 228
Isa 6:11

Until the destruction of Jerusalem by Babylon. (type of world). App: keep telling them, maybe
someone will listen. How long does God have. To keep sending people to tell us get your home right,
etc.
Conclusion/Appeal

Matt 13:16

Blessing two (show one today)

Mark 4:12

Perceive, understand, convert, sins forgiven. Blessing is that your sins will be forgiven. There are
conditions: Seeing, hearing, conversion, forgiveness of sins. Must meet the conditions to have the
full blessing. Tell me, how do you respond to the Word of God? He that hath ears to hear let him
hear.

Part 2: The farmer in this parable is planting field run crops. He is not planting raised bed crops.
There are a few differences with each. Field run crops are grown outdoors in soil that is not heavily
modified. While raised bed crops are grown in soils that are heavily modified. Those crops have the
advantage of receiving all the necessary nutrients in the soil. They have the right minerals, vitamins,
better moisture, pest control and climate control. Not so with the field run crops. They have
seasonal vagaries; they are susceptible to pests, climate, and weeds. And in this parable each
ground has an antagonist. The wayside is overcome by pests, the stony ground is overcome by
climate, and the thorny ground is over come by weeds. Only one survives.

Every ground has an antagonist (Represents those in the church)

Matt 13:3-9

Main objective is to bring forth fruit. Every stage in the development is stifled, except for the good
ground. Wayside germinates and is snatched; Stony ground spring up, has not root and withers
away; Thorny ground germinates, has root, and right when it is about to bring forth fruit it is
choked. Good ground endures all three and brings forth fruit.

Matt 13:19

Wayside & Wicked One

Luke 8:5

Human agencies and supernatural agencies are used to take away the Word. (people trying to force
truth, parents criticizing, friends coming and distracting from the message). The person gets
discouraged and is devoured up.

Matt 13:20-21

Stony place & the Sun. This person is very impulsive. This person doesn’t allow the fowls to
discourage him. But when persecution comes they can’t handle it

http://breachrepairers.webs.com/ 229
Matt 13:21 …not root…

Col 2:6-7 Rooted and built p in him, and stablished in the faith.

They live by feelings instead of faith.

Matt 13:22

Seed in the Thorns. This person are able to dodge the fowls, don’t distract me, don’t take away the
Word from me. I don’t care if you persecute me for what I believe.

Mark 4:7 And some fell among thorns, and the thorns grew up, and choked it, and it yielded no fruit.

Luke 8:7 …the thorns sprang up with it.

Both the crop and the thorns are growing together.

1 Pet 1:23 Being born again...seed…word of God.

Is this the same seed as the parable of the sower? Yes

1 Pet 2:1-2

Milk = doctrines, they can explain with accuracy.

1 Pet 2:3

Gracious. How? V.1 All have to do with the heart. Thorny ground hearers have experienced the
grace of God. But they are not cultivating that grace. These are the weeds!

Matt 13:23

Seed bears fruit 30, 60, 100. They all endure the fowls, sun, and weeds.

Luke 8:15 …heard the word, keep it, and bring forth fruit with patience”

Who does this refer to?

Rev 14:12 Here is the patience…

It is speaking about the 144,000. They endured the previous condition of each ground.

Mark 4:29 But when the fruit is brought forth, immediately he putteth in the sickle, because the
harvest is come.

When does this take place?

Rev 14:15 Thrust in thy sickle, and reap: for the time is come for thee to reap; for the harvest of the
earth is ripe.

Conclusion

http://breachrepairers.webs.com/ 230
Jas 5:7 Behold, the husbandman waiteth precious fruit of the earth”

Appeal: Is this your condition, maybe you are a wayside, stony, or thorny ground hearer, but God
says I am waiting on you. His arms outstretched come to me. I will give you the early rain. That is
the working of grace in the heart and life. It is power to bear forth fruit, and then He’ll give the latter
rain so you will be complete.

COL 33 By the parable of the sower, Christ illustrates the things of the kingdom of heaven, and the
work of the great Husbandman for His people. Like a sower in the field, He came to scatter the
heavenly grain of truth. And His parable teaching itself was the seed with which the most precious
truths of His grace were sown. Because of its simplicity the parable of the sower has not been
valued as it should be. From the natural seed cast into the soil, Christ desires to lead our minds to
the gospel seed, the sowing of which results in bringing man back to his loyalty to God. He who gave
the parable of the tiny seed is the Sovereign of heaven, and the same laws that govern earthly seed
sowing govern the sowing of the seeds of truth.

Part 3: Jesus presented the parable as a warning to those who were listening because He knew what
would take place in the future.

Represents the condition of the church prior to the Second Coming

Luke 8:5

Wayside “trodden down” by who?

Lk 21:20, 24 …Jerusalem shall be trodden down of the gentiles…

If you continue to be a wayside hearer you will perish when Jerusalem is destroyed. For us that
represents the end of the world.

Matt 13:20-21 …stony places…offended…

Matt 24:10

Many in the church will be offended. They are stony ground hearers. They are over come by false
Christ’s and false prophets because they rely on their feelings and emotions instead of faith. And
persecution causes them to fall away.

Matt 13:22 …thorns…care of this world…

Lk 21:34 …hearts…cares of this life…

Thorny ground hearer overtaken by the day of the Lord.

Rom 13:12-14 …day is at hand…put on the armour of light”

1 Thes 5:4-8 …day should overtake you..breastplate of faith and love…

Note: Thorny ground hearers started out well, they escaped the fowls, endured persecution, and then
they got weary, they fell asleep because His coming seemed so far away. They got weighed down with
the cares of this life, they got drunk, they lost the armor, and made provision for the flesh.

http://breachrepairers.webs.com/ 231
Matt 13:23 …good ground…beareth fruit….

Eph 5:9 …fruit of the Spirit [is] in all goodness and righteousness…

Picture of the Second Coming. What is He reaping? Fruit (144,000 called first fruits)

Majority will not go to heaven. PA: I realized I have been on all three grounds. 75% that I would not
make it. Same with the church because this shows their condition before the Second Coming. The
majority won’t make it.

Matt 7:13-14, 20-21

Many take the road to destruction, Few find road to life.

Con/Appeal. Sounds I want to hear

Isa 11:8 And the sucking child shall play on the hole of the asp, and the weaned child shall put his
hand on the cockatrice' den.

Zech 8:4-5 …streets of Jerusalem..boys and girls playing…

EW 19 Mount Zion was just before us, and on the mount was a glorious temple, and about it were
seven other mountains, on which grew roses and lilies. And I saw the little ones climb, or, if they
chose, use their little wings and fly, to the top of the mountains and pluck the never-fading flowers.

Is 30:26 Moon as bright as the sun, sun sevenfold brighter.

See God face to face. God is waiting on us. We can only have this if we are ready. Do you want this
experience.

13:3 And he spake many things unto them in parables, saying, Behold, a sower went forth to sow;

EGW: says if you understand this parable, it will help you understand all other parables.

Why?
Luke 8:11

Matt 13:19

When we study parables, we have to first define the symbols that are being used. In Matthew it is
assumed that the seed is the Word of God. Why a seed though? Inside of a seed is life. The seed has
to die and has to find the right conditions.

1 Thes 2:13

The Word of God has power. It has self fulfilling power.

Matt 13:22

This plant being described does not die. It springs up but just does not bear fruit. What is fruit in the
Bible?

http://breachrepairers.webs.com/ 232
Gal 5:22

Fruit of the Spirit

Prov 11:30-31

Parallel for the righteous is wise. The fear of the Lord is the beginning of wisdom. So a righteous
man is a wise man. Winning souls is a fruit of the righteous. The primary reason of this parable is
about winning souls.

Matt 13:8

13:4 And when he sowed, some seeds fell by the way side, and the fowls came and devoured them up:

13:5 Some fell upon stony places, where they had not much earth: and forthwith they sprung up, because
they had no deepness of earth:

13:6 And when the sun was up, they were scorched; and because they had no root, they withered away.

13:7 And some fell among thorns; and the thorns sprung up, and choked them:

13:8 But other fell into good ground, and brought forth fruit, some an hundredfold, some sixtyfold, some
thirtyfold.

13:9 Who hath ears to hear, let him hear.

13:10 And the disciples came, and said unto him, Why speakest thou unto them in parables?

Why did Jesus speak in parables?

Matt 13:11-15

To reveal the mysteries of the kingdom of heaven. Because they did not understand spiritual things.
To keep the truth from the enemy only those that had spiritual discernment could understand

13:11 He answered and said unto them, Because it is given unto you to know the mysteries of the
kingdom of heaven, but to them it is not given.

13:12 For whosoever hath, to him shall be given, and he shall have more abundance: but whosoever hath
not, from him shall be taken away even that he hath.

13:13 Therefore speak I to them in parables: because they seeing see not; and hearing they hear not,
neither do they understand.

13:14 And in them is fulfilled the prophecy of Esaias, which saith, By hearing ye shall hear, and shall not
understand; and seeing ye shall see, and shall not perceive:

13:15 For this people's heart is waxed gross, and their ears are dull of hearing, and their eyes they have
closed; lest at any time they should see with their eyes, and hear with their ears, and should understand
with their heart, and should be converted, and I should heal them.

13:16 But blessed are your eyes, for they see: and your ears, for they hear.

http://breachrepairers.webs.com/ 233
13:17 For verily I say unto you, That many prophets and righteous men have desired to see those things
which ye see, and have not seen them; and to hear those things which ye hear, and have not heard them.

13:18 Hear ye therefore the parable of the sower.

13:19 When any one heareth the word of the kingdom, and understandeth it not, then cometh the wicked
one, and catcheth away that which was sown in his heart. This is he which received seed by the way side.

Who does the sower represent?

Matt 13:37

The Son of Man What does the ground and seed represent?

Matthew 13:19

Luke 8:11

“Ground” – heart

“Seed” – The Word of the God

What is the common characteristic in all the groups of people? They all hear the Word of God What
does the four different grounds represent, give Bible texts? Sown by the wayside

Matt 13:19

Hears the word of God. Understands it not. Then Satan (the wicked one) comes by takes the words
out of their hearts. Sown on the stony place

Matt 13:20, 21

Luke 8:13

Hears the word, receives it joyfully. Has no root. Believe for a while and when temptation comes
they fall away. Sown on thorny ground

Matt 13:22

Luke 8:14

Hears the word. Thorns choking seed = Cares of this world, deceitfulness of riches, and pleasures of
this life choke the Word. Becomes unfruitful, brings no fruit. Sown on good ground

Matt 13:23

Luke 8:15

Hears the word. Understands it. In an honest and good heart hear the word and keep it. Bares and
produces much fruit with patience. According to Scripture what does it mean when someone hears
the Word of God?

http://breachrepairers.webs.com/ 234
Rom 10:17

They all have faith. Who else in the bible has faith and what lesson can we learn for that?

Jas 2:19, 20

The devils and believe and tremble. It is not enough to have faith, we must have faith that works.
Good ground contains manure, what does that mean if we want to have good ground?

Phil 3:8

Count ourselves as dung will be good ground for the Word of God to produce fruit in our lives.
According to the parable how can we bring forth fruit? Hear, understand—have faith. Produce
works. We must have a faith that works. What does it mean to have a root experience?

Eph 3:17

Col 2:6-7

Root is in a dark place, no light, all by itself – means that our root experience needs to be personal,
nobody around, our own private devotion lives is important for us being rooted in Christ. We are to
be rooted and grounded in love. We are to be rooted in Christ and established in the faith

Summarize: There are 4 groups of people. One that hears the truth but doesn’t understand it and
losses interest and no fruit produced. Another group receives the truth but has no deep root so
when trials come they fall out of the faith. Another group receives the truth but the cares and
pleasures of the world take priority in the life and the people becomes unfruitful in the truth. The
last group has a honest and good heart when he hears and understand the truth, that he keeps the
truth and produces much fruit
Which group are you? In order for us to produce fruit we must: Have a faith that works, its not
enough to believe; we are to understand, believe and keep the words we hear. He are to count
ourselves as nothing. We must have a personal experience with Jesus (root experience). We are to
be grounded in Christ, in His love.

13:20 But he that received the seed into stony places, the same is he that heareth the word, and anon
with joy receiveth it;

13:21 Yet hath he not root in himself, but dureth for a while: for when tribulation or persecution ariseth
because of the word, by and by he is offended.

13:22 He also that received seed among the thorns is he that heareth the word; and the care of this
world, and the deceitfulness of riches, choke the word, and he becometh unfruitful.

13:23 But he that received seed into the good ground is he that heareth the word, and understandeth it;
which also beareth fruit, and bringeth forth, some an hundredfold, some sixty, some thirty.

Sower and Wheat & Tares (Must go together). Sower covers from the gospel seed up to 2nd Coming.
Wheat and tares covers from gospel seed to end of the millennium. Wheat and tares defines what
type of seed was used in previous parable. Wheat and tares focuses on the good ground and moves
forward.

http://breachrepairers.webs.com/ 235
Wheat, Tares, and Servants

Matt 13:24-26 …good ground…forth fruit…

Explains good ground in previous parable. Wheat seed is what the previous farmer wanted to
produce

Matt 13:27-30

Focus on the attitude of the Servants. Servants = Front line workers. Wheat & Tares =
church
members.

Wheat & Tares

Matt 13:38 …children of the kingdom…children of the wicked one”

Children of the kingdom V.43 “righteous” (no iniquity, don’t offend).

Eph 1:4,7 Holy and without blame, live by Grace.

Rom 8:14 Led by the Spirit.

Children of the wicked one (devil) V.41 do iniquity and offend. 1 John 3:9-12 Seed remain in. Love
their brother.

Note: Tares have all the characteristics of the previous 3 grounds. Wicked one. Where did he get his
children? Wayside. All things that offend? Stony ground. Children of wicked one have problem with
lusts. Thorny ground.

Con/Appeal:

1 Jn 3:10 …loveth not his brother…

13:24 Another parable put he forth unto them, saying, The kingdom of heaven is likened unto a man
which sowed good seed in his field:

13:25 But while men slept, his enemy came and sowed tares among the wheat, and went his way.

1. What do these represent? (give Bible text)


o Sower = son of man (V37)
o Enemy = devil (V39)
o Good seed = children of the kingdom (V38)
 Seed = Word of God (Luke 8:11)
o Tares = children of the wicked one (V38)
o Field = this world (V38)
 Specifically in the world where the Christian are. The church.

http://breachrepairers.webs.com/ 236
o Harvest = the end of this world (V40)
o Reapers = Angels (V39)
2. Why is the kingdom of heaven likened to a man that sows good seed?
o Christ sought to turn their thoughts from the hope of an earthly kingdom to
heavenly kingdom.
3. What does the Bible say about the children of the kingdom Vs the children of the wicked
one?
o Children of the kingdom
 Converted (Mat 18:3)
o Children of the wicked one
 I John 3:8-24
4. When will the tares appear, and what makes the difference?
o The tares will not be recognized until wheat has sprung up and brought forth fruit
o That means the tares will grow together with the wheat and it will look just like the
wheat before the wheat has brought forth fruit
5. What do “blade was sprung up”, and “brought forth fruit” mean, and what do they
represent?
o “Blade was sprung up” means it is growing (Mark 4:26-28)
o After the seed is sown, there is a growing time taking place before its fruits appear.
o When we receive the word of God in our heart, we start growing spiritually. And
there may not be fruits right away, but there will come a point, when every
individual will bring forth his fruits.
o After we have received the Word of God. We shall produce fruits in 2 ways:
 Fruit of the spirit (Gal 5:22)
 One who is winning souls (Pro 11:30)
6. What did the servants ask, and what did they want to do?
o They asked why there were tares. Because they knew that only the good seed were
sown, but there were tares among the wheat in the field.
o The servants wanted to pluck the tares out.
7. Why can’t the tares be plucked, and how can it apply to us?
o Because both wheat and tares’ roots are intertwined together. If the tare is plucked
up, the root of the wheat will be injured
o If one of our family members committed a sin and God plucked him out of the
church right away, what will happen to our faith? It will shake. Right? God will not
judge His church right away because He gives us time and opportunity to change.
 Examples: God did not destroy Satan right away (optional)
8. When is the separation of wheat and tares taking place, and what does it represent?

http://breachrepairers.webs.com/ 237
o In the harvest = end of the world
o In the end of the world, wheat and tares, righteous and wicked, will be separated.
The righteous will be gathered and those that do iniquity will be burned in the fire(V
39-43)
 Hell fire is not taking place right now.
9. Based upon the answer in question 8, what should we be expecting in our church today and
what should we do?
o Since the separation will not take place until the 2nd coming of Jesus or the close of
probation for God’s church, then we will have 2 groups of Christians in our church,
wheat and tares.
o Humanly speaking, there is no way that we can tell who is the wheat and who are
the tares outwardly in our church. And it is really not our job to try to find out who
is the wheat and who are the tares, because we can’t read people’s thoughts or
hearts. Therefore we can’t judge people outwardly
o There are people who you will never know what they will become when they are
converted.
 Example: one the preacher used to have different colors of hair and tattoos
all over, but after conversion, he is a great preacher (optional)
 So if we see someone who doesn’t look like wheat to us, what we can do is to
pray for him.
10. How does this parable relate to the parable of sower?
o The parable of the sower reveals the 4 conditions of the hearts.
 Way side, stony ground, thorny place, and good ground.
o Out of these 4 conditions of the heart, they can be divided into 2 groups. First, the
seed of the word of God will grow in their hearts and bring forth fruits. Second, the
seed of the word of God will grow and die with no fruits or it will not grow at all.
11. How do I know if I am wheat? And what should I do if I am a tare?
o When the wheat grows, we will see fruit (V26)
o If we know that we are not wheat, does that mean we are hopeless? No
 if any man [be] in Christ, [he is] a new creature (II Cor 5:17)
 Christ alone can help us to have a complete character transformation. And
he alone can help us to be fruitful.
13:26 But when the blade was sprung up, and brought forth fruit, then appeared the tares also.

13:27 So the servants of the householder came and said unto him, Sir, didst not thou sow good seed in
thy field? from whence then hath it tares?

13:28 He said unto them, An enemy hath done this. The servants said unto him, Wilt thou then that we go
and gather them up?

13:29 But he said, Nay; lest while ye gather up the tares, ye root up also the wheat with them.

13:30 Let both grow together until the harvest: and in the time of harvest I will say to the reapers,

http://breachrepairers.webs.com/ 238
Gather ye together first the tares, and bind them in bundles to burn them: but gather the wheat into my
barn.

13:31 Another parable put he forth unto them, saying, The kingdom of heaven is like to a grain of
mustard seed, which a man took, and sowed in his field:

What is the common lesson that Jesus is trying to illustrate in all these parables?

He is trying to illustrate the kingdom of Heaven

What must we first understand?

Matt 6:33

We must first seek the kingdom of God

What is the kingdom of heaven?

Luke 17:20-21

Kingdom of God is in you

Rom 14:17

Kingdom of God is righteousness, peace and joy in the Holy Ghost. How can we receive the
Kingdom of Heaven?

Matt 12:28

Luke 8:1-2

Luke 10:9

We must have devils cast out of us

Healing

Matt 21:43

Gal 5:22-25

Fruits of the spirit

How can we share the Kingdom of Heaven?

Matt 10:7-8

13:32 Which indeed is the least of all seeds: but when it is grown, it is the greatest among herbs, and
becometh a tree, so that the birds of the air come and lodge in the branches thereof.

What are the details brought out in the parable of the mustard seed?
Least of all seeds, when grown is the greatest among herbs, becomes a tree and birds of the air

http://breachrepairers.webs.com/ 239
come and lodge in it

What does a seed represent?

Luke 8:11

Word of God

What does a tree represent?

Psa 1:3, 6

Righteous man

What application can we draw from this and how does this help us to understand more the
kingdom of heaven? The growth of a Christian is brought out. It begins with faith through studying
the word of God (Rom 10:17). It begins in a humble manner in the soil where no one can see it.
Greatness/success is not measured by the stature of a man but his faith and dependence on God.
Only then will God exalt man.

13:33 Another parable spake he unto them; The kingdom of heaven is like unto leaven, which a woman
took, and hid in three measures of meal, till the whole was leavened.

What is leaven?

Matt 16:6-12

Luke 12:1

1 Cor 5:6-8

Is this leaven in the parable representative of good or bad things? GOOD

How does it apply to those that are preparing for the kingdom of Heaven? Leaven cannot be
seen and so growth in God takes place in the heart. Illustrate it being opposite to what was read in
question 11 – that is the principles of heaven – opposite to hypocrisy. Ask them what is opposite to
hypocrisy.

What is the common illustration of the mustard seed and leaven? They both are hidden. One in
the earth, the other in the bread.

Matt 13:33

Leaven in the Bible is a symbol of sin. But here leaven represents something spiritual.

1 Thes 2:13

So the leaven in the parable represents God’s word. Leaven in reality is a form of bacteria. When it
goes inside of something it eats the sugar and then produces carbon dioxide to make it rise. It was
his in three measures of meal .

http://breachrepairers.webs.com/ 240
1 Thes 5:23

This represents a person, physical, mental, and spiritual.

13:34 All these things spake Jesus unto the multitude in parables; and without a parable spake he not
unto them:

Jesus and Parables


Why did Jesus use parables?

Matt 13:34

Eg. “The kingdom of Heaven is like…” Jesus always came down to their level to help them
understand. Helps them bridge from the unknown to the known.

The Parables
1. Define the symbols first.

13:35 That it might be fulfilled which was spoken by the prophet, saying, I will open my mouth in
parables; I will utter things which have been kept secret from the foundation of the world.

13:36 Then Jesus sent the multitude away, and went into the house: and his disciples came unto him,
saying, Declare unto us the parable of the tares of the field.

13:37 He answered and said unto them, He that soweth the good seed is the Son of man;

13:38 The field is the world; the good seed are the children of the kingdom; but the tares are the children
of the wicked one;

13:39 The enemy that sowed them is the devil; the harvest is the end of the world; and the reapers are
the angels.

13:40 As therefore the tares are gathered and burned in the fire; so shall it be in the end of this world.

When do the tares spring up? After the wheat has brought forth fruit. When Jesus taught parables,
they were full of meaning. When you study a parable never pass over anything as unimportant.
Everything that He gave was significant.

Matt 13:24-29

There are 2 groups, the man that sowed the seed and the servant. Define the symbols:

COL 70 (Chapter called “Tares”)

What does the field represent in the Bible?

Matt 13:37-38

It represents the world. In this world, where is God’s people to grow in the church

COL 70 “The field," Christ said, "is the world." But we must understand this as signifying the church
of Christ in the world. The parable is a description of that which pertains to the kingdom of God, His

http://breachrepairers.webs.com/ 241
work of salvation of men; and this work is accomplished through the church. True, the Holy Spirit has
gone out into all the world; everywhere it is moving upon the hearts of men; but it is in the church
that we are to grow and ripen for the garner of God.

When will the church be purified? At the harvest at the second coming. So in the church, do we
expect there to be tares? Of course! Always until the end of time. Why does God say we should not
uproot the tares? None of us are given the work of uprooting the tares. We can’t tell on the surface.
Only God knows. Tares and wheat look the same. Growing side by side, the roots will be
intertwined. If He does it any sooner, He would lose a lot more than just the tares. Hell fire is at the
end of the millennium – verse 40. This clearly states that the destruction of the wicked are at the
end of the world. Eg: Laborer Union in bound to be burn. This parable is always used in Hell Fire
evangelistic sermons.

13:41 The Son of man shall send forth his angels, and they shall gather out of his kingdom all things that
offend, and them which do iniquity;

13:42 And shall cast them into a furnace of fire: there shall be wailing and gnashing of teeth.

13:43 Then shall the righteous shine forth as the sun in the kingdom of their Father. Who hath ears to
hear, let him hear.

13:44 Again, the kingdom of heaven is like unto treasure hid in a field; the which when a man hath
found, he hideth, and for joy thereof goeth and selleth all that he hath, and buyeth that field.

What does the treasure represent: Treasure—the gospel

Prov 2:2-4

1 Cor 1:18-21

Rom 1:16

Col 2:3

Wisdom and knowledge

What is wisdom and knowledge compared to as?

Eccl 7:11, 12

John 14:6

Life – eternal life. Jesus

What does the field represent, that we may find Jesus and have eternal life?

John 5:39

The scriptures

How can we get this wisdom?

http://breachrepairers.webs.com/ 242
1 Cor 1:30

Col 1:27

We have to be in Christ

How can we have Christ in us?

Gal 2:20

Where do we find the treasure in the field, where do we find the gospel?

John 5:39

In the Scriptures. Therefore the field = the Scriptures. How can we practically “sell all that we
have”?

Phil 3:7, 8

Counting everything loss

Mark 10:21

Rich young ruler. Take up the cross. Follow Christ. What was this man’s heart in? Make
application,

Matt 6:21

In the treasure, in the gospel it was on Christ. Main point of the Hidden treasure: Teaches us the
value of the heavenly treasure (the gospel) and the effort that should be made to secure it

What is the major difference between these 2 stories? Similarities: Something is being bought and
in both cases, that which is being bought is obscure. Both are talking about the kingdom of Heaven.
Everything is sold.

Difference: Second man – he seeks for goodly pearls. First man – he stumbled across it by accident.
Pearls were the single most valuable item in Jesus’ day. Today we can’t put our finger on what is the
most valuable thing in the world. But everyone is seekers. Everyone has different things which are
valuable to them. A pearl in the Bible represents Jesus – it is the pinnacle of perfection. The pearl is
the only type of precious thing which you don’t have to modify, to cut to make it better.

13:45 Again, the kingdom of heaven is like unto a merchant man, seeking goodly pearls:

From the previous parable what does the Pearl represent? Pearl—Jesus Christ (Col. 2:3). What does
the merchant man represent. The class of people who were sincerely desiring truth. Why did Christ
use the parable of the hidden treasure and pearl? What are their similarities/differences? Both are
to teach us the value of the truth and how precious Christ is. Also how much effort we are to put in
to securing this treasure/Christ. Similarities/Differences: One man was going through a field one
day and came across the treasure, while the other parable the man was actually seeking for goodly
pearls and he found one of great price. Represent two groups of people that find Christ/truth, one
came across the truth by searching the other was just plowing and came across by accident. Main

http://breachrepairers.webs.com/ 243
point of the Pearl: We are to giving everything that we have to receive the Pearl/Christ. However
this parable also shows Christ (merchantman) seeking His lost inheritance, that pearl of price was
lost humanity, which Christ came to seek.

What does the Bible regard as a great price?

1 Pet 3:4

Meek and quiet spirit

How can we get this pearl of great price?

Matt 11:28-29

We must go to Jesus

What is the difference between the man with the treasure and the man with the pearl? The
man with the treasure – he wasn’t looking for it, he stumbled upon it. The man with the pearl – he
was seeking for that pearl.

What is the similarity between the two? Both sold all that they had for what they found

How can we sell all that we have and what does it mean?

Mark 10:28-30

Mark 8:34-37

To deny ourselves and give up everything for the cause of Christ

How often do we need to apply this principle and why? How is it applicable to us today?

Luke 9:23

This parable can have dual application. The merchant can be us or Jesus. Jesus came from afar
looking for goodly pearls. What did Jesus sell in order to save man? He veiled divinity with
humanity. The merchant was searching for a goodly pearl.

1 Pet 3:4 …a meek and quiet spirit, which is in the sight of God of great price.

How pearl is form. A piece of sand that irritate in the oyster, then it will form pearl from that
irritating piece. Why is the pearl as described as being of a great price? In heaven, it says that the
gates are of one pearl (Rev 21:21). Only those that have overcome sin in this life and formed the
character of Christ, only they will get into heaven.

13:46 Who, when he had found one pearl of great price, went and sold all that he had, and bought it.

13:47 Again, the kingdom of heaven is like unto a net, that was cast into the sea, and gathered of every
kind:

What does the fish represent? People. Jesus said I will make you fishers of men. What is the

http://breachrepairers.webs.com/ 244
gathering at the end of time? What is the net? The net is the gospel that will gather the people at the
end of time. Why is the time up? Because every fish has been caught in the net. All the gospel has
been preached and every fish has made a decision. That does not mean that everybody will be
saved. How to become a good fish? If any man be in Christ, he is a new creature.

2 Cor 5:17

What does the following represent: Casting of the net—is the preaching of the gospel.What does the
net represent?What does this parable teach us about who are in church?There will be good and bad
in the church . This reminds us of the parable of the wheat and tares.The wicked with only be
destroyed after the judgment Main point of the Net: That the gospel will be preached and both good
and evil will be in the church

Who uses a net? Fishermen

What does a fisherman represent?

Matt 4:18-20

Disciples of Christ. What does a disciple of Christ use to fish for men?

Mark 1:15

Mark 16:15-16, 20

13:48 Which, when it was full, they drew to shore, and sat down, and gathered the good into vessels, but
cast the bad away.

13:49 So shall it be at the end of the world: the angels shall come forth, and sever the wicked from
among the just,

13:50 And shall cast them into the furnace of fire: there shall be wailing and gnashing of teeth.

13:51 Jesus saith unto them, Have ye understood all these things? They say unto him, Yea, Lord.

13:52 Then said he unto them, Therefore every scribe which is instructed unto the kingdom of heaven is
like unto a man that is an householder, which bringeth forth out of his treasure things new and old.

There is a householder, a house with treasures. Luke 15, 16 has concurrent parables.

13:53 And it came to pass, that when Jesus had finished these parables, he departed thence.

13:54 And when he was come into his own country, he taught them in their synagogue, insomuch that
they were astonished, and said, Whence hath this man this wisdom, and these mighty works?

13:55 Is not this the carpenter's son? is not his mother called Mary? and his brethren, James, and Joses,
and Simon, and Judas?

13:56 And his sisters, are they not all with us? Whence then hath this man all these things?

13:57 And they were offended in him. But Jesus said unto them, A prophet is not without honour, save in
his own country, and in his own house.

http://breachrepairers.webs.com/ 245
13:58 And he did not many mighty works there because of their unbelief.

http://breachrepairers.webs.com/ 246
Chapter 14 – Feeding of the Five thousand
Reading
Desire of Ages – Chapter Give Ye Them to Eat.

14:1 At that time Herod the tetrarch heard of the fame of Jesus,

14:2 And said unto his servants, This is John the Baptist; he is risen from the dead; and therefore mighty
works do shew forth themselves in him.

14:3 For Herod had laid hold on John, and bound him, and put him in prison for Herodias' sake, his
brother Philip's wife.

14:4 For John said unto him, It is not lawful for thee to have her.

14:5 And when he would have put him to death, he feared the multitude, because they counted him as a
prophet.

14:6 But when Herod's birthday was kept, the daughter of Herodias danced before them, and pleased
Herod.

14:7 Whereupon he promised with an oath to give her whatsoever she would ask.

14:8 And she, being before instructed of her mother, said, Give me here John Baptist's head in a charger.

14:9 And the king was sorry: nevertheless for the oath's sake, and them which sat with him at meat, he
commanded it to be given her.

14:10 And he sent, and beheaded John in the prison.

14:11 And his head was brought in a charger, and given to the damsel: and she brought it to her mother.

14:12 And his disciples came, and took up the body, and buried it, and went and told Jesus.

14:13 When Jesus heard of it, he departed thence by ship into a desert place apart: and when the people
had heard thereof, they followed him on foot out of the cities.

Where did Jesus retreat with his disciples?

By ship to a desert place to rest a while. This means that Jesus and His disciples were tired from
their labors and wanted some relaxation time.

What happened when the people heard Jesus had gone?

John 6:2

Because they had seen the miracles Jesus had performed and probably others wanted His help.

How did Jesus react when He saw the people? Why? How can we have compassion on others?

Mark 6:34

http://breachrepairers.webs.com/ 247
He had compassion on the people. Why? Because the people looked like sheep without a shepherd.
How can we have compassion on others? Learn to see others as Christ saw them – sheep without a
shepherd. They also decided to follow.

14:14 And Jesus went forth, and saw a great multitude, and was moved with compassion toward them,
and he healed their sick.

How did Jesus show compassion? What can we learn from this?

Jesus showed his compassion through the following things.

Matt 14:14

He healed the sick.

Mark 6:34

He taught them many things.

Luke 9:11

He spake unto them of the kingdom of God, and healed them that had need of healing.

Application: We can show Christian compassion to others by: 1. Teaching and preaching the
kingdom of God. 2. Healing the sick NOTE: these are the 2 most important lines of gospel work that
Christ did without fail wherever He went. These 2 are the most effective methods that we can share
Christ with others.

14:15 And when it was evening, his disciples came to him, saying, This is a desert place, and the time is
now past; send the multitude away, that they may go into the villages, and buy themselves victuals.

What did the disciples ask Jesus to do at the end of the day? Why? Can you relate?

They asked Jesus to let the people go. Why? The disciples were probably tired from the day’s work,
irritated that the people had taken away their private time, hungry themselves. They just did not
want to entertain people. Can you relate? Do you sometimes feel this way? Especially after your day
of work (which isn’t even maybe gospel work?) – too tired to feed the sheep?

14:16 But Jesus said unto them, They need not depart; give ye them to eat.

What was Jesus response to the disciples?

He asked the disciples to feed the crowd. The sequence of the order of the bread and how it reached
the people: It went from: Jesus, to the Disciples, then to the People. Christ will continue to supply as
you are providing the bread to others.

14:17 And they say unto him, We have here but five loaves, and two fishes.

How did the disciples react? What evidence had Christ given them to react otherwise?

John 6:5-9

http://breachrepairers.webs.com/ 248
They doubted their ability to feed everyone. NOTE: their description of the meal – “small fishes”.
They were saying to Jesus, look how little resources we have. How are we going to feed everyone?
Reason to react this way: They did not have enough resources.

Christ’s evidence to react otherwise: Remember, what had Jesus been doing all day? Healing the
sick. Weren’t these miracles? How could the disciples have witnessed such an occurrence all around
them, and yet doubt the resources and ability to fulfill Christ’s command to them.

Application – How does this apply to us? Sometimes we feel that as long as we have helped the sick
or taught Bible studies, this should be enough – like the disciples we say let the people find their
own food, haven’t we helped them enough?
God’s people are sometimes much better at giving spiritual food, but lack the compassion and love
necessary to provide physically for others.

14:18 He said, Bring them hither to me.

14:19 And he commanded the multitude to sit down on the grass, and took the five loaves, and the two
fishes, and looking up to heaven, he blessed, and brake, and gave the loaves to his disciples, and the
disciples to the multitude.

14:20 And they did all eat, and were filled: and they took up of the fragments that remained twelve
baskets full.

What did Jesus do next? What can we learn from the order in which Christ distributed the
food?

Matt 14:18-20

1. Jesus asked the disciples to bring the loaves and fishes to Him.
o Application – if you feel that you have too little resources to help others, bring them to
Christ and He will provide for you and them.
2. Jesus prayed for the blessing on the food.
3. Jesus broke the bread and fishes and gave it to the disciples, and the disciples gave it to the
people.
4. Everyone was filled.

Question – what is the order that the bread is distributed? Christ > Disciples > lost sheep.

Spiritual Application – what spiritual application can we learn from this? Christ distributes His
word through His people, who are then to take it to others. This should be the structure for
outreach and evangelism – the apostles / evangelists / pastors are to train the church members –
especially those that are newly baptized, the more experienced should work with the less
experienced to reach those who have never heard the truth.

What does bread represent? And what are we to do with the bread according to this story?

John 6:35

Jesus = Bread of Life.

http://breachrepairers.webs.com/ 249
Matt 4:4

Bread = Word of God. Therefore, Jesus, the Bread of Life is found in God’s word. What is His
promise? If we go to Him, we will never hunger or thirst. If you have received from Christ, the living
Bread today, what are we to do with Him? Share Him, distribute Him to the crowd so that they may
be filled.

How many baskets were left over? What was Jesus trying to teach the disciples and us?

Matt 14:20

John 10:10

How many baskets left over? 12. How many disciples were there? 12. So there would have been 1
basket for each disciple. Christ wanted to not only show he could provide for each of the five
thousand and the disciples, but that there would be more than enough left over. Application – a life
in Christ is a life in abundance. When we follow God’s command, He will not only take care of our
needs, but will give us more than we need.

Compare and contrast the 3 different groups in this story. Which group might you belong to?
1. The crowd – desiring to be with Jesus, sheep without shepherd.
2. Disciples – helpers of Christ, wanting the people to leave by the evening, tired of helping.
3. Jesus – had compassion on the people, teaching and healing the sick, desiring also to feed
them physically.

Which group do you belong to? Discuss. Encourage sharing testimony.

According to this story, what does it mean when Christ asked Peter (and us) to feed His
sheep in John 21:15-17?

Feed my sheep. What does this include?

1. Sharing God’s word – spiritual food (Luke 9:11)


2. Healing the sick – health message (Luke 9:11)
3. most important from this study – feeding people physical food.

Many times we neglect the work of hospitality.

14:21 And they that had eaten were about five thousand men, beside women and children.

14:22 And straightway Jesus constrained his disciples to get into a ship, and to go before him unto the
other side, while he sent the multitudes away.

14:23 And when he had sent the multitudes away, he went up into a mountain apart to pray: and when
the evening was come, he was there alone.

14:24 But the ship was now in the midst of the sea, tossed with waves: for the wind was contrary.

14:25 And in the fourth watch of the night Jesus went unto them, walking on the sea.

http://breachrepairers.webs.com/ 250
14:26 And when the disciples saw him walking on the sea, they were troubled, saying, It is a spirit; and
they cried out for fear.

14:27 But straightway Jesus spake unto them, saying, Be of good cheer; it is I; be not afraid.

14:28 And Peter answered him and said, Lord, if it be thou, bid me come unto thee on the water.

Why didn’t Peter just step out? Peter knew that if Jesus told Him to come, He would make it
possible. All of Christ’s biddings are sure. He enables us to do that which He asks. Obedience to the
law of God is impossible except only through faith. If we lose our focus on Jesus Christ, we will not
be able to walk by faith.

14:29 And he said, Come. And when Peter was come down out of the ship, he walked on the water, to go
to Jesus.

Jesus and Peter walking on the water

Matt 14:22-33

Why did Peter ask Jesus before walking out on the water? Jesus said “Come”. Peter knew that if
Jesus said so, it will be done. All Jesus’ biddings are enabling. What made walking on the water
possible? The Word of God. Faith does the impossible. If God promises something, then it is
possible. Peter was familiar with water (Peter knew how to swim) and knew that it was not
possible to walk on water. But when Jesus said Come, he knew it was possible by faith. With the
power of God’s Word, we can do it. Carnal heart can’t do it, but God can do it for us. When we take
our eyes of Jesus, we lose our faith. The waves went in between. What gets us away from God?
Seeing the fault of others (murmuring). By seeing the fault of others, we fall easily. When we do, we
started to sink. The prayer that Peter prayed: “Lord, Help me!” God will be more than happy to help
you!

14:30 But when he saw the wind boisterous, he was afraid; and beginning to sink, he cried, saying, Lord,
save me.

14:31 And immediately Jesus stretched forth his hand, and caught him, and said unto him, O thou of little
faith, wherefore didst thou doubt?

14:32 And when they were come into the ship, the wind ceased.

14:33 Then they that were in the ship came and worshipped him, saying, Of a truth thou art the Son of
God.

14:34 And when they were gone over, they came into the land of Gennesaret.

14:35 And when the men of that place had knowledge of him, they sent out into all that country round
about, and brought unto him all that were diseased;

14:36 And besought him that they might only touch the hem of his garment: and as many as touched
were made perfectly whole.

http://breachrepairers.webs.com/ 251
Chapter 15
15:1 Then came to Jesus scribes and Pharisees, which were of Jerusalem, saying,

15:2 Why do thy disciples transgress the tradition of the elders? for they wash not their hands when they
eat bread.

15:3 But he answered and said unto them, Why do ye also transgress the commandment of God by your
tradition?

15:4 For God commanded, saying, Honour thy father and mother: and, He that curseth father or mother,
let him die the death.

15:5 But ye say, Whosoever shall say to his father or his mother, It is a gift, by whatsoever thou mightest
be profited by me;

Jesus fulfilled His role as a son on the cross. He gave His mother to the care of John (the disciple
whom Jesus loved). Our duty to our parents, it doesn’t matter how old you get. Jesus asked John to
take care of his mother, this act is an example for us. It doesn’t matter whether they have made
mistakes or not, we must honor them as much as we can within the laws of God.

15:6 And honour not his father or his mother, he shall be free. Thus have ye made the commandment of
God of none effect by your tradition.

Matt 16:6, 12

The traditions and commandments of men is the doctrines of the Pharisees and Sadducees. It can be
related to false doctrines in the Bible. If you add leaven to wine (fresh grape juice), what happens, it
becomes alcohol. Wine represents doctrines in the Bible. If the whole world drinks the wine from
the whore that makes them drunk (it is fermented). A error that will mix with God’s
commandments, the Sunday law.

15:7 Ye hypocrites, well did Esaias prophesy of you, saying,

15:8 This people draweth nigh unto me with their mouth, and honoureth me with their lips; but their
heart is far from me.

15:9 But in vain they do worship me, teaching for doctrines the commandments of men.

15:10 And he called the multitude, and said unto them, Hear, and understand:

15:11 Not that which goeth into the mouth defileth a man; but that which cometh out of the mouth, this
defileth a man.

15:12 Then came his disciples, and said unto him, Knowest thou that the Pharisees were offended, after
they heard this saying?

15:13 But he answered and said, Every plant, which my heavenly Father hath not planted, shall be
rooted up.

15:14 Let them alone: they be blind leaders of the blind. And if the blind lead the blind, both shall fall into
the ditch.

http://breachrepairers.webs.com/ 252
15:15 Then answered Peter and said unto him, Declare unto us this parable.

15:16 And Jesus said, Are ye also yet without understanding?

15:17 Do not ye yet understand, that whatsoever entereth in at the mouth goeth into the belly, and is
cast out into the draught?

15:18 But those things which proceed out of the mouth come forth from the heart; and they defile the
man.

15:19 For out of the heart proceed evil thoughts, murders, adulteries, fornications, thefts, false witness,
blasphemies:

15:20 These are the things which defile a man: but to eat with unwashen hands defileth not a man.

15:21 Then Jesus went thence, and departed into the coasts of Tyre and Sidon.

15:22 And, behold, a woman of Canaan came out of the same coasts, and cried unto him, saying, Have
mercy on me, O Lord, thou Son of David; my daughter is grievously vexed with a devil.

This woman was not a Jew. And because she was not a Jew, they basically looked down on those
that weren’t Jews. Jesus acted towards her that would teach forever a lesson to the disciples about
prejudice. How did she know about Jesus? She heard about Him. Her faith began to grow when she
heard about Jesus whom healed diseases no one else could cure. People come to Jesus either first or
last choice. Jarius’ daughter they came first, they had faith. Woman with an issue of blood she came
last. After all hope runs out, then they come to Jesus.

15:23 But he answered her not a word. And his disciples came and besought him, saying, Send her away;
for she crieth after us.

15:24 But he answered and said, I am not sent but unto the lost sheep of the house of Israel.

15:25 Then came she and worshipped him, saying, Lord, help me.

If Jesus does not answer our prayers, we must plead even more to Jesus. The woman who goes to
the Judge, she pleads until the judge does something.

15:26 But he answered and said, It is not meet to take the children's bread, and to cast it to dogs.

He didn’t answer her to treat her like how the other Jews treat her. He was acting out what He
never wanted them to do.

DA 401 The woman urged her case with increased earnestness, bowing at Christ's feet, and crying,
"Lord, help me." Jesus, still apparently rejecting her entreaties, according to the unfeeling prejudice
of the Jews, answered, "It is not meet to take the children's bread, and to cast it to dogs." This was
virtually asserting that it was not just to lavish the blessings brought to the favored people of God
upon strangers and aliens from Israel. This answer would have utterly discouraged a less earnest
seeker. But the woman saw that her opportunity had come. Beneath the apparent refusal of Jesus, she
saw a compassion that He could not hide. "Truth, Lord," she answered, "yet the dogs eat of the
crumbs which fall from their masters' table." While the children of the household eat at the father's
table, even the dogs are not left unfed. They have a right to the crumbs that fall from the table
abundantly supplied. So while there were many blessings given to Israel, was there not also a
blessing for her? She was looked upon as a dog, and had she not then a dog's claim to a crumb from
His bounty?

http://breachrepairers.webs.com/ 253
Mark 7:28-29

Because I have believed, therefore have I spoken. This woman believed that Jesus grant her request.

DA 402 This was the only miracle that Jesus wrought while on this journey. It was for the
performance of this act that He went to the borders of Tyre and Sidon. He wished to relieve the
afflicted woman, and at the same time to leave an example in His work of mercy toward one of a
despised people for the benefit of His disciples when He should no longer be with them. He wished to
lead them from their Jewish exclusiveness to be interested in working for others besides their own
people.

DA 403 Caste is hateful to God.

“Caste” class system in India.

These two cities were 8 hours apart. Jesus is willing to travel to such distances just to minister to
one person. If you’re only friends of the same race, something is wrong.

15:27 And she said, Truth, Lord: yet the dogs eat of the crumbs which fall from their masters' table.

This woman would not be turned back by anything.

DA 400 But although Jesus did not reply, the woman did not lose faith. As He passed on, as if not
hearing her, she followed Him, continuing her supplications. Annoyed by her importunities, the
disciples asked Jesus to send her away. They saw that their Master treated her with indifference, and
they therefore supposed that the prejudice of the Jews against the Canaanites was pleasing to Him.
But it was a pitying Saviour to whom the woman made her plea, and in answer to the request of the
disciples, Jesus said, "I am not sent but unto the lost sheep of the house of Israel.

Other sheep have I that are not of this fold. This statement does not exclude the woman that was
pleading.

15:28 Then Jesus answered and said unto her, O woman, great is thy faith: be it unto thee even as thou
wilt. And her daughter was made whole from that very hour.

15:29 And Jesus departed from thence, and came nigh unto the sea of Galilee; and went up into a
mountain, and sat down there.

15:30 And great multitudes came unto him, having with them those that were lame, blind, dumb,
maimed, and many others, and cast them down at Jesus' feet; and he healed them:

15:31 Insomuch that the multitude wondered, when they saw the dumb to speak, the maimed to be
whole, the lame to walk, and the blind to see: and they glorified the God of Israel.

15:32 Then Jesus called his disciples unto him, and said, I have compassion on the multitude, because
they continue with me now three days, and have nothing to eat: and I will not send them away fasting,
lest they faint in the way.

15:33 And his disciples say unto him, Whence should we have so much bread in the wilderness, as to fill
so great a multitude?

15:34 And Jesus saith unto them, How many loaves have ye? And they said, Seven, and a few little fishes.

http://breachrepairers.webs.com/ 254
15:35 And he commanded the multitude to sit down on the ground.

15:36 And he took the seven loaves and the fishes, and gave thanks, and brake them, and gave to his
disciples, and the disciples to the multitude.

15:37 And they did all eat, and were filled: and they took up of the broken meat that was left seven
baskets full.

15:38 And they that did eat were four thousand men, beside women and children.

15:39 And he sent away the multitude, and took ship, and came into the coasts of Magdala.

http://breachrepairers.webs.com/ 255
Chapter 16
16:1 The Pharisees also with the Sadducees came, and tempting desired him that he would shew them a
sign from heaven.

16:2 He answered and said unto them, When it is evening, ye say, It will be fair weather: for the sky is
red.

16:3 And in the morning, It will be foul weather to day: for the sky is red and lowring. O ye hypocrites, ye
can discern the face of the sky; but can ye not discern the signs of the times?

16:4 A wicked and adulterous generation seeketh after a sign; and there shall no sign be given unto it,
but the sign of the prophet Jonas. And he left them, and departed.

Is this principle true today about the weather? Yes.

“Good sunset” – good day tomorrow; red & lowering sunrise = bad weather. Is the sign of Jonas
talked anywhere else in the New Testament?

Matt 12:39-40

When did the 3 days and 3 nights begin for Jesus? Was Jesus in the earth 3 nights?

“in earth” is not always referring to the grave.

Matt 6:10

Thursday night was the night that Jesus decided He would bear the sins of man. It was from that
point on that He would be in the heart of the earth.

16:5 And when his disciples were come to the other side, they had forgotten to take bread.

16:6 Then Jesus said unto them, Take heed and beware of the leaven of the Pharisees and of the
Sadducees.

16:7 And they reasoned among themselves, saying, It is because we have taken no bread.

16:8 Which when Jesus perceived, he said unto them, O ye of little faith, why reason ye among yourselves,
because ye have brought no bread?

16:9 Do ye not yet understand, neither remember the five loaves of the five thousand, and how many
baskets ye took up?

16:10 Neither the seven loaves of the four thousand, and how many baskets ye took up?

16:11 How is it that ye do not understand that I spake it not to you concerning bread, that ye should
beware of the leaven of the Pharisees and of the Sadducees?

16:12 Then understood they how that he bade them not beware of the leaven of bread, but of the
doctrine of the Pharisees and of the Sadducees.

16:13 When Jesus came into the coasts of Caesarea Philippi, he asked his disciples, saying, Whom do men
say that I the Son of man am?

http://breachrepairers.webs.com/ 256
16:14 And they said, Some say that thou art John the Baptist: some, Elias; and others, Jeremias, or one of
the prophets.

16:15 He saith unto them, But whom say ye that I am?

16:16 And Simon Peter answered and said, Thou art the Christ, the Son of the living God.

16:17 And Jesus answered and said unto him, Blessed art thou, Simon Barjona: for flesh and blood hath
not revealed it unto thee, but my Father which is in heaven.

16:18 And I say also unto thee, That thou art Peter, and upon this rock I will build my church; and the
gates of hell shall not prevail against it.

If a Catholic said that Peter is the first Pope, what would you say? What is the strongest evidence
that the rock was not Peter?

“Peter” – Petros means small pebble or rolling stone.

“Rock” – Petra means a stone.

It is always only ever used in regard to Jesus.

1 Cor 10:4

Rock is using the Greek word Petra. So Jesus is going to be the foundation of the church.

Matt 16:23

Sometimes we tell people that they don’t have to do it, it’s extreme. If we lighten the obligation of
Jesus upon a person, that is Satan’s job.

16:19 And I will give unto thee the keys of the kingdom of heaven: and whatsoever thou shalt bind on
earth shall be bound in heaven: and whatsoever thou shalt loose on earth shall be loosed in heaven.

John 20:21, 23

The disciples that were sent by Jesus would also be sent the same way as the Father sent Jesus onto
the earth. Jesus never spoke of His own words but except that the Father gives it to Him.

2 Cor 5:18-21

God has given to us the ministry of reconciling people. We are dealing with the forgiveness of sins.
Jesus’ reconciliation was not imputing their trespasses unto them.

16:20 Then charged he his disciples that they should tell no man that he was Jesus the Christ.

16:21 From that time forth began Jesus to shew unto his disciples, how that he must go unto Jerusalem,
and suffer many things of the elders and chief priests and scribes, and be killed, and be raised again the
third day.

16:22 Then Peter took him, and began to rebuke him, saying, Be it far from thee, Lord: this shall not be
unto thee.

http://breachrepairers.webs.com/ 257
16:23 But he turned, and said unto Peter, Get thee behind me, Satan: thou art an offence unto me: for
thou savourest not the things that be of God, but those that be of men.

16:24 Then said Jesus unto his disciples, If any man will come after me, let him deny himself, and take up
his cross, and follow me.

16:25 For whosoever will save his life shall lose it: and whosoever will lose his life for my sake shall find
it.

16:26 For what is a man profited, if he shall gain the whole world, and lose his own soul? or what shall a
man give in exchange for his soul?

16:27 For the Son of man shall come in the glory of his Father with his angels; and then he shall reward
every man according to his works.

16:28 Verily I say unto you, There be some standing here, which shall not taste of death, till they see the
Son of man coming in his kingdom.

Did Jesus have an earthy human nature? Yes It is not natural for any person to want to suffer and to
choose rejection, pain, etc. We must deny ourselves. The natural heart desires to have the easy way.
The old man, unconverted heart. What evidence do we have that Jesus denied Himself?
Gethsemane. Not mine will but thine will be done. It’s one thing to do something for someone, but
even harder when they do not appreciate it. What does it mean to take up the cross? First is to deny
your natural inclination. To give up your desire for evil comfort. One thing we don’t talk about much
is the cross. Because it is unpopular. Taking up your cross—simply endure suffering etc. God tests
His people; the last 2 tests are money & power. Vs 26, if we have the whole world, and lost eternal
life, it is not worthy at all! Can have wealth? Yes, like Abraham and Job. But God will test us.

None of the disciples are alive today. So how do we comprehend this statement? Matt 17:1-4

http://breachrepairers.webs.com/ 258
Chapter 17
Two extreme in ministry: High mountain, never minister to other. Might have great devotion, but
never share with others. Being with other, but not really minister to them. We might doing a lot of
things, but not really have personal time with Jesus.

17:1 And after six days Jesus taketh Peter, James, and John his brother, and bringeth them up into an
high mountain apart,

“After six days” – so He was transfigured on the seventh day. Moses represents those who were
faithful to Jesus but died and were resurrected to Heaven. He can sympathize with Jesus because he
had the same experience. Elijah represents those who didn’t see death and were taken to Heaven.
He specifically represents the 144,000. Jezebel persecuted Elijah, he went through the difficult
times as well. He fell so great, and he asked for forgiveness, and he was taken to heaven after
he anointed other 3 workers of God. We have nothing to fear for the future, unless we forget the
blessings that God had given us. Peter, James and John were the ones that saw Jesus transfigured.
After 6 years the lands should rest and slaves could go free. No matter how you do for the time
sequence, we are really close now. Time setting is always wrong. After 6, 7 is rest. We need to be
ready at all time.

17:2 And was transfigured before them: and his face did shine as the sun, and his raiment was white as
the light.

17:3 And, behold, there appeared unto them Moses and Elias talking with him.

17:4 Then answered Peter, and said unto Jesus, Lord, it is good for us to be here: if thou wilt, let us make
here three tabernacles; one for thee, and one for Moses, and one for Elias.

17:5 While he yet spake, behold, a bright cloud overshadowed them: and behold a voice out of the cloud,
which said, This is my beloved Son, in whom I am well pleased; hear ye him.

17:6 And when the disciples heard it, they fell on their face, and were sore afraid.

17:7 And Jesus came and touched them, and said, Arise, and be not afraid.

17:8 And when they had lifted up their eyes, they saw no man, save Jesus only.

17:9 And as they came down from the mountain, Jesus charged them, saying, Tell the vision to no man,
until the Son of man be risen again from the dead.

17:10 And his disciples asked him, saying, Why then say the scribes that Elias must first come?

17:11 And Jesus answered and said unto them, Elias truly shall first come, and restore all things.

Is that a reference to anything that is written before?

Mal 4:5-6

Luke 1:16-17

When we talk about the Elijah message, it is not about the restoration of the home family. It is
specifically concerning the children of God back to the Heavenly Father. John the Baptist achieved

http://breachrepairers.webs.com/ 259
this by causing a great revival in Israel

17:12 But I say unto you, That Elias is come already, and they knew him not, but have done unto him
whatsoever they listed. Likewise shall also the Son of man suffer of them.

Malachi was referring to the great and dreadful day. But Jesus made a secondary application
referring to John the Baptist. That is not the primary application though because Elijah will come
before the great and dreadful day of the Lord.

17:13 Then the disciples understood that he spake unto them of John the Baptist.

17:14 And when they were come to the multitude, there came to him a certain man, kneeling down to
him, and saying,

17:15 Lord, have mercy on my son: for he is lunatick, and sore vexed: for ofttimes he falleth into the fire,
and oft into the water.

Satan’s purpose has always been to destroy the image of God in man. Piercing etc..

Mark 9:19-29

Mark 9:21

Why did Jesus ask the question? To make it more visibly the power of God.

Mark 9:22

The father doubted – “if you canst do anything”

Mark 9:23

The only thing that can limit the power of God.

Matt 15:38

Our lack of faith is what prevents the power of God from manifesting in our lives.

Matt 17:20 If ye have faith as a grain of mustard seed.

DA 431 If ye have faith as a grain of mustard seed," said Jesus, "ye shall say unto this mountain,
Remove hence to yonder place; and it shall remove." Though the grain of mustard seed is so small, it
contains that same mysterious life principle which produces growth in the loftiest tree. When the
mustard seed is cast into the ground, the tiny germ lays hold of every element that God has provided
for its nutriment, and it speedily develops a sturdy growth. If you have faith like this, you will lay hold
upon God's word, and upon all the helpful agencies He has appointed. Thus your faith will strengthen,
and will bring to your aid the power of heaven. The obstacles that are piled by Satan across your
path, though apparently as insurmountable as the eternal hills, shall disappear before the demand of
faith. "Nothing shall be impossible unto you.

Luke 17:6

There are 2 obstacles:

http://breachrepairers.webs.com/ 260
“Mountain” – it is big. It is visible to all.

“Tree” – a tree has roots, it is a deeper seated problem. We need a faith like mustard seed to
get rid of all the bad habits in our lives.

17:16 And I brought him to thy disciples, and they could not cure him.

17:17 Then Jesus answered and said, O faithless and perverse generation, how long shall I be with you?
how long shall I suffer you? bring him hither to me.

17:18 And Jesus rebuked the devil; and he departed out of him: and the child was cured from that very
hour.

17:19 Then came the disciples to Jesus apart, and said, Why could not we cast him out?

17:20 And Jesus said unto them, Because of your unbelief: for verily I say unto you, If ye have faith as a
grain of mustard seed, ye shall say unto this mountain, Remove hence to yonder place; and it shall
remove; and nothing shall be impossible unto you.

17:21 Howbeit this kind goeth not out but by prayer and fasting.

They had neglected to pray while Jesus was up in the mountain. They had started but they fell
asleep.

17:22 And while they abode in Galilee, Jesus said unto them, The Son of man shall be betrayed into the
hands of men:

17:23 And they shall kill him, and the third day he shall be raised again. And they were exceeding sorry.

17:24 And when they were come to Capernaum, they that received tribute money came to Peter, and
said, Doth not your master pay tribute?

If Jesus didn’t pay it would show disloyalty. If Jesus paid it, it would show that He was not a rabbi, or
a religious leader as it was free for them.

17:25 He saith, Yes. And when he was come into the house, Jesus prevented him, saying, What thinkest
thou, Simon? of whom do the kings of the earth take custom or tribute? of their own children, or of
strangers?

17:26 Peter saith unto him, Of strangers. Jesus saith unto him, Then are the children free.

17:27 Notwithstanding, lest we should offend them, go thou to the sea, and cast an hook, and take up the
fish that first cometh up; and when thou hast opened his mouth, thou shalt find a piece of money: that
take, and give unto them for me and thee.

Peter was a fisherman so that was not unusual. Jesus was going to pay the tribute but it was not
really by Him. The way that Jesus got His money was proof for His divinity. Jesus paid the tribute
but also showed evidence of His divinity as well.

http://breachrepairers.webs.com/ 261
Chapter 18
18:1 At the same time came the disciples unto Jesus, saying, Who is the greatest in the kingdom of
heaven?

Who is the greatest? Judas is the one who rouse up the discussion “who is the greatest?” among the
disciples.
The humble souls that trusted in God will succeed. Why did Jesus need 12 disciples? 12 personality
types. For every true, there are counterfeit. Jerusalem has 12 gates. Biblical basis of personality
type—12. Jealousy is the cruelest, is bondage. He needs everyone of us, we reflect God in different
ways. Each one of us is essential to God. Some are using they talents, some are hiding in the earth.

Real greatness

DA 437 Except ye be converted, and become as little children, ye shall not enter into the kingdom of
heaven." The simplicity, the self-forgetfulness, and the confiding love of a little child are the attributes
that Heaven values. These are the characteristics of real greatness.

18:2 And Jesus called a little child unto him, and set him in the midst of them,

18:3 And said, Verily I say unto you, Except ye be converted, and become as little children, ye shall not
enter into the kingdom of heaven.

Who is the greatest? In characteristics of a child is what Jesus considers great: simplicity. self-
forgetfulness, and confiding love.

18:4 Whosoever therefore shall humble himself as this little child, the same is greatest in the kingdom of
heaven.

Being born again and converted has to do with a characteristics of a child: Humble themselves as a
little child. When working with children, we must always be careful in how we portray the
character of Jesus Christ to them.

18:5 And whoso shall receive one such little child in my name receiveth me.

18:6 But whoso shall offend one of these little ones which believe in me, it were better for him that a
millstone were hanged about his neck, and that he were drowned in the depth of the sea.

Child Training
DA 515 As the mother teaches her children to obey her because they love her, she is teaching them
the first lessons in the Christian life.

We are to teach them obedience because of love.

DA 515 We should teach them to bring their sins to Jesus, asking His forgiveness, and believing that
He pardons and receives them as He received the children when He was personally on earth.

DA 515 He did not use one unkind or discourteous expression.

DA 516 Parents, in the training of your children, study the lessons that God has given in nature. If
you would train a pink, or rose, or lily, how would you do it? Ask the gardener by what process he

http://breachrepairers.webs.com/ 262
makes every branch and leaf to flourish so beautifully, and to develop in symmetry and loveliness. He
will tell you that it was by no rude touch, no violent effort; for this would only break the delicate
stems. It was by little attentions, often repeated. He moistened the soil, and protected the growing
plants from the fierce blasts and from the scorching sun, and God caused them to flourish and to
blossom into loveliness. In dealing with your children, follow the method of the gardener. By gentle
touches, by loving ministrations, seek to fashion their characters after the pattern of the character of
Christ.
Working with young people, we need repetition.

DA 516 Encourage the expression of love toward God and toward one another. The reason why there
are so many hardhearted men and women in the world is that true affection has been regarded as
weakness, and has been discouraged and repressed.

Showing appreciation to someone is not wrong.

DA 516 Do not weary them with long prayers and tedious exhortations, but through nature's object
lessons teach them obedience to the law of God.

DA 517 As you win their confidence in you as followers of Christ, it will be easy to teach them of the
great love wherewith He has loved us.

Approach them as one who desires their good, then you win their confidence and then big them
follow you.

18:7 Woe unto the world because of offences! for it must needs be that offences come; but woe to that
man by whom the offence cometh!

Did Jesus need to die? Yes. But does that mean Judas had to betray Him? No

18:8 Wherefore if thy hand or thy foot offend thee, cut them off, and cast them from thee: it is better for
thee to enter into life halt or maimed, rather than having two hands or two feet to be cast into
everlasting fire.

18:9 And if thine eye offend thee, pluck it out, and cast it from thee: it is better for thee to enter into life
with one eye, rather than having two eyes to be cast into hell fire.

Jesus was not speaking of literal dismemberment. How do we know that it is figurative? We can still
sin with one leg or arm or eye.

DA 439 Any habit or practice that would lead into sin, and bring dishonor upon Christ, would
better be put away, whatever the sacrifice. That which dishonors God cannot benefit the soul. The
blessing of heaven cannot attend any man in violating the eternal principles of right. And one sin
cherished is sufficient to work the degradation of the character, and to mislead others. If the foot or
the hand would be cut off, or even the eye would be plucked out, to save the body from death,
how much more earnest should we be to put away sin, that brings death to the soul!

Anything that you are holding on to. E.g.: relationship. You never know what God has prepared for
you.

18:10 Take heed that ye despise not one of these little ones; for I say unto you, That in heaven their
angels do always behold the face of my Father which is in heaven.

This proves to us that everyone is assigned an angel.

http://breachrepairers.webs.com/ 263
18:11 For the Son of man is come to save that which was lost.

18:12 How think ye? if a man have an hundred sheep, and one of them be gone astray, doth he not leave
the ninety and nine, and goeth into the mountains, and seeketh that which is gone astray?

This sheep could represent this earth and the ninety nine the other unfallen worlds.

18:13 And if so be that he find it, verily I say unto you, he rejoiceth more of that sheep, than of the ninety
and nine which went not astray.

18:14 Even so it is not the will of your Father which is in heaven, that one of these little ones should
perish.

18:15 Moreover if thy brother shall trespass against thee, go and tell him his fault between thee and him
alone: if he shall hear thee, thou hast gained thy brother.

Tell him in private. You must pick someone that is not partial to a particular person. Pick someone
who is mutual, has clear discernment judgment. You treat them like heathen, gentiles, as you want
to win them back.

18:16 But if he will not hear thee, then take with thee one or two more, that in the mouth of two or three
witnesses every word may be established.

18:17 And if he shall neglect to hear them, tell it unto the church: but if he neglect to hear the church, let
him be unto thee as an heathen man and a publican.

18:18 Verily I say unto you, Whatsoever ye shall bind on earth shall be bound in heaven: and whatsoever
ye shall loose on earth shall be loosed in heaven.

18:19 Again I say unto you, That if two of you shall agree on earth as touching any thing that they shall
ask, it shall be done for them of my Father which is in heaven.

18:20 For where two or three are gathered together in my name, there am I in the midst of them.

18:21 Then came Peter to him, and said, Lord, how oft shall my brother sin against me, and I forgive
him? till seven times?

18:22 Jesus saith unto him, I say not unto thee, Until seven times: but, Until seventy times seven.

Does Jesus mean that you just forgive them 490 times? No, it just meant to never stop forgiving.
Jesus also gave the Jews a time of probation. After 490 years, God’s forbearance to them would
close. But it doesn’t mean that you only forgive 490 times. There is a limit to God’s forbearance.

18:23 Therefore is the kingdom of heaven likened unto a certain king, which would take account of his
servants.

What was the reason why Jesus gave this parable? They thought that there was a limit to
forgiving somebody. They didn’t realize that forgiveness was more than just a one off event

Who does the king represent?

Matt 18:35

http://breachrepairers.webs.com/ 264
King = Heavenly Father

Psa 78:38

God is full of compassion, forgives, and does destroy us

Psa 86:5

God longs to forgive all our debt. He is merciful. God forgives us completely 100% of what
we confess

18:24 And when he had begun to reckon, one was brought unto him, which owed him ten thousand
talents.

18:25 But forasmuch as he had not to pay, his lord commanded him to be sold, and his wife, and children,
and all that he had, and payment to be made.

18:26 The servant therefore fell down, and worshipped him, saying, Lord, have patience with me, and I
will pay thee all.

18:27 Then the lord of that servant was moved with compassion, and loosed him, and forgave him the
debt.

18:28 But the same servant went out, and found one of his fellowservants, which owed him an hundred
pence: and he laid hands on him, and took him by the throat, saying, Pay me that thou owest.

How much did the first servant owe as compared to the fellow servant? What was the difference in
today’s value?

Note: the ratio of 10,000 talents to 100 pence = 1 million to 1 (1 pence = 1 day’s wage). 1 talent =
10,000 pence. Therefore 10,000 talents = 100 million pence. Today’s wage = $100 per day.
Therefore 10,000 talents = $10 billion (first servant). 100 pence = $10,000 (fellow servant)

What was the attitude of the first servant at the beginning with the king?

Matt 18:26

Humble, Meek, Begging.

18:29 And his fellowservant fell down at his feet, and besought him, saying, Have patience with me, and I
will pay thee all.

18:30 And he would not: but went and cast him into prison, till he should pay the debt.

What was the attitude of the first servant with his fellow servant?

Matt 18:28-30

Unforgiving, Unmerciful, Violent.

What was the plea of the first servant to the king and the plea of the fellow servant to the
first servant? They were exactly the same. When the fellow servant pleaded to the first servant, the
first servant should have been reminded of his own words that he spoke to the king and how the

http://breachrepairers.webs.com/ 265
king responded

How should the first servant have reacted to his fellow servant and why should he have
reacted that way? He should have had mercy on his fellow servant because the king forgave him

What is the important lesson that we must learn as we look at the first servant and how he
reacted? We must remember the mercy of God upon us and how much we were forgiven

Where is the root of the problem when we see someone who has an unforgiving spirit? They
either have never experienced true forgiveness from God, or they have forgotten that God has
forgiven them.

Where must forgiveness come from?

Matt 18:35

From the heart

What does it mean to forgive from the heart? Can we forgive someone NOT from the heart?
Discuss. Forgiveness is also emotional, not just a principle. There is emotional healing as well.
When people hurt us, it leaves an emotional scar. And that scar can only be healed if we learn to
forgive them. Forgiving not from the heart is surface forgiveness. We remember what they did to us
and still hold it against them and as a result we act differently towards them.

What are the consequences of cherishing an unforgiving spirit?

Matt 6:14-15

Jer 31:34

We will not be forgiven.

18:31 So when his fellowservants saw what was done, they were very sorry, and came and told unto
their lord all that was done.

18:32 Then his lord, after that he had called him, said unto him, O thou wicked servant, I forgave thee all
that debt, because thou desiredst me:

18:33 Shouldest not thou also have had compassion on thy fellowservant, even as I had pity on thee?

18:34 And his lord was wroth, and delivered him to the tormentors, till he should pay all that was due
unto him.

18:35 So likewise shall my heavenly Father do also unto you, if ye from your hearts forgive not every one
his brother their trespasses.

http://breachrepairers.webs.com/ 266
Chapter 19
19:1 And it came to pass, that when Jesus had finished these sayings, he departed from Galilee, and came
into the coasts of Judaea beyond Jordan;

19:2 And great multitudes followed him; and he healed them there.

19:3 The Pharisees also came unto him, tempting him, and saying unto him, Is it lawful for a man to put
away his wife for every cause?

19:4 And he answered and said unto them, Have ye not read, that he which made them at the beginning
made them male and female,

19:5 And said, For this cause shall a man leave father and mother, and shall cleave to his wife: and they
twain shall be one flesh?

19:6 Wherefore they are no more twain, but one flesh. What therefore God hath joined together, let not
man put asunder.

19:7 They say unto him, Why did Moses then command to give a writing of divorcement, and to put her
away?

19:8 He saith unto them, Moses because of the hardness of your hearts suffered you to put away your
wives: but from the beginning it was not so.

19:9 And I say unto you, Whosoever shall put away his wife, except it be for fornication, and shall marry
another, committeth adultery: and whoso marrieth her which is put away doth commit adultery.

19:10 His disciples say unto him, If the case of the man be so with his wife, it is not good to marry.

19:11 But he said unto them, All men cannot receive this saying, save they to whom it is given.

19:12 For there are some eunuchs, which were so born from their mother's womb: and there are some
eunuchs, which were made eunuchs of men: and there be eunuchs, which have made themselves eunuchs
for the kingdom of heaven's sake. He that is able to receive it, let him receive it.

19:13 Then were there brought unto him little children, that he should put his hands on them, and pray:
and the disciples rebuked them.

19:14 But Jesus said, Suffer little children, and forbid them not, to come unto me: for of such is the
kingdom of heaven.

19:15 And he laid his hands on them, and departed thence.

19:16 And, behold, one came and said unto him, Good Master, what good thing shall I do, that I may have
eternal life?

“What shall I do to inherit eternal life?” – There is an idea of works in the statement.

Background of the rich young ruler:

DA 520 He was a member of the honored council of the Jews, and Satan was tempting him with
flattering prospects of the future.

http://breachrepairers.webs.com/ 267
He was part of the Sanhedrin council. If he was young and already part of the council, he must have
been very bright. This young man, he pretty much had it all. However, in every heart, no matter how
great, if Jesus does not reside there, there is always a feeling of emptiness. We know this because he
approached Jesus.

19:17 And he said unto him, Why callest thou me good? there is none good but one, that is, God: but if
thou wilt enter into life, keep the commandments.
He didn’t acknowledge Him to be the Saviour. Jesus replied to test him. To see how this ruler
viewed Him. How did Jesus teach people to the way to eternal life? Obedience to the 10
commandments.

19:18 He saith unto him, Which? Jesus said, Thou shalt do no murder, Thou shalt not commit adultery,
Thou shalt not steal, Thou shalt not bear false witness,

What was the major problem the with rich young ruler? The love to God in the soul. The
commandments hang on two things: Love to God. Love to man. Jesus left out the first four
commandments. It would have made him question.

The order of the commandments that Jesus mentioned them in.

“Thou shalt do no murder” – commandment 6.

“Thou shalt not commit adultery” – commandment 7. Etc…

He left out commandment number 10. He also left out the first four as well.

19:19 Honour thy father and thy mother: and, Thou shalt love thy neighbour as thyself.

19:20 The young man saith unto him, All these things have I kept from my youth up: what lack I yet?

He was sincere in his reply:

DA 519 Christ was drawn to this young man. He knew him to be sincere in his assertion, "All these
things have I kept from my youth." The Redeemer longed to create in him that discernment which
would enable him to see the necessity of heart devotion and Christian goodness. He longed to see in
him a humble and contrite heart, conscious of the supreme love to be given to God, and hiding its lack
in the perfection of Christ.

We could be sincerely deceived to the state of our righteousness.

19:21 Jesus said unto him, If thou wilt be perfect, go and sell that thou hast, and give to the poor, and
thou shalt have treasure in heaven: and come and follow me.

Character perfection is possible. If he had sold all that he had, he could have been perfect. It was
possible to achieve. Here was the man’s dilemma: He was an honored member of the Sanhedrin
council. He was rich. He had ambitious projects. He had to give up everything! But Jesus promised
that he would have treasure in Heaven. There are many that are looking for their treasure on earth.
He was sorrowful because he wanted both. The conditions are exclusive, you have only have one or
the other.

19:22 But when the young man heard that saying, he went away sorrowful: for he had great possessions.

http://breachrepairers.webs.com/ 268
19:23 Then said Jesus unto his disciples, Verily I say unto you, That a rich man shall hardly enter into the
kingdom of heaven.

19:24 And again I say unto you, It is easier for a camel to go through the eye of a needle, than for a rich
man to enter into the kingdom of God.

“eye of a needle” – It was a door of one of the cities in Jerusalem. It only allowed the camel to get
through without anything on its back. Money is not the root of all evil, it is the love of money. We
can only cherish one idol before we lose eternal life.

19:25 When his disciples heard it, they were exceedingly amazed, saying, Who then can be saved?

19:26 But Jesus beheld them, and said unto them, With men this is impossible; but with God all things
are possible.

19:27 Then answered Peter and said unto him, Behold, we have forsaken all, and followed thee; what
shall we have therefore?

19:28 And Jesus said unto them, Verily I say unto you, That ye which have followed me, in the
regeneration when the Son of man shall sit in the throne of his glory, ye also shall sit upon twelve
thrones, judging the twelve tribes of Israel.

The heavenly reward is not comparable to the reward on earth. When we give up something, it will
mean nothing when given up for the cause of Christ and to follow him. We count it dung.

19:29 And every one that hath forsaken houses, or brethren, or sisters, or father, or mother, or wife, or
children, or lands, for my name's sake, shall receive an hundredfold, and shall inherit everlasting life.

19:30 But many that are first shall be last; and the last shall be first.

http://breachrepairers.webs.com/ 269
Chapter 20
 Blind Bartimaeus (Son of David) (29-34)

20:1 For the kingdom of heaven is like unto a man that is an householder, which went out early in the
morning to hire labourers into his vineyard.

Who did Jesus encounter before the telling of this parable?

Matt 19:16

Luke 18:18

Mark 10:17

Jesus encounter with the rich ruler who wanted to follow Him.

What was the man’s problem?

Mark 19:21-22

He was covetous. He could not give up everything to follow Jesus. See also Mark 19:18-19 –
Jesus purposely does not mention the 10th commandment, which deals with covetousness.

What is Peter implying from his question to Jesus? What kind of spirit is he showing?

Matt 19:27

Peter is asking what is in it for him and the other disciples since they have forsaken all to follow
Jesus. Peter is showing a pharisaical spirit. Its all about works and reward. But Peter's question,
"What shall we have therefore?" had revealed a spirit that uncorrected would unfit the disciples to
be messengers for Christ; for it was the spirit of a hireling. While they had been attracted by the
love of Jesus, the disciples were not wholly free from Pharisaism. They still worked with the
thought of meriting a reward in proportion to their labor. They cherished a spirit of self-exaltation
and self-complacency, and made comparisons among themselves. When one of them failed in any
particular, the others indulged feelings of superiority. {COL 396.1}

What issue do you think Jesus tried to address with the parable that followed? Peter’s spirit.

COL 396 Lest the disciples should lose sight of the principles of the gospel, Christ related to them a
parable illustrating the manner in which God deals with His servants, and the spirit in which He
desires them to labor for Him.

From previous parables, who is the householder, the laborers and the vineyard?

Householder = God. (Isa 5:7)

Vineyard = Israel, or spiritual Israel. (Isa 5:7)

http://breachrepairers.webs.com/ 270
The laborers = fellow laborer in gospel of Christ. (1 Thess 3:2)

What does the householder do throughout the day? What are the differences between the
laborers and what do they all have in common? He hires laborers throughout the day.
Difference: The laborers work for different lengths of time. Common: The laborers are all promised
one pence.

What did the laborers receive at the end of the day? The householder gave everyone one pence.

What was the attitude of the first laborers when they came to receive their wages? What was
their reaction when they received the same amount?

Matt 20:10-12

The first supposed they should receive more because they had been working longer. They
murmured against the householder.

What was the response of the householder?

Matt 20:13-15

He was fair in what they had agreed upon. It is not for the first laborers to judge if out of the
goodness of His heart He decides to reward those who joined at the end with the same reward since
all the wages are His to give.

What does the householder mean when He says “Is thine eye evil, because I am good”? They
looked upon the goodness of the householder / God because He saw fit to give everyone the same
reward, and accused God of being unfair (or evil). They were seeing this through their evil eyes.

Who do the first laborers represent today? They represent those who feel that because they
worked in the vineyard longer should be entitled to a bigger reward or wage. This is why the
Pharisees went out of their way to show the public their good deeds. They wanted recognition for
their perceive piety. What could the wage represent today? Recognition from others. Bigger roles in
ministry. Bigger mansions in heaven.

(You can read this passage if all Adventists)

COL 399-400 The first laborers of the parable represent those who, because of their services, claim
preference above others. They take up their work in a self-gratulatory spirit, and do not bring into it
self-denial and sacrifice. They may have professed to serve God all their lives; they may have been
foremost in enduring hardship, privation, and trial, and they therefore think themselves entitled to a
large reward. They think more of the reward than of the privilege of being servants of Christ. In their
view their labors and sacrifices entitle them to receive honor above others, and because this claim is
not recognized, they are offended. Did they bring into their work a loving, trusting spirit, they would
continue to be first; but their querulous, complaining disposition is un-Christlike, and proves them to
be untrustworthy. It reveals their desire for self-advancement, their distrust of God, and their jealous,
grudging spirit toward their brethren. The Lord's goodness and liberality is to them only an occasion
of murmuring. Thus they show that there is no connection between their souls and God. They do not

http://breachrepairers.webs.com/ 271
know the joy of co-operation with the Master Worker...There is nothing more offensive to God than
this narrow, self-caring spirit. He cannot work with any who manifest these attributes. They are
insensible to the working of His Spirit.

What spirit should we have instead?

1 Cor 6:20

Our life is not ours. It is bought with a price. The penny represents the blood of Jesus that paid for
our lives. Therefore whatever we do is in gratitude, not for greater reward.

Jer 9:23-24

Boasting in ourselves has no merit.

Eph 2:8-9

The penny also represents the grace of God by which we are saved, not our works lest any man
should boast.

Rom 4:1-5

We work because of our debt to grace, not for reward.

COL 402 The first and the last are to be sharers in the great, eternal reward, and the first should
gladly welcome the last. He who grudges the reward to another forgets that he himself is saved by
grace alone. The parable of the laborers rebukes all jealousy and suspicion. Love rejoices in the truth
and institutes no envious comparisons. He who possesses love compares only the loveliness of Christ
and his own imperfect character. This parable is a warning to all laborers, however long their service,
however abundant their labors, that without love to their brethren, without humility before God,
they are nothing. There is no religion in the enthronement of self. He who makes self-glorification his
aim will find himself destitute of that grace which alone can make him efficient in Christ's service.
Whenever pride and self-complacency are indulged, the work is marred.

What did Jesus mean when He said the first shall be last and the last shall be first? How does
that apply to us today? The first = those that have worked the longest for God. Why were they
last? They had the tendency to want more reward for their works, working their way to heaven. The
last = were the 11th hour workers. They were happy with whatever they got. How does this apply to
us today? Its those that have been working for God the longest and that can count their good works
for God, that have the most tendency to try to work their way to heaven – Pharisaism.

20:2 And when he had agreed with the labourers for a penny a day, he sent them into his vineyard.

20:3 And he went out about the third hour, and saw others standing idle in the marketplace,

20:4 And said unto them; Go ye also into the vineyard, and whatsoever is right I will give you. And they
went their way.

20:5 Again he went out about the sixth and ninth hour, and did likewise.

20:6 And about the eleventh hour he went out, and found others standing idle, and saith unto them, Why

http://breachrepairers.webs.com/ 272
stand ye here all the day idle?

20:7 They say unto him, Because no man hath hired us. He saith unto them, Go ye also into the vineyard;
and whatsoever is right, that shall ye receive.

20:8 So when even was come, the lord of the vineyard saith unto his steward, Call the labourers, and give
them their hire, beginning from the last unto the first.

20:9 And when they came that were hired about the eleventh hour, they received every man a penny.

20:10 But when the first came, they supposed that they should have received more; and they likewise
received every man a penny.

20:11 And when they had received it, they murmured against the goodman of the house,

20:12 Saying, These last have wrought but one hour, and thou hast made them equal unto us, which
have borne the burden and heat of the day.

20:13 But he answered one of them, and said, Friend, I do thee no wrong: didst not thou agree with me
for a penny?

20:14 Take that thine is, and go thy way: I will give unto this last, even as unto thee.

20:15 Is it not lawful for me to do what I will with mine own? Is thine eye evil, because I am good?

20:16 So the last shall be first, and the first last: for many be called, but few chosen.

20:17 And Jesus going up to Jerusalem took the twelve disciples apart in the way, and said unto them,

20:18 Behold, we go up to Jerusalem; and the Son of man shall be betrayed unto the chief priests and
unto the scribes, and they shall condemn him to death,

20:19 And shall deliver him to the Gentiles to mock, and to scourge, and to crucify him: and the third day
he shall rise again.

20:20 Then came to him the mother of Zebedee's children with her sons, worshipping him, and desiring a
certain thing of him.

20:21 And he said unto her, What wilt thou? She saith unto him, Grant that these my two sons may sit,
the one on thy right hand, and the other on the left, in thy kingdom.

20:22 But Jesus answered and said, Ye know not what ye ask. Are ye able to drink of the cup that I shall
drink of, and to be baptized with the baptism that I am baptized with? They say unto him, We are able.

20:23 And he saith unto them, Ye shall drink indeed of my cup, and be baptized with the baptism that I
am baptized with: but to sit on my right hand, and on my left, is not mine to give, but it shall be given to
them for whom it is prepared of my Father.

20:24 And when the ten heard it, they were moved with indignation against the two brethren.

20:25 But Jesus called them unto him, and said, Ye know that the princes of the Gentiles exercise
dominion over them, and they that are great exercise authority upon them.

20:26 But it shall not be so among you: but whosoever will be great among you, let him be your minister;

http://breachrepairers.webs.com/ 273
20:27 And whosoever will be chief among you, let him be your servant:

20:28 Even as the Son of man came not to be ministered unto, but to minister, and to give his life a
ransom for many.

20:29 And as they departed from Jericho, a great multitude followed him.

20:30 And, behold, two blind men sitting by the way side, when they heard that Jesus passed by, cried
out, saying, Have mercy on us, O Lord, thou Son of David.

20:31 And the multitude rebuked them, because they should hold their peace: but they cried the more,
saying, Have mercy on us, O Lord, thou Son of David.

20:32 And Jesus stood still, and called them, and said, What will ye that I shall do unto you?

20:33 They say unto him, Lord, that our eyes may be opened.

20:34 So Jesus had compassion on them, and touched their eyes: and immediately their eyes received
sight, and they followed him.

http://breachrepairers.webs.com/ 274
Chapter 21
21:1 And when they drew nigh unto Jerusalem, and were come to Bethphage, unto the mount of Olives,
then sent Jesus two disciples,

21:2 Saying unto them, Go into the village over against you, and straightway ye shall find an ass tied,
and a colt with her: loose them, and bring them unto me.

21:3 And if any man say ought unto you, ye shall say, The Lord hath need of them; and straightway he
will send them.

21:4 All this was done, that it might be fulfilled which was spoken by the prophet, saying,

21:5 Tell ye the daughter of Sion, Behold, thy King cometh unto thee, meek, and sitting upon an ass, and
a colt the foal of an ass.

21:6 And the disciples went, and did as Jesus commanded them,

21:7 And brought the ass, and the colt, and put on them their clothes, and they set him thereon.

21:8 And a very great multitude spread their garments in the way; others cut down branches from the
trees, and strawed them in the way.

21:9 And the multitudes that went before, and that followed, cried, saying, Hosanna to the Son of David:
Blessed is he that cometh in the name of the Lord; Hosanna in the highest.

21:10 And when he was come into Jerusalem, all the city was moved, saying, Who is this?

John 12:17-19

People that were thronged there were the people that Jesus had healed in some way or another.

Luke 19:39-40

It tells us that in the course of time, that a time would come when there would be an event that
would draw the people to the crucifixion. Prophecy would have been fulfilled one way or another.
Today, if we do not turn people to the event of the coming of Jesus, people out in the world will start
to cry out. The stones will cry out. The nature of prophecy, it has to be fulfilled.

“who is this?”

DA 578 Adam will tell you, It is the seed of the woman that shall bruise the serpent's head. Ask
Abraham, he will tell you, It is "Melchizedek King of Salem," King of Peace. Gen. 14:18. Jacob will tell
you, He is Shiloh of the tribe of Judah.Isaiah will tell you, "Immanuel," "Wonderful, Counselor, The
mighty God, The everlasting Father, The Prince of Peace." Isa. 7:14; 9:6. Jeremiah will tell you, The
Branch of David, "the Lord our Righteousness." Jer. 23:6. Daniel will tell you, He is the Messiah. Hosea
will tell you, He is "the Lord God of hosts; the Lord is His memorial." Hosea 12:5. John the Baptist will
tell you, He is "the Lamb of God, which taketh away the sin of the world." John 1:29. The great
Jehovah has proclaimed from His throne, "This is My beloved Son." Matt. 3:17. We, His disciples,
declare, This is Jesus, the Messiah, the Prince of life, the Redeemer of the world. And the prince of the
powers of darkness acknowledges Him, saying, "I know Thee who Thou art, the Holy One of God."
Mark 1:24.”

http://breachrepairers.webs.com/ 275
21:11 And the multitude said, This is Jesus the prophet of Nazareth of Galilee.

21:12 And Jesus went into the temple of God, and cast out all them that sold and bought in the temple,
and overthrew the tables of the moneychangers, and the seats of them that sold doves,

21:13 And said unto them, It is written, My house shall be called the house of prayer; but ye have made it
a den of thieves.
What was the reason why Jesus cleansed the temple the second time?

DA 590 He who had Himself given these prophecies now for the last time repeated the warning. In
fulfillment of prophecy the people had proclaimed Jesus king of Israel. He had received their homage,
and accepted the office of king. In this character He must act. He knew that His efforts to reform a
corrupt priesthood would be in vain; nevertheless His work must be done; to an unbelieving people
the evidence of His divine mission must be given.

Because they had pronounced Him king, then He had to do the duties of a king.

DA 591 They had felt that it was impossible for their undignified surrender to be repeated. Yet they
were now more terrified than before, and in greater haste to obey His command.

21:14 And the blind and the lame came to him in the temple; and he healed them.

21:15 And when the chief priests and scribes saw the wonderful things that he did, and the children
crying in the temple, and saying, Hosanna to the Son of David; they were sore displeased,

21:16 And said unto him, Hearest thou what these say? And Jesus saith unto them, Yea; have ye never
read, Out of the mouth of babes and sucklings thou hast perfected praise?

21:17 And he left them, and went out of the city into Bethany; and he lodged there.

21:18 Now in the morning as he returned into the city, he hungered.

21:19 And when he saw a fig tree in the way, he came to it, and found nothing thereon, but leaves only,
and said unto it, Let no fruit grow on thee henceforward for ever. And presently the fig tree withered
away.

The fig tree represented the Jewish nation. What was the problem? This tree had leaves but had no
fruit. Fig trees, when it has full leaves, it should have figs. Leaves represent outward righteousness.
Fig leaves served to cover the sins of Adam and Eve – self-righteousness. Then God covers them
with His righteousness. Fruit represents winning souls and the fruit of the spirit.

Matt 23:38

The act of cursing the tree was an act to show what would happen to the Jewish nation.

21:20 And when the disciples saw it, they marvelled, saying, How soon is the fig tree withered away!

21:21 Jesus answered and said unto them, Verily I say unto you, If ye have faith, and doubt not, ye shall
not only do this which is done to the fig tree, but also if ye shall say unto this mountain, Be thou removed,
and be thou cast into the sea; it shall be done.

21:22 And all things, whatsoever ye shall ask in prayer, believing, ye shall receive.

21:23 And when he was come into the temple, the chief priests and the elders of the people came unto

http://breachrepairers.webs.com/ 276
him as he was teaching, and said, By what authority doest thou these things? and who gave thee this
authority?

The reason they were trying to trip Him up by His words because they couldn’t trip Him up by His
works.

DA 594 With intense interest the multitude awaited the decision. They knew that the priests had
professed to accept the ministry of John, and they expected them to acknowledge without a question
that he was sent from God. But after conferring secretly together, the priests decided not to commit
themselves. Hypocritically professing ignorance, they said, "We cannot tell." "Neither tell I you," said
Christ, "by what authority I do these things…Scribes, priests, and rulers were all silenced. Baffled and
disappointed, they stood with lowering brows, not daring to press further questions upon Christ. By
their cowardice and indecision they had in a great measure forfeited the respect of the people, who
now stood by, amused to see these proud, self-righteous men defeated.

What is the reason why Jesus gave this parable? It was dealing with the authority of Jesus. It was
also dealing with the baptism of John, whether it was from heaven or not

What is the reason why the chief priests and elders were not willing to answer Jesus about
John’s baptism? They didn’t want to acknowledge that John was heaven sent because Jesus would
ask them why they didn’t acknowledge Him. They didn’t want to reject him because many people
believed that John was a prophet

Why didn’t the chief priests want to acknowledge that John’s baptism was from heaven?
What would be the implication if they did acknowledge it?

John 1:29

Matt 3:13-17

They didn’t want to acknowledge because if they accept John, then they must accept his testimony
of Jesus that He was the Lamb of God that was to take away the sin of the world. They must also
accept the baptism that he did for Jesus and how a voice came from heaven saying that Jesus was
the Son of God. Who does the father represent in the parable?

Matt 6:9

Represents God the Father. What does the vineyard represent?

Psa 80:8, 15

This is speaking of Israel

Isa 5:7

The vineyard is Israel. Who is Israel in our day? The church

What do we understand about the first son and who he represents by his initial response, “I will
not?” He was rebellious. He was living in open transgression. He refused to obey the words of God.
What happened with the first son after he said , “I will not?”

http://breachrepairers.webs.com/ 277
Matt 21:29

Matt 3:1-2

He repented. How is John the Baptist brought into the picture? He preached the message of
repentance. So the first son was one of those that heard the message of John the Baptist and
repented, which shows that John’s baptism was of heaven

What do we understand about the second son by his initial response, “I go, sir,” and the
actual outcome of his actions? He said that he was willing to go but his actions proved his words
worthless. A lot of people know how to talk but they don’t put their actions where their mouths are.
They like to pray, to act big and talk spiritual in front of others, but they do not do put it into
actions. They just have theory.

Who do the first and second sons represent?

Matt 21:31

First son represents the harlots and publicans. Second son represents the chief priests and elders.

What was the father asking the sons to do?

Matt 21:28, 31

Work in the vineyard. Do his will.

What is the will of the Father?

Psa 40:8

Heb 10:16

To delight to do his law, which is to keep the commandments because it is written in the heart

Practically speaking, how can we have the law written in our hearts and how can we keep it
there? Discuss.

Psa 1:2

The way that we can delight in the law and as a result have it written in our hearts is to meditate on
it day and night. Generate discussion about how we can keep it there. How do we make it practical?
It now governs our actions and decisions. The law cannot be in there and remain if we go to movie
theaters and clubs or listen to rock music, etc. Our thoughts are not on God. The law does not
remain there

What was the basis of the actions of why the first son obeyed and the second son didn’t obey?

Matt 21:32

John 14:15

http://breachrepairers.webs.com/ 278
The first group repented and believed. The second didn’t believe because they didn’t repent. The
reason why they didn’t believe it because they didn’t love God. And that is why they didn’t obey.
Obedience it the basis of all our actions.

Is belief enough? What else must we have?

Jas 2:19-20

Belief is not enough. We must have works that correspond to our belief. But the basis of our works
is built upon our belief. And if we have no works, it shows that there is something wrong with our
belief/faith. What is the main lesson that we can learn from this parable?

John 13:17 …if ye know these things, happy are ye if ye do them

Words are of no value unless they are accompanied with appropriate deeds.

21:24 And Jesus answered and said unto them, I also will ask you one thing, which if ye tell me, I in like
wise will tell you by what authority I do these things.

21:25 The baptism of John, whence was it? from heaven, or of men? And they reasoned with themselves,
saying, If we shall say, From heaven; he will say unto us, Why did ye not then believe him?

21:26 But if we shall say, Of men; we fear the people; for all hold John as a prophet.

21:27 And they answered Jesus, and said, We cannot tell. And he said unto them, Neither tell I you by
what authority I do these things.

21:28 But what think ye? A certain man had two sons; and he came to the first, and said, Son, go work to
day in my vineyard.

21:29 He answered and said, I will not: but afterward he repented, and went.

21:30 And he came to the second, and said likewise. And he answered and said, I go, sir: and went not.

21:31 Whether of them twain did the will of his father? They say unto him, The first. Jesus saith unto
them, Verily I say unto you, That the publicans and the harlots go into the kingdom of God before you.

21:32 For John came unto you in the way of righteousness, and ye believed him not: but the publicans
and the harlots believed him: and ye, when ye had seen it, repented not afterward, that ye might believe
him.

The Lord’s Vineyard (33-44)


21:33 Hear another parable: There was a certain householder, which planted a vineyard, and hedged it
round about, and digged a winepress in it, and built a tower, and let it out to husbandmen, and went into
a far country:

What was the context of why this parable was given? Jesus has just addressed the issue of why
the chief priests and elders did not believe John the Baptist and why they had rejected Him. Jesus
had also just given the parable about the two sons and He had shown how the chief priests and
elders were like the second son who said he would go and did not. They didn’t have fruits to show
for their profession. He had just showed them through the parable of the two sons the importance
of obedience

http://breachrepairers.webs.com/ 279
Who does this householder represent that built the vineyard and went into a far country?
And who does the vineyard represent?

Isa 5:7

The householder represents the Lord, which is God the Father. The greek word actually means
master of the house. The vineyard represents the house of Israel.

What does the hedging about represent? Hedging about means fence, enclosing barrier or
partition

Zech 2:5

The Lord is the wall and the glory is manifested in them.

Exo 33:18-19

Glory is the transcript of God’s character

Rom 7:12

The law is also the transcript of God’s character. Therefore, the law = glory = the wall

Why is a winepress built? The owner was obviously expecting good quality grapes to produce
good wine.

What does the tower represent?

2 Sam 22:3

God in the midst of them. How did the Israelite’s have God in the midst of them? – It was the
sanctuary.

21:34 And when the time of the fruit drew near, he sent his servants to the husbandmen, that they might
receive the fruits of it.

Who do the servants represent that were sent to receive the fruits of the vineyard?

Matt 21:34-36

2 Ki 17:13

Jer 25:4

Amos 3:7

The servants represent the prophets

What sort of environment is being described in Matt. 21:33 about the vineyard and how does
this apply to Israel?

http://breachrepairers.webs.com/ 280
Isa 5:2

God who had planted the vineyard gave them every opportunity to develop good fruits. He put a
fence around it. He built a tower for it. God had given Israel every opportunity to bear fruit. Note: A
winepress is just a trough to hold the wine/fruit that is harvested

What do the fruits represent?

Gal 5:22-23

Prov 11:30

The fruits represented the character of God. Also souls won to the kingdom.

21:35 And the husbandmen took his servants, and beat one, and killed another, and stoned another.

21:36 Again, he sent other servants more than the first: and they did unto them likewise.

21:37 But last of all he sent unto them his son, saying, They will reverence my son.

21:38 But when the husbandmen saw the son, they said among themselves, This is the heir; come, let us
kill him, and let us seize on his inheritance.

Who does the son represent?

Matt 21:37-38

John 3:16

Heb 1:2

The son represents Jesus

What is the reason why the husbandmen killed the son? Link this also to the previous
parable. They wanted the inheritance without producing fruit. What kind of inheritance? The
promised land – Canaan. They were looking for the wrong inheritance. They wanted the inheritance
without the son and the householder. They wanted the inheritance without obedience.

What will the winepress be used for at the end?

Rev 14:19, 20,

Rev 19:15

Those who failed to produce fruit (God’s character) for the winepress will themselves be used in
the winepress. God will tread on the wicked. They shall feel His wrath.

What was this parable foreshadowing?

Matt 21:43

http://breachrepairers.webs.com/ 281
It was foreshadowing the destruction of Jerusalem and the reason why it would be destroyed.

Contextually speaking, what is the result of disobedience or faith without works? Lack of
fruits. Rejecting the prophets. Crucifying the Son for the inheritance. How? People today are doing
the same thing by claiming the inheritance of God without actually believing the need for obedience.
End up in the winepress of the wrath of God (Rev. 14:20).

21:39 And they caught him, and cast him out of the vineyard, and slew him.

21:40 When the lord therefore of the vineyard cometh, what will he do unto those husbandmen?

21:41 They say unto him, He will miserably destroy those wicked men, and will let out his vineyard unto
other husbandmen, which shall render him the fruits in their seasons.

21:42 Jesus saith unto them, Did ye never read in the scriptures, The stone which the builders rejected,
the same is become the head of the corner: this is the Lord's doing, and it is marvellous in our eyes?

Heavenly kingdom of God.

DA 597 In quoting the prophecy of the rejected stone, Christ referred to an actual occurrence in the
history of Israel. The incident was connected with the building of the first temple. While it had a
special application at the time of Christ's first advent, and should have appealed with special force to
the Jews, it has also a lesson for us. When the temple of Solomon was erected, the immense stones for
the walls and the foundation were entirely prepared at the quarry; after they were brought to the
place of building, not an instrument was to be used upon them; the workmen had only to place them
in position. For use in the foundation, one stone of unusual size and peculiar shape had been brought;
but the workmen could find no place for it, and would not accept it. It was an annoyance to them as it
lay unused in their way. Long it remained a rejected stone. But when the builders came to the laying
of the corner, they searched for a long time to find a stone of sufficient size and strength, and of the
proper shape, to take that particular place, and bear the great weight which would rest upon it.

DA 597 Should they make an unwise choice for this important place, the safety of the entire building
would be endangered. They must find a stone capable of resisting the influence of the sun, of frost,
and of tempest. Several stones had at different times been chosen, but under the pressure of
immense weights they had crumbled to pieces. Others could not bear the test of the sudden
atmospheric changes. But at last attention was called to the stone so long rejected. It had been
exposed to the air, to sun and storm, without revealing the slightest crack. The builders examined
this stone. It had borne every test but one. If it could bear the test of severe pressure, they decided to
accept it for the cornerstone. The trial was made. The stone was accepted, brought to its assigned
position, and found to be an exact fit.

When the ground freezes the soil will contract. If it contracts, the foundation will move
down. When it heats up again, it will move up again. If a stone cannot resist frost, it will
crumble to pieces. It teaches us about the ministry of Christ. Christ has so long been
rejected, but the stones of man have proved to be insufficient.

We are stones that need to be chiseled at and worked at before being placed as part of the
temple of Matt 22:23-40

If Jesus answered incorrectly, it would show the folly of believing in a resurrection.

DA 605 Should He agree with them, He would give still further offense to the Pharisees. Should He
differ with them, they designed to hold His teaching up to ridicule.

http://breachrepairers.webs.com/ 282
They wanted to make this doctrine look silly. What is the source of erring? Not knowing the Bible.
Therefore Jesus says, “sanctify them through thy truth, thy word is truth.” Jesus brings us something
we know about Heaven – there is no more marriage. Your decisions that you make here, affect you
for eternity. The person that you decide to marry could decide whether you are saved or lost.

Matt 21:31-32

DA 606 God counts the things that are not as though they were. He sees the end from the beginning,
and beholds the result of His work as though it were now accomplished… The dead live unto Him.

When He says He is the God of Abraham, He means that he has already been redeemed.

Rom 4:17, 19

God counts those things that are not, as though they were. When God made the transition from
Abram to Abraham, he had zero children. God counts those things that aren’t, as though they were.

Matt 21:35

What do we know about this lawyer? He discerned in Jesus’ teaching as being truth.

DA 608 The heart of Jesus went out in pity to the honest scribe who had dared to face the frowns of
the priests and the threats of the rulers to speak the convictions of his heart…The scribe was near to
the kingdom of God, in that he recognized deeds of righteousness as more acceptable to God than
burnt offerings and sacrifices.

Matt 21:37-38

The first commandment is the greater. You cannot have true love to your neighbour unless you love
God supremely first. If you have love, you will be in obedience to everything.

DA 606 The Pharisees had exalted the first four commandments, which point out the duty of man to
his Maker, as of far greater consequence than the other six, which define man's duty to his fellow
man. As the result, they greatly failed of practical godliness. Jesus had shown the people their great
deficiency, and had taught the necessity of good works, declaring that the tree is known by its fruits.
For this reason He had been charged with exalting the last six commandments above the first four.

Mark 12:32-34

The lawyer saw the truth in what Jesus said, but the one thing he was missing was to
recognize Christ as the Saviour. He knew the purpose of the law was for obedience and not
all these ceremonial services.

21:43 Therefore say I unto you, The kingdom of God shall be taken from you, and given to a nation
bringing forth the fruits thereof.

21:44 And whosoever shall fall on this stone shall be broken: but on whomsoever it shall fall, it will grind
him to powder.

21:45 And when the chief priests and Pharisees had heard his parables, they perceived that he spake of
them.

http://breachrepairers.webs.com/ 283
21:46 But when they sought to lay hands on him, they feared the multitude, because they took him for a
prophet.

http://breachrepairers.webs.com/ 284
Chapter 22
22:1 And Jesus answered and spake unto them again by parables, and said,

What was the context of why this parable was given? It was still a continuation of what Jesus
was saying to the chief priests and elders from the previous chapter. Jesus was still addressing the
issue of why the Jewish nation didn’t bring forth any fruits.

Who do the king and the son symbolize in this parable?

1 Tim 1:17

King = God the Father

Matt 8:29

Son of God = Jesus

What does this marriage represent? Who is the son marrying?

Rev 19:7

There will be a marriage supper of the Lamb in the future.

Eph 5:23-25, 32

Christ is marrying His church, it is called a mystery.

Eph 1:9-10

The mystery is how God will unite Heaven and Earth. So this marriage is how God is going to unite
Heaven and Earth, Divinity with humanity. The son is marrying the church, those that are faithful to
Him.

What is the difference between the first call and the second call to the marriage?

Matt 22:3-4

The first call is just a general call which we are not given much detail. The second call is given after
the dinner is prepared and the oxen and the fatlings are killed.

What does the dinner and the oxen and fatlings being killed represent?

Jer 11:19

The killing of the oxen was the same as the killing of a lamb.

Isa 53:7

Jesus was that Lamb that was killed for us. This dinner represented the death of Christ. Communion.

http://breachrepairers.webs.com/ 285
Who do the kings servants represent? From the previous study it represents the prophets. These
are the texts that were used previously. Only go through them if there are people that missed the
previous study (Matt. 21:34-36; 2 Kings 17:13; Jer. 25:4; Amos 3:7). Contextually speaking it
represents the disciples before Christ died and also the apostles that gave the gospel message after
Christ died. What did the destruction of the city represent?

Matt 22:7

It represented the destruction of Jerusalem in the future. Note: The first part of this study is related
to the Jews and then not bearing fruit, which we looked at already in the previous parable. But now
we are going to look at the second part which is much more relevant to us.

What happened next? Who do the new invitees represent? The invitation was extended to
those out in the highways. Those whom an invitation had not previously been given. Application:
These represent the gentiles. The gospel was extended to them after the persecution of Stephen.
(Acts 8:4)

What was wrong with the man in verse 11?

Matt 22:11-12

This man received an invitation, but he chose not to wear a wedding garment. From verse 12 we
can see that the king was insulted and angered that someone had dared to enter the wedding feast
without a wedding garment. From the king’s response we can see that everyone who had received
an invitation would have been told that they need to wear a wedding garment.

What does the wedding garment represent? Who is the giver of this garment?

Rev 19:8

It represents the righteousness of the saints.

Eph 5:27

It represents the church as being pure.

Psa 119:172

It represents the church in obedience to the law of God which is righteous.

Rom 13:14

Putting on the garment means to put on Christ. In what way do we put on Christ? We put on
His righteousness/character. Christ supplies this garment. The king and his son.

Why is it so important for us to wear that wedding garment?

Isa 64:6

http://breachrepairers.webs.com/ 286
All our righteousness is as filthy rags

How can one practically wear that garment?

Gal 3:27

We must be baptized.

Rom 6:4-7

We must die to self and crucify self.

Who does that man represent in our world today that came in without a wedding garment?
Discuss. He represents those that claim to be Christian and want the benefits offered to the
followers of Christ, yet they do not feel the need to have a transformation of character. They feel no
need to obey the very thing that will change their characters, the law of God. They have never felt
true repentance of sin. They didn’t feel like they needed Christ’s righteousness. They are merely
hearers of the word.

Why was the man without the wedding garment speechless? How is that applicable to us
today? He was speechless because he didn’t have any excuse. At the end of time when we come
before God, we are not going to have any excuse as to why we didn’t form characters like Christ.
Like the previous parable, Christ has given us all the blessings to us that we may develop Christ-like
characters. Opportunities are given us, but it was up to us to make right use of those opportunities.

According to this parable, what will we be judged by at the end of time? Whether or not we put
on the wedding garment. Whether or not we obeyed the law of God. Whether or not we attained to
the character of Christ.

How can we be chosen today?

Matt 22:14

Many are called but few are chosen.

2 Thes 2:13

We have been chosen through sanctification of the Spirit and belief of the truth.

1 Pet 1:2

Elect (chosen) by sanctification and obedience.

John 17:17

We are sanctified by the truth. Therefore, we are chosen by whether or not we obey the
truth or not.

22:2 The kingdom of heaven is like unto a certain king, which made a marriage for his son,

22:3 And sent forth his servants to call them that were bidden to the wedding: and they would not come.

http://breachrepairers.webs.com/ 287
22:4 Again, he sent forth other servants, saying, Tell them which are bidden, Behold, I have prepared my
dinner: my oxen and my fatlings are killed, and all things are ready: come unto the marriage.

22:5 But they made light of it, and went their ways, one to his farm, another to his merchandise:

22:6 And the remnant took his servants, and entreated them spitefully, and slew them.

22:7 But when the king heard thereof, he was wroth: and he sent forth his armies, and destroyed those
murderers, and burned up their city.

22:8 Then saith he to his servants, The wedding is ready, but they which were bidden were not worthy.

22:9 Go ye therefore into the highways, and as many as ye shall find, bid to the marriage.

22:10 So those servants went out into the highways, and gathered together all as many as they found,
both bad and good: and the wedding was furnished with guests.

22:11 And when the king came in to see the guests, he saw there a man which had not on a wedding
garment:

22:12 And he saith unto him, Friend, how camest thou in hither not having a wedding garment? And he
was speechless.

22:13 Then said the king to the servants, Bind him hand and foot, and take him away, and cast him into
outer darkness; there shall be weeping and gnashing of teeth.

22:14 For many are called, but few are chosen.

22:15 Then went the Pharisees, and took counsel how they might entangle him in his talk.

DA 602 He had rebuked their hypocrisy and presumption, and in doing this He had stated a great
principle, a principle that clearly defines the limits of man's duty to the civil government and his duty
to God. In many minds a vexed question had been settled. Ever after they held to the right principle.
And although many went away dissatisfied, they saw that the principle underlying the question had
been clearly set forth, and they marveled at Christ's far-seeing discernment. Christ's reply was no
evasion, but a candid answer to the question. Holding in His hand the Roman coin, upon which were
stamped the name and image of Caesar, He declared that since they were living under the protection
of the Roman power, they should render to that power the support it claimed, so long as this did not
conflict with a higher duty. But while peaceably subject to the laws of the land, they should at all
times give their first allegiance to God.

22:16 And they sent out unto him their disciples with the Herodians, saying, Master, we know that thou
art true, and teachest the way of God in truth, neither carest thou for any man: for thou regardest not
the person of men.

22:17 Tell us therefore, What thinkest thou? Is it lawful to give tribute unto Caesar, or not?

22:18 But Jesus perceived their wickedness, and said, Why tempt ye me, ye hypocrites?

22:19 Shew me the tribute money. And they brought unto him a penny.

22:20 And he saith unto them, Whose is this image and superscription?

22:21 They say unto him, Caesar's. Then saith he unto them, Render therefore unto Caesar the things

http://breachrepairers.webs.com/ 288
which are Caesar's; and unto God the things that are God's.

22:22 When they had heard these words, they marvelled, and left him, and went their way.

22:23 The same day came to him the Sadducees, which say that there is no resurrection, and asked him,

22:24 Saying, Master, Moses said, If a man die, having no children, his brother shall marry his wife, and
raise up seed unto his brother.

22:25 Now there were with us seven brethren: and the first, when he had married a wife, deceased, and,
having no issue, left his wife unto his brother:

22:26 Likewise the second also, and the third, unto the seventh.

22:27 And last of all the woman died also.

22:28 Therefore in the resurrection whose wife shall she be of the seven? for they all had her.

22:29 Jesus answered and said unto them, Ye do err, not knowing the scriptures, nor the power of God.

22:30 For in the resurrection they neither marry, nor are given in marriage, but are as the angels of God
in heaven.

22:31 But as touching the resurrection of the dead, have ye not read that which was spoken unto you by
God, saying,

22:32 I am the God of Abraham, and the God of Isaac, and the God of Jacob? God is not the God of the
dead, but of the living.

22:33 And when the multitude heard this, they were astonished at his doctrine.

22:34 But when the Pharisees had heard that he had put the Sadducees to silence, they were gathered
together.

22:35 Then one of them, which was a lawyer, asked him a question, tempting him, and saying,

22:36 Master, which is the great commandment in the law?

22:37 Jesus said unto him, Thou shalt love the Lord thy God with all thy heart, and with all thy soul, and
with all thy mind.

22:38 This is the first and great commandment.

22:39 And the second is like unto it, Thou shalt love thy neighbour as thyself.

22:40 On these two commandments hang all the law and the prophets.

22:41 While the Pharisees were gathered together, Jesus asked them,

22:42 Saying, What think ye of Christ? whose son is he? They say unto him, The Son of David.

22:43 He saith unto them, How then doth David in spirit call him Lord, saying,

22:44 The LORD said unto my Lord, Sit thou on my right hand, till I make thine enemies thy footstool?

http://breachrepairers.webs.com/ 289
22:45 If David then call him Lord, how is he his son?

22:46 And no man was able to answer him a word, neither durst any man from that day forth ask him
any more questions.

http://breachrepairers.webs.com/ 290
Chapter 23 – Warning to the Scribes and Pharisees
Jesus was speaking to His followers as well as the multitude (Matthew 23:1) In the last week of His
ministry it is important that He speak plainly The people depended on the leadership without
studying out what they were taught Jesus removed the mask of those they were following.

Chapter Outline
 Position of the Scribes and Pharisees (1-12)
 Condition of the Scribes and Pharisees (13-31)
 Solution, Warning, and Final Judgment (32-39)

Position of the Scribes and Pharisees (1-12)

23:1 Then spake Jesus to the multitude, and to his disciples,

Speaking to the disciples and the multitude, all His followers. Necessary for Jesus to speak plainly so
the people wouldn’t be following their leaders who were leading them to eternal ruin

23:2 Saying, The scribes and the Pharisees sit in Moses' seat:

Verses 2 to 10 Authority & Power. Observation: Moses Seat, Rabbi, Master, Father, Bid-instruct,
Bind burdens.
Moses a very prominent individual, a leader of God’s people, used to build sanctuary. (App: NT
Pastors)

“Seat” – authority and power (Rev 13:2). Jesus recognized these were in power and authority
(Master, Rabbi,

“Father” – all terms of authority among Jewish people). They believed they had to instruct the
people to lead them to understanding salvation

23:3 All therefore whatsoever they bid you observe, that observe and do; but do not ye after their works:
for they say, and do not.

“Observe and do” – In chap. 15:1-10, Jesus exposed the false teachings of the Pharisees. Is this
contradiction? As long as they taught things according to the Word or Law of God we have
responsibility to follow that truth God can use all avenues to distribute His truth. The more light we
have, the more accountable or responsible we are for it.

“They say and do not” – Don’t live like them, but observe and do the teaching if it’s in harmony
with God’s will. We are to obey and follow the word of God

23:4 For they bind heavy burdens and grievous to be borne, and lay them on men's shoulders; but they
themselves will not move them with one of their fingers.

“Bind heavy burdens” – Requiring things to do or keep which God never had told should be kept
or followed
Sabbath forms and ceremonies

Isa 58:3

http://breachrepairers.webs.com/ 291
Fasting and afflicting their soul (DOA: 1. Garments; 2. Work; 3. Afflict Soul). Sanctuary language

Isa 58:4-6

“True fast” – let go of heavy burdensPharisees know about sanctuary but miss the heart experience

Isa 58:12

Prophecy of restoring the breach, the Sabbath. Sabbath is one of 10 commandments

“Law of God” –Most Holy Place

“True fast” – loosing heavy burdens, true Sabbath-keepers

Gal 6:2

Bearing burdens of one another

1 Jn 3:23

Love one another. How to keep the law of Christ? Bearing one another’s burdens and manifest love of God
toward each other. This is the experience of the true fast

Matt 11:28-30

Heavy laden, burden is light. Jesus example of true fast.

23:5 But all their works they do for to be seen of men: they make broad their phylacteries, and enlarge
the borders of their garments,

23:6 And love the uppermost rooms at feasts, and the chief seats in the synagogues,

23:7 And greetings in the markets, and to be called of men, Rabbi, Rabbi.

23:8 But be not ye called Rabbi: for one is your Master, even Christ; and all ye are brethren.

Matt 20:25

Gentile exercise Lordship. This should not be among you.

23:9 And call no man your father upon the earth: for one is your Father, which is in heaven.

23:10 Neither be ye called masters: for one is your Master, even Christ.

23:11 But he that is greatest among you shall be your servant.

“servant” – Leaders held up Moses in high position. They forgot one thing about Moses.

Num 12:3

Moses was the meekest man. They forgot the humility of Jesus. Also notice that Jesus speaks
to them in this way so that they can feel their need of Him once again.

http://breachrepairers.webs.com/ 292
23:12 And whosoever shall exalt himself shall be abased; and he that shall humble himself shall be
exalted.

What does this mean?

Lk 18:12-14

Pharisee is standing up. Publican is bowed down very low—humbling himself

“Shall be exalted” – justified

“Abase” – condemned

Dan 8:14

“Cleansed” = justified. God is looking for those who are humble like the publican and not proud like
Pharisees and Scribes Humility? Are they listening to the prophets? Are they experiencing the love
of Christ and manifesting it to others? Do we have righteousness of Jesus Christ? Where do we
stand?

Condition of the Scribes and Pharisees (13-31)


23:13 But woe unto you, scribes and Pharisees, hypocrites! for ye shut up the kingdom of heaven against
men: for ye neither go in yourselves, neither suffer ye them that are entering to go in.

23:14 Woe unto you, scribes and Pharisees, hypocrites! for ye devour widows' houses, and for a pretence
make long prayer: therefore ye shall receive the greater damnation.

“Long prayer” – Anyone pray long prayers? Jesus not condemning the duration of the prayer but
the motivation. A Pharisee’s Prayer

Lk 18:9-11

Believing you are righteous but despising others at the same time

23:15 Woe unto you, scribes and Pharisees, hypocrites! for ye compass sea and land to make one
proselyte, and when he is made, ye make him twofold more the child of hell than yourselves.

Do you Scribes and Pharisees win souls? Yes.

“Proselyte” – a convert. They even had zeal “compass sea and land.” But they make that soul
twofold more a child of hell than themselves. Both children of heaven and hell win souls, but what’s
the Difference? Child of hell makes hypocrites like himself. Child of heaven makes sons and
daughters of God.

23:16 Woe unto you, ye blind guides, which say, Whosoever shall swear by the temple, it is nothing; but
whosoever shall swear by the gold of the temple, he is a debtor!

23:17 Ye fools and blind: for whether is greater, the gold, or the temple that sanctifieth the gold?

23:18 And, Whosoever shall swear by the altar, it is nothing; but whosoever sweareth by the gift that is
upon it, he is guilty.

http://breachrepairers.webs.com/ 293
23:19 Ye fools and blind: for whether is greater, the gift, or the altar that sanctifieth the gift?

23:20 Whoso therefore shall swear by the altar, sweareth by it, and by all things thereon.

23:21 And whoso shall swear by the temple, sweareth by it, and by him that dwelleth therein.

23:22 And he that shall swear by heaven, sweareth by the throne of God, and by him that sitteth thereon.

23:23 Woe unto you, scribes and Pharisees, hypocrites! for ye pay tithe of mint and anise and cummin,
and have omitted the weightier matters of the law, judgment, mercy, and faith: these ought ye to have
done, and not to leave the other undone.

“Weightier matters” – Judgment, Mercy, Faith. Even if our money is helping to bring souls into the
kingdom we are lost because we don’t exercise compassion and love for our brother.

23:24 Ye blind guides, which strain at a gnat, and swallow a camel.

23:25 Woe unto you, scribes and Pharisees, hypocrites! for ye make clean the outside of the cup and of
the platter, but within they are full of extortion and excess.

23:26 Thou blind Pharisee, cleanse first that which is within the cup and platter, that the outside of them
may be clean also.

“Blind” - Church of Laodicea = Pharisaism

“Cleanse first the inside” – Fix the heart first and the rest will come, don’t put long dress first, they
will still be in their sins

23:27 Woe unto you, scribes and Pharisees, hypocrites! for ye are like unto whited sepulchres, which
indeed appear beautiful outward, but are within full of dead men's bones, and of all uncleanness.

23:28 Even so ye also outwardly appear righteous unto men, but within ye are full of hypocrisy and
iniquity.

23:29 Woe unto you, scribes and Pharisees, hypocrites! because ye build the tombs of the prophets, and
garnish the sepulchres of the righteous,

23:30 And say, If we had been in the days of our fathers, we would not have been partakers with them in
the blood of the prophets.

How many of you would’ve killed Isaiah? Put Christ on the cross? Every time we sin we do this. A
voice, walk ye in it – you disobey the prophet and thus slay them. Concealed sin – God gives us a
warning – exposing it in public – we want to slay them

23:31 Wherefore ye be witnesses unto yourselves, that ye are the children of them which killed the
prophets.

Solution, Warning, and Final Judgment (32-39)


23:32 Fill ye up then the measure of your fathers.

23:33 Ye serpents, ye generation of vipers, how can ye escape the damnation of hell?

http://breachrepairers.webs.com/ 294
23:34 Wherefore, behold, I send unto you prophets, and wise men, and scribes: and some of them ye shall
kill and crucify; and some of them shall ye scourge in your synagogues, and persecute them from city to
city:

“Prophets” – Given to help change the inside. 95% of Ellen G Whites writings discusses character
development.

“wise men and scribes” – Send one of your own rank to help you understand where you’re off the
path. Will we change? Will we listen?

23:35 That upon you may come all the righteous blood shed upon the earth, from the blood of righteous
Abel unto the blood of Zacharias son of Barachias, whom ye slew between the temple and the altar.

23:36 Verily I say unto you, All these things shall come upon this generation.

23:37 O Jerusalem, Jerusalem, thou that killest the prophets, and stonest them which are sent unto thee,
how often would I have gathered thy children together, even as a hen gathereth her chickens under her
wings, and ye would not!

23:38 Behold, your house is left unto you desolate.

23:39 For I say unto you, Ye shall not see me henceforth, till ye shall say, Blessed is he that cometh in the
name of the Lord.

http://breachrepairers.webs.com/ 295
Chapter 24
DA 628 In mercy to them He blended the description of the two great crises, leaving the disciples to
study out the meaning for themselves.

Jesus was describing the end of the world and the destruction of Jerusalem.

DA 627 As Christ's attention was attracted to the magnificence of the temple, what must have been
the unuttered thoughts of that Rejected One! The view before Him was indeed beautiful, but He said
with sadness, I see it all. The buildings are indeed wonderful. You point to these walls as apparently
indestructible; but listen to My words: The day will come when "there shall not be left one stone
upon another, that shall not be thrown down.

This was literally fulfilled

24:1 And Jesus went out, and departed from the temple: and his disciples came to him for to shew him
the buildings of the temple.

24:2 And Jesus said unto them, See ye not all these things? verily I say unto you, There shall not be left
here one stone upon another, that shall not be thrown down.

24:3 And as he sat upon the mount of Olives, the disciples came unto him privately, saying, Tell us, when
shall these things be? and what shall be the sign of thy coming, and of the end of the world?

The destruction of the temple was equivalent to being the end of the world for them.

24:4 And Jesus answered and said unto them, Take heed that no man deceive you.

Matt 24:4, 5, 24

Jesus had a lot of emphasis about this issue of deception. Barabbas and Jesus were stood next to
each other and people were asked to choose. There was a false and a truth represented. At the end
of time the papacy will be the major anti-Christ

24:5 For many shall come in my name, saying, I am Christ; and shall deceive many.

Read over. As we continue to read these texts we will see how many will be deceived. When we
read this text we think that many people shall come in the name of Christ, and many will be
deceived because they will believe that they are Christ. But for us that is pretty easy. If someone
where to come to us and say they are Christ you will probably have a hard time holding back your
laughter. So for good Bible students perhaps that kind of deception will not work. Perhaps it will be
more subtle than that. But notice what happens from verse 6 and onward.

24:6 And ye shall hear of wars and rumours of wars: see that ye be not troubled: for all these things must
come to pass, but the end is not yet.

“And ye shall hear of wars and rumours of wars” – I contribute this event to false Christs. False
Christs are behind wars and rumors of wars. And the Bible says

“see that ye be not troubled” – I am sure that many Seventh-day Adventist that lived during
World War 1 and World War 2 were tempted to think that the end is here. But the Bible says

http://breachrepairers.webs.com/ 296
“for all [these things] must come to pass, but the end is not yet” – Side Note: Just connect that to
Revelation 17 and you will have something very interesting.

24:7 For nation shall rise against nation, and kingdom against kingdom: and there shall be famines, and
pestilences, and earthquakes, in divers places.

“For nation shall rise against nation, and kingdom against kingdom” – Right here it shows the
condition of the world. It is simply saying that the nations and kingdoms are not united. They are
divided. And then the Bible says

“and there shall be famines, and pestilences, and earthquakes, in divers places” – Take these
three words: Famines, pestilences, and earthquakes and we are looking at natural disasters, or you
can just simply say disasters. Verse 8 the Bible says after the natural disasters

24:8 All these are the beginning of sorrows.

24:9 Then shall they deliver you up to be afflicted, and shall kill you: and ye shall be hated of all nations
for my name's sake.

Read over

“Then shall they…of all” – For my what? Verse 9 is very important. Notice what is happening here.
Who are they? Who are they? It is based upon the previous texts. They is referring to those who
come in the name of Christ and they is referring to the nations. So some how there is mixture
between the religious bodies and the political bodies and they are combined together. Perhaps
Church & State

“Then shall they deliver you up to be afflicted, and shall kill you: and ye shall be hated of all
nations for my name's sake” – Shall be hated of how many nations? All nations. For my name's?
Sake. Right here we see that all nations are united. They are united to persecute. Before they were
divided but now they are united. What took place in the middle? Natural disasters. So then they are
being hated for what? One thing is for sure that the nations were divided, and then you have
disasters, and some how those disasters bring the nations together as one. Are we seeing something
like this today? Yes Are we seeing global warming bringing nations together? Yes Are we seeing
terrorism bringing nations together? Yes. So the Bible says they are all united to HATE those who
uphold the name of God. Why? If I can read between the lines they are being HATED because all
nations are thinking that they are the ones who are causing these natural disasters. Do we see any
other place in the Bible that gives us a similar example? Was there a time when the people of God
were blamed for natural disasters? Yes, Elijah. Are thou the one who troubles Israel. You are the
one that is causing the heavens not to poor out the rain. You are causing this natural disaster, and
they wanted to kill him. But it is very interesting the Bible says

They are being HATED of all nations. For whose name? My name's sake. The name of God. What is
God's name? If we compare this with the book of Revelation since we know that Matt 24 & 25 is a
miniature picture of the book of Revelation. The name of God based upon the book of Revelation if
you look at chapter 14 you have a picture of the 144,000 and they have the Father's name written in
their foreheads. And Jesus said if you have seen me you have seen the Father. And then in Rev 7 you
have the 144,000 and they have what in their forehead? The Seal of the Living God. Therefore the
Name of God is connected to the Seal of the Living God. And the Seal of the Living God is what? The
Sabbath, within the 10 Commandment the Sabbath is the only commandment that has the name of
God. So they are being HATED for upholding the Name of God by keeping the 7th Day Sabbath. So

http://breachrepairers.webs.com/ 297
that HATRED will come from the outside of the church. And it will then come into God's church.
Next verse

24:10 And then shall many be offended, and shall betray one another, and shall hate one another.

“And then shall many be offended” – Shall many be what? Offended. Who are the many? People
who are in the church. The people who are in the church shall be offended. And shall do what?
Betray. Those who are offended. Guess what they are going to do?

“betray one another” – That phrase one another gives us a picture of fellowship. One another.
They are together. So within a close group they are HATING each other. HATRED comes from the
outside, and then the people inside begin to HATE each other. But who are hating each other? Those
who are OFFENDED. And those who are Offended they will BETRAY. And then the Bible says
“and shall hate one another” – And while they are HATING one another. Now they are prepared
for another deception. Verse 11

24:11 And many false prophets shall rise, and shall deceive many.

“shall deceive MANY” – Because when you are HATING each other, you are looking for direction,
you are looking for a leader. So then Satan sends false prophets to mislead the people who are
hating each other. But my question is. Why are the people in the church so easily offended? And I
begin to discover what the Bible has to say about why we so easily get offended. Do you remember
the parable that Jesus gave regarding the 4 grounds? The wayside, stony place, thorny place, and
the good ground. When you look at that parable the sower is spreading the seed in all 4 places. If I
was the farm manager I would fire him because 75% of the seed is going to the wrong place and
only 25% is going to the right place. And you ask yourself why is the sower wasting his seed on
these places that cannot produce any fruit.

Most likely the waste side is right next to the field that is the only way the seeds can go there. No
farmer would look at the way side and say this looks good let me put them there. Most likely he is
throwing the seed to the good ground, but perhaps the lighter seeds are carried by the wind. And
some how they land on the way side. And you ask why is he throwing his seed on the stony places.
Perhaps it is located somewhere between the waste side and the field. Why? Because when you
work on the farm not only do you harvest potatoes, but you also harvest stones, and what do you do
with them? You throw them to the edge of the field. And you ask yourself what are these thistles,
where are they growing? Perhaps right next to or close by the stony places because you cannot get
to them. They are growing between the stony place and the field.

Some people are in the way side. And I will illustrate this: Some people in the church they like to sit
way back there. And you know how it is. They like to sit way back there because they do not want to
be fully involved in the church. And then some people sit somewhere in-between. Perhaps that is a
plain illustration, but it is just to let you know we have 4 conditions of the heart. But I want to
concentrate on one ground and that is the stony place. This is what Jesus said

Matt 13:5-6 [5] Some fell upon stony places, where they had not much earth: and forthwith they
sprung up, because they had no deepness of earth: [6] And when the sun was up, they were scorched;
and because they had no root, they withered away.

“sun was up they were scorched” – they were what? Scorched because they had no? Root (they
withered away). Let us consider the interpretation that Jesus gave.

http://breachrepairers.webs.com/ 298
Matt 13:20 But he that received the seed into stony places, the same is he that heareth the word, and
anon with joy receiveth it;

“But he….heareth the word” –Stop right there. Question: Those who are in the stony places. Are
they hearing the word? Are they listening to sermons? Are they studying the Bible? Yes they are.
And because of that they should respond with Faith, because the Bible says that Faith comes by
hearing, and hearing by the Word of God. (Rom 10:17). So they should have faith. So it is very
possible that they have some elements of faith.

“But he…joy receiveth it” – What is the attitude of those who receive the word in the stony places?
JOY. Is that positive or negative? Positive. They get all excited! Did you hear that? That was so
powerful! That was soo good! Man that was wonderful! That is exactly what we need to hear. This is
what my elder needs to hear, my deacon needs to hear, my family needs to hear, this is what we
need to hear. They get all joyful and excited. But based upon the parable of Jesus what was the
problem? There was no root. Not that there wasn't literally any root. But it was not deep enough.
Next verse

Matt 13:21 Yet hath he not root in himself, but dureth for a while: for when tribulation or
persecution ariseth because of the word, by and by he is offended.

“Yet hath he not root in himself, but dureth for a while: for when tribulation or persecution
ariseth” – Yet hath he not what? Root But dureth for a what? While. Tribulation or persecution,
next word? Ariseth. In the parable what rises? The Sun rises. Question: Is the Sun for the plant or
against the plant? For the plant. Question: Is persecution and tribulation against you or for you? For
you. PERSECUTION AND TRIBULATION IS ONLY FOR YOU, IF YOU HAVE THE ROOT (CONNECT
WITH JAMES AND 1 PETER) . IF YOU DON'T HAVE THE ROOT IT IS AGAINST YOU!! WITH THE
ROOT EITHER YOU WILL WITHER AWAY OR EVEN GROW MORE STRONGER!! These people when
they hear the word they get all excited and joyful they have these emotional highs. But they do not
know who they are inside.

“when tribulation or persecution ariseth because of the word, by and by he is offended” –


because of the what? Word By and by he is? Offended. Now let me ask you a question. In Matthew
24 when God's church is being persecuted by the nations of the world, and the Bible says people in
the church are offended. Tell me from the bible why are they offended? Because they have no root.
Question: What is that root?

Eph 3:17 That Christ may dwell in your hearts by faith; that ye, being rooted and grounded in love.

“dwell in your hearts by faith” – By what? Faith

“rooted and grounded in love” – So tell me from the Bible what is that root? Love. It is so easy to
have a sprout, steam, and leaves: dress reform, diet reform, recreation reform, it is so easy to have
these beautiful green leaves, devotional life, and educational reform, etc. But it is very possible that
it is only a display and there is no root. There is no true love inside. How do we know if we have that
love or not? Here is how. A root is something that you don't display. And that you cannot see. It is
beneath the earth. No farmer will show off his roots. Come here and look at these roots. He only
shows off the fruit. So the ROOT represents who you are when people cannot see you. it represents
who you are in the dark cold room. Do you have the Love of God when no one is watching you? Who
are you when you are alone? If we have that root. When persecution or tribulation comes to us it
will only increase our love to God even more. Just knowing the prophecies of the Bible will not

http://breachrepairers.webs.com/ 299
prepare us, although it is very important. Just having the reforms will not prepare us although they
are very import to protect our faith experience in God. Just because we have true education. True
education that is only a form without the power will not prepare us. There must be a heart work.
Searching our hearts deeply that we know that we truly love God. And at the end the words of
Christ will prove true. If ye love me keep my commandments! May that be our experience today.

24:12 And because iniquity shall abound, the love of many shall wax cold.

24:13 But he that shall endure unto the end, the same shall be saved.

24:14 And this gospel of the kingdom shall be preached in all the world for a witness unto all nations;
and then shall the end come.

24:15 When ye therefore shall see the abomination of desolation, spoken of by Daniel the prophet, stand
in the holy place, (whoso readeth, let him understand:)

24:16 Then let them which be in Judaea flee into the mountains:

24:17 Let him which is on the housetop not come down to take any thing out of his house:

24:18 Neither let him which is in the field return back to take his clothes.

24:19 And woe unto them that are with child, and to them that give suck in those days!

Intro: 4 Case studies on the family (divorce and dysfunctional homes)

Warning to the Family

Matt 24:12

Love of many (family) waxes cold

Matt 24:15-19

Woe to the family why?

Matt 24:10

Mk 13:12

Family betrays each other

Lk 21:16, 12

Cause to be put to death, how? Church/State. Mystery of God (Restoring the Family).

Rev 10:6-7

Rev 11:15, 19

Mystery of God complete from 1844

http://breachrepairers.webs.com/ 300
Eph 1:9-10

Mystery of God (Unity between Heaven and Earth)

Eph 5:31-32

Two become one flesh Mystery of God

PK 678

Every institution restored. Family Counsel

Eph 5:25-29

1 Tm 5:8

Counsel to Husband.

Prov 31:10-11, 16, 26-27, 29-31

Wife needs to be virtuous

Prov 4:3, 14-19

Listen to Father’s instruction.

Appeal:

Mal 4:5-6

3rd Elijah will turn hearts of the fathers to the children. The scriptures present the condition of two
types of families in the last days. Families that have been restored. Families that are betraying each
other and causing them to be put to death.

EGW: How you live in the home shows if you are ready for heaven or not.

Will you do your part to complete the mystery of God in your home and heed the warning of Christ?

Woman

Prov 24:27 Prepare thy work without, and make it fit for thyself in the field; and afterwards build
thine house.

Prov 14:1 Every wise woman buildeth her house: but the foolish plucketh it down with her hands.

Prov 24:3 Through wisdom is an house builded; and by understanding it is established…

EGW Quotes

5T 541 Restrain Children -- The Lord Jesus, on the Mount of Olives, plainly stated that "because
iniquity shall abound, the love of many shall wax cold." He speaks of a class who have fallen from a
high state of spirituality.. The false idea entertained by many, that the restraining of children is

http://breachrepairers.webs.com/ 301
an injury, is ruining thousands upon thousands. Satan will surely take possession of the children
if you are not on your guard. Do not encourage their association with the ungodly. Draw them away.
Come out from among such yourselves, and show them that you are on the Lord's side.

AH 15 Home Is the Heart of All Activity.--Society is composed of families, and is what the heads of
families make it. Out of the heart are "the issues of life"; and the heart of the community, of the
church, and of the nation is the household. The well-being of society, the success of the church,
the prosperity of the nation, depend upon home influences. The elevation or deterioration of the
future of society will be determined by the manners and morals of the youth growing up around us.
As the youth are educated, and as their characters are molded in their childhood to virtuous habits, self-
control, and temperance, so will their influence be upon society. If they are left unenlightened and
uncontrolled, and as the result become self-willed, intemperate in appetite and passion, so will be
their future influence in molding society. (1) The company which the young now keep, (2) the habits
they now form, and (3) the principles they now adopt are the index to the state of society for years to
come.

PK 678 Every Institution to be Restored -- In the time of the end every divine institution is to be
restored.

24:20 But pray ye that your flight be not in the winter, neither on the sabbath day:

24:21 For then shall be great tribulation, such as was not since the beginning of the world to this time,
no, nor ever shall be.

24:22 And except those days should be shortened, there should no flesh be saved: but for the elect's sake
those days shall be shortened.

24:23 Then if any man shall say unto you, Lo, here is Christ, or there; believe it not.

24:24 For there shall arise false Christs, and false prophets, and shall shew great signs and wonders;
insomuch that, if it were possible, they shall deceive the very elect.

24:25 Behold, I have told you before.

24:26 Wherefore if they shall say unto you, Behold, he is in the desert; go not forth: behold, he is in the
secret chambers; believe it not.

24:27 For as the lightning cometh out of the east, and shineth even unto the west; so shall also the
coming of the Son of man be.

24:28 For wheresoever the carcase is, there will the eagles be gathered together.

24:29 Immediately after the tribulation of those days shall the sun be darkened, and the moon shall not
give her light, and the stars shall fall from heaven, and the powers of the heavens shall be shaken:

24:30 And then shall appear the sign of the Son of man in heaven: and then shall all the tribes of the
earth mourn, and they shall see the Son of man coming in the clouds of heaven with power and great
glory.

24:31 And he shall send his angels with a great sound of a trumpet, and they shall gather together his
elect from the four winds, from one end of heaven to the other.

24:32 Now learn a parable of the fig tree; When his branch is yet tender, and putteth forth leaves, ye
know that summer is nigh:

http://breachrepairers.webs.com/ 302
24:33 So likewise ye, when ye shall see all these things, know that it is near, even at the doors.

24:34 Verily I say unto you, This generation shall not pass, till all these things be fulfilled.

24:35 Heaven and earth shall pass away, but my words shall not pass away.

24:36 But of that day and hour knoweth no man, no, not the angels of heaven, but my Father only.

24:37 But as the days of Noe were, so shall also the coming of the Son of man be.

24:38 For as in the days that were before the flood they were eating and drinking, marrying and giving
in marriage, until the day that Noe entered into the ark,

24:39 And knew not until the flood came, and took them all away; so shall also the coming of the Son of
man be.

24:40 Then shall two be in the field; the one shall be taken, and the other left.

24:41 Two women shall be grinding at the mill; the one shall be taken, and the other left.

24:42 Watch therefore: for ye know not what hour your Lord doth come.

24:43 But know this, that if the goodman of the house had known in what watch the thief would come, he
would have watched, and would not have suffered his house to be broken up.

24:44 Therefore be ye also ready: for in such an hour as ye think not the Son of man cometh.

24:45 Who then is a faithful and wise servant, whom his lord hath made ruler over his household, to give
them meat in due season?

Which season does the faithful and wise servant give meat in? We have two passages that give us an
indication.

Matt 24:12 And because iniquity shall abound, the love of many shall wax cold.

This shows us that we are in a part of earth's history where iniquity will continue to abound. It will
get worse and worse. But out of the 4 seasons of the earth, which one is the coldest? Winter. So
these faithful and wise servants will be giving meat to in inhabitant of the earth during the winter.

Extra: Meat in due season meaning. When this part of the season comes, there is a specific type of
food that you should be given. This is what we call present truth.

Matt 24:32 Now learn a parable of the fig tree; When his branch is yet tender, and putteth forth
leaves, ye know that summer [is] nigh…

What is before summer? Spring and Winter. Note: The leaves during spring are like the signs of the
times. Investigative judgment happens during winter and spring. During the end of spring you see
the leaves growing back. This tells you that summer is around the corner. But what happens during
the summer time? The harvest. So at the second coming Christ is coming with a sickle in His hand
because He is going to do what? Reap (Rev 14:14) Christ's second coming is likened to the summer.

But the question is who is the wise and faithful servant? Those who are giving the 3 angel's
messages (but ultimately the 144,000). These messages prepare the harvest.

http://breachrepairers.webs.com/ 303
Extra: Fear God and give glory. The fear of the Lord is the beginning of wisdom Psalms 111:10 They
are the wise servants.

But there are two harvest (Rev 14:14; 14:15, 19-20) The righteous and the wicked. (The wise
servant and the evil servant). The evil servants say Jeremiah 8:20 "The harvest is past, the summer
is ended, and we are not saved."

24:46 Blessed is that servant, whom his lord when he cometh shall find so doing.

24:47 Verily I say unto you, That he shall make him ruler over all his goods.

24:48 But and if that evil servant shall say in his heart, My lord delayeth his coming;

24:49 And shall begin to smite his fellowservants, and to eat and drink with the drunken;

24:50 The lord of that servant shall come in a day when he looketh not for him, and in an hour that he is
not aware of,

24:51 And shall cut him asunder, and appoint him his portion with the hypocrites: there shall be
weeping and gnashing of teeth.

http://breachrepairers.webs.com/ 304
Chapter 25
25:1 Then shall the kingdom of heaven be likened unto ten virgins, which took their lamps, and went
forth to meet the bridegroom.

Matt 25 is a continuation of Christ’s sermon beginning in Matt 24. With this in mind what theme do
you think this parable is related to?

Matt 25:13

The time of the end, just before Jesus’ second coming.

Who were the virgins looking forward to meeting? Who does he represent? The bridegroom.
The bridegroom represents God

Isa 62:5

Specifically Jesus

Eph 5:23

The bridegroom is coming for the bride. Who does she represent?

Eph 5:23

The bride = church = God’s people. Christ is coming back for His people

What were the intentions of the virgins? Who do the virgins represent?

2 Corin 11:2

They were waiting to meet the bridegroom. They were part of the bridal party. Background: In
middle eastern weddings, the bridegroom would come to the bride’s house to receive her and bring
her back to the bridegroom’s house for the wedding feast. These virgins represent God’s people at
the end, who are part of the bridal party, waiting for Christ’s coming.

“chaste” = pure. These are people who profess to have a pure faith, a pure heart.

What do the lamps held by the virgins represent?

Psa 119:105

Lamps = word of God. These people professed their faith and purity on the word of God. They kept
the word.

What do the foolish and wise virgins have in common? What are their differences? Common:
Both are virgins, All have lamps, All fell asleep, All are woken up at midnight. Difference: The wise
virgins had extra oil in their vessels.

What does the oil represent?

http://breachrepairers.webs.com/ 305
Zech 4:2-4, 12

Zechariah asks what the candlestick and oil represents.

Zech 4:6

The angel answers that it represents the spirit of the Lord = Holy Spirit.

When did it become obvious that the foolish virgins were different from the wise?

Matt 25:5-7

After the delay. Whilst both fell asleep, lamps went out. When the midnight cry came.

What happens after the midnight cry? What does this represent in the end of time?

Matt 25:6

The coming of the bridegroom = Coming of Christ, that happens at the darkest hour of earth’s
history. Because the foolish virgins had no oil, what were they not able to do with their
lamps? What does this represent?

Matt 5:16

The foolish virgins were not able to keep their lamps lighted. Light = good works. These foolish
virgins were not shining with good works to glorify God. What parable have we studied that teaches
us how to shine with good works for God? Parable of talents.

Matt 25:14-30

What happened to the foolish virgins when they tried to enter the wedding feast?

Matt 25:12

He said he did not know them.

Who do the wise and foolish virgins represent in context of the end of time?

Matt 25:31-46

The wise virgins = those who shine the light of good works in a sin darkened world. They shine
their good works by helping others. From the parable of the 10 virgins, we know that this light is
powered by the oil, which represents the Holy Spirit.

The foolish virgins = Those who in time of delay lost their faith and failed to allow the Holy Spirit to
power their life. It’s only by being baptized by the Spirit as well as of water that we can be saved.
Baptism of the spirit shows itself in a converted life that sacrifices self and shines the light of good
works. As a result, they show no light = good works, during the time of darkness.

http://breachrepairers.webs.com/ 306
CONCLUSION: If we are to see Jesus at the end of time, the wise Christians must have that oil, which
is the Holy Spirit, which will sanctify us, transforming us so that our light will shine the light from
the Sun of righteousness. This will allow Jesus to recognize us when He comes for His bride.

25:2 And five of them were wise, and five were foolish.

25:3 They that were foolish took their lamps, and took no oil with them:

25:4 But the wise took oil in their vessels with their lamps.

25:5 While the bridegroom tarried, they all slumbered and slept.

25:6 And at midnight there was a cry made, Behold, the bridegroom cometh; go ye out to meet him.

25:7 Then all those virgins arose, and trimmed their lamps.

25:8 And the foolish said unto the wise, Give us of your oil; for our lamps are gone out.

25:9 But the wise answered, saying, Not so; lest there be not enough for us and you: but go ye rather to
them that sell, and buy for yourselves.

25:10 And while they went to buy, the bridegroom came; and they that were ready went in with him to
the marriage: and the door was shut.

25:11 Afterward came also the other virgins, saying, Lord, Lord, open to us.

25:12 But he answered and said, Verily I say unto you, I know you not.

25:13 Watch therefore, for ye know neither the day nor the hour wherein the Son of man cometh.

25:14 For the kingdom of heaven is as a man travelling into a far country, who called his own servants,
and delivered unto them his goods.

What was the context of why this parable was given? See chapter before. Note: This parable is
a continuation of Jesus message to His disciples in Matt 24. Matt 25 is a continuation of Matt 24.
Matt 24 is Jesus’ last instruction to His disciples before His death about the end of the world.
Therefore, this parable is given in relation to the time of the end.

Who does the man going to a far country symbolize in this parable? Note: Who is mentioned
returning later in the parable?

Matt 25:19, 31

Jesus.

When did the ‘certain man’ leave to go to a far country?

Acts 1:9

Rephrase question: When did Jesus leave? Jesus left this earth when He ascended into heaven, after
His resurrection.

Who do the servants represent?

http://breachrepairers.webs.com/ 307
John 12:26

Servants = followers of Christ.

Why is the concept of servants used? What lessons can we learn from this?

1 Cor 6:20

A follower of Christ is not his own. We have been bought with a price. All of Christ’s followers have
been bought with a price.

1 Pet 1:18-19

What price? The blood of Christ. His life. The prince of heaven, Heaven’s precious Son was
sacrificed.

2 Cor 5:15

Who did Jesus die for? Everyone. That means, that everybody, believers or unbelievers, are the
Lord’s property. But in this parable, only those who accept Christ are represented as His servants.

What does the man going to a far country do before he leaves? What does this symbolize?

Matt 25:14-15

He leaves his goods with his servants. Goods = talents.

Eph 4:8

Left behind gifts = apostles, pastors, etc.

John 16:7

When Jesus left, He left behind the Comforter, the Holy Spirit. What are the gifts of the Spirit?

1 Cor 12:8-11

Talents represent gifts of the Spirit.

How are the talents distributed? What does this mean?

Matt 25:15

According to his several ability.

“several” means one’s own. Gifts are given according to man’s ability to use them and handle them.
Some servants are more capable to handle larger responsibilities. APPLICATION: Sometimes we are
not as talented because God knows our capacity to use them. This does not mean we are not as
blessed as others. With more talents come more responsibility. But at the same time, the one with
just ONE talent is expected to use the little that he has as well.

http://breachrepairers.webs.com/ 308
According to the parable, how are talents multiplied? Discuss.

Matt. 25:16

By trading them. By putting them to work. What does it mean to hide your talent? Discuss. To
not use them, having them sit idle. To knowingly have it, but intentionally hide it.

What was the servant’s excuse for hiding his talent?

Matt 25:24

He had a false perception of who God really was:

“hard” – difficult God, harsh, judgmental.

“reaping where thou hast not sown and gathering where thou has not strewn” – asking
impossible things. The servant believes that he has been given an impossible task.

What is the real reason that is pointed out by the Lord to why this servant hid his talent?

Matt 25:26

Lord’s perception = “wicked” and “slothful.” Slothful means indolent, lazy, wanting to avoid activity.
He was lazy, not afraid. Being afraid was just an excuse.

In the context of this parable, what is an unprofitable servant? Discuss. He had a talent and he
heard the command of the Lord but never used it. Application: He was a Christian and was blessed
with at least one talent from God but he chose not to use it.

What is the end result of the unfaithful servant?

Matt 25:29-30, 41

Cast out into outer darkness, there shall be weeping and gnashing of teeth. This is symbolic of
eternal destruction in verse 41.

In the context of this parable, how do faithful servants use their talents?

Matt 25:34-40

To use the talents that God has given us to help those in need. Sometimes the talents that God gives
us we use for selfish purposes and not to help or bless others in winning them to the kingdom.

What lessons can we learn from what the servants did with the talents and the results?

Luke 16:10

Luke 12:48

It is not the amount of talents that you are given, but what you do with what is given you. If you

http://breachrepairers.webs.com/ 309
make an effort to use your talents, God will at the very least, reward you by doubling. God does not
give small increments. He gives double. God gives in abundance. All of God’s servants are to
improve on what they are given. Those who do nothing with their gifts but bury them will end up
with even less than what he started out with.

25:15 And unto one he gave five talents, to another two, and to another one; to every man according to
his several ability; and straightway took his journey.

25:16 Then he that had received the five talents went and traded with the same, and made them other
five talents.

25:17 And likewise he that had received two, he also gained other two.

25:18 But he that had received one went and digged in the earth, and hid his lord's money.

25:19 After a long time the lord of those servants cometh, and reckoneth with them.

25:20 And so he that had received five talents came and brought other five talents, saying, Lord, thou
deliveredst unto me five talents: behold, I have gained beside them five talents more.

25:21 His lord said unto him, Well done, thou good and faithful servant: thou hast been faithful over a
few things, I will make thee ruler over many things: enter thou into the joy of thy lord.

25:22 He also that had received two talents came and said, Lord, thou deliveredst unto me two talents:
behold, I have gained two other talents beside them.

25:23 His lord said unto him, Well done, good and faithful servant; thou hast been faithful over a few
things, I will make thee ruler over many things: enter thou into the joy of thy lord.

25:24 Then he which had received the one talent came and said, Lord, I knew thee that thou art an hard
man, reaping where thou hast not sown, and gathering where thou hast not strawed:

25:25 And I was afraid, and went and hid thy talent in the earth: lo, there thou hast that is thine.

25:26 His lord answered and said unto him, Thou wicked and slothful servant, thou knewest that I reap
where I sowed not, and gather where I have not strawed:

25:27 Thou oughtest therefore to have put my money to the exchangers, and then at my coming I should
have received mine own with usury.

25:28 Take therefore the talent from him, and give it unto him which hath ten talents.

25:29 For unto every one that hath shall be given, and he shall have abundance: but from him that hath
not shall be taken away even that which he hath.

25:30 And cast ye the unprofitable servant into outer darkness: there shall be weeping and gnashing of
teeth.

25:31 When the Son of man shall come in his glory, and all the holy angels with him, then shall he sit
upon the throne of his glory:

25:32 And before him shall be gathered all nations: and he shall separate them one from another, as a
shepherd divideth his sheep from the goats:

http://breachrepairers.webs.com/ 310
25:33 And he shall set the sheep on his right hand, but the goats on the left.

25:34 Then shall the King say unto them on his right hand, Come, ye blessed of my Father, inherit the
kingdom prepared for you from the foundation of the world:

25:35 For I was an hungred, and ye gave me meat: I was thirsty, and ye gave me drink: I was a stranger,
and ye took me in:

25:36 Naked, and ye clothed me: I was sick, and ye visited me: I was in prison, and ye came unto me.

25:37 Then shall the righteous answer him, saying, Lord, when saw we thee an hungred, and fed thee? or
thirsty, and gave thee drink?

25:38 When saw we thee a stranger, and took thee in? or naked, and clothed thee?

25:39 Or when saw we thee sick, or in prison, and came unto thee?

25:40 And the King shall answer and say unto them, Verily I say unto you, Inasmuch as ye have done it
unto one of the least of these my brethren, ye have done it unto me.

25:41 Then shall he say also unto them on the left hand, Depart from me, ye cursed, into everlasting fire,
prepared for the devil and his angels:

25:42 For I was an hungred, and ye gave me no meat: I was thirsty, and ye gave me no drink:

25:43 I was a stranger, and ye took me not in: naked, and ye clothed me not: sick, and in prison, and ye
visited me not.

25:44 Then shall they also answer him, saying, Lord, when saw we thee an hungred, or athirst, or a
stranger, or naked, or sick, or in prison, and did not minister unto thee?

25:45 Then shall he answer them, saying, Verily I say unto you, Inasmuch as ye did it not to one of the
least of these, ye did it not to me.

25:46 And these shall go away into everlasting punishment: but the righteous into life eternal.

http://breachrepairers.webs.com/ 311
Chapter 26
 Simon’s feast (6-13)

26:1 And it came to pass, when Jesus had finished all these sayings, he said unto his disciples,

26:2 Ye know that after two days is the feast of the passover, and the Son of man is betrayed to be
crucified.

26:3 Then assembled together the chief priests, and the scribes, and the elders of the people, unto the
palace of the high priest, who was called Caiaphas,

26:4 And consulted that they might take Jesus by subtilty, and kill him.

26:5 But they said, Not on the feast day, lest there be an uproar among the people.

26:6 Now when Jesus was in Bethany, in the house of Simon the leper,

What do we know about Simon? He was a leper. He was also a Pharisee. He was one of the few who
made a public commitment to Christ during His ministry. He had a courage to make a stand for
Jesus. He was responsible for leading Mary into sin.

What has just happened just prior to this feast? Lazarus was just raised from the dead. The
Pharisees wanted to kill Lazarus and Jesus. Judas was considered the most educated amongst all of
the disciples.

DA 559 His professed sympathy for the poor deceived them, and his artful insinuation caused them
to look distrustfully upon Mary's devotion.

Speaking of Mary:

John 12:1-5

The woman is Mary. Spikenard costed 300 pence.

Matt 20:2

People were paid a penny per day. This gives us an idea of how much this perfume cost. It was
equivalent to spending one years salary.

26:7 There came unto him a woman having an alabaster box of very precious ointment, and poured it on
his head, as he sat at meat.

26:8 But when his disciples saw it, they had indignation, saying, To what purpose is this waste?

One person who was instigating this statement was Judas. EGW tells us that he wanted the money
to go into the bag so he could use it for himself.

Luke 7:38

This account gives us this idea that she was crying. But it was a custom back then. How was she able
to get this ointment out of this alabaster box? In order for that scent to be noticed, the box had to be

http://breachrepairers.webs.com/ 312
broken. In order for the fragrance of His life to fill this world, His body had to be broken. We also
have to fall on the rock and be broken.

26:9 For this ointment might have been sold for much, and given to the poor.

26:10 When Jesus understood it, he said unto them, Why trouble ye the woman? for she hath wrought a
good work upon me.

26:11 For ye have the poor always with you; but me ye have not always.

26:12 For in that she hath poured this ointment on my body, she did it for my burial.

What was the purpose for which Mary gave the ointment? It was in favor for Him to ascend to the
throne, but He said that it was for His burial.

DA 560 She feared that her sister would reproach her for extravagance. The Master, too, might think
her improvident. Without apology or excuse she was about to shrink away, when the voice of her
Lord was heard, "Let her alone; why trouble ye her?" He saw that she was embarrassed and
distressed. He knew that in this act of service she had expressed her gratitude for the forgiveness of
her sins, and He brought relief to her mind. Lifting His voice above the murmur of criticism, He said,
"She hath wrought a good work on Me. For ye have the poor with you always, and whensoever ye will
ye may do them good: but Me ye have not always. She hath done what she could: she is come
aforehand to anoint My body to the burying.”

She was being criticized for an act out of love. The character of Judas:

DA 563 But the look which Jesus cast upon Judas convinced him that the Saviour penetrated his
hypocrisy, and read his base, contemptible character. And in commending Mary's action, which had
been so severely condemned, Christ had rebuked Judas. Prior to this, the Saviour had never given him
a direct rebuke. Now the reproof rankled in his heart. He determined to be revenged. From the
supper he went directly to the palace of the high priest, where he found the council assembled, and
he offered to betray Jesus into their hands.

Luke 7:40-50

Out of these two in the parable, Simon was forgiven more.

DA 566 Simon had led into sin the woman he now despised. She had been deeply wronged by him.
By the two debtors of the parable, Simon and the woman were represented. Jesus did not design to
teach that different degrees of obligation should be felt by the two persons, for each owed a debt of
gratitude that never could be repaid. But Simon felt himself more righteous than Mary, and Jesus
desired him to see how great his guilt really was. He would show him that his sin was greater than
hers, as much greater as a debt of five hundred pence exceeds a debt of fifty pence.

DA 566 And when they had nothing to pay, he frankly forgave them both. Tell Me therefore, which of
them will love him most? Simon answered and said, I suppose that he, to whom he forgave most. And
He said unto him, Thou hast rightly judged.

If you look back and realize how great a sinner you really were, you sense the gratitude of what God
had really done for you.

DA 564 Christ values acts of heartfelt courtesy. When anyone did Him a favor, with heavenly
politeness He blessed the actor. He did not refuse the simplest flower plucked by the hand of a child,
and offered to Him in love. He accepted the offerings of children, and blessed the givers, inscribing

http://breachrepairers.webs.com/ 313
their names in the book of life.

DA 563 Looking into the future, the Saviour spoke with certainty concerning His gospel. It was to be
preached throughout the world. And as far as the gospel extended, Mary's gift would shed its
fragrance, and hearts would be blessed through her unstudied act. Kingdoms would rise and fall; the
names of monarchs and conquerors would be forgotten; but this woman's deed would be
immortalized upon the pages of sacred history. Until time should be no more, that broken alabaster
box would tell the story of the abundant love of God for a fallen race.

DA 568 Jesus knows the circumstances of every soul. You may say, I am sinful, very sinful. You may
be; but the worse you are, the more you need Jesus. He turns no weeping, contrite one away. He does
not tell to any all that He might reveal, but He bids every trembling soul take courage. Freely will He
pardon all who come to Him for forgiveness and restoration.

26:13 Verily I say unto you, Wheresoever this gospel shall be preached in the whole world, there shall
also this, that this woman hath done, be told for a memorial of her.

26:14 Then one of the twelve, called Judas Iscariot, went unto the chief priests,

26:15 And said unto them, What will ye give me, and I will deliver him unto you? And they covenanted
with him for thirty pieces of silver.

Thirty pieces of silver was the price of a slave. Avarice and greed brought him to sell his master for
30 pieces of silver. It was just a progression of events starting from stealing money. Then he starts
criticizing people for using money unwisely. Ultimately, he sells his Master.

26:16 And from that time he sought opportunity to betray him.

26:17 Now the first day of the feast of unleavened bread the disciples came to Jesus, saying unto him,
Where wilt thou that we prepare for thee to eat the passover?

26:18 And he said, Go into the city to such a man, and say unto him, The Master saith, My time is at hand;
I will keep the passover at thy house with my disciples.

26:19 And the disciples did as Jesus had appointed them; and they made ready the passover.

26:20 Now when the even was come, he sat down with the twelve.

26:21 And as they did eat, he said, Verily I say unto you, that one of you shall betray me.

26:22 And they were exceeding sorrowful, and began every one of them to say unto him, Lord, is it I?

26:23 And he answered and said, He that dippeth his hand with me in the dish, the same shall betray me.

26:24 The Son of man goeth as it is written of him: but woe unto that man by whom the Son of man is
betrayed! it had been good for that man if he had not been born.

26:25 Then Judas, which betrayed him, answered and said, Master, is it I? He said unto him, Thou hast
said.

26:26 And as they were eating, Jesus took bread, and blessed it, and brake it, and gave it to the disciples,
and said, Take, eat; this is my body.

26:27 And he took the cup, and gave thanks, and gave it to them, saying, Drink ye all of it;

http://breachrepairers.webs.com/ 314
26:28 For this is my blood of the new testament, which is shed for many for the remission of sins.

26:29 But I say unto you, I will not drink henceforth of this fruit of the vine, until that day when I drink it
new with you in my Father's kingdom.

26:30 And when they had sung an hymn, they went out into the mount of Olives.

26:31 Then saith Jesus unto them, All ye shall be offended because of me this night: for it is written, I will
smite the shepherd, and the sheep of the flock shall be scattered abroad.

26:32 But after I am risen again, I will go before you into Galilee.

26:33 Peter answered and said unto him, Though all men shall be offended because of thee, yet will I
never be offended.

How did Peter react when Jesus said that He would be betrayed? What was his mistake?

Matt 26:31-34

He was so sure of himself that he would stay and die with Jesus. Sometimes we can be as sure of our
faith and zealousness for Jesus as well. We need to be careful of our confidence in ourselves. We
need to learn to distrust ourselves rather than be so confident. MISTAKE 1: Peter’s 1st mistake was
to be self confident when he should have put his confidence in Christ. LESSON: Don’t be confident in
self. Learn to trust only in Jesus and God’s guidance.

What was Peter’s mistake in the garden of Gethsemane?

Matt 26:40

Peter was sleeping rather than praying. MISTAKE 2: Peter did not pray, but fell asleep on Christ
instead. LESSON: When we forget to pray, we will inevitably fall asleep in our Christian walk.

How did Peter react when a mob came to take Jesus? What was his mistake?

John 18:10-11.

Peter tried to kill one of the members of the mob. In fact, the servant of the high priest. Question:
Was Peter just trying to cut off the ear? Do you think he was so good a swordsman as to try to scare
everyone away by cutting off an ear? I don’t think so. Most likely he was trying to kill the servant
and aim for the neck. However he missed and Jesus rebuked him. MISTAKE 3: Tried to defend
Christ with impulsive violence. LESSON: Do not try to defend Christ or God in a moment of impulse
and revenge. Christ does not expect us to react in violence for His defense. Christ’s response is meek
and lowly.

After Jesus was taken away, what did Peter do? What was his mistake?

Luke 22:54

Peter followed afar off. MISTAKE 4: Peter followed Christ from a distance. LESSON: When we follow
Jesus afar off, we are destined to deny Him. What should Peter have been doing? Following Jesus
closely, by His side.

http://breachrepairers.webs.com/ 315
Where was Peter as the trial began?

Matt 26:58

Sitting inside the hall to see the trial.

What did Peter do after a maid recognized him?

Mark 14:66-68

John 18:17-18

Peter denied Christ the first time. What else did Peter do? Moved out of the hall outside to the porch
and further away from Christ. He warmed himself in front of “a fire of coals” with the servants and
officers. MISTAKE 5: Denied Christ and moved further away from Him. LESSON: When we deny
Christ, we instinctively move further away from Him.

Why did Peter warm himself with the servants and officers? He wanted to make himself look
like he was one of those people who brought Jesus to the court. MISTAKE 6: Tried to hide himself in
the wrong crowd, to disguise his relationship with Christ. LESSON: Once we start to deny Christ and
move away from Him, we will next try to hide ourselves in the wrong crowd.

How was Peter’s behavior after each time he denied Christ?

Matt 26:70-74

Peter denied Christ stronger and stronger to the point where he was cursing. That is usually the
trend when we get used to denying Christ. After a while we not only deny Him, but we do it with
cursing and swearing.

Who did Peter see right after he denied Jesus?

Luke 22:60-61

Peter saw Jesus.

What did Peter hear right after he denied Jesus?

Matt 26:74

He heard the cock crewing. QUESTION: When does a cock usually crow? In the morning hours.

How did Peter respond to the cock crowing?

Matt 26:75

He remembered the words of Jesus and this made him weep bitterly.

In summary, what can we learn from Peter’s mistakes leading up to his fall? How do you

http://breachrepairers.webs.com/ 316
think Christ felt?

Luke 22:31-32

Self confidence, self reliance. Lack of prayer, falling asleep to Christ. Impulsive and violent. Followed
Christ from a distance. Denied Christ and moved further away. Tried to hide himself in the wrong
crowd. Denial for Christ became worse and worse to the point of cursing and swearing.

NOTE: Christ knew Peter was going to fall and had prayed for him already (Luke 22:31-32).
Likewise, Christ knows when we will fall and is praying for our conversion. When Christ looked at
Peter, his heart must have hurt, but He knew that Peter would make a change. Let us make sure that
we don’t continue to make the same mistakes as Peter.

26:34 Jesus said unto him, Verily I say unto thee, That this night, before the cock crow, thou shalt deny
me thrice.

26:35 Peter said unto him, Though I should die with thee, yet will I not deny thee. Likewise also said all
the disciples.

26:36 Then cometh Jesus with them unto a place called Gethsemane, and saith unto the disciples, Sit ye
here, while I go and pray yonder.

26:37 And he took with him Peter and the two sons of Zebedee, and began to be sorrowful and very
heavy.

26:38 Then saith he unto them, My soul is exceeding sorrowful, even unto death: tarry ye here, and
watch with me.

26:39 And he went a little further, and fell on his face, and prayed, saying, O my Father, if it be possible,
let this cup pass from me: nevertheless not as I will, but as thou wilt.

26:40 And he cometh unto the disciples, and findeth them asleep, and saith unto Peter, What, could ye
not watch with me one hour?

26:41 Watch and pray, that ye enter not into temptation: the spirit indeed is willing, but the flesh is
weak.

26:42 He went away again the second time, and prayed, saying, O my Father, if this cup may not pass
away from me, except I drink it, thy will be done.

26:43 And he came and found them asleep again: for their eyes were heavy.

26:44 And he left them, and went away again, and prayed the third time, saying the same words.

26:45 Then cometh he to his disciples, and saith unto them, Sleep on now, and take your rest: behold, the
hour is at hand, and the Son of man is betrayed into the hands of sinners.

26:46 Rise, let us be going: behold, he is at hand that doth betray me.

26:47 And while he yet spake, lo, Judas, one of the twelve, came, and with him a great multitude with
swords and staves, from the chief priests and elders of the people.

26:48 Now he that betrayed him gave them a sign, saying, Whomsoever I shall kiss, that same is he: hold
him fast.

http://breachrepairers.webs.com/ 317
26:49 And forthwith he came to Jesus, and said, Hail, master; and kissed him.

26:50 And Jesus said unto him, Friend, wherefore art thou come? Then came they, and laid hands on
Jesus, and took him.

26:51 And, behold, one of them which were with Jesus stretched out his hand, and drew his sword, and
struck a servant of the high priest's, and smote off his ear.

26:52 Then said Jesus unto him, Put up again thy sword into his place: for all they that take the sword
shall perish with the sword.

26:53 Thinkest thou that I cannot now pray to my Father, and he shall presently give me more than
twelve legions of angels?

26:54 But how then shall the scriptures be fulfilled, that thus it must be?

26:55 In that same hour said Jesus to the multitudes, Are ye come out as against a thief with swords and
staves for to take me? I sat daily with you teaching in the temple, and ye laid no hold on me.

26:56 But all this was done, that the scriptures of the prophets might be fulfilled. Then all the disciples
forsook him, and fled.

26:57 And they that had laid hold on Jesus led him away to Caiaphas the high priest, where the scribes
and the elders were assembled.

26:58 But Peter followed him afar off unto the high priest's palace, and went in, and sat with the
servants, to see the end.

26:59 Now the chief priests, and elders, and all the council, sought false witness against Jesus, to put him
to death;

26:60 But found none: yea, though many false witnesses came, yet found they none. At the last came two
false witnesses,

26:61 And said, This fellow said, I am able to destroy the temple of God, and to build it in three days.

26:62 And the high priest arose, and said unto him, Answerest thou nothing? what is it which these
witness against thee?

26:63 But Jesus held his peace. And the high priest answered and said unto him, I adjure thee by the
living God, that thou tell us whether thou be the Christ, the Son of God.

26:64 Jesus saith unto him, Thou hast said: nevertheless I say unto you, Hereafter shall ye see the Son of
man sitting on the right hand of power, and coming in the clouds of heaven.

26:65 Then the high priest rent his clothes, saying, He hath spoken blasphemy; what further need have
we of witnesses? behold, now ye have heard his blasphemy.

26:66 What think ye? They answered and said, He is guilty of death.

26:67 Then did they spit in his face, and buffeted him; and others smote him with the palms of their
hands,

26:68 Saying, Prophesy unto us, thou Christ, Who is he that smote thee?

http://breachrepairers.webs.com/ 318
26:69 Now Peter sat without in the palace: and a damsel came unto him, saying, Thou also wast with
Jesus of Galilee.

26:70 But he denied before them all, saying, I know not what thou sayest.

It is important to see how Peter’s experience relates to us.

DA 710 In the court a fire had been kindled; for it was the coldest hour of the night, being just before
the dawn. A company drew about the fire, and Peter presumptuously took his place with them. He
did not wish to be recognized as a disciple of Jesus. By mingling carelessly with the crowd, he hoped
to be taken for one of those who had brought Jesus to the hall.

John was in the judgment hall with Jesus. Peter tried to act as one of the crowd.

DA 712 In order to conceal his true feelings, he endeavored to join with the persecutors of Jesus in
their untimely jests. But his appearance was unnatural. He was acting a lie, and while seeking to talk
unconcernedly he could not restrain expressions of indignation at the abuse heaped upon his
Master.” –

Here is the three ways that Peter denied Jesus:

DA 710 "Art not thou also one of this Man's disciples?" Peter was startled and confused; the eyes of
the company instantly fastened upon him. He pretended not to understand her; but she was
persistent, and said to those around her that this man was with Jesus. Peter felt compelled to answer,
and said angrily, "Woman, I know Him not." This was the first denial, and immediately the cock crew.
O Peter, so soon ashamed of thy Master! so soon to deny thy Lord!

He was trying to act like one of the wolves.

DA 712 Attention was called to him the second time, and he was again charged with being a follower
of Jesus. He now declared with an oath, "I do not know the Man."

Christ’s disciples were known for their honesty and clarity of speech.

DA 712 An hour had passed, when one of the servants of the high priest, being a near kinsman of the
man whose ear Peter had cut off, asked him, "Did not I see thee in the garden with Him?" "Surely thou
art one of them: for thou art a Galilean, and thy speech agreeth thereto." At this Peter flew into a rage.
The disciples of Jesus were noted for the purity of their language, and in order fully to deceive his
questioners, and justify his assumed character, Peter now denied his Master with cursing and
swearing.

Peter denied Christ by what he said. Today there are two ways we can deny Christ: Through our
actions. And through our words.

DA 712 While the degrading oaths were fresh upon Peter's lips, and the shrill crowing of the cock
was still ringing in his ears, the Saviour turned from the frowning judges, and looked full upon His
poor disciple. At the same time Peter's eyes were drawn to his Master. In that gentle countenance he
read deep pity and sorrow, but there was no anger there.

If Jesus had refused Judas’ kiss, it would have shown us that we could justifiably repel those who
betray us. Why did Judas have to kiss Jesus? It was unclear which one Jesus was because all the
disciples were healing and teaching and preaching.

http://breachrepairers.webs.com/ 319
DA 714 Then came the third scene of abuse and mockery, worse even than that received from the
ignorant rabble. In the very presence of the priests and rulers, and with their sanction, this took
place. Every feeling of sympathy or humanity had gone out of their hearts. If their arguments were
weak, and failed to silence His voice, they had other weapons, such as in all ages have been used to
silence heretics,--suffering, and violence, and death.

DA 715 The angels of God faithfully recorded every insulting look, word, and act against their
beloved Commander. One day the base men who scorned and spat upon the calm, pale face of Christ
will look upon it in its glory, shining brighter than the sun.

26:71 And when he was gone out into the porch, another maid saw him, and said unto them that were
there, This fellow was also with Jesus of Nazareth.

26:72 And again he denied with an oath, I do not know the man.

26:73 And after a while came unto him they that stood by, and said to Peter, Surely thou also art one of
them; for thy speech bewrayeth thee.

26:74 Then began he to curse and to swear, saying, I know not the man. And immediately the cock crew.

26:75 And Peter remembered the word of Jesus, which said unto him, Before the cock crow, thou shalt
deny me thrice. And he went out, and wept bitterly.

http://breachrepairers.webs.com/ 320
Chapter 27
27:1 When the morning was come, all the chief priests and elders of the people took counsel against
Jesus to put him to death:

27:2 And when they had bound him, they led him away, and delivered him to Pontius Pilate the governor.

What did the priests and elders do and what was their intention? Took counsel against Jesus to
put him to death. Bound Jesus and carried him to away to Pilate.

Who is Pilate? Governor.

What did the priests accuse Jesus of?

Luke 23:2, 5

Perverting the nation. Forbidding to give tribute unto Caesar. Saying that he himself was a king.
Stirring up the people

What was Pilate’s reply? And what did he do after the priests pressured him more? What can
we learn from Pilate’s first mistake?

Luke 23:4-7

Pilate told the priests he found no fault with Jesus. After further pressure, Pilate tried to pass the
problem on to Herod. Application: Pilate sent Jesus away even though he found no fault in Him
because of the pressure of the people. Sometimes we tend to reject Jesus as well even though we
believe but because of peer pressure, we reject Him.

27:3 Then Judas, which had betrayed him, when he saw that he was condemned, repented himself, and
brought again the thirty pieces of silver to the chief priests and elders,

27:4 Saying, I have sinned in that I have betrayed the innocent blood. And they said, What is that to us?
see thou to that.

27:5 And he cast down the pieces of silver in the temple, and departed, and went and hanged himself.

27:6 And the chief priests took the silver pieces, and said, It is not lawful for to put them into the
treasury, because it is the price of blood.

27:7 And they took counsel, and bought with them the potter's field, to bury strangers in.

27:8 Wherefore that field was called, The field of blood, unto this day.

27:9 Then was fulfilled that which was spoken by Jeremy the prophet, saying, And they took the thirty
pieces of silver, the price of him that was valued, whom they of the children of Israel did value;

27:10 And gave them for the potter's field, as the Lord appointed me.

27:11 And Jesus stood before the governor: and the governor asked him, saying, Art thou the King of the
Jews? And Jesus said unto him, Thou sayest.

http://breachrepairers.webs.com/ 321
27:12 And when he was accused of the chief priests and elders, he answered nothing.

27:13 Then said Pilate unto him, Hearest thou not how many things they witness against thee?

27:14 And he answered him to never a word; insomuch that the governor marvelled greatly.

How did Jesus respond differently to Pilate’s two questions? What is the difference between
the two questions? What can we learn from the question Jesus chose to answer?

Matt 27:11-14

In verse 11 Pilate asks Jesus to identify Himself and He responds. In verses 12 to 14 Pilate asks
Jesus to respond to accusations and He chooses not to respond. Application: When Jesus was asked
concerning His identity, He answered. But when it comes to meeting the accusations, he did not say
anything. Question: According to Jesus’ example, when is the right time for us to answer? When we
are asked to identify ourselves as Christians. When shouldn’t we answer? When we are being
accused falsely. Too often Christians are tempted into fighting back false accusations and feel the
need to argue back, but instead, we should follow Christ’s example.

27:15 Now at that feast the governor was wont to release unto the people a prisoner, whom they would.

27:16 And they had then a notable prisoner, called Barabbas.

27:17 Therefore when they were gathered together, Pilate said unto them, Whom will ye that I release
unto you? Barabbas, or Jesus which is called Christ?

27:18 For he knew that for envy they had delivered him.

What does Pilate try to do next in handling Christ? What lesson can we learn from his second
mistake?

Matt 27:15-18

Pilate tries to give the Jews a choice because he has no intention of killing Jesus. He tries to save
Jesus by offering somebody worse. Rather than making the tough decision, Pilate tries to remain
neutral. Application: As Christians we should be prepared to make the tough decisions for Jesus. We
need people who can stand up for JESUS. In tough situations, do people know where we stand? Or
do we allow the crowd to influence our decisions? Pilate’s mistake = He did not pick a side to stand,
he tried to remain neutral.

27:19 When he was set down on the judgment seat, his wife sent unto him, saying, Have thou nothing to
do with that just man: for I have suffered many things this day in a dream because of him.

What does Pilate’s wife tell him? What does she call Jesus? Why do you think she was given a
dream? Pilate’s wife warns him to have nothing to do with Jesus’ death because she was warned in
a dream. She calls Jesus a just man. God is trying to give Pilate one last chance to do the right thing.

Who is Barabbas? Who does he represent? How does this apply to us?

Matt 27:6

A notable prisoner.

http://breachrepairers.webs.com/ 322
Mark 15:7

A murderer.

John 18:4

A robber.

Application: Barabbas represents Lucifer – the rebel, criminal, father of sin and death (murderer),
thief (stole God’s people from Him). We have 2 choices in life – choose Jesus OR Lucifer. We also
choose Barabbas over Jesus when we choose Lucifer.

27:20 But the chief priests and elders persuaded the multitude that they should ask Barabbas, and
destroy Jesus.

27:21 The governor answered and said unto them, Whether of the twain will ye that I release unto you?
They said, Barabbas.

27:22 Pilate saith unto them, What shall I do then with Jesus which is called Christ? They all say unto
him, Let him be crucified.

27:23 And the governor said, Why, what evil hath he done? But they cried out the more, saying, Let him
be crucified.

How does Pilate make his important decision over Christ in his third mistake?

Matt 27:22-23

Pilate bases his decision on the people. Who do the people represent? The majority. Pilate decides
on Christ by yielding to the earthly majority. Application: Never make your decisions for Christ
based on the majority, peer pressure is usually always wrong.

27:24 When Pilate saw that he could prevail nothing, but that rather a tumult was made, he took water,
and washed his hands before the multitude, saying, I am innocent of the blood of this just person: see ye
to it.

27:25 Then answered all the people, and said, His blood be on us, and on our children.

27:26 Then released he Barabbas unto them: and when he had scourged Jesus, he delivered him to be
crucified.

What does Pilate try to do to detach himself from this decision? What was his fourth
mistake?

Matt 27:24-26

Pilate washes his hand. The people ask for Christ’s blood to be upon them. Pilate’s fourth mistake =
thinking that water can wash away guilt.

In summary, what are the mistakes that Pilate made and what important lessons can we
learn from them? Sent Jesus away. He did not pick a side to stand on, tried to be neutral. Yielded to

http://breachrepairers.webs.com/ 323
the earthly majority. Thought that water could wash away guilt. What can we learn for ourselves
from this? When Jesus comes to us, don’t send Him away, especially if we find no fault in Him. Give
him a chance and ignore the peer pressure. Don’t be neutral in following Jesus. Study His word and
obey the convictions of our heart. Don’t yield to the majority (even in church) because they are
usually wrong. Don’t think that by remaining impartial we can be innocent.

27:27 Then the soldiers of the governor took Jesus into the common hall, and gathered unto him the
whole band of soldiers.

27:28 And they stripped him, and put on him a scarlet robe.

27:29 And when they had platted a crown of thorns, they put it upon his head, and a reed in his right
hand: and they bowed the knee before him, and mocked him, saying, Hail, King of the Jews!

27:30 And they spit upon him, and took the reed, and smote him on the head.

27:31 And after that they had mocked him, they took the robe off from him, and put his own raiment on
him, and led him away to crucify him.

27:32 And as they came out, they found a man of Cyrene, Simon by name: him they compelled to bear his
cross.

27:33 And when they were come unto a place called Golgotha, that is to say, a place of a skull,

27:34 They gave him vinegar to drink mingled with gall: and when he had tasted thereof, he would not
drink.

27:35 And they crucified him, and parted his garments, casting lots: that it might be fulfilled which was
spoken by the prophet, They parted my garments among them, and upon my vesture did they cast lots.

27:36 And sitting down they watched him there;

27:37 And set up over his head his accusation written, THIS IS JESUS THE KING OF THE JEWS.

27:38 Then were there two thieves crucified with him, one on the right hand, and another on the left.

27:39 And they that passed by reviled him, wagging their heads,

27:40 And saying, Thou that destroyest the temple, and buildest it in three days, save thyself. If thou be
the Son of God, come down from the cross.

27:41 Likewise also the chief priests mocking him, with the scribes and elders, said,

27:42 He saved others; himself he cannot save. If he be the King of Israel, let him now come down from
the cross, and we will believe him.

27:43 He trusted in God; let him deliver him now, if he will have him: for he said, I am the Son of God.

27:44 The thieves also, which were crucified with him, cast the same in his teeth.

27:45 Now from the sixth hour there was darkness over all the land unto the ninth hour.

27:46 And about the ninth hour Jesus cried with a loud voice, saying, Eli, Eli, lama sabachthani? that is to
say, My God, my God, why hast thou forsaken me?

http://breachrepairers.webs.com/ 324
27:47 Some of them that stood there, when they heard that, said, This man calleth for Elias.

27:48 And straightway one of them ran, and took a spunge, and filled it with vinegar, and put it on a
reed, and gave him to drink.

27:49 The rest said, Let be, let us see whether Elias will come to save him.

27:50 Jesus, when he had cried again with a loud voice, yielded up the ghost.

27:51 And, behold, the veil of the temple was rent in twain from the top to the bottom; and the earth did
quake, and the rocks rent;

27:52 And the graves were opened; and many bodies of the saints which slept arose,

27:53 And came out of the graves after his resurrection, and went into the holy city, and appeared unto
many.

27:54 Now when the centurion, and they that were with him, watching Jesus, saw the earthquake, and
those things that were done, they feared greatly, saying, Truly this was the Son of God.

27:55 And many women were there beholding afar off, which followed Jesus from Galilee, ministering
unto him:

27:56 Among which was Mary Magdalene, and Mary the mother of James and Joses, and the mother of
Zebedee's children.

27:57 When the even was come, there came a rich man of Arimathaea, named Joseph, who also himself
was Jesus' disciple:

27:58 He went to Pilate, and begged the body of Jesus. Then Pilate commanded the body to be delivered.

27:59 And when Joseph had taken the body, he wrapped it in a clean linen cloth,

27:60 And laid it in his own new tomb, which he had hewn out in the rock: and he rolled a great stone to
the door of the sepulchre, and departed.

27:61 And there was Mary Magdalene, and the other Mary, sitting over against the sepulchre.

27:62 Now the next day, that followed the day of the preparation, the chief priests and Pharisees came
together unto Pilate,

27:63 Saying, Sir, we remember that that deceiver said, while he was yet alive, After three days I will rise
again.

27:64 Command therefore that the sepulchre be made sure until the third day, lest his disciples come by
night, and steal him away, and say unto the people, He is risen from the dead: so the last error shall be
worse than the first.

27:65 Pilate said unto them, Ye have a watch: go your way, make it as sure as ye can.

27:66 So they went, and made the sepulchre sure, sealing the stone, and setting a watch.

http://breachrepairers.webs.com/ 325
Chapter 28
28:1 In the end of the sabbath, as it began to dawn toward the first day of the week, came Mary
Magdalene and the other Mary to see the sepulchre.

28:2 And, behold, there was a great earthquake: for the angel of the Lord descended from heaven, and
came and rolled back the stone from the door, and sat upon it.

28:3 His countenance was like lightning, and his raiment white as snow:

28:4 And for fear of him the keepers did shake, and became as dead men.

28:5 And the angel answered and said unto the women, Fear not ye: for I know that ye seek Jesus, which
was crucified.

28:6 He is not here: for he is risen, as he said. Come, see the place where the Lord lay.

28:7 And go quickly, and tell his disciples that he is risen from the dead; and, behold, he goeth before you
into Galilee; there shall ye see him: lo, I have told you.

28:8 And they departed quickly from the sepulchre with fear and great joy; and did run to bring his
disciples word.

28:9 And as they went to tell his disciples, behold, Jesus met them, saying, All hail. And they came and
held him by the feet, and worshipped him.

28:10 Then said Jesus unto them, Be not afraid: go tell my brethren that they go into Galilee, and there
shall they see me.

28:11 Now when they were going, behold, some of the watch came into the city, and shewed unto the
chief priests all the things that were done.

28:12 And when they were assembled with the elders, and had taken counsel, they gave large money
unto the soldiers,

28:13 Saying, Say ye, His disciples came by night, and stole him away while we slept.

28:14 And if this come to the governor's ears, we will persuade him, and secure you.

28:15 So they took the money, and did as they were taught: and this saying is commonly reported among
the Jews until this day.

28:16 Then the eleven disciples went away into Galilee, into a mountain where Jesus had appointed
them.

28:17 And when they saw him, they worshipped him: but some doubted.

28:18 And Jesus came and spake unto them, saying, All power is given unto me in heaven and in earth.

28:19 Go ye therefore, and teach all nations, baptizing them in the name of the Father, and of the Son,
and of the Holy Ghost:

28:20 Teaching them to observe all things whatsoever I have commanded you: and, lo, I am with you
alway, even unto the end of the world. Amen.

http://breachrepairers.webs.com/ 326
Mark - Jesus Christ as a Servant
Introduction
Author: John Mark, cousin of Barnabas, translator for Peter. He gives the gospel account of Jesus
according to Peter. John was a common Hebrew name and Mark was a common gentile name
coming from the Latin Marcus. He was an intimate friend and associate of Peter Acts 12:11-17.
Peter addresses him as my son 1 Peter 5:13 1 & 2 Peter should be studied in connection with the
gospel of Mark.

Note: Memorization Outline

 Do we find Mark anywhere else in the Bible?


o Acts and Paul’s writings
 He was also a contemporary but he was not a disciple
 So how did he get the information to write Mark?
o He talked with one of the disciples – Peter
o He had information which no other gospel writer has
 His account is very short
 We believe that it was the first gospel written
o About 32 years after Jesus died – 65 AD
o Why would it take so long for them to write it?
 They were busy evangelizing, etc.

Author and Title


Widespread evidence from the early church fathers affirms that Peter passed on reports of the
words and deeds of Jesus to his attendant and writer, John Mark.Internal evidence also supports the
Patristic testimony that Peter stands behind Mark's Gospel. Mark's account is especially vivid when
recounting incidents involving Peter. It presents the weaknesses of Peter, as well as the disciples as
a whole, and omits praiseworthy or noticeable references to Peter reported in Matthew and Luke. It
has also been observed that there exists a certain structural proximity between Peter's Caesarea
speech (Acts 10:34–43) and the Gospel of Mark.

Date and Location


The external and internal data most convincingly point to Rome as the place of composition and a
date for Mark in the mid- to late-50s a.d..

Theme
The ultimate purpose and theme of Mark is to present and defend Jesus' universal call to discipleship.
Mark returns often to this theme, and as the narrative unfolds he categorizes his main audience as
either followers or opponents of Jesus. The outline demonstrates that Mark's central effort in
presenting and supporting this call is to narrate the identity and teaching of Jesus. This fact implies
that discipleship for Mark is essentially a relationship with Jesus, not merely following a certain
code of conduct. Fellowship with Jesus marks the heart of the disciple's life, and this fellowship
includes trusting him, confessing him, taking note of his conduct, following his teaching, and being
shaped by a relationship to him. Discipleship also means being prepared to face the kind of
rejection that Jesus faced.

http://breachrepairers.webs.com/ 327
Key Themes
1. Jesus seeks to correct messianic expectations and 1:25, 34, 44; 3:12; 4:10–12; 5:18–
misunderstandings. 19, 43; 8:30; 9:9
2. Jesus is man. 3:5; 4:38; 6:6; 7:34; 8:12, 33;
10:14; 11:12; 14:33–42
3. Jesus is the Son of God. 1:11; 3:11; 5:7; 8:38; 9:7; 12:6–8;
13:32; 14:36, 61; 15:39
4. Jesus is the Son of Man with all power and authority. 1:16–34; 2:3–12, 23–28; 3:11;
4:35–41; 6:45–52; 7:1–23; 10:1–12
5. Jesus as the Son of Man must suffer. 8:31; 10:45; 14:21, 36
6. Jesus is Lord. 2:28; 12:35–37; 14:62
7. Jesus calls his followers to imitate him in humble 8:34–38; 9:35–37; 10:35–45
service, self-denial, and suffering.
8. Jesus teaches on the kingdom of God, and implies that ch. 4; cf. 1:15; 9:1; 14:25; 15:43
God continues to call a people to himself.

Purpose, Occasion, and Background


Though Mark wrote from Rome, the Gospel of Mark was composed for the wider church as the
record of the apostolic testimony of Peter. Even during the early Patristic period, Gentile Christians
were frequently mentioned as the recipients of this Gospel. Mark addresses an audience that is
largely unfamiliar with Jewish customs. He intends to familiarize them with those customs, because
only then will they understand the coming of Jesus as the culmination of God's work with Israel and
the entire world.

Distinctive Features

The Setting of Mark


The events in the book of Mark take place almost entirely within the
vicinity of Palestine, an area extending roughly from Caesarea Philippi
in the north to Beersheba in the south. During this time it was ruled
by the Roman Empire. The book opens with Jesus' baptism by John
during the rule of Pontius Pilate and the tetrarchs Antipas and Philip,
and it closes with Jesus' death and resurrection about three years
later.

History of Salvation Summary


Mark tells of Jesus' coming to bring everlasting salvation, as prophesied in the OT, and to triumph
over sin and Satan. The ultimate fulfillment comes with his crucifixion and resurrection.

Outline
1. Introduction (1:1–15)
2. Demonstration of Jesus' Authority (1:16–8:26)
a. Jesus' early Galilean ministry (1:16–3:12)
b. Jesus' later Galilean ministry (3:13–6:6)
i. Calling of the Twelve (3:13–35)
ii. Parables (4:1–34)
iii. Nature miracle, exorcism, and healing (4:35–5:43)

http://breachrepairers.webs.com/ 328
iv. Rejection at Nazareth (6:1–6)
c. Work beyond Galilee (6:7–8:26)
i. Sending of the Twelve (6:7–13)
ii. Death of John the Baptist (6:14–56)
iii. Teachings on moral defilement (7:1–23)
iv. Opening to Gentiles (7:24–30)
v. Additional miracles in Decapolis and Bethsaida (7:31–8:26)
3. Testing Jesus' Authority in Suffering (8:27–16:8)
a. Journey to Jerusalem (8:27–10:52)
i. Peter's confession (8:27–33)
ii. Call to discipleship (8:34–9:1)
iii. Transfiguration and healing (9:2–29)
iv. Instruction on discipleship: putting others first (9:30–50)
v. Instruction on discipleship: divorce, wealth, humility (10:1–52)
b. Entering and judging Jerusalem (11:1–13:37)
i. Triumphal entry to Jerusalem (11:1–11)
ii. Jesus' judgment on religious leaders (11:12–12:44)
iii. Jesus and the coming judgment (13:1–37)
c. Death and resurrection in Jerusalem (14:1–16:8)
i. Betrayal (14:1–52)
ii. Trial (14:53–15:20)
iii. Crucifixion and resurrection (15:21–16:8)
iv. [“Longer ending of Mark” (16:9–20; see note)]

http://breachrepairers.webs.com/ 329
Chapter 1
Chapter Outline
 John prepares the way through Baptism (1-8)
 Baptism & Temptation of Jesus in the wilderness (9-13)
 Call of Peter, Andrew, James and John (14-20)
 Christ's authority over all diseases and demons (21-34)
o Unclean spirit in Synagogue and Peter's mother in law.
 Healing of the leper and Christ's fame spreads abroad (35-45)
Summary: Christ's has authority over all manner of sicknesses and diseases. From minor sicknesses
such as a fever to severe sicknesses leprosy. And all demons obey His voice.

Overview Verses 1:1-2 = It begins with the gospel message being taken to the world by the
messenger of God. Chap 16:19-20 = It ends with the gospel being taken to the world by the
messengers of God. And this gospel is incomplete; it will be complete with you and me. And it talks
about the gospel going to the whole world. The book of Mark stresses action, movement. One word
is repeated throughout ‘straightway’.

John prepares the way through Baptism (1-8)

1:1 The beginning of the gospel of Jesus Christ, the Son of God;

What can you tell me about the gospel from this verse? It is about Jesus Christ, who is Jesus? The
Christ, and who is the Christ? The Son of God. Gospel means good news, glad tidings. What are the
glad tidings? That Jesus Christ is the Son of God. Son of God automatically makes Him divine. Jesus
shows He is human.

1:2 As it is written in the prophets, Behold, I send my messenger before thy face, which shall prepare thy
way before thee.

“As it is written in the prophets” – What is written in the prophets, the Old Testament? The
gospel of Jesus Christ, the Son of God. (See Romans 1:1-3)

“my messenger” – This shows that he has a message. And this message is a part of the gospel, and
this message will prepare the way for Him to come the first time. How will he prepare the way? By
giving a message

Amos 3:7 Surely the Lord GOD will do nothing, but he revealeth his secret unto his servants the
prophets.

1:3 The voice of one crying in the wilderness, Prepare ye the way of the Lord, make his paths straight.

“make his paths straight” – This is the message; make his paths straight. According to eastern
culture this language was used to usher in a king. What does that mean? The true message of the
gospel is that Jesus is a man, God, and King.

Note: Mal 3:1 Messenger prepares the way for him to enter His temple. That was on this earth when

http://breachrepairers.webs.com/ 330
Jesus came the first time; for our time it was before He went into the MHP. This phrase also shows
that He was a Priest.

App: Before God comes, He first sends a message. And that means He must have a messenger. A
mouth piece. So it must follow that before He comes back the second time He must have a
messenger. Remember Mk 16:19, what is there message? Jesus, a man, who is He? Lord, divine, and
he sat on the right hand of God, King. Same message. Practically how do we make his paths straight?

1:4 John did baptize in the wilderness, and preach the baptism of repentance for the remission of sins.

It didn’t mention his birth, who his parents were, etc.

“baptism” – this is how we make his paths straight, this is the message. App: We should be
propelled to take the gospel to the world. If not, we need to re-examine if we are ready to make his
paths straight, if we need to be re-baptized.

1:5 And there went out unto him all the land of Judaea, and they of Jerusalem, and were all baptized of
him in the river of Jordan, confessing their sins.

Confession of sin

1:6 And John was clothed with camel's hair, and with a girdle of a skin about his loins; and he did eat
locusts and wild honey;

He wore simple clothing, the people were attracted to the message, and not the man. He pointed
them to the message.

1:7 And preached, saying, There cometh one mightier than I after me, the latchet of whose shoes I am not
worthy to stoop down and unloose.

He didn’t mention his name? Why? Because of what there expectations were. He pointed them to
the scriptures first, took attention off of himself.

1:8 I indeed have baptized you with water: but he shall baptize you with the Holy Ghost.

This is how He is mightier than Him. How do we make his paths straight, confession, repentance,
and baptism of the water and Holy Ghost.

Baptism & Temptation of Jesus in the wilderness (9-13)


1:9 And it came to pass in those days, that Jesus came from Nazareth of Galilee, and was baptized of John
in Jordan.

“Jesus came from Nazareth” – What did verse 1 say? Jesus Christ. Here He is called Jesus, why?
Because He was not yet the Christ.

1:10 And straightway coming up out of the water, he saw the heavens opened, and the Spirit like a dove
descending
upon him:

Why does He mention the Holy Spirit here? This is how He became the Christ. What does Christ
mean? The anointed one. The Messiah.

http://breachrepairers.webs.com/ 331
Dan 9:25 Messiah

Jn 1:41 We have found the Messias, which is, being interpreted, the Christ.” (the anointed margin)

Acts 10:38 How God anointed Jesus of Nazareth with the Holy Ghost and with power.

1:11 And there came a voice from heaven, saying, Thou art my beloved Son, in whom I am well pleased.

“my beloved Son”

Mark 1:9 Jesus

Mark 1:10 Christ

Mark 1:11 Son of God

What is the gospel? Jesus Christ the Son of God. Make the Messiah’s path, the kings path straight.
Verses 1-11. The gospel put succinctly.

1:12 And immediately the Spirit driveth him into the wilderness.

1:13 And he was there in the wilderness forty days, tempted of Satan; and was with the wild beasts; and
the angels ministered unto him.

Call of Peter, Andrew, James and John (14-20)


1:14 Now after that John was put in prison, Jesus came into Galilee, preaching the gospel of the kingdom
of God,

1:15 And saying, The time is fulfilled, and the kingdom of God is at hand: repent ye, and believe the
gospel.

1:16 Now as he walked by the sea of Galilee, he saw Simon and Andrew his brother casting a net into the
sea: for they were fishers.

1:17 And Jesus said unto them, Come ye after me, and I will make you to become fishers of men.

1:18 And straightway they forsook their nets, and followed him.

1:19 And when he had gone a little further thence, he saw James the son of Zebedee, and John his
brother, who also were in the ship mending their nets.

1:20 And straightway he called them: and they left their father Zebedee in the ship with the hired
servants, and went after him.

Christ's authority over all diseases and demons (21-34)


1:21 And they went into Capernaum; and straightway on the sabbath day he entered into the synagogue,
and taught.

1:22 And they were astonished at his doctrine: for he taught them as one that had authority, and not as
the scribes.

1:23 And there was in their synagogue a man with an unclean spirit; and he cried out,

http://breachrepairers.webs.com/ 332
1:24 Saying, Let us alone; what have we to do with thee, thou Jesus of Nazareth? art thou come to destroy
us? I know thee who thou art, the Holy One of God.

1:25 And Jesus rebuked him, saying, Hold thy peace, and come out of him.

1:26 And when the unclean spirit had torn him, and cried with a loud voice, he came out of him.

1:27 And they were all amazed, insomuch that they questioned among themselves, saying, What thing is
this? what new doctrine is this? for with authority commandeth he even the unclean spirits, and they do
obey him.

1:28 And immediately his fame spread abroad throughout all the region round about Galilee.

1:29 And forthwith, when they were come out of the synagogue, they entered into the house of Simon and
Andrew, with James and John.

1:30 But Simon's wife's mother lay sick of a fever, and anon they tell him of her.

1:31 And he came and took her by the hand, and lifted her up; and immediately the fever left her, and she
ministered unto them.

1:32 And at even, when the sun did set, they brought unto him all that were diseased, and them that
were possessed with devils.

1:33 And all the city was gathered together at the door.

1:34 And he healed many that were sick of divers diseases, and cast out many devils; and suffered not
the devils to speak, because they knew him.

Healing of the leper and Christ's fame spreads abroad (35-45)


1:35 And in the morning, rising up a great while before day, he went out, and departed into a solitary
place, and there prayed.

1:36 And Simon and they that were with him followed after him.

1:37 And when they had found him, they said unto him, All men seek for thee.

1:38 And he said unto them, Let us go into the next towns, that I may preach there also: for therefore
came I forth.

1:39 And he preached in their synagogues throughout all Galilee, and cast out devils.

1:40 And there came a leper to him, beseeching him, and kneeling down to him, and saying unto him, If
thou wilt, thou canst make me clean.

Who came to Jesus and who does he represent spiritually? A leper. Note: The leper came to
Jesus. Not Jesus went to him. The leper = a sinner = all of us. How can we make this comparison?
Rom 3:23 and 6:23 tells us that all have sinned and our wages is death. So just like the leper, we are
on a death sentence

How did the leper come to Jesus? What does this represent? The leper was kneeling to Jesus.
This represents prayer.

http://breachrepairers.webs.com/ 333
What did the leper ask for? What does this for us? To be made clean. It meAnswer for the sinner
to free from sin.

What does the word beseech mean? What can we learn? To beg, to implore, plead. When is the
last time that you pleaded, begged, implored God to cleanse you from your sins? Or do you just ask
it out of habit and without much feeling involved.

1:41 And Jesus, moved with compassion, put forth his hand, and touched him, and saith unto him, I will;
be thou clean.

How did Jesus respond? What application can we draw from this? Jesus reacted with
compassion. What application can we draw? When you pray, Jesus is filled with compassion when
you are asking for forgiveness. He is not a tyrant sitting there ready to punish you

What does Christ’s hand represent?

Isa 41:10

Righteousness. Why did Jesus touch the man? Many instances He heals without touching. Why did
He have to touch him? Have you been touched by Jesus?

What did Jesus say to the leper? I wilt be thou clean. According to verse 41, what is the will of
God? Compare with other texts as well.

Mark 1:41

1 Thess 4:3

John 17:17

Answer: For us sinners to be clean. For us to be clean = free from sin.

1:42 And as soon as he had spoken, immediately the leprosy departed from him, and he was cleansed.

How was the leper healed? What has healing power?

Mark 1:42

Eph 5:26-27

Answer: As soon as Jesus spoke. Jesus’ words = word of God.

How soon was the leper healed? What can we learn?

Answer: Immediately. In summary: As soon as we pray to Jesus, beseeching him to forgive us and
make us clean, Jesus has compassion, and touches us with his righteousness. Then he speaks
through His word to heal us. Immediately, Christ’s word has healing power and can make us clean.
We don’t have to wait days, weeks, or months. We can be healed through Christ’s righteousness and
we should believe that we can be healed as long as we plead with Christ.

http://breachrepairers.webs.com/ 334
1:43 And he straitly charged him, and forthwith sent him away;

1:44 And saith unto him, See thou say nothing to any man: but go thy way, shew thyself to the priest, and
offer for thy cleansing those things which Moses commanded, for a testimony unto them.

1:45 But he went out, and began to publish it much, and to blaze abroad the matter, insomuch that Jesus
could no more openly enter into the city, but was without in desert places: and they came to him from
every quarter.

http://breachrepairers.webs.com/ 335
Chapter 2 – Healing of Man with Palsy
Summary: This chapter shows His authority over sin, interacting with sinners, fasting, and the
Sabbath. He is totally restructuring the Jewish economy. Placing everything back in its proper
sphere in relation to the original purpose of God for His people.

Chapter Outline
 Healing of the man sick with palsy (1-12)
 The call and feast of Levi Matthew (13-17)
 The question on fasting (18-22)
 Plucking corn on the Sabbath (23-28)

Healing of the man sick with palsy (1-12)


2:1 And again he entered into Capernaum after some days; and it was noised that he was in the house.

2:2 And straightway many were gathered together, insomuch that there was no room to receive them,
no, not so much as about the door: and he preached the word unto them.

Where is Jesus and what is He doing?

Mark 2:1-2

Answer: Jesus was in a house, preaching. Application: Equivalent of conducting prayer meeting or
vespers – it is a worship or gathering of followers.

What type of people could be listening to Him and blocking the way for the man that was sick of the
palsy?

Mark 1:16-20

His disciples were listening to Him

Mark 1:21-22

Those that heard His teaching at church

Mark 1:32

Those that He healed physically

Mark 2:6

Scribes

APPLICATION: Mixture of church attendees and the sick.

2:3 And they come unto him, bringing one sick of the palsy, which was borne of four.

http://breachrepairers.webs.com/ 336
Who does the man sick of the palsy represent today?

Those who are too crippled by life or circumstances to come to Jesus. They need their friends’ help.

According Luke 5:19, Mark 2:4, how hard was it to bring this sick man before Jesus?
Remember, who was blocking the way? And how many people did it take to carry this sick
man?
They tried through the door, the window. Eventually they had to lift him up on the roof, break it and
then lower him from the roof. They went to a lot of trouble to get him to Jesus. They were being
blocked by the followers of Jesus.

APPLICATION: sometimes we can block others to Christ by our actions or attitudes. We can block
others even by wanting to hear a good sermon. Good motive – wanting to be spiritually fed, but
there are others more in need but because we are stronger, we block them. It took 4 people to bring
the sick man to Christ.

In this verse who does the four men represent that were carrying this man sick of the palsy?
Friends. Those with burden to see their friends come to Christ so that they can be healed physically
and spiritually.

What personal lesson can we learn in bringing souls to Christ from the four men? When a
person comes to church, they need to make at least four friends. It takes team work to bring a soul
to Christ. It takes a lot of effort to bring a soul to Christ.

2:4 And when they could not come nigh unto him for the press, they uncovered the roof where he was:
and when they had broken it up, they let down the bed wherein the sick of the palsy lay.

2:5 When Jesus saw their faith, he said unto the sick of the palsy, Son, thy sins be forgiven thee.

What was Jesus’ response to the man sick of the palsy?

Son, thy sins be forgiven thee.

Do you think that that was a response that he was seeking for? Discuss.

Many people that are burdened with the guilt of sin first want to know that they are accepted by
Christ and that their sins are forgiven them. The physical healing can sometimes be desired more,
but usually it is the mind that wants to have peace first. It is that conscience that wants to be void of
guilt and shame and transgression.

2:6 But there were certain of the scribes sitting there, and reasoning in their hearts,

2:7 Why doth this man thus speak blasphemies? who can forgive sins but God only?

Who do the scribes represent today? Discuss it in relation to their response.

Mark 2:6-7

Learned men. The best theologians from the most acclaimed universities. APPLICATION: Some
theologians, despite studying the truth, are not converted and will fight against God’s authority or
will. We must be careful not to become like that.

http://breachrepairers.webs.com/ 337
2:8 And immediately when Jesus perceived in his spirit that they so reasoned within themselves, he said
unto them, Why reason ye these things in your hearts?

2:9 Whether is it easier to say to the sick of the palsy, Thy sins be forgiven thee; or to say, Arise, and take
up thy bed, and walk?

Was it easier for Jesus to offer forgiveness or for Him to heal the man? Discuss.

It would have been easier for Jesus to heal Him. Forgiveness signified that He had to make a
provision for forgiveness.

1 Cor. 15:17

If He had to be resurrected, that means He had to die before that. So forgiveness of sins comes from
the death and resurrection of Christ.

Heb 9:22

He had to shed His blood that He could give us remission of sins which is forgiveness.

According to the story, whose faith was exercised that caused this man to be healed? It was
the faith of four friends and the palsy man that enabled him to be healed (Mark 2:5). However, more
importantly without the faith of friends, palsy man would not have reached Jesus.

What lessons can we learn from the four men and their faith? How did Jesus see their faith?
Discuss.

Mark 2:5

We must accompany our faith with works. It is our works that demonstrate what type of faith we
have. The people listening had faith, but it didn’t help them because they had no works. Their faith
was united in winning souls for Christ. It is when we are working for Christ for those crippled by sin
that men will become united in faith.

2:10 But that ye may know that the Son of man hath power on earth to forgive sins, (he saith to the sick
of the palsy,)

2:11 I say unto thee, Arise, and take up thy bed, and go thy way into thine house.

2:12 And immediately he arose, took up the bed, and went forth before them all; insomuch that they
were all amazed, and glorified God, saying, We never saw it on this fashion.

The call and feast of Levi Matthew (13-17)


2:13 And he went forth again by the sea side; and all the multitude resorted unto him, and he taught
them.

2:14 And as he passed by, he saw Levi the son of Alphaeus sitting at the receipt of custom, and said unto
him, Follow me. And he arose and followed him.

2:15 And it came to pass, that, as Jesus sat at meat in his house, many publicans and sinners sat also
together with Jesus and his disciples: for there were many, and they followed him.

http://breachrepairers.webs.com/ 338
2:16 And when the scribes and Pharisees saw him eat with publicans and sinners, they said unto his
disciples, How is it that he eateth and drinketh with publicans and sinners?

2:17 When Jesus heard it, he saith unto them, They that are whole have no need of the physician, but they
that are sick: I came not to call the righteous, but sinners to repentance.

The question on fasting (18-22)


2:18 And the disciples of John and of the Pharisees used to fast: and they come and say unto him, Why do
the disciples of John and of the Pharisees fast, but thy disciples fast not?

2:19 And Jesus said unto them, Can the children of the bridechamber fast, while the bridegroom is with
them? as long as they have the bridegroom with them, they cannot fast.

2:20 But the days will come, when the bridegroom shall be taken away from them, and then shall they
fast in those days.

2:21 No man also seweth a piece of new cloth on an old garment: else the new piece that filled it up
taketh away from the old, and the rent is made worse.

2:22 And no man putteth new wine into old bottles: else the new wine doth burst the bottles, and the
wine is spilled, and the bottles will be marred: but new wine must be put into new bottles.

Plucking corn on the Sabbath (23-28)


2:23 And it came to pass, that he went through the corn fields on the sabbath day; and his disciples
began, as they went, to pluck the ears of corn.

2:24 And the Pharisees said unto him, Behold, why do they on the sabbath day that which is not lawful?

2:25 And he said unto them, Have ye never read what David did, when he had need, and was an hungred,
he, and they that were with him?

2:26 How he went into the house of God in the days of Abiathar the high priest, and did eat the
shewbread, which is not lawful to eat but for the priests, and gave also to them which were with him?

2:27 And he said unto them, The sabbath was made for man, and not man for the sabbath:

“Sabbath was made for man” – It was made for the needs of man, for the benefit of man. If a man
is hungry on the Sabbath we are to feed him. Same if he is hungry, without water or clothes.

“not man for the Sabbath” – Man was not made to determine what should or shouldn’t be done on
the Sabbath. To set up his own restrictions and rules on the Sabbath.

2:28 Therefore the Son of man is Lord also of the sabbath.

Because I did not create you to determine what should be done on the Sabbath. The one who has
authority over the Sabbath will dictate and give an example of what should be done on the Sabbath.

http://breachrepairers.webs.com/ 339
Chapter 3
Chapter Outline
 Healing of the withered hand on the Sabbath (1-6)
 Multitude from seven regions gathers at the sea of Capernaum (7-12)
 Ordination of the twelve disciples (13-19)
 Who are my brethren? & Warning against the sin of blasphemy (20-35)

Healing of the withered hand on the Sabbath (1-6)


3:1 And he entered again into the synagogue; and there was a man there which had a withered hand.

3:2 And they watched him, whether he would heal him on the sabbath day; that they might accuse him.

3:3 And he saith unto the man which had the withered hand, Stand forth.

3:4 And he saith unto them, Is it lawful to do good on the sabbath days, or to do evil? to save life, or to
kill? But they held their peace.

3:5 And when he had looked round about on them with anger, being grieved for the hardness of their
hearts, he saith unto the man, Stretch forth thine hand. And he stretched it out: and his hand was
restored whole as the other.

3:6 And the Pharisees went forth, and straightway took counsel with the Herodians against him, how
they might destroy him.

Multitude from seven regions gathers at the sea of Capernaum (7-12)


3:7 But Jesus withdrew himself with his disciples to the sea: and a great multitude from Galilee followed
him, and from Judaea,

3:8 And from Jerusalem, and from Idumaea, and from beyond Jordan; and they about Tyre and Sidon, a
great multitude, when they had heard what great things he did, came unto him.

3:9 And he spake to his disciples, that a small ship should wait on him because of the multitude, lest they
should throng him.

3:10 For he had healed many; insomuch that they pressed upon him for to touch him, as many as had
plagues.

3:11 And unclean spirits, when they saw him, fell down before him, and cried, saying, Thou art the Son of
God.

3:12 And he straitly charged them that they should not make him known.

Ordination of the twelve disciples (13-19)


3:13 And he goeth up into a mountain, and calleth unto him whom he would: and they came unto him.

3:14 And he ordained twelve, that they should be with him, and that he might send them forth to preach,

3:15 And to have power to heal sicknesses, and to cast out devils:

http://breachrepairers.webs.com/ 340
3:16 And Simon he surnamed Peter;

3:17 And James the son of Zebedee, and John the brother of James; and he surnamed them Boanerges,
which is, The sons of thunder:

3:18 And Andrew, and Philip, and Bartholomew, and Matthew, and Thomas, and James the son of
Alphaeus, and Thaddaeus, and Simon the Canaanite,

3:19 And Judas Iscariot, which also betrayed him: and they went into an house.

Who are my brethren? & Warning against the sin of blasphemy (20-35)
3:20 And the multitude cometh together again, so that they could not so much as eat bread.

3:21 And when his friends heard of it, they went out to lay hold on him: for they said, He is beside himself.

3:22 And the scribes which came down from Jerusalem said, He hath Beelzebub, and by the prince of the
devils casteth he out devils.

3:23 And he called them unto him, and said unto them in parables, How can Satan cast out Satan?

3:24 And if a kingdom be divided against itself, that kingdom cannot stand.

3:25 And if a house be divided against itself, that house cannot stand.

3:26 And if Satan rise up against himself, and be divided, he cannot stand, but hath an end.

3:27 No man can enter into a strong man's house, and spoil his goods, except he will first bind the strong
man; and then he will spoil his house.

3:28 Verily I say unto you, All sins shall be forgiven unto the sons of men, and blasphemies wherewith
soever they shall blaspheme:

3:29 But he that shall blaspheme against the Holy Ghost hath never forgiveness, but is in danger of
eternal damnation:

3:30 Because they said, He hath an unclean spirit.

3:31 There came then his brethren and his mother, and, standing without, sent unto him, calling him.

3:32 And the multitude sat about him, and they said unto him, Behold, thy mother and thy brethren
without seek for thee.

3:33 And he answered them, saying, Who is my mother, or my brethren?

3:34 And he looked round about on them which sat about him, and said, Behold my mother and my
brethren!

3:35 For whosoever shall do the will of God, the same is my brother, and my sister, and mother.

http://breachrepairers.webs.com/ 341
Chapter 4 – Disciples in the Storm
Chapter Outline
 Parable of the four grounds (1-23)
 Parables of the full corn in the ear and the mustard seed (24-34)
 Let us pass over to the other side (35-41)

Parable of the four grounds (1-23)


4:1 And he began again to teach by the sea side: and there was gathered unto him a great multitude, so
that he entered into a ship, and sat in the sea; and the whole multitude was by the sea on the land.

4:2 And he taught them many things by parables, and said unto them in his doctrine,

4:3 Hearken; Behold, there went out a sower to sow:

4:4 And it came to pass, as he sowed, some fell by the way side, and the fowls of the air came and
devoured it up.

4:5 And some fell on stony ground, where it had not much earth; and immediately it sprang up, because
it had no depth of earth:

4:6 But when the sun was up, it was scorched; and because it had no root, it withered away.

4:7 And some fell among thorns, and the thorns grew up, and choked it, and it yielded no fruit.

4:8 And other fell on good ground, and did yield fruit that sprang up and increased; and brought forth,
some thirty, and some sixty, and some an hundred.

4:9 And he said unto them, He that hath ears to hear, let him hear.

4:10 And when he was alone, they that were about him with the twelve asked of him the parable.

4:11 And he said unto them, Unto you it is given to know the mystery of the kingdom of God: but unto
them that are without, all these things are done in parables:

4:12 That seeing they may see, and not perceive; and hearing they may hear, and not understand; lest at
any time they should be converted, and their sins should be forgiven them.

4:13 And he said unto them, Know ye not this parable? and how then will ye know all parables?

4:14 The sower soweth the word.

4:15 And these are they by the way side, where the word is sown; but when they have heard, Satan
cometh immediately, and taketh away the word that was sown in their hearts.

4:16 And these are they likewise which are sown on stony ground; who, when they have heard the word,
immediately receive it with gladness;

4:17 And have no root in themselves, and so endure but for a time: afterward, when affliction or
persecution ariseth for the word's sake, immediately they are offended.

http://breachrepairers.webs.com/ 342
4:18 And these are they which are sown among thorns; such as hear the word,

4:19 And the cares of this world, and the deceitfulness of riches, and the lusts of other things entering in,
choke the word, and it becometh unfruitful.

4:20 And these are they which are sown on good ground; such as hear the word, and receive it, and bring
forth fruit, some thirtyfold, some sixty, and some an hundred.
4:21 And he said unto them, Is a candle brought to be put under a bushel, or under a bed? and not to be
set on a candlestick?

4:22 For there is nothing hid, which shall not be manifested; neither was any thing kept secret, but that
it should come abroad.

4:23 If any man have ears to hear, let him hear.

Parables of the full corn in the ear and the mustard seed (24-34)
4:24 And he said unto them, Take heed what ye hear: with what measure ye mete, it shall be measured to
you: and unto you that hear shall more be given.

4:25 For he that hath, to him shall be given: and he that hath not, from him shall be taken even that
which he hath.

4:26 And he said, So is the kingdom of God, as if a man should cast seed into the ground;

4:27 And should sleep, and rise night and day, and the seed should spring and grow up, he knoweth not
how.

4:28 For the earth bringeth forth fruit of herself; first the blade, then the ear, after that the full corn in
the ear.

4:29 But when the fruit is brought forth, immediately he putteth in the sickle, because the harvest is
come.

4:30 And he said, Whereunto shall we liken the kingdom of God? or with what comparison shall we
compare it?

4:31 It is like a grain of mustard seed, which, when it is sown in the earth, is less than all the seeds that
be in the earth:

4:32 But when it is sown, it groweth up, and becometh greater than all herbs, and shooteth out great
branches; so that the fowls of the air may lodge under the shadow of it.

4:33 And with many such parables spake he the word unto them, as they were able to hear it.

4:34 But without a parable spake he not unto them: and when they were alone, he expounded all things
to his disciples.

Let us pass over to the other side (35-41)


4:35 And the same day, when the even was come, he saith unto them, Let us pass over unto the other side.

What parable had Jesus been teaching before this encounter? What was the lesson about?

Mark 4:3-8

http://breachrepairers.webs.com/ 343
Matt 13:20

Rom 10:17

Parable of the sower. This is about the different grounds and how they receive the seed = word
(Matt 13:20). The main theme is about faith, because Rom 10:17 tells us that faith comes by
hearing, hearing by the word of God. Jesus is about to follow up this parable with a practical
example on faith. This is what this encounter is about.

What did Jesus ask the disciples to do?

Mark 4:35

He asked them to pass over onto the other side.

4:36 And when they had sent away the multitude, they took him even as he was in the ship. And there
were also with him other little ships.

4:37 And there arose a great storm of wind, and the waves beat into the ship, so that it was now full.

What happened next?

There was a great storm. It beat against the boat and filled it up.

4:38 And he was in the hinder part of the ship, asleep on a pillow: and they awake him, and say unto him,
Master, carest thou not that we perish?

Where was Jesus, and what was he doing?

Mark 4:38

He was asleep in the hinder part of the ship and He was fast asleep.

How do the disciples react? What does it tell us about their attitude?

The disciples call out to Jesus and ask Him if He cares whether they perish. When someone says
something like that, what are they implying? That you don’t care. Why did the disciples think that?
Because they were in the middle of a storm and Jesus was fast asleep. They probably felt Jesus did
not do his part in keeping everyone alive.

In what state was the boat in when the disciples called Christ? What do you think the
disciples had already been doing before calling out to Christ and what lesson can we learn
from this?

Mark 4:37

The boat was already full (v37). This means that the disciples only came to Jesus when the boat was
full. As fishermen, what would be their natural instinct as they saw the boat filling up? They would
try to use a bucket to scoop the water out. This means that by the time the disciples approached
Jesus, they had already tried everything they humanly could. All that was left was to cry out to Jesus.
Many of us try to save ourselves, we try working our way to heaven, before we will surrender all to

http://breachrepairers.webs.com/ 344
Jesus. If only the disciples had called out to Jesus earlier they would have saved themselves a lot of
work and grief.

What does Jesus do next?

Mark 4:39

He rebukes the wind and sea. He tells them “Peace, be still.”

If Jesus knew that the storm was coming, why did He first ask the disciples to go across the
sea?

Jesus wanted to teach the disciples a lesson about faith, or their lack of faith. Remember that He had
just been teaching about faith just before they got into the boat.

Application: Many times, Jesus deliberately brings us into the storms of life so that He can teach us
something or test our character to show us where we are at.

4:39 And he arose, and rebuked the wind, and said unto the sea, Peace, be still. And the wind ceased, and
there was a great calm.

4:40 And he said unto them, Why are ye so fearful? how is it that ye have no faith?

What two things did Jesus rebuke the disciples for? What lessons can we learn from this?

Answer: 1. For fearing. 2. For having no faith.

Application: Fear = Lack of faith. The spirit of blaming Christ came from the disciples’ lack of faith in
Jesus and first trying on their own human strength.

Lessons:
 1. The disciples forgot Jesus words of promise that they would pass over to the other side.
Lesson 1 = Remember God’s word and hold on to promises during storms.

 2. The disciples tried to save themselves on their own strength first. Righteousness by
works.
Lesson 2 = Have faith in God. Righteousness by faith.

 3. The disciples only called out to Christ when the boat was full.
Lesson 3 = Seek God first, not last.

 4. Their desperation, lack of faith caused them to doubt Christ’s care and love for them.
Lesson 4 = Do not fear, but remember God’s love.

What does the Bible tell us about fear and faith?

1 John 4:18

There is no fear in love. If we really love Jesus we will have faith.

http://breachrepairers.webs.com/ 345
Gal 5:6

Faith works by love.

Isa 57:20-21

There is no peace with the wicked. They are fearful. Why? Because they don’t love God and
they don’t trust Him.

Rom 5:1

If we are justified by faith, we don’t need to fear for our lives. Many are living by works and
because our works always fall short (it will always make the boat sink) that we become
fearful.

Application: The disciples did not have perfect love for Jesus. They did not completely trust Him at
His word. Those who have perfect love and faith will also have peace in their lives, despite the
storms. God’s people should be able to pass through storms, but still have perfect love and peace. It
doesn’t mean that all our storms will be taken away, but they are given so that it may reveal what
sort of faith we have been building. Trials don’t build faith, they reveal the sort of faith that we have.

What key attributes must the people at the end of time have that are brought out in this
story?

Rev 14:12

Patience – patience to endure storms of life. Faith – to believe that God will carry you through.

4:41 And they feared exceedingly, and said one to another, What manner of man is this, that even the
wind and the sea obey him?

http://breachrepairers.webs.com/ 346
Chapter 5 - Demoniacs of the Gadarenes | Women with Issue of
Blood
Reading
Desire of Ages – Chapter The Demoniacs

Chapter Outline
 Christ casts out legion into the swine (1-20)
 Resurrection of Jairus daughter and the Issue of Blood (21-43)

Christ casts out legion into the swine (1-20)


5:1 And they came over unto the other side of the sea, into the country of the Gadarenes.

5:2 And when he was come out of the ship, immediately there met him out of the tombs a man with an
unclean spirit,

Unclean spirit is a fallen angel

Parallel passage is Luke 8:27

5:3 Who had his dwelling among the tombs; and no man could bind him, no, not with chains:

What are some of the characteristics of the demoniacs?

Luke 8:27

He lived in tombs. No man could bind him with chains. No man could tame him. Always crying.
Always cutting himself. He wore no clothes. He had supernatural strength. This indicates what
Satan will do when he has control of us. When Satan does that, he is trying to destroy the image of
God in man.

Application: Do you think Satan is pleased when we pierce ourselves? Or when we get tattoos? Baal
worship, they cut themselves. Anytime you are destroying your body in some way, Satan is behind
that. He made them naked – that is a symbol of sin. Satan does not get you put your clothes on, he
gets you to take them off. When you are doing something night and day, you get no rest! Satan does
not give you any rest when he takes full control.

5:4 Because that he had been often bound with fetters and chains, and the chains had been plucked
asunder by him, and the fetters broken in pieces: neither could any man tame him.

5:5 And always, night and day, he was in the mountains, and in the tombs, crying, and cutting himself
with stones.

Night and day he was crying and cutting himself.

5:6 But when he saw Jesus afar off, he ran and worshipped him,

5:7 And cried with a loud voice, and said, What have I to do with thee, Jesus, thou Son of the most high
God? I adjure thee by God, that thou torment me not.

http://breachrepairers.webs.com/ 347
5:8 For he said unto him, Come out of the man, thou unclean spirit.

5:9 And he asked him, What is thy name? And he answered, saying, My name is Legion: for we are many.

What was the name of the unclean spirit? And what is the meaning of its name?

3000-6000 infantry troops and 100-200 cavalry. Application: How many demons do you have in
you? Selfishness, pride, etc…?

5:10 And he besought him much that he would not send them away out of the country.

5:11 Now there was there nigh unto the mountains a great herd of swine feeding.

5:12 And all the devils besought him, saying, Send us into the swine, that we may enter into them.

What did the unclean spirits enter after departing from the man? What applications can we
learn from this?

They entered the pigs. Application: God will take bad stuff away for us with good reason.

5:13 And forthwith Jesus gave them leave. And the unclean spirits went out, and entered into the swine:
and the herd ran violently down a steep place into the sea, (they were about two thousand;) and were
choked in the sea.

Two thousand swine perished.

Luke 8:30

Legion means many. A legion of Roman soldiers consists of a thousand soldiers.

5:14 And they that fed the swine fled, and told it in the city, and in the country. And they went out to see
what it was that was done.

What was the reaction of those that fed the swine?

They fled and told everyone in the city and country.

5:15 And they come to Jesus, and see him that was possessed with the devil, and had the legion, sitting,
and clothed, and in his right mind: and they were afraid.

What was the new look of the man that was possessed with the unclean spirit? Compare it to
his previous description. What lessons can we learn from that?

Sitting, clothed and in the right mind. Jesus is concerned with our actions, the way we look and our
mind. When Jesus cleans us up, He cleans us up in three ways: Our actions are different. Our
outward appearances are different. The way we think is different which affects the way that we talk
(Matt. 12:34)

What was the reaction of the people that came to see the reports? Why is it they reacted that
way? Discuss.

http://breachrepairers.webs.com/ 348
They were afraid. Why were they afraid? Maybe they were afraid of Jesus making them lose more
substances because the swine had drowned already

Application: Sometimes we become afraid of what Jesus might take away from us if we were to
follow Him. We are afraid to sacrifice.

5:16 And they that saw it told them how it befell to him that was possessed with the devil, and also
concerning the swine.

5:17 And they began to pray him to depart out of their coasts.

What was the final reaction of the townsfolk (Mark 5:17) as compared to the demoniac that
was now free (Mark 5:18)? Why do you think the townsfolk reacted that way?

They thought more of what they lost instead of a man saved to the kingdom of God. Sometimes we
look more on our own selfish things and are not willing to sacrifice those things which are harmful
to us – swine were unclean. They lost what was bad for them, yet they complained. Are we like that
sometimes?

What was the reaction of Jesus to the townsfolk and the demoniac? What lessons can we
learn from His reaction to both groups?

Jesus didn’t react or say anything to the townsfolk. Jesus said no to the man that was possessed
with the unclean spirit. Sometimes, the better work is done by the locals. No point to argue with
those who are the locals because they have more respect. So He left behind the man that was
possessed because he was a local. Sometimes Jesus says no to even the good things and we may be a
bigger blessing

5:18 And when he was come into the ship, he that had been possessed with the devil prayed him that he
might be
with him.

Luke 8:35

“sitting at the feet of Jesus” – the attitude of learning.

“clothed” – he must have gotten his clothes from Jesus

“In his right mind” – when we are apart from Christ, we are really out of our mind.

5:19 Howbeit Jesus suffered him not, but saith unto him, Go home to thy friends, and tell them how great
things the Lord hath done for thee, and hath had compassion on thee.

Principle of witnessing

Luke 8:38-39

The first area of witnessing is our own household.

DA 340 But they bore in their own persons the evidence that Jesus was the Messiah. They could tell
what they knew; what they themselves had seen, and heard, and felt of the power of Christ. This is

http://breachrepairers.webs.com/ 349
what everyone can do whose heart has been touched by the grace of God. –

DA 339 In doing this work they could receive a greater blessing than if, merely for benefit to
themselves, they had remained in His presence.

You can witness wherever you go – what Christ has done for you in your life.

What lesson can we learn from the man that was possessed by the unclean spirit who obeyed
Jesus’ command to go tell what great things God had done for him?

Mark 5:19-20

You don’t need to know much to share. Just your own experience what great things God has done
for you? Application: is it sometimes that you don’t think about what God has done for you and as a
result, you don’t share about Him at all? Witnessing begins with remembering the great things God
has done for you.

5:20 And he departed, and began to publish in Decapolis how great things Jesus had done for him: and
all men did marvel.

What was the fruit of the man who bore his simple testimony to the whole town?

Luke 8:40

As a result of his testimony, the whole town after that came to see Jesus. How does his simple
testimony apply to us today? What lesson can we learn from that?

Rev 12:11

Overcoming has a lot to do with bearing our own testimony, witnessing.

“Decapolis" – Took place during Jesus' second Galilean tour. Between the 2nd and 3rd Passover.
Jesus was 32 Note: He had 1 ½ years left on this earth. This increases the intensity of all His actions.
Between 60-75% of His work had been accomplished at this point, but He didn't have anytime to
loose. He knew He was going to die. Extra: I am beginning to see that there are more examples of
believing, in exercising faith in the gospels than there are of works. Not that works aren't important,
but they are put in their proper place. God is emphasizing to us BELIEVE!

Resurrection of Jairus daughter and the Issue of Blood (21-43)


5:21 And when Jesus was passed over again by ship unto the other side, much people gathered unto him:
and he was nigh unto the sea.

5:22 And, behold, there cometh one of the rulers of the synagogue, Jairus by name; and when he saw him,
he fell at his feet,

5:23 And besought him greatly, saying, My little daughter lieth at the point of death: I pray thee, come
and lay thy hands on her, that she may be healed; and she shall live.

5:24 And Jesus went with him; and much people followed him, and thronged him.

5:25 And a certain woman, which had an issue of blood twelve years,

http://breachrepairers.webs.com/ 350
The parallel passage of Mark 5:24-43 is Luke 8:40-56.

5:26 And had suffered many things of many physicians, and had spent all that she had, and was nothing
bettered, but rather grew worse,

What do we know about the past of the woman with the issue of blood?

Luke 8:43

She had tried many other physicians who had not made her better but worse. She had spent all her
living on trying to find healing.

Who does the woman represent today? Application: The woman represents those who have been
looking for healing – physical or spiritual. They have tried all types of solutions / physicians –
religions, worldly solutions – amusement or entertainment. But still they are not fulfilled or made
whole.

5:27 When she had heard of Jesus, came in the press behind, and touched his garment.

How did the woman know about Jesus? She had heard about Jesus, probably from the crowd or
through friends. What does the manner in which the woman approached Jesus and what she
did tell us about her? What application can we draw? She was too shy to speak to Jesus openly.
Instead of approaching Jesus directly, she came from behind him. Instead of asking Jesus upfront,
she preferred to touch his garment.

Application: This woman represents those in the crowd or church today who are scared to openly
seek Jesus. Maybe they are embarrassed of their condition or just naturally very shy. Shy people
should take comfort that Jesus does not miss them, even in the crowd.

Points: Issue of blood 12 years. Jesus was 20 years old when this condition began with her. He had
not yet been revealed as the Messiah. God the Father was orchestrating events that would reach a
pinnacle in Decapolis. For 12 years she sought help in vain. And it gives a picture that she became
poor. It says she spent all. And as she went from physician to physician her anxiety increased. (The
worse the condition, the greater the physician. As you go from physician to physician the quality
and skill of the physician increases, not decreases, later exalting Jesus above all earthly skill and
ability. And as the quality increases the cost rises, and she ends up spending all she had, but later
Jesus does it for free.) Her dependence upon man decreased. It also gives the picture of the
imbecility of man. As a result of seeing the physicians it says she grew worse. But at the same time it
shows an attitude of perseverance and determination. Perhaps if she had heard of Jesus sooner she
would not have responded in the same way. After 12 years Jesus comes. When she heard of Him.
And now Jesus comes. Do you think He knew about her condition? Yes, but He doesn't go to her
directly. He appears to be so busy. But He places Himself in close proximity to her giving her a
chance to seek Him.

“came in the press”

Mark 5:24 …much people followed him, and thronged him…

This happened at a time when all the attention was fixed upon Jesus. Every one was following Him
pressed together. No one wanted to loose their position. Look there He is! This is the one we have

http://breachrepairers.webs.com/ 351
heard about. It was like religious revival. And no one was paying attention to her. (It's like what
about me? He is not paying attention to me. What about my issue of blood?) Per App: But they were
not the real followers. She was the real follower. Per App: To follower after Jesus is more then just
hearing about Him or hearing the spoken word Sabbath after Sabbath. To be a true follower of Jesus
is to seek after Him for spiritual restoration. To overcome your issue of blood.

Mark 5:27 …touched His garment…

5:28 For she said, If I may touch but his clothes, I shall be whole.

Do you think she said this out loud? No. She said it within her heart. 12 years of disappointment and
grief were bottled up inside her. (Just like when you shake up carbonated water, when you open the
cap, POOF!) And she reaches out and touches the tip of His garment. Where do you think she got
this attitude from? From all exterior appearance Jesus was just another man. To her another
physician. She said I will be what? 'whole' The previous year Jesus went to the pool of Bethesida.
And Jesus asked the invalid of 38 years wilt thou be made whole? She had heard of His power. And
she knew He was not just another physician. He was THEE physician.

Mark 5:29 She touched Him and straight way her blood was dried up.

What action did the woman have to do to be healed? What was probably going through her
mind? She had to reach out to touch Jesus. She believed that if only she touched Jesus garments, she
could be made whole.

Where on Christ’s garment did she touch? What does it represent?

Luke 8:44

Num 15:38-39

She touched the border of Christ’s garment. What does it represent? In reaching out to touching the
border of Christ’s garment, the woman represents those who reach out for healing in faith, and are
willing to keep God’s commandments and no longer seek after the things of their own heart. This is
true conversion.

What does Jesus do as soon as he is touched? What can we learn from this?

Luke 8:45-46

He asks who touched him because he perceives virtue gone out of him.God notices the unnoticed
touch of faith of an earnest sinner seeking for healing above the crowd.

Discuss the reactions of Peter and the crowd. Why do you think Jesus felt the woman’s touch
over everyone else? Peter and the crowd had been pressed against Jesus and yet they had felt
nothing.

Application: There are many in the crowd who come to church every week who press against Jesus
but feel nothing. There is no change in their life, no healing, because they are not reaching out.
Maybe they are in the crowd because they find Christ interesting, or their friends invited them, etc,
but they themselves are not reaching out for anything in particular. Therefore, they will receive
nothing in particular.

http://breachrepairers.webs.com/ 352
Why did Jesus want the woman to share what she had done? What does it represent? Who in
particular did it encourage in the crowd?

Isa 43:12

Testify.

Matt 10:32

Confess. Jesus wanted the woman to make a public confession and testimony for everyone. It was
not enough for the woman to experience the healing power of Christ, she needed to confess it to all
men.

Rev 12:11

God’s people will overcome not just through the blood of the lamb, but by the word of their
testimony. What are they supposed to testify? How Christ has healed them from their sinful,
helpless lives. What is so important about sharing? By sharing how Jesus had healed her, this
woman would encourage others to reach out to Jesus. What do you think happened right after she
told everyone what happened? Probably everyone started trying to touch Jesus.

What can we learn about faith from the woman?

Faith involves:
 1. Understanding our needs – physical, spiritual. Do we see that there are things in our life
we cannot overcome? Can we see we need a Savior? Did the woman know how Christ was
going to heal her? No. Did she even know Christ? No. Then what was it that made Jesus save
her? Her desire and desperation to be saved. She needed a Savior.
 2. Reaching out to Christ with a determination that even if we touch the hem of his garment,
we can be healed.
o Can you imagine the woman reaching out with all the determination and
desperation? This is how our faith should be.
o Every day as reach out to Christ, we should reach out with desperation that He is
our only hope.
o We must realize that we can try everything in this world, but only Christ can truly
make us whole.
 3. Obedience to the law.

5:29 And straightway the fountain of her blood was dried up; and she felt in her body that she was
healed of that plague.

5:30 And Jesus, immediately knowing in himself that virtue had gone out of him, turned him about in the
press, and said, Who touched my clothes?

“virtue had gone out” – Who touched me? He could have just kept walking. Look at the nature of
His question. He could have said anything else. But when He asked this question the people would
begin to do what? Start thinking about touching Him. They would be like what do you mean? That is
why Peter said V.31 That means that Jesus wanted them to understand what it really means to
touch Him. Do you think He didn't know who she was?

http://breachrepairers.webs.com/ 353
Mark 5:32 …looked round about to see her…

Not look round to see the person who touched Him. It shows He already knew who it was. (Ex.
Remember Nathaniel, and the man who they brought threw the roof? He was already acquainted with
them.) He uses her as an example to show what it really means to touch Him! And it was because
she exercised faith that He uses her as an example.

PNA: You see we can be so close to Jesus. We can be bumping Him in a crowd with other people. But
just because we are nudging Him it does not mean we are truly TOUCHING! People around us tell us
He did this for me, He did that, did you hear about it? Did you hear that sermon, etc. That is like
bumping up against Him. But that is not enough. You must truly TOUCH HIM!

5:31 And his disciples said unto him, Thou seest the multitude thronging thee, and sayest thou, Who
touched me?

5:32 And he looked round about to see her that had done this thing.

5:33 But the woman fearing and trembling, knowing what was done in her, came and fell down before
him, and told him all the truth.

5:34 And he said unto her, Daughter, thy faith hath made thee whole; go in peace, and be whole of thy
plague.

“faith made thee whole” – Remember she said that in her mind. Now Jesus is confirming her
thoughts and actions. She believed and she acted on it. NOT ACTED THEN Believed. This word
whole includes forgiveness of sins as well as physical healing. (Ex. The man at the pool of
Betheisda)

PNA: What is your issue of blood? What has been plaguing you for so long? Many times in our walk
when we fall what do we do? Ask forgiveness and say Lord please help me, give me the power. Then
we get up try to do it in our own strength and fall again. It is like we rely on the act of repentance to
receive strength. BUT How was she made whole?

Mark 5:30 …virtue had gone out of Him…

This word virtue in the Greek means Dunamis power. She exercised FAITH and then received
POWER and then her blood was dried up. It's FAITH, POWER, Blood dried up. Not Repentance,
POWER, Blood dries up. In other words, you must believe that God can heal you first. And when you
exercise the Faith THEN the POWER is given, and then the blood dries up. Our problem is we don’t
believe.

Mark 5:34 …whole of thy plague….

Jesus just called her issue of blood a plague. It was a plague of blood. And she was cured from it.
Conclusion: Either you can exercise faith receive power and be cured of your plague of blood. Or
you can be a partaker of the plagues of blood when probation closes.

Miscellaneous: She has an interesting personality. We see excellent virtues: Determination;


perseverance; stamina, endurance, honesty. (she didn't just say I touched you. She told the whole
story. She didn't leave any thing out.) But at the same time she is shy, timid, and discreet. She

http://breachrepairers.webs.com/ 354
doesn't like to call attention to herself.

5:35 While he yet spake, there came from the ruler of the synagogue's house certain which said, Thy
daughter is dead: why troublest thou the Master any further?

5:36 As soon as Jesus heard the word that was spoken, he saith unto the ruler of the synagogue, Be not
afraid, only believe.

5:37 And he suffered no man to follow him, save Peter, and James, and John the brother of James.

5:38 And he cometh to the house of the ruler of the synagogue, and seeth the tumult, and them that wept
and wailed greatly.

5:39 And when he was come in, he saith unto them, Why make ye this ado, and weep? the damsel is not
dead, but sleepeth.

5:40 And they laughed him to scorn. But when he had put them all out, he taketh the father and the
mother of the damsel, and them that were with him, and entereth in where the damsel was lying.

5:41 And he took the damsel by the hand, and said unto her, Talitha cumi; which is, being interpreted,
Damsel, I say unto thee, arise.

5:42 And straightway the damsel arose, and walked; for she was of the age of twelve years. And they
were astonished with a great astonishment.

5:43 And he charged them straitly that no man should know it; and commanded that something should
be given her to eat.

http://breachrepairers.webs.com/ 355
Chapter 6
Chapter Outline
 Is this the carpenters Son? (1-6)
 Disciples receive power over sicknesses and demons (7-13)
 Herod kills John the Baptist (14-29)
 Feeding the 5,000 (30-44)
 Christ’s walks on water and heals in Gennsaret (45-56)

Is this the carpenters Son? (1-6)


6:1 And he went out from thence, and came into his own country; and his disciples follow him.

6:2 And when the sabbath day was come, he began to teach in the synagogue: and many hearing him
were astonished, saying, From whence hath this man these things? and what wisdom is this which is
given unto him, that even such mighty works are wrought by his hands?

6:3 Is not this the carpenter, the son of Mary, the brother of James, and Joses, and of Juda, and Simon?
and are not his sisters here with us? And they were offended at him.

6:4 But Jesus said unto them, A prophet is not without honour, but in his own country, and among his
own kin, and in his own house.

6:5 And he could there do no mighty work, save that he laid his hands upon a few sick folk, and healed
them.

6:6 And he marvelled because of their unbelief. And he went round about the villages, teaching.

Disciples receive power over sicknesses and demons (7-13)


6:7 And he called unto him the twelve, and began to send them forth by two and two; and gave them
power over unclean spirits;

6:8 And commanded them that they should take nothing for their journey, save a staff only; no scrip, no
bread, no money in their purse:

6:9 But be shod with sandals; and not put on two coats.

6:10 And he said unto them, In what place soever ye enter into an house, there abide till ye depart from
that place.

6:11 And whosoever shall not receive you, nor hear you, when ye depart thence, shake off the dust under
your feet for a testimony against them. Verily I say unto you, It shall be more tolerable for Sodom and
Gomorrha in the day of judgment, than for that city.

6:12 And they went out, and preached that men should repent.

6:13 And they cast out many devils, and anointed with oil many that were sick, and healed them.

Herod kills John the Baptist (14-29)


6:14 And king Herod heard of him; (for his name was spread abroad:) and he said, That John the Baptist

http://breachrepairers.webs.com/ 356
was risen from the dead, and therefore mighty works do shew forth themselves in him.

6:15 Others said, That it is Elias. And others said, That it is a prophet, or as one of the prophets.

6:16 But when Herod heard thereof, he said, It is John, whom I beheaded: he is risen from the dead.

6:17 For Herod himself had sent forth and laid hold upon John, and bound him in prison for Herodias'
sake, his brother Philip's wife: for he had married her.

6:18 For John had said unto Herod, It is not lawful for thee to have thy brother's wife.

6:19 Therefore Herodias had a quarrel against him, and would have killed him; but she could not:

6:20 For Herod feared John, knowing that he was a just man and an holy, and observed him; and when
he heard him, he did many things, and heard him gladly.

6:21 And when a convenient day was come, that Herod on his birthday made a supper to his lords, high
captains, and chief estates of Galilee;

6:22 And when the daughter of the said Herodias came in, and danced, and pleased Herod and them that
sat with him, the king said unto the damsel, Ask of me whatsoever thou wilt, and I will give it thee.

6:23 And he sware unto her, Whatsoever thou shalt ask of me, I will give it thee, unto the half of my
kingdom.

6:24 And she went forth, and said unto her mother, What shall I ask? And she said, The head of John the
Baptist.

6:25 And she came in straightway with haste unto the king, and asked, saying, I will that thou give me by
and by in a charger the head of John the Baptist.

6:26 And the king was exceeding sorry; yet for his oath's sake, and for their sakes which sat with him, he
would not reject her.

6:27 And immediately the king sent an executioner, and commanded his head to be brought: and he
went and beheaded him in the prison,

6:28 And brought his head in a charger, and gave it to the damsel: and the damsel gave it to her mother.

6:29 And when his disciples heard of it, they came and took up his corpse, and laid it in a tomb.

Feeding the 5,000 (30-44)


6:30 And the apostles gathered themselves together unto Jesus, and told him all things, both what they
had done, and what they had taught.

6:31 And he said unto them, Come ye yourselves apart into a desert place, and rest a while: for there
were many coming and going, and they had no leisure so much as to eat.

6:32 And they departed into a desert place by ship privately.

6:33 And the people saw them departing, and many knew him, and ran afoot thither out of all cities, and
outwent them, and came together unto him.

6:34 And Jesus, when he came out, saw much people, and was moved with compassion toward them,

http://breachrepairers.webs.com/ 357
because they were as sheep not having a shepherd: and he began to teach them many things.

6:35 And when the day was now far spent, his disciples came unto him, and said, This is a desert place,
and now the time is far passed:

6:36 Send them away, that they may go into the country round about, and into the villages, and buy
themselves bread: for they have nothing to eat.

6:37 He answered and said unto them, Give ye them to eat. And they say unto him, Shall we go and buy
two hundred pennyworth of bread, and give them to eat?

6:38 He saith unto them, How many loaves have ye? go and see. And when they knew, they say, Five, and
two fishes.

6:39 And he commanded them to make all sit down by companies upon the green grass.

6:40 And they sat down in ranks, by hundreds, and by fifties.

6:41 And when he had taken the five loaves and the two fishes, he looked up to heaven, and blessed, and
brake the loaves, and gave them to his disciples to set before them; and the two fishes divided he among
them all.

6:42 And they did all eat, and were filled.

6:43 And they took up twelve baskets full of the fragments, and of the fishes.

6:44 And they that did eat of the loaves were about five thousand men.

Christ’s walks on water and heals in Gennsaret (45-56)


6:45 And straightway he constrained his disciples to get into the ship, and to go to the other side before
unto Bethsaida, while he sent away the people.

6:46 And when he had sent them away, he departed into a mountain to pray.

6:47 And when even was come, the ship was in the midst of the sea, and he alone on the land.

6:48 And he saw them toiling in rowing; for the wind was contrary unto them: and about the fourth
watch of the night he cometh unto them, walking upon the sea, and would have passed by them.

6:49 But when they saw him walking upon the sea, they supposed it had been a spirit, and cried out:

6:50 For they all saw him, and were troubled. And immediately he talked with them, and saith unto
them, Be of good cheer: it is I; be not afraid.

6:51 And he went up unto them into the ship; and the wind ceased: and they were sore amazed in
themselves beyond measure, and wondered.

6:52 For they considered not the miracle of the loaves: for their heart was hardened.

6:53 And when they had passed over, they came into the land of Gennesaret, and drew to the shore.

6:54 And when they were come out of the ship, straightway they knew him,

6:55 And ran through that whole region round about, and began to carry about in beds those that were

http://breachrepairers.webs.com/ 358
sick, where they heard he was.

6:56 And whithersoever he entered, into villages, or cities, or country, they laid the sick in the streets,
and besought him that they might touch if it were but the border of his garment: and as many as
touched him were made whole.

http://breachrepairers.webs.com/ 359
Chapter 7
Chapter Outline
 Commandents of God vs. Tradition of men (1-23)
 Demon casts out of Syrophenecian woman (24-30)
 Ephatha (Blind and deaf man healed) (31-37)

Commandents of God vs. Tradition of men (1-23)


7:1 Then came together unto him the Pharisees, and certain of the scribes, which came from Jerusalem.

7:2 And when they saw some of his disciples eat bread with defiled, that is to say, with unwashen, hands,
they found fault.

7:3 For the Pharisees, and all the Jews, except they wash their hands oft, eat not, holding the tradition of
the elders.

7:4 And when they come from the market, except they wash, they eat not. And many other things there
be, which they have received to hold, as the washing of cups, and pots, brasen vessels, and of tables.

7:5 Then the Pharisees and scribes asked him, Why walk not thy disciples according to the tradition of
the elders, but eat bread with unwashen hands?

7:6 He answered and said unto them, Well hath Esaias prophesied of you hypocrites, as it is written, This
people honoureth me with their lips, but their heart is far from me.

7:7Howbeit in vain do they worship me, teaching for doctrines the commandments of men.

7:8 For laying aside the commandment of God, ye hold the tradition of men, as the washing of pots and
cups: and many other such like things ye do.

7:9 And he said unto them, Full well ye reject the commandment of God, that ye may keep your own
tradition.

7:10 For Moses said, Honour thy father and thy mother; and, Whoso curseth father or mother, let him die
the death:

7:11 But ye say, If a man shall say to his father or mother, It is Corban, that is to say, a gift, by
whatsoever thou mightest be profited by me; he shall be free.

7:12 And ye suffer him no more to do ought for his father or his mother;

7:13 Making the word of God of none effect through your tradition, which ye have delivered: and many
such like things do ye.

7:14 And when he had called all the people unto him, he said unto them, Hearken unto me every one of
you, and understand:

7:15 There is nothing from without a man, that entering into him can defile him: but the things which
come out of him, those are they that defile the man.

7:16 If any man have ears to hear, let him hear.

http://breachrepairers.webs.com/ 360
7:17 And when he was entered into the house from the people, his disciples asked him concerning the
parable.

7:18 And he saith unto them, Are ye so without understanding also? Do ye not perceive, that whatsoever
thing from without entereth into the man, it cannot defile him;

7:19 Because it entereth not into his heart, but into the belly, and goeth out into the draught, purging all
meats?

7:20 And he said, That which cometh out of the man, that defileth the man.

7:21 For from within, out of the heart of men, proceed evil thoughts, adulteries, fornications, murders,

7:22 Thefts, covetousness, wickedness, deceit, lasciviousness, an evil eye, blasphemy, pride, foolishness:

7:23 All these evil things come from within, and defile the man.

Demon casts out of Syrophenecian woman (24-30)


7:24 And from thence he arose, and went into the borders of Tyre and Sidon, and entered into an house,
and would have no man know it: but he could not be hid.

7:25 For a certain woman, whose young daughter had an unclean spirit, heard of him, and came and fell
at his feet:

7:26 The woman was a Greek, a Syrophenician by nation; and she besought him that he would cast forth
the devil out of her daughter.

7:27 But Jesus said unto her, Let the children first be filled: for it is not meet to take the children's bread,
and to cast it unto the dogs.

7:28 And she answered and said unto him, Yes, Lord: yet the dogs under the table eat of the children's
crumbs.

7:29 And he said unto her, For this saying go thy way; the devil is gone out of thy daughter.

7:30 And when she was come to her house, she found the devil gone out, and her daughter laid upon the
bed.

Ephatha (Blind and deaf man healed) (31-37)


7:31 And again, departing from the coasts of Tyre and Sidon, he came unto the sea of Galilee, through
the midst of the coasts of Decapolis.

7:32 And they bring unto him one that was deaf, and had an impediment in his speech; and they beseech
him to put his hand upon him.

7:33 And he took him aside from the multitude, and put his fingers into his ears, and he spit, and touched
his tongue;

7:34 And looking up to heaven, he sighed, and saith unto him, Ephphatha, that is, Be opened.

7:35 And straightway his ears were opened, and the string of his tongue was loosed, and he spake plain.

http://breachrepairers.webs.com/ 361
7:36 And he charged them that they should tell no man: but the more he charged them, so much the
more a great deal they published it;

7:37 And were beyond measure astonished, saying, He hath done all things well: he maketh both the deaf
to hear, and the dumb to speak.

http://breachrepairers.webs.com/ 362
Chapter 8 – Blind Man near Bethsaida
Chapter Outline
 Feeding of the 4,000 (1-9)
 Pharisees seek a sign (10-13)
 Beware of the leaven of Herod and Pharisees (14-21)
 Healing of the blind man with saliva (22-26)
 Who am I? (Confession of Peter) (27-38)

Feeding of the 4,000 (1-9)


8:1 In those days the multitude being very great, and having nothing to eat, Jesus called his disciples
unto him, and saith unto them,

8:2 I have compassion on the multitude, because they have now been with me three days, and have
nothing to eat:

8:3 And if I send them away fasting to their own houses, they will faint by the way: for divers of them
came from far.

8:4 And his disciples answered him, From whence can a man satisfy these men with bread here in the
wilderness?

8:5 And he asked them, How many loaves have ye? And they said, Seven.

8:6 And he commanded the people to sit down on the ground: and he took the seven loaves, and gave
thanks, and brake, and gave to his disciples to set before them; and they did set them before the people.

8:7 And they had a few small fishes: and he blessed, and commanded to set them also before them.

8:8 So they did eat, and were filled: and they took up of the broken meat that was left seven baskets.

8:9 And they that had eaten were about four thousand: and he sent them away.

Pharisees seek a sign (10-13)


8:10 And straightway he entered into a ship with his disciples, and came into the parts of Dalmanutha.

8:11 And the Pharisees came forth, and began to question with him, seeking of him a sign from heaven,
tempting him.

8:12 And he sighed deeply in his spirit, and saith, Why doth this generation seek after a sign? verily I say
unto you, There shall no sign be given unto this generation.

8:13 And he left them, and entering into the ship again departed to the other side.

Beware of the leaven of Herod and Pharisees (14-21)


8:14 Now the disciples had forgotten to take bread, neither had they in the ship with them more than one
loaf.

8:15 And he charged them, saying, Take heed, beware of the leaven of the Pharisees, and of the leaven of

http://breachrepairers.webs.com/ 363
Herod.

8:16 And they reasoned among themselves, saying, It is because we have no bread.

8:17 And when Jesus knew it, he saith unto them, Why reason ye, because ye have no bread? perceive ye
not yet, neither understand? have ye your heart yet hardened?

8:18 Having eyes, see ye not? and having ears, hear ye not? and do ye not remember?

8:19 When I brake the five loaves among five thousand, how many baskets full of fragments took ye up?
They say unto him, Twelve.

8:20 And when the seven among four thousand, how many baskets full of fragments took ye up? And they
said, Seven.

8:21 And he said unto them, How is it that ye do not understand?

Healing of the blind man with saliva (22-26)


8:22 And he cometh to Bethsaida; and they bring a blind man unto him, and besought him to touch him.

Who was brought to Jesus? Who brought the man?

Answer: A blind man. Someone who could not find Jesus for himself. Who brought the man? Most
likely people from the town. His friends.

In what manner did ‘they’ ask for Jesus’ help? What did ‘they’ want Jesus to do?

Answer: Like the nobleman (John 5:43-54), the blind man’s friends were beseeching Jesus. It says
he “besought” = beseeching = pleading or begging

Application: When we are trying to bring others to Christ, we should have a beseeching spirit. We
should be pleading with Christ on their behalf. And we know that Christ will have compassion. Too
often, we fail to care as much as we should about our family and friends’ salvation. They wanted
Jesus to touch him.

Application: Our most basic request to Jesus should be that He touches their lives.

8:23 And he took the blind man by the hand, and led him out of the town; and when he had spit on his
eyes, and put his hands upon him, he asked him if he saw ought.

What was the first thing that Jesus did? What application can we draw from this?

Answer: Jesus took the blind man by the hand and led him out of the city.

Application: Sometimes, those who are spiritually blind need to be lead out of the city. They need to
be separated from the places which are causing them to sin. They need to be lead away from their
friends. This means that we need to spend time with them. Holding their hand indicates that we
need to be there every step of the way, like a child. Words just won’t be enough, you need to
literally show them, hand hold them.

What was the next thing that Jesus did?

http://breachrepairers.webs.com/ 364
Answer: Jesus spit in his eyes and put his hands upon him.

What is a spiritual remedy for the eyes?

Rev. 3:18

Eyesalve. This blind man also represents those who are lukewarm / Laodicean because they
are also called blind (Rev 3:17). What does eyesalve represent? It provides sight to the spiritually
blind.

1 Sam 9:9

Those who see are the prophets. To anoint ourselves with eyesalve from God is to seek the
counsel of the prophets.

Rom 1:2
Where can we hear the words of the prophets? In the scriptures.

What is the purpose of spiritual sight?

Eph 1:18

To understand the hope of our calling and the inheritance for saints.

Acts 26:18

This verse gives 4 reasons: Turn from darkness to light (spiritual discernment). 2 Cor 4:16 –
knowledge of Jesus. To turn them from the power of Satan to God. What is the power of Satan? Heb
2:14 = power of death. What is the power of God? Rom 1:4 = resurrection. Summary – power of
Satan (death) to power of God (life in resurrection). Receive forgiveness of sins. Acts 5:31 – receive
forgiveness through repentance. Receive inheritance among them which are sanctified by faith
(likewise, in order to receive this inheritance, each individual must also be sanctified by faith. What
is this inheritance? 1 Cor 15:50-53 – eternal life. We see a process here from knowledge of Jesus to
eternal life.

Psa 119:18

What gives us light? Psalms 119:105 – Thy word is a lamp unto my feet, and a light unto my path.

8:24 And he looked up, and said, I see men as trees, walking.

What was the response when Jesus asked him what he saw? What does this represent?

Psa 1:3

Tree according to this verse represents a righteous man. The man looked up and said he saw men
walking as trees. What do trees represent according to the Bible?

Application: Sometimes when a person is first touched by Christ and as their spiritual eyes begin to
open, they look at all men as righteous. Or they follow men’s righteousness when they try to look

http://breachrepairers.webs.com/ 365
up. They see men as walking trees, walking righteous.

Application: His eyesight was blurry – could not clearly discern light and darkness, do not
completely understand forgiveness of sins, does not completely understand sanctification.

8:25 After that he put his hands again upon his eyes, and made him look up: and he was restored, and
saw every man clearly.

What happens to the man after Jesus touches him a second time?

Answer: The man looks up again. This time he sees men more clearly.

Application: As people are touched more by Christ and as they apply the eyesalve more to their
lives, they will see men more clearly. A more mature Christian relies less on men as they see men
more clearly for what they are – unrighteous. However, we should understand that young
Christians look to those who have lead them to Christ as trees of righteousness. We need to
continually point them up to Christ.

8:26 And he sent him away to his house, saying, Neither go into the town, nor tell it to any in the town.

What lessons can we learn from this encounter?


Answer: Sometimes spiritually blind need to be lead away from the city or places of temptation in
order to be healed. Blind men need eyesalve from Christ. Blind men need to be touched by Christ
personally. As blind men are touched by Christ, they will at first mistakenly see men as righteous
and look to them for direction. However, as they get to know Christ more, they will begin to better
spiritually discern which men are righteous.

Who am I? (Confession of Peter) (27-38)


8:27 And Jesus went out, and his disciples, into the towns of Caesarea Philippi: and by the way he asked
his disciples, saying unto them, Whom do men say that I am?

8:28 And they answered, John the Baptist: but some say, Elias; and others, One of the prophets.

8:29 And he saith unto them, But whom say ye that I am? And Peter answereth and saith unto him, Thou
art the Christ.

8:30 And he charged them that they should tell no man of him.

8:31 And he began to teach them, that the Son of man must suffer many things, and be rejected of the
elders, and of the chief priests, and scribes, and be killed, and after three days rise again.

8:32 And he spake that saying openly. And Peter took him, and began to rebuke him.

8:33 But when he had turned about and looked on his disciples, he rebuked Peter, saying, Get thee
behind me, Satan: for thou savourest not the things that be of God, but the things that be of men.

8:34 And when he had called the people unto him with his disciples also, he said unto them, Whosoever
will come after me, let him deny himself, and take up his cross, and follow me.

8:35 For whosoever will save his life shall lose it; but whosoever shall lose his life for my sake and the
gospel's, the same shall save it.

http://breachrepairers.webs.com/ 366
8:36 For what shall it profit a man, if he shall gain the whole world, and lose his own soul?

8:37 Or what shall a man give in exchange for his soul?

8:38 Whosoever therefore shall be ashamed of me and of my words in this adulterous and sinful
generation; of him also shall the Son of man be ashamed, when he cometh in the glory of his Father with
the holy angels.

http://breachrepairers.webs.com/ 367
Chapter 9
Chapter Outline
 The coming Kingdom of God (Transfiguration of Christ) (1-13)
 Cast out through Fasting and Prayer (14-29)
 Who is the greatest? (30-37)
 How to enter the kingdom of God (38-50)

The coming Kingdom of God (Transfiguration of Christ) (1-13)


9:1 And he said unto them, Verily I say unto you, That there be some of them that stand here, which shall
not taste of death, till they have seen the kingdom of God come with power.

9:2 And after six days Jesus taketh with him Peter, and James, and John, and leadeth them up into an
high mountain apart by themselves: and he was transfigured before them.

9:3 And his raiment became shining, exceeding white as snow; so as no fuller on earth can white them.

9:4 And there appeared unto them Elias with Moses: and they were talking with Jesus.

9:5 And Peter answered and said to Jesus, Master, it is good for us to be here: and let us make three
tabernacles; one for thee, and one for Moses, and one for Elias.

9:6 For he wist not what to say; for they were sore afraid.

9:7 And there was a cloud that overshadowed them: and a voice came out of the cloud, saying, This is my
beloved Son: hear him.

9:8 And suddenly, when they had looked round about, they saw no man any more, save Jesus only with
themselves.

9:9 And as they came down from the mountain, he charged them that they should tell no man what
things they had seen, till the Son of man were risen from the dead.

9:10 And they kept that saying with themselves, questioning one with another what the rising from the
dead should mean.

9:11 And they asked him, saying, Why say the scribes that Elias must first come?

9:12 And he answered and told them, Elias verily cometh first, and restoreth all things; and how it is
written of the Son of man, that he must suffer many things, and be set at nought.

9:13 But I say unto you, That Elias is indeed come, and they have done unto him whatsoever they listed,
as it is written of him.

Cast out through Fasting and Prayer (14-29)


9:14 And when he came to his disciples, he saw a great multitude about them, and the scribes
questioning with them.

9:15 And straightway all the people, when they beheld him, were greatly amazed, and running to him
saluted him.

http://breachrepairers.webs.com/ 368
9:16 And he asked the scribes, What question ye with them?

9:17 And one of the multitude answered and said, Master, I have brought unto thee my son, which hath a
dumb spirit;

9:18 And wheresoever he taketh him, he teareth him: and he foameth, and gnasheth with his teeth, and
pineth away: and I spake to thy disciples that they should cast him out; and they could not.

9:19 He answereth him, and saith, O faithless generation, how long shall I be with you? how long shall I
suffer you? bring him unto me.

9:20 And they brought him unto him: and when he saw him, straightway the spirit tare him; and he fell
on the ground, and wallowed foaming.

9:21 And he asked his father, How long is it ago since this came unto him? And he said, Of a child.

9:22 And ofttimes it hath cast him into the fire, and into the waters, to destroy him: but if thou canst do
any thing, have compassion on us, and help us.

9:23 Jesus said unto him, If thou canst believe, all things are possible to him that believeth.

9:24 And straightway the father of the child cried out, and said with tears, Lord, I believe; help thou mine
unbelief.

9:25 When Jesus saw that the people came running together, he rebuked the foul spirit, saying unto him,
Thou dumb and deaf spirit, I charge thee, come out of him, and enter no more into him.

9:26 And the spirit cried, and rent him sore, and came out of him: and he was as one dead; insomuch that
many said, He is dead.

9:27 But Jesus took him by the hand, and lifted him up; and he arose.

9:28 And when he was come into the house, his disciples asked him privately, Why could not we cast him
out?

9:29 And he said unto them, This kind can come forth by nothing, but by prayer and fasting.

Who is the greatest? (30-37)


9:30 And they departed thence, and passed through Galilee; and he would not that any man should know
it.

9:31 For he taught his disciples, and said unto them, The Son of man is delivered into the hands of men,
and they shall kill him; and after that he is killed, he shall rise the third day.

9:32 But they understood not that saying, and were afraid to ask him.

9:33 And he came to Capernaum: and being in the house he asked them, What was it that ye disputed
among yourselves by the way?

9:34 But they held their peace: for by the way they had disputed among themselves, who should be the
greatest.

9:35 And he sat down, and called the twelve, and saith unto them, If any man desire to be first, the same

http://breachrepairers.webs.com/ 369
shall be last of all, and servant of all.

9:36 And he took a child, and set him in the midst of them: and when he had taken him in his arms, he
said unto them,

9:37 Whosoever shall receive one of such children in my name, receiveth me: and whosoever shall
receive me, receiveth not me, but him that sent me.

How to enter the kingdom of God (38-50)


9:38 And John answered him, saying, Master, we saw one casting out devils in thy name, and he
followeth not us: and we forbad him, because he followeth not us.

9:39 But Jesus said, Forbid him not: for there is no man which shall do a miracle in my name, that can
lightly speak evil of me.

9:40 For he that is not against us is on our part.

9:41 For whosoever shall give you a cup of water to drink in my name, because ye belong to Christ, verily
I say unto you, he shall not lose his reward.

9:42 And whosoever shall offend one of these little ones that believe in me, it is better for him that a
millstone were hanged about his neck, and he were cast into the sea.

9:43 And if thy hand offend thee, cut it off: it is better for thee to enter into life maimed, than having two
hands to go into hell, into the fire that never shall be quenched:

9:44 Where their worm dieth not, and the fire is not quenched.

9:45 And if thy foot offend thee, cut it off: it is better for thee to enter halt into life, than having two feet
to be cast into hell, into the fire that never shall be quenched:

9:46 Where their worm dieth not, and the fire is not quenched.

9:47 And if thine eye offend thee, pluck it out: it is better for thee to enter into the kingdom of God with
one eye, than having two eyes to be cast into hell fire:

9:48 Where their worm dieth not, and the fire is not quenched.

9:49 For every one shall be salted with fire, and every sacrifice shall be salted with salt.

9:50 Salt is good: but if the salt have lost his saltness, wherewith will ye season it? Have salt in
yourselves, and have peace one with another.

Summary
For all intensive purposes Mark is finish with the main emphasis in his biography, which is Christ’s
role as a Servant. The demon that was cast out in this chapter was the most powerful of the
previous demons. This is the second climatic chapter in Mark. From this point his focus changes to
the closing scenes.

http://breachrepairers.webs.com/ 370
Chapter 10 – Conditions of the Kingdom of God
Chapter Outline
 Is it lawful to put away your wife? (1-12)
 Little children and the kingdom of God (13-16)
 Rich young ruler (kingdom of God) (17-31)
 Let us sit on your right and left hand (32-45)
 Blind Bartimaeus (Son of David) (46-52)

Is it lawful to put away your wife? (1-12)


10:1 And he arose from thence, and cometh into the coasts of Judaea by the farther side of Jordan: and
the people resort unto him again; and, as he was wont, he taught them again.

10:2 And the Pharisees came to him, and asked him, Is it lawful for a man to put away his wife? tempting
him.

10:3 And he answered and said unto them, What did Moses command you?

10:4 And they said, Moses suffered to write a bill of divorcement, and to put her away.

10:5 And Jesus answered and said unto them, For the hardness of your heart he wrote you this precept.

10:6 But from the beginning of the creation God made them male and female.

10:7 For this cause shall a man leave his father and mother, and cleave to his wife;

10:8 And they twain shall be one flesh: so then they are no more twain, but one flesh.

10:9 What therefore God hath joined together, let not man put asunder.

10:10 And in the house his disciples asked him again of the same matter.

10:11 And he saith unto them, Whosoever shall put away his wife, and marry another, committeth
adultery against her.

10:12 And if a woman shall put away her husband, and be married to another, she committeth adultery.

Little children and the kingdom of God (13-16)


10:13 And they brought young children to him, that he should touch them: and his disciples rebuked
those that brought them.

10:14 But when Jesus saw it, he was much displeased, and said unto them, Suffer the little children to
come unto me, and forbid them not: for of such is the kingdom of God.

10:15 Verily I say unto you, Whosoever shall not receive the kingdom of God as a little child, he shall not
enter therein.

10:16 And he took them up in his arms, put his hands upon them, and blessed them.

http://breachrepairers.webs.com/ 371
Rich young ruler (kingdom of God) (17-31)
10:17 And when he was gone forth into the way, there came one running, and kneeled to him, and asked
him, Good Master, what shall I do that I may inherit eternal life?

What is the significance of kneeling? What does this indicate about the rich young ruler? The
significance of kneeling was showing that the rich young ruler recognized Jesus as his master. This
is also shown by the fact that he called him good master as well. This indicated that he knew who
Jesus was and he knew that eternal life was to come from Jesus. Or at least Jesus knew the TRUE
way to eternal life. It almost showed a type of respect to Jesus so that he could gain favor as it is in
Eastern cultures.

10:18 And Jesus said unto him, Why callest thou me good? there is none good but one, that is, God.

10:19 Thou knowest the commandments, Do not commit adultery, Do not kill, Do not steal, Do not bear
false witness, Defraud not, Honour thy father and mother.

What commandments did Jesus list and why? If you need to, compare it with the
commandments in Exodus 20. He listed commandments 5-9.

Mark 12:30-31

Jesus wanted to show the rich young ruler one thing that he could do to inherit eternal life, which
was loving his neighbour. And Jesus was about to show that this rich young rulers actions was going
against the second great commandment.

Which of the second segment of the commandments were missing? What does this tell us
about the rich young ruler’s problem? The 10th commandment was missing – thou shalt not
covet. This indicates to us that maybe this rich young ruler is struggling with coveting.

What order did Jesus give these commandments in? Why do you think Jesus gave it that way?
The order: Adultery – 7th commandment, Kill – 6th commandment, Steal – 8th commandment, Lie
(twice) – 9th commandment, and Honor father and mother – 5th commandment. Adultery was
considered worse than killing. Jesus listed the commandments from worse to better. Almost giving
the rich young ruler a false sense of security because he was quoting commandments that he had
kept. Notice that Jesus repeated lying twice. It was almost to say to the rich young ruler, please
don’t lie, don’t lie about your true condition about keeping them up from your youth.

Note: Obviously we know the rich young rulers response. That doesn’t need a question. Please bridge it
by asking them “What was the rich young rulers reply?” Let them respond. And then just add a
comment to their answer saying that the rich young ruler was probably feeling more confident now
because he had kept all the commandments that Jesus spoken. Maybe feeling a bit closer to eternal life.

10:20 And he answered and said unto him, Master, all these have I observed from my youth.

10:21 Then Jesus beholding him loved him, and said unto him, One thing thou lackest: go thy way, sell
whatsoever thou hast, and give to the poor, and thou shalt have treasure in heaven: and come, take up
the cross, and follow me.

What was Jesus’ response to the rich young rulers reply? What important lesson can we
learn from Jesus’ reply?

http://breachrepairers.webs.com/ 372
First – Jesus beholding him loved him. Second – Jesus speaks the lack to the rich young ruler and
then gives him the remedy, to sell all that he has and to follow Him. Important lesson: Before Jesus
asks us to give up everything to follow Him, He shows us reassurance in His look of love that He will
take care of us and supply all our need. Jesus reminds us of His love for us that we are more
precious than a bird or a grass of the field.

10:22 And he was sad at that saying, and went away grieved: for he had great possessions.

Why was the rich young ruler grieved? Contrast this with godly sorrow. What is the
difference?

2 Cor 7:10

He was grieved because he had a lot of possessions. The difference between this grief and the godly
sorrow is: This grief: He was sad because he wasn’t willing to follow the words of Jesus that he may
have eternal life. This showed the true character of the rich young ruler. He wanted to have the
riches of heaven with the sacrifice on earth. He was grieved over his earthly possessions. Godly
sorrow: He ought to have repented and given up his possessions at any cost that he may win
heaven. Godly sorrow is a type of sorrow that repents from past wrongs when shown that error
from the word of God.

10:23 And Jesus looked round about, and saith unto his disciples, How hardly shall they that have riches
enter into the kingdom of God!

10:24 And the disciples were astonished at his words. But Jesus answereth again, and saith unto them,
Children, how hard is it for them that trust in riches to enter into the kingdom of God!

What does it mean to trust in riches?

Ps 49:6-8

Means to boast about your riches. It means to trust in the riches to an extent that you can buy
pardon or mercy from God. Trying to pay the debt that you owe to God. Ransom – giving money in
exchange for something else. This rich man was trying to give God his earthly wealth so that he
could buy heaven.

What lesson of covetousness can we learn from this story? Covetousness isn’t just about trying
to buy up all that others have (trying to keep up with the Jones’) and it isn’t just about wanting
everything you see out there. From this encounter, we can see that covetousness can also be
equated to being rich, and not using those riches to bless others. It can also mean having riches or
possessions and not being willing to give them up to follow Jesus.

Application: Are you truly willing to give up all to follow Jesus? Or are you just like the rich young
ruler, desiring it, wanting to know the way to eternal life, but not being willing to give up all on the
altar of sacrifice for Jesus.

What is the difference between 10th commandment and the other 5? The other 5
commandments are all showing how you deal with your friends and family. It has external fruits to
others. The 10th commandment of covetousness is dealing with the heart and is more personal. It
doesn’t affect others in a bad way. But neither does it affect that person personally in a bad way
either (temporarily on this earth). The problem is not so apparent as one cannot condemn another

http://breachrepairers.webs.com/ 373
for being rich. The underlying problem is not so obvious.

What is Jesus’ solution to those who trust in riches? How rich do you think this man would
have been after that?

Mark 10:21

Sell everything that you have. He would actually have been a very poor man. Nothing left of this
earth. Application: Are you willing to sell all that you have that God may grant you eternal life? Easy
to say, hard to do. How much do you struggle with even giving a bit of money to the Lord work and
treasury? How many times do we find ourselves like the rich young ruler?… grieved.

10:25 It is easier for a camel to go through the eye of a needle, than for a rich man to enter into the
kingdom of God.

Why did Christ use the example of a camel passing through the eye of the needle? This
situation of a camel passing through an eye of a needle is almost wholly impossible. Jesus wanted to
demonstrate that it is wholly impossible for a rich man to enter into the kingdom of heaven. Why?
Because they will trust in their riches too much, or they would become covetous and not use it to
bless others.

10:26 And they were astonished out of measure, saying among themselves, Who then can be saved?

Why were the disciples astonished? The disciples were astonished because Jesus’ reply
practically gave no hope of any rich man going to heaven.

10:27 And Jesus looking upon them saith, With men it is impossible, but not with God: for with God all
things are possible.

Contextually speaking, what does Jesus mean when He said “With men it is impossible, but
not with God: for with God all things are possible.” Within context, it doesn’t mean that we can
do everything. But that it is possible for a rich man to be saved. But how will he be saved? He must
cooperate with Christ and be willing to sell all that he has.

10:28 Then Peter began to say unto him, Lo, we have left all, and have followed thee.

10:29 And Jesus answered and said, Verily I say unto you, There is no man that hath left house, or
brethren, or sisters, or father, or mother, or wife, or children, or lands, for my sake, and the gospel's,

10:30 But he shall receive an hundredfold now in this time, houses, and brethren, and sisters, and
mothers, and children, and lands, with persecutions; and in the world to come eternal life.

What is Christ’s promise of reward? What lessons can we learn from this? Just because you
give up everything to follow Christ it doesn’t mean that you will be a pauper your whole life on this
earth. God blesses you with more. But we shouldn’t expect for more just because we sacrificed, then
that is wrong motive. Those that started out first in life may end up becoming last because of their
riches. And those that end up poorest and last in this world may end up first in the kingdom of God
because it is easier for a poor person to give up everything as there’s not much to give up.

10:31 But many that are first shall be last; and the last first.

http://breachrepairers.webs.com/ 374
Let us sit on your right and left hand (32-45)
10:32 And they were in the way going up to Jerusalem; and Jesus went before them: and they were
amazed; and as they followed, they were afraid. And he took again the twelve, and began to tell them
what things should happen unto him,

10:33 Saying, Behold, we go up to Jerusalem; and the Son of man shall be delivered unto the chief priests,
and unto the scribes; and they shall condemn him to death, and shall deliver him to the Gentiles:

10:34 And they shall mock him, and shall scourge him, and shall spit upon him, and shall kill him: and
the third day he shall rise again.

10:35 And James and John, the sons of Zebedee, come unto him, saying, Master, we would that thou
shouldest do for us whatsoever we shall desire.

10:36 And he said unto them, What would ye that I should do for you?

10:37 They said unto him, Grant unto us that we may sit, one on thy right hand, and the other on thy left
hand, in thy glory.

10:38 But Jesus said unto them, Ye know not what ye ask: can ye drink of the cup that I drink of? and be
baptized with the baptism that I am baptized with?

10:39 And they said unto him, We can. And Jesus said unto them, Ye shall indeed drink of the cup that I
drink of; and with the baptism that I am baptized withal shall ye be baptized:

10:40 But to sit on my right hand and on my left hand is not mine to give; but it shall be given to them for
whom it is prepared.

10:41 And when the ten heard it, they began to be much displeased with James and John.

10:42 But Jesus called them to him, and saith unto them, Ye know that they which are accounted to rule
over the Gentiles exercise lordship over them; and their great ones exercise authority upon them.

10:43 But so shall it not be among you: but whosoever will be great among you, shall be your minister:

10:44 And whosoever of you will be the chiefest, shall be servant of all.

10:45 For even the Son of man came not to be ministered unto, but to minister, and to give his life a
ransom for many.

Blind Bartimaeus (Son of David) (46-52)


10:46 And they came to Jericho: and as he went out of Jericho with his disciples and a great number of
people, blind Bartimaeus, the son of Timaeus, sat by the highway side begging.

10:47 And when he heard that it was Jesus of Nazareth, he began to cry out, and say, Jesus, thou Son of
David, have mercy on me.

10:48 And many charged him that he should hold his peace: but he cried the more a great deal, Thou Son
of David, have mercy on me.

10:49 And Jesus stood still, and commanded him to be called. And they call the blind man, saying unto
him, Be of good comfort, rise; he calleth thee.

http://breachrepairers.webs.com/ 375
10:50 And he, casting away his garment, rose, and came to Jesus.

10:51 And Jesus answered and said unto him, What wilt thou that I should do unto thee? The blind man
said unto him, Lord, that I might receive my sight.

10:52 And Jesus said unto him, Go thy way; thy faith hath made thee whole. And immediately he received
his sight, and followed Jesus in the way.

Summary
Verse 45 I believe is the verse which captures the theme of the book. Mark saves his final miracle
for this chapter. The blind man calls Jesus the Son of David preparing the way for Jesus to teach
what that title means. There are no more recorded miracles from chapter 11-16. Christ’s greatest
act of servitude is to lay down His life as a ransom for man!

http://breachrepairers.webs.com/ 376
Chapter 11
Chapter Outline
 Christ’s entry into Jerusalem (1-11)
 Curse of the fig tree (12-14)
 Cleansing the temple (15-18)
 Curse fig tree and have faith in God (19-26)
 By what authority do you do these things? (27-33)

Christ’s entry into Jerusalem (1-11)


11:1 And when they came nigh to Jerusalem, unto Bethphage and Bethany, at the mount of Olives, he
sendeth forth two of his disciples,

11:2 And saith unto them, Go your way into the village over against you: and as soon as ye be entered
into it, ye shall find a colt tied, whereon never man sat; loose him, and bring him.

11:3 And if any man say unto you, Why do ye this? say ye that the Lord hath need of him; and
straightway he will send him hither.

11:4 And they went their way, and found the colt tied by the door without in a place where two ways met;
and they loose him.

11:5 And certain of them that stood there said unto them, What do ye, loosing the colt?

11:6 And they said unto them even as Jesus had commanded: and they let them go.

11:7 And they brought the colt to Jesus, and cast their garments on him; and he sat upon him.

11:8 And many spread their garments in the way: and others cut down branches off the trees, and
strawed them in the way.

11:9 And they that went before, and they that followed, cried, saying, Hosanna; Blessed is he that cometh
in the name of the Lord:

11:10 Blessed be the kingdom of our father David, that cometh in the name of the Lord: Hosanna in the
highest.

11:11 And Jesus entered into Jerusalem, and into the temple: and when he had looked round about upon
all things, and now the eventide was come, he went out unto Bethany with the twelve.

Curse of the fig tree (12-14)


11:12 And on the morrow, when they were come from Bethany, he was hungry:

11:13 And seeing a fig tree afar off having leaves, he came, if haply he might find any thing thereon: and
when he came to it, he found nothing but leaves; for the time of figs was not yet.

11:14 And Jesus answered and said unto it, No man eat fruit of thee hereafter for ever. And his disciples
heard it.

http://breachrepairers.webs.com/ 377
Cleansing the temple (15-18)
11:15 And they come to Jerusalem: and Jesus went into the temple, and began to cast out them that sold
and bought in the temple, and overthrew the tables of the moneychangers, and the seats of them that
sold doves;

11:16 And would not suffer that any man should carry any vessel through the temple.

11:17 And he taught, saying unto them, Is it not written, My house shall be called of all nations the house
of prayer? but ye have made it a den of thieves.

11:18 And the scribes and chief priests heard it, and sought how they might destroy him: for they feared
him, because all the people was astonished at his doctrine.

Curse fig tree and have faith in God (19-26)


11:19 And when even was come, he went out of the city.

11:20 And in the morning, as they passed by, they saw the fig tree dried up from the roots.

11:21 And Peter calling to remembrance saith unto him, Master, behold, the fig tree which thou cursedst
is withered away.

11:22 And Jesus answering saith unto them, Have faith in God.

11:23 For verily I say unto you, That whosoever shall say unto this mountain, Be thou removed, and be
thou cast into the sea; and shall not doubt in his heart, but shall believe that those things which he saith
shall come to pass; he shall have whatsoever he saith.

11:24 Therefore I say unto you, What things soever ye desire, when ye pray, believe that ye receive them,
and ye shall have them.

11:25 And when ye stand praying, forgive, if ye have ought against any: that your Father also which is in
heaven may forgive you your trespasses.

11:26 But if ye do not forgive, neither will your Father which is in heaven forgive your trespasses.

By what authority do you do these things? (27-33)


11:27 And they come again to Jerusalem: and as he was walking in the temple, there come to him the
chief priests, and the scribes, and the elders,

11:28 And say unto him, By what authority doest thou these things? and who gave thee this authority to
do these things?

11:29 And Jesus answered and said unto them, I will also ask of you one question, and answer me, and I
will tell you by what authority I do these things.

11:30 The baptism of John, was it from heaven, or of men? answer me.

11:31 And they reasoned with themselves, saying, If we shall say, From heaven; he will say, Why then did
ye not believe him?

11:32 But if we shall say, Of men; they feared the people: for all men counted John, that he was a prophet
indeed.

http://breachrepairers.webs.com/ 378
11:33 And they answered and said unto Jesus, We cannot tell. And Jesus answering saith unto them,
Neither do I tell you by what authority I do these things.

http://breachrepairers.webs.com/ 379
Chapter 12
Chapter Outline
 Parable of the vineyard and the heir (1-12)
 Render unto Caesar (Question by Pharisees and Herodians) (13-17)
 Question on marriage and resurrection (Sadducee's) (18-27)
 Question on first commandment (Scribe) (28-34)
 Son of David, Beware of scribes, Two mites (35-45)

Parable of the vineyard and the heir (1-12)


12:1 And he began to speak unto them by parables. A certain man planted a vineyard, and set an hedge
about it, and digged a place for the winefat, and built a tower, and let it out to husbandmen, and went
into a far country.

12:2 And at the season he sent to the husbandmen a servant, that he might receive from the
husbandmen of the fruit of the vineyard.

12:3 And they caught him, and beat him, and sent him away empty.

12:4 And again he sent unto them another servant; and at him they cast stones, and wounded him in the
head, and sent him away shamefully handled.

12:5 And again he sent another; and him they killed, and many others; beating some, and killing some.

12:6 Having yet therefore one son, his wellbeloved, he sent him also last unto them, saying, They will
reverence my son.

12:7 But those husbandmen said among themselves, This is the heir; come, let us kill him, and the
inheritance shall be ours.

12:8 And they took him, and killed him, and cast him out of the vineyard.

12:9 What shall therefore the lord of the vineyard do? he will come and destroy the husbandmen, and
will give the vineyard unto others.

12:10 And have ye not read this scripture; The stone which the builders rejected is become the head of
the corner:

12:11 This was the Lord's doing, and it is marvellous in our eyes?

12:12 And they sought to lay hold on him, but feared the people: for they knew that he had spoken the
parable against them: and they left him, and went their way.

Render unto Caesar (Question by Pharisees and Herodians) (13-17)


12:13 And they send unto him certain of the Pharisees and of the Herodians, to catch him in his words.

12:14 And when they were come, they say unto him, Master, we know that thou art true, and carest for
no man: for thou regardest not the person of men, but teachest the way of God in truth: Is it lawful to
give tribute to Caesar, or not?

http://breachrepairers.webs.com/ 380
12:15 Shall we give, or shall we not give? But he, knowing their hypocrisy, said unto them, Why tempt ye
me? bring me a penny, that I may see it.

12:16 And they brought it. And he saith unto them, Whose is this image and superscription? And they
said unto him, Caesar's.

12:17 And Jesus answering said unto them, Render to Caesar the things that are Caesar's, and to God the
things that are God's. And they marvelled at him.

Question on marriage and resurrection (Sadducee's) (18-27)


12:18 Then come unto him the Sadducees, which say there is no resurrection; and they asked him,
saying,

12:19 Master, Moses wrote unto us, If a man's brother die, and leave his wife behind him, and leave no
children, that his brother should take his wife, and raise up seed unto his brother.

12:20 Now there were seven brethren: and the first took a wife, and dying left no seed.

12:21 And the second took her, and died, neither left he any seed: and the third likewise.

12:22 And the seven had her, and left no seed: last of all the woman died also.

12:23 In the resurrection therefore, when they shall rise, whose wife shall she be of them? for the seven
had her to wife.

12:24 And Jesus answering said unto them, Do ye not therefore err, because ye know not the scriptures,
neither the power of God?

12:25 For when they shall rise from the dead, they neither marry, nor are given in marriage; but are as
the angels which are in heaven.

12:26 And as touching the dead, that they rise: have ye not read in the book of Moses, how in the bush
God spake unto him, saying, I am the God of Abraham, and the God of Isaac, and the God of Jacob?

12:27 He is not the God of the dead, but the God of the living: ye therefore do greatly err.

Question on first commandment (Scribe) (28-34)


12:28 And one of the scribes came, and having heard them reasoning together, and perceiving that he
had answered them well, asked him, Which is the first commandment of all?

12:29 And Jesus answered him, The first of all the commandments is, Hear, O Israel; The Lord our God is
one Lord:

12:30 And thou shalt love the Lord thy God with all thy heart, and with all thy soul, and with all thy mind,
and with all thy strength: this is the first commandment.

12:31 And the second is like, namely this, Thou shalt love thy neighbour as thyself. There is none other
commandment greater than these.

12:32 And the scribe said unto him, Well, Master, thou hast said the truth: for there is one God; and there
is none other but he:

12:33 And to love him with all the heart, and with all the understanding, and with all the soul, and with

http://breachrepairers.webs.com/ 381
all the strength, and to love his neighbour as himself, is more than all whole burnt offerings and
sacrifices.

12:34 And when Jesus saw that he answered discreetly, he said unto him, Thou art not far from the
kingdom of God. And no man after that durst ask him any question.

Son of David, Beware of scribes, Two mites (35-45)


12:35 And Jesus answered and said, while he taught in the temple, How say the scribes that Christ is the
Son of David?

12:36 For David himself said by the Holy Ghost, The LORD said to my Lord, Sit thou on my right hand, till
I make thine enemies thy footstool.

12:37 David therefore himself calleth him Lord; and whence is he then his son? And the common people
heard him gladly.

12:38 And he said unto them in his doctrine, Beware of the scribes, which love to go in long clothing, and
love salutations in the marketplaces,

12:39 And the chief seats in the synagogues, and the uppermost rooms at feasts:

12:40 Which devour widows' houses, and for a pretence make long prayers: these shall receive greater
damnation.

12:41 And Jesus sat over against the treasury, and beheld how the people cast money into the treasury:
and many that were rich cast in much.

12:42 And there came a certain poor widow, and she threw in two mites, which make a farthing.

12:43 And he called unto him his disciples, and saith unto them, Verily I say unto you, That this poor
widow hath cast more in, than all they which have cast into the treasury:

12:44 For all they did cast in of their abundance; but she of her want did cast in all that she had, even all
her living.

Summary
One word that sums up this whole chapter is questions. Questions on punishment of the
husbandman, tribute, marriage and resurrection, the first commandment, and the Son of David.

http://breachrepairers.webs.com/ 382
Chapter 13
Chapter Outline
 Not one stone left for the temple (1-4)
 Signs of Christ's coming (5-13)
 Abomination of Desolation (14-23)
o Don't return to your house
o Pray that flight is not in winter
o Days shortened for the elect
o Personation of Christ
 Time frame, manner, and purpose of Coming (24-31)
o After these things
o Coming with power and glory
o Coming to deliver elect
 Warning to watch and be ready for Coming (32-37)

Not one stone left for the temple (1-4)


13:1 And as he went out of the temple, one of his disciples saith unto him, Master, see what manner of
stones and what buildings are here!

13:2 And Jesus answering said unto him, Seest thou these great buildings? there shall not be left one
stone upon another, that shall not be thrown down.

13:3 And as he sat upon the mount of Olives over against the temple, Peter and James and John and
Andrew asked him privately,

13:4 Tell us, when shall these things be? and what shall be the sign when all these things shall be
fulfilled?

Signs of Christ's coming (5-13)


13:5 And Jesus answering them began to say, Take heed lest any man deceive you:

13:6 For many shall come in my name, saying, I am Christ; and shall deceive many.

13:7 And when ye shall hear of wars and rumours of wars, be ye not troubled: for such things must needs
be; but the end shall not be yet.

13:8 For nation shall rise against nation, and kingdom against kingdom: and there shall be earthquakes
in divers places, and there shall be famines and troubles: these are the beginnings of sorrows.

13:9 But take heed to yourselves: for they shall deliver you up to councils; and in the synagogues ye shall
be beaten: and ye shall be brought before rulers and kings for my sake, for a testimony against them.

13:10 And the gospel must first be published among all nations.

http://breachrepairers.webs.com/ 383
13:11 But when they shall lead you, and deliver you up, take no thought beforehand what ye shall speak,
neither do ye premeditate: but whatsoever shall be given you in that hour, that speak ye: for it is not ye
that speak, but the Holy Ghost.

13:12 Now the brother shall betray the brother to death, and the father the son; and children shall rise
up against their parents, and shall cause them to be put to death.

13:13 And ye shall be hated of all men for my name's sake: but he that shall endure unto the end, the
same shall be saved.

Abomination of Desolation (14-23)


13:14 But when ye shall see the abomination of desolation, spoken of by Daniel the prophet, standing
where it ought not, (let him that readeth understand,) then let them that be in Judaea flee to the
mountains:

13:15 And let him that is on the housetop not go down into the house, neither enter therein, to take any
thing out of his house:

13:16 And let him that is in the field not turn back again for to take up his garment.

13:17 But woe to them that are with child, and to them that give suck in those days!

13:18 And pray ye that your flight be not in the winter.

13:19 For in those days shall be affliction, such as was not from the beginning of the creation which God
created unto this time, neither shall be.

13:20 And except that the Lord had shortened those days, no flesh should be saved: but for the elect's
sake, whom he hath chosen, he hath shortened the days.

13:21 And then if any man shall say to you, Lo, here is Christ; or, lo, he is there; believe him not:

13:22 For false Christs and false prophets shall rise, and shall shew signs and wonders, to seduce, if it
were possible, even the elect.

13:23 But take ye heed: behold, I have foretold you all things.

Time frame, manner, and purpose of Coming (24-31)


13:24 But in those days, after that tribulation, the sun shall be darkened, and the moon shall not give her
light,

13:25 And the stars of heaven shall fall, and the powers that are in heaven shall be shaken.

13:26 And then shall they see the Son of man coming in the clouds with great power and glory.

13:27 And then shall he send his angels, and shall gather together his elect from the four winds, from the
uttermost part of the earth to the uttermost part of heaven.

13:28 Now learn a parable of the fig tree; When her branch is yet tender, and putteth forth leaves, ye
know that summer is near:

13:29 So ye in like manner, when ye shall see these things come to pass, know that it is nigh, even at the
doors.

http://breachrepairers.webs.com/ 384
13:30 Verily I say unto you, that this generation shall not pass, till all these things be done.

13:31 Heaven and earth shall pass away: but my words shall not pass away.

Warning to watch and be ready for Coming (32-37)


13:32 But of that day and that hour knoweth no man, no, not the angels which are in heaven, neither the
Son, but the Father.

13:33 Take ye heed, watch and pray: for ye know not when the time is.

13:34 For the Son of man is as a man taking a far journey, who left his house, and gave authority to his
servants, and to every man his work, and commanded the porter to watch.

13:35 Watch ye therefore: for ye know not when the master of the house cometh, at even, or at midnight,
or at the cockcrowing, or in the morning:

13:36 Lest coming suddenly he find you sleeping.

13:37 And what I say unto you I say unto all, Watch.

http://breachrepairers.webs.com/ 385
Chapter 14
Satan was pressing upon Jesus that no appreciation was given for the sacrifice that Jesus was giving.
It was a test on Christ where He had to go in faith on His Father. He couldn’t rely on any external
source.

DA 687 The people who claim to be above all others in temporal and spiritual advantages have
rejected You. They are seeking to destroy You, the foundation, the center and seal of the promises
made to them as a peculiar people. One of Your own disciples, who has listened to Your instruction,
and has been among the foremost in church activities, will betray You. One of Your most zealous
followers will deny You. All will forsake You. Christ's whole being abhorred the thought. That those
whom He had undertaken to save, those whom He loved so much, should unite in the plots of Satan,
this pierced His soul. The conflict was terrible. Its measure was the guilt of His nation, of His accusers
and betrayer, the guilt of a world lying in wickedness. The sins of men weighed heavily upon Christ,
and the sense of God's wrath against sin was crushing out His life.

DA 687 How dark seemed the malignity of sin! Terrible was the temptation to let the human race
bear the consequences of its own guilt, while He stood innocent before God. If He could only know
that His disciples understood and appreciated this, He would be strengthened.

DA 690 The humanity of the Son of God trembled in that trying hour. He prayed not now for His
disciples that their faith might not fail, but for His own tempted, agonized soul. The awful moment
had come--that moment which was to decide the destiny of the world. The fate of humanity trembled
in the balance. Christ might even now refuse to drink the cup apportioned to guilty man.

What would have happened if Christ had gone back? Would it have been wrong? It wasn’t wrong.
Justice demands that the guilty receive their punishment. There would always be a question about
God’s character and angels and other unfallen beings could doubt God’s character.

DA 690 Three times has He uttered that prayer. Three times has humanity shrunk from the last,
crowning sacrifice.

DA 693 Having made the decision, He fell dying to the ground from which He had partially risen.
Where now were His disciples, to place their hands tenderly beneath the head of their fainting
Master, and bathe that brow, marred indeed more than the sons of men? The Saviour trod the wine
press alone, and of the people there was none with Him.

DA 693 In this awful crisis, when everything was at stake, when the mysterious cup trembled in the
hand of the sufferer, the heavens opened, a light shone forth amid the stormy darkness of the crisis
hour, and the mighty angel who stands in God's presence, occupying the position from which Satan
fell, came to the side of Christ. The angel came not to take the cup from Christ's hand, but to
strengthen Him to drink it, with the assurance of the Father's love.

Chapter Outline
 Simon’s feast (3-9)

14:1 After two days was the feast of the passover, and of unleavened bread: and the chief priests and the
scribes sought how they might take him by craft, and put him to death.

14:2 But they said, Not on the feast day, lest there be an uproar of the people.

14:3 And being in Bethany in the house of Simon the leper, as he sat at meat, there came a woman
having an alabaster box of ointment of spikenard very precious; and she brake the box, and poured it on

http://breachrepairers.webs.com/ 386
his head.

14:4 And there were some that had indignation within themselves, and said, Why was this waste of the
ointment made?

14:5 For it might have been sold for more than three hundred pence, and have been given to the poor.
And they murmured against her.

14:6 And Jesus said, Let her alone; why trouble ye her? she hath wrought a good work on me.

14:7 For ye have the poor with you always, and whensoever ye will ye may do them good: but me ye have
not always.

14:8 She hath done what she could: she is come aforehand to anoint my body to the burying.

14:9 Verily I say unto you, Wheresoever this gospel shall be preached throughout the whole world, this
also that she hath done shall be spoken of for a memorial of her.

14:10 And Judas Iscariot, one of the twelve, went unto the chief priests, to betray him unto them.

14:11 And when they heard it, they were glad, and promised to give him money. And he sought how he
might conveniently betray him.

14:12 And the first day of unleavened bread, when they killed the passover, his disciples said unto him,
Where wilt thou that we go and prepare that thou mayest eat the passover?

14:13 And he sendeth forth two of his disciples, and saith unto them, Go ye into the city, and there shall
meet you a man bearing a pitcher of water: follow him.

14:14 And wheresoever he shall go in, say ye to the goodman of the house, The Master saith, Where is the
guestchamber, where I shall eat the passover with my disciples?

14:15 And he will shew you a large upper room furnished and prepared: there make ready for us.

14:16 And his disciples went forth, and came into the city, and found as he had said unto them: and they
made ready the passover.

14:17 And in the evening he cometh with the twelve.

14:18 And as they sat and did eat, Jesus said, Verily I say unto you, One of you which eateth with me shall
betray me.

14:19 And they began to be sorrowful, and to say unto him one by one, Is it I? and another said, Is it I?

14:20 And he answered and said unto them, It is one of the twelve, that dippeth with me in the dish.

14:21 The Son of man indeed goeth, as it is written of him: but woe to that man by whom the Son of man
is betrayed! good were it for that man if he had never been born.

14:22 And as they did eat, Jesus took bread, and blessed, and brake it, and gave to them, and said, Take,
eat: this is my body.

14:23 And he took the cup, and when he had given thanks, he gave it to them: and they all drank of it.

14:24 And he said unto them, This is my blood of the new testament, which is shed for many.

http://breachrepairers.webs.com/ 387
14:25 Verily I say unto you, I will drink no more of the fruit of the vine, until that day that I drink it new
in the kingdom of God.

14:26 And when they had sung an hymn, they went out into the mount of Olives.

14:27 And Jesus saith unto them, All ye shall be offended because of me this night: for it is written, I will
smite the shepherd, and the sheep shall be scattered.

14:28 But after that I am risen, I will go before you into Galilee.

14:29 But Peter said unto him, Although all shall be offended, yet will not I.

14:30 And Jesus saith unto him, Verily I say unto thee, That this day, even in this night, before the cock
crow twice, thou shalt deny me thrice.

14:31 But he spake the more vehemently, If I should die with thee, I will not deny thee in any wise.
Likewise also said they all.

14:32 And they came to a place which was named Gethsemane: and he saith to his disciples, Sit ye here,
while I shall pray.

14:33 And he taketh with him Peter and James and John, and began to be sore amazed, and to be very
heavy;

14:34 And saith unto them, My soul is exceeding sorrowful unto death: tarry ye here, and watch.

14:35 And he went forward a little, and fell on the ground, and prayed that, if it were possible, the hour
might pass from him.

14:36 And he said, Abba, Father, all things are possible unto thee; take away this cup from me:
nevertheless not what I will, but what thou wilt.

14:37 And he cometh, and findeth them sleeping, and saith unto Peter, Simon, sleepest thou? couldest not
thou watch one hour?

14:38 Watch ye and pray, lest ye enter into temptation. The spirit truly is ready, but the flesh is weak.

14:39 And again he went away, and prayed, and spake the same words.

14:40 And when he returned, he found them asleep again, (for their eyes were heavy,) neither wist they
what to answer him.

14:41 And he cometh the third time, and saith unto them, Sleep on now, and take your rest: it is enough,
the hour is come; behold, the Son of man is betrayed into the hands of sinners.

14:42 Rise up, let us go; lo, he that betrayeth me is at hand.

14:43 And immediately, while he yet spake, cometh Judas, one of the twelve, and with him a great
multitude with swords and staves, from the chief priests and the scribes and the elders.

14:44 And he that betrayed him had given them a token, saying, Whomsoever I shall kiss, that same is
he; take him, and lead him away safely.

14:45 And as soon as he was come, he goeth straightway to him, and saith, Master, master; and kissed

http://breachrepairers.webs.com/ 388
him.

14:46 And they laid their hands on him, and took him.

14:47 And one of them that stood by drew a sword, and smote a servant of the high priest, and cut off his
ear.

14:48 And Jesus answered and said unto them, Are ye come out, as against a thief, with swords and with
staves to take me?

14:49 I was daily with you in the temple teaching, and ye took me not: but the scriptures must be
fulfilled.

14:50 And they all forsook him, and fled.

14:51 And there followed him a certain young man, having a linen cloth cast about his naked body; and
the young men laid hold on him:

14:52 And he left the linen cloth, and fled from them naked.

14:53 And they led Jesus away to the high priest: and with him were assembled all the chief priests and
the elders and the scribes.

14:54 And Peter followed him afar off, even into the palace of the high priest: and he sat with the
servants, and warmed himself at the fire.

14:55 And the chief priests and all the council sought for witness against Jesus to put him to death; and
found none.

14:56 For many bare false witness against him, but their witness agreed not together.

14:57 And there arose certain, and bare false witness against him, saying,

When the witnesses were brought in, they were brought in one at a time. In the mouth of two or
three witnesses shall everything be established. None could match. They were trying to get Him on
charges of sedition against the Roman government. The second one blasphemy.

DA 699 Christ read the priest's purpose as an open book.

DA 698 His counsel was sought and carried out as the voice of God.

DA 699 Turning upon His questioner, Jesus said, "Why askest thou Me?" Had not the priests and
rulers sent spies to watch His movements, and report His every word? Had not these been present at
every gathering of the people, and carried to the priests information of all His sayings and doings?
"Ask them which heard Me, what I have said unto them," replied Jesus; "behold, they know what I
said."

We know this court was done at night (Friday morning). Usually, it was done after the morning
sacrifice. Nicodemus and Joseph of Arimathaea were not present because they knew they would
defend Jesus.

DA 700 Christ suffered keenly under abuse and insult. At the hands of the beings whom He had
created, and for whom He was making an infinite sacrifice, He received every indignity. And He
suffered in proportion to the perfection of His holiness and His hatred of sin.

http://breachrepairers.webs.com/ 389
If Christ were to flash forth His divinity, everything would have been destroyed. The greatest
temptation comes to those who have the power to do so but do not. All who are godly in Christ
Jesus will suffer persecution. If you are not having persecution, chances are you are not living a
Godly life.

DA 704 Caiaphas had regarded Jesus as his rival. The eagerness of the people to hear the Saviour,
and their apparent readiness to accept His teachings, had aroused the bitter jealousy of the high
priest. But as Caiaphas now looked upon the prisoner, he was struck with admiration for His noble
and dignified bearing. A conviction came over him that this Man was akin to God. The next instant he
scornfully banished the thought.

DA 707 Never in afterlife did he forget that searching glance of the persecuted Son of God.

14:58 We heard him say, I will destroy this temple that is made with hands, and within three days I will
build another made without hands.

14:59 But neither so did their witness agree together.

14:60 And the high priest stood up in the midst, and asked Jesus, saying, Answerest thou nothing? what
is it which these witness against thee?

14:61 But he held his peace, and answered nothing. Again the high priest asked him, and said unto him,
Art thou the Christ, the Son of the Blessed?

14:62 And Jesus said, I am: and ye shall see the Son of man sitting on the right hand of power, and
coming in the clouds of heaven.

14:63 Then the high priest rent his clothes, and saith, What need we any further witnesses?

14:64 Ye have heard the blasphemy: what think ye? And they all condemned him to be guilty of death.

14:65 And some began to spit on him, and to cover his face, and to buffet him, and to say unto him,
Prophesy: and the servants did strike him with the palms of their hands.

14:66 And as Peter was beneath in the palace, there cometh one of the maids of the high priest:

14:67 And when she saw Peter warming himself, she looked upon him, and said, And thou also wast with
Jesus of Nazareth.

14:68 But he denied, saying, I know not, neither understand I what thou sayest. And he went out into the
porch; and the cock crew.

14:69 And a maid saw him again, and began to say to them that stood by, This is one of them.

14:70 And he denied it again. And a little after, they that stood by said again to Peter, Surely thou art one
of them: for thou art a Galilaean, and thy speech agreeth thereto.

14:71 But he began to curse and to swear, saying, I know not this man of whom ye speak.

14:72 And the second time the cock crew. And Peter called to mind the word that Jesus said unto him,
Before the cock crow twice, thou shalt deny me thrice. And when he thought thereon, he wept.

http://breachrepairers.webs.com/ 390
Chapter 15
Chapter Outline

15:1 And straightway in the morning the chief priests held a consultation with the elders and scribes
and the whole council, and bound Jesus, and carried him away, and delivered him to Pilate.

15:2 And Pilate asked him, Art thou the King of the Jews? And he answering said unto him, Thou sayest it.

15:3 And the chief priests accused him of many things: but he answered nothing.

15:4 And Pilate asked him again, saying, Answerest thou nothing? behold how many things they witness
against thee.

15:5 But Jesus yet answered nothing; so that Pilate marvelled.

15:6 Now at that feast he released unto them one prisoner, whomsoever they desired.

15:7 And there was one named Barabbas, which lay bound with them that had made insurrection with
him, who had committed murder in the insurrection.

15:8 And the multitude crying aloud began to desire him to do as he had ever done unto them.

15:9 But Pilate answered them, saying, Will ye that I release unto you the King of the Jews?

15:10 For he knew that the chief priests had delivered him for envy.

15:11 But the chief priests moved the people, that he should rather release Barabbas unto them.

15:12 And Pilate answered and said again unto them, What will ye then that I shall do unto him whom ye
call the King of the Jews?

15:13 And they cried out again, Crucify him.

15:14 Then Pilate said unto them, Why, what evil hath he done? And they cried out the more exceedingly,
Crucify him.

15:15 And so Pilate, willing to content the people, released Barabbas unto them, and delivered Jesus,
when he had scourged him, to be crucified.

15:16 And the soldiers led him away into the hall, called Praetorium; and they call together the whole
band.

15:17 And they clothed him with purple, and platted a crown of thorns, and put it about his head,

15:18 And began to salute him, Hail, King of the Jews!

15:19 And they smote him on the head with a reed, and did spit upon him, and bowing their knees
worshipped him.

15:20 And when they had mocked him, they took off the purple from him, and put his own clothes on him,
and led him out to crucify him.

http://breachrepairers.webs.com/ 391
15:21 And they compel one Simon a Cyrenian, who passed by, coming out of the country, the father of
Alexander and Rufus, to bear his cross.

15:22 And they bring him unto the place Golgotha, which is, being interpreted, The place of a skull.

15:23 And they gave him to drink wine mingled with myrrh: but he received it not.

15:24 And when they had crucified him, they parted his garments, casting lots upon them, what every
man should take.

15:25 And it was the third hour, and they crucified him.

15:26 And the superscription of his accusation was written over, THE KING OF THE JEWS.

15:27 And with him they crucify two thieves; the one on his right hand, and the other on his left.

15:28 And the scripture was fulfilled, which saith, And he was numbered with the transgressors.

15:29 And they that passed by railed on him, wagging their heads, and saying, Ah, thou that destroyest
the temple, and buildest it in three days,

15:30 Save thyself, and come down from the cross.

15:31 Likewise also the chief priests mocking said among themselves with the scribes, He saved others;
himself he cannot save.

15:32 Let Christ the King of Israel descend now from the cross, that we may see and believe. And they
that were crucified with him reviled him.

15:33 And when the sixth hour was come, there was darkness over the whole land until the ninth hour.

15:34 And at the ninth hour Jesus cried with a loud voice, saying, Eloi, Eloi, lama sabachthani? which is,
being interpreted, My God, my God, why hast thou forsaken me?

15:35 And some of them that stood by, when they heard it, said, Behold, he calleth Elias.

15:36 And one ran and filled a spunge full of vinegar, and put it on a reed, and gave him to drink, saying,
Let alone; let us see whether Elias will come to take him down.

15:37 And Jesus cried with a loud voice, and gave up the ghost.

15:38 And the veil of the temple was rent in twain from the top to the bottom.

15:39 And when the centurion, which stood over against him, saw that he so cried out, and gave up the
ghost, he said, Truly this man was the Son of God.

15:40 There were also women looking on afar off: among whom was Mary Magdalene, and Mary the
mother of James the less and of Joses, and Salome;

15:41 (Who also, when he was in Galilee, followed him, and ministered unto him;) and many other
women which came up with him unto Jerusalem.

15:42 And now when the even was come, because it was the preparation, that is, the day before the
sabbath,

http://breachrepairers.webs.com/ 392
15:43 Joseph of Arimathaea, an honourable counsellor, which also waited for the kingdom of God, came,
and went in boldly unto Pilate, and craved the body of Jesus.

15:44 And Pilate marvelled if he were already dead: and calling unto him the centurion, he asked him
whether he had been any while dead.

15:45 And when he knew it of the centurion, he gave the body to Joseph.

15:46 And he bought fine linen, and took him down, and wrapped him in the linen, and laid him in a
sepulchre which was hewn out of a rock, and rolled a stone unto the door of the sepulchre.

15:47 And Mary Magdalene and Mary the mother of Joses beheld where he was laid.

http://breachrepairers.webs.com/ 393
Chapter 16
Chapter Outline
 Mary, Mary, and Salome tell Peter the Christ is risen (1-8)
 Disciples don't believe Mary and the two disciples (9-13)
 Gospel commission to believe and cast out demons (14-20)

Mary, Mary, and Salome tell Peter the Christ is risen (1-8)
16:1 And when the sabbath was past, Mary Magdalene, and Mary the mother of James, and Salome, had
bought sweet spices, that they might come and anoint him.

Bringing the Spices (Not ready for His death)

Mark 16:1

Spices were brought after the Sabbath was past. Why? Note: It was the Jews custom to use Myrrh &
Aloe to embalm the body of those who died

Luke 23:54-56; 24:1

Prepared spices right before Sabbath came Note: These verses show that it took time to prepare
these spices. Perhaps almost an hour. They also needed at least 100 lbs. It was expensive so maybe
they needed to pull all of their funds together to buy it.

Using the Spices

John 19:39-40

Spices ready to anoint the body of Jesus

John 19:39

100 lbs weight – took time to make

Significance of the Spices

Jn 3:14

Jesus spoke of His death to Nicodemus

Jn 7:48, 50-51

Nicodemus knew they wanted to kill him. App: Those who are the closest to Jesus and hear His
words the most are the most prone to not be ready for the crises. Nicodemus was not in the inner
circle. He didn’t hear the words of Jesus every day. Jesus told the disciples several times He would
die. He told Nicodemus once that he would die.

Appeal/Con

http://breachrepairers.webs.com/ 394
Matt 2:11

Three wise men brought spices. They were ready for the 1st Coming.

Jn 19:39

Nicodemus was ready for His death. Spices used in sanctuary work. Who will be ready for the 2nd
coming? Those who have the spices, they are involved in the sanctuary work

Miscellaneous: Myrrh Gk 4666 - myrrh, a bitter gum and costly perfume which exudes from a
certain tree or shrub in Arabia and Ethiopia, or is obtained by incisions made in the bark: as an
antiseptic it was used for embalming3. Antiseptic - Capable of preventing infection by inhibiting the
growth of microorganisms. Embalming - 1. To treat (a corpse) with preservatives in order to
prevent decay. 3. To impart fragrance to; perfume: Spicy aromas embalmed the air.

Bible “Myrrh.” Principle ingredient for the holy anointing oil used in the Sanctuary.

Ex 30:23

Brought as a gift from the Wise men from the east.

Matt 2:11

16:2 And very early in the morning the first day of the week, they came unto the sepulchre at the rising
of the sun.

16:3 And they said among themselves, Who shall roll us away the stone from the door of the sepulchre?

16:4 And when they looked, they saw that the stone was rolled away: for it was very great.

16:5 And entering into the sepulchre, they saw a young man sitting on the right side, clothed in a long
white garment; and they were affrighted.

16:6 And he saith unto them, Be not affrighted: Ye seek Jesus of Nazareth, which was crucified: he is
risen; he is not here: behold the place where they laid him.

16:7 But go your way, tell his disciples and Peter that he goeth before you into Galilee: there shall ye see
him, as he said unto you.

16:8 And they went out quickly, and fled from the sepulchre; for they trembled and were amazed: neither
said they any thing to any man; for they were afraid.

Disciples don't believe Mary and the two disciples (9-13)


16:9 Now when Jesus was risen early the first day of the week, he appeared first to Mary Magdalene, out
of whom he had cast seven devils.

5BC 1114 Only the Father Could Release Christ.--He who died for the sins of the world was to remain

3Strong, J. 1996. The exhaustive concordance of the Bible : Showing every word of the test of the common English
version of the canonical books, and every occurence of each word in regular order. (electronic ed.) . Woodside
Bible Fellowship.: Ontario

http://breachrepairers.webs.com/ 395
in the tomb the allotted time. He was in that stony prison house as a prisoner of divine justice. He was
responsible to the Judge of the universe. He was bearing the sins of the world, and His Father only
could release Him. A strong guard of mighty angels kept watch over the tomb, and had a hand been
raised to remove the body, the flashing forth of their glory would have laid him who ventured
powerless on the earth.

MS 94, 1897 There was only one entrance to the tomb, and neither human force nor fraud could
tamper with the stone that guarded the entrance. Here Jesus rested during the Sabbath. But prophecy
had pointed out that on the third day Christ would rise from the dead. Christ Himself had assured His
disciples of this. "Destroy this temple," He said, "and in three days I will raise it up." Christ never
committed sin, neither was guile found in His mouth. His body was to come forth from the tomb
untarnished by corruption.

Ps 16:10 For thou wilt not leave my soul in hell; neither wilt thou suffer thine Holy One to see
corruption.

Heb 7845 Corruption = corruption 4, pit 14, destruction 2, grave 1; 23 (Pit for catching
lions, or of Sheol).

Ps 49:9 That he should still live for ever, [and] not see corruption.

Greek 1312 Corruption = corruption 6; 6 corruption, destruction. In the NT that destruction


which is effected by the decay of the body after death.

Acts 2:27

Acts 2:31

Acts 13:34-37

5BC 1113 (John 1:1-3, 14; Phil. 2:5-8; Col. 2:9; Heb. 1:6, 8; 2:14-17; 4:15). Deity Did Not Die.--Was
the human nature of the Son of Mary changed into the divine nature of the Son of God? No; the two
natures were mysteriously blended in one person--the man Christ Jesus. In Him dwelt all the fullness
of the Godhead bodily. When Christ was crucified, it was His human nature that died. Deity did not
sink and die; that would have been impossible. Christ, the sinless One, will save every son and
daughter of Adam who accepts the salvation proffered them, consenting to become the children of
God. The Saviour has purchased the fallen race with His own blood. This is a great mystery, a mystery
that will not be fully, completely understood in all its greatness until the translation of the redeemed
shall take place. Then the power and greatness and efficacy of the gift of God to man will be
understood. But the enemy is determined that this gift shall be so mystified that it will become as
nothingness (Letter 280, 1904). {.3}

5BC 1113 (Matt. 28:5, 6; Luke 24:5, 6; John 2:19; 10:17, 18; Acts 13:32, 33.) When the voice of the
angel was heard saying, "Thy Father calls thee," He who had said, "I lay down my life, that I might
take it again," "Destroy this temple, and in three days I will raise it up," came forth from the grave to
life that was in Himself. Deity did not die. Humanity died, but Christ now proclaims over the rent
sepulcher of Joseph, "I am the resurrection, and the life." In His divinity Christ possessed the
power to break the bonds of death. He declares that He had life in Himself to quicken whom
He will.

5BC 1113 I am the resurrection, and the life." He who had said, "I lay down my life, that I might take
it again," came forth from the grave to life that was in Himself. Humanity died: divinity did not die. In
His divinity, Christ possessed the power to break the bonds of death. He declares that He has life in
Himself to quicken whom He will. All created beings live by the will and power of God. They are

http://breachrepairers.webs.com/ 396
recipients of the life of the Son of God. However able and talented, however large their capacities,
they are replenished with life from the Source of all life. He is the spring, the fountain, of life. Only He
who alone hath immortality, dwelling in light and life, could say, "I have power to lay down my life,
and I have power to take it again." . . . Christ was invested with the right to give immortality. The life
which He had laid down in humanity, He again took up and gave to humanity. "I am come," He says,
"that they might have life, and that they might have it more abundantly" (YI Aug. 4, 1898).

16:10 And she went and told them that had been with him, as they mourned and wept.

16:11 And they, when they had heard that he was alive, and had been seen of her, believed not.

16:12 After that he appeared in another form unto two of them, as they walked, and went into the
country.

16:13 And they went and told it unto the residue: neither believed they them.

Gospel commission to believe and cast out demons (14-20)


16:14 Afterward he appeared unto the eleven as they sat at meat, and upbraided them with their
unbelief and hardness of heart, because they believed not them which had seen him after he was risen.

16:15 And he said unto them, Go ye into all the world, and preach the gospel to every creature.

16:16 He that believeth and is baptized shall be saved; but he that believeth not shall be damned.

16:17 And these signs shall follow them that believe; In my name shall they cast out devils; they shall
speak with new tongues;

16:18 They shall take up serpents; and if they drink any deadly thing, it shall not hurt them; they shall
lay hands on the sick, and they shall recover.

16:19 So then after the Lord had spoken unto them, he was received up into heaven, and sat on the right
hand of God.

16:20 And they went forth, and preached every where, the Lord working with them, and confirming the
word with signs following. Amen.

http://breachrepairers.webs.com/ 397
Luke - Humanity of Jesus Christ
Introduction
 Where did he get his material from?
o From others whom had written and seen – Luke 1:2, 3
 Luke 1:3
o Theophilus
 Theo – means God
 Philus – means friend
 So Theophilus means the friend of God – those who are the friends of God
 Nationality – a Gentile, a Greek
o Begins his genealogy from Adam
o Luke 3:1
o Tries to anchor the life of Jesus into secular history
o He was trying to reach the Gentile mind

Author: Luke = "light-giving." This whole book is an example of Luke giving light to Theophilus.
Theophilus was his student and high priest. Commonly believed that he came from Antioch. He was
a companion of Paul (2nd journey to Rome) both a physician and gentile. His literary style is Grecian.
He died at the age of 84 a martyr. Tradition states he was nailed to an olive tree. He sends this work
to Theophilus for examination and criticism. 6BC 1051

Author
The third Gospel does not name its author. This is also true of the book of Acts, the author's second
work. This does not mean, however, that the original readers did not know who wrote these two
books. The Lukan authorship of Luke–Acts is affirmed by both external evidence (church tradition)
and internal evidence. Church tradition supporting Luke as the author is both early (from the mid-
2nd century a.d.) and unanimous (it was never doubted until the 19th century). The “we” sections
of Acts (16:10–17; 20:5–21:18; 27:1–28:16) assume that the author was a companion of Paul and
participated in the events described in those sections. Thus the author of Acts was probably one of
Paul's companions listed in his letters written during those periods (Luke is listed in Col. 4:14; 2
Tim. 4:11; Philem. 24) and not one of those referred to in the third person in the “we” sections (cf.
Acts 20:4–5). It is known that the author was from the second generation of the early church, was
not an “eyewitness” of Jesus' ministry (Luke 1:2), and was a Gentile cf. Col. 4:14). All of this confirms
the tradition that Luke was the author of the third Gospel. Because Luke traveled with Paul, this
Gospel was received as having apostolic endorsement and authority from Paul and as a trustworthy
record of the gospel that Paul preached.

Date
The earliest possible date of Luke–Acts is immediately after the events that Luke recorded in Acts
28, c. a.d. 62. In fact, Luke could have been written slightly earlier, and Acts could have been
completed at that time. The specific date centers on two questions: would Luke have added to his
Gospel later, and did he make use of the Gospel of Mark in writing his own Gospel? If Luke wrote
Luke–Acts after the martyrdom of Paul (c. a.d. 64–67), some have suggested that the omission of the
details of Paul's trial and death seems strange (see Introduction to Acts: Date). In addition, Luke
makes no mention of the terrible persecution under Nero in a.d. 65 but gives a very positive picture
of Paul preaching the gospel in Rome for two years “with all boldness and without hindrance” (Acts

http://breachrepairers.webs.com/ 398
28:31), so he must have written sometime before 65. (Those who hold to a later date for Luke reply
that ending with Paul preaching in Rome is natural, because it shows that the preaching “to the end
of the earth” in Acts 1:8 has been fulfilled. Since a primary purpose of Luke–Acts is to tell the story
of how the gospel spread to the Gentile capital of Rome, when that purpose is accomplished the
story could come to a natural end, regardless of what happens to Paul.)

Concerning the relationship of Luke to Mark, the great majority of scholars believe that Luke made
use of Mark in writing his Gospel. There is no real difficulty in dating Mark in the mid- to late-50s
a.d. (see Introduction to Mark: Date and Location), which would allow for a date of Luke in the early
60s. Other scholars dispute this and claim that the predictions of the fall of Jerusalem in Luke
21:20–24 are so vivid that they could have been written only after the fact, which means that Luke
and Acts were written after a.d. 70. These scholars would then date Mark somewhat before Luke, in
the late 60s (after Peter's death). However, many evangelical scholars, who consider Luke 21:20–24
a predictive prophecy by Jesus, would hold to a date for Luke in the early 60s.

Theme
Luke wrote his Gospel so that his readers would understand that the gospel is for all, both Jews and
Gentiles alike, since Jesus is the promised one of God as prophesied in the OT and as attested
through God's saving activity in Jesus' life, death, and resurrection. In addition to this, Luke
emphasized the truthfulness of the Christian traditions his readers had been taught, so that by
believing in Jesus Christ, the Son of God, they would receive the promised Holy Spirit whom he
gives to all who follow him.

Shows His close connection with human needs.


Presents Him as a friend and partaker of humanity.

Observation – Of the 4 gospel writers Luke was the only Gentile. It is like the gospel account would
not be complete unless God choose a gentile to write it.

Luke 9:56 For the Son of man is not come to destroy men's lives, but to save [them].

Luke 19:10 For the Son of man is come to seek and to save that which was lost

Key Themes
1. God's sovereign rule over history. The promises God made through 13:33; 22:22, 42; Acts
the prophets are already being fulfilled. 1:16–17; 2:23; 4:28;
etc.
2. The arrival and actual presence of the kingdom of God. 11:2, 20; 16:16; 17:20–
Nevertheless, the consummation of the kingdom is still a future 21; 18:1–8; 21:27–28,
event, a blessed hope for which the church prays. 34–36; cf. Acts 1:11; 1
Cor. 16:22; Rev. 22:20
3. The coming and indwelling of the Holy Spirit upon Jesus and his 1:15–17, 35; 2:25–27;
followers. The Spirit is present in the Gospel of Luke, from the births 3:16, 22; 4:1, 14, 18;
of John the Baptist and Jesus to the end. The Spirit is present at 5:17; 24:49
Jesus' dedication in the temple, his baptism, temptation, early
ministry, and first sermon. The Holy Spirit is central to the message
of John the Baptist, and Jesus at his ascension promises the Spirit's
future coming in power.

http://breachrepairers.webs.com/ 399
4. The great reversal taking place in the world, in which the first are 1:48, 52–53; 6:20–26;
becoming last and the last are becoming first, the proud are being 13:30; 14:11; 18:14
brought low and the humble are being exalted. Luke places great
emphasis on God's love for the poor, tax collectors, outcasts, sinners,
women, Samaritans, and Gentiles. In keeping with this concern,
many of the episodes that appear only in Luke's Gospel feature the
welcome of an outcast (the Christmas shepherds, the Prodigal Son,
the persistent widow, Zacchaeus, etc.).
5. Believers are to live a life of prayer and practice good stewardship 3:21; 6:12; 9:18, 28–
with their possessions. In Luke's narrative, prayer occurs at every 29; 11:1–4; 12:33–34;
major point in Jesus' life: at his baptism; at his selection of the 16:9; 18:1; 22:32, 40,
Twelve; at Peter's confession; at Jesus' transfiguration; in his 46
teaching the Lord's Prayer; before Peter's denial; etc.
6. The danger of riches is constantly emphasized in Luke, for the love 6:20–26; 8:14; 12:13–
of riches chokes out the seed of the gospel and keeps it from 21; 16:10–13, 19–31;
becoming fruitful. This danger is so great that Jesus often warns his 18:22 (cf. 5:11; 14:33;
readers not to set their hearts upon riches and to give generously to Acts 2:44–45; 4:32);
the poor. The woes pronounced upon haughty rich people stand in Luke 21:3–4
sharp contrast to the blessings pronounced upon the humble poor.

Purpose, Occasion, and Background


Both Luke (1:3) and Acts (1:1) are addressed to “Theophilus,” and there is no reason to deny that he
was a real person, although attempts to identify him have been unsuccessful. Luke uses the same
description “most excellent” (Luke 1:3) in the book of Acts to describe the Roman governors Felix
(Acts 23:26; 24:2) and Festus (Acts 26:25). Theophilus was probably a man of wealth and social
standing, and “most excellent” served as a respectful form of address.

Luke's broader intended audience consisted primarily of Gentile Christians like Theophilus who
had already “been taught” (1:4) about Jesus. But Luke no doubt realized that his recounting of Jesus'
life and message would also be useful for evangelism among non-Christians. Luke probably had
several goals in writing:

1. To assure his readers of the certainty of what they had been taught. This is accomplished by
demonstrating his credentials as a historian (cf. 1:1–4, where Luke speaks of having “followed all
things closely for some time past” in order “to write an orderly account”). He also mentions that the
material he is sharing is well known (24:18; Acts 26:26). The fact that the material in Luke comes
from eyewitness testimony (Luke 1:2; 24:48; cf. Acts 1:8) further assures his readers that what they
were taught is certain. Luke also seeks to assure his readers by demonstrating that the events
recorded in Luke–Acts were the fulfillment of ancient prophecy (e.g., Luke 1:1; 3:4–6; 4:17–21;
7:22–23) and the fulfillment of Jesus' prophecies (e.g., 9:22, 44; 11:29–30; 13:32–34; 17:25; 18:31–
33).

2. To help his readers understand how Israel's rejection of Jesus and the Gentiles' entrance into the
kingdom of God are in accord with the divine plan. Luke emphasizes that Christianity is not a new
religion but rather the fulfillment and present-day expression of the religion of Abraham, Isaac, and
Jacob.

3. To clarify for his readers Jesus' teaching concerning the end times by showing that Jesus did not
teach that the parousia (return of Christ) would come immediately but that there would be a period

http://breachrepairers.webs.com/ 400
between his resurrection and his return (9:27; 19:11; 21:20–24; 22:69; Acts 1:6–9). Nevertheless,
Jesus would return (Luke 3:9, 17; 12:38–48; 18:8; 21:32) in bodily form (Acts 1:11), and believers
should live in watchful expectation (Luke 21:34–36).

4. To emphasize that his readers need not fear Rome. Luke hints at this theme by highlighting
Herod's and Pilate's desire to release Jesus and the Roman centurion's recognition of his innocence.
Luke also records (in Acts) several occasions where Roman authorities came to Paul's rescue. When
Roman officials did persecute, Luke explains that it was due to error and that the persecution
ceased immediately when the error was discovered (cf. Acts 16:22–39).

Location
Written in Achaia (Corinth) or Rome or Antioch. Around 63 A.D.

Distinctive Features
The Setting of Luke
The events in the book of Luke take place almost entirely within the
vicinity of Palestine, an area extending roughly from Caesarea Philippi
in the north to Beersheba in the south. During this time it was ruled by
the Roman Empire. The opening chapters describe events surrounding
Jesus' birth in Judea, where Herod had been appointed king by the
Romans. The closing chapters end with Jesus' death, resurrection, and
ascension during the rule of Pontius Pilate and the tetrarchs Antipas
and Philip.

Sermons
The Lord's Prayer (Takes you through the last scenes of Christ's life Luke 11)
The Lord's Prayer (Contextually, The Kingdom of God [Unity] Luke 11)
Three Parables of the Lost (Luke 15)
The Cup of Jesus (Luke 22 the Denial and Betrayal of Jesus)
The Trial of Jesus (Luke 23 Progression, reconcile, release of Bar)
Signs of His Return (Luke 21 False Christ's will deceive)

History of Salvation Summary


Jesus comes as the messianic King to deliver the poor and needy and downcast (4:18–19). He fulfills
the whole OT (24:44–47), especially its promises of everlasting salvation. The fulfillment of his
mission comes with his crucifixion and resurrection.

Outline
1. The Prologue (1:1–4)
2. The Infancy Narrative (1:5–2:52)
a. The birth of John the Baptist foretold (1:5–25)
b. The birth of Jesus foretold (1:26–38)
c. Mary visits Elizabeth (1:39–56)
d. The birth of John the Baptist (1:57–80)
e. The birth of Jesus Christ (2:1–52)
i. Jesus is born (2:1–20)

http://breachrepairers.webs.com/ 401
ii. Jesus presented in the temple (2:21–40)
iii. The boy Jesus in the temple (2:41–52)
3. Preparation for the Ministry of Jesus (3:1–4:15)
a. John the Baptist prepares the way (3:1–20)
b. Jesus' baptism, genealogy, and temptation (3:21–4:15)
i. Jesus' baptism (3:21–22)
ii. The genealogy of Jesus Christ (3:23–38)
iii. The temptation of Jesus (4:1–15)
4. The Ministry of Jesus in Galilee (4:16–9:50)
a. The beginning (4:16–5:16)
i. Jesus rejected at Nazareth (4:16–30)
ii. Jesus begins his healing ministry (4:31–41)
iii. Jesus preaches in synagogues (4:42–44)
iv. Jesus calls the first disciples (5:1–11)
v. Jesus cleanses a leper (5:12–16)
b. The beginning of controversy (5:17–6:11)
i. Jesus heals a paralytic (5:17–26)
ii. Jesus calls Levi (5:27–32)
iii. A question about fasting (5:33–39)
iv. Jesus is lord of the Sabbath (6:1–5)
v. A man with a withered hand (6:6–11)
c. Jesus teaches the disciples (6:12–49)
i. Jesus appoints twelve apostles (6:12–16)
ii. Jesus ministers to a great multitude (6:17–19)
iii. The Beatitudes (6:20–23)
iv. Jesus pronounces woes (6:24–26)
v. Love your enemies (6:27–36)
vi. Judging others (6:37–42)
vii. A tree and its fruit (6:43–45)
viii. Build your house on the rock (6:46–49)
d. Who is this Jesus? (7:1–50)
i. Jesus heals a centurion's servant (7:1–10)
ii. Jesus raises a widow's son (7:11–17)
iii. Messengers from John the Baptist (7:18–35)
iv. A sinful woman forgiven (7:36–50)
e. Jesus teaches in parables (8:1–21)
i. Women accompanying Jesus (8:1–3)
ii. The parable of the sower (8:4–8)
iii. The purpose of the parables (8:9–15)
iv. A lamp under a jar (8:16–18)
v. Jesus' mother and brothers (8:19–21)
f. Jesus, Lord of nature, demons, disease, and death (8:22–56)
i. Jesus calms a storm (8:22–25)
ii. Jesus heals a demon-possessed man (8:26–39)
iii. Jesus heals a woman and Jairus's daughter (8:40–56)
g. Jesus and the Twelve (9:1–50)
i. Jesus sends out the Twelve (9:1–6)
ii. Herod Antipas is perplexed by Jesus (9:7–9)
iii. Jesus feeds the 5,000 (9:10–17)
iv. Peter confesses Jesus as the Christ (9:18–20)

http://breachrepairers.webs.com/ 402
v. Jesus foretells his death (9:21–22)
vi. Jesus teaches the disciples (9:23–27)
vii. The transfiguration (9:28–36)
viii. The healing of a boy with an unclean spirit (9:37–43a)
ix. Jesus again foretells his death (9:43b–45)
x. Who is the greatest? (9:46–48)
xi. Anyone not against us is for us (9:49–50)
5. The Journey to Jerusalem (9:51–19:27)
a. The first mention of the journey to Jerusalem (9:51–13:21)
i. The mission to Samaria (9:51–56)
ii. The cost of following Jesus (9:57–62)
iii. The mission of the seventy-two (10:1–24)
iv. The parable of the good Samaritan (10:25–37)
v. Martha and Mary (10:38–42)
vi. The Lord's Prayer (11:1–13)
vii. Jesus and Beelzebul (11:14–23)
viii. The return of an unclean spirit (11:24–26)
ix. Various warnings and teachings (11:27–13:9)
x. Jesus heals on the Sabbath (13:10–17)
xi. The parables of the mustard seed and the leaven (13:18–21)
b. The second mention of the journey to Jerusalem (13:22–17:10)
i. The narrow door (13:22–30)
ii. Lament over Jerusalem (13:31–35)
iii. The healing of a man on the Sabbath (14:1–6)
iv. Various teachings and parables (14:7–17:10)
c. The third mention of the journey to Jerusalem (17:11–19:27)
i. Jesus cleanses ten lepers (17:11–19)
ii. The coming of the kingdom (17:20–37)
iii. The parable of the persistent widow (18:1–8)
iv. The parable of the Pharisee and the tax collector (18:9–14)
v. Jesus blesses the children (18:15–17)
vi. The rich ruler (18:18–30)
vii. Jesus foretells his death a third time (18:31–34)
viii. Jesus heals a blind beggar (18:35–43)
ix. Jesus and Zacchaeus (19:1–10)
x. The parable of the ten minas (19:11–27)
6. The Ministry of Jesus in Jerusalem (19:28–21:38)
a. The Triumphal Entry (19:28–40)
b. Jesus weeps over Jerusalem (19:41–44)
c. Jesus cleanses the temple (19:45–48)
d. The authority of Jesus challenged (20:1–8)
e. The parable of the wicked tenants (20:9–18)
f. Paying taxes to Caesar (20:19–26)
g. Sadducees ask about the resurrection (20:27–40)
h. Whose son is the Christ? (20:41–44)
i. Beware of the scribes (20:45–47)
j. The widow's offering (21:1–4)
k. Jesus foretells the destruction of the temple and Jerusalem (21:5–24)
i. Jesus foretells the destruction of the temple (21:5–6)
ii. Signs before the destruction (21:7–9)

http://breachrepairers.webs.com/ 403
iii. Nation will rise against nation (21:10–19)
iv. Jesus foretells the destruction of Jerusalem (21:20–24)
l. Jesus foretells the coming of the Son of Man (21:25–38)
i. The coming of the Son of Man (21:25–28)
ii. The lesson of the fig tree (21:29–33)
iii. Watch yourselves (21:34–38)
7. The Suffering and Death of Jesus (22:1–23:56)
a. The plot to kill Jesus and the Passover meal (22:1–38)
i. The plot to kill Jesus (22:1–6)
ii. Preparations for the Passover meal (22:7–13)
iii. The Passover meal and the institution of the Lord's Supper (22:14–23)
iv. Who is the greatest? (22:24–30)
v. Jesus foretells Peter's denial (22:31–34)
vi. Scripture must be fulfilled in Jesus (22:35–38)
b. The arrest and trial (22:39–23:56)
i. Jesus prays on the Mount of Olives (22:39–46)
ii. The betrayal and arrest of Jesus (22:47–53)
iii. Peter denies Jesus (22:54–62)
iv. Jesus is mocked (22:63–65)
v. Jesus before the council (22:66–71)
vi. Jesus before Pilate (23:1–5)
vii. Jesus before Herod Antipas (23:6–16)
viii. Pilate delivers Jesus to be crucified (23:18–25)
ix. The crucifixion (23:26–43)
x. The death of Jesus (23:44–49)
xi. Jesus is buried (23:50–56)
8. The Resurrection of Jesus (24:1–53)
a. The empty tomb (24:1–12)
b. Jesus' appearance on the road to Emmaus (24:13–35)
c. Jesus appears to his disciples (24:36–49)
d. The ascension of Jesus (24:50–53)

Chapter 1 - Gabriel's glad tidings of the birth of John & Jesus


Chapter 1 lays the foundation for the book.

1:9, 11 According to the custom of the priest's office, his lot was to burn incense when he
went into the temple of the Lord… altar of incense.

Luke 23:45 And the sun was darkened, and the veil of the temple was rent in the midst.

Greek 3485 Sanctuary (HP and MHP)

1:17 And he shall go before him in the spirit and power of Elias, to turn the hearts of the
fathers to the children, and the disobedient to the wisdom of the just; to make ready a people
prepared for the Lord.

Luke 2:40 And the child grew, and waxed strong in spirit, filled with wisdom: and the grace of God
was upon him.

http://breachrepairers.webs.com/ 404
Luke 2:52 And Jesus increased in wisdom and stature, and in favour with God and man

Luke 7:35 But wisdom is justified of all her children

Luke 11:31 The queen of the south shall rise up in the judgment with the men of this generation, and
condemn them: for she came from the utmost parts of the earth to hear the wisdom of Solomon; and,
behold, a greater than Solomon [is] here.

Luke 11:49 Therefore also said the wisdom of God, I will send them prophets and apostles, and
[some] of them they shall slay and persecute:

Luke 21:15 For I will give you a mouth and wisdom, which all your adversaries shall not be able to
gainsay nor resist.

1:32 He shall be great, and shall be called the [1]Son of the Highest: and the Lord God shall
give unto him the [2]throne of his [3] father David:

This verse shows that Jesus is both the Son of the Highest and the Son of David. It shows
that He is both Divine and human.

“Son of the Highest”

Luke 3:22 And the Holy Ghost descended in a bodily shape like a dove upon him, and a voice came
from heaven, which said, Thou art my beloved Son; in thee I am well pleased

Luke 4:41 And devils also came out of many, crying out, and saying, Thou art Christ the Son of God.
And he rebuking [them] suffered them not to speak: for they knew that he was Christ.

Luke 8:28 When he saw Jesus, he cried out, and fell down before him, and with a loud voice said,
What have I to do with thee, Jesus, [thou] Son of God most high? I beseech thee, torment me not.

Luke 9:35 And there came a voice out of the cloud, saying, This is my beloved Son: hear him.

Luke 22:70 Then said they all, Art thou then the Son of God? And he said unto them, Ye say that I
am.

“father David”

Luke 18:38 And he cried, saying, Jesus, [thou] son of David, have mercy on me.

Luke 18:39 And they which went before rebuked him, that he should hold his peace: but he cried so
much the more, [Thou] son of David, have mercy on me.

Luke 20:41 And he said unto them, How say they that Christ is David's son?

Luke 20:44 David theref ore calleth him Lord, how is he then his son?

“God shall give unto him the throne of his father David”

Luke 1:33 And he shall reign over the house of Jacob for ever; and of his kingdom there shall
be no end.

Is 9:6-7 [6] For unto us a child is born, unto us a son is given: and the government shall be upon his

http://breachrepairers.webs.com/ 405
shoulder: and his name shall be called Wonderful, Counsellor, The mighty God, The everlasting
Father, The Prince of Peace. [7] Of the increase of [his] government and peace [there shall be] no
end, upon the throne of David, and upon his kingdom, to order it, and to establish it with
judgment and with justice from henceforth even for ever. The zeal of the LORD of hosts will
perform this.

Dan 7:13-14 [13] I saw in the night visions, and, behold, [one] like the Son of man came with the
clouds of heaven, and came to the Ancient of days, and they brought him near before him. [14] And
there was given him dominion, and glory, and a kingdom, that all people, nations, and languages,
should serve him: his dominion [is] an everlasting dominion, which shall not pass away, and his
kingdom [that] which shall not be destroyed.

Luke 9:27 But I tell you of a truth, there be some standing here, which shall not taste of death, till
they see the kingdom of God.

Luke 11:2 When ye pray, say, Our Father which art in heaven, Hallowed be thy name. Thy kingdom
come.

Luke 13:29 nd they shall come from the east, and [from] the west, and from the north, and [from]
the south, and shall sit down in the kingdom of God

Luke 19:12 He said therefore, A certain nobleman went into a far country to receive for himself a
kingdom, and to return.

Luke 19:14 But his citizens hated him, and sent a message after him, saying, We will not have this
[man] to reign over us.

Luke 19:27 But those mine enemies, which would not that I should reign over them, bring hither,
and slay [them] before me.

Luke 22:29-30 And I appoint unto you a kingdom, as my Father hath appointed unto me; That ye
may eat and drink at my table in my kingdom, and sit on thrones judging the twelve tribes of Israel.

Luke 23:42 And he said unto Jesus, Lord, remember me when thou comest into thy kingdom.

Luke 1:50 And his mercy [is] on them that fear him from generation to generation

Luke 6:36 Be ye therefore merciful, as your Father also is merciful.

Luke 1:54-55 [54] He hath holpen his servant Israel, in remembrance of [his] mercy; [55]
As he spake to our fathers, to Abraham, and to his seed for ever.

Note: “that fear Him.” It shows that those who give the 3 angel's message are entitled to this
mercy. We are the last generation that will receive this mercy.

Luke 1:72 To perform the mercy [promised] to our fathers, and to remember his holy covenant;

Note: this covenant is described in verses 72-79

Deut 4:31 For the LORD thy God [is] a merciful God he will not..forget the covenant of thy fathers
which he sware unto them.

Luke 1:78, 77 Through the tender mercy of our God; whereby the dayspring from on high hath
visited us.. To give knowledge of salvation unto his people by the remission of their sins.

http://breachrepairers.webs.com/ 406
Luke 18:13-14 And the publican, standing afar off, would not lift up so much as [his] eyes unto
heaven, but smote upon his breast, saying, God be merciful to me a sinner. I tell you, this man went
down to his house justified [rather] than the other:

Extra:

Psa 67:1 God be merciful unto us, and bless us; [and] cause his face to shine upon us; Selah. (literally
Face to Face)

Ex 34:6-7 The LORD God, merciful.. Keeping mercy for thousands, forgiving iniquity and
transgression and sin, and that will by no means clear [the guilty]

Luke 6:36 Be ye therefore merciful, as your Father also is merciful.

Ps 18:25 With the merciful thou wilt show thyself merciful; (2 Sam 22:26, background)

Matt 5:7 Blessed [are] the merciful: for they shall obtain mercy.

Heb 4:16 Let us therefore come boldly unto the throne of grace, that we may obtain mercy, and find
grace to help in time of need

Heb 8:12 For I will be merciful to their unrighteousness, and their sins and their iniquities will I
remember no more.

Ps 51:1 Have mercy upon me, O God, according to thy lovingkindness: according unto the multitude
of thy tender mercies blot out my transgressions.

1 Pet 1:3-4 Blessed [be] the God and Father of our Lord Jesus Christ, which according to his
abundant mercy hath begotten us again unto a lively hope by the resurrection of Jesus Christ from
the dead, To an inheritance incorruptible, and undefiled, and that fadeth not away, reserved in
heaven for you.

Jude 1:21 Keep yourselves in the love of God, looking for the mercy of our Lord Jesus Christ unto
eternal life.

Titus 3:5

Note: Other things can be shown about John the Baptist, but I am focusing on Jesus.

Chapter Outline
 Ordered declaration of things believed among us (1-4)
 Gabriel sent to tell Zacharias that Elisabeth will have a son, John (5-25)
 Gabriel sent to tell Mary she will have a son, Jesus (26-38)
 Mary's salutation to Elisabeth (39-56)
 Elisabeth gives birth to John (57-66)
 Zachariah's prophecy concerning Jesus & John (67-80)

Ordered declaration of things believed among us (1-4)


1:1 Forasmuch as many have taken in hand to set forth in order adeclaration of those things which are

http://breachrepairers.webs.com/ 407
most surely
believed among us,

1:2 Even as they delivered them unto us, which from the beginning were eyewitnesses, and ministers of
the word;

1:3 It seemed good to me also, having had perfect understanding of all things from the very first, to write
unto thee in order, most excellent Theophilus,

1:4 That thou mightest know the certainty of those things, wherein thou hast been instructed.

Gabriel sent to tell Zacharias that Elisabeth will have a son, John (5-25)
1:5 ¶ There was in the days of Herod, the king of Judæa, a certain priest named Zacharias, of the course
of Abia: and his wife was of the daughters of Aaron, and her name was Elisabeth.

1:6 And they were both righteous before God, walking in all the commandments and ordinances of the
Lord blameless.

The Life of John the Baptist and how it relates to you and I

Luke 1:6, 11

Right side means favour. Christ on the right hand of the majesty on high. Sheep on the right, goats
on the left

What did Jesus say about John the Baptist?

Matt 11:11

Luke 7:28

In the sight of God, John was great. In the sight of man, he was nothing.

Luke 1:17

It was the fulfillment of the prophecy in Malachi 4. But it was only partially fulfilled

Matt 17:10-13

Although he did fill the spirit of Elijah, but it was only partial fulfillment – because it is referring to
the dreadful day of the Lord in Luke 1:17 – the second coming of Jesus. Why did John the Baptist
die? – there are end time applications for us.

Luke 1:17 …to turn the hearts of the fathers to the children, and the disobedient to the wisdom of
the just” – quoted from Malachi.

“Disobedient” – Is referring to the children in Malachi. John’s ministry was to awaken the children
of Israel to show them that the Messiah was here. John will turn the children of Israel back to God
their Father. The angel in Luke was interpreting this verse.

Luke 1:59-63

http://breachrepairers.webs.com/ 408
It was expected that John would follow in his father’s occupation – a priest. John was expected to do
exactly what his father did. But when John’s relatives asked them, they both said John. An angel
gave them that name. A lot of us have been living in families where we’ve been good people, good
Adventists. But there came a point in them where God had to change the pattern in the family and
had to call John to do something specifically unique so He gave him a special name. People may
have expectations on you, but you pray to God, because God has a specific calling to each one of you.
Your work may not seem that great in the eyes of the world, but today God is calling Elijah’s. He is
calling them to do a special work. When Jesus heard of John preaching, He knew that His time was
coming to reveal Himself to the world. Without there being another Elijah today, Jesus can’t come.

Luke 1:76-80
There was something similar described of Jesus – Luke 2:52. Three aspects were highlighted of
Jesus’ education:

“Mental” – wisdom

“Stature” – physical

“Spiritual” – favour with God

“Social” – and man

But with John it was in a different order

“Physical” – child grew

“Spiritual” – waxed strong in spirit

“Mental” - was in the deserts till the day of his showing unto Israel.

Moses spent forty studying in the wilderness – similar to John’s education.


 First 40 years in Egypt – he was training to be somebody
 Next 40 years – he was training to be nobody
 Last part of his life – he was showing people that he was nobody
John the Baptist did not even trust himself. He lived out in the wilderness but even then, he still
went and mingled with people in the city – kept up to date with the events of the world – that is the
Bible method of evangelism. This is the way Enoch lived as well. People were so wealthy back then
– the love of luxury and display. There is such occurring nowadays as well. What does modesty
mean? – not to attract attention to yourself. There was nothing that would point to himself. Jesus
came in the same way. John dressed like the old prophets – exactly like how Elijah dressed.

Luke 3:3

No one had really ever been baptized before in the Old Testament. But there were figures that
represented it: Naaman – leprosy is a symbol of sin in the Bible, Red sea deliverance, The flood. The
laver – washing before entering the holy place. What does the word “remission of sins” mean? The
word remiss means passing over. God would pass over them – the passover.

http://breachrepairers.webs.com/ 409
Luke 3:9

If you didn’t bring forth fruit, then you would be cast down and thrown into the fire. What does fruit
represent? – the fruit of the Spirit?

Luke 3:11

What was the problem in the society? – materialism. There is a parallel to our time as well.

Luke 3:12-13

There is covetousness – same as our time.

Luke 3:14

There is so much violence in our world today – just as back then.

Luke 3:19-20

What was the reason that John was put in prison? John reproved him for the adultery and
fornication. John represents the people that will prepare the way of Jesus’ second coming. We must
reprove the adultery between the church and the state.

1:7 And they had no child, because that Elisabeth was barren, and they both were now well stricken in
years.

1:8 And it came to pass, that while he executed the priest's office before God in the order of his course,

1:9 According to the custom of the priest's office, his lot was to burn incense when he went into the
temple of the Lord.

1:10 And the whole multitude of the people were praying without at the time of incense.

1:11 And there appeared unto him an angel of the Lord standing on the right side of the altar of incense.

1:12 And when Zacharias saw him, he was troubled, and fear fell upon him.

1:13 But the angel said unto him, Fear not, Zacharias: for thy prayer is heard; and thy wife Elisabeth
shall bear thee a son, and thou shalt call his name John.

1:14 And thou shalt have joy and gladness; and many shall rejoice at his birth.

1:15 For he shall be great in the sight of the Lord, and shall drink neither wine nor strong drink; and he
shall be filled with the Holy Ghost, even from his mother's womb.

1:16 And many of the children of Israel shall he turn to the Lord their God.

1:17 And he shall go before him in the spirit and power of Elias, to turn the hearts of the fathers to the
children, and
the disobedient to the wisdom of the just; to make ready a people prepared for the Lord.

1:18 And Zacharias said unto the angel, Whereby shall I know this? for I am an old man, and my wife

http://breachrepairers.webs.com/ 410
well stricken in years.

1:19 And the angel answering said unto him, I am Gabriel, that stand in the presence of God; and am sent
to speak unto thee, and to shew thee these glad tidings.

1:20 And, behold, thou shalt be dumb, and not able to speak, until the day that these things shall be
performed, because thou believest not my words, which shall be fulfilled in their season.

1:21 And the people waited for Zacharias, and marvelled that he tarried so long in the temple.

1:22 And when he came out, he could not speak unto them: and they perceived that he had seen a vision
in the temple: for he beckoned unto them, and remained speechless.

1:23 And it came to pass, that, as soon as the days of his ministration were accomplished, he departed to
his own house.

1:24 And after those days his wife Elisabeth conceived, and hid herself five months, saying,

1:25 Thus hath the Lord dealt with me in the days wherein he looked on me, to take away my reproach
among men.

Gabriel sent to tell Mary she will have a son, Jesus (26-38)
1:26 And in the sixth month the angel Gabriel was sent from God unto a city of Galilee, named Nazareth,

1:27 To a virgin espoused to a man whose name was Joseph, of the house of David; and the virgin's name
was Mary.

1:28 And the angel came in unto her, and said, Hail, thou that art highly favoured, the Lord is with thee:
blessed art thou among women.

1:29 And when she saw him, she was troubled at his saying, and cast in her mind what manner of
salutation this should be.

1:30 And the angel said unto her, Fear not, Mary: for thou hast found favour with God.

1:31 And, behold, thou shalt conceive in thy womb, and bring forth a son, and shalt call his name JESUS.

1:32 He shall be great, and shall be called the Son of the Highest: and the Lord God shall give unto him
the throne of his father David:

1:33 And he shall reign over the house of Jacob for ever; and of his kingdom there shall be no end.

1:34 Then said Mary unto the angel, How shall this be, seeing I know not a man?

1:35 And the angel answered and said unto her, The Holy Ghost shall come upon thee, and the power of
the Highest shall overshadow thee: therefore also that holy thing which shall be born of thee shall be
called the Son of God.

1:36 And, behold, thy cousin Elisabeth, she hath also conceived a son in her old age: and this is the sixth
month with her, who was called barren.

1:37 For with God nothing shall be impossible.

1:38 And Mary said, Behold the handmaid of the Lord; be it unto me according to thy word. And the angel

http://breachrepairers.webs.com/ 411
departed from her.

1:39 And Mary arose in those days, and went into the hill country with haste, into a city of Juda;

1:40 And entered into the house of Zacharias, and saluted Elisabeth.

1:41 And it came to pass, that, when Elisabeth heard the salutation of Mary, the babe leaped in her
womb; and Elisabeth was filled with the Holy Ghost:

1:42 And she spake out with a loud voice, and said, Blessed art thou among women, and blessed is the
fruit of thy womb.

1:43 And whence is this to me, that the mother of my Lord should come to me?

1:44 For, lo, as soon as the voice of thy salutation sounded in mine ears, the babe leaped in my womb for
joy.

1:45 And blessed is she that believed: for there shall be a performance of those things which were told
her from the Lord.

1:46 And Mary said, My soul doth magnify the Lord,

1:47 And my spirit hath rejoiced in God my Saviour.

1:48 For he hath regarded the low estate of his handmaiden: for, behold, from henceforth all generations
shall call me blessed.

1:49 For he that is mighty hath done to me great things; and holy is his name.

1:50 And his mercy is on them that fear him from generation to generation.

1:51 He hath shewed strength with his arm; he hath scattered the proud in the imagination of their
hearts.

1:52 He hath put down the mighty from their seats, and exalted them of low degree.

1:53 He hath filled the hungry with good things; and the rich he hath sent empty away.

1:54 He hath holpen his servant Israel, in remembrance of his mercy;

1:55 As he spake to our fathers, to Abraham, and to his seed for ever.

1:56 And Mary abode with her about three months, and returned to her own house.

Elisabeth gives birth to John (57-66)


1:57 Now Elisabeth's full time came that she should be delivered; and she brought forth a son.

1:58 And her neighbours and her cousins heard how the Lord had shewed great mercy upon her; and
they rejoiced with her.

1:59 And it came to pass, that on the eighth day they came to circumcise the child; and they called him
Zacharias, after the name of his father.

1:60 And his mother answered and said, Not so; but he shall be called John.

http://breachrepairers.webs.com/ 412
1:61 And they said unto her, There is none of thy kindred that is called by this name.

1:62 And they made signs to his father, how he would have him called.

1:63 And he asked for a writing table, and wrote, saying, His name is John. And they marvelled all.

1:64 And his mouth was opened immediately, and his tongue loosed, and he spake, and praised God.

1:65 And fear came on all that dwelt round about them: and all these sayings were noised abroad
throughout all the
hill country of Judæa.

1:66 And all they that heard them laid them up in their hearts, saying, What manner of child shall this
be! And the
hand of the Lord was with him.

Zachariah's prophecy concerning Jesus & John (67-80)


1:67 And his father Zacharias was filled with the Holy Ghost, and prophesied, saying,

1:68 Blessed be the Lord God of Israel; for he hath visited and redeemed his people,

1:69 And hath raised up an horn of salvation for us in the house of his servant David;

1:70 As he spake by the mouth of his holy prophets, which have been since the world began:

1:71 That we should be saved from our enemies, and from the hand of all that hate us;

1:72 To perform the mercy promised to our fathers, and to remember his holy covenant;

1:73 The oath which he sware to our father Abraham,

1:74 That he would grant unto us, that we being delivered out of the hand of our enemies might serve
him without
fear,

1:75 In holiness and righteousness before him, all the days of our life.

1:76 And thou, child, shalt be called the prophet of the Highest: for thou shalt go before the face of the
Lord to prepare his ways;

1:77 To give knowledge of salvation unto his people by the remission of their sins,

1:78 Through the tender mercy of our God; whereby the dayspring from on high hath visited us,

1:79 To give light to them that sit in darkness and in the shadow of death, to guide our feet into the way
of peace.

1:80 And the child grew, and waxed strong in spirit, and was in the deserts till the day of his shewing
unto Israel.

http://breachrepairers.webs.com/ 413
Chapter 2 - Jesus' birth & visit to Jerusalem
Chapter Outline
 Jesus born in Bethlehem during the days of taxing (1-7)
 Angels and Shepard's tell good tidings of His birth to many (8-20)
 Jesus circumcised and seen by Simeon and Anna (21-39)
 Jesus' growth and visit to Jerusalem (39-52)

Note: The Birth of Jesus brings salvation and redemption to Israel.1 “And it came to pass in those
days, that there went out a decree from Caesar Augustus, that all the world should be taxed”

Jesus born in Bethlehem during the days of taxing (1-7)


2:1 And it came to pass in those days, that there went out a decree from Caesar Augustus, that all the
world should
be taxed.

“Caesar Augustus” – Just as Cyrus was used by God to issue a decree freeing God’s people from
Babylonian according to His promise, now God is using Caesar Augustus to fulfill His promised of
sending a Savior into the world to be born in Bethlehem.

God’s providence – This shows another way that God works in fulfilling His promises in our lives. In
Luke 1,2 & Matt 1 God sends His angels to Zecharias, Mary, Joseph and the Shepherds giving specific
directions on what He will do, and what they are to do. Now He doesn’t send an angel to tell Mary
and Joseph to go to Bethlehem. He orchestrates events. I wonder how Mary and Joseph would have
reacted if an angel told them to go to Bethlehem?

“all the world should be taxed” – Two things are for the whole world in Luke 2: A decree from
Caesar and glad tidings of great joy. But Caesar’s decree ended with the then known world. God’s
tidings extend to everyone who has ever lived in the whole world.

2:2 (And this taxing was first made when Cyrenius was governor of Syria.)

2:3 And all went to be taxed, every one into his own city.

2:4 And Joseph also went up from Galilee, out of the city of Nazareth, into Judaea, unto the city of David,
which is called Bethlehem; (because he was of the house and lineage of David:)

Both Mary and Joseph were of the lineage of David

2:5 To be taxed with Mary his espoused wife, being great with child.

2:6 And so it was, that, while they were there, the days were accomplished that she should be delivered.

Luke 1 left her at 3 months pregnant. Wonder how many months she was when they left Galilee,
how long the trip was, and how long they stayed in Bethlehem.

2:7 And she brought forth her firstborn son, and wrapped him in swaddling clothes, and laid him in a
manger; because there was no room for them in the inn”

http://breachrepairers.webs.com/ 414
“firstborn son” – this implies that Mary had more children after the birth of Jesus. This is not
firstborn in the sense of preeminence like other passages of scripture.

Angels and Shepard's tell good tidings of His birth to many (8-20)
2:8 And there were in the same country shepherds abiding in the field, keeping watch over their flock by
night.

“same country” – Judea. Luke is emphasizing this because of all the inhabitants that were in Judea,
none of them realized that the fulfillment of God’s promise was taking place.

“shepherds” – Why did God come to these shepherds? Seems like they may have been the only
ones contemplating the theme of a coming Savior. Who else keep shepherds in the Bible? David and
Moses both prophesied of a coming King and Savior.

2:9 And, lo, the angel of the Lord came upon them, and the glory of the Lord shone round about them:
and they were sore afraid.

2:10 And the angel said unto them, Fear not: for, behold, I bring you good tidings of great joy, which shall
be to all people.

“good tidings of great joy” – This is the key phrase. What caused the shepherds to be sore afraid?
The glory of God. Glory means the brightness, radiant beams. And in the middle of the night they
would not be able to behold the sight. But what does glory represent? Good tidings of great joy.

“I bring you” – This shows that these tidings were given to them. Who gave them these tidings?
The Lord (v.9). They came from God, were given to the angels, and the angels gave them to the
shepherds.

“I bring you good tidings of great joy” – What did they bring? Good tidings of great joy. Who are
the tidings for? The shepherds (and all people), therefore ‘fear not’ fear not the glory of the Lord.

“great joy, which shall be to all people” – When the angel said all people it applies to Judea, and
the rest of the then known world, and finally all who have ever lived in the world. But it applied to
Palestine first. This joy is for Judea. This shows the condition of Judea and Palestine at this time.
They were full of sorrow. Why? They were waiting for a Savior. This shows the mindset of the
shepherds they were wondering about the Messiah so God appeared to them.

Good tidings also mean the gospel. Can you think of another gospel that is for all people? Rev 14:6
The Everlasting ‘good tidings’ to go to every nation, kindred, tongue, and people. MWhat are the
good tidings of great joy?

2:11 For unto you is born this day in the city of David a Saviour, which is Christ the Lord.

“City of David” – shows that the shepherds were in the same country, not the same city. What is the
good tidings of great joy?

“born this day” – (5 B.C.) – Shows that God fulfilled the specific day that He was to be born (Birth)

“City of David” – Shows location (and also the Messiah was to come from the lineage of David, a

http://breachrepairers.webs.com/ 415
King) (Location).

“Savior” – They needed to be redeemed

“Christ the Lord” – Messiah, the Divine Son of God (the one to be anointed) (Title)

App: God wants to dispel all of the sorrow in our world. The birth, location, and title of Jesus Christ
is the remedy for all sorrow in the world. The whole earth should break forth in gladness because
Jesus was born at the exact time that the Father promised, He was born as the seed or descendent
of David, and He is the anointed Savior who is God.

2:12 And this [shall be] a sign unto you; Ye shall find the babe wrapped in swaddling clothes, lying in a
manger.

“wrapped in swaddling clothes, lying in a manger” –We saw this in verse 7 “wrapped in
swaddling clothes, and laid him in a manger.” This shows that the angels were present when He
came out of the womb. All of the angels were waiting with abated breath. Not one would have
missed that event. They were without Michael, Jesus, the second member of the Godhead for nine
months!! From the time they were created the angels were always able to see Jesus. For thousands
of years and then all of a sudden He vanishes into thin air for nine months. And while He was in the
womb, humanity and divinity were joined together. His divinity was not walking around while His
humanity was growing in the womb, no, they were together. Just like at His death. Divinity didn’t
die, but divinity was present with that body. The opposite then is also true.

Now while Jesus was in the womb developing, all of heaven, and the unfallen worlds were without a
physical representation of the Father. God dwells in light unapproachable (1 Tim 6:16). The angels
approach His throne, but they don’t enter the light. They can hear His voice and see the magnificent
brilliance, but that is it. There is no Jesus to give shape to that voice and brightness. When Jesus said
if you have seen me you have seen the Father, that was also true when He was in the divine form of
God.

When angels and beings from the unfallen worlds desired an interview with God they can only see
light and hear a voice. That is it. Heaven at that time only consisted of the Father, The Holy Spirit,
and angels for nine months. And then the angels watched Him disappear and the next time they see
Him is as a helpless babe in the manger! What Love, how amazing, and with abated breath they
hasten to tell whoever will listen. The Savior is born, the Savior is born, and the people of God are
asleep! And then the Father says watch over Him, who? Their commander and chief…and they
humbly obey and only the wise men and shepherds understood the event. But you and I can
appreciate it now and come humbly to our Savior.

Sermon Title: “Nine Months Without One God”

2:13 And suddenly there was with the angel a multitude of the heavenly host praising God, and saying,

2:14 Glory to God in the highest, and on earth peace, good will toward men.

“on earth peace” – This shows God’s purpose for the earth. Peace. The opposite of peace is war,
variance, strife, discord, violence, etc. The earth at this time (and our time) is filled with sorrow and
strife. God wants to bring joy and peace. And once there is peace on earth and peace in heaven,
heaven and earth will be one. How? The Father will accomplish this through the kingdom of Christ

http://breachrepairers.webs.com/ 416
being established.

Luke 1:79 To give light to them that sit in darkness and [in] the shadow of death, to guide our feet
into the way of peace.

Luke 19:38 Blessed [be] the King that cometh in the name of the Lord: peace in heaven, and glory in
the highest.

Eph 2:14-17 [14] For he is our peace, who hath made both one, and hath broken down the middle
wall of partition [between us]. Having abolished in his flesh the enmity, [even] the law of
commandments [contained] in ordinances; for to make in himself of twain one new man, [so] making
peace And that he might reconcile both unto God in one body by the cross, having slain the enmity
thereby: And came and preached peace to you which were afar off, and to them that were nigh.

Rom 5:1 Therefore being justified by faith, we have peace with God through our Lord Jesus
Christ…For if, when we were enemies, we were reconciled to God by the death of his Son, much more,
being reconciled, we shall be saved by his life.

We were enemies, but now we are friends. We will also receive peace through the establishment of
the Messianic Kingdom after the millennium.

Is 9:6 For unto us a child is born, unto us a son is given: and the government shall be upon his
shoulder: and his name shall be called Wonderful, Counsellor, The mighty God, The everlasting
Father, The Prince of Peace. Of the increase of [his] government and peace [there shall be] no end,
upon the throne of David, and upon his kingdom, to order it, and to establish it with judgment and
with justice from henceforth even for ever. The zeal of the LORD of hosts will perform this.” (us –
meaning He is given to humanity forever as Son of man. Same as Jn 3:16 “he gave His only begotten
Son)

Note: “to order it” – this shows not only Christ giving instruction to His subjects, but also Him
organizing His kingdom. We can see a hierarchical structure. In which we reign with him also
having different responsibilities, but all being under Christ. This will be established at the second
coming, through the millennium, and then Christ turns the kingdom over to the Father. His kingdom
lasts forever, but He will be subject to the Father. The order we know of is that the 12 disciples will
have 12 thrones judging (Matt 19:28). The last generation that comes from Laodicea will share
Christ’s throne (Rev 3:21), and the 144,000 who come from the last generation, follow Christ
wherever He goes literally (Rev 14:1,4). This can be summed up in one word ‘principalities’ (Col
1:16). There will be judges, magistrates, kings, governors over dominions (I believe). This can also
be added to my study on Kingdom of God and Kingdom of Christ

“good will toward men” – This shows a vindication of God’s character. The angels are expressing
the motives behind the Father’s actions. Why? Because there is a fallen angel who accuses God of
wanting arbitrary obedience because He is unjust. We see the great controversy theme here.

Jer 29:11 For I know the thoughts that I think toward you, saith the LORD, thoughts of peace, and
not of evil, to give you an expected end.

2:15 And it came to pass, as the angels were gone away from them into heaven, the shepherds said one
to another, Let us now go even unto Bethlehem, and see this thing which is come to pass, which the Lord
hath made known unto us.

2:16 And they came with haste, and found Mary, and Joseph, and the babe lying in a manger.

http://breachrepairers.webs.com/ 417
“they came with haste” – this shows that they obeyed and had great anticipation

2:17 And when they had seen [it], they made known abroad the saying which was told them concerning
this child

2:18 And all they that heard [it] wondered at those things which were told them by the shepherds

Reminds me of Rev 13 the world wonders after the beast, interesting it is after his re-birth that the
world wonders after the beast. When Jesus was born they all wonder. Wonder meaning to worship
with adoration.

2:19 But Mary kept all these things, and pondered them in her heart.

2:20 And the shepherds returned, glorifying and praising God for all the things that they had heard and
seen, as it was told unto them.

“they had seen it….made known abroad”

“heard and seen” – Now we see why God chose these shepherds. He knew they would be
instruments in the hands of God to share this message with the Jews. God could have sent angels
everywhere, but he allowed man to act a part in proclaiming the glad tidings of great joy. God used
the shepherds to take the message to the Jews, and God used the Wise men to take the message to
Gentiles. Why were they able to share the message abroad? They had seen & heard

Acts 4:20 For we cannot but speak the things which we have seen and heard.” (Peter and John)

1 Jn 1:1,3 [1] That which was from the beginning, which we have heard, which we have seen with
our eyes, which we have looked upon, and our hands have handled, of the Word of life…[3] That
which we have seen and heard declare we unto you

App: We will only make know the message of salvation to others if we have seen and heard. It
means to experience what God has told us. What would have happened if the shepherds never acted
on the word. Then they would have only heard, they would not have experienced that word for
themselves. We need to not only hear the word, we need to act on it ‘with haste’.

Jesus circumcised and seen by Simeon and Anna (21-39)


2:21 And when eight days were accomplished for the circumcising of the child, his name was called
JESUS, which was so named of the angel before he was conceived in the womb.

2:22 And when the days of her purification according to the law of Moses were accomplished, they
brought him to Jerusalem, to present him to the Lord;

“days of purification” – After 40 days they brought Jesus to Jerusalem

Lev 12:2-4, 8 [2] Speak unto the children of Israel, saying, If a woman have conceived seed, and born
a man child: then she shall be unclean seven days; according to the days of the separation for her
infirmity shall she be unclean. [3] And in the eighth day the flesh of his foreskin shall be circumcised.
[4] And she shall then continue in the blood of her purifying three and thirty days; she shall touch
no hallowed thing, nor come into the sanctuary, until the days of her purifying be fulfilled. [8] And if
she be not able to bring a lamb, then she shall bring two turtles, or two young pigeons; the one for the
burnt offering, and the other for a sin offering: and the priest shall make an atonement for her, and

http://breachrepairers.webs.com/ 418
she shall be clean.

Note: Double check sin offering and purification.

DA 50 About forty days after the birth of Christ, Joseph and Mary took Him to Jerusalem, to
present Him to the Lord, and to offer sacrifice. This was according to the Jewish law, and as man's
substitute Christ must conform to the law in every particular. He had already been subjected to the
rite of circumcision, as a pledge of His obedience to the law”

App: From these verses we get Baby dedications.

2:23 (As it is written in the law of the Lord, Every male that openeth the womb shall be called holy to the
Lord;)

2:24 And to offer a sacrifice according to that which is said in the law of the Lord, A pair of turtledoves,
or two young pigeons.

2:25 And, behold, there was a man in Jerusalem, whose name was Simeon; and the same man was just
and devout, waiting for the consolation of Israel: and the Holy Ghost was upon him.

2:26 And it was revealed unto him by the Holy Ghost, that he should not see death, before he had seen the
Lord's Christ.

2:27 And he came by the Spirit into the temple: and when the parents brought in the child Jesus, to do for
him after the custom of the law,

“by the Holy Ghost” – We see the work of the Holy Ghost. In chap 1, by His power the Son of God
was implanted in the womb as a man. The incarnation.

Note: Treading softly it appears as if the Holy Ghost had the power to create the humanity of Jesus.
We know from this that the Holy Spirit has the power to give life. We see that again at the
resurrection. (Rom 8). We see Him now as the one who reveals the future to prophets. He also
directed him to go to the temple at that time. This man was use to being led by the Spirit. This is
what it means when the Bible says the Holy Spirit was upon him. He revealed and He led.

“Lord’s Christ” - Who is the Lord? The Father, one text showing that Lord can apply to the Father.
It can also apply to the Son and Holy Ghost. Christ means the anointed one. The Lord’s anointed

2:28 Then took he him up in his arms, and blessed God, and said,

2:29 Lord, now lettest thou thy servant depart in peace, according to thy word:

2:30 For mine eyes have seen thy salvation,

2:31 Which thou hast prepared before the face of all people;

2:32 A light to lighten the Gentiles, and the glory of thy people Israel.

2:33 And Joseph and his mother marvelled at those things which were spoken of him.

“Lord” – second time the Father is called Lord

http://breachrepairers.webs.com/ 419
“thy salvation” – same as Luke 2:10-11 Jesus is the Savior

“all people” – He is repeating what the angels said. Luke 2:10 “which shall be to all people. Luke 2
is an announcement of the glad tidings of great joy (the gospel) that is for all people. The gospel is
the birth, location (city of David), and title of Jesus. Savior, Christ, the Lord. This announcement was
made by:
 Angels (to shepherds)
 Shepherds (throughout Judea)
 Simeon (Mary, Joseph, Priests)
 Anna
 Jesus (Doctors of the Law)
To sum it up the birth of Jesus our Savior. (Born as a human). Theme of Luke 2

2:34 And Simeon blessed them, and said unto Mary his mother, Behold, this child is set for the fall and
rising again of many in Israel; and for a sign which shall be spoken against;

2:35 (Yea, a sword shall pierce through thy own soul also,) that the thoughts of many hearts may be
revealed.

Simeon is prophesying about the death of Jesus. How will He die? Passover

2:36 And there was one Anna, a prophetess, the daughter of Phanuel, of the tribe of Aser: she was of a
great age, and had lived with an husband seven years from her virginity;

2:37 And she was a widow of about fourscore and four years, which departed not from the temple, but
served God with fastings and prayers night and day.

2:38 And she coming in that instant gave thanks likewise unto the Lord, and spake of him to all them
that looked for redemption in Jerusalem.

“spake of him to all them that looked for redemption in Jerusalem” – This woman lived with
her husband for 7 years and then lived in the temple for the rest of her life, it appears. She is now 84
years of age. That means she was born around 89 B.C. she has spent many years praying and fasting
for the Messiah. And in the temple she saw the services that pre-figured the coming Messiah.

Jesus' growth and visit to Jerusalem (39-52)


2:39 And when they had performed all things according to the law of the Lord, they returned into Galilee,
to their own city Nazareth.

2:40 And the child grew, and waxed strong in spirit, filled with wisdom: and the grace of God was upon
him.

“child grew” – How did Jesus grow? Waxed strong in spirit – this refers to His mind

“grace of God” – How did Jesus wax strong in spirit and become full of wisdom? By the Grace of
God. So God gives us grace to grow. From this verse and onward we are going to see His wisdom
manifested, and just how strong in spirit He was.

http://breachrepairers.webs.com/ 420
“Filled with wisdom” – Jesus became a wise child. How?

2 Tim 3:15 And that from a child thou hast known the holy scriptures, which are able to make thee
wise unto salvation through faith which is in Christ Jesus.

What was Jesus learning? The Holy Scriptures, He was studying about the mission His Father sent
Him to finish. Salvation comes through Jesus, that was Him, the Christ, meaning He had to be
anointed, which also meant He was divine. He was God.

2:41 Now his parents went to Jerusalem every year at the feast of the passover.

2:42 And when he was twelve years old, they went up to Jerusalem after the custom of the feast.

“Passover”

Deut 16:1-2 [1] Observe the month of Abib, and keep the passover unto the LORD thy God: for in the
month of Abib the LORD thy God brought thee forth out of Egypt by night. [2] Thou shalt therefore
sacrifice the passover unto the LORD thy God, of the flock and the herd, in the place which the LORD
shall choose to place his name there.”

“Abib” – the first month of the Jews religious year. It is equivalent to March or April in our
year. The spring.

Ex 12:2-3, 5-6, 14 This month [shall be] unto you the beginning of months: it [shall be] the first
month of the year to you. Speak ye unto all the congregation of Israel, saying, In the tenth [day] of this
month they shall take to them every man a lamb, according to the house of [their] fathers, a lamb for
an house…Your lamb shall be without blemish, a male of the first year: ye shall take [it] out from
the sheep, or from the goats: And ye shall keep it up until the fourteenth day of the same month: and
the whole assembly of the congregation of Israel shall kill it in the evening And this day shall be unto
you for a memorial; and ye shall keep it a feast to the LORD throughout your generations; ye shall
keep it a feast by an ordinance for ever.

Note: Ex 12:14, 17; Ex 13:10 these verses call the Passover feast of unleavened bread, plus
the Lamb without blemish an ordinance. These things were done away at the death of Jesus.

Note: Col 2:14 “Blotting out the handwriting of ordinances that was against us, which was contrary
to us, and took it out of the way, nailing it to his cross”

Note: Dan 9:27 mentions oblations, which are sacrificial offerings. See Lev 2:1 & 3:1.

Ex 12:18 In the first [month], on the fourteenth day of the month at even, ye shall eat unleavened
bread, until the one and twentieth day of the month at even.

Ex 13:4,2 This day came ye out in the month Abib…Sanctify unto me all the firstborn, whatsoever
openeth the womb among the children of Israel, [both] of man and of beast: it [is] mine.

From these verses we can see how Jesus and His family kept the ceremonies. There homes had to be
without leaven for seven days, and then they ate unleavened bread from Abib 14 – Abib 21. On the
14th they ate the unleavened bread with the lamb, etc. And the Lamb was called the firstborn. Luke
2:7 “And she brought forth her firstborn son” for the antitypical Passover sacrifice

2:43 And when they had fulfilled the days, as they returned, the child Jesus tarried behind in Jerusalem;

http://breachrepairers.webs.com/ 421
and Joseph and his mother knew not of it.

2:44 But they, supposing him to have been in the company, went a day's journey; and they sought him
among their kinsfolk and acquaintance.

2:45 And when they found him not, they turned back again to Jerusalem, seeking him.

2:46 And it came to pass, that after three days they found him in the temple, sitting in the midst of the
doctors, both hearing them, and asking them questions.

2:47 And all that heard him were astonished at his understanding and answers.

“sitting in the midst of the doctors” – They were speaking about the temple and its services. This
is an explanation of verse 40. It shows how much wisdom He had. What did Jesus see in the temple?
Lamb, sacrifice, he was observing the daily service, because the yearly service was not until the end
of the year. They were there in the beginning of the year. And then all of the scriptures and texts He
memorized came together and He immediately went on His Father’s business. From this we see He
understood He was divine, and who His Father was by observing the sanctuary. He knew He was
the Lamb. Gen 22:12. Abraham said God will provide Himself a Lamb.

2:48 And when they saw him, they were amazed: and his mother said unto him, Son, why hast thou thus
dealt with us? behold, thy father and I have sought thee sorrowing.

2:49 And he said unto them, How is it that ye sought me? wist ye not that I must be about my Father's
business?”

“Father’s business” – The Father’s business has already been explained in the previous verses.
Glad tidings of great joy to all people. Salvation through the Christ, the Savior, of the seed of David.
He would bring salvation through the death of Jesus as typified in the sanctuary.

“my Father” – This shows at the age of 12 that Jesus has an understanding of His mission in this
world.

Luke 2:46 …hey found him in the temple…

This was during the Passover. When Jesus saw the Lamb He understood that He must give His life
as a sacrifice. But not only did He understand that He was to give His life as a sacrifice He
understood that He was God. How? Anytime Jesus says in reference to God ‘my Father’ it means that
He is the Son of God, which automatically makes Him God.

Jn 5:17 But Jesus answered them, My Father worketh hitherto, and I work. Therefore the Jews
sought the more to kill him, because he not only had broken the sabbath, but said also that God was
his Father, making himself equal with God.

2:50 And they understood not the saying which he spake unto them.

2:51 And he went down with them, and came to Nazareth, and was subject unto them: but his mother
kept all these sayings in her heart.

Shows He willingly obeyed them. ‘subject’ because He was actually God. His Father’s claims were
above His earthly parents claims, but for now, He subjected Himself until His time came. They didn’t
understand because there too were looking for the temporal establishment of the Messiah’s

http://breachrepairers.webs.com/ 422
kingdom. This is the second time Mary kept these things in her heart.

2:52 And Jesus increased in wisdom and stature, and in favour with God and man.

Summary
Luke 2 is an announcement of the glad tidings of great joy (the gospel) that is for all people. The
gospel is the birth, location (city of David), and title of Jesus. Savior, Christ, the Lord.

This announcement was made by: Angels (to shepherds), Shepherds (throughout Judea), Simeon
(Mary, Joseph, Priests), Anna, Jesus (Doctors of the Law). To sum it up the birth of Jesus our Savior.
(Born as a human). Theme of Luke 2.

http://breachrepairers.webs.com/ 423
Chapter 3 - Jesus' Baptism & Genealogy
Chapter Outline
 John preaches the baptism of repentance | Ministry of John Baptist (1-20)
 Jesus' baptism (21-23)
 The biological genealogy of Jesus (23-38)

John preaches the baptism of repentance | Ministry of John Baptist (1-


20)
3:1 Now in the fifteenth year of the reign of Tiberius Cæsar, Pontius Pilate being governor of Judæa, and
Herod being tetrarch of Galilee, and his brother Philip tetrarch of Ituraea and of the region of
Trachonitis, and Lysanias the tetrarch of Abilene,

3:2 Annas and Caiaphas being the high priests, the word of God came unto John the son of Zacharias in
the wilderness.

Note: Verses 1-2 Show the political and religious structure of Rome in the region of Palestine:
Caesar, Governor, Tetrarch and Priest It gives the picture like J. the Baptist was not intimidated by
this. Because he was in the wilderness and stood before the King of kings he need not fear.

"Tetrarch” – ruler of the 4th part of a province; Herod had the title of king.

“Iturea” – North east of Palestine

“Trachonitis” – East of Jordan (one of 5 regions that Rome divided it into). Abilene – Plain on the
east side of Lebanon.

3:3 And he came into all the country about Jordan, preaching the baptism of repentance for the
remission of sins;

3:4 As it is written in the book of the words of Esaias the prophet, saying, The voice of one crying in the
wilderness, Prepare ye the way of the Lord, make his paths straight.

3:5 Every valley shall be filled, and every mountain and hill shall be brought low; and the crooked shall
be made straight, and the rough ways shall be made smooth;

3:6 And all flesh shall see the salvation of God.

Note: V.6 climax; show's why John is baptizing. Contextually salvation is seeing Jesus. Luke doesn't
just want to show that Jesus was baptized and who baptized Him. But he is showing that Jesus was
the biological son of Adam.

Luke 3:22 Thou art my beloved son…

What kind of Son?

Luke 3:23-38

Luke 3:38 …son of God…

http://breachrepairers.webs.com/ 424
Thou art my beloved Son in divinity & humanity. And in speaking to Jesus thou art my beloved Son.
He was speaking to whom? Jesus as He came after His whole linage. He is a representative of
humanity.

3:7 Then said he to the multitude that came forth to be baptized of him, O generation of vipers, who hath
warned you to flee from the wrath to come?

3:8 Bring forth therefore fruits worthy of repentance, and begin not to say within yourselves, We have
Abraham to our father: for I say unto you, That God is able of these stones to raise up children unto
Abraham.

3:9 And now also the axe is laid unto the root of the trees: every tree therefore which bringeth not forth
good fruit is hewn down, and cast into the fire.

3:10 And the people asked him, saying, What shall we do then?

3:11 He answereth and saith unto them, He that hath two coats, let him impart to him that hath none;
and he that hath meat, let him do likewise.

3:12 Then came also publicans to be baptized, and said unto him, Master, what shall we do?

3:13 And he said unto them, Exact no more than that which is appointed you.

3:14 And the soldiers likewise demanded of him, saying, And what shall we do? And he said unto them,
Do violence to no man, neither accuse any falsely; and be content with your wages.

3:15 And as the people were in expectation, and all men mused in their hearts of John, whether he were
the Christ, or not;

3:16 John answered, saying unto them all, I indeed baptize you with water; but one mightier than I
cometh, the latchet of whose shoes I am not worthy to unloose: he shall baptize you with the Holy Ghost
and with fire:

3:17 Whose fan is in his hand, and he will throughly purge his floor, and will gather the wheat into his
garner; but the chaff he will burn with fire unquenchable.

3:18 And many other things in his exhortation preached he unto the people.

3:19 But Herod the tetrarch, being reproved by him for Herodias his brother Philip's wife, and for all the
evils which Herod had done,

3:20 Added yet this above all, that he shut up John in prison.

Jesus' baptism (21-23)


3:21 Now when all the people were baptized, it came to pass, that Jesus also being baptized, and praying,
the heaven was opened,

The Birth of Jesus – When Jesus was born we know that Herod tried to kill Him. In Rev 12 there
was a dragon trying to wipe out Jesus. So Satan was using Herod to try to wipe out Jesus. There is a
parallel once again with Moses and Jesus

Moses Jesus

http://breachrepairers.webs.com/ 425
Pharaoh tried to kill first born of all children of Herod tried to kill the babes in
Israel Bethlehem
Passed through the red sea Baptized
Wilderness for 40 years Temptation for 40 days
Died and resurrected Died and resurrected
In Heaven In Heaven

Luke 3:21

Tidings of the wilderness prophet and his wonderful announcement, spread throughout Galilee.
The message reached the peasants in the remotest hill towns, and the fisher folk by the sea, and in
these simple, earnest hearts found its truest response. In Nazareth it was told in the carpenter shop
that had been Joseph's, and One recognized the call. His time had come. Turning from His daily toil,
He bade farewell to His mother, and followed in the steps of His countrymen who were flocking to
the Jordan. Jesus knew that He had to do a special work for his Father, but now He recognized the
time had come.

Matt 3:13-17

Did Jesus need to be baptized? Why was Jesus baptized? To set us a proper example and to be
baptized in behalf of those that weren’t able to be baptized. Jesus and the Holy Spirit are two
separate beings. God the Father talked to Jesus – the Son. Therefore we see here three attributes:
Jesus, the Holy Spirit, and the Father. There are three distinct beings described here.

John 1:32-33

John knew that that man would be the Messiah be the sign of the Holy Spirit resting on Him. Of the
vast throng at the Jordan, few except John discerned the heavenly vision.

When Jesus was baptized, He remembered these words which helped Him to remember that He
was the Son of God and to endure the trials before Him. This symbolized that God was with us, and
also Jesus as God was with us as well

Matt 3:15 This is My beloved child, in whom I am well pleased.

And the word that was spoken to Jesus at the Jordan, "This is My beloved Son, in whom I am well pleased,"
embraces humanity.These words were for us as well.

3:22 And the Holy Ghost descended in a bodily shape like a dove upon him, and a voice came from
heaven, which said, Thou art my beloved Son; in thee I am well pleased.

The biological genealogy of Jesus (23-38)


3:23 And Jesus himself began to be about thirty years of age, being 3:as was supposed the son of Joseph,
which was the son of Heli,

3:24 Which was the son of Matthat, which was the son of Levi, which was the son of Melchi, which was the
son of Janna, which was the son of Joseph,

3:25 Which was the son of Mattathias, which was the son of Amos, which was the son of Naum, which
was the son of Esli, which was the son of Nagge,

http://breachrepairers.webs.com/ 426
3:26 Which was the son of Maath, which was the son of Mattathias, which was the son of Semei, which
was the son of Joseph, which was the son of Juda,

3:27 Which was the son of Joanna, which was the son of Rhesa, which was the son of Zorobabel, which
was the son of Salathiel, which was the son of Neri,

3:28 Which was the son of Melchi, which was the son of Addi, which was the son of Cosam, which was the
son of Elmodam, which was the son of Er,

3:29 Which was the son of Jose, which was the son of Eliezer, which was the son of Jorim, which was the
son of Matthat, which was the son of Levi,

3:30 Which was the son of Simeon, which was the son of Juda, which was the son of Joseph, which was the
son of Jonan, which was the son of Eliakim,

3:31 Which was the son of Melea, which was the son of Menan, which was the son of Mattatha, which was
the son of Nathan, which was the son of David,

3:32 Which was the son of Jesse, which was the son of Obed, which was the son of Booz, which was the
son of Salmon, which was the son of Naasson,

3:33 Which was the son of Aminadab, which was the son of Aram, which was the son of Esrom, which was
the son of Phares, which was the son of Juda,

3:34 Which was the son of Jacob, which was the son of Isaac, which was the son of Abraham, which was
the son of Thara, which was the son of Nachor,

3:35 Which was the son of Saruch, which was the son of Ragau, which was the son of Phalec, which was
the son of Heber, which was the son of Sala,

3:36 Which was the son of Cainan, which was the son of Arphaxad, which was the son of Sem, which was
the son of Noe, which was the son of Lamech,

3:37 Which was the son of Mathusala, which was the son of Enoch, which was the son of Jared, which was
the son of Maleleel, which was the son of Cainan,

3:38 Which was the son of Enos, which was the son of Seth, which was the son of Adam, which was the
son of God.

http://breachrepairers.webs.com/ 427
Chapter 4 - Jesus' temptation and ministry in Galilee
Chapter Outline
 The 3 temptations of Jesus (1-13)
 Jesus returns to Galilee & Nazareth (14-30)
 Jesus in Capernaum and departure to the rest of Galilee (31-44)

The 3 temptations of Jesus (1-13)


4:1 And Jesus being full of the Holy Ghost returned from Jordan, and was led by the Spirit into the
wilderness,

4:2 Being forty days tempted of the devil. And in those days he did eat nothing: and when they were
ended, he afterward hungered.

4:3 And the devil said unto him, If thou be the Son of God, command this stone that it be made bread.

4:4 And Jesus answered him, saying, It is written, That man shall not live by bread alone, but by every
word of God.

4:5 And the devil, taking him up into an high mountain, shewed unto him all the kingdoms of the world
in a moment of time.

4:6 And the devil said unto him, All this power will I give thee, and the glory of them: for that is delivered
unto me; and to whomsoever I will I give it.

The Third Temptation

Luke 4:5-6, 8

“Get thee behind me, Satan” – He recognized that it was Satan. At the start, Satan tried to make
Him think that He was the fallen angel. Satan didn’t really have the authority to give the earth to
anybody. What did these three temptations all have in common? Every sin in the world fall under
these categories Jesus answered it all with “It is written” – we can overcome all temptations in this
way.

4:7 If thou therefore wilt worship me, all shall be thine.

4:8 And Jesus answered and said unto him, Get thee behind me, Satan: for it is written, Thou shalt
worship the Lord thy God, and him only shalt thou serve.

4:9 And he brought him to Jerusalem, and set him on a pinnacle of the temple, and said unto him, If thou
be the Son of God, cast thyself down from hence:

4:10 For it is written, He shall give his angels charge over thee, to keep thee:

4:11 And in their hands they shall bear thee up, lest at any time thou dash thy foot against a stone.

4:12 And Jesus answering said unto him, It is said, Thou shalt not tempt the Lord thy God.

4:13 And when the devil had ended all the temptation, he departed from him for a season.

http://breachrepairers.webs.com/ 428
Jesus returns to Galilee & Nazareth (14-30)
4:14 ¶ And Jesus returned in the power of the Spirit into Galilee: and there went out a fame of him
through all the region round about.

4:15 And he taught in their synagogues, being glorified of all.

4:16 ¶ And he came to Nazareth, where he had been brought up: and, as his custom was, he went into the
synagogue on the sabbath day, and stood up for to read.

Why was Jesus allowed to stand up to read? He was considered a Rabbi and was a guest.

4:17 And there was delivered unto him the book of the prophet Esaias. And when he had opened the
book, he found the place where it was written,

4:18 The Spirit of the Lord is upon me, because he hath anointed me to preach the gospel to the poor; he
hath sent me to heal the brokenhearted, to preach deliverance to the captives, and recovering of sight to
the blind, to set at liberty them that are bruised,

4:19 To preach the acceptable year of the Lord.

Jesus read from Isaiah 61:1-2. What was so amazing about the statement that He read that caused
all to look at Him?

DA 237 Jesus stood before the people as a living expositor of the prophecies concerning Himself.
Explaining the words He had read, He spoke of the Messiah as a reliever of the oppressed, a liberator
of captives, a healer of the afflicted, restoring sight to the blind, and revealing to the world the light of
truth. His impressive manner and the wonderful import of His words thrilled the hearers with a
power they had never felt before. The tide of divine influence broke every barrier down; like Moses,
they beheld the Invisible. As their hearts were moved upon by the Holy Spirit, they responded with
fervent amens and praises to the Lord.

Delivery and content make a good sermon. It’s ok to say “Amen” in a church because it happened
back then.

DA 237 But when Jesus announced, "This day is this scripture fulfilled in your ears," they were
suddenly recalled to think of themselves, and of the claims of Him who had been addressing them.
They, Israelites, children of Abraham, had been represented as in bondage…Who is this Jesus? they
questioned. He who had claimed for Himself the glory of the Messiah was the son of a carpenter, and
had worked at His trade with His father Joseph. They had seen Him toiling up and down the hills, they
were acquainted with His brothers and sisters, and knew His life and labors. They had seen Him
develop from childhood to youth, and from youth to manhood. Although His life had been spotless,
they would not believe that He was the Promised One.

They were struggling with the fact that Jesus was the Promised One.

4:20 And he closed the book, and he gave it again to the minister, and sat down. And the eyes of all them
that were in the synagogue were fastened on him.

4:21 And he began to say unto them, This day is this scripture fulfilled in your ears.

4:22 And all bare him witness, and wondered at the gracious words which proceeded out of his mouth.
And they said, Is not this Joseph's son?

http://breachrepairers.webs.com/ 429
4:23 And he said unto them, Ye will surely say unto me this proverb, Physician, heal thyself: whatsoever
we have heard done in Capernaum, do also here in thy country.

It was a major thoroughfare. So His fame would be spread worldwide. Application: When
evangelizing, we must go where there are a lot of people. It was nicknamed “His own city”

DA 253 Jesus met the people on their own ground, as one who was acquainted with their
perplexities.

Don’t give them what they don’t need. (ie Sept 11 – grief, why does God allow suffering?)

DA 253 His language was pure, refined, and clear as a running stream. His voice was as music to
those who had listened to the monotonous tones of the rabbis. But while His teaching was simple, He
spoke as one having authority.

He didn’t adapt his language with His surroundings. He had voice modulation – it wasn’t
monotonous.

DA 254 His messages of mercy were varied to suit His audience. He knew "how to speak a word in
season to him that is weary" (Isa. 50:4); for grace was poured upon His lips, that He might convey to
men in the most attractive way the treasures of truth. He had tact to meet the prejudiced minds, and
surprise them with illustrations that won their attention. Through the imagination He reached the
heart. His illustrations were taken from the things of daily life, and although they were simple, they
had in them a wonderful depth of meaning.

When you can, use illustrations to explain what you want to convey.

DA 254 Christ never flattered men. He never spoke that which would exalt their fancies and
imaginations, nor did He praise them for their clever inventions; but deep, unprejudiced thinkers
received His teaching, and found that it tested their wisdom. One of the things that we should avoid is
make the audience feel comfortable in their state of spirituality and in their life.

Never flatter people – it will give them pride.

DA 256 He did not dream of becoming a terror to the world and the reproach of his family. He
thought his time could be spent in innocent folly. But once in the downward path, his feet rapidly
descended. Intemperance and frivolity perverted the noble attributes of his nature, and Satan took
absolute control of him.

Sin can lead to a downfall very quickly.

DA 258 History is repeating. With the open Bible before them, and professing to reverence its
teachings, many of the religious leaders of our time are destroying faith in it as the word of God. They
busy themselves with dissecting the word, and set their own opinions above its plainest statements.

Criticism

4:24 And he said, Verily I say unto you, No prophet is accepted in his own country.

4:25 But I tell you of a truth, many widows were in Israel in the days of Elias, when the heaven was shut
up three years and six months, when great famine was throughout all the land;

4:26 But unto none of them was Elias sent, save unto Sarepta, a city of Sidon, unto a woman that was a

http://breachrepairers.webs.com/ 430
widow.

The miracle that happened to this woman was of giving light to her.

4:27 And many lepers were in Israel in the time of Eliseus the prophet; and none of them was cleansed,
saving Naaman the Syrian.

Both Elijah and Elisha were prophets and both dealt with Gentiles. God sent them both out of the
country. Many times we do not appreciate the people around us until they are not there.

4:28 And all they in the synagogue, when they heard these things, were filled with wrath,

4:29 And rose up, and thrust him out of the city, and led him unto the brow of the hill whereon their city
was built, that they might cast him down headlong.

4:30 But he passing through the midst of them went his way,

Jesus in Capernaum and departure to the rest of Galilee (31-44)


4:31 And came down to Capernaum, a city of Galilee, and taught them on the sabbath days.

4:32 And they were astonished at his doctrine: for his word was with power.

4:33 ¶ And in the synagogue there was a man, which had a spirit of an unclean devil, and cried out with
a loud voice,

4:34 Saying, Let us alone; what have we to do with thee, thou Jesus of Nazareth? art thou come to destroy
us? I know thee who thou art; the Holy One of God.

4:35 And Jesus rebuked him, saying, Hold thy peace, and come out of him. And when the devil had thrown
him in the midst, he came out of him, and hurt him not.

4:36 And they were all amazed, and spake among themselves, saying, What a word is this! for with
authority and power he commandeth the unclean spirits, and they come out.

4:37 And the fame of him went out into every place of the country round about.

4:38 ¶ And he arose out of the synagogue, and entered into Simon's house. And Simon's wife's mother
was taken with a great fever; and they besought him for her.

4:39 And he stood over her, and rebuked the fever; and it left her: and immediately she arose and
ministered unto them.

4:40 ¶ Now when the sun was setting, all they that had any sick with divers diseases brought them unto
him; and he laid his hands on every one of them, and healed them.

4:41 And devils also came out of many, crying out, and saying, Thou art Christ the Son of God. And he
rebuking them suffered them not to speak: for they knew that he was Christ.

4:42 And when it was day, he departed and went into a desert place: and the people sought him, and
came unto him, and stayed him, that he should not depart from them.

4:43 And he said unto them, I must preach the kingdom of God to other cities also: for therefore am I
sent.

http://breachrepairers.webs.com/ 431
4:44 And he preached in the synagogues of Galilee.

http://breachrepairers.webs.com/ 432
Chapter 5 - Jesus came to call sinners to repentance (Jesus'
ministry)
Chapter Outline
 The call of Peter, James, and John by Gennesaret (1-11)
 Jesus heals the leper and His fame goes abroad (12-16)
 Jesus heals the man with palsy before Pharisees (17-26)
 Call and feast of Levi-Matthew & Parable of new wine and old bottles (27-39)

Note: He is the only man on earth who can forgive sins. And He calls all sinners to experience this.
What makes it possible for Him to forgive sins as man? Chap 4 He overcame sin as Man. So because
of this He has power on earth to forgive sins. Remember Jn 8? He that is without sin cast the first
stone. If you are without sin you are qualified to condemn. The opposite is true if you are without
sin you are qualified to justify.

The call of Peter, James, and John by Gennesaret (1-11)


5:1 And it came to pass, that, as the people pressed upon him to hear the word of God, he stood by the
lake of Gennesaret,

5:2 And saw two ships standing by the lake: but the fishermen were gone out of them, and were washing
their nets.

Jesus looked for these 2 boats because there were so many people that He needed breathing place.
Fishing was done at night so the fish cannot see the net.

What was Jesus doing when he met Simon and his friends?

Matt 4:17

Preaching the word. If we want to make disciples for Christ, we must likewise be doing the work.
What message in particular was he preaching? Matt 4:17 – the kingdom is at hand. Likewise, we
should be preaching the same message today in order to make strong disciples.

What were the fishermen doing? And what did that tell us about the stage they were at in
their fishing? Washing their nets. What did this mean? That they had finished fishing already and
were getting ready to finish their work.

5:3 And he entered into one of the ships, which was Simon's, and prayed him that he would thrust out a
little from the land. And he sat down, and taught the people out of the ship.

Simon Peter had already heard Jesus. But when they heard, they went back to their former
employment. Jesus teaches them a lesson on faith but there are a few objections that they have to
overcome: The fish can see the nets during the day. It is not a good idea to take advice from a
carpenter. They had just cleaned all their nets. They had toiled all night and caught nothing.

What was the first step that Jesus took in engaging Simon? What can we learn from this?
Jesus entered into Simon's ship.

http://breachrepairers.webs.com/ 433
Application: We have to enter into someone's ship, their life, their work, their surroundings, etc.
Jesus uses his boat. He makes the effort to come into his life and be a part of it.

What did Jesus ask Simon to do when He got on the boat? What application can we draw?
Jesus asked Simon to thrust out a little from the land. Who was on the land? v1. lots of people. What
is Jesus asking Simon to do? To step away a little from all the commotion and busyness.

Application: Who could Simon represent? Who are those we may need to thrust out only a little?
When we first disciple someone, we need to bring them out a little. Don’t throw them into the deep
end. Simon could represent those who are visitors, those hearing the truth for the first time. Could
also represent newly baptized. How do we thrust out a little for each group? New visitor – just to
join socials or CARE group, NOT communion or even church service yet. Newly baptized – just to
help out in ministry, NOT be a leader. Share with the work the talents they have.

What was Simon doing while Jesus taught? What can we learn from this? Simon watched while
Jesus taught.

Application: Simon needed to be an observer first, while Jesus taught. He needed to sit and watch
Jesus teach. Likewise, before we can become a disciple of Christ, we need to hear the word of God
for ourselves. Also, next step in discipleship, we need to bring the new disciple as observers to
watch us teach. This is called mentorship.

5:4 Now when he had left speaking, he said unto Simon, Launch out into the deep, and let down your nets
for a draught.

What did Jesus ask Simon to do after He had finished preaching? What lesson can we learn from
this? Jesus asks Peter to launch out into the deep. What does deep represent?

“Deep” – taking a bigger risk now. Taking the next big step in their relationship. Taking a bigger
step in faith with Jesus. We need to take people out step by step and not expect them to launch out
into the deep when you first meet them. Faith must be built up slowly.

NOTE: Jesus stayed with Simon all this time in the boat. He could have just asked Simon to launch
out into the deep by himself. Jesus could have jumped off the boat and swam back to shore and ask
Simon to launch into the deep himself. But as a good discipler and mentor, Jesus stayed with Simon
all the way until he had caught fish.

Application: A good disciple should stick with their student until you have helped them catch fishes
(souls) for Christ.

5:5 And Simon answering said unto him, Master, we have toiled all the night, and have taken nothing:
nevertheless at thy word I will let down the net.

“at thy word” – they did it. This teaches a lesson of faith. Abraham had to hope against hope for his
son Isaac and to sacrifice his son.

1st part of the miracle: their nets brake because of so much fish then both ships started to sink
because there was so much fish.

What does Simon’s response tell us about his thinking? Answer: It showed that Simon doubted.

http://breachrepairers.webs.com/ 434
He was just fishing the whole night without any success. Did he have reason to doubt? YES. Jesus
was not a fisherman, He was a carpenter’s son (Matt. 13:55). Peter was a tried fisherman, used to
working on the sea. Logic demanded that it would be impossible to catch anything during this time
of the day, and on top of that, they had just finished cleaning their nets.

Despite Simon’s doubt, what was positive about his response to Jesus? And what important
lesson can we learn from this? Although he had initially responded with doubt, there was a key
word in his response that we can learn from. What is it? It is the word “nevertheless.” Yes, he gave
evidence of a fruitless toil on the sea without any catch, “nevertheless…” That word showed that
even though he didn’t agree, he would do it anyways. So although Simon was doubtful, he still went
ahead and obeyed. Faith is still putting into action those things which seem impossible, but still
obeying because the Master said so.

5:6 And when they had this done, they inclosed a great multitude of fishes: and their net brake.

5:7 And they beckoned unto their partners, which were in the other ship, that they should come and help
them. And they came, and filled both the ships, so that they began to sink.

What did Simon do when he saw the abundance of fish that he had caught? What lesson can
we learn about discipleship from this? He called his friends to join.

Application: A person who is truly excited by fishing for God cannot help but call others to help.
This is how discipleship grows.

5:8 When Simon Peter saw it, he fell down at Jesus' knees, saying, Depart from me; for I am a sinful man,
O Lord.

At what point did Simon Peter fall down and exclaim that he was a sinful man, and why?
Simon Peter fell down when he saw that the boats were about to sink because of the multitude of
fishes. Why did he do that? It was at that point that he realized that God still blessed with the
abundance of the fishes despite the doubt that he held in his own mind. God was still gracious to
bless without condition of Simon completely believing. But he was rewarded because of his
“nevertheless.” If Peter had reasoned in his own heart and not said “nevertheless,” he would never
have caught that multitude of fishes. Many times we give way to unbelief too easy and use our own
human logic to go against the word of God, and as a result we lose the blessing that Jesus wants to
give us if only we had obeyed.

Jesus knew that Peter wanted to be a fisher of men

DA 246 Peter exclaimed, "Depart from me; for I am a sinful man;" yet he clung to the feet of Jesus,
feeling that he could not be parted from Him. –

Jesus spoke the language of love that Peter would understand. He had to show Peter how much He
cared. The greatest evidence to a fisherman of love to him would be a great catch. Everybody is
different. By showing love, Peter said that he was a sinful man

Rom 2:4

God speaks the language that we can understand

DA 246 It was after Peter had been led to self-renunciation and dependence upon divine power that

http://breachrepairers.webs.com/ 435
he received the call to his work for Christ.

There must be self renunciation before we can be called – to admit that we are a sinful man

Matt 4:19

Jesus wants us to be fisher of men. It is only Jesus that calls us – there is by no other way that we are
called.

Matt 4:20

They didn’t put all the things away expecting to come back if things didn’t work out. If the Lord calls
us, don’t look back. Don’t expect to come back to your former job.

Matt 4:21-22

The disciples also left their family behind. We know that Peter was married because his mother in
law was sick. So how do we harmonize this that he didn’t provide for his family? EGW defines the
word “hate” in the following verse:

Luke 14:26 If any [man] come to me, and hate not his father, and mother, and wife, and children, and
brethren, and sisters, yea, and his own life also, he cannot be my disciple.

We must ask what God wants us to do, not follow our families wishes.

5:9 For he was astonished, and all that were with him, at the draught of the fishes which they had taken:

Before Jesus told Peter, Andrew, James and John to follow Him fully, what did He prove? Jesus
had to prove that He could make them millionaires if He wanted to by giving them such a huge
catch. Jesus proved that He could take care of their physical needs. Has Jesus proved to YOU that He
can take care of your physical needs? If YES, then ask the following question – Then what is
stopping us from following Jesus all the way and forsaking all that we have? Maybe we haven’t seen
the sinfulness of our own unbelief. Maybe we are waiting for a bigger miracle to happen before we
follow Him fully.

5:10 And so was also James, and John, the sons of Zebedee, which were partners with Simon. And Jesus
said unto Simon, Fear not; from henceforth thou shalt catch men.

“fear not” – do not be afraid for the Lord will provide. God has something for you, you will be a soul
winner if you follow Christ.

5:11 And when they had brought their ships to land, they forsook all, and followed him.

When did Peter, Andrew, James and John forsake everything? They forsook everything when
they hit the land. They didn’t clean their nets. They didn’t sell all the fish they just caught so they
could support themselves a while longer. If we want to follow Jesus, we need to make that decision
and then put it into action right away. Why? Because the longer we wait, the harder it will get and
the more excuses we will find not to forsake everything. With the experience with Jesus and His
miracle fresh in their minds, they made their choice and put it into action straight away. Maybe
there are some that heard the call of Jesus many years ago but they let that decision just linger and
as a result, the cares of the world choked that decision. It is not too late to want to follow Jesus all

http://breachrepairers.webs.com/ 436
the way. It may mean that you would have to sacrifice some things in life, some creature comforts.
But God will take are of you. Do you want to follow Jesus all the way today and forsake all in life for
Him?

Jesus heals the leper and His fame goes abroad (12-16)
5:12 ¶ And it came to pass, when he was in a certain city, behold a man full of leprosy: who seeing Jesus
fell on his face, and besought him, saying, Lord, if thou wilt, thou canst make me clean.

5:13 And he put forth his hand, and touched him, saying, I will: be thou clean. And immediately the
leprosy departed from him.

5:14 And he charged him to tell no man: but go, and shew thyself to the priest, and offer for thy
cleansing, according as Moses commanded, for a testimony unto them.

5:15 But so much the more went there a fame abroad of him: and great multitudes came together to
hear, and to be healed by him of their infirmities.

5:16 ¶ And he withdrew himself into the wilderness, and prayed.

Jesus heals the man with palsy before Pharisees (17-26)


Point: Son of man can forgive sins on earth.

5:17 And it came to pass on a certain day, as he was teaching, that there were Pharisees and doctors of
the law sitting by, which were come out of every town of Galilee, and Judæa, and Jerusalem: and the
power of the Lord was present to heal them.

5:18 ¶ And, behold, men brought in a bed a man which was taken with a palsy: and they sought means to
bring him in, and to lay him before him.

5:19 And when they could not find by what way they might bring him in because of the multitude, they
went upon the housetop, and let him down through the tiling with his couch into the midst before Jesus.

5:20 And when he saw their faith, he said unto him, Man, thy sins are forgiven thee.

5:21 And the scribes and the Pharisees began to reason, saying, Who is this which speaketh
blasphemies? Who can forgive sins, but God alone?

5:22 But when Jesus perceived their thoughts, he answering said unto them, What reason ye in your
hearts?

5:23 Whether is easier, to say, Thy sins be forgiven thee; or to say, Rise up and walk?

5:24 But that ye may know that the Son of man hath power upon earth to forgive sins, 5:he said unto the
sick of the palsy, I say unto thee, Arise, and take up thy couch, and go into thine house.

5:25 And immediately he rose up before them, and took up that whereon he lay, and departed to his own
house, glorifying God.

5:26 And they were all amazed, and they glorified God, and were filled with fear, saying, We have seen
strange things to day.

Call and feast of Levi-Matthew & Parable of new wine and old bottles

http://breachrepairers.webs.com/ 437
(27-39)
5:27 ¶ And after these things he went forth, and saw a publican, named Levi, sitting at the receipt of
custom: and he said unto him, Follow me.

5:28 And he left all, rose up, and followed him.

5:29 And Levi made him a great feast in his own house: and there was a great company of publicans and
of others that sat down with them.

5:30 But their scribes and Pharisees murmured against his disciples, saying, Why do ye eat and drink
with publicans and sinners?

5:31 And Jesus answering said unto them, They that are whole need not a physician; but they that are
sick.

5:32 I came not to call the righteous, but sinners to repentance.

5:33 ¶ And they said unto him, Why do the disciples of John fast often, and make prayers, and likewise
the disciples of the Pharisees; but thine eat and drink?

5:34 And he said unto them, Can ye make the children of the bridechamber fast, while the bridegroom is
with them?

5:35 But the days will come, when the bridegroom shall be taken away from them, and then shall they
fast in those days.

5:36 ¶ And he spake also a parable unto them; No man putteth a piece of a new garment upon an old; if
otherwise, then both the new maketh a rent, and the piece that was taken out of the new agreeth not
with the old.

5:37 And no man putteth new wine into old bottles; else the new wine will burst the bottles, and be
spilled, and the bottles shall perish.

5:38 But new wine must be put into new bottles; and both are preserved.

5:39 No man also having drunk old wine straightway desireth new: for he saith, The old is better.

http://breachrepairers.webs.com/ 438
Chapter 6
Reading
 Chap 29 – The Sabbath
 Desire of Ages – Chapter 30 – He Ordained Twelve

Chapter Outline
 Disciples of Jesus eat corn on the Sabbath (1-5)
 Jesus heals on the Sabbath (Withered hand) (6-12)
 Jesus chooses 12 disciples (13-16)
 Sermon on the mount (Blessings, woes, parable) (17-49)

Disciples of Jesus eat corn on the Sabbath (1-5)


6:1 And it came to pass on the second sabbath after the first, that he went through the corn fields; and
his disciples plucked the ears of corn, and did eat, rubbing them in their hands.

Plucking is a kind of reaping. Rubbing is a kind of threshing.

6:2 And certain of the Pharisees said unto them, Why do ye that which is not lawful to do on the sabbath
days?

6:3 And Jesus answering them said, Have ye not read so much as this, what David did, when himself was
an hungred, and they which were with him;

6:4 How he went into the house of God, and did take and eat the shewbread, and gave also to them that
were with him; which it is not lawful to eat but for the priests alone?

When Jesus was saying this, what was the reason behind the argument to defend His actions? Why
bring up David and the temple? You could be using someone else’s actions to justify His argument.
But Jesus was showing that there was a connection between that of David and of plucking of corn.
What is holy right now? The Sabbath

DA 285 If it was right for David to satisfy his hunger by eating of the bread that had been set apart to
a holy use, then it was right for the disciples to supply their need by plucking the grain upon the
sacred hours of the Sabbath.

What is both holy? Sabbath and the bread.

DA 284 When accused of Sabbathbreaking at Bethesda, Jesus defended Himself by affirming His
Sonship to God, and declaring that He worked in harmony with the Father. Now that the disciples are
attacked, He cites His accusers to examples from the Old Testament, acts performed on the Sabbath
by those who were in the service of God.

David was also doing God’s work, and in doing God’s work, you are allowed to profane the Sabbath.

DA 285 Have ye not read in the law, how that on the Sabbath days the priests in the temple profane
the Sabbath, and are blameless? But I say unto you, That in this place is one greater than the temple."
"The Son of man is Lord also of the Sabbath.

http://breachrepairers.webs.com/ 439
The majority of the work of a pastor is on the Sabbath and he is being paid for that. So on the
Sabbath we can be busy doing things, but yet not defile the Sabbath.

6:5 And he said unto them, That the Son of man is Lord also of the sabbath.

Why does He say the word “also?” Jesus has authority over not, not just heaven and the universe,
but He is also above the Sabbath, Lord of the Sabbath.

Jesus heals on the Sabbath (Withered hand) (6-12)


6:6 And it came to pass also on another sabbath, that he entered into the synagogue and taught: and
there was a
man whose right hand was withered.

6:7 And the scribes and Pharisees watched him, whether he would heal on the sabbath day; that they
might find an accusation against him.

6:8 But he knew their thoughts, and said to the man which had the withered hand, Rise up, and stand
forth in the midst. And he arose and stood forth.

6:9 Then said Jesus unto them, I will ask you one thing; Is it lawful on the sabbath days to do good, or to
do evil? to save life, or to destroy it?

6:10 And looking round about upon them all, he said unto the man, Stretch forth thy hand. And he did so:
and his hand was restored whole as the other.

6:11 And they were filled with madness; and communed one with another what they might do to Jesus.

DA 286 It was a maxim among the Jews that a failure to do good, when one had opportunity, was to
do evil; to neglect to save life was to kill. Thus Jesus met the rabbis on their own ground. "But they
held their peace. And when He had looked round about on them with anger, being grieved for the
hardness of their hearts, He saith unto the man, Stretch forth thine hand. And he stretched it out: and
his hand was restored whole as the other." Mark 3:4, 5.

False religion. They are designed to bring out the good in man. They all end up putting man below
creature.

Is 58:13-14

Ps 37:4

6:12 And it came to pass in those days, that he went out into a mountain to pray, and continued all night
in prayer to God.

Jesus chooses 12 disciples (13-16)


6:13 ¶ And when it was day, he called unto him his disciples: and of them he chose twelve, whom also he
named
apostles;

DA 291 Jesus had called His disciples that He might send them forth as His witnesses, to declare to
the world what they had seen and heard of Him. Their office was the most important to which human
beings had ever been called, and was second only to that of Christ Himself. They were to be workers
together with God for the saving of the world. As in the Old Testament the twelve patriarchs stand as
representatives of Israel, so the twelve apostles were to stand as representatives of the gospel

http://breachrepairers.webs.com/ 440
church.

“Twelve” represents leadership in the Bible – the order of God’s government. Twelve gates have
the names of the twelve tribes above them. Woman with the twelve stars – represents the old and
new testament church. Described before the child is born. It represents the government of His
church.

Rev 12:1

Woman with the twelve stars – represents the old and new testament church. Described before the
child is born. It represents the government of His church.

DA 293 While Jesus was preparing the disciples for their ordination, one who had not been
summoned urged his presence among them. It was Judas Iscariot, a man who professed to be a
follower of Christ. He now came forward, soliciting a place in this inner circle of disciples. With great
earnestness and apparent sincerity he declared, "Master, I will follow Thee whithersoever Thou
goest.”

Eleven were there and then comes one.

Matt 8:19

Described as Judas Iscariot in Desire of Ages. Judas

Luke 9:57-58

Jesus was emphasizing the poverty of His government. If Jesus was to establish the kingdom, Judas
wanted to be right there.

DA 294 The disciples were anxious that Judas should become one of their number. He was of
commanding appearance, a man of keen discernment and executive ability, and they commended
him to Jesus as one who would greatly assist Him in His work. They were surprised that Jesus
received him so coolly.

Those who we sometimes exalt are not the ones that God may call. Outward appearance is nothing
in God’s eyes. He doesn’t call the brightest.

DA 294 Yet when Judas joined the disciples, he was not insensible to the beauty of the character of
Christ. He felt the influence of that divine power which was drawing souls to the Saviour. He who
came not to break the bruised reed nor quench the smoking flax would not repulse this soul while
even one desire was reaching toward the light. The Saviour read the heart of Judas; He knew the
depths of iniquity to which, unless delivered by the grace of God, Judas would sink. In connecting this
man with Himself, He placed him where he might, day by day, be brought in contact with the
outflowing of His own unselfish love. If he would open his heart to Christ, divine grace would banish
the demon of selfishness, and even Judas might become a subject of the kingdom of God.

By allowing Judas to join the twelve, He puts him in the most favorable position to be saved. If
anyone is lost, it is not because God did not try to save them. This is the danger of looking at
outward appearances.

DA 295 All the disciples had serious faults when Jesus called them to His service. Even John, who
came into closest association with the meek and lowly One, was not himself naturally meek and

http://breachrepairers.webs.com/ 441
yielding. He and his brother were called "the sons of thunder." While they were with Jesus, any slight
shown to Him aroused their indignation and combativeness. Evil temper, revenge, the spirit of
criticism, were all in the beloved disciple. He was proud, and ambitious to be first in the kingdom of
God. But day by day, in contrast with his own violent spirit, he beheld the tenderness and
forbearance of Jesus, and heard His lessons of humility and patience. He opened his heart to the
divine influence, and became not only a hearer but a doer of the Saviour's words. Self was hid in
Christ. He learned to wear the yoke of Christ and to bear His burden.

What was the difference between John and James and Judas. John and James became not only a
hearer but also a doer of the Word, but Judas:

DA 295 He was self-sufficient, and instead of resisting temptation, he continued to follow his
fraudulent practices. Christ was before him, a living example of what he must become if he reaped
the benefit of the divine mediation and ministry; but lesson after lesson fell unheeded on the ears of
Judas.

As John and James saw Jesus, they desired to be more and more like Jesus. But Judas blocked out
Jesus and persisted in his own way. Judas was given every opportunity to know who he was, but he
conceded to transgression.

DA 297 And He is just as willing to manifest His power through us as through the first disciples.
However imperfect and sinful we may be, the Lord holds out to us the offer of partnership with
Himself, of apprenticeship to Christ. He invites us to come under the divine instruction, that, uniting
with Christ, we may work the works of God.

Jesus didn’t call perfect men back then. If we are cherishing known sin, when God calls us and if we
are willing to renounce our sins, then He can use us. So we have to be willing to be perfect to be
called by Christ.

6:14 Simon, (whom he also named Peter), and Andrew his brother, James and John, Philip and
Bartholomew,

6:15 Matthew and Thomas, James the son of Alphæus, and Simon called Zelotes,

6:16 And Judas the brother of James, and Judas Iscariot, which also was the traitor.

Sermon on the mount (Blessings, woes, parable) (17-49)


6:17 ¶ And he came down with them, and stood in the plain, and the company of his disciples, and a great
multitude of people out of all Judæa and Jerusalem, and from the sea coast of Tyre and Sidon, which
came to hear him, and to be healed of their diseases;

6:18 And they that were vexed with unclean spirits: and they were healed.

6:19 And the whole multitude sought to touch him: for there went virtue out of him, and healed them all.

6:20 ¶ And he lifted up his eyes on his disciples, and said, Blessed be ye poor: for yours is the kingdom of
God.

6:21 Blessed are ye that hunger now: for ye shall be filled. Blessed are ye that weep now: for ye shall
laugh.

6:22 Blessed are ye, when men shall hate you, and when they shall separate you from their company,
and shall reproach you, and cast out your name as evil, for the Son of man's sake.

http://breachrepairers.webs.com/ 442
6:23 Rejoice ye in that day, and leap for joy: for, behold, your reward is great in heaven: for in the like
manner did their fathers unto the prophets.

6:24 But woe unto you that are rich! for ye have received your consolation.

6:25 Woe unto you that are full! for ye shall hunger. Woe unto you that laugh now! for ye shall mourn
and weep.

6:26 Woe unto you, when all men shall speak well of you! for so did their fathers to the false prophets.

6:27 ¶ But I say unto you which hear, Love your enemies, do good to them which hate you,

6:28 Bless them that curse you, and pray for them which despitefully use you.

6:29 And unto him that smiteth thee on the one cheek offer also the other; and him that taketh away thy
cloke forbid not to take thy coat also.

6:30 Give to every man that asketh of thee; and of him that taketh away thy goods ask them not again.

6:31 And as ye would that men should do to you, do ye also to them likewise.

6:32 For if ye love them which love you, what thank have ye? for sinners also love those that love them.

6:33 And if ye do good to them which do good to you, what thank have ye? for sinners also do even the
same.

6:34 And if ye lend to them of whom ye hope to receive, what thank have ye? for sinners also lend to
sinners, to receive as much again.

6:35 But love ye your enemies, and do good, and lend, hoping for nothing again; and your reward shall
be great, and
ye shall be the children of the Highest: for he is kind unto the unthankful and to the evil.

6:36 Be ye therefore merciful, as your Father also is merciful.

6:37 Judge not, and ye shall not be judged: condemn not, and ye shall not be condemned: forgive, and ye
shall be forgiven:

6:38 Give, and it shall be given unto you; good measure, pressed down, and shaken together, and
running over, shall men give into your bosom. For with the same measure that ye mete withal it shall be
measured to you again.

6:39 And he spake a parable unto them, Can the blind lead the blind? shall they not both fall into the
ditch?

6:40 The disciple is not above his master: but every one that is perfect shall be as his master.

6:41 And why beholdest thou the mote that is in thy brother's eye, but perceivest not the beam that is in
thine own eye?

6:42 Either how canst thou say to thy brother, Brother, let me pull out the mote that is in thine eye, when
thou
thyself beholdest not the beam that is in thine own eye? Thou hypocrite, cast out first the beam out of
thine own eye, and then shalt thou see clearly to pull out the mote that is in thy brother's eye.

http://breachrepairers.webs.com/ 443
6:43 For a good tree bringeth not forth corrupt fruit; neither doth a corrupt tree bring forth good fruit.

6:44 For every tree is known by his own fruit. For of thorns men do not gather figs, nor of a bramble bush
gather they grapes.

6:45 A good man out of the good treasure of his heart bringeth forth that which is good; and an evil man
out of the evil treasure of his heart bringeth forth that which is evil: for of the abundance of the heart his
mouth speaketh.

6:46 ¶ And why call ye me, Lord, Lord, and do not the things which I say?

6:47 Whosoever cometh to me, and heareth my sayings, and doeth them, I will shew you to whom he is
like:

6:48 He is like a man which built an house, and digged deep, and laid the foundation on a rock: and when
the flood arose, the stream beat vehemently upon that house, and could not shake it: for it was founded
upon a rock.

6:49 But he that heareth, and doeth not, is like a man that without a foundation built an house upon the
earth; against which the stream did beat vehemently, and immediately it fell; and the ruin of that house
was great.

Luke 6:6-11

DA 286 It was a maxim among the Jews that a failure to do good, when one had opportunity, was to
do evil; to neglect to save life was to kill. Thus Jesus met the rabbis on their own ground. "But they
held their peace. And when He had looked round about on them with anger, being grieved for the
hardness of their hearts, He saith unto the man, Stretch forth thine hand. And he stretched it out: and
his hand was restored whole as the other." Mark 3:4, 5.

False religion. They are designed to bring out the good in man. They all end up putting man below
creature.

Is 58:13-14

Ps 37:4

http://breachrepairers.webs.com/ 444
Chapter 7 – The Centurion | Jesus a Prophet of God
Observation: 7:16, 19, 39, 49 [16]…prophet is risen up among us…[19] Art thou he that should
come...[39]...if he were a prophet...[49]…who is this that forgiveth sins also…

Note: Over riding question is who is this man?

Chronology: This was Jesus' last visit to Bethany. DA 557.2 "Christ's last visit to Bethany"
This Simon was a Pharisee who had been healed of leprosy. He reached Bethany Six days
before the Passover this happened after Lazarus had been raised from the dead.

Chapter Outline
 Centurion's faith (1-10)
 Resurrection of widow's son in Nain (11-17)
 John questions Jesus (John more than a prophet) (18-35)
 Simon's feast (Mary Magdalene's faith) (36-50)

Centurion's faith (1-10)


7:1 Now when he had ended all his sayings in the audience of the people, he entered into Capernaum.

7:2 And a certain centurion's servant, who was dear unto him, was sick, and ready to die.

As in previous encounters, in what manner did the centurion approach Jesus?

By beseeching Him. Pleading his case.

For whom was the centurion beseeching?

His servant who was dear unto him.

7:3 And when he heard of Jesus, he sent unto him the elders of the Jews, beseeching him that he would
come and heal his servant.

What did the centurion do once he heard of Jesus?

He asked the Jewish elders to speak to Jesus.

7:4 And when they came to Jesus, they besought him instantly, saying, That he was worthy for whom he
should do this:

7:5 For he loveth our nation, and he hath built us a synagogue.

On what basis did the elders think the centurion was worthy of Jesus’ help? How can we be
similar to the elders today?

Luke 7:4-5

Because he loved the Jewish nation. He had helped them build a synagogue. What did the Jewish

http://breachrepairers.webs.com/ 445
elders value? The centurion’s support of the Jewish nation. The centurion’s wealth and contribution
of the synagogue. How can we be like the elders today? By looking only to help and honor only
those with people with wealth.

What does the Bible warn against this kind of attitude?

Jas 2:1-9

The danger of the elder’s attitude is that it will lead to prejudice. According to this passage, it is a sin
to respect one class over another (rich over poor).

What was Jesus response? Jesus decided to go to the Centurion’s house. So despite the wrong
attitude of the elders, Jesus decided to listen for the centurion’s sake.

7:6 Then Jesus went with them. And when he was now not far from the house, the centurion sent friends
to him, saying unto him, Lord, trouble not thyself: for I am not worthy that thou shouldest enter under
my roof:

7:7 Wherefore neither thought I myself worthy to come unto thee: but say in a word, and my servant
shall be healed.

Contrast the Centurion’s response to the elders? Who do the elders and centurion represent
and what lessons can we learn from their attitude?

Luke 7:6-7

The elders felt the centurion was more than worthy of Jesus’ attention. The centurion was ashamed
to even be in the presence of Christ. He felt unworthy of His holy company. Centurion also
respected Jesus by addressing him Lord. Not even Nicodemus addressed Jesus with this title.

Application: Jewish elders = those who have been in the church for a long time. Centurion = pagan,
someone who has just learnt about Jesus.

Lesson 1: often it is those who have just heard about Jesus and maybe never even been Christian
who feel ashamed, whereas those who have grown up in church, or Christian by heritage or birth
who feel they have a right to Jesus’ help.

Lesson 2: first step in asking for Jesus’ help is a humble heart which recognizes his/her own
unworthiness.

How did the Centurion want Jesus to heal? What application can we draw? “but say in a word.”
The Centurion believed in Jesus so much, that all he wanted Jesus to do was say the words.

7:8 For I also am a man set under authority, having under me soldiers, and I say unto one, Go, and he
goeth; and to another, Come, and he cometh; and to my servant, Do this, and he doeth it.

Who was the Centurion comparing Jesus to? What was he trying to say about Jesus? Himself.
The Centurion was saying that just as he was a captain of the Roman army and people obeyed his
command, Jesus was lord over the earth and likewise all creation would listen to Christ’s command.
And if Jesus was said the word, the disease would depart. (DA 316.2) Compare the Centurion’s
attitude with the Nobleman’s.

http://breachrepairers.webs.com/ 446
John 4:47-48

Nobleman – would only believe in Jesus if He performed the miracle of healing his son. Centurion –
pleaded with Christ not to perform the miracle in his home, but just to say the words. For him it was
enough for Jesus just to speak.

What can we learn about faith from the Centurion’s example?

Matt 8:10

He realized his unworthiness before Christ rather than feeling like he deserved it because of his
status or birth right. He didn’t need outward signs or miracles to believe in Christ. He trusted Jesus
at His word and that was enough for him.

Discuss Jesus’ conclusion

Matt 8:10-12

Because of the attitude of the Jewish elders, many of God’s people would not be in heaven at the
table with Abraham, Isaac, Jacob. Instead, it would be those who were outside the kingdom of Israel.
Remember that the centurion was not a Jew, he was a Roman gentile.

Application: Likewise today, many who are born into the Adventist church and claim to be God’s
people by heritage who will not be sitting at the table. We will be surprised that it will be many
come from the outside, maybe later in life, who will be saved. It is those who can have the same
experience as the Centurion. Even if we have been born in church, we need to have a daily fresh
experience with Christ. Daily we need to feel our unworthiness. We need to take God’s word and
trust in it without expecting great signs and wonders and this will be great faith in God’s eyes.

7:9 When Jesus heard these things, he marvelled at him, and turned him about, and said unto the people
that followed him, I say unto you, I have not found so great faith, no, not in Israel.

7:10 And they that were sent, returning to the house, found the servant whole that had been sick.

Resurrection of widow's son in Nain (11-17)


7:11 And it came to pass the day after, that he went into a city called Nain; and many of his disciples
went with him, and much people.

7:12 Now when he came nigh to the gate of the city, behold, there was a dead man carried out, the only
son of his mother, and she was a widow: and much people of the city was with her.

7:13 And when the Lord saw her, he had compassion on her, and said unto her, Weep not.

7:14 And he came and touched the bier: and they that bare him stood still. And he said, Young man, I say
unto thee, Arise.

7:15 And he that was dead sat up, and began to speak. And he delivered him to his mother.

7:16 And there came a fear on all: and they glorified God, saying, That a great prophet is risen up among
us; and, That God hath visited his people.

http://breachrepairers.webs.com/ 447
7:17 And this rumour of him went forth throughout all Judaea, and throughout all the region round
about.

John questions Jesus (John more than a prophet) (18-35)


7:18 And the disciples of John shewed him of all these things.

7:19 And John calling unto him two of his disciples sent them to Jesus, saying, Art thou he that should
come? or look we for another?

7:20 When the men were come unto him, they said, John Baptist hath sent us unto thee, saying, Art thou
he that should come? or look we for another?

7:21 And in that same hour he cured many of their infirmities and plagues, and of evil spirits; and unto
many that were blind he gave sight.

7:22 Then Jesus answering said unto them, Go your way, and tell John what things ye have seen and
heard; how that the blind see, the lame walk, the lepers are cleansed, the deaf hear, the dead are raised,
to the poor the gospel is preached.

7:23 And blessed is he, whosoever shall not be offended in me.

7:24 And when the messengers of John were departed, he began to speak unto the people concerning
John, What went ye out into the wilderness for to see? A reed shaken with the wind?

7:25 But what went ye out for to see? A man clothed in soft raiment? Behold, they which are gorgeously
apparelled, and live delicately, are in kings' courts.

7:26 But what went ye out for to see? A prophet? Yea, I say unto you, and much more than a prophet.

7:27 This is he, of whom it is written, Behold, I send my messenger before thy face, which shall prepare
thy way before thee.

7:28 For I say unto you, Among those that are born of women there is not a greater prophet than John
the Baptist: but he that is least in the kingdom of God is greater than he.

7:29 And all the people that heard him, and the publicans, justified God, being baptized with the baptism
of John.

7:30 But the Pharisees and lawyers rejected the counsel of God against themselves, being not baptized of
him.

7:31 And the Lord said, Whereunto then shall I liken the men of this generation? and to what are they
like?

7:32 They are like unto children sitting in the marketplace, and calling one to another, and saying, We
have piped unto you, and ye have not danced; we have mourned to you, and ye have not wept.

7:33 For John the Baptist came neither eating bread nor drinking wine; and ye say, He hath a devil.

7:34 The Son of man is come eating and drinking; and ye say, Behold a gluttonous man, and a
winebibber, a friend of publicans and sinners!

7:35 But wisdom is justified of all her children.

http://breachrepairers.webs.com/ 448
Simon's feast (Mary Magdalene's faith) (36-50)
7:36 And one of the Pharisees desired him that he would eat with him. And he went into the Pharisee's
house, and sat down to meat.

7:37 And, behold, a woman in the city, which was a sinner, when she knew that Jesus sat at meat in the
Pharisee's house, brought an alabaster box of ointment,

7:38 And stood at his feet behind him weeping, and began to wash his feet with tears, and did wipe them
with the hairs of her head, and kissed his feet, and anointed them with the ointment.

7:39 Now when the Pharisee which had bidden him saw it, he spake within himself, saying, This man, if
he were a prophet, would have known who and what manner of woman this is that toucheth him: for she
is a sinner.

7:40 And Jesus answering said unto him, Simon, I have somewhat to say unto thee. And he saith, Master,
say on.

7:41 There was a certain creditor which had two debtors: the one owed five hundred pence, and the
other fifty.

7:42 And when they had nothing to pay, he frankly forgave them both. Tell me therefore, which of them
will love him most?

7:43 Simon answered and said, I suppose that he, to whom he forgave most. And he said unto him, Thou
hast rightly judged.

7:44 And he turned to the woman, and said unto Simon, Seest thou this woman? I entered into thine
house, thou gavest me no water for my feet: but she hath washed my feet with tears, and wiped them
with the hairs of her head.

7:45 Thou gavest me no kiss: but this woman since the time I came in hath not ceased to kiss my feet.

7:46 My head with oil thou didst not anoint: but this woman hath anointed my feet with ointment.

7:47 Wherefore I say unto thee, Her sins, which are many, are forgiven; for she loved much: but to whom
little is forgiven, the same loveth little.

7:48 And he said unto her, Thy sins are forgiven.

7:49 And they that sat at meat with him began to say within themselves, Who is this that forgiveth sins
also?

7:50 And he said to the woman, Thy faith hath saved thee; go in peace.

http://breachrepairers.webs.com/ 449
Chapter 8
Chapter Outline
 Parable of the sower (1-21)
 Jesus rebukes the sea and the legions of devils (22-40)
 Jesus heals the issue of blood and raises Jairus' daughter (41-56)

Parable of the sower (1-21)


8:1 And it came to pass afterward, that he went throughout every city and village, preaching and
shewing the glad tidings of the kingdom of God: and the twelve were with him,

8:2 And certain women, which had been healed of evil spirits and infirmities, Mary called Magdalene, out
of whom went seven devils,

8:3 And Joanna the wife of Chuza Herod's steward, and Susanna, and many others, which ministered
unto him of their substance.

8:4 And when much people were gathered together, and were come to him out of every city, he spake by
a parable:

8:5 A sower went out to sow his seed: and as he sowed, some fell by the way side; and it was trodden
down, and the fowls of the air devoured it.

8:6 And some fell upon a rock; and as soon as it was sprung up, it withered away, because it lacked
moisture.

8:7 And some fell among thorns; and the thorns sprang up with it, and choked it.

8:8 And other fell on good ground, and sprang up, and bare fruit an hundredfold. And when he had said
these things, he cried, He that hath ears to hear, let him hear.

8:9 And his disciples asked him, saying, What might this parable be?

8:10 And he said, Unto you it is given to know the mysteries of the kingdom of God: but to others in
parables; that seeing they might not see, and hearing they might not understand.

8:11 Now the parable is this: The seed is the word of God.

8:12 Those by the way side are they that hear; then cometh the devil, and taketh away the word out of
their hearts, lest they should believe and be saved.

8:13 They on the rock are they, which, when they hear, receive the word with joy; and these have no root,
which for a while believe, and in time of temptation fall away.

8:14 And that which fell among thorns are they, which, when they have heard, go forth, and are choked
with cares and riches and pleasures of this life, and bring no fruit to perfection.

8:15 But that on the good ground are they, which in an honest and good heart, having heard the word,
keep it, and bring forth fruit with patience.

8:16 No man, when he hath lighted a candle, covereth it with a vessel, or putteth it under a bed; but

http://breachrepairers.webs.com/ 450
setteth it on a candlestick, that they which enter in may see the light.

8:17 For nothing is secret, that shall not be made manifest; neither any thing hid, that shall not be
known and come abroad.

8:18 Take heed therefore how ye hear: for whosoever hath, to him shall be given; and whosoever hath
not, from him shall be taken even that which he seemeth to have.

8:19 Then came to him his mother and his brethren, and could not come at him for the press.

8:20 And it was told him by certain which said, Thy mother and thy brethren stand without, desiring to
see thee.

8:21 And he answered and said unto them, My mother and my brethren are these which hear the word of
God, and do it.

Jesus rebukes the sea and the legions of devils (22-40)


8:22 Now it came to pass on a certain day, that he went into a ship with his disciples: and he said unto
them, Let us go over unto the other side of the lake. And they launched forth.

8:23 But as they sailed he fell asleep: and there came down a storm of wind on the lake; and they were
filled with water, and were in jeopardy.

8:24 And they came to him, and awoke him, saying, Master, master, we perish. Then he arose, and
rebuked the wind and the raging of the water: and they ceased, and there was a calm.

8:25 And he said unto them, Where is your faith? And they being afraid wondered, saying one to another,
What manner of man is this! for he commandeth even the winds and water, and they obey him.

8:26 And they arrived at the country of the Gadarenes, which is over against Galilee.

8:27 And when he went forth to land, there met him out of the city a certain man, which had devils long
time, and ware no clothes, neither abode in any house, but in the tombs.

8:28 When he saw Jesus, he cried out, and fell down before him, and with a loud voice said, What have I
to do with thee, Jesus, thou Son of God most high? I beseech thee, torment me not.

8:29 (For he had commanded the unclean spirit to come out of the man. For oftentimes it had caught
him: and he was kept bound with chains and in fetters; and he brake the bands, and was driven of the
devil into the wilderness.)

8:30 And Jesus asked him, saying, What is thy name? And he said, Legion: because many devils were
entered into him.

8:31 And they besought him that he would not command them to go out into the deep.

8:32 And there was there an herd of many swine feeding on the mountain: and they besought him that
he would suffer them to enter into them. And he suffered them.

8:33 Then went the devils out of the man, and entered into the swine: and the herd ran violently down a
steep place into the lake, and were choked.

8:34 When they that fed them saw what was done, they fled, and went and told it in the city and in the
country.

http://breachrepairers.webs.com/ 451
8:35 Then they went out to see what was done; and came to Jesus, and found the man, out of whom the
devils were departed, sitting at the feet of Jesus, clothed, and in his right mind: and they were afraid.

8:36 They also which saw it told them by what means he that was possessed of the devils was healed.

8:37 Then the whole multitude of the country of the Gadarenes round about besought him to depart
from them; for they were taken with great fear: and he went up into the ship, and returned back again.

8:38 Now the man out of whom the devils were departed besought him that he might be with him: but
Jesus sent him away, saying,

8:39 Return to thine own house, and shew how great things God hath done unto thee. And he went his
way, and published throughout the whole city how great things Jesus had done unto him.

8:40 And it came to pass, that, when Jesus was returned, the people gladly received him: for they were all
waiting for him.

Jesus heals the issue of blood and raises Jairus' daughter (41-56)
8:41 And, behold, there came a man named Jairus, and he was a ruler of the synagogue: and he fell down
at Jesus' feet, and besought him that he would come into his house:

8:42 For he had one only daughter, about twelve years of age, and she lay a dying. But as he went the
people thronged him.

8:43 And a woman having an issue of blood twelve years, which had spent all her living upon physicians,
neither could be healed of any,

8:44 Came behind him, and touched the border of his garment: and immediately her issue of blood
stanched.

8:45 And Jesus said, Who touched me? When all denied, Peter and they that were with him said, Master,
the multitude throng thee and press thee, and sayest thou, Who touched me?

8:46 And Jesus said, Somebody hath touched me: for I perceive that virtue is gone out of me.

8:47 And when the woman saw that she was not hid, she came trembling, and falling down before him,
she declared unto him before all the people for what cause she had touched him, and how she was healed
immediately.

8:48 And he said unto her, Daughter, be of good comfort: thy faith hath made thee whole; go in peace.

8:49 While he yet spake, there cometh one from the ruler of the synagogue's house, saying to him, Thy
daughter is dead; trouble not the Master.

8:50 But when Jesus heard it, he answered him, saying, Fear not: believe only, and she shall be made
whole.

8:51 And when he came into the house, he suffered no man to go in, save Peter, and James, and John, and
the father and the mother of the maiden.

8:52 And all wept, and bewailed her: but he said, Weep not; she is not dead, but sleepeth.

http://breachrepairers.webs.com/ 452
8:53 And they laughed him to scorn, knowing that she was dead.

8:54 And he put them all out, and took her by the hand, and called, saying, Maid, arise.

8:55 And her spirit came again, and she arose straightway: and he commanded to give her meat.

8:56 And her parents were astonished: but he charged them that they should tell no man what was done.

http://breachrepairers.webs.com/ 453
Chapter 9 - The Kingdom of God?

Chapter Outline
 Jesus sends 12 disciples to preach the kingdom of God (1-10)
 Feeding the 5,000 (10-17)
 Jesus asks Peter who He is (Jesus explains His kingdom) (18-27)
 Jesus is transfigured (28-36)
 Casting out of devil/Jesus to be delivered up/Who is the greatest? (37-50)
 Jesus goes to Jerusalem (through Samaria, others want to follow) (51-62)

Observation: Who is He?

Mk 9:9 ...who is this...

Mk 9:18 ...whom say the people that I am?...

Mk 9:20 ...whom say ye that I am?...

Mk 9:35 ...This is my beloved Son...

Observation: Kingdom of God

Mk 9:2 ...Kingdom of God...

Mk 9:11 ...Kingdom of God...

Mk 9:27 ...Kingdom of God...

Mk 9:46 ...which of them should be greatest...

Mk 9:60 ...Kingdom of God...

Mk 9:62 ...Kingdom of God...

Observation: Sufferings of Jesus

Mk 9:22 ...Son of man: suffer, rejected, slain, rise...

Mk 9:31 ...spake of his decease...

Mk 9:44 ...Son of man shall be delivered in the hands of men...

Mk 9:51 ...time was come…should be received up...

Observation: Follow me

Mk 9:23 ...follow me...

Mk 9:59 ...follow me...

http://breachrepairers.webs.com/ 454
Mk 9:60 ...follow me...

Conclusion: The broadest theme is the Kingdom of God. Follow me. How? By experiencing His
suffering, rejection, death, and resurrection. Then you will be fit to enter into the Kingdom of God.
And this kingdom will be established at His second coming. And Luke gives an example; the
transfiguration. Moses and Elijah both lost their lives for Christ, so in the end they saved them. They
received eternal life.

Jesus sends 12 disciples to preach the kingdom of God (1-10)


9:1 Then he called his twelve disciples together, and gave them power and authority over all devils, and
to cure diseases.

9:2 And he sent them to preach the kingdom of God, and to heal the sick.

9:3 And he said unto them, Take nothing for your journey, neither staves, nor scrip, neither bread,
neither money; neither have two coats apiece.

9:4 And whatsoever house ye enter into, there abide, and thence depart.

9:5 And whosoever will not receive you, when ye go out of that city, shake off the very dust from your feet
for a testimony against them.

9:6 And they departed, and went through the towns, preaching the gospel, and healing every where.

9:7 Now Herod the tetrarch heard of all that was done by him: and he was perplexed, because that it was
said of some, that John was risen from the dead;

9:8 And of some, that Elias had appeared; and of others, that one of the old prophets was risen again.

9:9 And Herod said, John have I beheaded: but who is this, of whom I hear such things? And he desired to
see him.

9:10 And the apostles, when they were returned, told him all that they had done. And he took them, and
went aside privately into a desert place belonging to the city called Bethsaida.

Feeding the 5,000 (11-17)


9:11 And the people, when they knew it, followed him: and he received them, and spake unto them of the
kingdom of God, and healed them that had need of healing.

9:12 And when the day began to wear away, then came the twelve, and said unto him, Send the multitude
away, that they may go into the towns and country round about, and lodge, and get victuals: for we are
here in a desert place.

9:13 But he said unto them, Give ye them to eat. And they said, We have no more but five loaves and two
fishes; except we should go and buy meat for all this people.

9:14 For they were about five thousand men. And he said to his disciples, Make them sit down by fifties in
a company.

9:15 And they did so, and made them all sit down.

9:16 Then he took the five loaves and the two fishes, and looking up to heaven, he blessed them, and

http://breachrepairers.webs.com/ 455
brake, and gave to the disciples to set before the multitude.

9:17 And they did eat, and were all filled: and there was taken up of fragments that remained to them
twelve baskets.

Jesus asks Peter who He is (Jesus explains His kingdom) (18-27)


9:18 And it came to pass, as he was alone praying, his disciples were with him: and he asked them,
saying, Whom say the people that I am?

9:19 They answering said, John the Baptist; but some say, Elias; and others say, that one of the old
prophets is risen again.

9:20 He said unto them, But whom say ye that I am? Peter answering said, The Christ of God.

9:21 And he straitly charged them, and commanded them to tell no man that thing;

9:22 Saying, The Son of man must suffer many things, and be rejected of the elders and chief priests and
scribes, and be slain, and be raised the third day.

9:23 And he said to them all, If any man will come after me, let him deny himself, and take up his cross
daily, and follow me.

9:24 For whosoever will save his life shall lose it: but whosoever will lose his life for my sake, the same
shall save it.

9:25 For what is a man advantaged, if he gain the whole world, and lose himself, or be cast away?

9:26 For whosoever shall be ashamed of me and of my words, of him shall the Son of man be ashamed,
when he shall come in his own glory, and in his Father's, and of the holy angels.

9:27 But I tell you of a truth, there be some standing here, which shall not taste of death, till they see the
kingdom of God.

Jesus is transfigured (28-36)


9:28 And it came to pass about an eight days after these sayings, he took Peter and John and James, and
went up into a mountain to pray.

9:29 And as he prayed, the fashion of his countenance was altered, and his raiment was white and
glistering.

9:30 And, behold, there talked with him two men, which were Moses and Elias:

9:31 Who appeared in glory, and spake of his decease which he should accomplish at Jerusalem.

9:32 But Peter and they that were with him were heavy with sleep: and when they were awake, they saw
his glory, and the two men that stood with him.

9:33 And it came to pass, as they departed from him, Peter said unto Jesus, Master, it is good for us to be
here: and let us make three tabernacles; one for thee, and one for Moses, and one for Elias: not knowing
what he said.

9:34 While he thus spake, there came a cloud, and overshadowed them: and they feared as they entered
into the cloud.

http://breachrepairers.webs.com/ 456
9:35 And there came a voice out of the cloud, saying, This is my beloved Son: hear him.

9:36 And when the voice was past, Jesus was found alone. And they kept it close, and told no man in those
days any of those things which they had seen.

Casting out of devil/Jesus to be delivered up/Who is the greatest? (37-


50)
9:37 And it came to pass, that on the next day, when they were come down from the hill, much people met
him.

9:38 And, behold, a man of the company cried out, saying, Master, I beseech thee, look upon my son: for
he is mine only child.

9:39 And, lo, a spirit taketh him, and he suddenly crieth out; and it teareth him that he foameth again,
and bruising him hardly departeth from him.

9:40 And I besought thy disciples to cast him out; and they could not.

9:41 And Jesus answering said, O faithless and perverse generation, how long shall I be with you, and
suffer you? Bring thy son hither.

9:42 And as he was yet a coming, the devil threw him down, and tare him. And Jesus rebuked the unclean
spirit, and healed the child, and delivered him again to his father.

9:43 And they were all amazed at the mighty power of God. But while they wondered every one at all
things which Jesus did, he said unto his disciples,

9:44 Let these sayings sink down into your ears: for the Son of man shall be delivered into the hands of
men.

9:45 But they understood not this saying, and it was hid from them, that they perceived it not: and they
feared to ask him of that saying.

9:46 Then there arose a reasoning among them, which of them should be greatest.

9:47 And Jesus, perceiving the thought of their heart, took a child, and set him by him,

9:48 And said unto them, Whosoever shall receive this child in my name receiveth me: and whosoever
shall receive me receiveth him that sent me: for he that is least among you all, the same shall be great.

9:49 And John answered and said, Master, we saw one casting out devils in thy name; and we forbad him,
because he followeth not with us.

9:50 And Jesus said unto him, Forbid him not: for he that is not against us is for us.

Jesus goes to Jerusalem (through Samaria, others want to follow) (51-62)


9:51 And it came to pass, when the time was come that he should be received up, he stedfastly set his face
to go to Jerusalem,

9:52 And sent messengers before his face: and they went, and entered into a village of the Samaritans, to
make ready for him.

http://breachrepairers.webs.com/ 457
9:53 And they did not receive him, because his face was as though he would go to Jerusalem.

Because of the prejudice, they told Him not to come. They didn’t realize what they were doing to
miss at that time.
DA 487 It is no part of Christ's mission to compel men to receive Him. It is Satan, and men actuated
by his spirit, that seek to compel the conscience. Under a pretense of zeal for righteousness, men who
are confederate with evil angels bring suffering upon their fellow men, in order to convert them to
their ideas of religion; but Christ is ever showing mercy, ever seeking to win by the revealing of His
love.” –

When we have external forms of religion, it’s often to hide what is lacking inside.

Luke 14:23

Jesus never forced. We can look at this word “compel” in the original language. Where else was the
similar idea conveyed?

Luke 24:29

9:54 And when his disciples James and John saw this, they said, Lord, wilt thou that we command fire to
come down from heaven, and consume them, even as Elias did?

9:55 But he turned, and rebuked them, and said, Ye know not what manner of spirit ye are of.

9:56 For the Son of man is not come to destroy men's lives, but to save them. And they went to another
village.

9:57 And it came to pass, that, as they went in the way, a certain man said unto him, Lord, I will follow
thee whithersoever thou goest.

Judas had a problem with worldly greatness. We know this because Jesus made a statement about
His poverty. However, Jesus never asked Judas to leave. But He always made it clear what Jesus had
come to do.

9:58 And Jesus said unto him, Foxes have holes, and birds of the air have nests; but the Son of man hath
not where to lay his head.

9:59 And he said unto another, Follow me. But he said, Lord, suffer me first to go and bury my father.

Jesus was testing whether they would be willing to sacrifice all to follow Him. There will always be a
point in time where we will be at our lowest. But it may be then that Jesus will ask us to follow Him.
Elisha had this similar experience first let me go back to my family first. Does God not like family?
Why wasn’t he allowed to bid farewell to His family? It was to show him that his calling had to be
first. His commission to the role of a prophet had to supreme. The first issue is that we must be
willing to forsake everything else to follow Jesus.

9:60 Jesus said unto him, Let the dead bury their dead: but go thou and preach the kingdom of God.

That is not literal because the dead know not anything. How do we know that it is referring to those
that are spiritually dead? The next thing that is mentioned is to go and preach the kingdom of God.

9:61 And another also said, Lord, I will follow thee; but let me first go bid them farewell, which are at
home at my house.

http://breachrepairers.webs.com/ 458
9:62 And Jesus said unto him, No man, having put his hand to the plough, and looking back, is fit for the
kingdom of God.

This is very similar to the call of Elisha.

“looking back” – Lot’s wife. This is a salvational issue, because then God says to us that we are not
fit for His kingdom. Be careful not to look at people at what you want to be like in ministry.

http://breachrepairers.webs.com/ 459
Chapter 10 –
Chapter Outline
 Jesus sends out 70 disciples (1-24)
 How to inherit eternal life (Good Samaritan) (25-37)
 Mary and Martha receive Jesus (38-42)

Jesus sends out 70 disciples (1-24)


10:1 After these things the Lord appointed other seventy also, and sent them two and two before his face
into every city and place, whither he himself would come.

Gospel ministry – laboring two and two. They compliment each other.

10:2 Therefore said he unto them, The harvest truly is great, but the labourers are few: pray ye therefore
the Lord of the harvest, that he would send forth labourers into his harvest.

10:3 Go your ways: behold, I send you forth as lambs among wolves.

What did this expression indicate? It suggested difficulty and danger in their paths. Jesus didn’t use
an animal that had any defense – lamb.

10:4 Carry neither purse, nor scrip, nor shoes: and salute no man by the way.

“Purse” – Carries money

“Scrip” – Carries bread.

“Shoes” – Clothing.

Luke 22:35-38

Parallel passage. There are 12 disciples and Peter said here are 2 swords and Jesus said it is
enough.It could not have been a literal sword. The ministry of Jesus had changed. The message that
they were to bear now was to be different. It was to be a cutting message.

10:5 And into whatsoever house ye enter, first say, Peace be to this house.

10:6 And if the son of peace be there, your peace shall rest upon it: if not, it shall turn to you again.

10:7 And in the same house remain, eating and drinking such things as they give: for the labourer is
worthy of his hire. Go not from house to house.

They that which are ministers of the gospel should live the gospel.

10:8 And into whatsoever city ye enter, and they receive you, eat such things as are set before you:

10:9 And heal the sick that are therein, and say unto them, The kingdom of God is come nigh unto you.

10:10 But into whatsoever city ye enter, and they receive you not, go your ways out into the streets of the

http://breachrepairers.webs.com/ 460
same, and say,

Why say this? They need to understand the solemnity that when they are rejecting you, they are
rejecting the gospel message.

10:11 Even the very dust of your city, which cleaveth on us, we do wipe off against you: notwithstanding
be ye sure of this, that the kingdom of God is come nigh unto you.

10:12 But I say unto you, that it shall be more tolerable in that day for Sodom, than for that city.

10:13 Woe unto thee, Chorazin! woe unto thee, Bethsaida! for if the mighty works had been done in Tyre
and Sidon, which have been done in you, they had a great while ago repented, sitting in sackcloth and
ashes.

10:14 But it shall be more tolerable for Tyre and Sidon at the judgment, than for you.

10:15 And thou, Capernaum, which art exalted to heaven, shalt be thrust down to hell.

10:16 He that heareth you heareth me; and he that despiseth you despiseth me; and he that despiseth me
despiseth him that sent me.

10:17 And the seventy returned again with joy, saying, Lord, even the devils are subject unto us through
thy name.

10:18 And he said unto them, I beheld Satan as lightning fall from heaven.

10:19 Behold, I give unto you power to tread on serpents and scorpions, and over all the power of the
enemy: and nothing shall by any means hurt you.

10:20 Notwithstanding in this rejoice not, that the spirits are subject unto you; but rather rejoice,
because your names are written in heaven.

10:21 In that hour Jesus rejoiced in spirit, and said, I thank thee, O Father, Lord of heaven and earth, that
thou hast hid these things from the wise and prudent, and hast revealed them unto babes: even so,
Father; for so it seemed good in thy sight.

10:22 All things are delivered to me of my Father: and no man knoweth who the Son is, but the Father;
and who the Father is, but the Son, and he to whom the Son will reveal him.

10:23 And he turned him unto his disciples, and said privately, Blessed are the eyes which see the things
that ye see:

10:24 For I tell you, that many prophets and kings have desired to see those things which ye see, and
have not seen them; and to hear those things which ye hear, and have not heard them.

How to inherit eternal life (Good Samaritan) (25-37)

What question lead to the telling of this parable? What shall I do to inherit eternal life?
Therefore, this parable is going to give us important information about what we need to do to
inherit eternal life.

What kind of lawyer is the person asking the question? Look up the word in the
concordance.

http://breachrepairers.webs.com/ 461
Luke 10:25

From G3551; according (or pertaining) to law, that is, legal (ceremonially); as noun, an expert in the
(Mosaic) law: - about the law, lawyer. This person is a doctor of the law.

What does the word tempt mean? Look up the word in the concordance.

Luke 10:25

From G1537 and G3985; to test thoroughly: - tempt. This doctor of the law was trying to thoroughly
test Jesus.

What does Jesus immediately point the doctor to? What application can we draw from this?

Luke 10:26

Jesus points the doctor back to the law. Application: Somehow, the keeping of the law is important
to inheriting eternal life. Question: What about the law is important? Keeping it. Many people say
the law was done away with. If it was truly done away with, then why would Jesus link it back to the
lawyer’s question of what must we do to inherit eternal life. Remember, the keeping of the law
won’t save, but the breaking of it will condemn us. We are saved by grace, not works. Yet works is
still important to a Christian’s life.

What was the lawyer’s answer? Where was he quoting from?

Deut 6:1-5

Love God with all thy heart.

Lev 19:18

Love thy neighbor as thyself. These were all from the Old Testament. Many people think that this is
only a New Testament commandment and that these replaced the 10 commandments.

How does the lawyer’s answer relate to God’s law, the Ten Commandments?

John 14:15

Rom 13:10

The first 4 commandments relate to our love relationship with God. Go through each
commandment. Eg. If we love God, we would not worship another god, nor would we create graven
images because God is a living god – imagine if we just talked to an image of our friend, nor would
we use God’s name in vain. If we loved someone, would we forget their birthday? The last 6
commandments relate to how we should love our neighbors. Go through the last 6. There is also a
process: If we don’t love God, we won’t keep the first 4 commandments. If we don’t love God and
keep the first 4 commandments, we won’t be compelled to love our neighbors. If we don’t love our
neighbors we won’t truly be compelled to keep the last 6 commandments.

Was the lawyer satisfied with Jesus’ reply?

http://breachrepairers.webs.com/ 462
Luke 10:29

No. He asked another question because he wanted to justify himself. He felt a need to justify himself
because he knew he was not keeping the commandments. When a person tries to justify himself,
they try to put themselves into a light which helps them to think they are doing right so that their
own conscience won’t condemn them. Example – when your mother tells you not to eat the cake
she baked and yet you do, a person may try to justify themselves by making excuses such as “I was
hungry,” “I didn’t have breakfast,” “You didn’t tell me properly or I didn’t hear it properly.” That is
self justification.

What was Jesus’ purpose for telling this parable?

Luke 10:29

To answer the lawyer’s question about who is his neighbor. What happened to the ‘certain man’?
Who does he represent? The man fell among thieves, was robbed, stripped of his raiment,
wounded, left half dead. The man represents those who are injured in life. How? Through
circumstances, people, etc. He also represents the whole human race – because we were robbed by
Satan, our garments – Christ’s righteousness was taken away (Adam and Eve were naked), and we
were left to die from the consequences of sin – the wages of sin is death.

Why didn’t the priest and Levite stop to help the stranger? They probably felt it was beneath
themselves. They had more important tasks such as religious duties.

Who do the priest and Levite represent today? Church leaders who are more concerned with
their priestly duties than helping those in need. But also all Christians

1 Pet 2:9

What is a Levite – they were the tribe that was chosen for the services of the sanctuary. Therefore,
the priest and levites also represent God’s people / spiritual Jews who are more interested in
performing church duties such as preaching, teaching, and other more external religious duties,
than the less noticeable, private acts of kindness.

Why did the Samaritan help the injured stranger? What can we imply about the priest and
Levite then?

Luke 10:33

He was moved with compassion. The priest and the Levite had no compassion.

What is a Samaritan and who does it represent?

John 4:9

Samaritan is an inhabitant of Samaria whom the Jews hated. They were considered as outcasts and
dogs and the Jews wished to have no interaction with them. Those who are not even God’s people,
but have shown acts of kindness to God’s people. Also represents Christ, because He came to save
us, fallen humanity, who have been robbed of our inheritance and left to die from the death

http://breachrepairers.webs.com/ 463
sentence of sin.

What did the Samaritan do that distinguished him from the priest and Levite? What do these
actions teach us?

Luke 10:33-37

1. He had compassion. It begins with our heart first. Has it been changed by Christ. 2. He bound up
the man’s wounds – sometimes we have to help heal a person’s physical needs first. 3. He poured in
oil and wine – oil = represents Holy Spirit (Zech. 4:1-6), wine = blood of Jesus which cleanses us
from all sin (Luke 22:20; Rev. 1:5). After you have ministered to a person physically, minister to
them also spiritually and show them Christ. 4. He set the man on his own beast and brought him to
an inn and took care of him – the Samaritan carried him, bore his burden on his ass – we have to be
willing to help bear other people’s burdens. 5. He payed for the man’s stay at the hotel – sometimes
it takes our own monetary possessions to minister to a person. Self sacrificing effort. The lawyer
summarizing mentioned that the neighbor is someone we show mercy to.

What principle was Christ trying to illustrate in this parable?

Gal 5:14

To inherit eternal life, we must through God’s help keep His commandments. In order to keep God’s
commandments, we need to learn to love God and love our neighbor. To love our neighbor and keep
God’s commandments, we must first learn that our neighbor also includes those whom we may hate
because of racial or other types of prejudice, those who are left dying by the road of life. To love our
neighbor means to show compassion and mercy to these people. To sacrifice our religious duties
sometimes and sacrifice time to help those in need.

10:25 And, behold, a certain lawyer stood up, and tempted him, saying, Master, what shall I do to inherit
eternal life?

What is the requirement with this situation and the rich young ruler? Obedience to the
commandments.

10:26 He said unto him, What is written in the law? how readest thou?

10:27 And he answering said, Thou shalt love the Lord thy God with all thy heart, and with all thy soul,
and with all thy strength, and with all thy mind; and thy neighbour as thyself.

We know this reference is related to the law that it is our duty to God and our duty to man. The
common thing was that the Pharisees performed all these ceremonies for outward appearance. The
good Samaritan was a true story. The Levite and the priest was there in the audience. At the same
time it is a parable.

10:28 And he said unto him, Thou hast answered right: this do, and thou shalt live.

10:29 But he, willing to justify himself, said unto Jesus, And who is my neighbour?

10:30 And Jesus answering said, A certain man went down from Jerusalem to Jericho, and fell among
thieves, which stripped him of his raiment, and wounded him, and departed, leaving him half dead.

http://breachrepairers.webs.com/ 464
The story of the good Samaritan is very much related to the fall of man.

“A certain [man] went down from Jerusalem to Jericho” – Jerusalem as a lot higher. Going from
a city of God (peace) down into Jericho.

Jerusalem = "set ye double peace"

Gr 2410 “Jericho = "place of fragrance." A noted city, abounding in balsam, honey, cyprus,
myrobalanus, roses and other fragrant products. It was near the north shore of the Dead Sea in the
tribe of Benjamin, between Jerusalem and the Jordan River

“fell among thieves, which stripped him of his raiment, and wounded [him], and departed,
leaving [him] half dead” – It was his fault that he traveled alone.

“Raiment” – lost his righteousness.

“Half dead” – man was now mortal.

10:31 And by chance there came down a certain priest that way: and when he saw him, he passed by on
the other side.

“Priests” – representing the sanctuary and ceremonial services.

10:32 And likewise a Levite, when he was at the place, came and looked on him, and passed by on the
other side.

“Levite” – what is the difference between the priest and the Levite? What was the role of the
Levite? They were expositors of the law.

Application: Ceremonies cannot save you and works of the law cannot save you either.

10:33 But a certain Samaritan, as he journeyed, came where he was: and when he saw him, he had
compassion on him,

Jesus was considered a Samaritan

John 8:48

10:34 And went to him, and bound up his wounds, pouring in oil and wine, and set him on his own beast,
and brought him to an inn, and took care of him.

Samaritan. They were a mixture of Jews and idol worshippers.

Instead of the Israelites transporting into Assyria, colonies were sent over, formed of various nations,
Chaldeans, Cutheans, Syrians, Arabs, and others; these mingled with the native population, forming
an amalgamation of religion and superstition; thus the Israelites with their own national worship
gave birth to the people and the religion of the Samaritans.

“Oil” – Representing the Holy Spirit - Sanctification.

“Wine” – Representing Jesus’ blood - Justification. Justification and sanctification.

http://breachrepairers.webs.com/ 465
“Beast” – Representing the church.

10:35 And on the morrow when he departed, he took out two pence, and gave them to the host, and said
unto him, Take care of him; and whatsoever thou spendest more, when I come again, I will repay thee.

“Two pence” – The old and the new testament.

10:36 Which now of these three, thinkest thou, was neighbour unto him that fell among the thieves?

10:37 And he said, He that shewed mercy on him. Then said Jesus unto him, Go, and do thou likewise.

The main point of the chapter on the good Samaritan is to show true religion.

DA 497 In the story of the good Samaritan, Christ illustrates the nature of true religion. He shows
that it consists not in systems, creeds, or rites, but in the performance of loving deeds, in bringing the
greatest good to others, in genuine goodness.

DA 498 The way to dispel darkness is to admit light. The best way to deal with error is to present
truth.

DA 503 Thus the question, "Who is my neighbor?" is forever answered. Christ has shown that our
neighbor does not mean merely one of the church or faith to which we belong. It has no reference to
race, color, or class distinction. Our neighbor is every person who needs our help. Our neighbor is
every soul who is wounded and bruised by the adversary. Our neighbor is everyone who is the
property of God.

DA 504 Sin is the greatest of all evils, and it is ours to pity and help the sinner. There are many who
err, and who feel their shame and their folly. They are hungry for words of encouragement. They look
upon their mistakes and errors, until they are driven almost to desperation. These souls we are not to
neglect. If we are Christians, we shall not pass by on the other side, keeping as far as possible from
the very ones who most need our help. When we see human beings in distress, whether through
affliction or through sin, we shall never say, This does not concern me.

DA 504 “Ye which are spiritual, restore such an one in the spirit of meekness." Gal. 6:1. By faith and
prayer press back the power of the enemy. Speak words of faith and courage that will be as a healing
balsam to the bruised and wounded one. Many, many, have fainted and become discouraged in the
great struggle of life, when one word of kindly cheer would have strengthened them to overcome.
Never should we pass by one suffering soul without seeking to impart to him of the comfort
wherewith we are comforted of God.”

Mary and Martha receive Jesus (38-42)


10:38 Now it came to pass, as they went, that he entered into a certain village: and a certain woman
named Martha received him into her house.

What do we know about Martha from the Bible? Who could she represent in this story? She
was sister of Mary and Lazarus. She is the person who initiates things. Martha represents those who
are very busy people, probably leaders. Ones who like to initiate things.

What did she ask Jesus to do? What does this tell us about her character? She asked Jesus to
come to her house. She had the gift of hospitality. Also she tends to initiate things: She invites Jesus
to her house. She is the first out of the two sisters to approach Jesus when Lazarus dies. (John
11:20)

http://breachrepairers.webs.com/ 466
What do we know about Mary from the Bible?

John 12:2-3

John 19:25

John 20:1

Sister of Martha and Lazarus. She was the one who washed Jesus feet with expensive perfume. She
was at the cross when Jesus died. She was the first to the tomb on Sunday morning.

10:39 And she had a sister called Mary, which also sat at Jesus' feet, and heard his word.

What are we to do at Jesus’ feet from Mary’s example? By sitting at the feet of Jesus, it is
referring to hearing His words. How can we practically do that? Studying the Bible ourselves,
hearing sermons. But in this case, a personal time, hearing Jesus’ words is emphasized.

What else does it mean to be at Jesus’ feet? What lessons can we learn?

Luke 8:41

We can “besought”. We must have a beseeching attitude at Jesus’ feet. What does that mean? Plead,
beg. Plead for what?

Mark 1:40

Matt 8:5

Mar 7:32

Every time the word beseeching is mentioned in reference to Jesus, it is in reference to asking or
pleading for healing or to be made clean. Therefore, at the feet of Jesus we can beseech for our own
physical or spiritual healing and the healing of others. We call this prayer and repentance.

Matt 15:30

Luke 8:35

We can receive healing and be in the right frame of mind by being at the feet of Jesus. Therefore, by
hearing Christ’s word and begging for healing, Christ will answer and heal us, and put us in the right
frame of mind.

John 11:32

We can lay our grief at Jesus feet.

John 12:3

Mark 14:8

Anoint Jesus. What does it mean to anoint someone? Christ told everyone that by anointing his feet,

http://breachrepairers.webs.com/ 467
Mary was anointing him for his death. Therefore, it means to remember the death of Christ. His
crucifixion. Lesson: Its important for us to regularly remember the price that was paid by Jesus for
our salvation. It is the power unto salvation (1 Cor 1:18, Rom 1:16)

Luke 17:16

Worship and praise God for healing us. Lesson: Be thankful and realize what God has healed us
from. We cannot do this unless we realize the severe price that was paid at the cross.

10:40 But Martha was cumbered about much serving, and came to him, and said, Lord, dost thou not
care that my sister hath left me to serve alone? bid her therefore that she help me.

Who was Martha serving? What lead to Martha getting upset? Martha was serving Jesus.
Martha was cumbered with much serving. What does cumbered mean? Burdened, hindered,
obstructed.

What lessons can we learn from Martha’s experience?


1. It is possible to be serving Jesus and yet cumbered with too much serving.
2. If we continue to persist with too much serving, we will start to become bitter or angry,
pointing at others who are sitting at Jesus feet. We will also cry out to Jesus, criticizing him
for not commanding our brothers and sisters to help.
3. We begin to think that Jesus does not care. Eventually we will burn out.

10:41 And Jesus answered and said unto her, Martha, Martha, thou art careful and troubled about many
things:

“careful” – full of care, worried. Application: These are people who have invited Jesus into their
home. So we can see that we are talking about 2 kinds of Christians. We need both Martha’s and
Mary’s.

Martha was busy serving Jesus. But many are working for the Lord at the neglect of sitting at His
feet. No service is effective if we have not made a connection with Christ.

Mary was commended: Because she sat at His feet and heard His words.

“sat at Jesus’ feet” – a learner. Willing to learn. Humility.

10:42 But one thing is needful: and Mary hath chosen that good part, which shall not be taken away
from her.

From His response, what is more important to Jesus? What does that teach us about serving?
According to Jesus response, it is more important to spend time at Jesus feet first. What does this
teach us about serving?
1. We need to know Him first, before we serve Him. This includes our daily life. It is not much
point serving Christ, when you did not even spend time at His feet first.
Note: There will come a time at the end, when Jesus will say to those who just serve, but
never took the time to hear, that He never knew them. (Matt 7:23)
2. (NOTE to church leaders especially) Before we get new members to serve in church, we
should make sure they are well acquainted with Jesus and have learned to spend time at His

http://breachrepairers.webs.com/ 468
feet. We may cause them to leave the church if we force them to serve at the wrong time. We
must all be sensitive to each individual’s needs. Some need to spend more time at Christ’s
feet and should not be criticized for doing so.

Does this mean that its not important to serve? Is hearing God’s word enough?

Jas 1:22-25

No. Its just as important for us to be doers of the Word. Otherwise we will soon forget what we have
heard or read. Christianity is practical, not just theory. We must practice what we preach!

Who should we be more like then? Martha or Mary? Neither. We should learn to be a balance of
both. You cannot just sit and hear God’s word. Neither can you be a doer of the word without first
sitting at Jesus’ feet. NOTE: because Mary spent more intimate time with Christ, she loved Him
more, and this is shown in how she treated Christ. She knew His heart better. (Ref to comments on
Mark 10:38).

“Mary hath chosen that good part, which shall not be taken away from her” – Those words
that He was teaching Mary would be the thing that would grant to her eternal life. He priority is that
we should have God’s word abiding in our hearts and then work. Because works are also important.

http://breachrepairers.webs.com/ 469
Chapter 11

Chapter Outline
 Pray that God's kingdom comes (Unity) (1-13)
 Satan’s kingdom is divided (14-28)
 Sign of Jonas the Prophet & Judgment (29-36)
 Woes to Pharisees, Scribes, and Lawyers (37-54)

Mark 11:15 He casteth out devils through Beelzebub

He addresses this topic in verses 17-28

Mark 11:16 sought of him a sign from heaven

He address this in verses 29-36

Observations:

 Kingdom
o God's kingdom is united, pray for this
o Satan's kingdom cannot stand divided
 Fullness
o Eye evil, body full of darkness, eye single, body full of light. (deliver us from evil,
deliver us from seeking signs, which ultimately prevents us from being condemned
in the judgment.)
o Pharisees are full of wickedness
o People full of unclean spirits

Pray that God's kingdom comes (Unity) (1-13)


11:1 And it came to pass, that, as he was praying in a certain place, when he ceased, one of his disciples
said unto him, Lord, teach us to pray, as John also taught his disciples.

The Hour of Temptation

Lord's Prayer

Lk 11:1-4

Mat 6:13

Unity, reconciliation, Kingdom of God comes

Lk 11:4 Lead us not into temptation…

Last Scenes in the Life of Christ to be repeated

Lk 11:3 …bread…

http://breachrepairers.webs.com/ 470
Lk 22:19 …took bread…

In the upper room

Lk 11:2 …thy will be done…

Lk 22:42 …not my will…

Geths/Betrayal/Kings

Lk 11:4 …forgive everyone…

Lk 23:34 …forgive them…

Cross, death of Jesus. Note: Jesus is walking them through the last scenes of His life. And
these will be repeated in our day.

Lk 21:12 …they…kings and rulers…

Note: Who are they? V.8 False Christ's; & V.10 Nations, you have a union between the religious
bodies and the political bodies. They are delivering us to churches and kings.

Lk 21:16

Betrayal & Death.

The Hour of temptation

Lk 22:1-23; 31-34; 39-46; 54-62

Betrayal and Denial of Peter

Rev 3:8-12

Temptation, Test, Sealing

Rev 3:12 …name of my God

Seal of God

Rev 14:1 …Father's name…foreheads…

Rev 7:3 …Sealed…foreheads…

Rev 13:3, 8, 16

Temptation and test is over the Sabbath Vs. Sunday issue. Note: The hour of temptation comes after
the loud cry.

See EGW Quotes. Note: The hour of temptation is the test that God's people must endure upon pain
of death. After the hour of temptation God's people are sealed.

http://breachrepairers.webs.com/ 471
Appeal

Luke 21:36 Watch ye therefore, and pray always, that ye may be accounted worthy to escape all
these things that shall come to pass, and to stand before the Son of man.

Luke 22:40 Pray that ye enter not into temptation…

The Hour of Temptation

EGW Quotes:

Scenes of Rejection Repeated -- When Christ was upon this earth, the world preferred Barabbas. And
today the world and the churches are making the same choice. The scenes of the betrayal, the
rejection, and the crucifixion of Christ have been re-enacted, and will again be re-enacted on an
immense scale. {RH, January 30, 1900 par. 8}[Added 6-3-05]

Name of My God is Seal – " The 144,000 were all sealed and perfectly united. On their foreheads was
written, God, New Jerusalem, and a glorious star containing Jesus' new name." EW 15

Hour of temptation after Loud Cry – " The Loud Cry… I saw another mighty angel commissioned to
descend to the earth, to unite his voice with the third angel, and give power and force to his
message… The work of this angel comes in at the right time to join in the last great work of the third
angel's message as it swells to a loud cry. And the people of God are thus prepared to stand in the
hour of temptation, which they are soon to meet." EW 277

Hour of temptation after Loud Cry 2 – " God's people were strengthened by the excellent glory which
rested upon them in rich abundance and prepared them to endure the hour of temptation." EW 279

Hour of temptation defined – " This message [The Third Angel's Message] was designed to put the
children of God upon their guard, by showing them the hour of temptation and anguish that was
before them. Said the angel, "They will be brought into close combat with the beast and his image.
Their only hope of eternal life is to remain steadfast. Although their lives are at stake, they must
hold fast the truth." EW 254

11:2 And he said unto them, When ye pray, say, Our Father which art in heaven, Hallowed be thy name.
Thy kingdom come. Thy will be done, as in heaven, so in earth.

11:3 Give us day by day our daily bread.

11:4 And forgive us our sins; for we also forgive every one that is indebted to us. And lead us not into
temptation; but deliver us from evil.

11:5 And he said unto them, Which of you shall have a friend, and shall go unto him at midnight, and say
unto him, Friend, lend me three loaves;

What is the theme of this passage? These parables are about prayer.

Who does the friend represent?

http://breachrepairers.webs.com/ 472
Luke 11:5

“Friend” = God. It has a capital ‘F’ at the end of verse 5. Bread represents not just physical needs,
but according to Matt 4, it also represents Jesus, the bread of life.

11:6 For a friend of mine in his journey is come to me, and I have nothing to set before him?

What is the lesson learnt in verse 6?

Luke 11:6

When we ask, it should not be for selfish purpose. This man wanted 3 loaves to feed someone else.
Not for his own personal gain. Our prayers should be consumed with thoughts of how to get the
bread of life into other’s hands. John 17:19 – Christ’s attitude as always one of self sacrifice.

11:7 And he from within shall answer and say, Trouble me not: the door is now shut, and my children are
with me in bed; I cannot rise and give thee.

What aspect of prayer can we learn from verse 7?

Luke 11:7

Sometimes God gives us no reply OR negative reply to test our desire or intentions of the heart.
Sometimes he may even permit trials to come our way which seem to point us to a NO answer just
to test our faith.

11:8 I say unto you, Though he will not rise and give him, because he is his friend, yet because of his
importunity he will rise and give him as many as he needeth.

Why is prayer answered according to verse 8?

Luke 11:8

Importunity = persistence (to the point of annoyance). When we ask in prayer, we must be
persistent.
What attitude should we have to be persistent? Faith. We must really believe in what we are asking
for. Real desire. We will only keep asking for something that we really really really want. Think
about a child who has his mind on some toys. They just keep asking for it.

What is the result?

Luke 11:8

God gives as many as he needeth. Needeth for what? To feed others. Notice it talks about one friend
helping another friend to feed his friends. This is friendship evangelism. You can’t pray for someone
who doesn’t mean anything to you.

What is the criteria for asking?

Matt 21:22

http://breachrepairers.webs.com/ 473
Jn 14:15, 21

Jn 15:7

1 Jn 2:3-5

1 Jn 3:22

1 Jn 5:14-15

Ask and believe. What does it mean to believe?

1 John 5:1-6

In order to ask in Christ’s name, you must believe in Him.

What does it mean to seek?

Matt 6:33

If you are not seeking the kingdom of God, then your prayers will not be answered. God is only
looking for those who have an interest in the principals and character that fit His kingdom.

What does it mean to knock?

John 10:7

Jesus is the door. If we are not knocking through Him, then God will not hear us.

Rev 3:8

And Jesus the door will not open if we do not keep His word and do not deny His name.

What else is there that will keep God from hearing us?

Mal 3:7-8, 10-12

God will bless if we first give. Otherwise God will withhold.

11:9 And I say unto you, Ask, and it shall be given you; seek, and ye shall find; knock, and it shall be
opened unto you.

11:10 For every one that asketh receiveth; and he that seeketh findeth; and to him that knocketh it shall
be opened.

11:11 If a son shall ask bread of any of you that is a father, will he give him a stone? or if he ask a fish,
will he for a fish give him a serpent?

11:12 Or if he shall ask an egg, will he offer him a scorpion?

According to Luke 11:11-12 how are we to treat prayer? As asking our Father. God is to be a

http://breachrepairers.webs.com/ 474
father to us. What does it mean to treat God like a father? (1) Intimacy. (2) That He has our best
interests at heart. Can you imagine calling the King of the universe Father? Do you even realize
what a privilege that is?

11:13 If ye then, being evil, know how to give good gifts unto your children: how much more shall your
heavenly Father give the Holy Spirit to them that ask him?

Contextually, what gifts has God given us according to the verse?

Luke 11:13

God’s gift comes through the Holy Spirit. We must pray for the Holy Spirit. He is also involved in our
praying (Rom 8:26). We must ask for the Holy Spirit’s inspiration in our prayers.

What are the benefits of this gift?

Fruit of the Spirit = Gal 5:22

Gifts = Rom 12:6, 1 Cor 12:1-13

Conclusion:

1. Pray unselfishly
2. Pray persistently
3. Expect trials and prolonged response for our character dev
4. Ask and believe (keep God’s commandments)
5. Make sure you are seeking the Kingdom of God first
6. Knock through Jesus Christ
7. Make sure we are not robbing God through tithe and offering
8. Treat God as Father (know the privilege of this)
9. Expect gifts through the Holy Spirit
a. gift of transformed character
b. ministry gifts – skills and talents

Satan’s kingdom is divided (14-28)


11:14 And he was casting out a devil, and it was dumb. And it came to pass, when the devil was gone out,
the dumb spake; and the people wondered.

11:15 But some of them said, He casteth out devils through Beelzebub the chief of the devils.

11:16 And others, tempting him, sought of him a sign from heaven.

11:17 But he, knowing their thoughts, said unto them, Every kingdom divided against itself is brought to
desolation; and a house divided against a house falleth.

11:18 If Satan also be divided against himself, how shall his kingdom stand? because ye say that I cast
out devils through Beelzebub.

11:19 And if I by Beelzebub cast out devils, by whom do your sons cast them out? therefore shall they be
your judges.

http://breachrepairers.webs.com/ 475
11:20 But if I with the finger of God cast out devils, no doubt the kingdom of God is come upon you.

11:21 When a strong man armed keepeth his palace, his goods are in peace:

11:22 But when a stronger than he shall come upon him, and overcome him, he taketh from him all his
armour wherein he trusted, and divideth his spoils.

11:23 He that is not with me is against me: and he that gathereth not with me scattereth.

11:24 When the unclean spirit is gone out of a man, he walketh through dry places, seeking rest; and
finding none, he saith, I will return unto my house whence I came out.

11:25 And when he cometh, he findeth it swept and garnished.

11:26 Then goeth he, and taketh to him seven other spirits more wicked than himself; and they enter in,
and dwell there: and the last state of that man is worse than the first.

11:27 And it came to pass, as he spake these things, a certain woman of the company lifted up her voice,
and said unto him, Blessed is the womb that bare thee, and the paps which thou hast sucked.

11:28 But he said, Yea rather, blessed are they that hear the word of God, and keep it.

Sign of Jonas the Prophet & Judgment (29-36)


11:29 And when the people were gathered thick together, he began to say, This is an evil generation:
they seek a sign; and there shall no sign be given it, but the sign of Jonas the prophet.

11:30 For as Jonas was a sign unto the Ninevites, so shall also the Son of man be to this generation.

11:31 The queen of the south shall rise up in the judgment with the men of this generation, and condemn
them: for she came from the utmost parts of the earth to hear the wisdom of Solomon; and, behold, a
greater than Solomon is here.

11:32 The men of Nineve shall rise up in the judgment with this generation, and shall condemn it: for
they repented at the preaching of Jonas; and, behold, a greater than Jonas is here.

11:33 No man, when he hath lighted a candle, putteth it in a secret place, neither under a bushel, but on
a candlestick, that they which come in may see the light.

11:34 The light of the body is the eye: therefore when thine eye is single, thy whole body also is full of
light; but when thine eye is evil, thy body also is full of darkness.

11:35 Take heed therefore that the light which is in thee be not darkness.

11:36 If thy whole body therefore be full of light, having no part dark, the whole shall be full of light, as
when the bright shining of a candle doth give thee light.

Woes to Pharisees, Scribes, and Lawyers (37-54)


11:37 And as he spake, a certain Pharisee besought him to dine with him: and he went in, and sat down
to meat.

11:38 And when the Pharisee saw it, he marvelled that he had not first washed before dinner.

11:39 And the Lord said unto him, Now do ye Pharisees make clean the outside of the cup and the platter;

http://breachrepairers.webs.com/ 476
but your inward part is full of ravening and wickedness.

11:40 Ye fools, did not he that made that which is without make that which is within also?

11:41 But rather give alms of such things as ye have; and, behold, all things are clean unto you.

11:42 But woe unto you, Pharisees! for ye tithe mint and rue and all manner of herbs, and pass over
judgment and the love of God: these ought ye to have done, and not to leave the other undone.

11:43 Woe unto you, Pharisees! for ye love the uppermost seats in the synagogues, and greetings in the
markets.

11:44 Woe unto you, scribes and Pharisees, hypocrites! for ye are as graves which appear not, and the
men that walk over them are not aware of them.

11:45 Then answered one of the lawyers, and said unto him, Master, thus saying thou reproachest us
also.

11:46 And he said, Woe unto you also, ye lawyers! for ye lade men with burdens grievous to be borne,
and ye yourselves touch not the burdens with one of your fingers.

11:47 Woe unto you! for ye build the sepulchres of the prophets, and your fathers killed them.

11:48 Truly ye bear witness that ye allow the deeds of your fathers: for they indeed killed them, and ye
build their sepulchres.

11:49 Therefore also said the wisdom of God, I will send them prophets and apostles, and some of them
they shall slay and persecute:

11:50 That the blood of all the prophets, which was shed from the foundation of the world, may be
required of this generation;

11:51 From the blood of Abel unto the blood of Zacharias, which perished between the altar and the
temple: verily I say unto you, It shall be required of this generation.

11:52 Woe unto you, lawyers! for ye have taken away the key of knowledge: ye entered not in yourselves,
and them that were entering in ye hindered.

11:53 And as he said these things unto them, the scribes and the Pharisees began to urge him
vehemently, and to provoke him to speak of many things:

11:54 Laying wait for him, and seeking to catch something out of his mouth, that they might accuse him.

http://breachrepairers.webs.com/ 477
Chapter 12 - Jesus as the Judge!
Summary: The over riding thought in this chapter is that Jesus as a man will judge! You have one
concept that is repeated throughout this chapter. Division (or to separate). It is basically divided
into 3 parts: Jesus as judge; when He will work as judge; And the reward He will give as judge.

And this chapter explains who the two groups are: Those who confess and deny. It explains what
will be divided, the inheritance. And it explains when the inheritance will be divided, during the
wedding, and what the final reward will be: sit in the kingdom and eternal destruction. It also
explains where the division will take place: The family. Contextually speaking who is the wise and
evil servant? Wise = those who confess Christ before men. Evil = Those who deny before men.

Observation:

Mk 12:14 ...judge or divider over you?...

Mk 12:13 ...divide the inheritance...

Mk 12:32 ...give you the kingdom...

Mk 12:42 ...make ruler….give portion...

Mk 12:46 ...appoint him his portion...

Mk 12:51-53 ...division..divided...

Mk 12:57 ...judge what is right...

See sermon on Books of Record. Beware associated with pretending Matt 7:15, 10:17; Mk 12:38

Chapter Outline
 Who Jesus will confess before the angels (1-12)
 Jesus as judge or divider (warning against coveting) (13-21)
 Father's pleasure to give the kingdom (22-40)
 Faithful and evil servant (portions divided) (41-53)
 Judge among yourselves what is right (54-59)

Who Jesus will confess before the angels (1-12)


12:1 In the mean time, when there were gathered together an innumerable multitude of people,
insomuch that they trode one upon another, he began to say unto his disciples first of all, Beware ye of
the leaven of the Pharisees, which is hypocrisy.

12:2 For there is nothing covered, that shall not be revealed; neither hid, that shall not be known.

12:3 Therefore whatsoever ye have spoken in darkness shall be heard in the light; and that which ye
have spoken in the ear in closets shall be proclaimed upon the housetops.

12:4 And I say unto you my friends, Be not afraid of them that kill the body, and after that have no more
that they can do.

http://breachrepairers.webs.com/ 478
12:5 But I will forewarn you whom ye shall fear: Fear him, which after he hath killed hath power to cast
into hell; yea, I say unto you, Fear him.

12:6 Are not five sparrows sold for two farthings, and not one of them is forgotten before God?

12:7 But even the very hairs of your head are all numbered. Fear not therefore: ye are of more value
than many sparrows.

12:8 Also I say unto you, Whosoever shall confess me before men, him shall the Son of man also confess
before the angels of God:

12:9 But he that denieth me before men shall be denied before the angels of God.

12:10 And whosoever shall speak a word against the Son of man, it shall be forgiven him: but unto him
that blasphemeth against the Holy Ghost it shall not be forgiven.

12:11 And when they bring you unto the synagogues, and unto magistrates, and powers, take ye no
thought how or what thing ye shall answer, or what ye shall say:

12:12 For the Holy Ghost shall teach you in the same hour what ye ought to say.

Jesus as judge or divider (warning against coveting) (13-21)


12:13 And one of the company said unto him, Master, speak to my brother, that he divide the inheritance
with me.

12:14 And he said unto him, Man, who made me a judge or a divider over you?

12:15 And he said unto them, Take heed, and beware of covetousness: for a man's life consisteth not in
the abundance of the things which he possesseth.

12:16 And he spake a parable unto them, saying, The ground of a certain rich man brought forth
plentifully:

Why did Jesus give this parable? What was He dealing with and why did He have to deal with it?
Ans: Covetousness, and two brothers that were arguing over inheritance which wasn’t about how
fair they were, but about them being greedy and wanting the greater part of the inheritance

What does this teach us about the mission of Christ and also the mission of His disciples that are to
live today? Ans: We shouldn’t get involved with worldly disputes that involve selfish interests. We
should never get involved in family disputes.

12:17 And he thought within himself, saying, What shall I do, because I have no room where to bestow
my fruits?

According to the parable, where does covetousness begin? Ans: He said within himself.
Covetousness begins in the heart.

What enabled this man to build bigger barns which showed that he was covetous? Ans: Prosperous
land. Stress the following points: Was there anything wrong with having excess? No. Was there
anything wrong with the ground bringing forth plentifully? No

What lessons can we learn from the previous question?

http://breachrepairers.webs.com/ 479
Matt 19:23

1 Tim 6:10

Luke 12:34

Ans: He became covetous at the point when the ground brought forth plentifully. Sometimes, God
keeps us poor that He may save us into the kingdom. That’s why it says that hardly shall a rich man
enter into heaven. What should this man have done with the excess of food/wealth that he got that
year?

Matt 10:8
Ans: When we get things freely, we should give freely back. The problem with us is, the more we get
sometimes, the more we want. As soon as we receive we want to spend

According to this parable what does it mean to be covetous? Ans: Having goods that you rightfully
earned, but you didn’t give it to others for their benefit but stored it up for yourselves. So it goes
beyond just being greedy. Jesus is dealing with the root of the issue, which is selfishness. Greed is
just a manifestation of a heart of covetousness/selfishness.

Contextually, what is the meaning of a fool?

Psa 14:1-4

Ans: One that does not regard God, and as a result does not regard His children and the work
needed to be done on this earth

How will our soul be found wanting in the last days? And how does it relate to this parable?

Luke 12:20

Matt 25:31-46

Jas 1:27

Ans: When we fail to look out for those less fortunate than ourselves. This man had every
opportunity to do so, but instead, he hoarded it all for himself. Just thought about himself only.
What was the purpose behind Jesus’ question which He didn’t give an answer for - “whose shall
those things be, which thou has provided?” And how should this change our attitude on this earth
while we are alive? Discuss

Ans: The rich man’s fate was death. But after we die, all the things that we worked for come to
naught. Everything that we put our hard work into, will go others. Someone else will inherit it.
Sometimes, even to strangers.

Matt 6:19

It didn’t even benefit this man one bit in the after life with all that he stored up. When we work, we
should live balanced. Keep that which you need, and help others as much as possible. It’s our good
works that show what sort of character we are.

http://breachrepairers.webs.com/ 480
What does it mean to be rich toward God?

Luke 12:21, 33-34

Mark 10:21

Matt 6:19-20

1 Tim 6:17-19

In God’s eyes one is not rich when he lays treasures for himself, therefore the reverse is true, one is
truly rich when he sells his excess that he has and gives alms. Putting our treasures in the kingdom
of heaven. How do we do this? Sell all that we have. Take up the cross (sacrifice).Follow Christ. Then
we shall have treasure in heaven and be rich in God’s eyes. Rich in good works. Ready to distribute,
lay hold on eternal life

12:18 And he said, This will I do: I will pull down my barns, and build greater; and there will I bestow all
my fruits and my goods.

12:19 And I will say to my soul, Soul, thou hast much goods laid up for many years; take thine ease, eat,
drink, and be merry.

12:20 But God said unto him, Thou fool, this night thy soul shall be required of thee: then whose shall
those things be, which thou hast provided?

12:21 So is he that layeth up treasure for himself, and is not rich toward God.

Father's pleasure to give the kingdom (22-40)


12:22 And he said unto his disciples, Therefore I say unto you, Take no thought for your life, what ye
shall eat; neither for the body, what ye shall put on.

12:23 The life is more than meat, and the body is more than raiment.

12:24 Consider the ravens: for they neither sow nor reap; which neither have storehouse nor barn; and
God feedeth them: how much more are ye better than the fowls?

12:25 And which of you with taking thought can add to his stature one cubit?

12:26 If ye then be not able to do that thing which is least, why take ye thought for the rest?

12:27 Consider the lilies how they grow: they toil not, they spin not; and yet I say unto you, that Solomon
in all his glory was not arrayed like one of these.

12:28 If then God so clothe the grass, which is to day in the field, and to morrow is cast into the oven; how
much more will he clothe you, O ye of little faith?

12:29 And seek not ye what ye shall eat, or what ye shall drink, neither be ye of doubtful mind.

12:30 For all these things do the nations of the world seek after: and your Father knoweth that ye have
need of these things.

12:31 But rather seek ye the kingdom of God; and all these things shall be added unto you.

http://breachrepairers.webs.com/ 481
12:32 Fear not, little flock; for it is your Father's good pleasure to give you the kingdom.

12:33 Sell that ye have, and give alms; provide yourselves bags which wax not old, a treasure in the
heavens that faileth not, where no thief approacheth, neither moth corrupteth.

12:34 For where your treasure is, there will your heart be also.

12:35 Let your loins be girded about, and your lights burning;

12:36 And ye yourselves like unto men that wait for their lord, when he will return from the wedding;
that when he cometh and knocketh, they may open unto him immediately.

12:37 Blessed are those servants, whom the lord when he cometh shall find watching: verily I say unto
you, that he shall gird himself, and make them to sit down to meat, and will come forth and serve them.

12:38 And if he shall come in the second watch, or come in the third watch, and find them so, blessed are
those servants.

12:39 And this know, that if the goodman of the house had known what hour the thief would come, he
would have watched, and not have suffered his house to be broken through.

12:40 Be ye therefore ready also: for the Son of man cometh at an hour when ye think not.

Faithful and evil servant (portions divided) (41-53)


12:41 Then Peter said unto him, Lord, speakest thou this parable unto us, or even to all?

12:42 And the Lord said, Who then is that faithful and wise steward, whom his lord shall make ruler over
his household, to give them their portion of meat in due season?

12:43 Blessed is that servant, whom his lord when he cometh shall find so doing.

12:44 Of a truth I say unto you, that he will make him ruler over all that he hath.

12:45 But and if that servant say in his heart, My lord delayeth his coming; and shall begin to beat the
menservants and maidens, and to eat and drink, and to be drunken;

12:46 The lord of that servant will come in a day when he looketh not for him, and at an hour when he is
not aware, and will cut him in sunder, and will appoint him his portion with the unbelievers.

12:47 And that servant, which knew his lord's will, and prepared not himself, neither did according to
his will, shall be beaten with many stripes.

12:48 But he that knew not, and did commit things worthy of stripes, shall be beaten with few stripes.
For unto whomsoever much is given, of him shall be much required: and to whom men have committed
much, of him they will ask the more.

12:49 I am come to send fire on the earth; and what will I, if it be already kindled?

12:50 But I have a baptism to be baptized with; and how am I straitened till it be accomplished!

12:51 Suppose ye that I am come to give peace on earth? I tell you, Nay; but rather division:

12:52 For from henceforth there shall be five in one house divided, three against two, and two against

http://breachrepairers.webs.com/ 482
three.

12:53 The father shall be divided against the son, and the son against the father; the mother against the
daughter, and the daughter against the mother; the mother in law against her daughter in law, and the
daughter in law against her mother in law.

Judge among yourselves what is right (54-59)


12:54 And he said also to the people, When ye see a cloud rise out of the west, straightway ye say, There
cometh a shower; and so it is.

12:55 And when ye see the south wind blow, ye say, There will be heat; and it cometh to pass.

12:56 Ye hypocrites, ye can discern the face of the sky and of the earth; but how is it that ye do not
discern this time?

12:57 Yea, and why even of yourselves judge ye not what is right?

12:58 When thou goest with thine adversary to the magistrate, as thou art in the way, give diligence that
thou mayest be delivered from him; lest he hale thee to the judge, and the judge deliver thee to the
officer, and the officer cast thee into prison.

12:59 I tell thee, thou shalt not depart thence, till thou hast paid the very last mite.

http://breachrepairers.webs.com/ 483
Chapter 13
Why is Jerusalem going to perish? Because they would not repent. They would not be loosed from
their infirmity. They did have the experience of the mustard seed, leaven, and they didn't enter in at
the strait gate.

Note: Gives another picture of judgment. 3 years there was an investigation of the fig tree. No fruit.
When will people try to enter in the strait gate? When the Master shuts the door. Probation closes.

Chapter Outline
 Repent or perish (Pilate mingled blood) (1-10)
 Woman healed who had infirmity 18 years (On Sabbath) (10-17)
 Kingdom of God (Mustard Seed, Leaven, Strait gate) (18-30)
 Jerusalem left desolate (31-35)

Repent or perish (Pilate mingled blood-fig tree) (1-10)


13:1 There were present at that season some that told him of the Galilaeans, whose blood Pilate had
mingled with their sacrifices.

13:2 And Jesus answering said unto them, Suppose ye that these Galilaeans were sinners above all the
Galilaeans, because they suffered such things?

13:3 I tell you, Nay: but, except ye repent, ye shall all likewise perish.

13:4 Or those eighteen, upon whom the tower in Siloam fell, and slew them, think ye that they were
sinners above all men that dwelt in Jerusalem?

13:5 I tell you, Nay: but, except ye repent, ye shall all likewise perish.

13:6 He spake also this parable; A certain man had a fig tree planted in his vineyard; and he came and
sought fruit thereon, and found none.

Who is this parable directed to? What are their attitudes? To those who were judging this killed
by Pilate and the tower of Siloam. Their attitude was that those who had been killed by tragic
events were being judged severely by God. This was a message of reproof on the victim’s lives.
Those who were judging concluded themselves as favorites of God because they were not facing
such judgments.

COL 213 The Jews especially regarded calamity as a judgment on the account of the sufferer’s sin,
and those who told of this act of violence did so with secret satisfaction.

These people had a false view of God and His character. This parable will be told to correct that.

What 2 themes are linked with one another through this parable? Give some. Bible texts that
further prove this was Christ’s intent for coming to Earth?

Judgment & Mercy.

Luke 9:56

http://breachrepairers.webs.com/ 484
John 3:17

Christ did not come to condemn but to show mercy. Inevitably there will be judgment.

What event is Jesus talking about in verse 5? Remember his audience. Jesus was talking about
the destruction of Jerusalem. Verse 4 gives us the clue that the people telling him these events were
from Jerusalem and were comparing the victim’s misfortunes with their own safety in Jerusalem.
Therefore, Jesus prophetically foretells a similar fate for Jerusalem.

13:7 Then said he unto the dresser of his vineyard, Behold, these three years I come seeking fruit on this
fig tree, and find none: cut it down; why cumbereth it the ground?

Who are the characters in this parable? The owner. The dresser. The vineyard and fig tree

Who was represented by the vineyard and fig tree?

Isa 5:7

Ps 1:3

Children of Israel. The generation to whom the Savior had come were represented by the fig tree in
the Lord’s vineyard – within the circle of His special care and blessing.

What had the owner done for this tree?

Isa 5:1-2

God had given the Jewish nation every opportunity to succeed and produce fruit.

What kind of fruit was the owner and dresser looking for?

Heb 13:15

John 15:8-10

Eph 5:9

Gal 5:22-23

Prov 11:30

Thankfulness, obedience, goodness and righteousness, winning souls. Fruit of individual =


righteousness = character. Fruit of the righteous = souls won.

Who is represented by the owner of the vineyard?

Matt 15:13

God the Father represents the owner.

What characteristics do we know about the dresser? What is His attitude? Who does the

http://breachrepairers.webs.com/ 485
dresser represent? He looks after the vineyard. He asks for more time to dig about the vineyard
and dung it. He agrees with the owner about the sentence if the tree still does not bear fruit. His
attitude = He still wants to save the Fig tree. What does it mean to dig and dung it? To try to revive
it with more nutrients. The dresser is Jesus. When did Jesus try to revive Israel? When he came to
Earth. He spent most of His time with the Jews. Even after his death, probation was extended until
the stoning of Steven. Then the final execution in destruction of Jerusalem in 70 AD.

What will happen to a fig tree that is cut down? What is the application for us today?

John 15:5-6

It will be cast into fire. The lake of fire at the end of time.

Who is the fig tree represented by today and how can this fig tree avoid being cut down? We
are represented by the fig tree. Spiritual Israel, God’s people. Those who profess to keep the
commandments of God and have the faith of Jesus. Jesus has extended probation time even to us.
The fact that we are still alive means that Jesus has asked God to extend our probation. Producing
fruit. Why aren’t we producing fruit then? No conversion. Need for change.

13:8 And he answering said unto him, Lord, let it alone this year also, till I shall dig about it, and dung it:

13:9 And if it bear fruit, well: and if not, then after that thou shalt cut it down.

Woman healed who had infirmity 18 years (On Sabbath) (10-17)


13:10 And he was teaching in one of the synagogues on the sabbath.

13:11 And, behold, there was a woman which had a spirit of infirmity eighteen years, and was bowed
together, and could in no wise lift up herself.

13:12 And when Jesus saw her, he called her to him, and said unto her, Woman, thou art loosed from
thine infirmity.

13:13 And he laid his hands on her: and immediately she was made straight, and glorified God.

13:14 And the ruler of the synagogue answered with indignation, because that Jesus had healed on the
sabbath day, and said unto the people, There are six days in which men ought to work: in them therefore
come and be healed, and not on the sabbath day.

13:15 The Lord then answered him, and said, Thou hypocrite, doth not each one of you on the sabbath
loose his ox or his ass from the stall, and lead him away to watering?

13:16 And ought not this woman, being a daughter of Abraham, whom Satan hath bound, lo, these
eighteen years, be loosed from this bond on the sabbath day?

13:17 And when he had said these things, all his adversaries were ashamed: and all the people rejoiced
for all the glorious things that were done by him.

Kingdom of God (Mustard Seed, Leaven, Strait gate) (18-30)


13:18 Then said he, Unto what is the kingdom of God like? and whereunto shall I resemble it?

13:19 It is like a grain of mustard seed, which a man took, and cast into his garden; and it grew, and
waxed a great tree; and the fowls of the air lodged in the branches of it.

http://breachrepairers.webs.com/ 486
13:20 And again he said, Whereunto shall I liken the kingdom of God?

13:21 It is like leaven, which a woman took and hid in three measures of meal, till the whole was
leavened.

13:22 And he went through the cities and villages, teaching, and journeying toward Jerusalem.

13:23 Then said one unto him, Lord, are there few that be saved? And he said unto them,

13:24 Strive to enter in at the strait gate: for many, I say unto you, will seek to enter in, and shall not be
able.

13:25 When once the master of the house is risen up, and hath shut to the door, and ye begin to stand
without, and to knock at the door, saying, Lord, Lord, open unto us; and he shall answer and say unto
you, I know you not whence ye are:

13:26 Then shall ye begin to say, We have eaten and drunk in thy presence, and thou hast taught in our
streets.

13:27 But he shall say, I tell you, I know you not whence ye are; depart from me, all ye workers of
iniquity.

13:28 There shall be weeping and gnashing of teeth, when ye shall see Abraham, and Isaac, and Jacob,
and all the prophets, in the kingdom of God, and you yourselves thrust out.

13:29 And they shall come from the east, and from the west, and from the north, and from the south, and
shall sit down in the kingdom of God.

13:30 And, behold, there are last which shall be first, and there are first which shall be last.

Jerusalem left desolate (31-35)


13:31 The same day there came certain of the Pharisees, saying unto him, Get thee out, and depart
hence: for Herod will kill thee.

13:32 And he said unto them, Go ye, and tell that fox, Behold, I cast out devils, and I do cures to day and
to morrow, and the third day I shall be perfected.

13:33 Nevertheless I must walk to day, and to morrow, and the day following: for it cannot be that a
prophet perish out of Jerusalem.

13:34 O Jerusalem, Jerusalem, which killest the prophets, and stonest them that are sent unto thee; how
often would I have gathered thy children together, as a hen doth gather her brood under her wings, and
ye would not!

13:35 Behold, your house is left unto you desolate: and verily I say unto you, Ye shall not see me, until the
time come when ye shall say, Blessed is he that cometh in the name of the Lord.

http://breachrepairers.webs.com/ 487
Chapter 14 – Jesus' Great Supper
Observations: This chapter shows: Where to sit at the feast, who to invite to the supper, who will
taste of the supper, and who will be the disciples from the supper.

Chapter Outline
 Jesus heals man with dropsy on the Sabbath (1-6)
 Parable of where to sit at the wedding feast (7-11)
 Who to invite to a supper (12-14)
 The great supper (15-24)
 Who is worthy to be Christ disciple (25-35)

Jesus heals man with dropsy on the Sabbath (1-6)


14:1 And it came to pass, as he went into the house of one of the chief Pharisees to eat bread on the
sabbath day, that they watched him.

14:2 And, behold, there was a certain man before him which had the dropsy.

14:3 And Jesus answering spake unto the lawyers and Pharisees, saying, Is it lawful to heal on the
sabbath day?

14:4 And they held their peace. And he took him, and healed him, and let him go;

14:5 And answered them, saying, Which of you shall have an ass or an ox fallen into a pit, and will not
straightway pull him out on the sabbath day?

14:6 And they could not answer him again to these things.

Parable of where to sit at the wedding feast (7-11)


14:7 And he put forth a parable to those which were bidden, when he marked how they chose out the
chief rooms; saying unto them,

14:8 When thou art bidden of any man to a wedding, sit not down in the highest room; lest a more
honourable man than thou be bidden of him;

14:9 And he that bade thee and him come and say to thee, Give this man place; and thou begin with
shame to take the lowest room.

14:10 But when thou art bidden, go and sit down in the lowest room; that when he that bade thee
cometh, he may say unto thee, Friend, go up higher: then shalt thou have worship in the presence of
them that sit at meat with thee.

14:11 For whosoever exalteth himself shall be abased; and he that humbleth himself shall be exalted.

Who to invite to a supper (12-14)


14:12 Then said he also to him that bade him, When thou makest a dinner or a supper, call not thy
friends, nor thy brethren, neither thy kinsmen, nor thy rich neighbours; lest they also bid thee again, and
a recompence be made thee.

http://breachrepairers.webs.com/ 488
What was the context of why Jesus was giving the parable and to whom was He speaking it
to? There were people there that thought they were going to heaven for sure. The feast was made
for only the Jews and none of the Gentiles, nor sick, nor maimed. So Jesus wanted to illustrate to
them with the feast – that just as they were leaving out people from the feast, God’s mercy was
about to run out on them as well. He was speaking this parable to the chief Pharisees (Luke 14:1)

What does the supper represent and what are the implications? Blessings offered through the
gospel. The blessings were not to be kept just between the Jewish nation but to all the world.

Who were invited to the great supper and who do they represent? 1st—the friends = Jewish
nation. 2nd—the poor, the injured (maimed), blind, which were within the city walls. 3rd—Highway
& hedges= everyone outside of Judaism,whole unbeliever nations; first to the highway = teachers
and leaders in the world (rich) and secondly the hedge = the poor class of society.

14:13 But when thou makest a feast, call the poor, the maimed, the lame, the blind:

14:14 And thou shalt be blessed; for they cannot recompense thee: for thou shalt be recompensed at the
resurrection of the just.

The great supper (15-24)


14:15 And when one of them that sat at meat with him heard these things, he said unto him, Blessed is he
that shall eat bread in the kingdom of God.

14:16 Then said he unto him, A certain man made a great supper, and bade many:

14:17 And sent his servant at supper time to say to them that were bidden, Come; for all things are now
ready.

14:18 And they all with one consent began to make excuse. The first said unto him, I have bought a piece
of ground, and I must needs go and see it: I pray thee have me excused.

14:19 And another said, I have bought five yoke of oxen, and I go to prove them: I pray thee have me
excused.

14:20 And another said, I have married a wife, and therefore I cannot come.

What were the excuses that were given? What are the common characteristics of the
excuses?

Excuses given

Luke 14:18-20

The word excuse means “to beg off, to decline, shun, avoid.” Common characteristics: None of them
were to be found of real necessity. The ground – it was already bought. His interest was all
absorbed in his purchase. The oxen – it had already been purchased as well. The proving of them
was only to satisfy the interest of the buyer, but it wouldn’t have made a difference in his purchase.
The wife – they were married already. He could have brought his wife and both would have been
welcome. But he had his own plans for enjoyment. All their minds were preoccupied with other
things, the things God gives them become the very excuse of not being able to accept the invitation

http://breachrepairers.webs.com/ 489
(gospel).

Application: Many times our excuses are so foolish in the eyes of God. And many times man can also
see through them. We put God off for a more convenient season (Acts 24:25)

Before you can accept the invitation to the wedding feast, what must you first do? You must
make all your interests subordinate to the one purpose of receiving Christ and His righteousness.

14:21 So that servant came, and shewed his lord these things. Then the master of the house being angry
said to his servant, Go out quickly into the streets and lanes of the city, and bring in hither the poor, and
the maimed, and the halt, and the blind.

14:22 And the servant said, Lord, it is done as thou hast commanded, and yet there is room.

14:23 And the lord said unto the servant, Go out into the highways and hedges, and compel them to come
in, that my house may be filled.

What did the lord mean when he told the servant to “compel them to come in”? Luke 14:23

Rev 22:17

Isa 55:1

It denotes urgency of the invitation and effectiveness in transmitting the message not necessary
force. God forces no one. He makes the invitation (Rev. 22:17, Isa. 55:1)

In the context of the study, what does it mean to hate family or friends in order to follow
Jesus? If we are not willing to sacrifice even the pleasures of the home or the company of family
that we may do God’s work, then we won’t follow Him.

What is the result of rejecting the invitation to the supper?

Luke 14:24

Rev 19:9, 17-18

They shall not taste of the supper feast. You either go to the supper or you become supper.

14:24 For I say unto you, That none of those men which were bidden shall taste of my supper.

Who is worthy to be Christ disciple (25-35)


14:25 And there went great multitudes with him: and he turned, and said unto them,

14:26 If any man come to me, and hate not his father, and mother, and wife, and children, and brethren,
and sisters, yea, and his own life also, he cannot be my disciple.

14:27 And whosoever doth not bear his cross, and come after me, cannot be my disciple.

14:28 For which of you, intending to build a tower, sitteth not down first, and counteth the cost, whether
he have sufficient to finish it?

14:29 Lest haply, after he hath laid the foundation, and is not able to finish it, all that behold it begin to

http://breachrepairers.webs.com/ 490
mock him,

14:30 Saying, This man began to build, and was not able to finish.

14:31 Or what king, going to make war against another king, sitteth not down first, and consulteth
whether he be able with ten thousand to meet him that cometh against him with twenty thousand?

14:32 Or else, while the other is yet a great way off, he sendeth an ambassage, and desireth conditions of
peace.

14:33 So likewise, whosoever he be of you that forsaketh not all that he hath, he cannot be my disciple.

14:34 Salt is good: but if the salt have lost his savour, wherewith shall it be seasoned?

14:35 It is neither fit for the land, nor yet for the dunghill; but men cast it out. He that hath ears to hear,
let him hear.

http://breachrepairers.webs.com/ 491
Chapter 15 – 3 Parables of the lost
Note: Percentage of loss increases from 1% to 10% to 50%

COL 198 3 Parables represent strays -- The parables of the lost sheep, the lost coin, and the prodigal
son, bring out in distinct lines God's pitying love for those who are straying from Him. Although
they have turned away from God, He does not leave them in their misery. He is full of kindness and
tender pity toward all who are exposed to the temptations of the artful foe.

COL 193 First 2 Parables - This parable, like the preceding, sets forth the loss of something which
with proper search may be recovered, and that with great joy. But the two parables represent
different classes. The lost sheep knows that it is lost. It has left the shepherd and the flock, and it
cannot recover itself. It represents those who realize that they are separated from God and who are
in a cloud of perplexity, in humiliation, and sorely tempted. The lost coin represents those who are
lost in trespasses and sins, but who have no sense of their condition. They are estranged from God,
but they know it not. Their souls are in peril, but they are unconscious and unconcerned. In this
parable Christ teaches that even those who are indifferent to the claims of God are the objects of His
pitying love. They are to be sought for that they may be brought back to God.

GW 170 Revivals.--When the Lord works through human instrumentalities, when men are moved
with power from on high, Satan leads his agents to cry, "Fanaticism!" and to warn people not to go to
extremes. Let all be careful how they raise this cry; for though there is counterfeit coin, this does not
lower the value of that which is genuine. Because there are spurious revivals and spurious
conversions, it does not follow that all revivals are to be held in suspicion. Let us not show the
contempt manifested by the Pharisees when they said, "This man receiveth sinners." [LUKE 15:2.]
There is enough in the life of Christ to teach us not to sneer at His work in the conversion of souls.
The manifestation of God's renewing grace on sinful men causes angels to rejoice, but often this work
has, through unbelief, been termed fanaticism, and the messenger through whom God has worked
has been spoken of as having zeal that is not according to knowledge.

Church to work for those who have left -- “ --The Lost Sheep -- The Lord is giving the erring, the weak
and trembling, and even those who have apostatized from the truth, a special call to come fully into
the fold. But many have not learned that they have a special duty to go and search for these lost
sheep. The Pharisees murmured because Jesus received publicans and common sinners, and ate with
them. In their self-righteousness they despised these poor sinners who gladly heard the words of
Jesus. To rebuke this spirit in the scribes and Pharisees, and leave an impressive lesson for all, the
Lord gave the parable of the lost sheep. Notice in particular the following points: The ninety and nine
sheep are left, and diligent search is made for the one that is lost. The entire effort is made for the
unfortunate sheep. So should the effort of the church be directed in behalf of those members who are
straying from the fold of Christ. And have they wandered far away? do not wait till they return before
you try to help them, but go in search of them.When the lost sheep was found, it was borne home
with joy, and much rejoicing followed. This illustrates the blessed, joyful work of laboring for
theerring. The church that engages successfully in this work, is a happy church.” LS 187

1SM 339 The Parable Represents -- The beautiful parable that Christ gave of the one lost sheep, of the
shepherd that left the ninety and nine to go in search of that which was lost, illustrates the work of
Christ, the sinner's condition, and the rejoicing of the universe over the salvation of the soul.

Chapter Outline
 Parable of one lost sheep (1-7)
 Parable of the one lost coin (8-10)
 Parable of the prodigal son (11-32)

http://breachrepairers.webs.com/ 492
Parable of one lost sheep (1-7)
15:1 Then drew near unto him all the publicans and sinners for to hear him.

Who is Jesus speaking this parable to?

Luke 15:1-3

Pharisees and scribes

15:2 And the Pharisees and scribes murmured, saying, This man receiveth sinners, and eateth with
them.

These verses show the social structure of Palestine. The Social Structure. Lk 15:1-2 = Four classes
and Two groups. One group: Draws near to hear Jesus. Other group: Draws near to murmur and
find fault These two verses show 4 classes of people in society and two groups. Notice in verse 1
you have the first group: Publicans and sinners. In verse 2 you have the second group: Pharisees
and scribes

App: This is how man looks upon society, this is how he views things, but this is not how God sees
society. The Bible says that 'All have sinned and come short of the glory of God'. It doesn't matter
which position you hold in society, in your neighborhood, or in the church self-supporting or the
conference, whether you are president, secretary, treasurer, you're all sinners in the eyes of God.

Luke 15:1 …drew near...to hear him…

The publicans were looked upon as betrayers because they exacted taxes. They were rejected. And
the sinners were looked upon as not being the lowest of the low. And this group drew near in order
to hear the words of Jesus. Very interesting the previous verse says 'He that hath ears to ear let him
hear' meaning those who have a desire to understand let him understand. Who are those who have
ears to hear? Who are those who want to understand? The publicans and sinners, they drew near to
hear, they had ears to hear.

App: Only when you realize that you deserve to be rejected, and you realize that you are a sinner in
need of a Savior. Then you will have ears to hear. Then and not until then will you draw near to
hear. Otherwise you will murmur.

Luke 15:2 …scribes murmured…

And this group they drew to murmur.

App: You have two groups in the church those who draw near to Christ to hear the words of Christ.
And those who draw near to murmur. They come to church, chapel, and other religious events in
order to criticize and look down upon others. They say look at her skirt, look at his shirt, you should
eat like this and not like that, etc. And they are so busy doing this that they miss the words of life.

App 2: There is a danger, the higher your status becomes in life, in the business world, or in the
church, the greater the chances are for you to no longer pay attention to the words of Jesus.

Three Parables. Lk 15:3-32. And after this picture Jesus presents three parables. The lost sheep, the
lost coin, and the lost son. These parables all emphasize one thing. The lost. And right after this

http://breachrepairers.webs.com/ 493
picture 3 parables are given. We are told by the servant of the Lord that these parables represent
God's love for those who are straying away from Him (COL 198). But my question is why are they
lost? Why are people straying into the woods, why are they hidden under the couch, why are they in
the pig pin?

Luke 14:34 …Salt is good…

What is the function of Salt? It preserves, it keeps in the same condition or state. In the 3 parables
something was lost? What does that show? The Salt was not preserving them. Who was that? The
leaders, the Pharisees and Scribes

App/Appeal: A big reason why so many people are lost in our church is because the leaders have
lost their preserving influence. They are too busy murmuring and finding fault and because of that
many are being led astray. And I pray that the leaders at Hartland have this preserving influence.
And I pray that we as God's people are not repulsing people, but we are bringing them closer to
Christ. Are you preserving others this morning? If not, let us listen to His words today because the
time is too short we need to save as many as we can before it is over.

Luke 14:34 …but if the salt have lost his savour, wherewith shall it be seasoned? It is neither fit for
the land, nor yet for the dunghill; [but] men cast it out. He that hath ears to hear, let him hear…

15:3 And he spake this parable unto them, saying,

15:4 What man of you, having an hundred sheep, if he lose one of them, doth not leave the ninety and
nine in the wilderness, and go after that which is lost, until he find it?

15:5 And when he hath found it, he layeth it on his shoulders, rejoicing.

15:6 And when he cometh home, he calleth together his friends and neighbours, saying unto them,
Rejoice with me; for I have found my sheep which was lost.

15:7 I say unto you, that likewise joy shall be in heaven over one sinner that repenteth, more than over
ninety and nine just persons, which need no repentance.

Parable of the one lost coin (8-10)


15:8 Either what woman having ten pieces of silver, if she lose one piece, doth not light a candle, and
sweep the house, and seek diligently till she find it?

15:9 And when she hath found it, she calleth her friends and her neighbours together, saying, Rejoice
with me; for I have found the piece which I had lost.

What does the silver represent? Sinners – one sinner that repenteth. Why does God represent a
sinner to a coin? Made in the image of the maker. What is the difference between the prodigal son,
sheep and coin?Coin doesn’t know it’s lost. Woman is a church. Psalms 119 equates His word with
His law. Candle represents the Word of God. Dirt represents the error of doctrines. What is the one
commandment that is lost? – The fourth. The Sabbath truth is found when the doctrines of man
are swept away.

Matt 13:53-58

Parallel chapter – John 6:42; Matt 13:55

http://breachrepairers.webs.com/ 494
John 6:25-27

Sometimes people come for ulterior motive, not to have the Word of God.

John 6:32-33

Manna represents Jesus

Num 21:5

Loatheth. It is too simple. Sometime we detest studying the Bible because it is just too simple. The
manna comes in early. And when we wake up late, it is gone. You must find God early. Wherever
you are with God in your experience, He will make sure you have enough.

15:10 Likewise, I say unto you, there is joy in the presence of the angels of God over one sinner that
repenteth.

What do the 2 parables in common? Both are lost. Both owners were searching. Both are found.
There was rejoicing when they are found

What are the differences of these 2 parables? Sheep knows when he is lost, but the silver doesn’t
know it is lost. Sheep has life, silver has no life. One is found by a shepherd, the other one is found
by a woman

What does the lost sheep and lost silver represent?

Isa 53:6

We all like sheep gone astray. Sheep: has life, but lost. Application: used to have Christ, but now lost
and doesn’t know how to come back. Also represents fallen world. The 99 sheep are those who
were never lost. Silver: has no life. Application: never used to know Christ. Doesn’t know it is lost.

What does it mean when he “layeth it on his shoulders”?

Isa 9:6

The government shall be upon his shoulders. The kingdom of God will be given to Christ. Every
sheep rescued will be a part of this new government.

What does the woman represent?

Eph 5:25

Woman represents the church.

What does the woman do after she loses the coin? Light a candle and sweep the house and seek
diligently.

What does the candle light represent?

Ps 119:105

http://breachrepairers.webs.com/ 495
Zech 4:2-6

Word of God and the Holy Spirit.

How much silver did the woman have? How many lost? What else does the silver represent?

Ps 12:6

Ps 119:172

Isa 1:10

The woman had 10 pieces of silver and lost 1. The silver = word of God = commandments. This
woman has lost the commandments of God. It is a church or people that is not keeping all the
commandments of God.

What has the woman/church lost? The woman is in darkness (spiritual darkness). This is a
church that has lost one of the commandments. She has to light a candle (Holy Spirit) and
eventually finds it and rejoices.

What can we learn from these 2 parables?

Luke 5:32

What were the Pharisees questioning Jesus about? They were questioning why Jesus was
associating with the sinners. These parables is trying to teach us Christ and Heaven’s attitude – to
seek for the sinners. The life of the church needs to be one that is continually seeking the lost. That
is the attitude of Christ.

Parable of the prodigal son (11-32)


15:11 And he said, A certain man had two sons:

Who is this story about? The father and his 2 sons. Many times we think this parable is only about
the prodigal son, and we forget the other son.

15:12 And the younger of them said to his father, Father, give me the portion of goods that falleth to me.
And he divided unto them his living.

What was the younger son asking for? What was his attitude?

Luke 15:12

His inheritance. He was asking for something he felt was rightfully his – “falleth to me.” No
gratitude. Felt that he deserved his inheritance.

15:13 And not many days after the younger son gathered all together, and took his journey into a far
country, and there wasted his substance with riotous living.

From the actions in verse 13, what can we conclude about the attitude of the younger son
toward the Father? As soon as the son could get away, he went wild. This son must have felt

http://breachrepairers.webs.com/ 496
restricted at home. He misunderstood and rejected the Father’s love and generosity for
restrictiveness. As a result, he wanted to get out and experience his own ‘freedom’. This son was
yearning for freedom.

Contextually, what does it mean to live riotously? How about our own application? Can you
share your own experiences? Share personal testimony. Riotous living is living a life without
accountability, away from the presence of the Father, with obedience to the Father. When we live a
life without God and His direction, we are wasting our substance.

15:14 And when he had spent all, there arose a mighty famine in that land; and he began to be in want.

What does the famine represent in verse 14?

Amos 8:11

Famine = lack of hearing of God’s word. Process of leaving the Father’s presence is riotous living
and eventuating in a famine of God’s word.

15:15 And he went and joined himself to a citizen of that country; and he sent him into his fields to feed
swine.

What does the younger son do as a result of the famine? What does that represent?

Luke 15:15

Joins himself to a citizen of that country. When there is a famine of the word of God in our lives, we
seek to join ourselves with the people of the world.

Rather than be joined with the world who should we be joined with?

1 Cor 1:10

1 Cor 6:17

We should be joined with Christ, having his mind.

What is the significance of the son feeding swine? Being a Jew, hearing this story, they would
have been repulsed that someone would stoop to feeding swine – unclean animals. The further we
go from God’s presence, the lower the things we will stoop to.

15:16 And he would fain have filled his belly with the husks that the swine did eat: and no man gave unto
him.

15:17 And when he came to himself, he said, How many hired servants of my father's have bread enough
and to spare, and I perish with hunger!

15:18 I will arise and go to my father, and will say unto him, Father, I have sinned against heaven, and
before thee,

15:19 And am no more worthy to be called thy son: make me as one of thy hired servants.

What realization does the son come to in verses 17-19? What spiritual state is this? The son

http://breachrepairers.webs.com/ 497
realizes that it is better to be a servant of the father. Conviction and repentance.

15:20 And he arose, and came to his father. But when he was yet a great way off, his father saw him, and
had compassion, and ran, and fell on his neck, and kissed him.

What doesn’t the son realize about the Father while he has been away?

Luke 15:20

He has been waiting every day for the son to return.

15:21 And the son said unto him, Father, I have sinned against heaven, and in thy sight, and am no more
worthy to be called thy son.

15:22 But the father said to his servants, Bring forth the best robe, and put it on him; and put a ring on
his hand, and shoes on his feet:

What does the Father give to the son? What do they represent?

Luke 15:22

Isa 61:10

Esther 8:8

Isa 8:16

Heb 10:16

Ps 119:11

Eph 6:15

Robe, ring, shoes.

“Robe” = Jesus’ robe of righteousness, covering our sins

“Ring” = seal = seal of law in disciples = seal of God’s law

“Shoes” = gospel commission

What is the reaction of the older son? What was his problem?

Luke 7:47

1 John 4:8

Anger. Lack of love, for the Father, and as result for his brother. Feels that the younger son does not
deserve this reward. Feels that he has been under appreciated for not leaving.

Contrast the younger son with the older son? What are the characteristics and weaknesses of
both? Where can we find both? Younger son = ungrateful of Father’s love and wants to leave,

http://breachrepairers.webs.com/ 498
loves the world, left home (church), went to low point and came back repentant. WEAKNESS = loves
the things of the world, desires to be part of the world, but still have the Father’s inheritance. Older
son = ungrateful of his inheritance because of lack of praise and reward, self righteous, criticizes
others. WEAKNESS = never left the church but becomes colder and more critical (loves to point out
the sins of others), until he cannot even rejoice when a brother returns. The longer you are in
church, the more danger of having a cold heart that is no longer grateful. We can find both in
church, in the world.

15:23 And bring hither the fatted calf, and kill it; and let us eat, and be merry:

15:24 For this my son was dead, and is alive again; he was lost, and is found. And they began to be merry.

15:25 Now his elder son was in the field: and as he came and drew nigh to the house, he heard musick
and dancing.

15:26 And he called one of the servants, and asked what these things meant.

15:27 And he said unto him, Thy brother is come; and thy father hath killed the fatted calf, because he
hath received him safe and sound.

15:28 And he was angry, and would not go in: therefore came his father out, and intreated him.

15:29 And he answering said to his father, Lo, these many years do I serve thee, neither transgressed I at
any time thy commandment: and yet thou never gavest me a kid, that I might make merry with my
friends:

15:30 But as soon as this thy son was come, which hath devoured thy living with harlots, thou hast killed
for him the fatted calf.

15:31 And he said unto him, Son, thou art ever with me, and all that I have is thine.

15:32 It was meet that we should make merry, and be glad: for this thy brother was dead, and is alive
again; and was lost, and is found.

http://breachrepairers.webs.com/ 499
Chapter 16
Chapter Outline
 Parable of the unjust steward (1-13)
 The law will not fail (14-18)
 Parable of the Rich man and Lazarus (19-31)

Parable of the unjust steward (1-13)

16:1 And he said also unto his disciples, There was a certain rich man, which had a steward; and the
same was accused unto him that he had wasted his goods.

Col 369

Unjust steward = publicans and Pharisees. Rich man = God (When applied to Pharisee's). Rich man
= Worldly man (When applied to publicans). Just remember that the unjust steward represents
BOTH publicans and Pharisees. 369.3 From this paragraph it is safe to say that the oil was entrusted
to the unjust steward (representing the leaders of Israel) for benevolent purposes. Or to use for
good. Then you can go to the story of the good Samaritan who used oil to bind up the wound, etc. It
also represents the Word of God. What about wheat? PRAISE GOD THE HOLY SPIRIT WAS WITH
ME! The Pharisee's were about to loose the stewardship. Here is my confirmation: 369.5 "The
stewardship was soon to be taken from them, "

Biblically speaking: Unjust steward = Publican. How? Luke 15:1 "Publicans." Naturally speaking,
who were stewards? Think of their occupation. Publicans

Unjust steward = Pharisees How?

Luke 16:15 …justify yourselves…

It means they were not just before God. So they were unjust.

Luke 16:14 …Pharisees…covetous…

They were coveting goods.

Luke 16:3, 20 …to beg I am ashamed… [20] ...begger named Lazarus…

I don't want to be like Lazarus. I don't want to be a gentile. Who are the just stewards? Luke 14:14
'resurrection of the just' (no unjust stewards will be in the first resurrection Who are they? Those
who make a feast for the poor, maimed, lame and blind. Who do you feed them? Bread

Goods: Wheat

Acts 27:38

Used for food. To feed those who are hungry. Those who are weak and starving. What do you make
with wheat? Bread (it sustains life)

http://breachrepairers.webs.com/ 500
Luke 4:4 ..man shall not live by bread alone…Word of God..

Luke 11:3 …Give us day by day our daily bread..

Luke 22:19 …bread, and gave thanks, and brake [it], and gave unto them, saying, This is my
body which is given for you…

Represents the body of Jesus Christ. They were suppose to impart a knowledge to everyone
concerning the death of Jesus Christ for the sins of the world as seen through the ceremonial law.

What was the context of why this parable was given? Who was Jesus speaking to? See chapter
before. What is in the previous chapter? Parable of the prodigal son, the lost coin and the lost
sheep. In chapter 15 Jesus established the need to reach out to sinners. Jesus was addressing the
Pharisees and scribes who said in verse 2 “This man receiveth sinners and eateth with them”

In the parable, what does the rich man demand from the steward and why was he asking for
it? What does that represent?

Luke 16:1-2

He had heard that he was wasting his goods, he was not using his money wisely, most likely
cheating his master of what he had. He asked for the accounts of what he was doing and told him he
couldn’t be his steward anymore.

“goods” = talents. Matt 25:14-15.

“given account of thy stewardship” = judgment.

What do we understand about this steward when he says “I cannot dig; to beg I am
ashamed?” I cannot dig. It’s not that he COULDN’T dig, but he didn’t want to. This showed that he
was lazy. He had been living the high life all this time and was not used to hard labor. To beg I am
ashamed. He was proud.

Realizing that he has little time left before he loses his stewardship, what does the servant
do and why? What does the servant do? Calls every one of his lord’s debtors. NOTE: “every one” –
this servant was desperate and was not about to leave any stone unturned. Application: how
desperate are we about our eternal situation? Would we make friends of everyone if faced with the
same situation as this unjust steward? Why?

Luke 16:4

He was going to try to make friends of them. He is insuring his future security.

What did the lord commend the unjust steward for? He was commending him for his
shrewdness, for doing wisely.

Luke 16:8

Notice that he was not commending him for his dishonesty

Why did the lord commend the unjust steward? Because the unjust steward was thinking about

http://breachrepairers.webs.com/ 501
his future security. He was wise enough to realize that when the lord found out he was dishonest,
he went out to make sure of his own financial security before he lost his job by giving reduction of
the debt owed so that he would have friends to take care of him when he was kicked out

What is Jesus trying to imply by giving this parable?

Luke 16:8

Wealthy greedy people are doing more and working more intensely and working more sincerely to
ensure their future financial security than the sons of light are working to ensure their future
eternal security. This also tells us that there is a work for us to do if we want to ensure our own
eternal security as well. We actual have PERSONAL responsibility and commitment in our own
destiny where we will end up. Some do the bare minimum to scrape into the kingdom of
heaven.Jesus is trying to illustrate that if we work so hard for our financial security, why do we now
work doubly hard, not just doing the bare minimum, to ensure our eternal security?

To whom was Jesus speaking this parable and why? The disciples (Luke 16:1). Because Jesus
wanted them to improve on what the Pharisees were doing. They were to redeem the time for God’s
people who had been wasting His goods. They would be the ones to bring the gospel to others and
make friends

Who was listening to this parable?

Luke 15:1, 2

Luke 16:1

Scribes and Pharisees. Disciples. Larger multitude – sinners and publicans.

Who was entrusted great stewardship responsibilities and great management duties out of
the three groups of people that were listening to Jesus? Who does it represent today? It was
the scribes and Pharisees. They had been given much light from God and much blessings. Those
that have been given much blessing from God. IE. SDA’s have the truth, people with great wealth
(eg. Job). Represents also God’s people and God’s church.

Why did Jesus speak this parable for the Pharisees? They had been given much light and God
was about to take account of their stewardship. Jesus was trying to explain to them what they could
do to be ready when God takes account. To help them realize their desperate situation and a
solution for it. They were being called to account for their stewardship.

What is the remedy in the context of the parable if we are found not being faithful stewards
of the lord’s money? How can we secure our eternal security and redeem the time we have
left?

Luke 16:9

By imparting. When you look at this parable, although the steward was asking for the debt, he was
really giving them discount. He was giving them back what they really owed. So it is if we want to
redeem the time. We need to give and impart God’s blessings to others. Make friends of the
mammon of unrighteousness – take care of them, help those of the world, the sinners. That, when

http://breachrepairers.webs.com/ 502
ye fail, they may receive you into everlasting habitations – it is not referring to the wicked receiving
us, but rather that because of the good works towards to unrighteous, our deeds towards them will
merit favor for us in the sight of God. Basically it is saying that works do matter. Read SOP quote
below for more understanding. Teachers, please use the SOP quote below just as reference. Don’t
quote it during the study. It’s just here for more background information:

COL 373 God and Christ and angels are all ministering to the afflicted, the suffering, and the sinful.
Give yourself to God for this work, use His gifts for this purpose, and you enter into partnership with
heavenly beings. Your heart will throb in sympathy with theirs. You will be assimilated to them in
character. To you these dwellers in the eternal tabernacles will not be strangers. When earthly things
shall have passed away, the watchers at heaven's gates will bid you welcome.

How then do we apply the last few verses in context of the parable?

Luke 16:10-12

If we are not faithful in what we have here on this earth and giving it back to God, He cannot commit
to us His eternal treasures. How do we be faithful in this respect? Use our possessions to benefit
others, give to the poor and sinners. Otherwise, if we were to be admitted into heaven, we would
just be selfish seeing all the glorious treasures there. Faithfulness has got much to do with
imparting blessings and using our goods to bless humanity.

16:2 And he called him, and said unto him, How is it that I hear this of thee? give an account of thy
stewardship; for thou mayest be no longer steward.

“Steward” – nothing was his own. We have been entrusted with His Word. We must also be
stewards of our time. There are 2 characteristics that typify the Jews here. What are they?

16:3 Then the steward said within himself, What shall I do? for my lord taketh away from me the
stewardship: I cannot dig; to beg I am ashamed.

“to beg I am ashamed” – They are proud.

“I cannot dig” – They are lazy.

The Jews isolated themselves with the Gentiles, because they are doing things that the Jews think is
apostasy. We have this kind of mentality sometimes, but you can punish them for what they don’t
know of. This also shows our time too. We are lazy and proud as well. God tells us to do our best to
be like the ants. Winter is coming, we must be prepared.

16:4 I am resolved what to do, that, when I am put out of the stewardship, they may receive me into their
houses.

When I lose my job, I have something to do still.

16:5 So he called every one of his lord's debtors unto him, and said unto the first, How much owest thou
unto my lord?

16:6 And he said, An hundred measures of oil. And he said unto him, Take thy bill, and sit down quickly,
and write fifty.

16:7 Then said he to another, And how much owest thou? And he said, An hundred measures of wheat.

http://breachrepairers.webs.com/ 503
And he said unto him, Take thy bill, and write fourscore.

16:8 And the lord commended the unjust steward, because he had done wisely: for the children of this
world are in their generation wiser than the children of light.

16:9 And I say unto you, Make to yourselves friends of the mammon of unrighteousness; that, when ye
fail, they may receive you into everlasting habitations.

The difference between this parable is this is what the world is doing, and this is what the Jews
were doing. This steward used the masters good to prepare for his future security. The point of this
parable is to teach us to prepare for future security. To barter what God has given us to prepare for
the kingdom of Heaven. The Gentiles were the friends of mammon to the Jews that the Jews could
reach out to in order to get to Heaven. But the Jews were failing to make use of the blessings from
God to prepare for the future on earth and also in Heaven.

16:10 He that is faithful in that which is least is faithful also in much: and he that is unjust in the least is
unjust also in much.

16:11 If therefore ye have not been faithful in the unrighteous mammon, who will commit to your trust
the true riches?

16:12 And if ye have not been faithful in that which is another man's, who shall give you that which is
your own?

16:13 No servant can serve two masters: for either he will hate the one, and love the other; or else he will
hold to the one, and despise the other. Ye cannot serve God and mammon.

The law will not fail (14-18)


16:14 And the Pharisees also, who were covetous, heard all these things: and they derided him.

16:15 And he said unto them, Ye are they which justify yourselves before men; but God knoweth your
hearts: for that which is highly esteemed among men is abomination in the sight of God.

16:16 The law and the prophets were until John: since that time the kingdom of God is preached, and
every man presseth into it.

16:17 And it is easier for heaven and earth to pass, than one tittle of the law to fail.

16:18 Whosoever putteth away his wife, and marrieth another, committeth adultery: and whosoever
marrieth her that is put away from her husband committeth adultery.

Parable of the Rich man and Lazarus (19-31)


16:19 There was a certain rich man, which was clothed in purple and fine linen, and fared sumptuously
every day:

Is this story symbolic or literal? Give Bible texts to support your answers. Symbolic.
Abraham’s bosom cannot hold all the dead. His bosom must be symbolic. How could Lazarus stretch
his finger across if heaven and hell were separated by a great gulf? If I were in hell, I wouldn’t
asking for a drop of water, I would be asking for the Niagra falls to fall on me. The phrase “There
was a certain man” in v19 shows that this is a parable and not a true story

Luke 14:16

http://breachrepairers.webs.com/ 504
Luke 15:11

Luke 16:1

Eccl 9:5-6

The dead know not anything.

16:20 And there was a certain beggar named Lazarus, which was laid at his gate, full of sores,

Who are the characters of this story? Rich man, Lazarus and Abraham

16:21 And desiring to be fed with the crumbs which fell from the rich man's table: moreover the dogs
came and licked his sores.

16:22 And it came to pass, that the beggar died, and was carried by the angels into Abraham's bosom:
the rich man also died, and was buried;

16:23 And in hell he lift up his eyes, being in torments, and seeth Abraham afar off, and Lazarus in his
bosom.

16:24 And he cried and said, Father Abraham, have mercy on me, and send Lazarus, that he may dip the
tip of his finger in water, and cool my tongue; for I am tormented in this flame.

16:25 But Abraham said, Son, remember that thou in thy lifetime receivedst thy good things, and
likewise Lazarus evil things: but now he is comforted, and thou art tormented.

What was the fate of the rich man and Lazarus? One went to hell and the other went to heaven

Who do the rich man and Lazarus represent literally and symbolically in our world today?

Jas 2:5

The rich and the poor in the world. Rich – no faith. Poor – have faith.

How can we have faith today?

Rom 10:17

We can have faith through hearing the word of God

What must go together with faith?

Jas 2:17-20

Works must go together with faith

16:26 And beside all this, between us and you there is a great gulf fixed: so that they which would pass
from hence to you cannot; neither can they pass to us, that would come from thence.

16:27 Then he said, I pray thee therefore, father, that thou wouldest send him to my father's house:

http://breachrepairers.webs.com/ 505
16:28 For I have five brethren; that he may testify unto them, lest they also come into this place of
torment.

What works was the rich man missing in this parable?

Luke 16:19-25

Matt 25:34-46

He was missing the works of helping/feeding the poor.

In the rich man’s first request, what was he asking for and why was it denied? He was asking
for comfort and possibly redemption from Lazarus who had been saved. He wanted Abraham to
send Lazarus to put out the fire.

Application: This deals with the misconception today and possibly in that day that the wicked who
died would be given a second chance after the judgment was finished and that the resurrected
righteous would be sent back to try to save the wicked again. Those in heaven will not be able to
save those in hell.

Why was his request denied? Because he was already given the benefit of a good environment as
opposed to Lazarus. He had more than enough chance. He was already dead, probation closed, and
there was a gulf between saved and condemned that could not be breached, even by God.

What was the rich man implying about God in his second request? That God was not doing
enough to save the living. That his warning was not good enough. The rich man was suggesting that
if God raised righteous dead (v27) or send them back down, their testimony would be the most
convincing. However that is not true because Jesus raised Lazarus and they still didn’t believe Him

In the response of Abraham in regard to the rich man’s brothers, what can we learn about
the rich man and why he ended up in hell? What else was he missing?

Luke 16:29-31

He lived a life of sensationalism because he thought that Lazarus coming back from the dead would
be sufficient enough to warn his brothers. He didn’t follow the writings of Moses and the prophets –
basically he didn’t follow the Bible. What was Moses and the prophets to them in their day? It was
the Bible

Why is Moses outlined here and not just mentioned with the prophets? What was so special
about mentioning Moses?

John 1:17, 45

He is mentioned always in reference to the law. It’s not just the prophets, but also the law.

What law was the rich man breaking?

Jas 2:8-10

Matt 22:35-40

http://breachrepairers.webs.com/ 506
Luke 16:25

He didn’t love his neighbor. He didn’t feed the poor (Lazarus)

What does it mean to be covetous according to this parable? To not love your neighbor. How
does faith work?

Gal 5:6

Faith works by love.

Why do you think that Abraham was brought into the picture to discuss with the rich man?
What was Jesus trying to illustrate?

Jas 2:21-24

Abraham demonstrated that he was justified through his works and that his faith was strong in the
Lord. But it was also through Abraham that we have a more clear understanding of what faith is and
when righteousness is imparted to us. Basically, Abraham is a demonstration of how faith and
works go together

In summary, what were the main problems with the rich man and how can we avoid being
like him today?

Mark 10:24

The rich man trusted too much in his riches and didn’t have faith towards God. He didn’t love his
neighbor. His works showed that there was something wrong with his faith. Miss understanding of
God. If we want to avoid being like the rich man, we must have a faith built upon the word of which
works my love. And out of that love and faith will come works naturally for our brethren. Discuss:
What things do we do that make us like the rich man?

16:29 Abraham saith unto him, They have Moses and the prophets; let them hear them.

16:30 And he said, Nay, father Abraham: but if one went unto them from the dead, they will repent.

16:31 And he said unto him, If they hear not Moses and the prophets, neither will they be persuaded,
though one rose from the dead.

http://breachrepairers.webs.com/ 507
Chapter 17 – Salvation through faith not sight | Redemption
through obedience
Chapter Outline
 Woe to whom offences come (1-4)
 Increase our faith (5-10)
 Cleansing of the 10 lepers (Faith is increased by obeying) (11-19)
 Signs of His return (20-37)

Note: DA 506 "Some of the Pharisees had come to Jesus demanding "when the kingdom of God
should come." More than three years had passed since John the Baptist gave the message that
like a trumpet call had sounded through the land, "The kingdom of heaven is at hand."

Luke 17 (20-22) chronologically takes place 3 years after Jesus' baptism.

Woe to whom offences come (1-4)


17:1 Then said he unto the disciples, It is impossible but that offences will come: but woe unto him,
through whom they come!

17:2 It were better for him that a millstone were hanged about his neck, and he cast into the sea, than
that he should offend one of these little ones.

17:3 Take heed to yourselves: If thy brother trespass against thee, rebuke him; and if he repent, forgive
him.

17:4 And if he trespass against thee seven times in a day, and seven times in a day turn again to thee,
saying, I repent; thou shalt forgive him.

Increase our faith (5-10)


17:5 And the apostles said unto the Lord, Increase our faith.

17:6 And the Lord said, If ye had faith as a grain of mustard seed, ye might say unto this sycamine tree,
Be thou plucked up by the root, and be thou planted in the sea; and it should obey you.

Why was this story given? They asked God to help them increase their faith. What does this story
teach? What is the duty that we are to do that Jesus was referring to, based on the question that
they asked? It was to exercise faith. Why say unprofitable servants? Faith doesn’t merit us anything.

17:7 But which of you, having a servant plowing or feeding cattle, will say unto him by and by, when he is
come from the field, Go and sit down to meat?

17:8 And will not rather say unto him, Make ready wherewith I may sup, and gird thyself, and serve me,
till I have eaten and drunken; and afterward thou shalt eat and drink?

17:9 Doth he thank that servant because he did the things that were commanded him? I trow not.

17:10 So likewise ye, when ye shall have done all those things which are commanded you, say, We are
unprofitable servants: we have done that which was our duty to do.

http://breachrepairers.webs.com/ 508
Cleansing of the 10 lepers (Faith is increased by obeying) (11-19)
17:11 And it came to pass, as he went to Jerusalem, that he passed through the midst of Samaria and
Galilee.

Where did Jesus pass through? What does midst mean? Jesus passed through a place in
between Samaria and Galilee. NOTE: Samaria = Samaritan, Galilee = Jews. That means this place had
a mixture of people that were probably neither here nor there. Neither Jew nor Gentile. Mixed race.
Application: What kind of people are described in the Bible as neither here nor there? Neither hot
nor cold? Laodicean church.

How did the lepers approach Jesus? What did they want?

Luke 17:2

From afar. They cried out to Jesus for mercy. They wanted healing (if you look at the rest of the
passage)

Contrast this with the approach of the leper in Mark 1:41. How does this apply to us?

Mark 1:40-41

The leper in this story comes right up to Jesus and allows Jesus to touch him. The ten lepers in Luke
17 stood afar off and Jesus was not able to touch him. Application: there are some people in church
who are content to ask for healing from Jesus afar off. What does it mean to stand afar off for us
today? Half hearted? Not willing to go all the way? Pray without necessarily meaning it?

17:12 And as he entered into a certain village, there met him ten men that were lepers, which stood afar
off:

17:13 And they lifted up their voices, and said, Jesus, Master, have mercy on us.

17:14 And when he saw them, he said unto them, Go shew yourselves unto the priests. And it came to
pass, that, as they went, they were cleansed.

What does Jesus ask them to do? Contrast with the leper in Mark 1. Jesus asks them to show
themselves to the priest. In Mark 1, Jesus touches the leper and speaks words of healing. Why does
Jesus address the two groups differently? Because of the difference of their approach. Application:
When we stand afar off to ask for healing, Jesus has to test our sincerity by asking us to do more.
When we come right up to Jesus personally and ask for healing, Jesus can touch us directly.

Where did the lepers need to go to show themselves to the priest? What did it represent?
Jerusalem. This where the temple was. The temple represented the Jewish ceremonies and
traditions. What kind of ceremonies do we have in church today? Sabbath worship, Communion
service.

What happened as they were running? The lepers were healed.

17:15 And one of them, when he saw that he was healed, turned back, and with a loud voice glorified
God,

17:16 And fell down on his face at his feet, giving him thanks: and he was a Samaritan.

http://breachrepairers.webs.com/ 509
What was different about this man? He turned back to thank Jesus. He was also a Samaritan. The
fact that the Bible mentions that he is a Samaritan most likely means that the other nine were Jews.

Why do you think the other nine continued running? What application can we draw from
this? Since the nine may have been Jews, they most likely ran all the way to Jerusalem because
acceptance back to the Jewish culture and religion was more important to them. The acceptance of
the Jewish priests was more important than their gratefulness to Christ. Application: Sometimes,
especially after we were just baptized, or even on a daily basis, we are so busy with doing things for
the Lord, getting involved in all the ceremonies or traditions, that we forget to take time to thank
the Lord. Even in our prayer life, we are so busy asking for things, that we don’t spend time
thanking Jesus for what He has done.

What lesson from the previous passage was Jesus trying to illustrate in this encounter?

Luke 17:7-10

Jesus was trying to teach that if we just simply obey what He says, we are called unprofitable
servants. The nine lepers simply obeyed what Jesus commanded. But the one did MORE than Jesus
asked. What does unprofitable mean? Not profitable. Not beneficial to the master.

What duty makes us only an unprofitable servant? Discuss.

Eccl 12:13

Keeping the ten commandments. Eg. Coming to church every Sabbath only makes us an
unprofitable servant.

What did the one leper do that made him a profitable servant?

Rom 12:1-2

He was grateful enough to come back and thank Christ. By doing that He gave glory to God. What is
a profitable servant? What does God consider a reasonable service? One that does more than God
asks out of gratefulness of the heart. What is more than what is contained in the law? God asks us to
be a living sacrifice. To give Him our hearts, our hobbies, our music, our diet. All of these things are
not contained in the law, but if we want to give glory to God, to be profitable to Him, we should love
Him enough, be thankful enough to give Him all of our lives.

What was the result of the one leper’s actions?

Luke 17:19

1 Thess 5:23

He was made whole. What does it mean to be made whole? To be sanctified, our whole spirit, body,
mind be preserved blameless till Jesus comes. NOTE: Its possible to be cleansed by the blood of
Christ (justified), but never made whole (sanctified – involves changing our old life, our habits,
becoming a living sacrifice).

17:17 And Jesus answering said, Were there not ten cleansed? but where are the nine?

http://breachrepairers.webs.com/ 510
17:18 There are not found that returned to give glory to God, save this stranger.

17:19 And he said unto him, Arise, go thy way: thy faith hath made thee whole.

Signs of His return (20-37)


17:20 And when he was demanded of the Pharisees, when the kingdom of God should come, he answered
them and said, The kingdom of God cometh not with observation:

They are still asking about the kingdom of God that John the Baptist was talking about.

DA 509 The kingdom of God comes not with outward show. The gospel of the grace of God, with its
spirit of self-abnegation, can never be in harmony with the spirit of the world. The two principles are
antagonistic. "The natural man receiveth not the things of the Spirit of God: for they are foolishness
unto him: neither can he know them, because they are spiritually discerned." 1 Cor. 2:14.

What kind of a kingdom were the Pharisees looking for? An earthly kingdom.

DA 509 Yet the Saviour attempted no civil reforms. He attacked no national abuses, nor condemned
the national enemies. He did not interfere with the authority or administration of those in power. He
who was our example kept aloof from earthly governments. Not because He was indifferent to the
woes of men, but because the remedy did not lie in merely human and external measures. To be
efficient, the cure must reach men individually, and must regenerate the heart.

Should we vote? In a way we are responsible for what the government does. Sister White says we
should not vote.

17:21 Neither shall they say, Lo here! or, lo there! for, behold, the kingdom of God is within you.

17:22 And he said unto the disciples, The days will come, when ye shall desire to see one of the days of
the Son of man, and ye shall not see it.

17:23 And they shall say to you, See here; or, see there: go not after them, nor follow them.

17:24 For as the lightning, that lighteneth out of the one part under heaven, shineth unto the other part
under heaven; so shall also the Son of man be in his day.

17:25 But first must he suffer many things, and be rejected of this generation.

17:26 And as it was in the days of Noe, so shall it be also in the days of the Son of man.

17:27 They did eat, they drank, they married wives, they were given in marriage, until the day that Noe
entered into the ark, and the flood came, and destroyed them all.

17:28 Likewise also as it was in the days of Lot; they did eat, they drank, they bought, they sold, they
planted, they builded;

17:29 But the same day that Lot went out of Sodom it rained fire and brimstone from heaven, and
destroyed them all.

17:30 Even thus shall it be in the day when the Son of man is revealed.

17:31 In that day, he which shall be upon the housetop, and his stuff in the house, let him not come down

http://breachrepairers.webs.com/ 511
to take it away: and he that is in the field, let him likewise not return back.

17:32 Remember Lot's wife.

17:33 Whosoever shall seek to save his life shall lose it; and whosoever shall lose his life shall preserve it.

17:34 I tell you, in that night there shall be two men in one bed; the one shall be taken, and the other
shall be left.

17:35 Two women shall be grinding together; the one shall be taken, and the other left.

17:36 Two men shall be in the field; the one shall be taken, and the other left.

17:37 And they answered and said unto him, Where, Lord? And he said unto them, Wheresoever the body
is, thither will the eagles be gathered together.

http://breachrepairers.webs.com/ 512
Chapter 18 – Prayer & Faith?
Chapter Outline
 Pray continually (regarding your adversaries) (1-8)
 Prayer of justification (Pharisee and the Publican, be humble) (9-17)
 How to inherit eternal life (18-30)
 Blind man asks for mercy (31-43)

Note: V.7 reminds me of the church of Philadelphia & the souls under the altar. It paints the picture
of God as judge. Note: Jesus is now closer to Jerusalem. He is passing through Jericho.

Pray continually (regarding your adversaries) (1-8)


18:1 And he spake a parable unto them to this end, that men ought always to pray, and not to faint;

God always want us to pray always. By praying we show that we have faith. (Heb 11:6). What do we
expect to learn from this parable?

Luke 18:1

Prayer, praying always without fainting or becoming weary. Why is this parable relevant to us?

Luke 17:26, 27

Because it under the context of the last days, this is how we are to pray in the last days

18:2 Saying, There was in a city a judge, which feared not God, neither regarded man:

That judge does and does not represent God. The woman who goes to him is a symbol of us who go
to God. But this judge does not fear God.

18:3 And there was a widow in that city; and she came unto him, saying, Avenge me of mine adversary.

18:4 And he would not for a while: but afterward he said within himself, Though I fear not God, nor
regard man;

18:5 Yet because this widow troubleth me, I will avenge her, lest by her continual coming she weary me.

18:6 And the Lord said, Hear what the unjust judge saith.

18:7 And shall not God avenge his own elect, which cry day and night unto him, though he bear long with
them?

What are the characteristics of the unjust judge and the character of God?

Luke 18:2-7

Rom 3:26

1 John 4:8

http://breachrepairers.webs.com/ 513
Isa 1:18

Unjust judge: No regard to right, nor pity for suffering. Showed indifference and hardheartedness.
Hears her plea because he didn’t want to hear her plea anymore and didn’t want to be troubled by
the women God the righteous judge:

Rom 3:26

Just and the justifier of those that believe in Him

1 John 4:8

God is love. Compassion.

Isa 1:18

He wants to reason with us and makes us white as snow. Worketh speedily. What does a woman
represent in the Bible?

Jer 6:2

Isa 51:16

Eph 5:25

God’s people, His church. This woman in the parable represents God’s people pleading for
deliverance

The woman asks for deliverance from the adversary, who does the adversary represent and
what does he do?

1 Pet 5:8

Rev 12:10

“Adversary” – Satan. Devour. Accuse the brethren day and night. What accusations does Satan
accuse God’s people?

Job 1:7-12

God blesses all those that follow Him, people only follow Christ because they are blessed by God

Zech 3:1-4

He is guilty, a sinner. Filthy garments = character/righteousness stained by sin (Isaiah 64:6)

Jude 9

Cannot have life. Moses was resurrected by faith before Christ had died (Matt. 17:1-4)

http://breachrepairers.webs.com/ 514
When will God avenge the elect? Contextually

Luke 18:8

When the Son of man comes. What other words does the Bible use to describe “avenge?”

Deut 32:43

Jer 46:10

Rom 12:19

Nah 1:2

Vengeance and also wrath

When is the day of God’s vengeance/wrath?

Rev 15:1

Rev 6:16-17

When the seven last plagues are poured out. It is just prior to the second coming.

What must take place before God can avenge His people?

Rev 6:10

Judgment must take place

Why is God bearing long with them (elect)?

Luke 18:7

Jas 5:7

Because they are persistent in praying and studying the word, even though He seemingly delays.
God waits patiently for the precious fruit of the earth

How do the elect cry day and night?

Luke 18:1, 7

Psa 1:2

Pray. Mediating upon the law

What are the characteristics of the elect?

Luke 18:7

Rom 12:19-20

http://breachrepairers.webs.com/ 515
Matt 25:31-40

Pray day and night. They are persistent. They have faith. They do not avenge but rather are nice and
loving back to their enemies.

18:8 I tell you that he will avenge them speedily. Nevertheless when the Son of man cometh, shall he find
faith on the earth?

Prayer of justification (Pharisee and the Publican, be humble) (9-17)


18:9 And he spake this parable unto certain which trusted in themselves that they were righteous, and
despised others:

Take note, who is Jesus speaking this parable to?

Luke 18:9

He is speaking to those that trust in themselves that they are righteous, and despise others. Namely,
Jesus is speaking to the Pharisees – we know because of the parable itself.

18:10 Two men went up into the temple to pray; the one a Pharisee, and the other a publican.

Who are the two groups of people represented in this parable?

Luke 18:10

Pharisee and Publican

18:11 The Pharisee stood and prayed thus with himself, God, I thank thee, that I am not as other men
are, extortioners, unjust, adulterers, or even as this publican.

18:12 I fast twice in the week, I give tithes of all that I possess.

18:13 And the publican, standing afar off, would not lift up so much as his eyes unto heaven, but smote
upon his breast, saying, God be merciful to me a sinner.

Postures in prayer – both were standing. EGW says that it is a privilege to kneel. Acts 2 – they were
sitting when they received the Holy Spirit.

Characteristics of a Pharisee: It is comparative righteousness. It is how much light you have


received and how much you are living up to which you know. Everyone is at different levels with
God. For whom much have been given, much more is been requires. Works are important, but
nothing that we do can ever help us to be right in the sight of God. It is His blood and Jesus that
makes us worthy.

We must know that we are ungodly. God justifies the ungodly, the sinners.

Psa 103:12

God show us mercy by remove our sins. Justify = forgiven; this man went a way justified.

http://breachrepairers.webs.com/ 516
18:14 I tell you, this man went down to his house justified rather than the other: for every one that
exalteth himself shall be abased; and he that humbleth himself shall be exalted.

Compare these two groups of people and what is the difference between the two of them?
Please discuss.

Luke 18:11-14

Pharisee:
o Self-righteous
o Satisfied with his own character
o Judges others by his own righteousness
o Feels no conviction that he is a sinner
o Proud, self-sufficient, self-confident
o Exalts himself
Publican:
o Realizes that he is a sinner in need of a Saviour – realizes his need
o Distrusts himself and relies fully on Christ’s merits
o Has understanding of self and doesn’t compare himself to others – if we want to, we will
always find a
person in this world that is worse that ourselves. We must compare ourselves to Jesus
o Humbles himself

Sometimes we can be in church worshipping the same God, but yet not feel like we really need Him.

Who was it that went to his house justified?

Luke 18:14

The publican

What does it mean to be justified?

Job 25:4

To be clean

Isa 53:11

For our iniquities to be borne away from us

How can a person be justified or cleansed?

1 John 1:9

We must confess, we must realize that we have sinned against God. We must acknowledge our
iniquity.

Psa 51:4

Jer 3:13

http://breachrepairers.webs.com/ 517
In order for us to confess our sins we must first realize that we are sinners and must have a true
knowledge of ourselves – we must have a true realization of self. There are many out there that are
self-deceived. They think that they are so good.

Ezek 36:25-26

He will sprinkle clean water upon us that we may be clean – this is referring to a new heart that He
will give us in the next verse. What does the Pharisee and the church of Laodicea in Revelation
3 have in common?

Rev 3:17

“knowest not” – they do not have a true picture of themselves. They are self deceived. If they ask
who is the Laodicean church, just mention it is God’s last day church and we can study into that in
the future.

What is Christ’s remedy for people who have no knowledge of who they really are?

Rev 3:18

Gold tried in the fire. White raiment. Anoint eyes with eye salve. What does gold represent?

1 Pet 1:7

It represents the trials of our faith. Note, it does not represent simply faith. So if we want a true
knowledge of ourselves, we must go through trials. It is in trials that we remain humble. Trails are
not just for wicked people. God uses trials to bring us closer to Him. Expound on this faith, its not
just one that is being tried but its a faith that works by love (Gal. 5:6). Because we can have all the
external products of faith but if it is not motivated by love its not what God is wanting. And that is
one thing that the Pharisee didn't have, love. They had the external fruits but a love that motivated
their faith they didn’t possess.

What does white raiment represent?

Rev 19:8

Represents righteousness of Christ

1 John 2:1

It represents Jesus Christ who is the righteous One.

When we put on that white raiment, we put on Jesus. People now see Jesus, not us. God now sees
righteous Jesus, not sinful me. The more we compare ourselves to Jesus, the more humble we will
be become because we will realize how perfect Jesus is and how sinful we are. We need this so we
will always have a right estimate of ourselves and not become critical and finger-pointing of others.
Those that become judgmental and harsh are those that are furthest from Christ because they
stopped comparing themselves to Him.

http://breachrepairers.webs.com/ 518
What does eyesalve represent?

1 Sam 9:9

Seer is called a prophet to help them see. Laodicean church needed eyesalve that they may see. We
need prophet to help us realize our need of self. Prophets wrote the Bible and it is the Bible that
tells us about Jesus (John 5:39) which is our garment of righteousness.

Please summarize what are the three things that we need to keep us humble like the
publican? Trials, Comparing ourselves to Jesus, The Bible.

What was Christ’s mission and counsel that we may remain humble and dependent on Christ
today?

Luke 5:31

They that are whole need not a physician

Matt 23:12

And whosoever shall exalt himself shall be abased; and he that shall humble himself shall be
exalted.

Conclusion: Let us learn to pray this prayer today no matter how far we have come in our Christian
walk, that we may depend on Christ everyday for strength:

“Lord, take my heart; for I cannot give it. It is Thy property. Keep it pure, for I cannot keep it for Thee.
Save me in spite of myself, my weak, unchristlike self. Mold me, fashion me, raise me into a pure and
holy atmosphere, where the rich current of Thy love can flow through my soul.”

18:15 And they brought unto him also infants, that he would touch them: but when his disciples saw it,
they rebuked them.

18:16 But Jesus called them unto him, and said, Suffer little children to come unto me, and forbid them
not: for of such is the kingdom of God.

18:17 Verily I say unto you, Whosoever shall not receive the kingdom of God as a little child shall in no
wise enter therein.

How to inherit eternal life (18-30)


18:18 And a certain ruler asked him, saying, Good Master, what shall I do to inherit eternal life?

18:19 And Jesus said unto him, Why callest thou me good? none is good, save one, that is, God.

18:20 Thou knowest the commandments, Do not commit adultery, Do not kill, Do not steal, Do not bear
false witness, Honour thy father and thy mother.

18:21 And he said, All these have I kept from my youth up.

18:22 Now when Jesus heard these things, he said unto him, Yet lackest thou one thing: sell all that thou
hast, and distribute unto the poor, and thou shalt have treasure in heaven: and come, follow me.

http://breachrepairers.webs.com/ 519
18:23 And when he heard this, he was very sorrowful: for he was very rich.

18:24 And when Jesus saw that he was very sorrowful, he said, How hardly shall they that have riches
enter into the kingdom of God!

18:25 For it is easier for a camel to go through a needle's eye, than for a rich man to enter into the
kingdom of God.

18:26 And they that heard it said, Who then can be saved?

18:27 And he said, The things which are impossible with men are possible with God.

18:28 Then Peter said, Lo, we have left all, and followed thee.

18:29 And he said unto them, Verily I say unto you, There is no man that hath left house, or parents, or
brethren, or wife, or children, for the kingdom of God's sake,

18:30 Who shall not receive manifold more in this present time, and in the world to come life everlasting.

18:31 Then he took unto him the twelve, and said unto them, Behold, we go up to Jerusalem, and all
things that are written by the prophets concerning the Son of man shall be accomplished.

18:32 For he shall be delivered unto the Gentiles, and shall be mocked, and spitefully entreated, and
spitted on:

18:33 And they shall scourge him, and put him to death: and the third day he shall rise again.

18:34 And they understood none of these things: and this saying was hid from them, neither knew they
the things which were spoken.

Blind man asks for mercy (35-43)


18:35 And it came to pass, that as he was come nigh unto Jericho, a certain blind man sat by the way side
begging:

What do we know about Bartimaeus before he met Jesus? He was blind. Sitting by way side.
Begging.

18:36 And hearing the multitude pass by, he asked what it meant.

How did Bartimaeus hear about Jesus?

Mark 10:46

The multitude, while they were passing by. Who did the multitude consist of? The disciples and the
crowd.

18:37 And they told him, that Jesus of Nazareth passeth by.

What did the crowd think of Jesus from their answer? Who was he to them? Just Jesus of
Nazareth, a man. Nothing special. Perhaps they may have recognized him at most as a prophet.

18:38 And he cried, saying, Jesus, thou Son of David, have mercy on me.

http://breachrepairers.webs.com/ 520
How does the blind man address Jesus? What did it show?

Matthew 22:42

Bartimaeus refers to Jesus as the Son of David. He recognizes Christ’s ancestry, his royalty and
messiahship. Such respect.

Why does Bartimaeus ask Jesus to have mercy on him? He recognizes his sinfulness. He desires
healing physically and spiritually. He knows that if God wills, He can heal him.

18:39 And they which went before rebuked him, that he should hold his peace: but he cried so much the
more, Thou Son of David, have mercy on me.

How does the crowd react? They try to rebuke him to hold his peace. They were embarrassed of
him.

What is Bartimaeus response and what can we learn from it? Despite the crowd’s desire for
him to keep quiet, Bartimaeus cried out even louder. He would not let the crowd stop him from
getting Jesus’ attention. Application: Important to be persistent when we want Christ. It won’t be an
easy road to have physical or spiritual healing, but we have to keep appealing to Christ and not give
up, even though people may be embarrassed of us or write us off. Likewise we should never give up
on anyone.

18:40 And Jesus stood, and commanded him to be brought unto him: and when he was come near, he
asked him,

How did Jesus respond? He commanded Bartimaeus to be brought to Him. Application: What is
Jesus desire for everyone who wants Him, no matter how embarrassing they may be? To be brought
to Him.

What does the blind man do before he goes to see Jesus? What does this symbolize?

Mark 10:50

Casts away his garment. Application: What does this mean? When we want to see Jesus and ask for
healing, we must be prepared to cast away our old garment. Our old habits and selfish desires. You
can’t serve both masters.

18:41 Saying, What wilt thou that I shall do unto thee? And he said, Lord, that I may receive my sight.

What does Jesus ask them? Why? What does Jesus ask them? What wilt thou that I shall do unto
thee? Application: Why? He wants to check that they truly want healing. Sometimes when we come
to Christ asking for healing, He will check our sincerity. He wants to know if we truly desire it. He
also wants to hear us profess it. To tell publicly. What is a public way that we can profess to want
healing from Jesus? Baptism and testimony. We are baptized because we want to show publicly that
we want healing from Christ.

What is the blind man’s reply? What does this symbolize? Lord that I may receive sight, that
their eyes may be opened. Application: What does it mean to have our eyes opened?

Matt 20:33

http://breachrepairers.webs.com/ 521
Psa 119:18

Behold wondrous things out of the law.

Is 42:7

Release prisoners from darkness.

Acts 26:18

Receive forgiveness of sins and inheritance among them which are sanctified by faith.

18:42 And Jesus said unto him, Receive thy sight: thy faith hath saved thee.

What does Jesus do? And what happened as a result?

Matthew 20:34

1. Jesus had compassion and touched their eyes.


2. Jesus spoke – receive thy sight, they faith has healed thee.
3. Bartimaeus immediately received his sight.
4. Bartimaeus followed Christ, glorifying Him.
5. All the people also gave praise to God.

Compare and contrast the different groups in this encounter.


1. Blind Man:
o Physically blind, but saw Jesus spiritually.
o Just met Jesus.
o Had bigger need for Christ – were willing to embarrass themselves by crying out.
2. Multitude – disciples and crowd:
o Had physical sight, but spiritually blind to who Jesus was.
o Knew Jesus for longer.
o Needed Christ less.

CONCLUSION: Sometimes those who first hear of Christ have a greater sense of how great He is
than those who have been with him a long time.

What lesson do we learn about faith from this encounter? Which parable in Luke 18 is Jesus
trying to illustrate through this encounter?

Luke 18:1-8

Faith is persistent. This encounter illustrates Jesus parable about the unjust judge at the beginning
of Luke 18 – how to ask with a persistent faith until your prayer is answered. Where else in the
Bible does it speak about praying with faith?

http://breachrepairers.webs.com/ 522
Jas 1:6

Do not waver.

18:43 And immediately he received his sight, and followed him, glorifying God: and all the people, when
they saw it, gave praise unto God.

http://breachrepairers.webs.com/ 523
Chapter 19 – Conditions of being a part of God’s house
Chapter Outline
 Story of Zacchaeus (a child of Abraham) (1-10)
 Parable of the Nobleman receiving a kingdom (occupy till I come) (11-27)
 Jesus' entry into Jerusalem (28-44)
 Jesus cleanses the temple the second time (45-48)
Climatic verse: V.10 "For the Son of man is come to seek and to save that which was lost"
Jesus finally arrives at Jerusalem. It took 10 chapters

The story from verse 1-9 is about salvation. It talks about the principles of Christian enterprise.

Story of Zacchaeus (a child of Abraham) (1-10)


19:1 And Jesus entered and passed through Jericho.

“Jericho” – it is a beautiful place.

19:2 And, behold, there was a man named Zacchaeus, which was the chief among the publicans, and he
was rich.

What do we know about Zaccheus? Chief among publicans. He was rich. Man of little stature

What is a publican? Simply a tax collector. Most seemed to be dishonest.

What were publicans known for?

Matt 5:46-47

Matt 9:10-11

Matt 10:3

Matt 18:17

Matt 21:31-32

Despised just like sinners. Matthew was a publican. See what else you can find from the Bible texts.
Note: Zaccheus was the CHIEF among publicans. Even worse. Publicans: They were Jews, It’s like a
Jew working for the Germans. They associated publicans with harlots. Zacchaeus was hated. He was
dishonest. He was the head tax collector.

19:3 And he sought to see Jesus who he was; and could not for the press, because he was little of stature.

What was Zaccheus trying to do? He sought to see Jesus who he was.

Why wasn’t Zaccheus able to see Jesus? He was of little stature – too short. There were people
there that were taller than him, blocking his view.

http://breachrepairers.webs.com/ 524
Who could Zaccheus represent? Those who are spiritually dwarfed. Those who are being blocked
by others from seeing Jesus.

Is 59:2

Our iniquities have separated us from God. Zacchaeus represents a person lost in sin.
Who could the tall people represent? Contrast it with who Zaccheus represents. People who
block spiritually dwarfed people from seeing Jesus. Sometimes you can act so spiritually tall that
you block the others view of Jesus. Eg. Publican and Pharisee (Luke 18:10-14). Note: These people
are able to see Jesus. And rather than helping others to see Him too, they block them, probably
because of their past or current occupation or stigma (Zaccheus was a hated tax collector)

19:4 And he ran before, and climbed up into a sycomore tree to see him: for he was to pass that way.

Sycamore tree: The fruit of the sycamore tree was like figs. What role did the tree play? It put him
up where he could see Jesus. Jesus was lifted up by a tree to restore connection between mad and
God.

How was Zaccheus able to see Jesus? Climbed a tree.

What does a tree represent? Contrast this with those blocking Zaccheus view.

Psa 1:2-3, 6

Tree = righteous man. Contrast: The tree was able to help Zaccheus see Jesus. A righteous man is
one who is able to help others see Jesus. The tall people saw Jesus, were in contact with Him, but
blocked Zaccheus from seeing Jesus. Where do you think you would find tall people? In church.
They see Jesus, but block others from Him.

Note: Zaccheus had to use his own choice and effort to climb the tree. The righteous man can only
do so much to help those who want to see Jesus. There must be a cooperation and effort on the part
of the seeker.

19:5 And when Jesus came to the place, he looked up, and saw him, and said unto him, Zacchaeus, make
haste, and come down; for to day I must abide at thy house.

Jesus specifically told him that He was going to stay at his house. Jesus knew about Zacchaeus. He
told him He was going to spend time with him. Zacchaeus had heard that one of his most trusted
disciples was a publican. It gave him hope.

Rom 2:4

The goodness of God leads us to repentance.

19:6 And he made haste, and came down, and received him joyfully.

Zacchaeus acted on His appeal.

19:7 And when they saw it, they all murmured, saying, That he was gone to be guest with a man that is a
sinner.

Who are they that murmured? Most likely those who were blocking Zaccheus view. The

http://breachrepairers.webs.com/ 525
spiritually tall people. Interestingly, they saw only Zaccheus as a sinner, not themselves.

One of our greatest problems that we have today in bringing people to Jesus is the church. It is
because the people are not ready. We can stand in the same way of someone of is trying to make an
effort to know Jesus.

19:8 And Zacchaeus stood, and said unto the Lord; Behold, Lord, the half of my goods I give to the poor;
and if I have taken any thing from any man by false accusation, I restore him fourfold.

He was giving away his life of sin.

What was the turning point that caused Zaccheus to give up all his goods? What can we learn
from this? When Jesus spoke to Zaccheus and told him He was coming over to his house. What can
we learn from this? Many people are keen to see Jesus, but people are blocking them with their self
righteousness and making religion so tall that it blocks the true object. However, we can still reach
these people just through coming to their house for visit, eating with them, then opening Christ to
them. There is a group of people that need to be reached through personal touch and home visits.
Fellowship in the home and dinner has a part to play in soul winning.

What can we learn about restitution from the story of Zaccheus? Give to the poor. Return 4
times as much.

Compare Zaccheus story with the rich young ruler? The disciples were astonished at Jesus
comment about the rich man but Zaccheus proves that a rich man can go to heaven. But what did it
take? Do you think Zaccheus was still rich after he gave half his possessions to poor and 4 times to
those whom he had cheated? No.

What does a rich man have to do to enter heaven? Sacrifice all and follow Jesus.

Matt 13:45-46

Interesting that Zaccheus did exactly what Jesus had asked the rich man to do, except he didn’t need
to be asked by Jesus. He was convicted of it. It was probably the same for the rich man. By the time
Jesus asked, he had already rejected the conviction of the Holy Spirit.

19:9 And Jesus said unto him, This day is salvation come to this house, forsomuch as he also is a son of
Abraham.

Jesus only said that salvation had come to his house after he had repented.

19:10 For the Son of man is come to seek and to save that which was lost.

Parable of the Nobleman receiving a kingdom (occupy till I come) (11-


27)
19:11 And as they heard these things, he added and spake a parable, because he was nigh to Jerusalem,
and because they thought that the kingdom of God should immediately appear.

19:12 He said therefore, A certain nobleman went into a far country to receive for himself a kingdom,
and to return.

19:13 And he called his ten servants, and delivered them ten pounds, and said unto them, Occupy till I

http://breachrepairers.webs.com/ 526
come.

19:14 But his citizens hated him, and sent a message after him, saying, We will not have this man to
reign over us.

19:15 And it came to pass, that when he was returned, having received the kingdom, then he
commanded these servants to be called unto him, to whom he had given the money, that he might know
how much every man had gained by trading.

19:16 Then came the first, saying, Lord, thy pound hath gained ten pounds.

19:17 And he said unto him, Well, thou good servant: because thou hast been faithful in a very little, have
thou authority over ten cities.

19:18 And the second came, saying, Lord, thy pound hath gained five pounds.

19:19 And he said likewise to him, Be thou also over five cities.

19:20 And another came, saying, Lord, behold, here is thy pound, which I have kept laid up in a napkin:

19:21 For I feared thee, because thou art an austere man: thou takest up that thou layedst not down, and
reapest that thou didst not sow.

19:22 And he saith unto him, Out of thine own mouth will I judge thee, thou wicked servant. Thou
knewest that I was an austere man, taking up that I laid not down, and reaping that I did not sow:

19:23 Wherefore then gavest not thou my money into the bank, that at my coming I might have required
mine own with usury?

19:24 And he said unto them that stood by, Take from him the pound, and give it to him that hath ten
pounds.

19:25 (And they said unto him, Lord, he hath ten pounds.)

19:26 For I say unto you, That unto every one which hath shall be given; and from him that hath not,
even that he hath shall be taken away from him.

19:27 But those mine enemies, which would not that I should reign over them, bring hither, and slay
them before me.

Jesus' entry into Jerusalem (28-44)


19:28 And when he had thus spoken, he went before, ascending up to Jerusalem.

19:29 And it came to pass, when he was come nigh to Bethphage and Bethany, at the mount called the
mount of Olives, he sent two of his disciples,

19:30 Saying, Go ye into the village over against you; in the which at your entering ye shall find a colt
tied, whereon yet never man sat: loose him, and bring him hither.

19:31 And if any man ask you, Why do ye loose him? thus shall ye say unto him, Because the Lord hath
need of him.

19:32 And they that were sent went their way, and found even as he had said unto them.

http://breachrepairers.webs.com/ 527
19:33 And as they were loosing the colt, the owners thereof said unto them, Why loose ye the colt?

19:34 And they said, The Lord hath need of him.

19:35 And they brought him to Jesus: and they cast their garments upon the colt, and they set Jesus
thereon.

19:36 And as he went, they spread their clothes in the way.

19:37 And when he was come nigh, even now at the descent of the mount of Olives, the whole multitude
of the disciples began to rejoice and praise God with a loud voice for all the mighty works that they had
seen;

19:38 Saying, Blessed be the King that cometh in the name of the Lord: peace in heaven, and glory in the
highest.

19:39 And some of the Pharisees from among the multitude said unto him, Master, rebuke thy disciples.

19:40 And he answered and said unto them, I tell you that, if these should hold their peace, the stones
would immediately cry out.

19:41 And when he was come near, he beheld the city, and wept over it,

19:42 Saying, If thou hadst known, even thou, at least in this thy day, the things which belong unto thy
peace! but now they are hid from thine eyes.

19:43 For the days shall come upon thee, that thine enemies shall cast a trench about thee, and compass
thee round, and keep thee in on every side,

19:44 And shall lay thee even with the ground, and thy children within thee; and they shall not leave in
thee one stone upon another; because thou knewest not the time of thy visitation.

Jesus cleanses the temple the second time (45-48)


19:45 And he went into the temple, and began to cast out them that sold therein, and them that bought;

19:46 Saying unto them, It is written, My house is the house of prayer: but ye have made it a den of
thieves.

19:47 And he taught daily in the temple. But the chief priests and the scribes and the chief of the people
sought to destroy him,

19:48 And could not find what they might do: for all the people were very attentive to hear him.

http://breachrepairers.webs.com/ 528
Chapter 20
Chapter Outline
 Pharisees ask what authority (parable of vineyard given to others) (1-19)
 Spies question Tribute to Caesar (19-26)
 Sadducees question resurrection and marriage (27-39)
 How is Christ David's Son, and beware of scribes (40-47)

Pharisees ask what authority (parable of vineyard given to others) (1-


19)
20:1 And it came to pass, that on one of those days, as he taught the people in the temple, and preached
the gospel, the chief priests and the scribes came upon him with the elders,

20:2 And spake unto him, saying, Tell us, by what authority doest thou these things? or who is he that
gave thee this authority?

20:3 And he answered and said unto them, I will also ask you one thing; and answer me:

20:4 The baptism of John, was it from heaven, or of men?

20:5 And they reasoned with themselves, saying, If we shall say, From heaven; he will say, Why then
believed ye him not?

20:6 But and if we say, Of men; all the people will stone us: for they be persuaded that John was a
prophet.

20:7 And they answered, that they could not tell whence it was.

20:8 And Jesus said unto them, Neither tell I you by what authority I do these things.

20:9 Then began he to speak to the people this parable; A certain man planted a vineyard, and let it forth
to husbandmen, and went into a far country for a long time.

20:10 And at the season he sent a servant to the husbandmen, that they should give him of the fruit of
the vineyard: but the husbandmen beat him, and sent him away empty.

20:11 And again he sent another servant: and they beat him also, and entreated him shamefully, and
sent him away empty.

20:12 And again he sent a third: and they wounded him also, and cast him out.

20:13 Then said the lord of the vineyard, What shall I do? I will send my beloved son: it may be they will
reverence him when they see him.

20:14 But when the husbandmen saw him, they reasoned among themselves, saying, This is the heir:
come, let us kill him, that the inheritance may be ours.

20:15 So they cast him out of the vineyard, and killed him. What therefore shall the lord of the vineyard
do unto them?

http://breachrepairers.webs.com/ 529
20:16 He shall come and destroy these husbandmen, and shall give the vineyard to others. And when
they heard it, they said, God forbid.

20:17 And he beheld them, and said, What is this then that is written, The stone which the builders
rejected, the same is become the head of the corner?

20:18 Whosoever shall fall upon that stone shall be broken; but on whomsoever it shall fall, it will grind
him to powder.

Spies question Tribute to Caesar (19-26)


20:19 And the chief priests and the scribes the same hour sought to lay hands on him; and they feared
the people: for they perceived that he had spoken this parable against them.

20:20 And they watched him, and sent forth spies, which should feign themselves just men, that they
might take hold of his words, that so they might deliver him unto the power and authority of the
governor.

20:21 And they asked him, saying, Master, we know that thou sayest and teachest rightly, neither
acceptest thou the person of any, but teachest the way of God truly:

20:22 Is it lawful for us to give tribute unto Caesar, or no?

20:23 But he perceived their craftiness, and said unto them, Why tempt ye me?

20:24 Shew me a penny. Whose image and superscription hath it? They answered and said, Caesar's.

20:25 And he said unto them, Render therefore unto Caesar the things which be Caesar's, and unto God
the things which be God's.

20:26 And they could not take hold of his words before the people: and they marvelled at his answer, and
held their peace.

Sadducees question resurrection and marriage (27-39)


20:27 Then came to him certain of the Sadducees, which deny that there is any resurrection; and they
asked him,

20:28 Saying, Master, Moses wrote unto us, If any man's brother die, having a wife, and he die without
children, that his brother should take his wife, and raise up seed unto his brother.

20:29 There were therefore seven brethren: and the first took a wife, and died without children.

20:30 And the second took her to wife, and he died childless.

20:31 And the third took her; and in like manner the seven also: and they left no children, and died.

20:32 Last of all the woman died also.

20:33 Therefore in the resurrection whose wife of them is she? for seven had her to wife.

20:34 And Jesus answering said unto them, The children of this world marry, and are given in marriage:

20:35 But they which shall be accounted worthy to obtain that world, and the resurrection from the

http://breachrepairers.webs.com/ 530
dead, neither marry, nor are given in marriage:

20:36 Neither can they die any more: for they are equal unto the angels; and are the children of God,
being the children of the resurrection.

20:37 Now that the dead are raised, even Moses shewed at the bush, when he calleth the Lord the God of
Abraham, and the God of Isaac, and the God of Jacob.

20:38 For he is not a God of the dead, but of the living: for all live unto him.

20:39 Then certain of the scribes answering said, Master, thou hast well said.

How is Christ David's Son, and beware of scribes (40-47)


20:40 And after that they durst not ask him any question at all.

20:41 And he said unto them, How say they that Christ is David's son?

20:42 And David himself saith in the book of Psalms, The LORD said unto my Lord, Sit thou on my right
hand,

20:43 Till I make thine enemies thy footstool.

20:44 David therefore calleth him Lord, how is he then his son?

20:45 Then in the audience of all the people he said unto his disciples,

20:46 Beware of the scribes, which desire to walk in long robes, and love greetings in the markets, and
the highest seats in the synagogues, and the chief rooms at feasts;

20:47 Which devour widows' houses, and for a shew make long prayers: the same shall receive greater
damnation.

http://breachrepairers.webs.com/ 531
Chapter 21 - The Second Coming of Jesus
Observation: Same picture seen in 2 Thess 1:4-8 Patience, vengeance, 2nd Coming. In Luke 21:19
'Patience'
 Persecution
 Vengeance
 Redemption

Applications
 Before Kings and Rules (12)

Chapter Outline
 Widow with the two mites (1-4)
 Persecution & Betrayal of God's people (5-19)
 Days of Vengeance on Jerusalem (20-24)
 The Second Coming (signs and seasons) (25-33)
 Preparation to be ready for the Second Coming (34-38)

Widow with the two mites (1-4)


21:1 And he looked up, and saw the rich men casting their gifts into the treasury.

21:2 And he saw also a certain poor widow casting in thither two mites.

21:3 And he said, Of a truth I say unto you, that this poor widow hath cast in more than they all:

21:4 For all these have of their abundance cast in unto the offerings of God: but she of her penury hath
cast in all the living that she had.

When you give till it hurts, God will bless the offering that you give.

Persecution & Betrayal of God's people (5-19)


21:5 And as some spake of the temple, how it was adorned with goodly stones and gifts, he said,

21:6 As for these things which ye behold, the days will come, in the which there shall not be left one stone
upon another, that shall not be thrown down.

21:7 And they asked him, saying, Master, but when shall these things be? and what sign will there be
when these things shall come to pass?

21:8 And he said, Take heed that ye be not deceived: for many shall come in my name, saying, I am
Christ; and the time draweth near: go ye not therefore after them.

Many will come in my name saying I am Christ. So we have false Christ's coming and saying the time
draweth near. It is a statement that creates trust. No one in the Christian world is going to deny that
the coming of Christ is near. So just right there they are being prepared for deception. And Jesus
says

http://breachrepairers.webs.com/ 532
“go ye not therefore after them” – Cf. Luke17:23 'nor follow them' So the false Christ's are saying
follow me. Just like Jesus said to His disciples.

21:9 But when ye shall hear of wars and commotions, be not terrified: for these things must first come to
pass; but the end is not by and by.

The false Christ's are the ones responsible for wars and commotions. They are behind them.

“wars and commotions” – Wars describe military conflict. Commotions describe social unrest
more so in the communities. Commotion (AHD) – “Civil disturbance or insurrection; disorder”

“when ye shall hear of wars and commotions” – Who are they hearing this from? False Christ's.
The false Christ's are speaking about wars and commotions, and they are using this upheaval to say
'the time draweth near.” From this verse we can see that false Christ's are using the signs of the
times. Both groups use the signs of the times. The true followers of Christ are saying that the signs
of the times show us that Jesus will return. And the false Christ's are using signs of the times to say
His return is near. But the false Christ's are using these signs to deceive the people in following the
false Christ. But the end is not yet.

21:10 Then said he unto them, Nation shall rise against nation, and kingdom against kingdom:

21:11 And great earthquakes shall be in divers places, and famines, and pestilences; and fearful sights
and great signs shall there be from heaven.

21:12 But before all these, they shall lay their hands on you, and persecute you, delivering you up to the
synagogues, and into prisons, being brought before kings and rulers for my name's sake.

“Before Kings and Rulers” – The signs of the times show that soon we will be brought before kings
and rulers. (Extra: What kind of men do you think they will be? Lk 13:1 Pilate mingles blood with
sacrifices) How will you respond? What will you do?

Accusations against Jesus

Lk 23:2 …accuse Him…

Perverting the nation & Forbidding to give tribute. These accusations will be repeated in the last
days.

Lk 23:5

They were 'more fierce' intensity increases

Lk 23:10 …stood and vehemently accused' men of war 'mocked…

Lk 23:16, 21 …Chastise… [21] …crucify him…

Sentence against Jesus

Lk 23:18, 19 [18]…Away with this man [19] …sedition..murder…

Lk 23:20 …Willing to release...Crucify him…

http://breachrepairers.webs.com/ 533
Lk 23:24 …Pilate gave sentence…as they required…

Con/Appeal

Lk 23:25 And he released unto them him that for sedition and murder was cast into prison, whom
they had desired; but he delivered Jesus to their will.

When you look at what Jesus did for you are you willing to be accused in the last days even unto
death? By the grace of God I want to say I will. How about you?

21:13 And it shall turn to you for a testimony.

21:14 Settle it therefore in your hearts, not to meditate before what ye shall answer:

21:15 For I will give you a mouth and wisdom, which all your adversaries shall not be able to gainsay
nor resist.

21:16 And ye shall be betrayed both by parents, and brethren, and kinsfolks, and friends; and some of
you shall they cause to be put to death.

21:17 And ye shall be hated of all men for my name's sake.

21:18 But there shall not an hair of your head perish.

21:19 In your patience possess ye your souls.

Days of Vengeance on Jerusalem (20-24)


21:20 And when ye shall see Jerusalem compassed with armies, then know that the desolation thereof is
nigh.

21:21 Then let them which are in Judaea flee to the mountains; and let them which are in the midst of it
depart out; and let not them that are in the countries enter thereinto.

21:22 For these be the days of vengeance, that all things which are written may be fulfilled.

21:23 But woe unto them that are with child, and to them that give suck, in those days! for there shall be
great distress in the land, and wrath upon this people.

21:24 And they shall fall by the edge of the sword, and shall be led away captive into all nations: and
Jerusalem shall be trodden down of the Gentiles, until the times of the Gentiles be fulfilled.

The Second Coming (signs and seasons) (25-33)


21:25 And there shall be signs in the sun, and in the moon, and in the stars; and upon the earth distress
of nations, with perplexity; the sea and the waves roaring;

21:26 Men's hearts failing them for fear, and for looking after those things which are coming on the
earth: for the powers of heaven shall be shaken.

21:27 And then shall they see the Son of man coming in a cloud with power and great glory.

21:28 And when these things begin to come to pass, then look up, and lift up your heads; for your
redemption draweth nigh.

http://breachrepairers.webs.com/ 534
21:29 And he spake to them a parable; Behold the fig tree, and all the trees;

21:30 When they now shoot forth, ye see and know of your own selves that summer is now nigh at hand.

21:31 So likewise ye, when ye see these things come to pass, know ye that the kingdom of God is nigh at
hand.

21:32 Verily I say unto you, This generation shall not pass away, till all be fulfilled.

21:33 Heaven and earth shall pass away: but my words shall not pass away.

Preparation to be ready for the Second Coming (34-38)


21:34 And take heed to yourselves, lest at any time your hearts be overcharged with surfeiting, and
drunkenness, and cares of this life, and so that day come upon you unawares.

21:35 For as a snare shall it come on all them that dwell on the face of the whole earth.

21:36 Watch ye therefore, and pray always, that ye may be accounted worthy to escape all these things
that shall come to pass, and to stand before the Son of man.

21:37 And in the day time he was teaching in the temple; and at night he went out, and abode in the
mount that is called the mount of Olives.

21:38 And all the people came early in the morning to him in the temple, for to hear him.

http://breachrepairers.webs.com/ 535
Chapter 22 - Betrayal and Denial of the Son of Man
The majority of the chapter covers the betrayal of Jesus Christ by Judas and the denial of Jesus
Christ by Peter.

 Betrayal
o It shows who incited the betrayal (Satan)
o It shows who Judas consulted with in his betrayal
o It shows when Judas was identified as the betrayer
o And it shows the betrayal itself.
 Denial
o It shows when Peter said he wouldn't deny
o It shows when Peter denied
o It shows Peter's reaction to his denial
 Observation:
o Peter denied Jesus before men. According to Luke 12 at that moment Peter was
disqualified from Jesus being able to confess Peter in heaven before the angels of
God (If Jesus would have been in heaven.)
o It shows how Jesus is one with us in temptation as a man. (James 1:13 'God cannot
be tempted'). And it shows what that temptation was 'remove this cup'. The cup was
the betrayal of Judas and the denial of Peter. It is natural for us to expect our
enemies to betray and deny us. It is harder to experience from people who we have
lived with for 3 years.(Jesus tried to save Judas) Jesus is giving us an example of how
to endure betrayal and denial as humans. Jesus just told them in Luke 21 that they
would be betrayed by friends, brethren, kinsfolk, etc. Then He gives an
example of how to endure it. This chapter ends with the second coming. Because
He endured the cup He would return. Luke 21 ends with 2nd coming because we
endure betrayal we will be ready. Etc.
o It shows that an angel had to strengthen Him. Why? Because He was a Man. And His
humanity in and of itself could not endure the cup.
o Blood hit the ground. The same ground that these two men were taken out of. The
life of the flesh is in the blood. He would give His life for those He created, and His
life would also restore the world He created (work on that)
Chapter Outline
 Judas makes a covenant to betray Jesus (1-6)
 The Passover supper & who is the greatest (7-30)
 Peter pledges not to deny Christ (31-38)
 Gethsemane (39-53)
 Trial of Jesus and the denial of Peter (54-71)

http://breachrepairers.webs.com/ 536
Judas makes a covenant to betray Jesus (1-6)
22:1 Now the feast of unleavened bread drew nigh, which is called the Passover.

22:2 And the chief priests and scribes sought how they might kill him; for they feared the people.

22:3 Then entered Satan into Judas surnamed Iscariot, being of the number of the twelve.

22:4 And he went his way, and communed with the chief priests and captains, how he might betray him
unto them.

22:5 And they were glad, and covenanted to give him money.

22:6 And he promised, and sought opportunity to betray him unto them in the absence of the multitude.

The Passover supper & who is the greatest (7-30)


22:7 Then came the day of unleavened bread, when the passover must be killed.

22:8 And he sent Peter and John, saying, Go and prepare us the passover, that we may eat.

22:9 And they said unto him, Where wilt thou that we prepare?

22:10 And he said unto them, Behold, when ye are entered into the city, there shall a man meet you,
bearing a pitcher of water; follow him into the house where he entereth in.

22:11 And ye shall say unto the goodman of the house, The Master saith unto thee, Where is the
guestchamber, where I shall eat the passover with my disciples?

22:12 And he shall shew you a large upper room furnished: there make ready.

Why not just tell them where they were going to eat? Judas would have went straightway to tell the
priests where they were meeting.

22:13 And they went, and found as he had said unto them: and they made ready the passover.

22:14 And when the hour was come, he sat down, and the twelve apostles with him.

22:15 And he said unto them, With desire I have desired to eat this passover with you before I suffer:

22:16 For I say unto you, I will not any more eat thereof, until it be fulfilled in the kingdom of God.

22:17 And he took the cup, and gave thanks, and said, Take this, and divide it among yourselves:

22:18 For I say unto you, I will not drink of the fruit of the vine, until the kingdom of God shall come.

22:19 And he took bread, and gave thanks, and brake it, and gave unto them, saying, This is my body
which is given for you: this do in remembrance of me.

22:20 Likewise also the cup after supper, saying, This cup is the new testament in my blood, which is
shed for you.

22:21 But, behold, the hand of him that betrayeth me is with me on the table.

22:22 And truly the Son of man goeth, as it was determined: but woe unto that man by whom he is

http://breachrepairers.webs.com/ 537
betrayed!

22:23 And they began to enquire among themselves, which of them it was that should do this thing.

22:24 And there was also a strife among them, which of them should be accounted the greatest.

22:25 And he said unto them, The kings of the Gentiles exercise lordship over them; and they that
exercise authority upon them are called benefactors.

22:26 But ye shall not be so: but he that is greatest among you, let him be as the younger; and he that is
chief, as he that doth serve.

22:27 For whether is greater, he that sitteth at meat, or he that serveth? is not he that sitteth at meat?
but I am among you as he that serveth.

22:28 Ye are they which have continued with me in my temptations.

22:29 And I appoint unto you a kingdom, as my Father hath appointed unto me;

22:30 That ye may eat and drink at my table in my kingdom, and sit on thrones judging the twelve tribes
of Israel.

Peter pledges not to deny Christ (31-38)


22:31 And the Lord said, Simon, Simon, behold, Satan hath desired to have you, that he may sift you as
wheat:

22:32 But I have prayed for thee, that thy faith fail not: and when thou art converted, strengthen thy
brethren.

22:33 And he said unto him, Lord, I am ready to go with thee, both into prison, and to death.

22:34 And he said, I tell thee, Peter, the cock shall not crow this day, before that thou shalt thrice deny
that thou knowest me.

22:35 And he said unto them, When I sent you without purse, and scrip, and shoes, lacked ye any thing?
And they said, Nothing.

22:36 Then said he unto them, But now, he that hath a purse, let him take it, and likewise his scrip: and
he that hath no sword, let him sell his garment, and buy one.

22:37 For I say unto you, that this that is written must yet be accomplished in me, And he was reckoned
among the transgressors: for the things concerning me have an end.

22:38 And they said, Lord, behold, here are two swords. And he said unto them, It is enough.

Gethsemane (39-53)
22:39 And he came out, and went, as he was wont, to the mount of Olives; and his disciples also followed
him.

22:40 And when he was at the place, he said unto them, Pray that ye enter not into temptation.

22:41 And he was withdrawn from them about a stone's cast, and kneeled down, and prayed,

http://breachrepairers.webs.com/ 538
22:42 Saying, Father, if thou be willing, remove this cup from me: nevertheless not my will, but thine, be
done.

22:43 And there appeared an angel unto him from heaven, strengthening him.

22:44 And being in an agony he prayed more earnestly: and his sweat was as it were great drops of
blood falling down to the ground.

22:45 And when he rose up from prayer, and was come to his disciples, he found them sleeping for
sorrow,

22:46 And said unto them, Why sleep ye? rise and pray, lest ye enter into temptation.

22:47 And while he yet spake, behold a multitude, and he that was called Judas, one of the twelve, went
before them, and drew near unto Jesus to kiss him.

22:48 But Jesus said unto him, Judas, betrayest thou the Son of man with a kiss?

22:49 When they which were about him saw what would follow, they said unto him, Lord, shall we smite
with the sword?

22:50 And one of them smote the servant of the high priest, and cut off his right ear.

22:51 And Jesus answered and said, Suffer ye thus far. And he touched his ear, and healed him.

22:52 Then Jesus said unto the chief priests, and captains of the temple, and the elders, which were come
to him, Be ye come out, as against a thief, with swords and staves?

22:53 When I was daily with you in the temple, ye stretched forth no hands against me: but this is your
hour, and the power of darkness.

Trial of Jesus and the denial of Peter (54-71)


22:54 Then took they him, and led him, and brought him into the high priest's house. And Peter followed
afar off.

22:55 And when they had kindled a fire in the midst of the hall, and were set down together, Peter sat
down among them.

22:56 But a certain maid beheld him as he sat by the fire, and earnestly looked upon him, and said, This
man was also with him.

22:57 And he denied him, saying, Woman, I know him not.

22:58 And after a little while another saw him, and said, Thou art also of them. And Peter said, Man, I am
not.

22:59 And about the space of one hour after another confidently affirmed, saying, Of a truth this fellow
also was with him: for he is a Galilaean.

22:60 And Peter said, Man, I know not what thou sayest. And immediately, while he yet spake, the cock
crew.

22:61 And the Lord turned, and looked upon Peter. And Peter remembered the word of the Lord, how he
had said unto him, Before the cock crow, thou shalt deny me thrice.

http://breachrepairers.webs.com/ 539
22:62 And Peter went out, and wept bitterly.

22:63 And the men that held Jesus mocked him, and smote him.

22:64 And when they had blindfolded him, they struck him on the face, and asked him, saying, Prophesy,
who is it that smote thee?

22:65 And many other things blasphemously spake they against him.

22:66 And as soon as it was day, the elders of the people and the chief priests and the scribes came
together, and led him into their council, saying,

22:67 Art thou the Christ? tell us. And he said unto them, If I tell you, ye will not believe:

22:68 And if I also ask you, ye will not answer me, nor let me go.

22:69 Hereafter shall the Son of man sit on the right hand of the power of God.

22:70 Then said they all, Art thou then the Son of God? And he said unto them, Ye say that I am.

22:71 And they said, What need we any further witness? for we ourselves have heard of his own mouth.

http://breachrepairers.webs.com/ 540
Chapter 23 - Jesus before Pilate and Herod

Chapter Outline
 Jesus before Pilate the first time (1-7)
 Jesus before Herod (8-12)
 Jesus before Pilate the second time (13-26)
 Jesus goes to Calvary (27-34)
 Death and burial of Jesus (35-56)

Jesus before Pilate the first time (1-7)


23:1 And the whole multitude of them arose, and led him unto Pilate.

"led him unto Pilate"

23:2 And they began to accuse him, saying, We found this fellow perverting the nation, and forbidding to
give tribute to Caesar, saying that he himself is Christ a King.

“Tribute” – A gift, payment, declaration, or other acknowledgment of gratitude, respect, or


admiration: In paying tribute it shows your respect, gratitude. I believe these accusations will be
repeated in the last days.

23:3 And Pilate asked him, saying, Art thou the King of the Jews? And he answered him and said, Thou
sayest it.

Look at how Pilate responds to these accusations. The Jews mentioned three different accusations
and as Pilate was listening to these accusations there was only one that caught his attention. It's like
he didn't even care about the other two. As Pilate saw Jesus' disposition, His bearing, and His
composer, he was not convinced that the Pharisees were telling the truth. When he heard the first
accusation and looked at Jesus, no, that can't be true, second accusation, the same. But when he
heard the third accusation because of what he saw he was convinced that this Man was a King. As
he looked at Jesus He saw a man with soiled clothing, worn sandals and holes in His clothes. His
apparel didn't prick the conscience of Pilate. His external appearance testified against Him being a
King, but His character swung the balance of the thoughts of Pilate to one fact, HE IS A KING!

App: What do people see when they see you? Do they know that you are a child of the King? Or
would your very countenance condemn you? What would government officials think if you were
accused today?

23:4 Then said Pilate to the chief priests and to the people, I find no fault in this man.

Read over

23:5 And they were the more fierce, saying, He stirreth up the people, teaching throughout all Jewry,
beginning from Galilee to this place.

They were the what? 'more fierce' saying.. And then Pilate sends Him to Herod. Verse 9

http://breachrepairers.webs.com/ 541
23:6 When Pilate heard of Galilee, he asked whether the man were a Galilaean.

23:7 And as soon as he knew that he belonged unto Herod's jurisdiction, he sent him to Herod, who
himself also was at Jerusalem at that time.

Jesus before Herod (8-12)


23:8 And when Herod saw Jesus, he was exceeding glad: for he was desirous to see him of a long season,
because he had heard many things of him; and he hoped to have seen some miracle done by him.

23:9 Then he questioned with him in many words; but he answered him nothing.

23:10 And the chief priests and scribes stood and vehemently accused him.

“vehemently accused him”

23:11 And Herod with his men of war set him at nought, and mocked him, and arrayed him in a gorgeous
robe, and sent him again to Pilate.

“mocked him” – If you pay attention very carefully you will see that the intensity of the accusations
increase in increments. The energy level increases. They said this Man was perverting the nation…,
and then they were more fierce, then they were vehement, and finally they mocked Him. They got
physical. This will happen again in the last days.

App: The only way you will respond to vehement accusations in last days the way a Christian
should is if you are like the MAN Christ Jesus.

23:12 And the same day Pilate and Herod were made friends together: for before they were at enmity
between themselves.

Even in His trial and mockery He could not help but bring reconciliation.

App: I believe this will happen again. Ex. California Senator Vs. Virginian Senator arguing over gay
rights, abortion, they separate and become enemies, and when the SDA's come to their attention.
They agree and become reconciled. I pray their fate will be different from Herod's and Pilate's.

App: God will save senators and government officials through our testimony. And now the sentence

Jesus before Pilate the second time (13-26)


23:13 And Pilate, when he had called together the chief priests and the rulers and the people,

23:14 Said unto them, Ye have brought this man unto me, as one that perverteth the people: and, behold,
I, having examined him before you, have found no fault in this man touching those things whereof ye
accuse him:

23:15 No, nor yet Herod: for I sent you to him; and, lo, nothing worthy of death is done unto him.

23:16 I will therefore chastise him, and release him.

23:17 (For of necessity he must release one unto them at the feast.)

23:18 And they cried out all at once, saying, Away with this man, and release unto us Barabbas:

http://breachrepairers.webs.com/ 542
23:19 (Who for a certain sedition made in the city, and for murder, was cast into prison.)

23:20 Pilate therefore, willing to release Jesus, spake again to them.

23:21 But they cried, saying, Crucify him, crucify him.

23:22 And he said unto them the third time, Why, what evil hath he done? I have found no cause of death
in him: I will therefore chastise him, and let him go.

23:23 And they were instant with loud voices, requiring that he might be crucified. And the voices of
them and of the chief priests prevailed.

23:24 And Pilate gave sentence that it should be as they required.

23:25 And he released unto them him that for sedition and murder was cast into prison, whom they had
desired; but he delivered Jesus to their will.

“for sedition and murder” – Sedition = (AHD) Conduct or language inciting rebellion against the
authority of a state. Here we find that the same accusations that were given against Jesus are now
found in Barabbas. Don't tell me he wasn't perverting the nation and I'm sure he wasn't saying give
tribute to Ceasar, but they wanted the one who was guilty to be released. For sedition and murder
he was imprisoned.

App: Have you ever been guilty of these things? The bible says in 1 John 3:15 Whosoever hateth his
brother is a murderer. Have you ever hated anyone? Your mother, father, friend or loved one. Have
you ever stolen something from the government? Do you think you weren't perverting the nation?
SO WHO IS BARABBAS THIS MORNING?

Con/Appeal: Personal testimony. When you look at what He has done for you, are you willing to go
before kings and rulers to face accusations and even death? I want to say by the grace of God. I am
willing.

23:26 And as they led him away, they laid hold upon one Simon, a Cyrenian, coming out of the country,
and on him they laid the cross, that he might bear it after Jesus.

Jesus goes to Calvary (27-34)


23:27 And there followed him a great company of people, and of women, which also bewailed and
lamented him.

Encounter 1 – Women Of Jerusalem

Why do you think the women were crying? They lamented Jesus. They pitied Him.

23:28 But Jesus turning unto them said, Daughters of Jerusalem, weep not for me, but weep for
yourselves, and for your children.

What was Jesus reply and what can we learn from Him for our own cross experience? Worry
about your own future. Soon they would say that it would be blessed to be barren. During the dark
ages, many of God’s people had to suffer in watching their children killed before them. The Jews
suffered many atrocities during destruction of Jerusalem and the world wars. APPLICATION: When
we are carrying our own cross and others cry for us, we can point them to their own future. What

http://breachrepairers.webs.com/ 543
kind of future? Point to prophecy. Our cross is experience is a way to point others to their own
futures and the prophecies in God’s Word.

Who could the women represent? A group who can see Christ, cry for Christ but don’t necessarily
know him.

23:29 For, behold, the days are coming, in the which they shall say, Blessed are the barren, and the
wombs that never bare, and the paps which never gave suck.

23:30 Then shall they begin to say to the mountains, Fall on us; and to the hills, Cover us.

23:31 For if they do these things in a green tree, what shall be done in the dry?

23:32 And there were also two other, malefactors, led with him to be put to death.

23:33 And when they were come to the place, which is called Calvary, there they crucified him, and the
malefactors, one on the right hand, and the other on the left.

23:34 Then said Jesus, Father, forgive them; for they know not what they do. And they parted his
raiment, and cast lots.

Death and burial of Jesus (35-56)


23:35 And the people stood beholding. And the rulers also with them derided him, saying, He saved
others; let him save himself, if he be Christ, the chosen of God.

23:36 And the soldiers also mocked him, coming to him, and offering him vinegar,

23:37 And saying, If thou be the king of the Jews, save thyself.

Encounter 2 – ROMAN SOLDIERS

What were the Roman soldiers trying to do?

Luke 23:36-37, 47

They mocked Jesus. They tried giving Him vinegar. The centurion eventually acknowledged Christ
as righteous. NOTE: the heathens were convicted as a result of Christ’s actions and encounters on
the cross.

What can we learn from Jesus’ action for our own cross experience?

Matt 27:34

Jesus was bearing his cross, but He still wouldn’t take substances that would cloud His mind.
APPLICATION: we should not try to avoid the pain and suffering of our cross experience by
numbing our senses with substances or other solutions.

Encounter 3 – Mary, Mother Of Jesus

What was Jesus trying to do for His mother?

John 19:25-27

http://breachrepairers.webs.com/ 544
He wanted John to take care of her in His place.

What can we learn from this encounter for our own cross experience?

Matt 27:19

Even while being crucified, Jesus could still remember His mother. Likewise, when we are bearing
our cross we should not forget the welfare of our loved ones.

23:38 And a superscription also was written over him in letters of Greek, and Latin, and Hebrew, THIS IS
THE KING OF THE JEWS.

23:39 And one of the malefactors which were hanged railed on him, saying, If thou be Christ, save thyself
and us.

23:40 But the other answering rebuked him, saying, Dost not thou fear God, seeing thou art in the same
condemnation?

23:41 And we indeed justly; for we receive the due reward of our deeds: but this man hath done nothing
amiss.

23:42 And he said unto Jesus, Lord, remember me when thou comest into thy kingdom.

23:43 And Jesus said unto him, Verily I say unto thee, To day shalt thou be with me in paradise.

Encounter 5 – The Thief

What was the problem with the attitude of one of the thieves? How are we like this thief?
Luke 23:39

He mocked Jesus and asked Him to prove Himself by saving Himself and the thieves.APPLICATION:
We can be like this thief when during our trials we mock Christ by criticizing Him and challenging
Him to get us out of our miseries or consequences of our sins. Faith does not demand that God save
us from every circumstance.

How did the other thief respond to Jesus? How are we like this thief?

Luke 23:40-43

The other thief rebukes his fellow thief and recognizes Jesus as the Son of God. He recognizes that
both of the thieves deserved their sentences, but Jesus did not. He asks Jesus to remember Him and
Jesus rewards his faith by assuring him of salvation. Application: We can be like this thief when we
recognize how deserving we are of the consequences of our sin, and how deserving we are of our
crosses, and recognize that Jesus died for us, who deserved no punishment. This is the right attitude
to approach Jesus, and Jesus in turn will respond favorably. NOTE: This thief did not demand
anything of Christ, but asked that Jesus remember Him. Likewise, we have no right to demand
anything of God. With meekness and humility, we should plead for God to remember us.

23:44 And it was about the sixth hour, and there was a darkness over all the earth until the ninth hour.

23:45 And the sun was darkened, and the veil of the temple was rent in the midst.

http://breachrepairers.webs.com/ 545
23:46 And when Jesus had cried with a loud voice, he said, Father, into thy hands I commend my spirit:
and having said thus, he gave up the ghost.

Encounter 5 – God, The Father

Why did Jesus say “Why hast thou forsaken Me”??

Matthew 27:46

John 12:49-50

Jesus spoke only those things from the Father.

John 2:19-21

Therefore, the Father had told Jesus that He would raise Him up from the dead in three days.

2 Cor 5:21

Isa 59:2

At that very moment, God the Father could not look upon Jesus, because He was on the cross and
made to be sin. Also, the Father was showing His wrath to the Son.

What can we learn from Jesus’ next two statements to the Father?

Luke 23:34

Jesus asked the Father to forgive His enemies, His persecutors. APPLICATION 1 = learn to ask the
Father to forgive your enemies during your cross experience.

Luke 23:46

Even though Jesus could not see past the cross He submitted His life in God’s hands.. Application 2 =
Likewise when we feel has though God left us or are going through trials and can’t see beyond this
trial (feels as though you are going to die or lose everything), be like Jesus and submit your life in
the Father’s hands.

Each encounter at the cross gives us an example of how we should act during the times when
we are bearing our cross. Summarize the lessons learnt.

Summary:
Encounter 1 = When others mourn for us, point them to their own futures and prophecy.
Encounter 2 = Do not try to avoid the pain of our cross experience by taking substances.
Encounter 3 = Don’t stop caring for our loved ones during our cross experiences.
Encounter 4 = Don’t demand that God prove Himself by saving you or others. Recognize your own
unworthiness and deserving of the punishment for your sins,
Encounter 5 = Forgive your enemies and submit to God even if you can’t see past your cross.

23:47 Now when the centurion saw what was done, he glorified God, saying, Certainly this was a

http://breachrepairers.webs.com/ 546
righteous man.

23:48 And all the people that came together to that sight, beholding the things which were done, smote
their breasts, and returned.

23:49 And all his acquaintance, and the women that followed him from Galilee, stood afar off, beholding
these things.

23:50 And, behold, there was a man named Joseph, a counsellor; and he was a good man, and a just:

23:51 (The same had not consented to the counsel and deed of them;) he was of Arimathaea, a city of the
Jews: who also himself waited for the kingdom of God.

23:52 This man went unto Pilate, and begged the body of Jesus.

23:53 And he took it down, and wrapped it in linen, and laid it in a sepulchre that was hewn in stone,
wherein never man before was laid.

23:54 And that day was the preparation, and the sabbath drew on.

23:55 And the women also, which came with him from Galilee, followed after, and beheld the sepulchre,
and how his body was laid.

23:56 And they returned, and prepared spices and ointments; and rested the sabbath day according to
the commandment.

http://breachrepairers.webs.com/ 547
Chapter 24 - The Resurrection of Jesus
Chapter Outline
 Mary and others tell disciples Jesus has risen (1-12)
 Walk to Emmaus (13-35)
 Jesus in the upper room with His disciples (36-49)
 Jesus returns to heaven (50-53)

Mary and others tell disciples Jesus has risen (1-12)


24:1 Now upon the first day of the week, very early in the morning, they came unto the sepulchre,
bringing the spices which they had prepared, and certain others with them.

24:2 And they found the stone rolled away from the sepulchre.

24:3 And they entered in, and found not the body of the Lord Jesus.

24:4 And it came to pass, as they were much perplexed thereabout, behold, two men stood by them in
shining garments:

24:5 And as they were afraid, and bowed down their faces to the earth, they said unto them, Why seek ye
the living among the dead?

24:6 He is not here, but is risen: remember how he spake unto you when he was yet in Galilee,

24:7 Saying, The Son of man must be delivered into the hands of sinful men, and be crucified, and the
third day rise again.

24:8 And they remembered his words,

24:9 And returned from the sepulchre, and told all these things unto the eleven, and to all the rest.

24:10 It was Mary Magdalene, and Joanna, and Mary the mother of James, and other women that were
with them, which told these things unto the apostles.

24:11 And their words seemed to them as idle tales, and they believed them not.

24:12 Then arose Peter, and ran unto the sepulchre; and stooping down, he beheld the linen clothes laid
by themselves, and departed, wondering in himself at that which was come to pass.

Walk to Emmaus (13-35)


24:13 And, behold, two of them went that same day to a village called Emmaus, which was from
Jerusalem about threescore furlongs.

Where were these disciples going? Emmaus. Note: Threescore furlongs = 7.5 Miles (12 km)

24:14 And they talked together of all these things which had happened.

24:15 And it came to pass, that, while they communed together and reasoned, Jesus himself drew near,
and went with them.

http://breachrepairers.webs.com/ 548
24:16 But their eyes were holden that they should not know him.

24:17 And he said unto them, What manner of communications are these that ye have one to another, as
ye walk, and are sad?

Why do you think “their eyes were holden”?

Luke 24:14-17

What do you think was the subject of these disciples’ conversation? The clue is in Jesus’ comment to
them in verse 17 – “sad.” They were so fixed (“holden”) on their grief and disappointment at Jesus’
death that they were unable to recognize. Their hopes were crushed because of Jesus’ death. They
could not see beyond that and as a result could not recognize the resurrected Jesus.

How can we be like these disciples? Sometimes when God allows something precious to us to be
taken away, just as Jesus was taken away from the disciples, our hopes can be crushed. And we are
so disappointed that we can’t see past the gloom. We choose to focus on the negatives rather than
the positives. Remember, every situation can be turned into a positive.

What do you think was Jesus’ intent in coming to these disciples?

Luke 24:14-17

Jesus wanted to comfort these disciples and refocus them. Let us look at how he does it.

Why do you think Jesus asks the obvious question in verse 17? What is He trying to get the
disciples to do? What lesson can we learn in comforting others?

24:18 And the one of them, whose name was Cleopas, answering said unto him, Art thou only a stranger
in Jerusalem, and hast not known the things which are come to pass there in these days?

24:19 And he said unto them, What things? And they said unto him, Concerning Jesus of Nazareth, which
was a prophet mighty in deed and word before God and all the people:

24:20 And how the chief priests and our rulers delivered him to be condemned to death, and have
crucified him.

24:21 But we trusted that it had been he which should have redeemed Israel: and beside all this, to day is
the third day since these things were done.

Why do you think Jesus asks the obvious question in verse 17? What is He trying to get the
disciples to do? What lesson can we learn in comforting others?

Luke 24:17-21

To give them a chance to pour their hearts out and get their burdens off their chest. This is how
Jesus expects us to pray to Him. To give Him the burdens of our hearts so that He can comfort us.
We should be honest and direct with Jesus, just as these disciples were.

What is the focus of the disciples in their disappointment?

Luke 24:20

http://breachrepairers.webs.com/ 549
Christ’s death and crucifixion.

What should have been the focus instead?

Luke 24:21

Matt 12:40

Matt 27:62-63

Jesus had prophesied His own death and resurrection. If His death had taken place, the disciples
should have believed that His resurrection was going to take place. Even the Pharisees and scribes
believed and were afraid of this.

24:22 Yea, and certain women also of our company made us astonished, which were early at the
sepulchre;

24:23 And when they found not his body, they came, saying, that they had also seen a vision of angels,
which said that he was alive.

24:24 And certain of them which were with us went to the sepulchre, and found it even so as the women
had said: but him they saw not.

How had the disciples informed of Jesus’ resurrection already? Why did they not believe?

Luke 24:22-24

The disciples had been informed through the women who had gone to the tomb early that morning.
Why did they not believe? Rather than taking this as good news based on what Christ had told them
while alive, they chose to focus on their grief instead. IMPORTANT: Why were they consumed in
their grief? See Luke 24:21 – they had trusted that Christ would be the earthly redeemer. He did not
fulfill their expectations from a human stand point. Application: When Christ does not answer our
prayers in the way that we expect from a human stand point, we must be careful not to lose all hope
and dwell on our grief. We should look for the way in which God is trying to answer our prayer.

What was Jesus’ solution for bringing the disciples out of their grief?

Luke 24:25-27

Jesus revealed to them from the scriptures.

24:25 Then he said unto them, O fools, and slow of heart to believe all that the prophets have spoken:

24:26 Ought not Christ to have suffered these things, and to enter into his glory?

24:27 And beginning at Moses and all the prophets, he expounded unto them in all the scriptures the
things concerning himself.

What was the importance of showing the disciples “from Moses” especially for us?

Luke 24:27

http://breachrepairers.webs.com/ 550
Jesus’ showed the importance of the Old Testament even for us today in revealing Christ. Note: He
expounded “concerning Himself”

24:28 And they drew nigh unto the village, whither they went: and he made as though he would have
gone further.

Why did Jesus make as if he wanted to go further?To test their sincerity and desire to know the
truth because they pleaded with Him to stay with them.

24:29 But they constrained him, saying, Abide with us: for it is toward evening, and the day is far spent.
And he went in to tarry with them.

24:30 And it came to pass, as he sat at meat with them, he took bread, and blessed it, and brake, and
gave to them.

24:31 And their eyes were opened, and they knew him; and he vanished out of their sight.

When were their eyes finally opened? When Jesus broke bread with them. How is Jesus able to
break bread with us today? Through His word. That is why it is important for us to pray “Give us
this day our daily bread”.

What was it that lead their hearts to burn within them?

Luke 24:29

“while he talked with us” = conversing with Christ through prayer.

“while he opened the scriptures to us” = conversing with Christ through the Word (the
Bible).

What was the result of having their hearts burning and their eyes opened?

Luke 24:33
:
They went to share the good news that Jesus was resurrected with their friends. APPLICATION: the
result of having our hearts burn within us and our eyes opened should be to go out and share the
good news about Christ.

24:32 And they said one to another, Did not our heart burn within us, while he talked with us by the way,
and while he opened to us the scriptures?

24:33 And they rose up the same hour, and returned to Jerusalem, and found the eleven gathered
together, and them that were with them,

24:34 Saying, The Lord is risen indeed, and hath appeared to Simon.

24:35 And they told what things were done in the way, and how he was known of them in breaking of
bread.

http://breachrepairers.webs.com/ 551
Jesus in the upper room with His disciples (36-49)
24:36 And as they thus spake, Jesus himself stood in the midst of them, and saith unto them, Peace be
unto you.

24:37 But they were terrified and affrighted, and supposed that they had seen a spirit.

24:38 And he said unto them, Why are ye troubled? and why do thoughts arise in your hearts?

24:39 Behold my hands and my feet, that it is I myself: handle me, and see; for a spirit hath not flesh and
bones, as ye see me have.

24:40 And when he had thus spoken, he shewed them his hands and his feet.

24:41 And while they yet believed not for joy, and wondered, he said unto them, Have ye here any meat?

24:42 And they gave him a piece of a broiled fish, and of an honeycomb.

24:43 And he took it, and did eat before them.

24:44 And he said unto them, These are the words which I spake unto you, while I was yet with you, that
all things must be fulfilled, which were written in the law of Moses, and in the prophets, and in the
psalms, concerning me.

24:45 Then opened he their understanding, that they might understand the scriptures,

24:46 And said unto them, Thus it is written, and thus it behoved Christ to suffer, and to rise from the
dead the third day:

24:47 And that repentance and remission of sins should be preached in his name among all nations,
beginning at Jerusalem.

24:48 And ye are witnesses of these things.

24:49 And, behold, I send the promise of my Father upon you: but tarry ye in the city of Jerusalem, until
ye be endued with power from on high.

Jesus returns to heaven (50-53)


24:50 And he led them out as far as to Bethany, and he lifted up his hands, and blessed them.

24:51 And it came to pass, while he blessed them, he was parted from them, and carried up into heaven.

24:52 And they worshipped him, and returned to Jerusalem with great joy:

24:53 And were continually in the temple, praising and blessing God. Amen.

http://breachrepairers.webs.com/ 552
John
Introduction
Why was the book written?

Jn 20:31 – Anointed One, Messiah, Son of God

Jn 20:30 – Everything not written

Jn 21:25 – Many miracles and deeds were done, but the world couldn't contain every act.

4 Major Parts: (4 Passovers)

Jn 2:13 – 1st Pass

Jn 6:4 – 2nd Pass

Jn 7:2 – Indicate 3rd Pass

Jn 11:55 – 4th Pass

Book of John based on the Passover of Jess

Comparison with Revelation:

Lamb of God

Disciples following the Lamb

Nathaniel No Guile, King of Israel, those that overcome, how do they overcome? (144,000)
through

Prayer

MHP Prayer – Jesus John 17

Conclusion: Do you want to be a living manifestation of the work of the Lamb.

Author and Title


The title says that the Gospel was written by John, and other evidence identifies this John as the son
of Zebedee. The internal evidence indicates that the author was (1) an apostle (1:14; cf. 2:11;
19:35), (2) one of the 12 disciples (“the disciple whom Jesus loved”; 13:23; 19:26; 20:2; 21:20; cf.
21:24–25), and, still more specifically, (3) John the son of Zebedee (note the association of “the
disciple whom Jesus loved” with Peter in 13:23–24; 18:15–16; 20:2–9; 21:2–23; cf. Luke 22:8; Acts
1:13; 3:1–4:37; 8:14–25; Gal. 2:9).

http://breachrepairers.webs.com/ 553
Date and Place of Writing
The most likely date of writing is the period between a.d. 70 (the date of the destruction of the
temple) and a.d. 100 (the end of John's lifetime), but there is not enough evidence to be much more
precise. A date subsequent to a.d. 70 is suggested, among other things, by the references in 6:1 and
21:1 to the Sea of Tiberias (a name widely used for the Sea of Galilee only toward the end of the 1st
century), the reference in 21:19 to Peter's martyrdom (which according to patristic evidence
occurred in a.d. 65 or 66), and the lack of reference to the Sadducees (who ceased to be a Jewish
religious party after a.d. 70).
The most likely place of writing is Ephesus in Asia Minor (modern-day Turkey), which was one of
the most important urban centers of the Roman Empire at the time. However, the readership
envisioned by John's Gospel transcends any one historical setting.

Theme
The theme of John's Gospel is that Jesus is the promised Messiah and Son of God. By believing in
Jesus, people can have eternal life (cf. 20:30–31).

Purpose, Occasion, and Background


The Gospel of John was written by the apostle John, the son of Zebedee, a Palestinian Jew and a
member of Jesus' inner apostolic circle during his earthly ministry. John's original audience
consisted of both Jews and Gentiles living in the larger Greco-Roman world in Ephesus and beyond
toward the close of the first century a.d. He frequently explains Jewish customs and Palestinian
geography and translates Aramaic terms into Greek (see note on 1:38), thus showing awareness of
non-Jewish readers. He also presents Jesus as the Word become flesh against the backdrop of Greek
thought that included Stoicism and early Gnosticism. But John also shows awareness of Jewish
readers as he demonstrates Jesus to be the Jewish Messiah, the fulfillment of many OT themes, and
the Son of God who was sent by God the Father to reveal the only true God and to provide
redemption for humanity.

The purpose statement in 20:30–31 makes it appear that John wrote with an evangelistic intent.
However, his depth of teaching shows that he wanted readers not only to come to initial saving faith
in Jesus but also to grow into a rich, well-informed faith. John's central contention is that Jesus is the
long-awaited Messiah and Son of God, and that by believing in him people may have eternal life. To
this end, he marshals the evidence of several selected messianic signs performed by Jesus and of a
series of witnesses to Jesus—including the Scriptures, John the Baptist, Jesus himself, God the
Father, Jesus' works, the Spirit, and John himself. It is also likely that John sought to present Jesus as
the new temple and center of worship for God's people, a concept that would be especially forceful
if the date of composition (as seems likely) was subsequent to a.d. 70 (the time of the destruction of
the Jerusalem temple).

Key Themes
1. Jesus is God. 1:1–2, 18; 5:17–18; 8:58–59; 10:30–33; 20:28
2. Jesus existed before the creation of the 1:1–2; 8:58; 17:5, 24
world.
3. Jesus has supernatural knowledge. 1:48; 2:4, 19, 23–25; 3:14; 4:17–18; 6:51, 70; 8:28;
9:3; 10:15, 17–18; 11:4, 14; 12:24, 32; 13:10–11,
38; 21:18–19
4. Jesus is the Messiah and the Son of God. 1:36, 41, 49; 3:18; 4:25, 29; 5:25; 7:26, 27, 31, 41,

http://breachrepairers.webs.com/ 554
42; 9:22; 10:24, 36; 11:4, 27; 12:34; 19:7; 20:30–31
5. Jesus is the “I am.” 4:26; 6:20, 35, 48, 51; 8:12, 18, 24, 28, 58; 9:5; 10:7,
9, 11, 14; 11:25; 13:19; 14:6; 15:1; 18:5–6 (cf. Ex.
3:14–15; Isa. 41:4; 43:10–13, 25; 45:18; 51:12;
52:6)
6. Jesus, the sent Son, reflects the sender. 3:17, 35–36; 5:19–26; 6:40; 8:35–36; 14:13; 17:1
7. Jesus is the fulfillment of Jewish 1:29, 36; 2:14–22, esp. v. 21; 4:23–24; 8:12; 9:5;
festivals and institutions (including the 19:14
temple).
8. Jesus is the giver of eternal life. 1:4; 3:15–16, 36; 4:14, 36; 5:24, 26, 39–40; 6:27,
33, 35, 40, 47–48, 51, 53–54, 68; 8:12; 10:10, 25,
28; 11:25; 12:25, 50; 14:6; 17:2–3; 20:31
9. The signs of Jesus show that he is the 2:1–11, 13–22; 4:46–54; 5:1–15; 6:1–15; 9:1–41;
Messiah (cf. also Jesus as the Messiah and 11:1–44
Son of God, above).
10. The witnesses to Jesus testify that he is 1:7–8, 15, 19, 32, 34; 3:11, 32–33; 4:39; 5:31–39;
the Messiah. 8:14, 18; 10:25; 15:26–27; 18:37; 19:35; 21:24
11. Father, Son, and Spirit are united in 14:17–18, 23, 26; 15:26; 20:21–22
their work of revelation and redemption.
12. Jesus' death is the basis of salvation. 1:29; 3:14–15; 6:51–58; 10:15; 11:50–52; 12:24;
15:13
13. God is sovereign in salvation. 3:21; 5:21; 6:37–45, 64–65; 10:16, 26–30; 15:16;
17:2, 6, 9
14. Salvation is obtained through 1:12; 3:15, 16; 5:24; 6:29, 35; 8:24; 11:25–27, 42;
believing in Jesus as the Messiah and the 12:44; 17:8, 21; 20:31
Son of God.
15. Believers can experience the benefits 3:18, 36; 4:23; 5:24; 6:39–40; 10:10, 26–29; 11:25–
of salvation already in the here and now, 26
during this present evil age.
16. Believers are called to continue Jesus' 4:38; 15:16; 17:18; 20:21–22
mission (cf. also Jesus as the sent Son,
above).

Distinctive Features

The Setting of John


The events of the Gospel of John take place in Palestine, incorporated
into the Roman Empire in 63 b.c. Appointed by the Romans as king
over the Jews in 37 b.c., Herod the Great ruled until his death in 4 b.c.
The Romans divided his kingdom among his descendants. The
predominantly Gentile region of the Decapolis, or “Ten Cities,” was a
loose confederation of semiautonomous cities administered by the
Roman legate of Syria.

History of Salvation Summary


Jesus comes as God in the flesh (1:14), the revealer of the Father

http://breachrepairers.webs.com/ 555
(14:9), and the messianic King (1:41, 49; 4:25; 6:15). He fulfills the OT and its symbols, especially its
promises of everlasting salvation. The ultimate fulfillment comes with his crucifixion and
resurrection.

Outline
1. Prologue: The Incarnate Word (1:1–18)
2. The Signs of the Messiah (1:19–12:50)
a. John the Baptist's witness and the first week of Jesus' ministry (1:19–2:11)
b. Jesus' ministry in Jerusalem, Judea, Samaria, and to Gentiles (2:12–4:54)
c. Mounting Jewish opposition, additional signs (5:1–10:42)
d. The final Passover: the ultimate sign and the aftermath (11:1–12:19)
e. The approaching Gentiles and the Messiah's rejection by the Jews (12:20–50)
3. The Farewell Discourse and the Passion Narrative (13:1–20:31)
a. The cleansing and instruction of the new messianic community and Jesus' final
prayer (13:1–17:26)
b. Jesus' arrest, trials, death, and burial (18:1–19:42)
c. Jesus' resurrection, appearances, and sending of his disciples (20:1–29)
d. Purpose statement: Jesus the Messiah, the Son of God (20:30–31)
4. Epilogue: The Roles of Peter and of the Disciple Whom Jesus Loved (21:1–25)

http://breachrepairers.webs.com/ 556
Chapter 1 – Identity of the Prophet of God
Reading
 Desire of Ages chapter 14.

Chapter Outline
 The Word, Life and Light (1-5)
 John the Baptist to bear witness (6-36)
 John's two disciples abide with Jesus (37-42)
 The calling of Philip (43-45)
 Nathanael (46-51)

The Word, Life and Light (1-5)


1:1 In the beginning was the Word, and the Word was with God, and the Word was God.

“beginning” – Means commencement

“the Word” – The meaning of the Word is “something said”, “motive”, “expression.” Therefore the
Word reveals to us the motive of God. It is also the expression of God. We can discern the motive
and expression of God through the Word.

What was from the beginning? The Word

What was with God? The Word

What was God? The Word In the Jewish mind, this was automatic. Other gods had images and all
different ways to visualize. But Jehovah, the God of Israel, all they could see was the shekina glory,
just the light. When people ask them about their God, they have never seen Him. All they knew
about Him was that He speaks. You must understand the power behind it. “My God speaks, and your
gods don’t. That distinguishes my God from all others.” That’s the reason why the book of Genesis
begins with “God said,” “God said,” “God said.”

Therefore, we know three things about the Word: It was in the beginning, It was with God, and It
was God. This establishes that there are at least two entities to the Godhead, God and the Word.

That is the outline of God. In the whole book of John, that’s what John does.

1:2 The same was in the beginning with God.

Who was with the Word from the beginning? God. Why is the verse basically a repeat of the first
verse? John really wants us to know that the Word and God were from the beginning and they were
with each other. That is the emphasis of the two verses so far. Why is it so important that the Word
was in the beginning with God?

http://breachrepairers.webs.com/ 557
John 15:27 And ye also shall bear witness, because ye have been with me from the beginning.

The Word was with God from the beginning. The Word bears witness of God. What is another word
for witness? Testify. Therefore, the Word testifies of God

Gen 1:1 In the beginning God created the heaven and the earth.

From the commencement of earth’s history, the Word and God existed. Both of them had a part to
play in creation. Therefore, when God spake:

Gen 1:3 And God said…

When we speak, it is an expression of what we feel inside. God was expressing Himself at creation.
Creation fully reflected what God felt inside. But the Word is also an expression. Therefore, the
Word and creation are an expression of God.

Heb 1:1-3 [1] God, who at sundry times and in divers manners spake in time past unto the fathers by
the prophets, Hath in these last days spoken unto us by [his] Son, whom he hath appointed heir of
all things, by whom also he made the worlds; Who being the brightness of [his] glory, and the
express image of his person, and upholding all things by the word of his power, when he had by
himself purged our sins, sat down on the right hand of the Majesty on high;

1:3 All things were made by him; and without him was not any thing made that was made.

“All things were made by him” – Everything was made by the Word

“without him was not any thing made that was made” – With the Word, nothing was made.
Creation would not have been possible without Him.

Why does John start his book like this? What is the purpose of the book?

John 20:30-31[30] And many other signs truly did Jesus in the presence of his disciples, which are
not written in this book: [31] But these are written, that ye might believe that Jesus is the Christ, the
Son of God; and that believing ye might have life through his name.

The purpose of the book is John is that in reading it we may believe that Jesus is the Christ, the Son
of God, and have life through His name. So why would John begin with the Creation by Jesus Christ?
(PO: If we understand that Christ is the Creator, we will be able to understand that Jesus is the Son
of God.)

(PO: Many people have asked how it is fair that one man Jesus Christ could die for us? The gospel of
John shows us why Jesus could be our substitute on the cross. Verse 4 tells us that in Jesus was life.
Christ life was un-borrowed therefore being God, He could lay down His life.)

Why could all things be made by him? Because in him (Jesus) was life.

1:4 In him was life; and the life was the light of men.

“In him” – Referring to the Word

“and the life was the light of men” – John will talk more about the life and the light in this book.

http://breachrepairers.webs.com/ 558
How that Word was life and light.

“Life” – light of men

1:5 And the light shineth in darkness; and the darkness comprehended it not.

When did light first shine in darkness?

Gen 1:3-5 [3] And God said, Let there be light: and there was light. And God saw the light, that [it
was] good: and God divided the light from the darkness. [4] And God called the light Day, and the
darkness he called Night. [5] And the evening and the morning were the first day.

Creation. How did it shine in darkness? God spoke and it appeared. How was the life the light of
men?

John the Baptist to bear witness (6-36)


1:6 ¶ There was a man sent from God, whose name was John.

Essentially what we see that the sole purpose of the ministry of John, is to testify of God,
contextually, the Word who has life, and provides the light of men. Studying a little deeper it almost
seems like John is sent to help the darkness comprehend the light, by testifying the the Word. So
contextually the prophet has this purpose. Deeper, why is John taking the time to say this? God
sends a man, to testify of another Man.... Role of a prophet: make the path to God clearer. Men have
rejected the Light of the Word so God sends the prophet to create the bridge... So why Has God
given us Ellen White???

1:7 The same came for a witness, to bear witness of the Light, that all men through him might believe.

1:8 He was not that Light, but was sent to bear witness of that Light.

1:9 That was the true Light, which lighteth every man that cometh into the world.

1:10 He was in the world, and the world was made by him, and the world knew him not.

1:11 He came unto his own, and his own received him not.

So Jesus came to the world which was made by Him, and the world knew Him now, but not only
that His own received Him not. So from this verse you can see that The role of John the Baptist was
important because he played a part in clearing up the misconceptions that people had about the
role of the Messiah, also notice verse 12.

1:12 But as many as received him, to them gave he power to become the sons of God, even to them that
believe on his name:

“But” gives us the idea of a change or contrast. It seems that “His own” received Him not because
they knew Him not. However a transition takes place. The Word is giving power to become sons of
God, even to those that believe on His name. Now according to this verse how would we define that
power that God gave to those who received Him? The fact that the Word was made flesh in verse
14. We can conclude that this manifestation was needed because the world “knew Him not.” So the
incarnation of the Word atleast does two things, provides power and also helps the world to know
the World.

http://breachrepairers.webs.com/ 559
“sons of God” – What is the definition of a son of God according to these verses? being born, not of
blood, nor of the will of the flesh, nor of the will of man, but of God. It also means to believe on the
name of the Word. Conclusion: It is by the Word that we are born of God, the manifestation of the
Word being made flesh, provides power for us to become Sons of God and as we understand the
Word made flesh we will receive Him, this sounds a lot like 1 John 3 doesn’t it?

1:13 Which were born, not of blood, nor of the will of the flesh, nor of the will of man, but of God.

1:14 And the Word was made flesh, and dwelt among us, (and we beheld his glory, the glory as of the
only begotten of the Father), full of grace and truth.

1:15 ¶ John bare witness of him, and cried, saying, This was he of whom I spake, He that cometh after me
is preferred before me: for he was before me.

1:16 And of his fulness have all we received, and grace for grace.

1:17 For the law was given by Moses, but grace and truth came by Jesus Christ.

1:18 No man hath seen God at any time; the only begotten Son, which is in the bosom of the Father, he
hath declared him.

1:19 ¶ And this is the record of John, when the Jews sent priests and Levites from Jerusalem to ask him,
Who art thou?

Why did they go to John? They wanted to find out who they should be working for:

DA 133 Already their power over the people was waning. It was becoming a serious question how to
maintain their position. In the hope of arriving at some conclusion, they dispatched to the Jordan a
deputation of priests and Levites to confer with the new teacher.

Characteristics of the Sanhedrin


 Made up from the priesthood
 There were 71 people in the council
 They were all to be advanced in years – though not aged (Paul, Nicodemus)
 They had to be married and to have children – to show that they had more compassion,
more humane
 Men of learning – religion, history and general knowledge

DA 133 It was well known that the seventy weeks of Daniel's prophecy, covering the Messiah's
advent, were nearly ended; and all were eager to share in that era of national glory which was then
expected.

There were many false Christ’s at that time and example is Barabbas. Why couldn’t the people see
that the prophecy was going to be fulfilled? – pride and envy. In the same way, we can’t see today
how fast the events can pass

1:20 And he confessed, and denied not; but confessed, I am not the Christ.

DA 135 The words "that prophet" had reference to Moses. The Jews had been inclined to the belief

http://breachrepairers.webs.com/ 560
that Moses would be raised from the dead, and taken to heaven. They did not know that he had
already been raised.

The Jews expected a literal fulfillment of the prophecy.

DA 135 Jesus afterward said, referring to John, "If ye are willing to receive it, this is Elijah, which is to
come." Matt. 11:14, R. V. John came in the spirit and power of Elijah, to do such a work as Elijah did. If
the Jews had received him, it would have been accomplished for them. But they did not receive his
message. To them he was not Elijah. He could not fulfill for them the mission he came to accomplish.

If they had accepted his message, he to them would have been Elijah – turn the hearts of the
children to the father and vice versa. Back then many people did not turn their hearts back to their
heavenly Father. John was Elijah, but the people did not believe that He was the Elijah. But in
Malachi the greater fulfillment of the prophecy was to happen just before Jesus came again – the
SDA church

1:21 And they asked him, What then? Art thou Elias? And he saith, I am not. Art thou that prophet? And
he answered, No.

1:22 Then said they unto him, Who art thou? that we may give an answer to them that sent us. What
sayest thou of thyself?

1:23 He said, I am the voice of one crying in the wilderness, Make straight the way of the Lord, as said the
prophet Esaias.

Is 40:3

What is the literal interpretation of this verse?

“in the desert” – he preached in the wilderness. He was making a “spiritual” making the hills and
mountains low and the valleys will be brought up.

“Mountains” – pride

“Valleys” – humility and self-sacrificing

1:24 And they which were sent were of the Pharisees.

1:25 And they asked him, and said unto him, Why baptizest thou then, if thou be not that Christ, nor
Elias, neither that prophet?

1:26 John answered them, saying, I baptize with water: but there standeth one among you, whom ye
know not;

1:27 He it is, who coming after me is preferred before me, whose shoe's latchet I am not worthy to
unloose.

1:28 These things were done in Bethabara beyond Jordan, where John was baptizing.

1:29 ¶ The next day John seeth Jesus coming unto him, and saith, Behold the Lamb of God, which taketh
away the sin of the world.

1:30 This is he of whom I said, After me cometh a man which is preferred before me: for he was before

http://breachrepairers.webs.com/ 561
me.

1:31 And I knew him not: but that he should be made manifest to Israel, therefore am I come baptizing
with water.

1:32 And John bare record, saying, I saw the Spirit descending from heaven like a dove, and it abode
upon him.

1:33 And I knew him not: but he that sent me to baptize with water, the same said unto me, Upon whom
thou shalt see the Spirit descending, and remaining on him, the same is he which baptizeth with the Holy
Ghost.

1:34 And I saw, and bare record that this is the Son of God.

1:35 ¶ Again the next day after John stood, and two of his disciples;

1:36 And looking upon Jesus as he walked, he saith, Behold the Lamb of God!

John's two disciples abide with Jesus (37-42)


1:37 And the two disciples heard him speak, and they followed Jesus.

1:38 Then Jesus turned, and saw them following, and saith unto them, What seek ye? They said unto him,
Rabbi, (which is to say, being interpreted, Master), where dwellest thou?

“where dwellest thou” – These 2 disciples are Andrew and John. They asked Jesus “Where
dwellest thou?” Why? They wanted to sit at His feet, to spend some quality time with Him. In every
group of people you meet, there are always those whom we call seekers. John and Andrew
represented that group.

1:39 He saith unto them, Come and see. They came and saw where he dwelt, and abode with him that
day: for it was about the tenth hour.

“Come and see” – Could we say to those seekers “come and see” to allow them to taste of Jesus. To
represent Jesus. If you come and see, you will see Jesus in me – how many of us can say that
confidently, that we can represent Jesus. They would see what Jesus eats, how He lives, etc…We
must be transparent with one another – so transparent that they are able to see Jesus in us.

1:40 One of the two which heard John speak, and followed him, was Andrew, Simon Peter's brother.

1:41 He first findeth his own brother Simon, and saith unto him, We have found the Messias, which is,
being interpreted, the Christ.

In order for Jesus to be Christ, He must be the anointed One – at His Baptism – AD 27. Until Messiah
the Prince – the anointed One.

1:42 And he brought him to Jesus. And when Jesus beheld him, he said, Thou art Simon the son of Jona:
thou shalt be called Cephas, which is by interpretation, A stone.

Everybody will have stars in their crowns – working for the salvation of souls. Andrew went in
search of Simon. He went and searched for his family first. Family is the first area of missionary
work. A change of name signifies a change in character. Simon means a stone and Cephas means a
stone .

http://breachrepairers.webs.com/ 562
The calling of Philip (43-45)
1:43 ¶ The day following Jesus would go forth into Galilee, and findeth Philip, and saith unto him, Follow
me.

1:44 Now Philip was of Bethsaida, the city of Andrew and Peter.

1:45 Philip findeth Nathanael, and saith unto him, We have found him, of whom Moses in the law, and
the prophets, did write, Jesus of Nazareth, the son of Joseph.

Nathanael (46-51)
1:46 And Nathanael said unto him, Can there any good thing come out of Nazareth? Philip saith unto
him, Come and see.

“Can any good thing come out of Nazareth?” – It was notorious for being wicked. Does our
environment necessarily dictate how we will be? – No Jesus grew up in one of the most wicked
areas, yet He was still the Son of God.

1:47 Jesus saw Nathanael coming to him, and saith of him, Behold an Israelite indeed, in whom is no
guile!

“Jesus saw Nathanael coming to him, and saith of him, Behold an Israelite indeed, in whom is
no guile!” – This man was actively meditating and praying. God will reveal truth to those who are
seeking and studying.

1:48 Nathanael saith unto him, Whence knowest thou me? Jesus answered and said unto him, Before
that Philip called thee, when thou wast under the fig tree, I saw thee.

1:49 Nathanael answered and saith unto him, Rabbi, thou art the Son of God; thou art the King of Israel.

1:50 Jesus answered and said unto him, Because I said unto thee, I saw thee under the fig tree, believest
thou? thou shalt see greater things than these.

1:51 And he saith unto him, Verily, verily, I say unto you, Hereafter ye shall see heaven open, and the
angels of God ascending and descending upon the Son of man.

What did Jesus mean about Jacob’s ladder?

DA 142 Here Christ virtually says, On the bank of the Jordan the heavens were opened, and the Spirit
descended like a dove upon Me. That scene was but a token that I am the Son of God. If you believe on
Me as such, your faith shall be quickened. You shall see that the heavens are opened, and are never to
be closed. I have opened them to you. The angels of God are ascending, bearing the prayers of the
needy and distressed to the Father above, and descending, bringing blessing and hope, courage, help,
and life, to the children of men.

http://breachrepairers.webs.com/ 563
Chapter 2 – The Miracle at Cana | Cleansing of the Temple
The wedding feast at Cana

Chapter Outline
 Jesus’ first miracle – the marriage at Cana (1-11)
 Jesus, His mother, brethren and disciples go down to Capernaum (12-13)
 Jesus cleanses the temple | Beginning of Jesus ministry (14-17)
 The Jews asks for a sign and Jesus speaks of the temple of His body (18-22)
 Jesus knows what is in man (23-25)

Jesus’ first miracle – the marriage at Cana (1-11)


2:1 And the third day there was a marriage in Cana of Galilee; and the mother of Jesus was there:

These wedding feasts were for more than one day.

2:2 And both Jesus was called, and his disciples, to the marriage.

Why was Jesus called? He was a relative. It was a little village – Cana. Jesus came to the marriage –
what does that mean? He sanctions gatherings of social enjoyment. Jesus was not a hermit, he didn’t
condemn it. It shows that He also advocates marriage.

2:3 And when they wanted wine, the mother of Jesus saith unto him, They have no wine.

Why did Jesus’ mother, Mary, turn to Jesus to tell Him there was no wine? How could He have
helped? She wanted Him to help with the situation of the wine. He could have helped by just
performing a miracle.

What did Jesus’ mother really want Jesus to do in telling him this situation? She wanted Him
to work this miracle to prove to others that He was the Son of God. She wanted others to know
about who her son truly was. Something that she had probably kept a secret for the past 30 years of
her life since His birth (Luke 2:15-19)

What do we understand about Mary’s thoughts and feelings about her Son and what she
understood about who Jesus really was and His mission? She was just like any mother today,
proud of her own Son and who He really was. Imagine what sort of position she would be elevated
to if people knew that she was the mother to the Son of God, or at least someone who could work
powerful miracles in healing and turning water into wine. This showed that she, even as a mother,
still didn’t fully comprehend Christ’s mission and why He came to this earth. She thought He was
going to reign as king on the earth like everyone else.

2:4 Jesus saith unto her, Woman, what have I to do with thee? mine hour is not yet come.

“Woman” – Jesus’ reply in just simple saying “Woman” was not a sign of disrespect but was of
custom. Jesus was still honoring His mother there. Mary set it up so that God could work a miracle.
God would never do for us what we could do ourselves. We have a part to play.

http://breachrepairers.webs.com/ 564
In Jesus’ reply to His mother what important lesson can we learn as parents or future-
parents or even as the child?

Luke 2:48-49

Eph. 6:1

As parents, even if we know the power that our children may have, we must not use it for our own
selfish purposes, and we must not turn the children’s minds to our own but always into the
submission of God. The claims of God are above ties of human relationship. No earthly attraction
should turn our feet from the path in which He bids us walk. Luke 2:48-49 – Jesus was about His
Heavenly Father’s business. Even from a young age He was learning to do the will of God. Eph. 6:1 –
as children we must obey our parents IN THE LORD. God is still the first. If our parents speak
contrary to what the will of God is, we must follow God’s will.

In Jesus’ reply to His mother, what does this help us to understand about Christ’s mission
here on this earth?

Rom. 5:6

Every part of Christ’s life and mission here on this earth had an appointed time. His life was a
fulfillment to a set time. Rom. 5:6 – at the appointed time, Christ died for our sins.

2:5 His mother saith unto the servants, Whatsoever he saith unto you, do it.

“Whatsoever he saith unto you, do it” – especially in regard to diet, etc…

2:6 And there were set there six waterpots of stone, after the manner of the purifying of the Jews,
containing two or three firkins apiece.

The firkins can show that Jesus did not make fermented wine. If that was so, chances are that
people would pass out, etc…

2:7 Jesus saith unto them, Fill the waterpots with water. And they filled them up to the brim.

According to the Bible, what does the water represent?

John 2:6-8

John 3:5

Rom. 6:3

The water represents the baptism into His death

What does the wine represent?

Luke 22:17-18, 20

Heb 9:22

The wine represents His blood which He shed for the remission of sins.

http://breachrepairers.webs.com/ 565
What was the main ingredient that turned the water into wine?

Ps 33:6, 9

It was the word of Jesus. The word of God has creative power. When Jesus speaks, it happens. Who
was it that bore the waterpots to the feast and what important lessons can we learn from
this?

James 2:20

It was the servants who bore the waterpots to the feast. It is by human hands that the blessing of
the wine (the blood of Christ) can be taken to the whole world. The word may have all the life giving
power in it, but it is through our faith that works that other may be blessed and nourished. We have
a responsibility to bring to the world those waterpots full of Christ’s blood and sacrifice.

How do we know that this wine was just pure grape juice and not the fermented wine which
we think of today? (Clue: 6 waterpots, 2 or 3 firkins a piece, approx. Eastern wedding usually
has around 300 guests)?

1 firkin = 11 gallons/41 liters. Minimum that 6 waterpots can hold is 2 firkins. Therefore total = 12
firkins = 12 x 11 = 132 gallons. Therefore, each person would have drunk about ½ gallon just from
what Jesus made. Maximum that 6 waterpots can hold is 3 firkins. Therefore total = 18 firkins = 18 x
11 = 198 gallons. Therefore, each person would have drunk about 2/3 gallon just from what Jesus
made.

Keep in mind that the people had already “well drunk” and this wine was coming in at the tail end of
the wedding. If this was fermented wine, it probably would have stoned a lot of people there and
hurt them internally, if not killing them. Why would Jesus create something that would be so
harmful to so many people? If there are still more questions, take the rest offline and do a study
with them on wine in the future.

2:8 And he saith unto them, Draw out now, and bear unto the governor of the feast. And they bare it.

2:9 When the ruler of the feast had tasted the water that was made wine, and knew not whence it was:
2:but the servants which drew the water knew; the governor of the feast called the bridegroom,

2:10 And saith unto him, Every man at the beginning doth set forth good wine; and when men have well
drunk, then that which is worse: but thou hast kept the good wine until now.

In this world today, the normal sequence of events is that it is good usually. But it goes from good to
worse. The principle that it teaches us that in the Christian life what starts out good will get better,
not worse. Jesus turned water into wine. Moses turned water into blood.

DA 145 As the guests assemble, many seem to be preoccupied with some topic of absorbing interest.
A suppressed excitement pervades the company. Little groups converse together in eager but quiet
tones, and wondering glances are turned upon the Son of Mary. As Mary had heard the disciples'
testimony in regard to Jesus, she had been gladdened with the assurance that her long-cherished
hopes were not in vain. Yet she would have been more than human if there had not mingled with this
holy joy a trace of the fond mother's natural pride. As she saw the many glances bent upon Jesus, she
longed to have Him prove to the company that He was really the Honored of God. She hoped there

http://breachrepairers.webs.com/ 566
might be opportunity for Him to work a miracle before them.

Mary was no different when she saw Jesus. She had the same feelings that a natural mother
would have. Yet the Catholics exalt her like a God. It showed her humanity not seen
anywhere else.

DA 148 As men set forth the best wine first, then afterward that which is worse, so does the world
with its gifts. That which it offers may please the eye and fascinate the senses, but it proves to be
unsatisfying. The wine turns to bitterness, the gaiety to gloom. That which was begun with songs and
mirth ends in weariness and disgust. But the gifts of Jesus are ever fresh and new. The feast that He
provides for the soul never fails to give satisfaction and joy. Each new gift increases the capacity of
the receiver to appreciate and enjoy the blessings of the Lord. He gives grace for grace. There can be
no failure of supply. If you abide in Him, the fact that you receive a rich gift today insures the
reception of a richer gift tomorrow.

What important lesson can we learn from verse 10 in its application to us today?

In this world, the best things are often put first to attract our senses and at the tail end, the worst
comes. But in the things of Christ, everything is ever fresh and new and it gets better with every
new thing that comes from Christ (John 1:50). It never fails to provide satisfaction and joy

2:11 This beginning of miracles did Jesus in Cana of Galilee, and manifested forth his glory; and his
disciples believed on him.

In summary, what spiritual application can we draw from the water, the wine and the
wedding feast? It is only through Jesus’ baptism into death (the water) and his blood (the wine)
that we can have an opportunity to enter into the wedding feast in heaven.

Jesus, His mother, brethren and disciples go down to Capernaum (12-13)


2:12 ¶ After this he went down to Capernaum, he, and his mother, and his brethren, and his disciples:
and they continued there not many days.

2:13 ¶ And the Jews' passover was at hand, and Jesus went up to Jerusalem,

The beginning of Jesus’ ministry (14-17)


2:14 And found in the temple those that sold oxen and sheep and doves, and the changers of money
sitting:

What 3 things does the temple symbolize in the Bible? The sanctuary, symbol of our body, or
represents the church. You could not buy anything in the temple unless they used the temple
currency

DA 155 The dealers demanded exorbitant prices for the animals sold, and they shared their profits
with the priests and rulers, who thus enriched themselves at the expense of the people.

We see here the combination of church and state. This is resulting in extortion. Application: We can
see these things happening today – getting money together.

DA 157 There came to this feast those who were suffering, those who were in want and distress. The
blind, the lame, the deaf, were there. Some were brought on beds. Many came who were too poor to

http://breachrepairers.webs.com/ 567
purchase the humblest offering for the Lord, too poor even to buy food with which to satisfy their
own hunger.

God designed it that everyone, no matter how poor you were, could bring offerings to God. But the
priests made it so that expensive that the poor were left out.

2:15 And when he had made a scourge of small cords, he drove them all out of the temple, and the sheep,
and the oxen; and poured out the changers' money, and overthrew the tables;

Is it wrong to be angry?

Eph 4:26 Be ye angry, and sin not: let not the sun go down upon your wrath:

There is a time to be angry – when there is extortion, unjustness, etc. It is called righteous
indignation. As Christ starts His ministry, Christ had to cleanse the temple. What is the spiritual
application?

DA 161 No man can of himself cast out the evil throng that have taken possession of the heart. Only
Christ can cleanse the soul temple. But He will not force an entrance.

When a person decides to accept Christ, the body has to be cleansed as well – physically and
spiritually. It is so important to be cleansed physically because God speaks to our mind. If our mind
is not healthy, then God cannot communicate to us because our mind will not be clear.

2:16 And said unto them that sold doves, Take these things hence; make not my Father's house an house
of merchandise.

2:17 And his disciples remembered that it was written, The zeal of thine house hath eaten me up.

“The zeal of thine house hath eaten me up” – what does it mean? The zeal to do what God wants
has overtaken Jesus.

DA 158 Looking upon Christ, they behold divinity flash through the garb of humanity.

Before Christ passes away, He cleanses the temple again.

The Jews asks for a sign and Jesus speaks of the temple of His body (18-
22)
2:18 ¶ Then answered the Jews and said unto him, What sign shewest thou unto us, seeing that thou
doest these things?

2:19 Jesus answered and said unto them, Destroy this temple, and in three days I will raise it up.

2:20 Then said the Jews, Forty and six years was this temple in building, and wilt thou rear it up in three
days?

2:21 But he spake of the temple of his body.

Verse 21 is one way that we can show that the temple symbolizes the body

2:22 When therefore he was risen from the dead, his disciples remembered that he had said this unto
them; and they believed the scripture, and the word which Jesus had said.

http://breachrepairers.webs.com/ 568
Jesus knows what is in man (23-25)
2:23 ¶ Now when he was in Jerusalem at the passover, in the feast day, many believed in his name, when
they saw the miracles which he did.

2:24 But Jesus did not commit himself unto them, because he knew all men,

“did not commit himself unto them” – who is them? The “them” is referring to those that had
seen and believed in the miracles of Jesus. The same group that cried crucify Him were the same
ones that were present there that Jesus was referring to.

2:25 And needed not that any should testify of man: for he knew what was in man.

http://breachrepairers.webs.com/ 569
Chapter 3 – Nicodemus | Ministry of John (Conditions of
Salvation-In reach)
Chapter Outline
 Jesus’ interview with Nicodemus (1-21)
 Jesus and His disciples go into the land of Judea and baptize (22)
 John the Baptist is baptizing in Aenon (23-24)
 John testifies of Jesus again – He must increase, I must decrease (25-36)
The importance of this chapter

DA 176 In the interview with Nicodemus, Jesus unfolded the plan of salvation, and His mission to the
world. In none of His subsequent discourses did He explain so fully, step by step, the work necessary
to be done in the hearts of all who would inherit the kingdom of heaven.

Introduction of Nicodemus

DA 167 Nicodemus held a high position of trust in the Jewish nation. He was highly educated, and
possessed talents of no ordinary character, and he was an honored member of the national council.
Though rich, learned, and honored, he had been strangely attracted by the humble Nazarene.

He was a member of the Sanhedrin. EGW says that he has talents of no ordinary character. Even the
Sanhedrin would probably have looked at him as a bright man.

DA 176 For a time Nicodemus did not publicly acknowledge Christ, but he watched His life, and
pondered His teachings. In the Sanhedrin council he repeatedly thwarted the schemes of the priests
to destroy Him. When at last Jesus was lifted up on the cross, Nicodemus remembered the teaching
upon Olivet: "As Moses lifted up the serpent in the wilderness, even so must the Son of man be lifted
up: that whosoever believeth in Him should not perish, but have eternal life." The light from that
secret interview illumined the cross upon Calvary, and Nicodemus saw in Jesus the world's
Redeemer.

DA 177 After the Lord's ascension, when the disciples were scattered by persecution, Nicodemus
came boldly to the front…He was scorned and persecuted by those who had paid him reverence in
other days. He became poor in this world's goods; yet he faltered not in the faith which had its
beginning in that night conference with Jesus.

Jesus’ interview with Nicodemus (1-21)


3:1 There was a man of the Pharisees, named Nicodemus, a ruler of the Jews:

Who is Nicodemus?

John 7:50-52

John 19:39

A pharisee. Who is a Pharisee? Get them to discuss and give their opinion. A member of the Jewish
sect noted for strict obedience to Jewish traditions

3:2 The same came to Jesus by night, and said unto him, Rabbi, we know that thou art a teacher come

http://breachrepairers.webs.com/ 570
from God: for no man can do these miracles that thou doest, except God be with him.

When did Nicodemus come to Jesus? And who do you think he represents?

John 3:10, 2

Nicodemus came to Jesus by night. Why did he go to Jesus by night? What did it indicate about his
character? He didn’t want people to follow his example in going to Jesus. So he was a very cautious
man. Thought things out well. Why? He was scared to let anyone know he was seeing Jesus.
Nicodemus represents those who grew up in the church - they know the scriptures and the
traditions (eg. Going to church on Sabbath) but they don’t know Christ. Nicodemus was ashamed to
be seen with Christ – those who grow up in church tend to be ashamed of association with Christs.

“Rabbi” – Calling Jesus Rabbi seemed like it was an expression of faith. It sounded like flattery, but
really it was an expression of doubt.

3:3 Jesus answered and said unto him, Verily, verily, I say unto thee, Except a man be born again, he can
not see the kingdom of God.

“Verily, verily” – when Jesus says that, it is always important.

Matt 18:3, 4 – very similar

We see here that righteousness by faith and humility are very closely related. This is related to
being born again as well. Jesus used a statement that was almost humiliating to him. Jesus didn’t
pull back but plainly told him that he wasn’t ready for the kingdom of Heaven.

Application: We take a different approach to those who don’t know the truth. But to those who
know the truth, we must be more direct and take a more direct approach. If you don’t have a love
for that person when you are rebuking them, it will show through. You are just fighting with Satan’s
weapon because it is carnal. But you need to win their trust as well. You can’t be so straight forward
that the words of truth have no place in their heart.

What does it mean to be born again? Literally go through a baby being born. If you are born again
as a baby, what does that mean? New and young. Do not remember past anymore, clean mind

3:4 Nicodemus saith unto him, How can a man be born when he is old? can he enter the second time into
his mother's womb, and be born?

Nicodemus did something very similar to the woman at the well:

“How can a man be born when he is old? can he enter the second time into his mother's
womb, and be born?” – He tried to avoid something that he knew was hitting close to his heart. He
was also applying what Jesus was saying literally, not spiritually. It is addressed in verse 12:

John 3:12 If I have told you earthly things, and ye believe not, how shall ye believe, if I tell you [of]
heavenly things?

It showed that he was taking things literally, and not spiritually. Sometimes we can listen to a
sermon and it may not mean anything to us. Maybe it might be possible that we aren’t spiritually
discerning things.

http://breachrepairers.webs.com/ 571
3:5 Jesus answered, Verily, verily, I say unto thee, Except a man be born of water and of the Spirit, he can
not enter into the kingdom of God.

What do we need to be born again with?

John 3:4-5

The spirit and the water.

What does it mean to be born of water? Baptism – this shows that it is not enough just to be born
by the water (baptism) but we also need to be born by the Spirit. Too many Christians are just born
with water, just like Nicodemus. But they are not born of the Spirit

What does it mean to be born of the spirit?

Eph 6:17

1 Pet 1:23

The Holy Spirit works through the word of God, and we must be born of the word.

“be born of the water and spirit” – What does the spirit indicate? Renewing of the heart by the
Spirit of God.

3:6 That which is born of the flesh is flesh; and that which is born of the Spirit is spirit.

“born of the flesh” – what does it mean? Referring to humanity and also carnality. So when we are
born we are born sinful, not born in sin. To be born not of the flesh, we must be born of the spirit.
Repentance and baptism, then followed by the renewing of the mind by the Holy Spirit

3:7 Marvel not that I said unto thee, Ye must be born again.

People change differently, some slowly over time while others instantly.

3:8 The wind bloweth where it listeth, and thou hearest the sound thereof, but canst not tell whence it
cometh, and whither it goeth: so is every one that is born of the Spirit.

What does Jesus liken the Holy Spirit to and why? The wind. Discuss – let them talk abut it. But
guide it along these following guidelines. We can’t see the wind but we can see the effects of it. We
may not see the Holy Spirit, but we can see the fruits of the converted person. They testify. This is
describing conversion process. We can’t tell exactly every working of the Holy Spirit but we see its
effects in peoples lives. The agency of change comes by the word of God. Sometimes people’s
conversion process takes a long time. Some are sudden. We can’t always pin point it to one exact
event, but we can see the gradual change.

3:9 Nicodemus answered and said unto him, How can these things be?

3:10 Jesus answered and said unto him, Art thou a master of Israel, and knowest not these things?

How can we be born again? By the word of God

http://breachrepairers.webs.com/ 572
1 Pet 1:23

How does reading the Bible lead to repentance? We realize the love of God. It contains the law of
God which points out our sin. When we see the love of God, we feel the need to be born again.

3:11 Verily, verily, I say unto thee, We speak that we do know, and testify that we have seen; and ye
receive not our witness.

3:12 If I have told you earthly things, and ye believe not, how shall ye believe, if I tell you of heavenly
things?

3:13 And no man hath ascended up to heaven, but he that came down from heaven, even the Son of man
which is in heaven.

3:14 ¶ And as Moses lifted up the serpent in the wilderness, even so must the Son of man be lifted up:

3:15 That whosoever believeth in him should not perish, but have eternal life.

3:16 ¶ For God so loved the world, that he gave his only begotten Son, that whosoever believeth in him
should not perish, but have everlasting life.

Why is Jesus called only-begotten Son? It makes you think that Christ is offspring of the father? If so,
who is his mother? then the start quoting Proverbs chapter 8, where it says Jesus came out from the
father. But we don’t have time to deal with that today.

Begotten, biologically you are the offspring of your parents. But we cannot take this word
biologically, God is beyond biology and the confinements of our human flesh. Thus the word
begotten cannot be used from a biological standpoint, but rather from the spiritual. Thus it has
another meaning than expressing offspring. Thus according to the Jewish mind if you say you are
begotten from Joseph you are saying you receive his inheritance. Matthew 1 uses the word begat
numerous times. That genealogy is not only to show the biological line of Jesus but his heritage, that
the inheritance of the promise given to Abraham was passed on from father to son.

Begotten means more than offspring here; it is talking about inheritance. Thus Jesus receives all the
inheritance from the father. Thus when God gave his only begotten son, He gave all His inheritance.
What is involved in this inheritance? The glory of God. The ways only begotten is used in the Bible:

John 1:14 And the Word was made flesh, and dwelt among us, (and we beheld his glory, the glory as
of the only begotten of the Father,) full of grace and truth.

This verse describes Jesus as the word, which is an expression of the father. At the same time he is
called the only begotten. Thus Jesus is the only expression of the inheritance of glory of the father.
He is the only one who can truly clearly express who the Father is. The Holy Spirit expresses the
Son. All three Gods are very humble. Father says, look at Son; the Son, look at Father; the Spirit, look
at the Son. Only begotten means Jesus is the only one who can express the father; in that sense he is
unique.

John 1:18 No man hath seen God at any time; the only begotten Son, which is in the bosom of the
Father, he hath declared [him].

The bible is giving the idea that no one has seen the father and that Jesus is the only one who can
declare him. Again it is saying that Jesus is the only expression of the glory of God.

http://breachrepairers.webs.com/ 573
2 Cor 4:6 For God, who commanded the light to shine out of darkness, hath shined in our hearts, to
[give] the light of the knowledge of the glory of God in the face of Jesus Christ.

Col 1:15 Who is the image of the invisible God, the firstborn of every creature:

Another bible text says that Jesus is the expression of God.

Heb 1:3 Who being the brightness of [his] glory, and the express image of his person, and upholding
all things by the word of his power, when he had by himself purged our sins, sat down on the right
hand of the Majesty on high;

“who being” – present tense. The word only begotten is describing that Jesus is unique.

Heb 11:17 By faith Abraham, when he was tried, offered up Isaac: and he that had received the
promises offered up his only begotten [son].

Abraham had two sons, but Isaac was unique because he had the promises. Jesus is unique because
he is the only who can express God’s image. God is the thought and He is the word. the Holy Spirit is
the one who makes the word reality. Before Jesus came to this world, what did He look like?
Nebuchadnezzar was able to tell that He was the Son of God, based on the characteristics. He called
Him Son of God. Though in Daniel 7 and 10, Jesus is called Son of man. When Jesus was only the Son
of God, not son of man, he had his only bodily image to express God’s character. He also had His
glory of God to express God’s character.

Phil 2:5-8 [5] Let this mind be in you, which was also in Christ Jesus: [6] Who, being in the form of
God, (Jesus was the only expression of the Father in the form of God) thought it not robbery to be
equal with God: [7] But made himself of no reputation, and took upon him the form of a servant, and
was made in the likeness of men: [8] And being found in fashion as a man, he humbled himself, and
became obedient unto death, even the death of the cross.

When Jesus became a man, he gave up the form of God. Before Jesus came to this world, he was the
express image of the father, the only expression of the father in the form of God. When he came to
this world, he was the only expression of God’s image or glory in the form of man. He is the only one
in the entire universe who can express who the father is in the form of man, not in the form of God
anymore. When he became man he gave that up.

John 3:16 …For God so loved the world, that he gave…

The father gave up the possibility to express himself in his son in the form of God forever. When
Jesus went back to heaven, when he was resurrected, he didn’t take the form of God but retained
the form of man and will keep that forever. He is no longer has the form of God any more. Forever
he became human. That was one of the big sacrifices that the Father made. If Jesus would commit a
sin, than the father forever, in a sense, lost the only expression of the father. When Jesus was
resurrected, he is now called the first begotten of the dead.

Rev 1:5 And from Jesus Christ, [who is] the faithful witness, [and] the first begotten of the dead.

Heb 1:5-6 [5] For unto which of the angels said he at any time, Thou art my Son, this day have I
begotten thee? And again, I will be to him a Father, and he shall be to me a Son? [6] And again, when
he bringeth in the firstbegotten into the world, he saith, And let all the angels of God worship him.

http://breachrepairers.webs.com/ 574
This is talking about after his resurrection, Acts 13:33. The father said that this day he has begotten
him; then he said that he brought the first begotten. When did this happen.

Acts 13:33 God hath fulfilled the same unto us their children, in that he hath raised up Jesus again; as
it is also written in the second psalm, Thou art my Son, this day have I begotten thee.

John 3:16 that God gave his only begotten; thus he was already begotten and God’s son; but here it
says Jesus was begotten at his resurrection.

When Jesus came to this world, he took upon him the form of man. If he wanted to, he could have
gone back to heaven. Even in Gethsemane, he could have gone back to his throne as how things
were. But when he decided to die for man, it confirmed his decision to be one of us, never to change.
When he died as a sinner, he forever identified himself as us. Because when you die, you cannot
change. He could have gone back to take his form of God, but when he died it was forever done. So
when Jesus was resurrected to the father, he now had a new son, in the form of man forever. When
he died and resurrected, the father said, you are my son, and are in the form of human. God says
that he was the first begotten of the dead. Though Moses was resurrected, without Jesus’
resurrection, no one else could be resurrected. If I say I have only son; I have one. If I say this is first
son, I have more. Now he is called first begotten, meaning there can be more. Thus, whoever dies
and resurrects like Jesus, and has immortality can be 2nd, 3rd, etc begotten of the father. Now God
has many sons to express his character in human form.

The ultimate inheritance is when you receive immortality, but your sins have to be blotted out to
receive this, but for your sins to be blotted out you must live by faith as though you are sons of God.
This is what Jesus means when he says, you must be born again. Romans 6 says that we are buried
with him by baptism and we are raised with him.

1 Pet 1:4 To an inheritance incorruptible, and undefiled, and that fadeth not away, reserved in
heaven for you,

Immortality is reserved in heaven for you

Rom 8:14-17 [14] For as many as are led by the Spirit of God, they are the sons of God. [15] For ye
have not received the spirit of bondage again to fear; but ye have received the Spirit of adoption,
whereby we cry, Abba, Father. [16] The Spirit itself beareth witness with our spirit, that we are the
children of God: [17] And if children, then heirs; heirs of God, and joint-heirs with Christ; if so be that
we suffer with [him], that we may be also glorified together.

Because Jesus retains his human form forever, when we receive immortality in human form, we are
exactly he same as Jesus. Now the father has many sons and daughters.

John 12:24 Verily, verily, I say unto you, Except a corn of wheat fall into the ground and die, it
abideth alone: but if it die, it bringeth forth much fruit.

When Jesus died, he brought forth much fruit. One seed is given up but the fruit itself has many
seeds that look just like the original. We will look just like Jesus, then Jesus will come with the
sickle. We are joint heirs with Christ. We are considered in the universe as sons of God; we are
much higher than Adam and Eve.

Rom 8:29 For whom he did foreknow, he also did predestinate [to be] conformed to the image of his
Son, that he might be the firstborn among many brethren.

http://breachrepairers.webs.com/ 575
1 Cor15:20-23 [20] But now is Christ risen from the dead, [and] become the firstfruits of them that
slept. [21] For since by man [came] death, by man [came] also the resurrection of the dead. [22] For
as in Adam all die, even so in Christ shall all be made alive. [23] But every man in his own order:
Christ the firstfruits; afterward they that are Christ's at his coming.

Jesus is the first fruits

1 John 3:1-2 [1] Behold, what manner of love the Father hath bestowed upon us, that we should be
called the sons of God: therefore the world knoweth us not, because it knew him not. [2] Beloved,
now are we the sons of God, and it doth not yet appear what we shall be: but we know that, when he
shall appear, we shall be like him; for we shall see him as he is.

In order to see God and not die, you have to have immortality….we don’t have immortality now, but
we can be called sons of God in character.

1 John 3:3 And every man that hath this hope in him purifieth himself, even as he is pure.
In order to receive immortality, we purify ourselves, but in the process of that we can be called sons
of God by faith, because we don’t look like him. We live for this hope.

Rom 4:25 Who was delivered for our offences, and was raised again for our justification.

Rom 5:1-2 [1] Therefore being justified by faith, we have peace with God through our Lord Jesus
Christ: [2] By whom also we have access by faith into this grace wherein we stand, and rejoice in
hope of the glory of God.

When you are justified by faith, The son of God became a son of man so that sons of man can
become sons of God. God gave his only begotten son so that in the end he will have many sons unto
glory. This is what the Father has done for you and me. Keep this hope. You are truly somebody in
the eyes of God.

3:17 For God sent not his Son into the world to condemn the world; but that the world through him might
be saved.

3:18 ¶ He that believeth on him is not condemned: but he that believeth not is condemned already,
because he hath not believed in the name of the only begotten Son of God.

3:19 And this is the condemnation, that light is come into the world, and men loved darkness rather than
light, because their deeds were evil.

According to the passage, why is it that people do not believe?

John 3:18-21

Because people loved darkness rather than light.

Why is it that they loved darkness? Because their deeds were evil. Because they don’t like to be
reproved.

Illustration: Sometimes we hate the person better than us, not because they are evil, but because
they are good and we are evil. We don’t like it how they make us look bad. Some of us are too proud
and don’t like to be told we are bad or wrong. So we go do the opposite

http://breachrepairers.webs.com/ 576
So how is a person condemned? Does Jesus do the condemning?

John 3:18

We are condemned when we don’t believe. Illustration: A person gives us the directions to a house
and we don’t believe them. Who is on the losing end? The person that doesn’t believe and follow
what the other said.

So in context, what does it really mean to be born again? Born of the Bible – study it daily. We
must believe what we read. We must not just believe, but also do the truth (faith and works).

3:20 For every one that doeth evil hateth the light, neither cometh to the light, lest his deeds should be
reproved.

3:21 But he that doeth truth cometh to the light, that his deeds may be made manifest, that they are
wrought in God.

In verses 9-21, what word is being repeated over and over again that helps us to understand more
about being born again? Believe. It seems that a key to being born again of water and Spirit is to
believe. Believe what though? Believe in Jesus.

Jesus and His disciples go into the land of Judea and baptize (22)
3:22 ¶ After these things came Jesus and his disciples into the land of Judæa; and there he tarried with
them, and baptized.

John the Baptist is baptizing in Aenon (23-24)


3:23 ¶ And John also was baptizing in Ænon near to Salim, because there was much water there: and
they came, and were baptized.

At Aenon there was much water there.

3:24 For John was not yet cast into prison.

John testifies of Jesus again – He must increase, I must decrease (25-36)


3:25 ¶ Then there arose a question between some of John's disciples and the Jews about purifying.

3:26 And they came unto John, and said unto him, Rabbi, he that was with thee beyond Jordan, to whom
thou barest witness, behold, the same baptizeth, and all men come to him.

3:27 John answered and said, A man can receive nothing, except it be given him from heaven.

Everything that we have is given from God.

3:28 Ye yourselves bear me witness, that I said, I am not the Christ, but that I am sent before him.

3:29 He that hath the bride is the bridegroom: but the friend of the bridegroom, which standeth and
heareth him, rejoiceth greatly because of the bridegroom's voice: this my joy therefore is fulfilled.

John was the messenger. In olden times, when the bride and the groom were not able to meet, there
were messengers that went between the two to deliver the messages

http://breachrepairers.webs.com/ 577
3:30 He must increase, but I must decrease.

This is one of the most profound statements of humility

DA 179 Those who are true to their calling as messengers for God will not seek honor for themselves.
Love for self will be swallowed up in love for Christ.

Before you enter into the work, He will make you realize how “nothing” you are. Moses’ life can be
divided into 3 sections. First 40 years: He was becoming somebody. The next 40 years: He was
being taught he was nobody. The last 40 years: He was being shown how God can use a “nobody.”
We must always give glory to God for the good work that He has done in us.

John 5:35

The more a candle burns, the less it will become. It is a fitting example of who John the Baptist was
– until the flame was extinguished. John could sustain the loss of popularity, of crowds – it really
showed how he was a great prophet of God.

3:31 He that cometh from above is above all: he that is of the earth is earthly, and speaketh of the earth:
he that cometh from heaven is above all.

3:32 And what he hath seen and heard, that he testifieth; and no man receiveth his testimony.

3:33 He that hath received his testimony hath set to his seal that God is true.

3:34 For he whom God hath sent speaketh the words of God: for God giveth not the Spirit by measure
unto him.

All prophets are equally inspired.

3:35 The Father loveth the Son, and hath given all things into his hand.

3:36 He that believeth on the Son hath everlasting life: and he that believeth not the Son shall not see life;
but the wrath of God abideth on him.

http://breachrepairers.webs.com/ 578
Chapter 4 – Women at the Well | the Nobleman (Message to the
Gentiles-Outreach)
Chapter Outline
 Jesus departs because of the issue of baptism between Himself and John (1-3)
 Jesus and the Samaritan woman (4-30)
 The meat of Jesus – to do the will of the Father and to finish His work (31-38)
 Jesus abides two days with the Samaritans and many more believe (39-42)
 Jesus’ second miracle in Galilee – He heals the son of a noble man by speaking (43-54)

Jesus departs because of the issue of baptism between Himself and John
(1-3)
4:1 When therefore the Lord knew how the Pharisees had heard that Jesus made and baptized more
disciples than John,

When working for God, we must avoid conflict as much as possible. Even if there is conflict amongst
the conflict of God’s people.

4:2 (Though Jesus himself baptized not, but his disciples,)

4:3 He left Judæa, and departed again into Galilee.

Jesus and the Samaritan woman (4-30)


4:4 And he must needs go through Samaria.

It was natural to go through Samaria to go from Judaea to Galilee

What town did Jesus pass through? Who were these people?

John 4:4

Jesus passed through Samaria

Matt 10:5

Samaritans were Gentiles, not considered part of God’s people

2 Kings 17:29

They were idol worshippers

John 8:48

The Jews really despised the Samaritans, even going so far as to saying they had devils.

4:5 Then cometh he to a city of Samaria, which is called Sychar, near to the parcel of ground that Jacob
gave to his son Joseph.

http://breachrepairers.webs.com/ 579
4:6 Now Jacob's well was there. Jesus therefore, being wearied with his journey, sat thus on the well: and
it was about the sixth hour.

At what time of the day did Jesus meet the woman at the well? It was about the sixth hour
which is six hours added to 6am because the Jews considered that the start of the day. The woman
at the well went to Jesus at the sixth hour. Nicodemus came searching for Jesus at the late hour of
the night. Sixth hour is 12 in the afternoon.

4:7 There cometh a woman of Samaria to draw water: Jesus saith unto her, Give me to drink.

Why was the woman drawing water from the well at this hour? What does that tell us about
her? 12 noon was the hottest time of the day. People usually drew it at the early morning or late
afternoon when the sun was about to go down. It was not a sensible thing to do to get water when
the sun was the hottest and the highest. This showed that this woman was an outcast. She did not
want to meet other women as there was a stigma attached to her.

The offer of a kindness might have been rejected; but trust awakens trust. – DA p184

Why did Jesus talk to her first? Contrast this with Nicodemus. Who does Nicodemus
represent today?
Who does the woman represent? Jesus talked to her first to break through the social barriers of a
Jew talking to a Samaritan. Remember that Jews really despised Samaritans. Her also talked to her
first because maybe she thought that no one would talk to her because of the social stigma attached
to her, as her being an outcast. Contrast with Nicodemus: Nicodemus came to Jesus, Jesus didn’t go
to him. Jesus purposely reached out to this woman.

Who do these two characters represent? Nicodemus: He represents those in the church thinking
that they are saved and have been baptized. These types of people we don’t need to go to them.
When they realize they need help, they will know where to go and find help. Samaritan woman: She
represents the outcasts of society today. Those that no one wants to be associated with. We must
purposefully go and seek out the unconverted and rejects that are outside of the church and not
wait for them to come and look for us.

How can we follow Christ’s example in asking for water, when reaching out to people today?

John 3:4-5

He asked for water to connect with her. He gave her opportunity to help Him first before He offered
her everlasting water. In our own outreach and witnessing, do we take the time to connect with
people? What are some ways that you can ask for water from the unconverted? Teachers, think of
examples

4:8 ( For his disciples were gone away unto the city to buy meat.)

4:9 Then saith the woman of Samaria unto him, How is it that thou, being a Jew, askest drink of me,
which am a woman of Samaria? for the Jews have no dealings with the Samaritans.

Why was the woman surprised at Jesus asking for water? What can we learn from this in
reaching out to others? Jews usual had not dealings with Samaritans at all. Jesus broke through
social tradition and stigma. He speaks to a Samaritan. He speaks to a woman. He speaks to an

http://breachrepairers.webs.com/ 580
outcast. We need to learn to get out of our own comfort zone of who we usually associate ourselves
with. There are some that are out there that would be very surprised that we just befriend them.
Would we go out of our way to speak to those with whom stigma is attached?

4:10 Jesus answered and said unto her, If thou knewest the gift of God, and who it is that saith to thee,
Give me to drink; thou wouldest have asked of him, and he would have given thee living water.

What is Jesus trying to do in verse 10? What is her perception of the gift of God? What
important lesson do we learn from this? In Middle East, water was literally called the gift of God.
Jesus uses this meaning to catch her attention and transition it to a spiritual meaning. It is good for
us to understand the parables of the Bible and use these object lessons to turn something that is
talked about by the world into something spiritual.

Jesus points to Himself as the gift of God. When we reach out to others, we must always point them
to Jesus first. The gift of God. Our goal should be to lead them to Christ first. Many times we are
tempted to jump straight to people’s problems and sins. But we should offer Christ the solution
first. Many times we, like this woman, do not know who the person really is like. You can only judge
what is going on in the inside. It’s not the outward appearance. Water was the common theme at
the well. When we talk about things to people, we must put things in general interest but at the
same time, apply spiritual things to our talk.

4:11 The woman saith unto him, Sir, thou hast nothing to draw with, and the well is deep: from whence
then hast thou that living water?

Jesus fanned the flames of desire to know more. That is the same duty that we have. We must put
the gospel into a desirable state that people will want to know more.

4:12 Art thou greater than our father Jacob, which gave us the well, and drank thereof himself, and his
children, and his cattle?

Why did she ask about Jacob?

Because they are at Jacob’s well and Jesus is claiming to offer greater water than the patriach’s well.

4:13 Jesus answered and said unto her, Whosoever drinketh of this water shall thirst again:

4:14 But whosoever drinketh of the water that I shall give him shall never thirst; but the water that I
shall give him shall be in him a well of water springing up into everlasting life.

What is Jesus doing in verses 13-14? What is he transitioning to?

He is trying to transition from the literal to the spiritual. We need to learn to use literal examples
and try and turn it into a spiritual conversation. Jesus says whoever drinks will never thirst again.
Here He is enticing her with a life that is in abundance. A life that will be completely satisfied. Are
we able to talk about our lives like that? When we witness for the Lord, we first lead them to Christ,
then we must tell them of a life that they can have where they will never thirst. We must also be
able to show it in our own lives and attitude.

4:15 The woman saith unto him, Sir, give me this water, that I thirst not, neither come hither to draw.

Jesus accomplished what He intended to do – to stir up her desire.

http://breachrepairers.webs.com/ 581
4:16 Jesus saith unto her, Go, call thy husband, and come hither.

4:17 The woman answered and said, I have no husband. Jesus said unto her, Thou hast well said, I have
no husband:

Before we can partake of the living water, we must be able to renounce of the secret sins in our life.

Amos 8:11

Without the Word of God, there are people that are thirsting. She had been trying to fill an empty
place in her heart. Righteousness only covers us when we are willing to renounce our sins. When
working with peoples secret sins, there should not be people there. It would be embarrassing. Jesus
said her secret sins in private.

4:18 For thou hast had five husbands; and he whom thou now hast is not thy husband: in that saidst thou
truly.

What does Jesus address in verse 16-18? What is His method? After revealing her to Christ and
a life in abundance, Jesus now proceeds to mention her problem. Do you notice the tone? How does
Jesus approach it? Its almost an after thought. A “by the way, you have a problem”. There is no
condemnation. There isn’t a detailed solution offered. Why? Because Jesus already revealed Himself
and a life that will never thirst as the solution. Many times, we feel compelled to point out people’s
sins, make a big deal about it, offer 101 solutions. But really, Christ doesn’t ask us to do that.
Conviction is the role of the Holy Spirit. Our role is to offer Christ and show Him abundantly in our
lives. Then impartially point the problem and let the Holy Spirit work.

4:19 The woman saith unto him, Sir, I perceive that thou art a prophet.

4:20 Our fathers worshipped in this mountain; and ye say, that in Jerusalem is the place where men
ought to worship.

How does the Samaritan woman respond? What is the issue she raises? Put that in modern
day language?

John 4:19-20

She is embarrassed. She tries to distract Jesus by asking a question. What is she trying to ask?
Basically what is the true religion. She asking Jesus what is the bottom line? Should I worship the
Jewish way, the Samaritan way, etc. Which is the best way to worship? Modern day context, many
Christians are debating over which is the best way to worship. More praise worship? Clapping
hands? Bands? More prayer? Etc. Next Jesus gives the answer plain and simple

4:21 Jesus saith unto her, Woman, believe me, the hour cometh, when ye shall neither in this mountain,
nor yet at Jerusalem, worship the Father.

4:22 Ye worship ye know not what: we know what we worship: for salvation is of the Jews.

4:23 But the hour cometh, and now is, when the true worshippers shall worship the Father in spirit and
in truth: for
the Father seeketh such to worship him.

http://breachrepairers.webs.com/ 582
4:24 God is a Spirit: and they that worship him must worship him in spirit and in truth.

What is true worship according to Jesus reply?

John 4:21-24

True religion = Worship the Father in spirit and truth. Through the Holy Spirit and the Bible. Note:
has nothing to do with the song service or speaking in tongues.

4:25 The woman saith unto him, I know that Messias cometh, which is called Christ: when he is come, he
will tell us all things.

4:26 Jesus saith unto her, I that speak unto thee am he.

Contrast Jesus reply to her with His reply to Nicodemus. Why was He so direct in this case?

John 4:25-26

Nicodemus already knew the truth. He had grown up with the scriptures. You could say he had
grown up in church all his life. Jesus did not need to beat around the bush with Nicodemus. Nor did
Jesus want to explain everything. He left Nicodemus to do some searching himself. Whereas with
the woman at the well, Jesus did more socializing, more explanation, more open and clearer with
His answers. This is the same tactics we can use for gentiles vs those grown up in church

4:27 ¶ And upon this came his disciples, and marvelled that he talked with the woman: yet no man said,
What seekest thou? or, Why talkest thou with her?

4:28 The woman then left her waterpot, and went her way into the city, and saith to the men,

4:29 Come, see a man, which told me all things that ever I did: is not this the Christ?

What does she do after this? What can we learn from this?

John 4:28-29

The woman goes back to her village and tells everyone of Christ. She is not baptized, but starts to
witness. We know that we are talking to the convicted, when they start to witness to their friends,
even before baptism. She shows fruits of her belief before she is even baptized.

4:30 Then they went out of the city, and came unto him.

The meat of Jesus – to do the will of the Father and to finish His work
(31-38)
4:31 ¶ In the mean while his disciples prayed him, saying, Master, eat.

4:32 But he said unto them, I have meat to eat that ye know not of.

4:33 Therefore said the disciples one to another, Hath any man brought him ought to eat?

4:34 Jesus saith unto them, My meat is to do the will of him that sent me, and to finish his work.

4:35 Say not ye, There are yet four months, and then cometh harvest? behold, I say unto you, Lift up your

http://breachrepairers.webs.com/ 583
eyes, and look on the fields; for they are white already to harvest.

What should we not say? There are yet four months, and [then] cometh harvest? We must look
with our eyes unto the field. There are people ready to be reaped. There are people looking unto
Heaven hoping that they would come and explain the scripture unto them.

4:36 And he that reapeth receiveth wages, and gathereth fruit unto life eternal: that both he that soweth
and he that reapeth may rejoice together.

4:37 And herein is that saying true, One soweth, and another reapeth.

4:38 I sent you to reap that whereon ye bestowed no labour: other men laboured, and ye are entered into
their labours.

Jesus abides two days with the Samaritans and many more believe (39-
42)
4:39 ¶ And many of the Samaritans of that city believed on him for the saying of the woman, which
testified, He told me all that ever I did.

4:40 So when the Samaritans were come unto him, they besought him that he would tarry with them:
and he abode there two days.

4:41 And many more believed because of his own word;

4:42 And said unto the woman, Now we believe, not because of thy saying: for we have heard him
ourselves, and know that this is indeed the Christ, the Saviour of the world.

What is the result of the woman’s witness? What does it mean to truly believe?

John 4:39-42

She brings them all to see Jesus and they believe through their own personal experience with Him.
If you truly believe that Jesus is the Christ, you will bring others to Him.

Summary

John 4:7 - Jesus asks for water.


John 4:9 - Jesus breaks through stigma & social tradition
John 4:10 - Jesus offers the gift of God
John 4:13-14 - Jesus then describes a life in Him
John 4:16-18 - Jesus points out her problem
John 4:19-20 - Woman’s response, tries to divert
John 4:21-24 - Jesus’ response, no condemnation, points to true religion
John 4:28-29 - The result, woman witnesses to village

Process for witnessing to gentiles:


1. Ask for water, establish connection (remember this may take time)
2. Present Christ.
3. Present a life in abundance, never thirsting (example in life, not only talk).
4. Impartially mention problem & let the Holy Spirit work.
5. Present the true religion.

http://breachrepairers.webs.com/ 584
6. Encourage your new friend to share Christ to others.
7. They will bring others to Christ.

Jesus’ second miracle in Galilee – He heals the son of a noble man by


speaking (43-54)
4:43 ¶ Now after two days he departed thence, and went into Galilee.

Never neglect the opportunities to talk to a person. That one person may be a door to another
community of people that may never have been reached except through that person.

4:44 For Jesus himself testified, that a prophet hath no honour in his own country.
Sometimes when people know you so well, they may know all your faults and it makes it harder for
you to gain respect.

4:45 Then when he was come into Galilee, the Galilæans received him, having seen all the things that he
did at Jerusalem at the feast: for they also went unto the feast.

How did the nobleman know about Jesus? He had heard about Jesus from others. He had heard
that Jesus was coming to Galilee.

4:46 So Jesus came again into Cana of Galilee, where he made the water wine. And there was a certain
nobleman, whose son was sick at Capernaum.

4:47 When he heard that Jesus was come out of Judæa into Galilee, he went unto him, and besought him
that he would come down, and heal his son: for he was at the point of death.

Why did the nobleman want to meet Jesus? How do you think the nobleman must have felt
when asking Jesus?

He wanted Jesus to heal his son, who was at the point of death. He must have been desperate for
Jesus help. It says he “besought” = beseeching = pleading or begging. We see already that Jesus’
influence was increasing. News is spreading about Him.

4:48 Then said Jesus unto him, Except ye see signs and wonders, ye will not believe.

Why did Jesus say “Except ye see signs and wonders”? Many of us today need those to believe. If
Jesus does this or that, then we will believe. But the nobleman is casting himself out, he has no hope
left. God often allows us to reach a point where we recognize all human help is futile. When we
realize that only God can help us, then faith is pushing out. Sometimes, God allows us to experience
failure, so that we will learn to depend on God. God cannot work with us when we feel that we can
do everything ourselves.

What was Jesus' response to the nobleman and what does this tell us about what the
nobleman was thinking?

Except ye see signs and wonders ye will not believe. Jesus was rebuking the nobleman for his
thoughts. Jesus could read his thoughts. Why? In the nobleman’s mind, he would only believe in
Jesus IF He could heal his son. Otherwise He would not believe. Do we do the same thing today? In
what way? Share personal experiences. Many times we pray for something, we plead God for an
answer to our request, and in the back of our mind, we think… if God does not answer, then I will

http://breachrepairers.webs.com/ 585
stop believing. Its almost like we threaten God.

What is wrong with basing our belief on signs and wonders alone? Can you give a Bible text?

Matt. 24:24

Talks of the last days. Jesus said at the end of earth’s history, even false christs could perform signs
and wonders.

2 Thess. 2:9

Signs and wonders can come from Jesus or false Christ’s or the devil himself.

Rev 13:13

The beast of revelation 13 can also performs miracles in the last days, and if we place our beliefs
solely in signs and wonders, we will be deceived

4:49 The nobleman saith unto him, Sir, come down ere my child die.

How did the nobleman respond? What can we learn from this?

It was almost as if Jesus' response was calculated to reveal the nobleman's thoughts and may have
discouraged him from going on, but he kept persevering. What lesson can we learn from this?

Lesson 1: There is still a need to continue to persevere to ask. This shows that we aren’t just
believing in signs and wonders. If the nobleman had not repented he probably would have stopped.
But he saw his true condition, changed and continued to persevere to ask for his son.

Lesson 2: Even though we may have this type of thinking, Christ still has compassion on us. God will
answer our prayers immediately if we repent and continue to plead our cause.

4:50 Jesus saith unto him, Go thy way; thy son liveth. And the man believed the word that Jesus had
spoken unto him, and he went his way.

On what basis was the nobleman’s son healed? How soon?

John 4:50

2 criteria: Jesus spoke. The nobleman believed. Immediately. The hour he believed and acted upon
it.

4:51 And as he was now going down, his servants met him, and told him, saying, Thy son liveth.

4:52 Then enquired he of them the hour when he began to amend. And they said unto him, Yesterday at
the seventh hour the fever left him.

How did the nobleman show his faith?

John 4:51

By believing Jesus’ words and going his way.

http://breachrepairers.webs.com/ 586
Compare the reactions of Nobleman, Nicodemus, woman at the well. How did Jesus treat
each one in comparison to their reactions?

Key text on methods of teaching, preaching, and ministering to God’s people both inside and outside
of the church.

Notice that God intervened each time, and enlightened Jesus with wisdom on how to
minister to the different types of people. In each situation, Jesus knew what was going on
inside their mind. Application for teachers if that in order to properly minister to your
students, you must know them, care about them. After they realize that you care they are
more than willing to take your counsel. Application for Bible workers, is that both churched
(in this case a wealthy leader), un churched (in this case poor), and churched (in this case
wealthy church member).

Nicodemus was looking for the way to eternal life, but did not act upon it. Remember, Nicodemus
represents someone grown up in church and a leader even.

Woman at the well was not looking for Jesus, but believed in Him because of His words and brought
even more people to see Jesus. She represents someone new to Jesus, who doesn’t know the truth.

Nobleman was looking for healing of his son and nothing more. If Jesus healed his son he would
believe in him. He represents a rich man in church.

Jesus dealt with Nicodemus and the Nobleman very abruptly. Didn’t waste any time to rebuke them.
But He was more soft spoken (tender) and round about with the woman at the well because she did
not know the truth. She had not grown up with the scriptures.

What lessons can we learn about faith from this encounter? Our faith must not be based on
signs or wonders = miracles. We believe Jesus based on His Word. Rom 10:17 – Faith comes by
hearing, hearing by the word. When we repent, God can answer immediately.

4:53 So the father knew that it was at the same hour, in the which Jesus said unto him, Thy son liveth:
and himself believed, and his whole house.

We see here the power of God’s Word. Lessons of faith: Nobleman believed the Word of God. It had
creative power – as soon as Jesus spoke, it was done.

4:54 This is again the second miracle that Jesus did, when he was come out of Judæa into Galilee.

http://breachrepairers.webs.com/ 587
Chapter 5 – Healing at Bethesda
Chapter Outline
 Jesus heals the cripple man at Bethesda (1-9)
 The cripple man knows not who healed him so he cannot tell the Jews (10-13)
 The Jews find out it is Jesus and seek to persecute and slay Him for healing on the Sabbath
(14-16)
 Jews seek to kill Jesus more because He calls God His Father – making Himself equal with
God (17-18)
 Jesus explains His relationship between Himself and the Father (19-47)

Jesus heals the cripple man at Bethesda (1-9)


5:1 After this there was a feast of the Jews; and Jesus went up to Jerusalem.

5:2 Now there is at Jerusalem by the sheep market a pool, which is called in the Hebrew tongue Bethesda,
having five porches.

What 2 places are mentioned in this story?

John 5:1-3

There was a sheep market, a place with sheep. By the sheep, there was a pool called Bethesda.

What does Bethesda mean? Look in concordance. Who were found there?

House of kindness. Multitude of impotent folk, of blind, halt, withered.

How are the 2 places mentioned related? What spiritual applications can we learn?

Sheep market = where sheep are gathered. Where do spiritual sheep gather? Church. Bethesda =
House of kindness. What kind of people are gathered here? Impotent. Conclusion: Church is not
only a place where spiritual sheep are gathered, but also a place of kindness where the impotent
can come for healing.

5:3 In these lay a great multitude of impotent folk, of blind, halt, withered, waiting for the moving of the
water.

5:4 For an angel went down at a certain season into the pool, and troubled the water: whosoever then
first after the troubling of the water stepped in was made whole of whatsoever disease he had.

5:5 And a certain man was there, which had an infirmity thirty and eight years.

He was the worst case. Why did Jesus choose the worst case? If He could heal this man, then He can
heal anyone.

5:6 When Jesus saw him lie, and knew that he had been now a long time in that case, he saith unto him,
Wilt thou be made whole?

http://breachrepairers.webs.com/ 588
What do we know about the man from this passage? Who could he represent?

He was at the pool of Bethesda. He had his infirmities for 38 years. But nobody was there to help
him. When he tried by himself, others would step down before him. Who does he represent? Those
who are helpless and crippled. Although they want to be healed, they cannot help themselves.
Sounds like the wretched man in Romans 7:15-24. A person that knows where to find spiritual
healing, he may come to the pool of Bethesda or even is among the sheep, but never have been able
to overcome. Helplessly lost in sinful habits. They can see the pool, but just can’t get to it.

What does Jesus ask the man? What is Christ’s desire? Then why does Jesus need to ask?

Wilt thou be made whole? The man heard a voice. Then he saw this face with loving compassion.
Christ’s desire according to Mark 1:41 is always that we would be made whole. Then why does
Jesus ask? For some people, like the leper in Mark 1, Christ waited for him to come. However, for
others who were too sick, too crippled, Christ went to them. But always, Christ gave each individual
the power of choice. He waited for them to tell him they wanted healing.

5:7 The impotent man answered him, Sir, I have no man, when the water is troubled, to put me into the
pool: but while I am coming, another steppeth down before me.

“I have no man” – He was depending on the arm of flesh. What is the man’s response? Summarize
his response.

He gives excuses for why he cannot be healed. Application: Don’t we often do the same thing? Christ
asks if we want to be healed, and we start giving responses for why it wouldn’t work or why it
hasn’t worked in the past.

What did Christ ask him to do? How was the man healed? What lessons about faith can we
learn?

Jas 2:20

What did Christ ask him to do? Jesus asked the man to rise, take up his bed and walk. How was the
man healed? By obeying Christ’s command. What lessons about faith can we learn?

1. Although Christ is able to make us whole, we will not realize our healing until we walk in
faith. The man did not realize he was healed until he actually got up. Therefore, we can
believe all the scriptures we want, until we put it into action, spiritual healing or cleansing is
not realized. James 2:20 – Faith without works is dead!

2. We can only be made whole by exercising faith in God’s word. NOTE: Christ did not even ask
the man to believe in Christ. Just to obey His word there is power to heal.

3. We must believe even if our body does not feel it. The man could have asked Jesus to make
him whole first, but that would not have healed him. The process of being made whole or
healing comes in the ACT of obedience. Faith must not be based on feeling BUT on
obedience even when we don’t feel or understand.

Spiritually, in what way are we asked to rise up and walk?

http://breachrepairers.webs.com/ 589
Rom 6:4

We are buried with Christ in baptism, and raised up to walk in the newness of life.

John 3:8

This is not just baptism of water, but also baptism of the Holy Spirit.

5:8 Jesus saith unto him, Rise, take up thy bed, and walk.

5:9 And immediately the man was made whole, and took up his bed, and walked: and on the same day
was the sabbath.

IMPORTANT: healing took place on the Sabbath

The cripple man knows not who healed him so he cannot tell the Jews
(10-13)
5:10 ¶ The Jews therefore said unto him that was cured, It is the sabbath day: it is not lawful for thee to
carry thy bed.

5:11 He answered them, He that made me whole, the same said unto me, Take up thy bed, and walk.

5:12 Then asked they him, What man is that which said unto thee, Take up thy bed, and walk?

5:13 And he that was healed wist not who it was: for Jesus had conveyed himself away, a multitude being
in that place.

How did the Jews react to the healed man? How can we react in the same way today to those
who are healed, but carrying their beds.

John 5:9-13

Rather than rejoice that one of the sick had been healed, all the Jews could see was that the man
was carrying his bed on the Sabbath.

Application: Many times, we as spiritual Jews, rather than rejoice that a newly baptized convert has
been saved from the condemnation of sin, see only that they are not at our level. They are still
carrying their bed on the Sabbath. They still have not learned all the fine points of keeping the
Sabbath or may not know all the lingo, or still not dressed in suit and tie for church. We have to be
careful not to be like the Jews in Christ’s time.

What were Jesus’ final words to the man? What is Christ’s desire for those who have been
healed spiritually?

Matt 12:43-45

2 Pet 2:20-21

Christ wants us to sin no more after He has healed us from the condemnation of sin. After giving our
life to the Lord through baptism, his command to us is sin no more, lest a worse thing come unto
thee. How is their last state worse? Because now they have the knowledge of the truth and they

http://breachrepairers.webs.com/ 590
intentionally turn back. Before they didn't have the knowledge of salvation. Also, because they
knew what they were saved out from but are willing to go back into it after tasting the goodness of
God and His mercy.

The Jews find out it is Jesus and seek to persecute and slay Him for
healing on the Sabbath (14-16)
5:14 Afterward Jesus findeth him in the temple, and said unto him, Behold, thou art made whole: sin no
more, lest a worse thing come unto thee.

“sin no more” – If we sin against light after we receive it, we get worse. Jesus said “sin no more”. It
is possible to be perfect.

5:15 The man departed, and told the Jews that it was Jesus, which had made him whole.

5:16 And therefore did the Jews persecute Jesus, and sought to slay him, because he had done these
things on the sabbath day.

We see how to keep the Sabbath:

DA 207 The necessities of life must be attended to, the sick must be cared for, the wants of the needy
must be supplied. He will not be held guiltless who neglects to relieve suffering on the Sabbath. God's
holy rest day was made for man, and acts of mercy are in perfect harmony with its intent. God does
not desire His creatures to suffer an hour's pain that may be relieved upon the Sabbath or any other
day….The demands upon God are even greater upon the Sabbath than upon other days….God does
not wait for the Sabbath to pass before He grants these requests. Heaven's work never ceases, and
men should never rest from doing good. The Sabbath is not intended to be a period of useless
inactivity.

If you do not relieve suffering on the Sabbath, you will be held guilty

DA 207 but as God ceased His labor of creating, and rested upon the Sabbath and blessed it, so man is
to leave the occupations of his daily life, and devote those sacred hours to healthful rest, to worship,
and to holy deeds.

Healthful rest, Worship, Holy deeds

Jews seek to kill Jesus more because He calls God His Father (17-18)
5:17 ¶ But Jesus answered them, My Father worketh hitherto, and I work.

5:18 Therefore the Jews sought the more to kill him, because he not only had broken the sabbath, but
said also that God was his Father, making himself equal with God.

Jesus explains His relationship between Himself and the Father (19-47)
5:19 Then answered Jesus and said unto them, Verily, verily, I say unto you, The Son can do nothing of
himself, but what he seeth the Father do: for what things soever he doeth, these also doeth the Son
likewise.

5:20 For the Father loveth the Son, and sheweth him all things that himself doeth: and he will shew him
greater works than these, that ye may marvel.

5:21 For as the Father raiseth up the dead, and quickeneth them; even so the Son quickeneth whom he

http://breachrepairers.webs.com/ 591
will.

5:22 For the Father judgeth no man, but hath committed all judgment unto the Son:

5:23 That all men should honour the Son, even as they honour the Father. He that honoureth not the Son
honoureth not the Father which hath sent him.

5:24 Verily, verily, I say unto you, He that heareth my word, and believeth on him that sent me, hath
everlasting life, and shall not come into condemnation; but is passed from death unto life.

Jesus is explaining the spiritual teaching behind the miracle: When the sinner heard the Word of
Jesus, he believed, so he passed from death into everlasting life.

“hath everlasting life” – that is present tense. If you have eternal life, can you die? – yes you can
die.

“when the dead shall hear the voice of the Son of God”

DA 209 Christ declares that even now the power which gives life to the dead is among them, and they
are to behold its manifestation. This same resurrection power is that which gives life to the soul
"dead in trespasses and sins." Eph. 2:1. The dominion of evil is broken, and through faith the soul is
kept from sin. He who opens his heart to the Spirit of Christ becomes a partaker of that mighty power
which shall bring forth his body from the grave.

5:25 Verily, verily, I say unto you, The hour is coming, and now is, when the dead shall hear the voice of
the Son of God: and they that hear shall live.

5:26 For as the Father hath life in himself; so hath he given to the Son to have life in himself;

The verse seems to indicate that the power was given to Him. The ability of having life in Himself
was given by the Father.

5:27 And hath given him authority to execute judgment also, because he is the Son of man.

5:28 Marvel not at this: for the hour is coming, in the which all that are in the graves shall hear his voice,

5:29 And shall come forth; they that have done good, unto the resurrection of life; and they that have
done evil, unto the resurrection of damnation.

This is an excellent doctrinal text – proves that the people that are dead are in the graves. It also
shows that there are 2 resurrections

5:30 I can of mine own self do nothing: as I hear, I judge: and my judgment is just; because I seek not
mine own will, but the will of the Father which hath sent me.

5:31 If I bear witness of myself, my witness is not true.

5:32 ¶ There is another that beareth witness of me; and I know that the witness which he witnesseth of
me is true.

5:33 Ye sent unto John, and he bare witness unto the truth.

5:34 But I receive not testimony from man: but these things I say, that ye might be saved.

http://breachrepairers.webs.com/ 592
5:35 He was a burning and a shining light: and ye were willing for a season to rejoice in his light.

5:36 ¶ But I have greater witness than that of John: for the works which the Father hath given me to
finish, the same works that I do, bear witness of me, that the Father hath sent me.

5:37 And the Father himself, which hath sent me, hath borne witness of me. Ye have neither heard his
voice at any time, nor seen his shape.

5:38 And ye have not his word abiding in you: for whom he hath sent, him ye believe not.

5:39 ¶ Search the scriptures; for in them ye think ye have eternal life: and they are they which testify of
me.

There is a typo in the verse. It should be a “but” not an “and”

5:40 And ye will not come to me, that ye might have life.

5:41 I receive not honour from men.

5:42 But I know you, that ye have not the love of God in you.

5:43 I am come in my Father's name, and ye receive me not: if another shall come in his own name, him
ye will receive.

5:44 How can ye believe, which receive honour one of another, and seek not the honour that cometh from
God only?

5:45 Do not think that I will accuse you to the Father: there is one that accuseth you, even Moses, in
whom ye trust.

5:46 For had ye believed Moses, ye would have believed me: for he wrote of me.

5:47 But if ye believe not his writings, how shall ye believe my words?

http://breachrepairers.webs.com/ 593
Chapter 6 – The True Bread
Reading
 Desire of Ages – Chapter A Night on the Lake
 Desire of Ages – Chapter 30 – He Ordained Twelve

Chapter Outline
 Jesus feeds the multitude and they testify that He is a prophet (1-14)
 Jesus walks on the sea and brings the disciples boat over to Capernaum (15-21)
 Jesus exhorts the multitude to labour for the meat that does not perish (22-29)
 Jesus introduces Himself as the bread of life, the living bread (30-52)
 Jesus exhorts them to eat his flesh and drink his blood (53-65)
 Separation of true and false disciples (66-71)

Jesus feeds the multitude and they testify that He is a prophet (1-14)
6:1 After these things Jesus went over the sea of Galilee, which is the sea of Tiberias.

6:2 And a great multitude followed him, because they saw his miracles which he did on them that were
diseased.

6:3 And Jesus went up into a mountain, and there he sat with his disciples.

6:4 And the passover, a feast of the Jews, was nigh.

6:5 When Jesus then lifted up his eyes, and saw a great company come unto him, he saith unto Philip,
Whence shall we buy bread, that these may eat?

6:6 And this he said to prove him: for he himself knew what he would do.

6:7 Philip answered him, Two hundred pennyworth of bread is not sufficient for them, that every one of
them may take a little.

6:8 One of his disciples, Andrew, Simon Peter's brother, saith unto him,

6:9 There is a lad here, which hath five barley loaves, and two small fishes: but what are they among so
many?

6:10 And Jesus said, Make the men sit down. Now there was much grass in the place. So the men sat
down, in number about five thousand.

6:11 And Jesus took the loaves; and when he had given thanks, he distributed to the disciples, and the
disciples to them that were set down; and likewise of the fishes as much as they would.

6:12 When they were filled, he said unto his disciples, Gather up the fragments that remain, that nothing
be lost.

6:13 Therefore they gathered them together, and filled twelve baskets with the fragments of the five
barley loaves, which remained over and above unto them that had eaten.

http://breachrepairers.webs.com/ 594
6:14 Then those men, when they had seen the miracle that Jesus did, said, This is of a truth that prophet
that should come into the world.

Jesus walks on the sea and brings the disciples boat over to Capernaum
(15-21)
6:15 When Jesus therefore perceived that they would come and take him by force, to make him a king, he
departed again into a mountain himself alone.

6:16 And when even was now come, his disciples went down unto the sea,

6:17 And entered into a ship, and went over the sea toward Capernaum. And it was now dark, and Jesus
was not come to them.

6:18 And the sea arose by reason of a great wind that blew.

6:19 So when they had rowed about five and twenty or thirty furlongs, they see Jesus walking on the sea,
and drawing nigh unto the ship: and they were afraid.

6:20 But he saith unto them, It is I; be not afraid.

6:21 Then they willingly received him into the ship: and immediately the ship was at the land whither
they went.

Desire of Ages says they were in the middle of the lake, but once they saw Jesus, they then reached
the land.

Jesus exhorts the multitude to labour for the meat that does not perish
(22-29)
6:22 The day following, when the people which stood on the other side of the sea saw that there was
none other boat there, save that one whereinto his disciples were entered, and that Jesus went not with
his disciples into the boat, but that his disciples were gone away alone;

6:23 (Howbeit there came other boats from Tiberias nigh unto the place where they did eat bread, after
that the Lord had given thanks:)

6:24 When the people therefore saw that Jesus was not there, neither his disciples, they also took
shipping, and came to Capernaum, seeking for Jesus.

6:25 And when they had found him on the other side of the sea, they said unto him, Rabbi, when camest
thou hither?

6:26 Jesus answered them and said, Verily, verily, I say unto you, Ye seek me, not because ye saw the
miracles, but because ye did eat of the loaves, and were filled.

6:27 Labour not for the meat which perisheth, but for that meat which endureth unto everlasting life,
which the Son of man shall give unto you: for him hath God the Father sealed.

6:28 Then said they unto him, What shall we do, that we might work the works of God?

6:29 Jesus answered and said unto them, This is the work of God, that ye believe on him whom he hath
sent.

http://breachrepairers.webs.com/ 595
Jesus introduces Himself as the bread of life, the living bread (30-52)
6:30 They said therefore unto him, What sign shewest thou then, that we may see, and believe thee?
what dost thou work?

6:31 Our fathers did eat manna in the desert; as it is written, He gave them bread from heaven to eat.

6:32 Then Jesus said unto them, Verily, verily, I say unto you, Moses gave you not that bread from
heaven; but my Father giveth you the true bread from heaven.

6:33 For the bread of God is he which cometh down from heaven, and giveth life unto the world.

6:34 Then said they unto him, Lord, evermore give us this bread.

6:35 And Jesus said unto them, I am the bread of life: he that cometh to me shall never hunger; and he
that believeth on me shall never thirst.

6:36 But I said unto you, That ye also have seen me, and believe not.

6:37 All that the Father giveth me shall come to me; and him that cometh to me I will in no wise cast out.

6:38 For I came down from heaven, not to do mine own will, but the will of him that sent me.

6:39 And this is the Father's will which hath sent me, that of all which he hath given me I should lose
nothing, but should raise it up again at the last day.

6:40 And this is the will of him that sent me, that every one which seeth the Son, and believeth on him,
may have everlasting life: and I will raise him up at the last day.

Jesus made it very plain when He was speaking about manna, He was talking about Himself.

Num 21:5

The Bible says that the people murmured. The first step in the plan of salvation is that the Father
will draw them (v 44). If you eat the flesh of Jesus and drink His blood, will you have eternal life?
Yes. Can you die? Yes. Can you have eternal life now? Yes. It is very clear here that it is possible.
Eternal life begins now. What does eating of His body and drinking His blood mean?

6:41 The Jews then murmured at him, because he said, I am the bread which came down from heaven.

6:42 And they said, Is not this Jesus, the son of Joseph, whose father and mother we know? how is it then
that he saith, I came down from heaven?

6:43 Jesus therefore answered and said unto them, Murmur not among yourselves.

6:44 No man can come to me, except the Father which hath sent me draw him: and I will raise him up at
the last day.

6:45 It is written in the prophets, And they shall be all taught of God. Every man therefore that hath
heard, and hath learned of the Father, cometh unto me.

6:46 Not that any man hath seen the Father, save he which is of God, he hath seen the Father.

6:47 Verily, verily, I say unto you, He that believeth on me hath everlasting life.

http://breachrepairers.webs.com/ 596
6:48 I am that bread of life.

6:49 Your fathers did eat manna in the wilderness, and are dead.

6:50 This is the bread which cometh down from heaven, that a man may eat thereof, and not die.

6:51 I am the living bread which came down from heaven: if any man eat of this bread, he shall live for
ever: and the bread that I will give is my flesh, which I will give for the life of the world.

6:52 The Jews therefore strove among themselves, saying, How can this man give us his flesh to eat?

Jesus exhorts them to eat his flesh and drink his blood (53-65)
6:53 Then Jesus said unto them, Verily, verily, I say unto you, Except ye eat the flesh of the Son of man,
and drink his blood, ye have no life in you.

6:54 Whoso eateth my flesh, and drinketh my blood, hath eternal life; and I will raise him up at the last
day.

6:55 For my flesh is meat indeed, and my blood is drink indeed.

6:56 He that eateth my flesh, and drinketh my blood, dwelleth in me, and I in him.

6:57 As the living Father hath sent me, and I live by the Father: so he that eateth me, even he shall live by
me.

6:58 This is that bread which came down from heaven: not as your fathers did eat manna, and are dead:
he that eateth of this bread shall live for ever.

6:59 These things said he in the synagogue, as he taught in Capernaum.

6:60 Many therefore of his disciples, when they had heard this, said, This is an hard saying; who can hear
it?

They knew that He was not here to give them the temporal power that they were wanting (EGW).
“This is an hard saying.”

6:61 When Jesus knew in himself that his disciples murmured at it, he said unto them, Doth this offend
you?

6:62 What and if ye shall see the Son of man ascend up where he was before?

6:63 It is the spirit that quickeneth; the flesh profiteth nothing: the words that I speak unto you, they are
spirit, and they are life.

Jesus said unto His disciples if you eat my flesh and drink My blood, you shall have life. But the
words that I speak unto you, they are spirit and life.

6:64 But there are some of you that believe not. For Jesus knew from the beginning who they were that
believed not, and who should betray him.

6:65 And he said, Therefore said I unto you, that no man can come unto me, except it were given unto
him of my Father.

http://breachrepairers.webs.com/ 597
Separation of true and false disciples (66-71)
6:66 From that time many of his disciples went back, and walked no more with him.

This is called the Crisis in Galilee. Many walked not with Jesus after this.

DA 391 Christ had spoken a sacred, eternal truth regarding the relation between Himself and His
followers. He knew the character of those who claimed to be His disciples, and His words tested their
faith. He declared that they were to believe and act upon His teaching. All who received Him would
partake of His nature, and be conformed to His character. This involved the relinquishment of their
cherished ambitions. It required the complete surrender of themselves to Jesus. They were called to
become self-sacrificing, meek and lowly in heart. They must walk in the narrow path traveled by the
Man of Calvary, if they would share in the gift of life and the glory of heaven.

DA 391 The test was too great. The enthusiasm of those who had sought to take Him by force and
make Him king grew cold. This discourse in the synagogue, they declared, had opened their eyes.
Now they were undeceived. In their minds His words were a direct confession that He was not the
Messiah, and that no earthly rewards were to be realized from connection with Him. They had
welcomed His miracle-working power; they were eager to be freed from disease and suffering; but
they would not come into sympathy with His self-sacrificing life. They cared not for the mysterious
spiritual kingdom of which He spoke. The insincere, the selfish, who had sought Him, no longer
desired Him. If He would not devote His power and influence to obtaining their freedom from the
Romans, they would have nothing to do with Him.”

Many begin but not many finish.

6:67 Then said Jesus unto the twelve, Will ye also go away?

6:68 Then Simon Peter answered him, Lord, to whom shall we go? thou hast the words of eternal life.

6:69 And we believe and are sure that thou art that Christ, the Son of the living God.

6:70 Jesus answered them, Have not I chosen you twelve, and one of you is a devil?

This church (SDA church) has problems. But the Lord will purge His people at the end of time. You
must be sure that Jesus is the Christ, the Son of the living God. This happens right after the story in
Matthew 14.

Judas clearly felt the power of Jesus in his life.

DA 716 But Judas did not come to the point of surrendering himself fully to Christ. He did not give up
his worldly ambition or his love of money.

The history of Judas shows us the danger of continually cherishing one sin.

DA 717 Judas was blinded to his own weakness of character, and Christ placed him where he would
have an opportunity to see and correct this.

If you have been given a position by God, it shows that God has put you where you are so you can
see who you really are.

DA 718 He felt the satisfaction that always comes in service to God.

Judas felt the same sense of satisfaction as we did when we serve God. Judas is really not all that

http://breachrepairers.webs.com/ 598
different from us.

DA 719 He regarded himself as farsighted, and thought he could see that Jesus would have no honor,
and that He could bestow no high position upon His followers. He determined not to unite himself so
closely to Christ but that he could draw away. He would watch. And he did watch.

The turning point for Judas was when he saw that following Jesus added no temporal advantage. If
we realize what God’s calling is for us, would we still follow Him?

DA 720 In all that Christ said to His disciples, there was something with which, in heart, Judas
disagreed. Under his influence the leaven of disaffection was fast doing its work. The disciples did not
see the real agency in all this; but Jesus saw that Satan was communicating his attributes to Judas,
and thus opening up a channel through which to influence the other disciples.

Satan only needed just one to do his work to influence the rest.

DA 720 Wounded pride and desire for revenge broke down the barriers, and the greed so long
indulged held him in control. But Judas was not yet wholly hardened. Even after he had twice pledged
himself to betray the Saviour, there was opportunity for repentance. At the Passover supper Jesus
proved His divinity by revealing the traitor's purpose. He tenderly included Judas in the ministry to
the disciples. But the last appeal of love was unheeded. Then the case of Judas was decided, and the
feet that Jesus had washed went forth to the betrayer's work.

DA 721 As the trial drew to a close, Judas could endure the torture of his guilty conscience no longer.
Suddenly a hoarse voice rang through the hall, sending a thrill of terror to all hearts: He is innocent;
spare Him, O Caiaphas! The tall form of Judas was now seen pressing through the startled throng. His
face was pale and haggard, and great drops of sweat stood on his forehead. Rushing to the throne of
judgment, he threw down before the high priest the pieces of silver that had been the price of his
Lord's betrayal. Eagerly grasping the robe of Caiaphas, he implored him to release Jesus, declaring
that He had done nothing worthy of death. Caiaphas angrily shook him off, but was confused, and
knew not what to say. The perfidy of the priests was revealed. It was evident that they had bribed the
disciple to betray his Master.

DA 722 Judas now cast himself at the feet of Jesus, acknowledging Him to be the Son of God, and
entreating Him to deliver Himself. The Saviour did not reproach His betrayer. He knew that Judas did
not repent; his confession was forced from his guilty soul by an awful sense of condemnation and a
looking for of judgment, but he felt no deep, heartbreaking grief that he had betrayed the spotless
Son of God, and denied the Holy One of Israel.

DA 722 The priests had been willing to make Judas their tool; but they despised his baseness. When
he turned to them with confession, they spurned him.

In the kingdom of Satan, there isn’t really any true affection.

DA 722 In falling, his body had been horribly mangled, and dogs were now devouring it. His remains
were immediately buried out of sight; but there was less mockery among the throng, and many a pale
face revealed the thoughts within. Retribution seemed already visiting those who were guilty of the
blood of Jesus.

6:71 He spake of Judas Iscariot the son of Simon: for he it was that should betray him, being one of the
twelve.

http://breachrepairers.webs.com/ 599
Chapter 7
Chapter Outline

7:1 After these things Jesus walked in Galilee: for he would not walk in Jewry, because the Jews sought to
kill him.

This could have been about middle of the week, Wednesday. The theme of the conversation is Jesus.

7:2 Now the Jews' feast of tabernacles was at hand.

7:3 His brethren therefore said unto him, Depart hence, and go into Judaea, that thy disciples also may
see the works that thou doest.

7:4 For there is no man that doeth any thing in secret, and he himself seeketh to be known openly. If thou
do these things, shew thyself to the world.

7:5 For neither did his brethren believe in him.

7:6 Then Jesus said unto them, My time is not yet come: but your time is alway ready.

“My time is not yet come” – there is time that is OK to lie, that is not true. Jesus wants to go
unknown, not because He wants to lie.

7:7 The world cannot hate you; but me it hateth, because I testify of it, that the works thereof are evil.

7:8 Go ye up unto this feast: I go not up yet unto this feast; for my time is not yet full come.

7:9 When he had said these words unto them, he abode still in Galilee.

7:10 But when his brethren were gone up, then went he also up unto the feast, not openly, but as it were
in secret.

7:11 Then the Jews sought him at the feast, and said, Where is he?

7:12 And there was much murmuring among the people concerning him: for some said, He is a good
man: others said, Nay; but he deceiveth the people.

7:13 Howbeit no man spake openly of him for fear of the Jews.

7:14 Now about the midst of the feast Jesus went up into the temple, and taught.

7:15 And the Jews marvelled, saying, How knoweth this man letters, having never learned?

They didn’t understand how someone who hadn’t been to the rabbinical schools could teach so
well. People want to know “does he knows?” Can he do the work? You will have a place to work out
in the field if you really study and know what you had learnt. It is the quality of the materials you
learnt, not the quantity.

7:16 Jesus answered them, and said, My doctrine is not mine, but his that sent me.

7:17 If any man will do his will, he shall know of the doctrine, whether it be of God, or whether I speak of
myself.

http://breachrepairers.webs.com/ 600
The greatest condition to understanding the truth. Before we can understand truth, we must be
willing. God will not reveal to you more truth unless you would be willing to do what God would
want you to do. Pre-requisite to understand the truth, you must set your mind to be willing to learn
what God wants you to do or to change.

7:18 He that speaketh of himself seeketh his own glory: but he that seeketh his glory that sent him, the
same is true, and no unrighteousness is in him.

Key text on characteristic of Man of sin.

A sign that a person is not sent from God is that he is going to be seeking his own glory.

7:19 Did not Moses give you the law, and yet none of you keepeth the law? Why go ye about to kill me?

Jesus gives them a sign that He is divine by reading their thoughts.

7:20 The people answered and said, Thou hast a devil: who goeth about to kill thee?

7:21 Jesus answered and said unto them, I have done one work, and ye all marvel.

7:22 Moses therefore gave unto you circumcision; (not because it is of Moses, but of the fathers;) and ye
on the sabbath day circumcise a man.

7:23 If a man on the sabbath day receive circumcision, that the law of Moses should not be broken; are ye
angry at me, because I have made a man every whit whole on the sabbath day?

7:24 Judge not according to the appearance, but judge righteous judgment.

7:25 Then said some of them of Jerusalem, Is not this he, whom they seek to kill?

7:26 But, lo, he speaketh boldly, and they say nothing unto him. Do the rulers know indeed that this is the
very Christ?

7:27 Howbeit we know this man whence he is: but when Christ cometh, no man knoweth whence he is.

7:28 Then cried Jesus in the temple as he taught, saying, Ye both know me, and ye know whence I am:
and I am not come of myself, but he that sent me is true, whom ye know not.

They made allusions to His birth that He was not divine.

7:29 But I know him: for I am from him, and he hath sent me.

7:30 Then they sought to take him: but no man laid hands on him, because his hour was not yet come.

Does providence overrule in peoples dealings with us? Yes

7:31 And many of the people believed on him, and said, When Christ cometh, will he do more miracles
than these which this man hath done?

7:32 The Pharisees heard that the people murmured such things concerning him; and the Pharisees and
the chief priests sent officers to take him.

http://breachrepairers.webs.com/ 601
7:33 Then said Jesus unto them, Yet a little while am I with you, and then I go unto him that sent me.

7:34 Ye shall seek me, and shall not find me: and where I am, thither ye cannot come.

7:35 Then said the Jews among themselves, Whither will he go, that we shall not find him? will he go
unto the dispersed among the Gentiles, and teach the Gentiles?

7:36 What manner of saying is this that he said, Ye shall seek me, and shall not find me: and where I am,
thither ye cannot come?

7:37 In the last day, that great day of the feast, Jesus stood and cried, saying, If any man thirst, let him
come unto me, and drink.

7:38 He that believeth on me, as the scripture hath said, out of his belly shall flow rivers of living water.

Water can represent righteousness and can also represent His truth and Word. Blessed are they
that thirst after righteousness. Jesus offers the living water in the end of the ceremonies. Jesus uses
it with the woman at the well.

Amos 8:11

Famine & thirst for the word of God. River of living water. The woman at the well become the spring
of water, bring the blessing to others. They will not only be the recipients but also the giver.

7:39 (But this spake he of the Spirit, which they that believe on him should receive: for the Holy Ghost
was not yet given; because that Jesus was not yet glorified.)

7:40 Many of the people therefore, when they heard this saying, said, Of a truth this is the Prophet.

7:41 Others said, This is the Christ. But some said, Shall Christ come out of Galilee?

7:42 Hath not the scripture said, That Christ cometh of the seed of David, and out of the town of
Bethlehem, where David was?

DA 457 It was generally believed that Christ would be born at Bethlehem, but that after a time He
would disappear, and at His second appearance none would know whence He came. There were not
a few who held that the Messiah would have no natural relationship to humanity. And because the
popular conception of the glory of the Messiah was not met by Jesus of Nazareth, many gave heed to
the suggestion, "Howbeit we know this Man whence He is: but when Christ cometh, no man knoweth
whence He is."

There were 3 common errors about the coming of the Messiah at that time: Born in Jerusalem, He
would have no relationship with humanity—divine, He would come in great glory. It was because
they weren’t studying the Bible for themselves. Human interpretation of themselves. A lot of people
today are not studying the scriptures for themselves. So when they are looking for Jesus’ 2nd
coming, not people will know about the true signs of His coming. Because of this people will have
misconceptions.

7:43 So there was a division among the people because of him.

7:44 And some of them would have taken him; but no man laid hands on him.

7:45 Then came the officers to the chief priests and Pharisees; and they said unto them, Why have ye not

http://breachrepairers.webs.com/ 602
brought him?

7:46 The officers answered, Never man spake like this man.

7:47 Then answered them the Pharisees, Are ye also deceived?

7:48 Have any of the rulers or of the Pharisees believed on him?

7:49 But this people who knoweth not the law are cursed.

7:50 Nicodemus saith unto them, (he that came to Jesus by night, being one of them,)

When Nicodemus first came to Jesus, he came because he wanted to question His authority to teach.
Since then Nicodemus has changed a lot. He is defending Christ from within.

7:51 Doth our law judge any man, before it hear him, and know what he doeth?

Connect with James 1.

7:52 They answered and said unto him, Art thou also of Galilee? Search, and look: for out of Galilee
ariseth no prophet.

But now they answered back to Nicodemus. But when Nicodemus stood up for Jesus now, others
could see that he was defending Jesus.

7:53 And every man went unto his own house.

http://breachrepairers.webs.com/ 603
Chapter 8 – Women taken in Adultery
DA 459 On the last day of the feast, the officers sent out by the priests and rulers to arrest Jesus,
returned without Him. They were angrily questioned, "Why have ye not brought Him?" With solemn
countenance they answered, "Never man spake like this Man."

DA 460 But in the early morning He returned to the temple, and as the people gathered about Him,
He sat down and taught them.

It seems that this chapter is after the feast of tabernacles.

Chapter Outline

8:1 Jesus went unto the mount of Olives.

8:2 And early in the morning he came again into the temple, and all the people came unto him; and he
sat down, and taught them.

8:3 And the scribes and Pharisees brought unto him a woman taken in adultery; and when they had set
her in the midst,

Where did the Pharisees bring the woman? Where would that be today?

In the middle of the church, maybe prayer meeting, maybe church service – Christ must have been
teaching or preaching.

How would you react if someone brought an adulterer or sinner into church?

What would you do if someone dragged a woman caught in adultery or some other crime and threw
them in the middle of church?

8:4 They say unto him, Master, this woman was taken in adultery, in the very act.

This woman was caught in adultery in public. Her sin, everyone knew. God’s people are also often
referred to as adulterers (James 4). This experience can be applied to a church. Her sin was
discovered publicly. We have a choice right now to allow our sins to be covered by His
righteousness or we can hide them now and they will become public later. Be sure that your sin will
find you out. There is nothing that is done is secret that will not be reveled someday. So we can
openly confess our sins now or someday everyone will know. Either here on earth or in Heaven.

8:5 Now Moses in the law commanded us, that such should be stoned: but what sayest thou?

What can we tell about the Pharisees’ attitude in verse 4-5?

Pharisees = so called spiritual leaders / authority figures. They told Jesus that the law of Moses said
they could stone her, but wanted his opinion.

Their attitude: As church leaders, instead of thinking of how they could help the woman, they were
thinking of how they should and could punish her. Instead of dealing with this matter in private,
they were trying to drag it out before everyone to see. They had no love or compassion for this
woman. They were just out to destroy her.

http://breachrepairers.webs.com/ 604
NOTE: they must have been waiting for a crime like this to happen in order for them to catch her in
the very act. The Pharisees were just waiting for this to happen. Some people are like that. Rather
than helping, they are just waiting to catch you in the very act.

What was the trap? If Jesus said stone her, He would be defying the authority of Rome. If Jesus said
not to stone her, then He would be defying the law of Moses.

8:6 This they said, tempting him, that they might have to accuse him. But Jesus stooped down, and with
his finger wrote on the ground, as though he heard them not.

Why were the Pharisees consulting Jesus?

Because they wanted to trap him. They wanted to accuse Him of going against the law of Moses or
presuming to execute law of the land which only the Romans had power to do. So the Pharisees
were trying to trap Jesus into a reply. Christ was very merciful to just write and not say their secret
sins out. Christ wrote the commandments in stone, but He was merciful in writing their sins in the
dust.

What did Jesus do and what is the significance of His response?

John 8:6-8

He didn’t get into a legal or theological debate. He didn’t say anything at all. He got down and
started writing. Application: Sometimes, it’s better not to say anything when you are challenged.
Just keep quiet and let the finger of God do the talking.

What does the finger represent?

Ps 8:3

Involved in Creation.

Exo 31:18

10 commandments.

Matt 12:28

Jesus cast out devils by the Spirit of God.

Luke 11:20

Parallel passage to above verse. Spirit of God = Finger of God.

So who convicted their conscience in verse 9?

John 8:9

The Finger of God = Holy Spirit.

http://breachrepairers.webs.com/ 605
What did Jesus do and what is the significance of His response? He didn’t get into a legal or
theological debate. He didn’t say anything at all. He got down and started writing. Application:
Sometimes, its better not to say anything when you are challenged. Just keep quiet and let the finger
of God do the talking.

8:7 So when they continued asking him, he lifted up himself, and said unto them, He that is without sin
among you, let him first cast a stone at her.

What is the prerequisite to them that can cast the first stone: They must be without sin.

Rom 8:3

Jesus condemned sin in the flesh. The weakness of the law was in the flesh, not the law. In order to
condemn sin, you have to lead a perfect life. Jesus kept the law perfectly, and so He was able to
condemn sin in the flesh. If you think you are ok, then you haven’t studied the life of Jesus, because
His life is perfect.

8:8 And again he stooped down, and wrote on the ground.

8:9 And they which heard it, being convicted by their own conscience, went out one by one, beginning at
the eldest, even unto the last: and Jesus was left alone, and the woman standing in the midst.

Why did it begin with the eldest?

Eze 9:6

Judgment begins at the house of God. Then they began at the ancient men. Therefore, judgment will
begin with the oldest and those that have the greatest accountability. In the parable of the talents,
he started reckoning with the man that had the most talents. Age is important because those that
live the longest have the most opportunity to know the truth.

8:10 When Jesus had lifted up himself, and saw none but the woman, he said unto her, Woman, where
are those thine accusers? hath no man condemned thee?

How did Jesus get to be alone with the woman?

By allowing the Holy Spirit to convict the Pharisees and Scribes. We need to allow God to remove
those who judge so that we can help the sinner. Rather than debating with those who judge, we
should let the Spirit of God handle them.

Spiritual application (optional, for Adventists): At the end of time, the Spirit of God through the
latter rain will cause those with a judging spirit to leave. It will leave Jesus and the woman alone.
Who does the woman represent? Church.

8:11 She said, No man, Lord. And Jesus said unto her, Neither do I condemn thee: go, and sin no more.

DA 462 This was to her the beginning of a new life, a life of purity and peace, devoted to the service
of God. In the uplifting of this fallen soul, Jesus performed a greater miracle than in healing the most
grievous physical disease; He cured the spiritual malady which is unto death everlasting. This
penitent woman became one of His most steadfast followers. With self-sacrificing love and devotion
she repaid His forgiving mercy.

http://breachrepairers.webs.com/ 606
Matt 12:20

What does it mean that “bruised reed shall he not break.”It was a bruised reed already. It is
about to fall or break off. But Jesus would not break it. This woman was in this position,
expecting to die with public humiliation. You never know how close a person is to being
broken. We must not quench the smoking flax or break the bruised reed. If Jesus had
rebuked Peter, he may have been broken. But because Jesus didn’t, Peter became one of the
most devoted disciples, dying for Jesus.

What was Christ’s solution to the situation? How can we apply this to our own situations
today?

“Neither do I condemn thee” – How do we apply this to ourselves? We need to learn to look at
others, no matter how bad a crime they have done, as a fellow sinner. We ourselves are sinners. If
Christ didn’t condemn her, neither do we have a right to condemn others. It also means looking at
them as someone we want to help, rather than accuse (condemn). Many times, when a person falls,
we want to go over all their past rather than dealing with the problem.

“Go and sin no more” – How about this line? What does this mean? We need to encourage those
whom we are helping to sin no more. In order for us to do this, we need to spend the time to show
them. If we just say to them “Neither do I condemn thee” and just leave them, then they will just fall
back into sin. There is no substance to our words.

The natural question that people will ask if we say this is “How?” It’s not easy to go and sin no more.
Many times, we need help for healing. So this involves many countless nights praying for them,
spending time together. Being there for each other. This is what it means to ask people to “Go and
sin no more”. So be prepared to sacrifice for them, just as Christ sacrificed for the woman. The only
reason why He could say that is because He was about to die on the cross for her. We also need to
point them to Jesus and His sacrifice for them. Let His love and forgiveness overwhelm them.

How was Christ able to say “Neither do I condemn thee?” He would sacrifice for the woman.
There is no forgiveness without sacrifice. After complete forgiveness, there is still the command to
Go and sin no more.

Application: We need to be able to sacrifice our time, lives for people if we want to follow in Christ’s
footsteps.

From Jesus’ example, what steps should we take in dealing with people like the woman
caught in adultery?

Lessons:
1. Deal with sensitive situations in private with as few people as possible.
(Jesus waited until all the Pharisees had gone before dealing with the woman)
2. Look to help not to destroy. If Jesus had consented, the Pharisees would have stoned the
woman.
3. Remember our own sins before dealing with other’s sins.
4. Pray that the Holy Spirit will convict the accusers of their sins too.
5. Allow the Holy Spirit to convict them of their sins.

http://breachrepairers.webs.com/ 607
6. Bring them to the feet of Jesus.
7. Learn to utter the words – neither do I condemn thee and Go and sin no more.

8:12 Then spake Jesus again unto them, saying, I am the light of the world: he that followeth me shall not
walk in darkness, but shall have the light of life.

The words that Jesus spoke, are they in direct contrast to something that just happened in that
chapter? What is in chapter 8 and 9 that deals with light and darkness? If you are in darkness, then
you are blind, you cannot see. Jesus gave evidence that He was divine, by healing a man born blind.

8:13 The Pharisees therefore said unto him, Thou bearest record of thyself; thy record is not true.

8:14 Jesus answered and said unto them, Though I bear record of myself, yet my record is true: for I
know whence I came, and whither I go; but ye cannot tell whence I come, and whither I go.

8:15 Ye judge after the flesh; I judge no man.

8:16 And yet if I judge, my judgment is true: for I am not alone, but I and the Father that sent me.

8:17 It is also written in your law, that the testimony of two men is true.

8:18 I am one that bear witness of myself, and the Father that sent me beareth witness of me.

8:19 Then said they unto him, Where is thy Father? Jesus answered, Ye neither know me, nor my Father:
if ye had known me, ye should have known my Father also.

8:20 These words spake Jesus in the treasury, as he taught in the temple: and no man laid hands on him;
for his hour was not yet come.

8:21 Then said Jesus again unto them, I go my way, and ye shall seek me, and shall die in your sins:
whither I go, ye cannot come.

8:22 Then said the Jews, Will he kill himself? because he saith, Whither I go, ye cannot come.

8:23 And he said unto them, Ye are from beneath; I am from above: ye are of this world; I am not of this
world.

8:24 I said therefore unto you, that ye shall die in your sins: for if ye believe not that I am he, ye shall die
in your sins.

8:25 Then said they unto him, Who art thou? And Jesus saith unto them, Even the same that I said unto
you from the beginning.

8:26 I have many things to say and to judge of you: but he that sent me is true; and I speak to the world
those things which I have heard of him.

8:27 They understood not that he spake to them of the Father.

8:28 Then said Jesus unto them, When ye have lifted up the Son of man, then shall ye know that I am he,
and that I do nothing of myself; but as my Father hath taught me, I speak these things.

8:29 And he that sent me is with me: the Father hath not left me alone; for I do always those things that
please him.

http://breachrepairers.webs.com/ 608
8:30 As he spake these words, many believed on him.

8:31 Then said Jesus to those Jews which believed on him, If ye continue in my word, then are ye my
disciples indeed;

If you “continue” in my word. You must continue in the truth. This freedom is referring to freedom
from sin (Rom 7).

8:32 And ye shall know the truth, and the truth shall make you free.

8:33 They answered him, We be Abraham's seed, and were never in bondage to any man: how sayest
thou, Ye shall be made free?

8:34 Jesus answered them, Verily, verily, I say unto you, Whosoever committeth sin is the servant of sin.

8:35 And the servant abideth not in the house for ever: but the Son abideth ever.

8:36 If the Son therefore shall make you free, ye shall be free indeed.

8:37 I know that ye are Abraham's seed; but ye seek to kill me, because my word hath no place in you.

8:38 I speak that which I have seen with my Father: and ye do that which ye have seen with your father.

8:39 They answered and said unto him, Abraham is our father. Jesus saith unto them, If ye were
Abraham's children, ye would do the works of Abraham.

8:40 But now ye seek to kill me, a man that hath told you the truth, which I have heard of God: this did
not Abraham.

8:41 Ye do the deeds of your father. Then said they to him, We be not born of fornication; we have one
Father, even God.

8:42 Jesus said unto them, If God were your Father, ye would love me: for I proceeded forth and came
from God; neither came I of myself, but he sent me.

8:43 Why do ye not understand my speech? even because ye cannot hear my word.

8:44 Ye are of your father the devil, and the lusts of your father ye will do. He was a murderer from the
beginning, and abode not in the truth, because there is no truth in him. When he speaketh a lie, he
speaketh of his own: for he is a liar, and the father of it.

8:45 And because I tell you the truth, ye believe me not.

8:46 Which of you convinceth me of sin? And if I say the truth, why do ye not believe me?

8:47 He that is of God heareth God's words: ye therefore hear them not, because ye are not of God.

8:48 Then answered the Jews, and said unto him, Say we not well that thou art a Samaritan, and hast a
devil?

8:49 Jesus answered, I have not a devil; but I honour my Father, and ye do dishonour me.

8:50 And I seek not mine own glory: there is one that seeketh and judgeth.

http://breachrepairers.webs.com/ 609
8:51 Verily, verily, I say unto you, If a man keep my saying, he shall never see death.

8:52 Then said the Jews unto him, Now we know that thou hast a devil. Abraham is dead, and the
prophets; and thou sayest, If a man keep my saying, he shall never taste of death.

8:53 Art thou greater than our father Abraham, which is dead? and the prophets are dead: whom
makest thou thyself?

8:54 Jesus answered, If I honour myself, my honour is nothing: it is my Father that honoureth me; of
whom ye say, that he is your God:

8:55 Yet ye have not known him; but I know him: and if I should say, I know him not, I shall be a liar like
unto you: but I know him, and keep his saying.

8:56 Your father Abraham rejoiced to see my day: and he saw it, and was glad.

Jesus was basically announcing that He was eternal. How did Abraham see Jesus’ day? Jesus
illustrated to Abraham through the experience with Isaac. The location where Abraham sacrificed
Isaac was where Jesus was crucified and was also where the temple was to be.

8:57 Then said the Jews unto him, Thou art not yet fifty years old, and hast thou seen Abraham?

8:58 Jesus said unto them, Verily, verily, I say unto you, Before Abraham was, I am.

8:59 Then took they up stones to cast at him: but Jesus hid himself, and went out of the temple, going
through the midst of them, and so passed by.

http://breachrepairers.webs.com/ 610
Chapter 9 – The Man Born Blind
Chapter Outline

9:1 And as Jesus passed by, he saw a man which was blind from his birth.

9:2 And his disciples asked him, saying, Master, who did sin, this man, or his parents, that he was born
blind?

According to verse 2, how did the Jews view sin and suffering?

It was generally believed by the Jews that sin is punished in this life. Every affliction was regarded
as the penalty of some wrongdoing, either of the sufferer himself or of his parents. Many attributed
it to be a punishment from God, that they were born in sin. NOTE: story of Job shows us that
affliction is not always God sends punishments for sin.

Nobody is born a sinner. Original sin is used most in Ps 51:5

Rom 1:3

If David was born a sinner, and Jesus was made according to the flesh, then Jesus was also born a
sinner. Sin is only sin when we know about it.

John 9:41

This is referring to the knowledge of the sin. In times of ignorance, God winked.

Deut 1:39

This indicates that because they didn’t know, God allowed them to go into the land of Canaan. Until
they know, then it is not imputed to them as sin.

9:3 Jesus answered, Neither hath this man sinned, nor his parents: but that the works of God should be
made manifest in him.

What was Jesus’ reply? What lessons can we learn?

Christ replied that neither parents nor blind man had sinned. Sickness is not necessarily a result of
an individual’s or their parent’s personal sins. Lesson: Sometimes God allows sickness to occur so
that He can shine through the situation.

9:4 I must work the works of him that sent me, while it is day: the night cometh, when no man can work.

9:5 As long as I am in the world, I am the light of the world.

Why did Jesus say “I am the light of the world”?

2 Cor 4:6

Jesus is the light, through which we can have more knowledge of the glory of God. Key word is
knowledge. Therefore, while Christ was in the world, his actions gave us greater knowledge of the

http://breachrepairers.webs.com/ 611
glory of God.

How does the light shine now that Jesus is no longer in the world?

2 Cor 4:4

Through the glorious gospel – record of His life.

Matt 5:14

God’s people who live the gospel life will be a light for the world. Are we shining for Christ to keep
the light of this world alive?

9:6 When he had thus spoken, he spat on the ground, and made clay of the spittle, and he anointed the
eyes of the blind man with the clay,

Jesus healed by speaking. But why did He use clay with spittle? Was it any less of a miracle? He was
illustrating the combination with natural means with the miracle will be performed in the latter
days.

9:7 And said unto him, Go, wash in the pool of Siloam, (which is by interpretation, Sent.) He went his way
therefore, and washed, and came seeing.

What was involved in healing the blind man? What did Jesus do? What did the blind man
have to do?

Process: Jesus spits in the clay and makes spittle for the blind man. Jesus applies it to his eye.
Commands the man to wash in water. The man obeys and he is healed.

Application: If we want to be healed from spiritual blindness, there are two parts. Jesus’ part –
making the spittle. Our part – going to the pool of Siloam to wash.

What does the clay and spittle represent?

Rev 3:18

Church of Laodicea is also considered blind. Both are anointed with something. Blind man with clay
+ spittle AND Laodicean church with eyesalve. Therefore, clay + spittle = eyesalve. What is the
eyesalve? According to 1 Sam 9:9 – a prophet is someone who helps you see spiritually. Rom 1:2 –
the writings of the prophets are contained in the scriptures. So how does a prophet help us see?
Through His writings.

How are we washed?

Eph 5:26

We are washed by the word = Bible. Therefore: Christ provides the eyesalve – the Bible. However, it
is our choice to then wash ourselves – also using the Word. Therefore, the Word is inspired by God,
but it is useless until we use it to wash our spiritual eyes.

9:8 ¶ The neighbours therefore, and they which before had seen him that he was blind, said, Is not this he

http://breachrepairers.webs.com/ 612
that sat and begged?

9:9 Some said, This is he: others said, He is like him: but he said, I am he.

9:10 Therefore said they unto him, How were thine eyes opened?

9:11 He answered and said, A man that is called Jesus made clay, and anointed mine eyes, and said unto
me, Go to the pool of Siloam, and wash: and I went and washed, and I received sight.

9:12 Then said they unto him, Where is he? He said, I know not.

9:13 ¶ They brought to the Pharisees him that aforetime was blind.

9:14 And it was the sabbath day when Jesus made the clay, and opened his eyes.

9:15 Then again the Pharisees also asked him how he had received his sight. He said unto them, He put
clay upon mine eyes, and I washed, and do see.

9:16 Therefore said some of the Pharisees, This man is not of God, because he keepeth not the sabbath
day. Others said, How can a man that is a sinner do such miracles? And there was a division among
them.

How did the Pharisees react to the healing?

They were not happy. They accused Jesus of being not of God because He healed on the Sabbath.

Based on the question before, why do you think Jesus healed the man in such a complicated
way?

In other words, why didn’t Christ just speak the words? Why spit and make eyesalve, then ask the
man to wash? All these were forbidden activities on the Sabbath. Jesus agitated the issue to teach
them about true Sabbath.

9:17 They say unto the blind man again, What sayest thou of him, that he hath opened thine eyes? He
said, He is a prophet.

What does the blind man think of Jesus? He believes Jesus is a prophet. Such a simple faith. It’s
hard for him to deny what Jesus has done for him. He was blind, now he is healed. Application: It’s
hard for others to argue with what Jesus has done for you. Nobody can argue with a personal
testimony.

9:18 But the Jews did not believe concerning him, that he had been blind, and received his sight, until
they called the parents of him that had received his sight.

Do the Pharisees believe? Contrast this with the blind man’s belief.

John 9:18-38

No. They keep justifying their unbelief even after proof from the parents. They get more and more
angry until they finally cast out the blind man.

Contrast: See the contrast between the blind man’s simple faith and belief in Jesus as a prophet and
then worshipping him as Son of God in v.35-38 – it just grows and grows. vs. the Pharisees unbelief

http://breachrepairers.webs.com/ 613
which has to be supported by more and more ridiculous claims until all they can do is cast the man
out.

Application: This proves that miracles will not help to change people’s minds to believe God. We
must exercise simple faith and accept the things that God is trying to teach us.

Why couldn’t the Pharisees accept what Jesus had done? What was the main issue that Jesus
was dealing with?

John 9:16

Why couldn’t the Pharisees accept what Jesus had done? Because Jesus had changed their
concept of how to keep Sabbath. They could not accept what Jesus was showing them because it
meant changing their traditions. Even a miracle that proved Jesus was Son of God could not
persuade them. Why? Because the Pharisees were the teachers of the law. It would be humiliating
for them to accept change.

What was the main issue that Jesus was dealing with? The concept of how to keep the Sabbath.
Jesus purposely healed the man in a way that would contradict the Pharisees view of keeping the
Sabbath.

Application Question: How does this apply to us? Are there traditions or beliefs we have learnt
growing up that God is showing us we need to change right now? Will we be like the Pharisees,
thinking we are smarting than God? Will we look for ways to reject the truth that is being given to
us?
Some of us may have grown up as Buddhist, Hindhu or even as Christian but not understood
everything from the Bible. I hope that as you hear God’s word and see changes in your life, you will
be like the blind man, rather than the Pharisees.

What methods did Jesus use to reach blind man vs. the Pharisees in this encounter?

How did Jesus reach the blind man? By healing him (medical missionary work). Who does the blind
man represent? Those who are helpless, in need, disabled, poor, uneducated.

How did Jesus try to reach the Pharisees? By the testimony of the blind man. NOTE: Jesus heals
this man so that he can be a witness to the Pharisees. Christ used a blind man with little or no
education to speak to the learned Pharisees. Reminds us of 1 Cor 1:27 – God has chosen to use the
foolish things to confound the wise. Why? Because not even a theologian cannot argue with a
changed life unless their heart is hardened against God. Who do the Pharisees represent? Teachers
of the law, learned men, accomplished, leaders of society.

What can we learn from this? Many times we try to aim for those high in society – learned people
because we think we can reach them through logic or reasoning. However, in this story, we see that
the best strategy for reaching the community is to find those in most need and help them with
physical healing. Then they will be an unwavering testimony to the learned ones. When a blind,
uneducated man can speak with power and authority… that has much more effect than a person
who is a teacher of the law speaking. So the process is, help people with greatest physical and
spiritual needs, then turn them into a witness to their own community.

9:19 And they asked them, saying, Is this your son, who ye say was born blind? how then doth he now

http://breachrepairers.webs.com/ 614
see?

9:20 His parents answered them and said, We know that this is our son, and that he was born blind:

9:21 But by what means he now seeth, we know not; or who hath opened his eyes, we know not: he is of
age; ask him: he shall speak for himself.

9:22 These words spake his parents, because they feared the Jews: for the Jews had agreed already, that
if any man did confess that he was Christ, he should be put out of the synagogue.

9:23 Therefore said his parents, He is of age; ask him.

9:24 Then again called they the man that was blind, and said unto him, Give God the praise: we know
that this man is a sinner.

9:25 He answered and said, Whether he be a sinner or no, I know not: one thing I know, that, whereas I
was blind, now I see.

9:26 Then said they to him again, What did he to thee? how opened he thine eyes?

9:27 He answered them, I have told you already, and ye did not hear: wherefore would ye hear it again?
will ye also be his disciples?

9:28 Then they reviled him, and said, Thou art his disciple; but we are Moses' disciples.

9:29 We know that God spake unto Moses: as for this fellow, we know not from whence he is.

9:30 The man answered and said unto them, Why herein is a marvellous thing, that ye know not from
whence he is, and yet he hath opened mine eyes.

9:31 Now we know that God heareth not sinners: but if any man be a worshipper of God, and doeth his
will, him he heareth.

9:32 Since the world began was it not heard that any man opened the eyes of one that was born blind.

9:33 If this man were not of God, he could do nothing.

9:34 They answered and said unto him, Thou wast altogether born in sins, and dost thou teach us? And
they cast him out.

Those that are going to be sharing the truth and what Christ has done, they will be cast out.

9:35 Jesus heard that they had cast him out; and when he had found him, he said unto him, Dost thou
believe on the Son of God?

9:36 He answered and said, Who is he, Lord, that I might believe on him?

9:37 And Jesus said unto him, Thou hast both seen him, and it is he that talketh with thee.

9:38 And he said, Lord, I believe. And he worshipped him.

9:39 ¶ And Jesus said, For judgment I am come into this world, that they which see not might see; and
that they which see might be made blind.

http://breachrepairers.webs.com/ 615
They really didn’t believe that Jesus was the Messiah. They had evidence, but they were really
spiritually blind and rejected Him.

9:40 And some of the Pharisees which were with him heard these words, and said unto him, Are we blind
also?

9:41 Jesus said unto them, If ye were blind, ye should have no sin: but now ye say, We see; therefore your
sin remaineth.

http://breachrepairers.webs.com/ 616
Chapter 10 – The true Shepard
Chapter Outline
10:1 Verily, verily, I say unto you, He that entereth not by the door into the sheepfold, but climbeth up
some other way, the same is a thief and a robber.

The thief and robber always tends towards to leading them away from God.

10:2 But he that entereth in by the door is the shepherd of the sheep.

The sheep hear his voice. Each one of us need to hear the voice of God for ourselves.

10:3 To him the porter openeth; and the sheep hear his voice: and he calleth his own sheep by name, and
leadeth them out.

10:4 And when he putteth forth his own sheep, he goeth before them, and the sheep follow him: for they
know his voice.

10:5 And a stranger will they not follow, but will flee from him: for they know not the voice of strangers.

10:6 This parable spake Jesus unto them: but they understood not what things they were which he spake
unto them.

10:7 Then said Jesus unto them again, Verily, verily, I say unto you, I am the door of the sheep.

Why would Jesus describe Himself as the door? A door is not only a means of keeping things in but
also of keeping things out. In a salvational sense, when Jesus is the door, the only way to enter in is
through Him.

10:8 All that ever came before me are thieves and robbers: but the sheep did not hear them.

10:9 I am the door: by me if any man enter in, he shall be saved, and shall go in and out, and find
pasture.

"I am the door: by me if any man enter in, he shall be saved" – How do we enter in?

Matt 7:13-14

"narrow is the way" Jesus said John 14:6 "I am the way" so Jesus is saying "I am the narrow way"
and this way leads to life, and few there be that find it. Parallel: a lot of people don’t want to live a
self-sacrificing life, etc (Dress reform, health reform)

Jer 17:20-21

By bearing no burdens on the Sabbath, and resting in Christ (Christ is the gate to Jerusalem, those
who enter the new Jerusalem shall be saved)

Matt 11:28-30

Burden is light

TMK 82 "By me," said Christ, "if any man enter in, he shall be saved, and shall go in and out, and find

http://breachrepairers.webs.com/ 617
pasture" (John 10:9). Let earth be glad, let the inhabitants of the world rejoice, that Christ has
bridged the gulf which sin had made, and has bound earth and heaven together. A highway has been
cast up for the ransomed of the Lord. The weary and heavy laden may come unto Him and find rest to
their souls.

Jer 22:2-4

We enter in Christ, be executing (upholding) judgment, righteousness, by taking care of strangers,


widows, the fatherless, and then we will enter in New Jerusalem riding in Chariots, etc

Heb 3:19

Enter in by belief (not unbelief) which is faith. Who enters in?

Isa 26:2 Open ye the gates, that the righteous nation which keepeth the truth may enter in.

What is truth?

Psa 119:151 …commandments are truth…

Connect: Revelation 14:12 Here is the patience of the saints: here are they that keep the
commandments of God, and the faith of Jesus. So those who enter in the gate, keep the
commandments of God & the faith of Jesus

Exo 33:19 I will proclaim the name of the Lord before thee.

Exo 34:6 …and proclaimed…truth

Connect: Revelation 14:1 144,000 "Father's name written in their foreheads." The 144,000 have a
close, intimate, strict, rigorous, walk with Christ, and because of this they have entered into Christ,
and will receive everlasting life. What else is truth?

John 14:6

Jesus

John 17:17

Word of God

John 16:13

Spirit is truth

John 8:32

Truth will make you free, from?

Rom 6:18

Made free from sin become servants

http://breachrepairers.webs.com/ 618
Rev 1:1

Servants understand book of Revelation, which is a revelation of Jesus, so they have a more
complete understanding of the character of Christ, and this enables them to give the loud cry.

John 10:16 …they shall hear my voice.

Rev18:1-4

Will recognize the loud cry as the voice of Jesus, saying come out of her and join my fold. The loud
cry will be given by the publishing work and 144,000

John 10:9 …shall go in and out, and find pasture…

Jesus = Shepard

His people = flock

Spiritually speaking how is the flock feed? Through the Word of God. It is like we come to Jesus
(6:53 – the flock must eat the flesh and drink the blood of Christ in order to have life") and we eat,
this sustains us spiritually and we go out, and share.

Gen 47:1-4

"Land of Canaan" was the promised land, it really refers to the New Jerusalem, that will be here
after the millennium. But right now the "land of Canaan" is this earth, they are saying there is going
to be a famine in the land (Amos 8:11), and their will be no pasture (word of God) for the world.

Para: soon there will be a famine and only those who have been eating the flesh and drinking the
blood of Christ will have pasture: Is bread and water will be sure (Is 33:16) because they will dwell
in the land of Goshen (drawing near); spiritually they will be so near to Christ, that when the famine
comes on this earth 'land of Canaan' they will still be feed spiritually. Jesus is the bread & water.
Powerful!! Tighten up

John 10:12 wolf catcheth them, and scattereth the sheep.

Psa 100:3

Ezek 34:31

Sheep = God's people

Jer 23:1-2

Wolves = Pastors. Wolves are the Pastors that don't feed the people of God, or visit the people of
God. (Jer 23:1-6)

Ezek 22:27

They destroy the people of God for money, they don't care about anything but a paycheck, who

http://breachrepairers.webs.com/ 619
cares if they understand what we believe as a people, who cares if they don't understand that Ellen
G. White is a prophet of God, let them come to church with jewelry and tight skirts, let them play
jazz, rap, and rock 'n' roll in the church , let them eat what they want to eat, who cares? Give me a
paycheck.

Matt 7:15

(additional): Wolves = false prophets

Acts 20:29-30

“Wolves” – those who come in teaching perverse things, new theology, no sanctuary, Jesus took the
nature of Adam before the fall, we can sin until Jesus comes, to draw away disciples after them. Who
is the hireling? Tasks that are considered menial or offensive: health reform, dress reform, not
going to the movies, not cursing, yet they are miserable, they feel like if they continue they will win
heaven. The hireling is those caught up in a works religion, they are trying to earn heaven (depart
from me I never knew you) Why isn't the Pastor considered the hireling? Because Christ is making a
distinction between the True Shepard (Himself) and the hireling (one trying to work to gain a
reward). It shows that the one who receive such rich blessings from God, should be concerned over
the works of the church, what Pastor comes in, what they preach. But he is trying to earn salvation
so he really isn't concerned about the rest. Study more: could be the Elders too!

Things to still study. Righteousness is free from sin, they are justified and sanctified. Christ was
made sin for them that they may receive righteousness. Righteous nation who are they?

1 Pet 2:9 Peculiar people, royal priesthood

Role of a priest is intercession for sins of the people, weeping between the porch and the altar,
those who are preparing to be sealed.

10:10 The thief cometh not, but for to steal, and to kill, and to destroy: I am come that they might have
life, and that they might have it more abundantly.

Every false teacher is to bring away, to take away into misery. What God wants us to do is to
experience the fullness of health, happiness and life.

10:11 I am the good shepherd: the good shepherd giveth his life for the sheep.

10:12 But he that is an hireling, and not the shepherd, whose own the sheep are not, seeth the wolf
coming, and leaveth the sheep, and fleeth: and the wolf catcheth them, and scattereth the sheep.

10:13 The hireling fleeth, because he is an hireling, and careth not for the sheep.

There are 2 contrasts, a hireling versus a shepherd. Both take care of the sheep. Difference:
Hireling: When problems come, they leave. Shepherd: But we can be shepherds as well which stay
to protect the sheep. What are you today? A shepherd or a hireling? Some are in it because they are
getting some kind of money or worldly esteem. Some do it because they are told to, and others do it
because they love God.

10:14 I am the good shepherd, and know my sheep, and am known of mine.

10:15 As the Father knoweth me, even so know I the Father: and I lay down my life for the sheep.

http://breachrepairers.webs.com/ 620
10:16 And other sheep I have, which are not of this fold: them also I must bring, and they shall hear my
voice; and there shall be one fold, and one shepherd.

If we apply the fold to the church, in this time and this age, the other sheep would be found not of
the SDA church. God is looking for shepherds who would call them out.

10:17 Therefore doth my Father love me, because I lay down my life, that I might take it again.

If Jesus didn’t want to die, no one could have stopped Him from doing it. The divinity of Christ it
seems that He has the power to do it, but yet not so in the next verse:

John 14:28

We know that they are all God, but it seems that Jesus is also applying the principle by let man
esteem the other better than himself. But we know that within Christ, He has the power of life in
Himself.

10:18 No man taketh it from me, but I lay it down of myself. I have power to lay it down, and I have
power to take it again. This commandment have I received of my Father.

10:19 There was a division therefore again among the Jews for these sayings.

10:20 And many of them said, He hath a devil, and is mad; why hear ye him?

10:21 Others said, These are not the words of him that hath a devil. Can a devil open the eyes of the
blind?

10:22 And it was at Jerusalem the feast of the dedication, and it was winter.

10:23 And Jesus walked in the temple in Solomon's porch.

10:24 Then came the Jews round about him, and said unto him, How long dost thou make us to doubt? If
thou be the Christ, tell us plainly.

10:25 Jesus answered them, I told you, and ye believed not: the works that I do in my Father's name, they
bear witness of me.

10:26 But ye believe not, because ye are not of my sheep, as I said unto you.

10:27 My sheep hear my voice, and I know them, and they follow me:

10:28 And I give unto them eternal life; and they shall never perish, neither shall any man pluck them
out of my hand.

This passage it used to advocate the false doctrine of once saved always saved. Verse 28 is used to
advocate this false doctrine.

John 10:27

“follow” – it is continual

Luke 8:12-13

http://breachrepairers.webs.com/ 621
They believed at one point but fell away.

Heb 10:38

What is another term for just? Righteous. You cannot draw back if you are already lost. So at one
point in time, this person must have been saved.

Luke 12:42-46

What indicates that that second servant was faithful at one time? He was given authority in that
household. But then he fell away.

Matt 18:23-35

Verse 32-34. This shows that forgiveness is conditional and the sins are only covered until it is
blotted out.

Heb 6:4

You can say for sure that they were saved and sealed with the Spirit.

Heb 6:6

This statement is quite plain. They were partakers of the Holy Spirit and were once enlightened, but
they fell away.

1 Tim 4:1

“some shall depart from the faith” indicating that they were once in the faith.

Rev 2:4, 5

Thos has left thy first love. Repent and do the first works.

Matt 24:13

Why endure if you are already saved?

1 John 1:7

What need is there of walking in the light if we have already been saved (this may be what a person
may ask who believes in once saved always saved)

Rom 11:22

This shows the idea of continuing.

2 Pet 1:10

10:29 My Father, which gave them me, is greater than all; and no man is able to pluck them out of my

http://breachrepairers.webs.com/ 622
Father's hand.

10:30 I and my Father are one.

10:31 Then the Jews took up stones again to stone him.

10:32 Jesus answered them, Many good works have I shewed you from my Father; for which of those
works do ye stone me?

10:33 The Jews answered him, saying, For a good work we stone thee not; but for blasphemy; and
because that thou, being a man, makest thyself God.

10:34 Jesus answered them, Is it not written in your law, I said, Ye are gods?

10:35 If he called them gods, unto whom the word of God came, and the scripture cannot be broken;

10:36 Say ye of him, whom the Father hath sanctified, and sent into the world, Thou blasphemest;
because I said, I am the Son of God?

10:37 If I do not the works of my Father, believe me not.

10:38 But if I do, though ye believe not me, believe the works: that ye may know, and believe, that the
Father is in me, and I in him.

10:39 Therefore they sought again to take him: but he escaped out of their hand,

10:40 And went away again beyond Jordan into the place where John at first baptized; and there he
abode.

10:41 And many resorted unto him, and said, John did no miracle: but all things that John spake of this
man were true.

10:42 And many believed on him there.

http://breachrepairers.webs.com/ 623
Chapter 11 – The Ressurection and the Life
It was the raising of Lazarus that was to culminate to the death of Jesus.

Chapter Outline

11:1 Now a certain man was sick, named Lazarus, of Bethany, the town of Mary and her sister Martha.

11:2 (It was that Mary which anointed the Lord with ointment, and wiped his feet with her hair, whose
brother Lazarus was sick.)

11:3 Therefore his sisters sent unto him, saying, Lord, behold, he whom thou lovest is sick.

Lazarus was very close to Jesus.

DA 524 The Saviour blessed all who sought His help; He loves all the human family, but to some He is
bound by peculiarly tender associations.

Jesus was closer to John, James and Peter. Why was He able to be closer to them? They sought Jesus
more. God can love us, the more that we love Him.

11:4 When Jesus heard that, he said, This sickness is not unto death, but for the glory of God, that the Son
of God might be glorified thereby.

11:5 Now Jesus loved Martha, and her sister, and Lazarus.

11:6 When he had heard therefore that he was sick, he abode two days still in the same place where he
was.

11:7 Then after that saith he to his disciples, Let us go into Judaea again.

11:8 His disciples say unto him, Master, the Jews of late sought to stone thee; and goest thou thither
again?

11:9 Jesus answered, Are there not twelve hours in the day? If any man walk in the day, he stumbleth not,
because he seeth the light of this world.

11:10 But if a man walk in the night, he stumbleth, because there is no light in him.

11:11 These things said he: and after that he saith unto them, Our friend Lazarus sleepeth; but I go, that
I may awake him out of sleep.

11:12 Then said his disciples, Lord, if he sleep, he shall do well.

11:13 Howbeit Jesus spake of his death: but they thought that he had spoken of taking of rest in sleep.

11:14 Then said Jesus unto them plainly, Lazarus is dead.

11:15 And I am glad for your sakes that I was not there, to the intent ye may believe; nevertheless let us
go unto him.

11:16 Then said Thomas, which is called Didymus, unto his fellowdisciples, Let us also go, that we may
die with him.

http://breachrepairers.webs.com/ 624
11:17 Then when Jesus came, he found that he had lain in the grave four days already.

11:18 Now Bethany was nigh unto Jerusalem, about fifteen furlongs off:

11:19 And many of the Jews came to Martha and Mary, to comfort them concerning their brother.

11:20 Then Martha, as soon as she heard that Jesus was coming, went and met him: but Mary sat still in
the house.

11:21 Then said Martha unto Jesus, Lord, if thou hadst been here, my brother had not died.

11:22 But I know, that even now, whatsoever thou wilt ask of God, God will give it thee.

11:23 Jesus saith unto her, Thy brother shall rise again.

11:24 Martha saith unto him, I know that he shall rise again in the resurrection at the last day.

11:25 Jesus said unto her, I am the resurrection, and the life: he that believeth in me, though he were
dead, yet shall he live:

11:26 And whosoever liveth and believeth in me shall never die. Believest thou this?

11:27 She saith unto him, Yea, Lord: I believe that thou art the Christ, the Son of God, which should come
into the world.

11:28 And when she had so said, she went her way, and called Mary her sister secretly, saying, The
Master is come, and calleth for thee.

11:29 As soon as she heard that, she arose quickly, and came unto him.

11:30 Now Jesus was not yet come into the town, but was in that place where Martha met him.

11:31 The Jews then which were with her in the house, and comforted her, when they saw Mary, that she
rose up hastily and went out, followed her, saying, She goeth unto the grave to weep there.

11:32 Then when Mary was come where Jesus was, and saw him, she fell down at his feet, saying unto
him, Lord, if thou hadst been here, my brother had not died.

11:33 When Jesus therefore saw her weeping, and the Jews also weeping which came with her, he
groaned in the spirit, and was troubled,

11:34 And said, Where have ye laid him? They said unto him, Lord, come and see.

11:35 Jesus wept.

11:36 Then said the Jews, Behold how he loved him!

11:37 And some of them said, Could not this man, which opened the eyes of the blind, have caused that
even this man should not have died?

11:38 Jesus therefore again groaning in himself cometh to the grave. It was a cave, and a stone lay upon
it.

11:39 Jesus said, Take ye away the stone. Martha, the sister of him that was dead, saith unto him, Lord,

http://breachrepairers.webs.com/ 625
by this time he stinketh: for he hath been dead four days.

11:40 Jesus saith unto her, Said I not unto thee, that, if thou wouldest believe, thou shouldest see the
glory of God?

11:41 Then they took away the stone from the place where the dead was laid. And Jesus lifted up his eyes,
and said, Father, I thank thee that thou hast heard me.

11:42 And I knew that thou hearest me always: but because of the people which stand by I said it, that
they may believe that thou hast sent me.

11:43 And when he thus had spoken, he cried with a loud voice, Lazarus, come forth.

11:44 And he that was dead came forth, bound hand and foot with graveclothes: and his face was bound
about with a napkin. Jesus saith unto them, Loose him, and let him go.

11:45 Then many of the Jews which came to Mary, and had seen the things which Jesus did, believed on
him.

11:46 But some of them went their ways to the Pharisees, and told them what things Jesus had done.

11:47 Then gathered the chief priests and the Pharisees a council, and said, What do we? for this man
doeth many miracles.

11:48 If we let him thus alone, all men will believe on him: and the Romans shall come and take away
both our place and nation.

11:49 And one of them, named Caiaphas, being the high priest that same year, said unto them, Ye know
nothing at all,

11:50 Nor consider that it is expedient for us, that one man should die for the people, and that the whole
nation perish not.

11:51 And this spake he not of himself: but being high priest that year, he prophesied that Jesus should
die for that nation;

11:52 And not for that nation only, but that also he should gather together in one the children of God
that were scattered abroad.

11:53 Then from that day forth they took counsel together for to put him to death.

11:54 Jesus therefore walked no more openly among the Jews; but went thence unto a country near to
the wilderness, into a city called Ephraim, and there continued with his disciples.

11:55 And the Jews' passover was nigh at hand: and many went out of the country up to Jerusalem
before the passover, to purify themselves.

11:56 Then sought they for Jesus, and spake among themselves, as they stood in the temple, What think
ye, that he will not come to the feast?

11:57 Now both the chief priests and the Pharisees had given a commandment, that, if any man knew
where he were, he should shew it, that they might take him.

http://breachrepairers.webs.com/ 626
Chapter 12 -
Chapter Outline
 Simon's feast (Mary Magdalene's faith) (1-9)
12:1 Then Jesus six days before the passover came to Bethany, where Lazarus was which had been dead,
whom he raised from the dead.

How many days before death of Christ was this event? Six days before Passover which was
when Jesus died.

Who are the different groups that Jesus deals with in this encounter? What do we know from
the Bible about them? NOTE – Compare the passages. Please discuss together.

MARY

John 12:3

John 11:1-5

Luke 7:37, 39

Luke 10:41

She was sister of Lazarus and Martha. Also from Bethany. She liked to sit at Jesus feet. She was
called a sinner in John. It seemed common knowledge that she was a sinner – meaning that it must
have been some public sin that she was performing at the time.

SIMON

Matt 26:6

Luke 7:36

Simon was the host of this feast. What do we know about him? A Leper. Probably healed by Jesus,
otherwise he would not be able to hold a feast at his house and come into contact with so many
people. A Pharisee. Held a feast for Jesus. Lived in Bethany. Father of Judas Iscariot. (John 12:4)

JUDAS

John 12:4.

Disciple of Jesus. The one who ends up betraying him. Selfish and greedy. Simon’s son.

12:2 There they made him a supper; and Martha served: but Lazarus was one of them that sat at the
table with him.

12:3 Then took Mary a pound of ointment of spikenard, very costly, and anointed the feet of Jesus, and
wiped his feet with her hair: and the house was filled with the odour of the ointment.

What happens as Mary begins to apply the ointment? The odor fills the whole room. People in

http://breachrepairers.webs.com/ 627
the room begin to notice.

How did each group respond to Jesus? NOTE – Compare the passages. Please discuss together.

MARY
Mark 14:3-5

Luke 7:37-38

Weeping (she believed Jesus would die soon). Bought an alabaster box of ointment for Christ. Spent
300 pence on a gift for Jesus (or denarius in Greek). This almost 1 year’s wage. How do we know?
Matt 20:2 – a labourer was hired for one penny or denarius per day. Anointed Christ. Sat at Jesus’
feet. Washed Jesus’ feet

SIMON

Luke 7:36, 39

He respected Jesus otherwise he wouldn’t hold a feast for Him. Then he looks down on Jesus for
allowing a ‘sinner’ to wash his feet. This leads to Simon questioning whether Jesus is a prophet.

DISCIPLES & JUDAS

Matt

John 12:4-6

Indignation. Despised Mary’s action. Why? Because they thought Mary was wasting her money on
ointment. Who was the main person influencing their attitude? Judas.What was his motive? Greed.

How did Jesus respond to each group? NOTE – Compare the passages.

MARY

Love and appreciation for her expensive gift to him. Forgiveness for her sins. Memorialized her
actions in the gospel (written in all 4 gospel accounts)

SIMON

Luke 7:40-47

Tells him a parable to rebuke him. Why a parable? He tells him in an indirect way. Cushions the
rebuke with a parable.

DISCIPLES & JUDAS

John 12:4

Open rebuke. What was the result? Judas decided to go and betray Jesus right after this.

Who does each group represent today?

http://breachrepairers.webs.com/ 628
MARY

Those who are repentant, weeping, adoring Christ. Those who are willing to give expensive gifts to
the Lord and his work rather than themselves. Their actions are like a sweet odor that fills the
room. Christ most favors his people who are constantly at his feet, repentant, desiring to anoint
Him, wanting to pay respect and homage.

SIMON

A newer Christian. Someone who just came to Christ. Or one who desired to do good (held the feast
for Jesus). But ultimately he was carried away by the crowd sentiment. He looked down on the
woman because of her past sins. Christ rebuked him and helped him to realize his sins.

DISCIPLES & JUDAS

Those who have been with Christ so long, but lost touch. Those who are proud of their spiritual
position and use it to put others down. Those who have personal agendas, driven by self gain.
Would rather make a spectacle of the sinner who is paying Christ respect. What will be their end
result? Eventually, you could fall away from Christ.

Despite Jesus’ loving rebuke, what did Judas decide to do after this?

Mark 26:14-16

Judas decided to betray Jesus to the chief priests for 30 pieces of silver.

What did Jesus mean by his conclusion in Luke 7:47?

Luke 7:47

Rom 6:1-2

Was Jesus trying to say that the bigger the sinner you are, the more you will feel forgiveness and the
more you will love God? No, Paul says that we should not continue sin just because we have grace.
So the Bible is not talking about going out and sinning more so that we can be forgiven more. Jesus
was talking about our frame of mind. Do we truly understand in our mind how much Christ has
sacrificed for our sin? For those who truly feel the magnitude of their own sins, they will feel deeper
repentance and be more grateful to Christ. It’s all about how you view yourself.

12:4 Then saith one of his disciples, Judas Iscariot, Simon's son, which should betray him,

12:5 Why was not this ointment sold for three hundred pence, and given to the poor?

12:6 This he said, not that he cared for the poor; but because he was a thief, and had the bag, and bare
what was put therein.

12:7 Then said Jesus, Let her alone: against the day of my burying hath she kept this.

12:8 For the poor always ye have with you; but me ye have not always.

http://breachrepairers.webs.com/ 629
12:9 Much people of the Jews therefore knew that he was there: and they came not for Jesus' sake only,
but that they might see Lazarus also, whom he had raised from the dead.

12:10 But the chief priests consulted that they might put Lazarus also to death;

12:11 Because that by reason of him many of the Jews went away, and believed on Jesus.

12:12 On the next day much people that were come to the feast, when they heard that Jesus was coming
to Jerusalem,

12:13 Took branches of palm trees, and went forth to meet him, and cried, Hosanna: Blessed is the King
of Israel that cometh in the name of the Lord.

12:14 And Jesus, when he had found a young ass, sat thereon; as it is written,

12:15 Fear not, daughter of Sion: behold, thy King cometh, sitting on an ass's colt.

12:16 These things understood not his disciples at the first: but when Jesus was glorified, then
remembered they that these things were written of him, and that they had done these things unto him.

12:17 The people therefore that was with him when he called Lazarus out of his grave, and raised him
from the dead, bare record.

12:18 For this cause the people also met him, for that they heard that he had done this miracle.

12:19 The Pharisees therefore said among themselves, Perceive ye how ye prevail nothing? behold, the
world is gone after him.

12:20 And there were certain Greeks among them that came up to worship at the feast:

12:21 The same came therefore to Philip, which was of Bethsaida of Galilee, and desired him, saying, Sir,
we would see Jesus.

12:22 Philip cometh and telleth Andrew: and again Andrew and Philip tell Jesus.

12:23 And Jesus answered them, saying, The hour is come, that the Son of man should be glorified.

12:24 Verily, verily, I say unto you, Except a corn of wheat fall into the ground and die, it abideth alone:
but if it die, it bringeth forth much fruit.

DA 623 And the law of self-sacrifice is the law of self-preservation.

In the vegetable kingdom, in order for something to survive, it must die. Jesus was referring to
Himself. It can also show that we must die to self before we can bring much fruit and connect with
Rom 6 and baptism. Does the act of baptism actually cause the person to experience newness of life
in Jesus? No, only total renunciation of all known sins does that.

12:25 He that loveth his life shall lose it; and he that hateth his life in this world shall keep it unto life
eternal.

We can choose what we have in the after life. We can determine our future now on this earth. God
gives to each one equal opportunity and it is up to us to make use of them.

12:26 If any man serve me, let him follow me; and where I am, there shall also my servant be: if any man

http://breachrepairers.webs.com/ 630
serve me, him will my Father honour.

12:27 Now is my soul troubled; and what shall I say? Father, save me from this hour: but for this cause
came I unto this hour.

DA 624 In anticipation Christ was already drinking the cup of bitterness. His humanity shrank from
the hour of abandonment, when to all appearance He would be deserted even by God, when all would
see Him stricken, smitten of God, and afflicted. He shrank from public exposure, from being treated as
the worst of criminals, from a shameful and dishonored death. A foreboding of His conflict with the
powers of darkness, a sense of the awful burden of human transgression, and the Father's wrath
because of sin caused the spirit of Jesus to faint, and the pallor of death to overspread His
countenance.

12:28 Father, glorify thy name. Then came there a voice from heaven, saying, I have both glorified it, and
will glorify it again.

The whole life of Christ was always glorifying His Father’s name.

12:29 The people therefore, that stood by, and heard it, said that it thundered: others said, An angel
spake to him.

The Greeks didn’t understand what was spoken from Heaven.

DA 625 But the inquiring Greeks saw the cloud, heard the voice, comprehended its meaning, and
discerned Christ indeed; to them He was revealed as the Sent of God.

12:30 Jesus answered and said, This voice came not because of me, but for your sakes.

12:31 Now is the judgment of this world: now shall the prince of this world be cast out.

12:32 And I, if I be lifted up from the earth, will draw all men unto me.

Lifting up signifies His death on the cross. Was Jesus lifted up? Did He draw all men unto Him? We
can’t be drawn against our own will. Jesus draws us to Him through His love and goodness. If we
exalt Jesus, people will be attracted to Him.

12:33 This he said, signifying what death he should die.

12:34 The people answered him, We have heard out of the law that Christ abideth for ever: and how
sayest thou, The Son of man must be lifted up? who is this Son of man?

12:35 Then Jesus said unto them, Yet a little while is the light with you. Walk while ye have the light, lest
darkness come upon you: for he that walketh in darkness knoweth not whither he goeth.

12:36 While ye have light, believe in the light, that ye may be the children of light. These things spake
Jesus, and departed, and did hide himself from them.

12:37 But though he had done so many miracles before them, yet they believed not on him:

12:38 That the saying of Esaias the prophet might be fulfilled, which he spake, Lord, who hath believed
our report? and to whom hath the arm of the Lord been revealed?

12:39 Therefore they could not believe, because that Esaias said again,

http://breachrepairers.webs.com/ 631
12:40 He hath blinded their eyes, and hardened their heart; that they should not see with their eyes, nor
understand with their heart, and be converted, and I should heal them.

12:41 These things said Esaias, when he saw his glory, and spake of him.

12:42 Nevertheless among the chief rulers also many believed on him; but because of the Pharisees they
did not confess him, lest they should be put out of the synagogue:

12:43 For they loved the praise of men more than the praise of God.

In our time today, we could do something for public applause more than for the love of God.

12:44 Jesus cried and said, He that believeth on me, believeth not on me, but on him that sent me.

12:45 And he that seeth me seeth him that sent me.

12:46 I am come a light into the world, that whosoever believeth on me should not abide in darkness.

12:47 And if any man hear my words, and believe not, I judge him not: for I came not to judge the world,
but to save the world.

12:48 He that rejecteth me, and receiveth not my words, hath one that judgeth him: the word that I have
spoken, the same shall judge him in the last day.

12:49 For I have not spoken of myself; but the Father which sent me, he gave me a commandment, what I
should say, and what I should speak.

12:50 And I know that his commandment is life everlasting: whatsoever I speak therefore, even as the
Father said unto me, so I speak.

http://breachrepairers.webs.com/ 632
Chapter 13
Chapter Outline
13:1 Now before the feast of the passover, when Jesus knew that his hour was come that he should
depart out of this world unto the Father, having loved his own which were in the world, he loved them
unto the end.

13:2 And supper being ended, the devil having now put into the heart of Judas Iscariot, Simon's son, to
betray him;

13:3 Jesus knowing that the Father had given all things into his hands, and that he was come from God,
and went to God;

13:4 He riseth from supper, and laid aside his garments; and took a towel, and girded himself.

The ordinance of humility was given verse.

DA 644 Bitter shame and humiliation filled their hearts. They understood the unspoken rebuke, and
saw themselves in altogether a new light.

Characteristics of Judas

DA 644 If there was a highest place, Judas was determined to have it, and that place was thought to
be next to Christ. And Judas was a traitor.

DA 645 Before the Passover Judas had met a second time with the priests and scribes, and had closed
the contract to deliver Jesus into their hands. Yet he afterward mingled with the disciples as though
innocent of any wrong, and interested in the work of preparing for the feast. The disciples knew
nothing of the purpose of Judas. Jesus alone could read his secret. Yet He did not expose him. Jesus
hungered for his soul.

DA 645 When the Saviour's hands were bathing those soiled feet, and wiping them with the towel,
the heart of Judas thrilled through and through with the impulse then and there to confess his sin.
But he would not humble himself. He hardened his heart against repentance; and the old impulses,
for the moment put aside, again controlled him…After seeing Him degrade Himself, as he thought, he
was confirmed in his purpose to disown Him, and confess himself deceived. He was possessed by a
demon, and he resolved to complete the work he had agreed to do in betraying his Lord.

The love of Jesus was so strongly felt to him. The demon that possessed Judas was the demon of
greed. One of the motives that Judas had to betray Jesus was to not humiliate him in public.

13:5 After that he poureth water into a bason, and began to wash the disciples' feet, and to wipe them
with the towel wherewith he was girded.

13:6 Then cometh he to Simon Peter: and Peter saith unto him, Lord, dost thou wash my feet?

13:7 Jesus answered and said unto him, What I do thou knowest not now; but thou shalt know hereafter.

13:8 Peter saith unto him, Thou shalt never wash my feet. Jesus answered him, If I wash thee not, thou
hast no part with me.

13:9 Simon Peter saith unto him, Lord, not my feet only, but also my hands and my head.

http://breachrepairers.webs.com/ 633
13:10 Jesus saith to him, He that is washed needeth not save to wash his feet, but is clean every whit: and
ye are clean, but not all.

What did the foot washing symbolize?

DA 646 He desired by that very act to wash the alienation, jealousy, and pride from their hearts.

13:11 For he knew who should betray him; therefore said he, Ye are not all clean.

13:12 So after he had washed their feet, and had taken his garments, and was set down again, he said
unto them, Know ye what I have done to you?

13:13 Ye call me Master and Lord: and ye say well; for so I am.

13:14 If I then, your Lord and Master, have washed your feet; ye also ought to wash one another's feet.

The blood of Jesus also represents life that comes from His words by partaking of His life.

DA 653 The Passover wine, untouched by fermentation, is on the table. These emblems Christ
employs to represent His own unblemished sacrifice. Nothing corrupted by fermentation, the symbol
of sin and death, could represent the "Lamb without blemish and without spot." 1 Peter 1:19.

13:15 For I have given you an example, that ye should do as I have done to you.

This is why we still wash one another’s feet today.

13:16 Verily, verily, I say unto you, The servant is not greater than his lord; neither he that is sent
greater than he that sent him.

13:17 If ye know these things, happy are ye if ye do them.

13:18 I speak not of you all: I know whom I have chosen: but that the scripture may be fulfilled, He that
eateth bread with me hath lifted up his heel against me.

13:19 Now I tell you before it come, that, when it is come to pass, ye may believe that I am he.

It was to give them faith in Him that He does know the future. That He truly knows the end from the
beginning. That is why we emphasize prophecy as a means to know that we have a God.

Heb 9:16, 17

“testament” – covenant or a will. A will does not go into effect until after that person dies. The
ordinance of humility and Lord’s supper was only effectual after His death, because it was a
commemoration to His death.

1 Cor 11:23-26

It is clear here that the bread and the wine represent His body and blood broken for us.

DA 650 This ordinance is Christ's appointed preparation for the sacramental service. While pride,
variance, and strife for supremacy are cherished, the heart cannot enter into fellowship with Christ.
We are not prepared to receive the communion of His body and His blood. Therefore it was that Jesus
appointed the memorial of His humiliation to be first observed.

http://breachrepairers.webs.com/ 634
What is the practical way that we are partakers of Him? By His Word.

13:20 Verily, verily, I say unto you, He that receiveth whomsoever I send receiveth me; and he that
receiveth me receiveth him that sent me.

13:21 When Jesus had thus said, he was troubled in spirit, and testified, and said, Verily, verily, I say unto
you, that one of you shall betray me.

DA 653 These words convinced the false disciple that Christ read his secret purpose. Now Christ
spoke out more plainly. As they were seated at the table He said, looking upon His disciples, "I speak
not of you all: I know whom I have chosen: but that the scripture may be fulfilled, He that eateth
bread with Me hath lifted up his heel against Me.

DA 654 A cloud settled over them all, a premonition of some dreadful calamity, the nature of which
they did not understand. As they ate in silence, Jesus said, "Verily I say unto you, that one of you shall
betray Me." At these words amazement and consternation seized them.

DA 654 Amid the confusion of questions and expressions of astonishment, Judas had not heard the
words of Jesus in answer to John's question. But now, to escape the scrutiny of the disciples, he asked
as they had done, "Master, is it I?" Jesus solemnly replied, "Thou hast said."

DA 654 In surprise and confusion at the exposure of his purpose, Judas rose hastily to leave the
room. "Then said Jesus unto him, That thou doest, do quickly. . . . He then having received the sop
went immediately out: and it was night." Night it was to the traitor as he turned away from Christ
into the outer darkness.

Why would Jesus say that thou doest, do quickly? To give him still that chance to repent.

DA 655 But although surprised and alarmed at the discovery of his guilt, Judas became only the more
determined. From the sacramental supper he went out to complete the work of betrayal.

DA 655 Nothing that could be done to save Judas had been left undone.

The reason why a person will be lost at the end of time will be their own fault.

DA 655 Though Jesus knew Judas from the beginning, He washed his feet. And the betrayer was
privileged to unite with Christ in partaking of the sacrament. A long-suffering Saviour held out every
inducement for the sinner to receive Him, to repent, and to be cleansed from the defilement of sin.
This example is for us. When we suppose one to be in error and sin, we are not to divorce ourselves
from him. By no careless separation are we to leave him a prey to temptation, or drive him upon
Satan's battleground. This is not Christ's method. It was because the disciples were erring and faulty
that He washed their feet, and all but one of the twelve were thus brought to repentance.

DA 656 Christ's example forbids exclusiveness at the Lord's Supper. It is true that open sin excludes
the guilty. This the Holy Spirit plainly teaches. 1 Cor. 5:11. But beyond this none are to pass judgment.
God has not left it with men to say who shall present themselves on these occasions. For who can
read the heart? Who can distinguish the tares from the wheat? "Let a man examine himself, and so let
him eat of that bread, and drink of that cup." For "whosoever shall eat this bread, and drink this cup
of the Lord, unworthily, shall be guilty of the body and blood of the Lord." "He that eateth and
drinketh unworthily, eateth and drinketh damnation to himself, not discerning the Lord's body." 1
Cor. 11:28, 27, 29.

Those that are known in open sin cannot partake of the Lord’s supper.

http://breachrepairers.webs.com/ 635
1 Cor 11:27

What does it mean to eat and drink unworthily?

1 Cor 11:29-30

If a person hasn’t been baptized or made a public commitment to Jesus, would they have a true
understanding of what it is about? If you do not understand what the communion is about, then you
will be drinking and eating it unworthily.

13:22 Then the disciples looked one on another, doubting of whom he spake.

13:23 Now there was leaning on Jesus' bosom one of his disciples, whom Jesus loved.

13:24 Simon Peter therefore beckoned to him, that he should ask who it should be of whom he spake.

13:25 He then lying on Jesus' breast saith unto him, Lord, who is it?

13:26 Jesus answered, He it is, to whom I shall give a sop, when I have dipped it. And when he had dipped
the sop, he gave it to Judas Iscariot, the son of Simon.

13:27 And after the sop Satan entered into him. Then said Jesus unto him, That thou doest, do quickly.

13:28 Now no man at the table knew for what intent he spake this unto him.

13:29 For some of them thought, because Judas had the bag, that Jesus had said unto him, Buy those
things that we have need of against the feast; or, that he should give something to the poor.

13:30 He then having received the sop went immediately out: and it was night.

13:31 Therefore, when he was gone out, Jesus said, Now is the Son of man glorified, and God is glorified
in him.

13:32 If God be glorified in him, God shall also glorify him in himself, and shall straightway glorify him.

13:33 Little children, yet a little while I am with you. Ye shall seek me: and as I said unto the Jews,
Whither I go, ye cannot come; so now I say to you.

13:34 A new commandment I give unto you, That ye love one another; as I have loved you, that ye also
love one another.

13:35 By this shall all men know that ye are my disciples, if ye have love one to another.

When people see we have genuine love one for another, they will know that we are the disciples of
Jesus.

13:36 Simon Peter said unto him, Lord, whither goest thou? Jesus answered him, Whither I go, thou canst
not follow me now; but thou shalt follow me afterwards.

13:37 Peter said unto him, Lord, why cannot I follow thee now? I will lay down my life for thy sake.

13:38 Jesus answered him, Wilt thou lay down thy life for my sake? Verily, verily, I say unto thee, The
cock shall not crow, till thou hast denied me thrice.

http://breachrepairers.webs.com/ 636
Peter felt that it was cruel to be saying that to him.

Matt 26:33-35

Peter wasn’t the only one that was so vehement about not denying Jesus. But it is a good illustration
about when we read the Bible or hear a sermon. God is directly speaking to us, but we feel that that
message is not for us. Was Peter sincere enough? Sincerity was not enough. Peter did not know
himself. We make resolutions that we are going to do certain things, but the Lord brings us to places
where we recognize who we really are. Paul counts him as the least of all the disciples. When people
think they are going to stand, they will fall.

http://breachrepairers.webs.com/ 637
Chapter 14
Chapter Outline
14:1 Let not your heart be troubled: ye believe in God, believe also in me.

14:2 In my Father's house are many mansions: if it were not so, I would have told you. I go to prepare a
place for you.

14:3 And if I go and prepare a place for you, I will come again, and receive you unto myself; that where I
am, there ye may be also.

14:4 And whither I go ye know, and the way ye know.

14:5 Thomas saith unto him, Lord, we know not whither thou goest; and how can we know the way?

14:6 Jesus saith unto him, I am the way, the truth, and the life: no man cometh unto the Father, but by
me.

Whenever you want to define truth, the way or eternal life, this verse if very good.

14:7 If ye had known me, ye should have known my Father also: and from henceforth ye know him, and
have seen him.

14:8 Philip saith unto him, Lord, shew us the Father, and it sufficeth us.

14:9 Jesus saith unto him, Have I been so long time with you, and yet hast thou not known me, Philip? he
that hath seen me hath seen the Father; and how sayest thou then, Shew us the Father?

14:10 Believest thou not that I am in the Father, and the Father in me? the words that I speak unto you I
speak not of myself: but the Father that dwelleth in me, he doeth the works.

14:11 Believe me that I am in the Father, and the Father in me: or else believe me for the very works'
sake.

14:12 Verily, verily, I say unto you, He that believeth on me, the works that I do shall he do also; and
greater works than these shall he do; because I go unto my Father.

DA 664 Jesus revealed no qualities, and exercised no powers, that men may not have through faith in
Him.

Is it possible for men to read other men’s hearts then? Did Jesus ever read peoples hearts? Nothing
is impossible for us through Jesus if we have faith. The Holy Spirit can impress us with the thoughts
of other people.

DA 664 By this Christ did not mean that the disciples' work would be of a more exalted character
than His, but that it would have greater extent.

14:13 And whatsoever ye shall ask in my name, that will I do, that the Father may be glorified in the Son.

14:14 If ye shall ask any thing in my name, I will do it.

14:15 If ye love me, keep my commandments.

http://breachrepairers.webs.com/ 638
Today everybody is saying that they love Jesus. But if you love God, it is manifested in obedience.

John 14:21

14:16 And I will pray the Father, and he shall give you another Comforter, that he may abide with you for
ever;

14:17 Even the Spirit of truth; whom the world cannot receive, because it seeth him not, neither knoweth
him: but ye know him; for he dwelleth with you, and shall be in you.

14:18 I will not leave you comfortless: I will come to you.

14:19 Yet a little while, and the world seeth me no more; but ye see me: because I live, ye shall live also.

14:20 At that day ye shall know that I am in my Father, and ye in me, and I in you.

14:21 He that hath my commandments, and keepeth them, he it is that loveth me: and he that loveth me
shall be loved of my Father, and I will love him, and will manifest myself to him.

14:22 Judas saith unto him, not Iscariot, Lord, how is it that thou wilt manifest thyself unto us, and not
unto the world?

14:23 Jesus answered and said unto him, If a man love me, he will keep my words: and my Father will
love him, and we will come unto him, and make our abode with him.

14:24 He that loveth me not keepeth not my sayings: and the word which ye hear is not mine, but the
Father's which sent me.

14:25 These things have I spoken unto you, being yet present with you.

14:26 But the Comforter, which is the Holy Ghost, whom the Father will send in my name, he shall teach
you all things, and bring all things to your remembrance, whatsoever I have said unto you.

The functions of the Holy Spirit: Teacher and brings all things to remembrance.

14:27 Peace I leave with you, my peace I give unto you: not as the world giveth, give I unto you. Let not
your heart be troubled, neither let it be afraid.

14:28 Ye have heard how I said unto you, I go away, and come again unto you. If ye loved me, ye would
rejoice, because I said, I go unto the Father: for my Father is greater than I.

Always esteeming each other better than yourself.

14:29 And now I have told you before it come to pass, that, when it is come to pass, ye might believe.

14:30 Hereafter I will not talk much with you: for the prince of this world cometh, and hath nothing in
me.

When Satan came to Jesus, he confined no responsive cord in any temptation. In order to be
tempted, there has to be an internal source, something that responds to the temptation. God cannot
be tempted. But the fact that Jesus could be tempted, at one point in His life, there could be
something in Jesus to be tempted.

14:31 But that the world may know that I love the Father; and as the Father gave me commandment,

http://breachrepairers.webs.com/ 639
even so I do. Arise, let us go hence.

http://breachrepairers.webs.com/ 640
Chapter 15 – The true Vine
The Jews viewed the vine as the most noble.

DA 674 The Saviour had been explaining to His disciples His mission to the world, and the spiritual
relation to Him which they were to sustain. Now He illustrates the lesson. The moon is shining bright,
and reveals to Him a flourishing grapevine. Drawing the attention of the disciples to it, He employs it
as a symbol.

The closer you are to the vine, the stronger it will be.

Chapter Outline

15:1 I am the true vine, and my Father is the husbandman.

15:2 Every branch in me that beareth not fruit he taketh away: and every branch that beareth fruit, he
purgeth it, that it may bring forth more fruit.

“purgeth it” – It is pruned. Pruning actually concentrates the energy of the plant into the
production of fruit. The reason God gives us trials is so that the focus of us is to produce fruit.

15:3 Now ye are clean through the word which I have spoken unto you.

God justifies us by faith, and faith comes from the Word of God.

15:4 Abide in me, and I in you. As the branch cannot bear fruit of itself, except it abide in the vine; no
more can ye, except ye abide in me.

DA 676 From the chosen twelve who had followed Jesus, one as a withered branch was about to be
taken away; the rest were to pass under the pruning knife of bitter trial.

The word “relationship” is used too loosely in connection with Christ. When we sin we are not in a
saving relationship anymore.

15:5 I am the vine, ye are the branches: He that abideth in me, and I in him, the same bringeth forth
much fruit: for without me ye can do nothing.

DA 671 This is the only effectual teacher of divine truth. Only when the truth is accompanied to the
heart by the Spirit will it quicken the conscience or transform the life. One might be able to present
the letter of the word of God, he might be familiar with all its commands and promises; but unless the
Holy Spirit sets home the truth, no souls will fall on the Rock and be broken. No amount of education,
no advantages, however great, can make one a channel of light without the co-operation of the Spirit
of God.” –

Without the divine aid of the Holy Spirit, we can go out and do ministry, people will come, but the
issue is are the souls converted? Do they hunger after the truth? Any ministry for God, all their
success hinges on whether they are connected to God or not.

15:6 If a man abide not in me, he is cast forth as a branch, and is withered; and men gather them, and
cast them into the fire, and they are burned.

15:7 If ye abide in me, and my words abide in you, ye shall ask what ye will, and it shall be done unto you.

http://breachrepairers.webs.com/ 641
15:8 Herein is my Father glorified, that ye bear much fruit; so shall ye be my disciples.

15:9 As the Father hath loved me, so have I loved you: continue ye in my love.

15:10 If ye keep my commandments, ye shall abide in my love; even as I have kept my Father's
commandments, and abide in his love.

15:11 These things have I spoken unto you, that my joy might remain in you, and that your joy might be
full.

15:12 This is my commandment, That ye love one another, as I have loved you.

15:13 Greater love hath no man than this, that a man lay down his life for his friends.

15:14 Ye are my friends, if ye do whatsoever I command you.

15:15 Henceforth I call you not servants; for the servant knoweth not what his lord doeth: but I have
called you friends; for all things that I have heard of my Father I have made known unto you.

15:16 Ye have not chosen me, but I have chosen you, and ordained you, that ye should go and bring forth
fruit, and that your fruit should remain: that whatsoever ye shall ask of the Father in my name, he may
give it you.

“ordained” – set apart. They were set apart to win souls.

15:17 These things I command you, that ye love one another.

15:18 If the world hate you, ye know that it hated me before it hated you.

15:19 If ye were of the world, the world would love his own: but because ye are not of the world, but I
have chosen you out of the world, therefore the world hateth you.

15:20 Remember the word that I said unto you, The servant is not greater than his lord. If they have
persecuted me, they will also persecute you; if they have kept my saying, they will keep yours also.

15:21 But all these things will they do unto you for my name's sake, because they know not him that sent
me.

15:22 If I had not come and spoken unto them, they had not had sin: but now they have no cloke for their
sin.

This tells us more about the nature of sin.

15:23 He that hateth me hateth my Father also.

15:24 If I had not done among them the works which none other man did, they had not had sin: but now
have they both seen and hated both me and my Father.

Similar to what was described in verse 22. The fool can say in their heart that there is no God. But
when they look at the works in nature, they can not but testify that there is a God. The Gentiles saw
Jesus in the works of nature.

15:25 But this cometh to pass, that the word might be fulfilled that is written in their law, They hated me
without a cause.

http://breachrepairers.webs.com/ 642
15:26 But when the Comforter is come, whom I will send unto you from the Father, even the Spirit of
truth, which proceedeth from the Father, he shall testify of me:

It distinguishes between the three.

John 16:7

It distinguishes here between the them. In the different parts of the Bible, there is different
emphasis.

Matt 28:19, 20

Matt 3:16, 17

15:27 And ye also shall bear witness, because ye have been with me from the beginning.

http://breachrepairers.webs.com/ 643
Chapter 16 – Coming of the Comforter
Chapter Outline
16:1 These things have I spoken unto you, that ye should not be offended.

16:2 They shall put you out of the synagogues: yea, the time cometh, that whosoever killeth you will
think that he doeth God service.

What 3 fulfillments do we have in this verse? Immediate primary context here when the Pharisees
were combining to kill Jesus. Papal persecution. End of time – death decree will go forward and they
will think they are doing right by killing us. Therefore, Jesus is a type for the future of what we need
to follow. Back then, they were also combining state and church together to kill Jesus. There are a
lot of similarities between the closing events of the life of Jesus and the closing events of earths
history.

16:3 And these things will they do unto you, because they have not known the Father, nor me.

16:4 But these things have I told you, that when the time shall come, ye may remember that I told you of
them. And these things I said not unto you at the beginning, because I was with you.

16:5 But now I go my way to him that sent me; and none of you asketh me, Whither goest thou?

16:6 But because I have said these things unto you, sorrow hath filled your heart.

16:7 Nevertheless I tell you the truth; It is expedient for you that I go away: for if I go not away, the
Comforter will not come unto you; but if I depart, I will send him unto you.

16:8 And when he is come, he will reprove the world of sin, and of righteousness, and of judgment:

Three things will be reproved (convince): Sin, righteousness, and judgment. The Holy Spirit plays
three roles: He will convince us of our sin. He is the power within us to help us to live righteous
lives. This judgment is referring to the judgment on Satan. Up till the cross, it was not clear to
everyone the stand point of what Satan took.

16:9 Of sin, because they believe not on me;

16:10 Of righteousness, because I go to my Father, and ye see me no more;

16:11 Of judgment, because the prince of this world is judged.

16:12 I have yet many things to say unto you, but ye cannot bear them now.

16:13 Howbeit when he, the Spirit of truth, is come, he will guide you into all truth: for he shall not speak
of himself; but whatsoever he shall hear, that shall he speak: and he will shew you things to come.

16:14 He shall glorify me: for he shall receive of mine, and shall shew it unto you.

16:15 All things that the Father hath are mine: therefore said I, that he shall take of mine, and shall shew
it unto you.

16:16 A little while, and ye shall not see me: and again, a little while, and ye shall see me, because I go to
the Father.

http://breachrepairers.webs.com/ 644
16:17 Then said some of his disciples among themselves, What is this that he saith unto us, A little while,
and ye shall not see me: and again, a little while, and ye shall see me: and, Because I go to the Father?

16:18 They said therefore, What is this that he saith, A little while? we cannot tell what he saith.

16:19 Now Jesus knew that they were desirous to ask him, and said unto them, Do ye enquire among
yourselves of that I said, A little while, and ye shall not see me: and again, a little while, and ye shall see
me?

16:20 Verily, verily, I say unto you, That ye shall weep and lament, but the world shall rejoice: and ye
shall be sorrowful, but your sorrow shall be turned into joy.

16:21 A woman when she is in travail hath sorrow, because her hour is come: but as soon as she is
delivered of the child, she remembereth no more the anguish, for joy that a man is born into the world.

16:22 And ye now therefore have sorrow: but I will see you again, and your heart shall rejoice, and your
joy no man taketh from you.

16:23 And in that day ye shall ask me nothing. Verily, verily, I say unto you, Whatsoever ye shall ask the
Father in my name, he will give it you.

16:24 Hitherto have ye asked nothing in my name: ask, and ye shall receive, that your joy may be full.

16:25 These things have I spoken unto you in proverbs: but the time cometh, when I shall no more speak
unto you in proverbs, but I shall shew you plainly of the Father.

16:26 At that day ye shall ask in my name: and I say not unto you, that I will pray the Father for you:

16:27 For the Father himself loveth you, because ye have loved me, and have believed that I came out
from God.

16:28 I came forth from the Father, and am come into the world: again, I leave the world, and go to the
Father.

16:29 His disciples said unto him, Lo, now speakest thou plainly, and speakest no proverb.

16:30 Now are we sure that thou knowest all things, and needest not that any man should ask thee: by
this we believe that thou camest forth from God.

16:31 Jesus answered them, Do ye now believe?

16:32 Behold, the hour cometh, yea, is now come, that ye shall be scattered, every man to his own, and
shall leave me alone: and yet I am not alone, because the Father is with me.

16:33 These things I have spoken unto you, that in me ye might have peace. In the world ye shall have
tribulation: but be of good cheer; I have overcome the world.

http://breachrepairers.webs.com/ 645
Chapter 17 – Prayer of Unity
EGW tells us that the 17th chapter should be read everyday or by the disciples of the believers of
Jesus today.

Chapter Outline
17:1 These words spake Jesus, and lifted up his eyes to heaven, and said, Father, the hour is come; glorify
thy Son, that thy Son also may glorify thee:

17:2 As thou hast given him power over all flesh, that he should give eternal life to as many as thou hast
given him.

17:3 And this is life eternal, that they might know thee the only true God, and Jesus Christ, whom thou
hast sent.

It is not simple a mere intellectual understanding. It is an intimate understanding.

17:4 I have glorified thee on the earth: I have finished the work which thou gavest me to do.

17:5 And now, O Father, glorify thou me with thine own self with the glory which I had with thee before
the world was.

17:6 I have manifested thy name unto the men which thou gavest me out of the world: thine they were,
and thou gavest them me; and they have kept thy word.

17:7 Now they have known that all things whatsoever thou hast given me are of thee.

17:8 For I have given unto them the words which thou gavest me; and they have received them, and have
known surely that I came out from thee, and they have believed that thou didst send me.

17:9 I pray for them: I pray not for the world, but for them which thou hast given me; for they are thine.

17:10 And all mine are thine, and thine are mine; and I am glorified in them.

17:11 And now I am no more in the world, but these are in the world, and I come to thee. Holy Father,
keep through thine own name those whom thou hast given me, that they may be one, as we are.

17:12 While I was with them in the world, I kept them in thy name: those that thou gavest me I have kept,
and none of them is lost, but the son of perdition; that the scripture might be fulfilled.

17:13 And now come I to thee; and these things I speak in the world, that they might have my joy fulfilled
in themselves.

17:14 I have given them thy word; and the world hath hated them, because they are not of the world,
even as I am not of the world.

The preaching of the gospel is divisive. Why preaching is not more pointed today because the
messenger themselves are not able to justify themselves living up to the light that they should be.

17:15 I pray not that thou shouldest take them out of the world, but that thou shouldest keep them from
the evil.

17:16 They are not of the world, even as I am not of the world.

http://breachrepairers.webs.com/ 646
17:17 Sanctify them through thy truth: thy word is truth.

In verse 15, we are to be kept from evil. Sanctify means we are to be set apart. So to keep us from
evil, it can only be done through the Word. Error has no power to sanctify.

17:18 As thou hast sent me into the world, even so have I also sent them into the world.

17:19 And for their sakes I sanctify myself, that they also might be sanctified through the truth.

The leader has a great influence upon the people that he is leading. You cannot expect people to be
converted in your ministry if you are not converted.

17:20 Neither pray I for these alone, but for them also which shall believe on me through their word;

17:21 That they all may be one; as thou, Father, art in me, and I in thee, that they also may be one in us:
that the world may believe that thou hast sent me.

As we come closer to Christ, we will find that our minds are together. You cannot have unity apart
from truth. Until everyone is converted, there will be no real unity.

17:22 And the glory which thou gavest me I have given them; that they may be one, even as we are one:

17:23 I in them, and thou in me, that they may be made perfect in one; and that the world may know that
thou hast sent me, and hast loved them, as thou hast loved me.

17:24 Father, I will that they also, whom thou hast given me, be with me where I am; that they may
behold my glory, which thou hast given me: for thou lovedst me before the foundation of the world.

17:25 O righteous Father, the world hath not known thee: but I have known thee, and these have known
that thou hast sent me.

17:26 And I have declared unto them thy name, and will declare it: that the love wherewith thou hast
loved me may be in them, and I in them.

http://breachrepairers.webs.com/ 647
Chapter 18
Chapter Outline
18:1 When Jesus had spoken these words, he went forth with his disciples over the brook Cedron, where
was a garden, into the which he entered, and his disciples.

18:2 And Judas also, which betrayed him, knew the place: for Jesus ofttimes resorted thither with his
disciples.

18:3 Judas then, having received a band of men and officers from the chief priests and Pharisees, cometh
thither with lanterns and torches and weapons.

18:4 Jesus therefore, knowing all things that should come upon him, went forth, and said unto them,
Whom seek ye?

18:5 They answered him, Jesus of Nazareth. Jesus saith unto them, I am he. And Judas also, which
betrayed him, stood with them.

18:6 As soon then as he had said unto them, I am he, they went backward, and fell to the ground.

18:7 Then asked he them again, Whom seek ye? And they said, Jesus of Nazareth.

18:8 Jesus answered, I have told you that I am he: if therefore ye seek me, let these go their way:

18:9 That the saying might be fulfilled, which he spake, Of them which thou gavest me have I lost none.

18:10 Then Simon Peter having a sword drew it, and smote the high priest's servant, and cut off his right
ear. The servant's name was Malchus.

18:11 Then said Jesus unto Peter, Put up thy sword into the sheath: the cup which my Father hath given
me, shall I not drink it?

The tongue also represents a sword in the Bible. Application: When people are about to face a crisis,
they will bring out their sword (tongue). They will prepare words that are cutting. If you can bare
insult and people criticizing you and you don’t wound then, that is a indication that you are close to
Jesus.

18:12 Then the band and the captain and officers of the Jews took Jesus, and bound him,

18:13 And led him away to Annas first; for he was father in law to Caiaphas, which was the high priest
that same year.

18:14 Now Caiaphas was he, which gave counsel to the Jews, that it was expedient that one man should
die for the people.

18:15 And Simon Peter followed Jesus, and so did another disciple: that disciple was known unto the high
priest, and went in with Jesus into the palace of the high priest.

18:16 But Peter stood at the door without. Then went out that other disciple, which was known unto the
high priest, and spake unto her that kept the door, and brought in Peter.

18:17 Then saith the damsel that kept the door unto Peter, Art not thou also one of this man's disciples?
He saith, I am not.

http://breachrepairers.webs.com/ 648
18:18 And the servants and officers stood there, who had made a fire of coals; for it was cold: and they
warmed themselves: and Peter stood with them, and warmed himself.

18:19 The high priest then asked Jesus of his disciples, and of his doctrine.

18:20 Jesus answered him, I spake openly to the world; I ever taught in the synagogue, and in the temple,
whither the Jews always resort; and in secret have I said nothing.

18:21 Why askest thou me? ask them which heard me, what I have said unto them: behold, they know
what I said.

18:22 And when he had thus spoken, one of the officers which stood by struck Jesus with the palm of his
hand, saying, Answerest thou the high priest so?

18:23 Jesus answered him, If I have spoken evil, bear witness of the evil: but if well, why smitest thou me?

18:24 Now Annas had sent him bound unto Caiaphas the high priest.

18:25 And Simon Peter stood and warmed himself. They said therefore unto him, Art not thou also one of
his disciples? He denied it, and said, I am not.

18:26 One of the servants of the high priest, being his kinsman whose ear Peter cut off, saith, Did not I
see thee in the garden with him?

18:27 Peter then denied again: and immediately the cock crew.

18:28 Then led they Jesus from Caiaphas unto the hall of judgment: and it was early; and they
themselves went not into the judgment hall, lest they should be defiled; but that they might eat the
passover.

The trial of Jesus took place in the wee early hours of the morning.

DA 725 But there was something in the prisoner that held Pilate back from this. He dared not do it.
He read the purposes of the priests. He remembered how, not long before, Jesus had raised Lazarus, a
man that had been dead four days; and he determined to know, before signing the sentence of
condemnation, what were the charges against Him, and whether they could be proved.

John 18:30

When Pilate saw Jesus, he knew that Jesus was more than an ordinary man.

DA 725 Pilate was not a just or a conscientious judge; but weak though he was in moral power, he
refused to grant this request. He would not condemn Jesus until a charge had been brought against
Him.

DA 726 Pilate saw through their purpose. He did not believe that the prisoner had plotted against the
government. His meek and humble appearance was altogether out of harmony with the charge. Pilate
was convinced that a deep plot had been laid to destroy an innocent man who stood in the way of the
Jewish dignitaries. Turning to Jesus he asked, "Art Thou the King of the Jews?" The Saviour answered,
"Thou sayest it." And as He spoke, His countenance lighted up as if a sunbeam were shining upon it.

Pilate is still at a point that he can be innocent of the blood of Jesus.

http://breachrepairers.webs.com/ 649
Luke 23:4

Jesus gave evidence that He was more than divine. Jesus was given over to Herod but he was a
different man now:

DA 730 Herod's conscience was now far less sensitive than when he had trembled with horror at the
request of Herodias for the head of John the Baptist. For a time he had felt the keen stings of remorse
for his terrible act; but his moral perceptions had become more and more degraded by his licentious
life. Now his heart had become so hardened that he could even boast of the punishment he had
inflicted upon John for daring to reprove him. And he now threatened Jesus, declaring repeatedly that
he had power to release or to condemn Him. But no sign from Jesus gave evidence that He heard a
word.

Herod already had tremendous light from John the Baptist.

DA 730 But Christ's silence was the severest rebuke that He could have given. Herod had rejected the
truth spoken to him by the greatest of the prophets, and no other message was he to receive.” –
DA 731 No sooner were these words spoken than a rush was made for Christ. Like wild beasts, the
crowd darted upon their prey. Jesus was dragged this way and that, Herod joining the mob in seeking
to humiliate the Son of God. Had not the Roman soldiers interposed, and forced back the maddened
throng, the Saviour would have been torn in pieces.

DA 731 While the rude throng were bowing in mockery before Him, some who came forward for that
purpose turned back, afraid and silenced. Herod was convicted. The last rays of merciful light were
shining upon his sin-hardened heart. He felt that this was no common man; for divinity had flashed
through humanity. At the very time when Christ was encompassed by mockers, adulterers, and
murderers, Herod felt that he was beholding a God upon His throne.

The way that Jesus flashed forth His divinity was by remaining calm and not saying anything.

John 18:13-16

Pilate’s character was to just make the people happy.

John 18:39

John 18:20-24

Matt 27:19

DA 732 In answer to Christ's prayer, the wife of Pilate had been visited by an angel from heaven, and
in a dream she had beheld the Saviour and conversed with Him. Pilate's wife was not a Jew, but as
she looked upon Jesus in her dream, she had no doubt of His character or mission. She knew Him to
be the Prince of God. She saw Him on trial in the judgment hall. She saw the hands tightly bound as
the hands of a criminal. She saw Herod and his soldiers doing their dreadful work. She heard the
priests and rulers, filled with envy and malice, madly accusing. She heard the words, "We have a law,
and by our law He ought to die." She saw Pilate give Jesus to the scourging, after he had declared, "I
find no fault in Him." She heard the condemnation pronounced by Pilate, and saw him give Christ up
to His murderers. She saw the cross uplifted on Calvary. She saw the earth wrapped in darkness, and
heard the mysterious cry, "It is finished." Still another scene met her gaze. She saw Christ seated
upon the great white cloud, while the earth reeled in space, and His murderers fled from the
presence of His glory. With a cry of horror she awoke, and at once wrote to Pilate words of warning.

DA 732 Pilate's face grew pale.

http://breachrepairers.webs.com/ 650
John 19:7-11

DA 738 When Pilate saw that he could prevail nothing, but that rather a tumult was made, he took
water, and washed his hands before the multitude, saying, I am innocent of the blood of this just
Person: see ye to it." In fear and self-condemnation Pilate looked upon the Saviour. In the vast sea of
upturned faces, His alone was peaceful. About His head a soft light seemed to shine. Pilate said in his
heart, He is a God. Turning to the multitude he declared, I am clear of His blood. Take ye Him, and
crucify Him. But mark ye, priests and rulers, I pronounce Him a just man. May He whom He claims as
His Father judge you and not me for this day's work. Then to Jesus he said, Forgive me for this act; I
cannot save You. And when he had again scourged Jesus, he delivered Him to be crucified.

Once you know you can never put off a decision. You can never avoid what you are going to do with
Jesus.

18:29 Pilate then went out unto them, and said, What accusation bring ye against this man?

18:30 They answered and said unto him, If he were not a malefactor, we would not have delivered him
up unto thee.

18:31 Then said Pilate unto them, Take ye him, and judge him according to your law. The Jews therefore
said unto him, It is not lawful for us to put any man to death:

18:32 That the saying of Jesus might be fulfilled, which he spake, signifying what death he should die.

18:33 Then Pilate entered into the judgment hall again, and called Jesus, and said unto him, Art thou the
King of the Jews?

18:34 Jesus answered him, Sayest thou this thing of thyself, or did others tell it thee of me?

18:35 Pilate answered, Am I a Jew? Thine own nation and the chief priests have delivered thee unto me:
what hast thou done?

18:36 Jesus answered, My kingdom is not of this world: if my kingdom were of this world, then would my
servants fight, that I should not be delivered to the Jews: but now is my kingdom not from hence.

18:37 Pilate therefore said unto him, Art thou a king then? Jesus answered, Thou sayest that I am a king.
To this end was I born, and for this cause came I into the world, that I should bear witness unto the truth.
Every one that is of the truth heareth my voice.

18:38 Pilate saith unto him, What is truth? And when he had said this, he went out again unto the Jews,
and saith unto them, I find in him no fault at all.

18:39 But ye have a custom, that I should release unto you one at the passover: will ye therefore that I
release unto you the King of the Jews?

18:40 Then cried they all again, saying, Not this man, but Barabbas. Now Barabbas was a robber.

http://breachrepairers.webs.com/ 651
Chapter 19
Chapter Outline

19:1 Then Pilate therefore took Jesus, and scourged him.

19:2 And the soldiers platted a crown of thorns, and put it on his head, and they put on him a purple
robe,

19:3 And said, Hail, King of the Jews! and they smote him with their hands.

19:4 Pilate therefore went forth again, and saith unto them, Behold, I bring him forth to you, that ye may
know that I find no fault in him.

19:5 Then came Jesus forth, wearing the crown of thorns, and the purple robe. And Pilate saith unto
them, Behold the man!

19:6 When the chief priests therefore and officers saw him, they cried out, saying, Crucify him, crucify
him. Pilate saith unto them, Take ye him, and crucify him: for I find no fault in him.

19:7 The Jews answered him, We have a law, and by our law he ought to die, because he made himself
the Son of God.

19:8 When Pilate therefore heard that saying, he was the more afraid;

19:9 And went again into the judgment hall, and saith unto Jesus, Whence art thou? But Jesus gave him
no answer.

19:10 Then saith Pilate unto him, Speakest thou not unto me? knowest thou not that I have power to
crucify thee, and have power to release thee?

19:11 Jesus answered, Thou couldest have no power at all against me, except it were given thee from
above: therefore he that delivered me unto thee hath the greater sin.

19:12 And from thenceforth Pilate sought to release him: but the Jews cried out, saying, If thou let this
man go, thou art not Caesar's friend: whosoever maketh himself a king speaketh against Caesar.

19:13 When Pilate therefore heard that saying, he brought Jesus forth, and sat down in the judgment
seat in a place that is called the Pavement, but in the Hebrew, Gabbatha.

19:14 And it was the preparation of the passover, and about the sixth hour: and he saith unto the Jews,
Behold your King!

19:15 But they cried out, Away with him, away with him, crucify him. Pilate saith unto them, Shall I
crucify your King? The chief priests answered, We have no king but Caesar.

19:16 Then delivered he him therefore unto them to be crucified. And they took Jesus, and led him away.

19:17 And he bearing his cross went forth into a place called the place of a skull, which is called in the
Hebrew Golgotha:

19:18 Where they crucified him, and two other with him, on either side one, and Jesus in the midst.

19:19 And Pilate wrote a title, and put it on the cross. And the writing was, JESUS OF NAZARETH THE

http://breachrepairers.webs.com/ 652
KING OF THE JEWS.

19:20 This title then read many of the Jews: for the place where Jesus was crucified was nigh to the city:
and it was written in Hebrew, and Greek, and Latin.

19:21 Then said the chief priests of the Jews to Pilate, Write not, The King of the Jews; but that he said, I
am King of the Jews.

19:22 Pilate answered, What I have written I have written.

19:23 Then the soldiers, when they had crucified Jesus, took his garments, and made four parts, to every
soldier a part; and also his coat: now the coat was without seam, woven from the top throughout.

19:24 They said therefore among themselves, Let us not rend it, but cast lots for it, whose it shall be: that
the scripture might be fulfilled, which saith, They parted my raiment among them, and for my vesture
they did cast lots. These things therefore the soldiers did.

19:25 Now there stood by the cross of Jesus his mother, and his mother's sister, Mary the wife of
Cleophas, and Mary Magdalene.

19:26 When Jesus therefore saw his mother, and the disciple standing by, whom he loved, he saith unto
his mother, Woman, behold thy son!

19:27 Then saith he to the disciple, Behold thy mother! And from that hour that disciple took her unto his
own home.

19:28 After this, Jesus knowing that all things were now accomplished, that the scripture might be
fulfilled, saith, I thirst.

19:29 Now there was set a vessel full of vinegar: and they filled a spunge with vinegar, and put it upon
hyssop, and put it to his mouth.

19:30 When Jesus therefore had received the vinegar, he said, It is finished: and he bowed his head, and
gave up the ghost.

19:31 The Jews therefore, because it was the preparation, that the bodies should not remain upon the
cross on the sabbath day, (for that sabbath day was an high day,) besought Pilate that their legs might
be broken, and that they might be taken away.

19:32 Then came the soldiers, and brake the legs of the first, and of the other which was crucified with
him.

19:33 But when they came to Jesus, and saw that he was dead already, they brake not his legs:

19:34 But one of the soldiers with a spear pierced his side, and forthwith came there out blood and
water.

19:35 And he that saw it bare record, and his record is true: and he knoweth that he saith true, that ye
might believe.

19:36 For these things were done, that the scripture should be fulfilled, A bone of him shall not be
broken.

19:37 And again another scripture saith, They shall look on him whom they pierced.

http://breachrepairers.webs.com/ 653
19:38 And after this Joseph of Arimathaea, being a disciple of Jesus, but secretly for fear of the Jews,
besought Pilate that he might take away the body of Jesus: and Pilate gave him leave. He came therefore,
and took the body of Jesus.

19:39 And there came also Nicodemus, which at the first came to Jesus by night, and brought a mixture
of myrrh and aloes, about an hundred pound weight.

19:40 Then took they the body of Jesus, and wound it in linen clothes with the spices, as the manner of
the Jews is to bury.

19:41 Now in the place where he was crucified there was a garden; and in the garden a new sepulchre,
wherein was never man yet laid.

19:42 There laid they Jesus therefore because of the Jews' preparation day; for the sepulchre was nigh at
hand.

http://breachrepairers.webs.com/ 654
Chapter 20 - Resurrection of Jesus
Chapter Outline
20:1 The first day of the week cometh Mary Magdalene early, when it was yet dark, unto the sepulchre,
and seeth the stone taken away from the sepulchre.

20:2 Then she runneth, and cometh to Simon Peter, and to the other disciple, whom Jesus loved, and
saith unto them, They have taken away the Lord out of the sepulchre, and we know not where they have
laid him.

20:3 Peter therefore went forth, and that other disciple, and came to the sepulchre.

20:4 So they ran both together: and the other disciple did outrun Peter, and came first to the sepulchre.

20:5 And he stooping down, and looking in, saw the linen clothes lying; yet went he not in.

20:6 Then cometh Simon Peter following him, and went into the sepulchre, and seeth the linen clothes lie,

20:7 And the napkin, that was about his head, not lying with the linen clothes, but wrapped together in a
place by itself.

20:8 Then went in also that other disciple, which came first to the sepulchre, and he saw, and believed.

20:9 For as yet they knew not the scripture, that he must rise again from the dead.

20:10 Then the disciples went away again unto their own home.

20:11 But Mary stood without at the sepulchre weeping: and as she wept, she stooped down, and looked
into the sepulchre,

20:12 And seeth two angels in white sitting, the one at the head, and the other at the feet, where the body
of Jesus had lain.

20:13 And they say unto her, Woman, why weepest thou? She saith unto them, Because they have taken
away my Lord, and I know not where they have laid him.

20:14 And when she had thus said, she turned herself back, and saw Jesus standing, and knew not that it
was Jesus.

20:15 Jesus saith unto her, Woman, why weepest thou? whom seekest thou? She, supposing him to be the
gardener, saith unto him, Sir, if thou have borne him hence, tell me where thou hast laid him, and I will
take him away.

20:16 Jesus saith unto her, Mary. She turned herself, and saith unto him, Rabboni; which is to say,
Master.

20:17 Jesus saith unto her, Touch me not; for I am not yet ascended to my Father: but go to my brethren,
and say unto them, I ascend unto my Father, and your Father; and to my God, and your God.

20:18 Mary Magdalene came and told the disciples that she had seen the Lord, and that he had spoken
these things unto her.

20:19 Then the same day at evening, being the first day of the week, when the doors were shut where the

http://breachrepairers.webs.com/ 655
disciples were assembled for fear of the Jews, came Jesus and stood in the midst, and saith unto them,
Peace be unto you.

20:20 And when he had so said, he shewed unto them his hands and his side. Then were the disciples
glad, when they saw the Lord.

20:21 Then said Jesus to them again, Peace be unto you: as my Father hath sent me, even so send I you.

20:22 And when he had said this, he breathed on them, and saith unto them, Receive ye the Holy Ghost:

20:23 Whose soever sins ye remit, they are remitted unto them; and whose soever sins ye retain, they are
retained.

20:24 But Thomas, one of the twelve, called Didymus, was not with them when Jesus came.

20:25 The other disciples therefore said unto him, We have seen the Lord. But he said unto them, Except I
shall see in his hands the print of the nails, and put my finger into the print of the nails, and thrust my
hand into his side, I will not believe.

20:26 And after eight days again his disciples were within, and Thomas with them: then came Jesus, the
doors being shut, and stood in the midst, and said, Peace be unto you.

20:27 Then saith he to Thomas, Reach hither thy finger, and behold my hands; and reach hither thy
hand, and thrust it into my side: and be not faithless, but believing.

20:28 And Thomas answered and said unto him, My Lord and my God.

20:29 Jesus saith unto him, Thomas, because thou hast seen me, thou hast believed: blessed are they that
have not seen, and yet have believed.

20:30 And many other signs truly did Jesus in the presence of his disciples, which are not written in this
book:

20:31 But these are written, that ye might believe that Jesus is the Christ, the Son of God; and that
believing ye might have life through his name.

http://breachrepairers.webs.com/ 656
Chapter 21 – Peter at the Sea of Tiberias
Chapter Outline

21:1 After these things Jesus shewed himself again to the disciples at the sea of Tiberias; and on this wise
shewed he himself.

What event did Jesus remind Peter of when he and other disciples caught the multitude of
fishes in the sea of Tiberias?

John 21:1-6

Luke 5:4-6

21:2 There were together Simon Peter, and Thomas called Didymus, and Nathanael of Cana in Galilee,
and the sons of Zebedee, and two other of his disciples.

21:3 Simon Peter saith unto them, I go a fishing. They say unto him, We also go with thee. They went
forth, and entered into a ship immediately; and that night they caught nothing.

21:4 But when the morning was now come, Jesus stood on the shore: but the disciples knew not that it
was Jesus.

What hour of day did this event take place? Morning. Remember what happened in the morning
at the high priest’s place? Peter heard the cock crow 3 times.

21:5 Then Jesus saith unto them, Children, have ye any meat? They answered him, No.

21:6 And he said unto them, Cast the net on the right side of the ship, and ye shall find. They cast
therefore, and now they were not able to draw it for the multitude of fishes.

21:7 Therefore that disciple whom Jesus loved saith unto Peter, It is the Lord. Now when Simon Peter
heard that it was the Lord, he girt his fisher's coat unto him, (for he was naked,) and did cast himself into
the sea.

21:8 And the other disciples came in a little ship; (for they were not far from land, but as it were two
hundred cubits,) dragging the net with fishes.

21:9 As soon then as they were come to land, they saw a fire of coals there, and fish laid thereon, and
bread.

What else did Peter see when he came to the shore to see Jesus?

John 21:9

A fire of coals, fish, and bread. What events did this remind Peter of?

John 18:17-18

Matt 14:17

Fire of coals = Peter’s denial of Christ in front of fire of coals.

http://breachrepairers.webs.com/ 657
Fish and Bread = feeding of the 5,000.

21:10 Jesus saith unto them, Bring of the fish which ye have now caught.

21:11 Simon Peter went up, and drew the net to land full of great fishes, an hundred and fifty and three:
and for all there were so many, yet was not the net broken.

21:12 Jesus saith unto them, Come and dine. And none of the disciples durst ask him, Who art thou?
knowing that it was the Lord.

21:13 Jesus then cometh, and taketh bread, and giveth them, and fish likewise.

21:14 This is now the third time that Jesus shewed himself to his disciples, after that he was risen from
the dead.

21:15 So when they had dined, Jesus saith to Simon Peter, Simon, son of Jonas, lovest thou me more than
these? He saith unto him, Yea, Lord; thou knowest that I love thee. He saith unto him, Feed my lambs.

21:16 He saith to him again the second time, Simon, son of Jonas, lovest thou me? He saith unto him, Yea,
Lord; thou knowest that I love thee. He saith unto him, Feed my sheep.

21:17 He saith unto him the third time, Simon, son of Jonas, lovest thou me? Peter was grieved because
he said unto him the third time, Lovest thou me? And he said unto him, Lord, thou knowest all things;
thou knowest that I love thee. Jesus saith unto him, Feed my sheep.

21:18 Verily, verily, I say unto thee, When thou wast young, thou girdedst thyself, and walkedst whither
thou wouldest: but when thou shalt be old, thou shalt stretch forth thy hands, and another shall gird
thee, and carry thee whither thou wouldest not.

21:19 This spake he, signifying by what death he should glorify God. And when he had spoken this, he
saith unto him, Follow me.

21:20 Then Peter, turning about, seeth the disciple whom Jesus loved following; which also leaned on his
breast at supper, and said, Lord, which is he that betrayeth thee?

21:21 Peter seeing him saith to Jesus, Lord, and what shall this man do?

What was the condition of the fishing net after the great catch? How is this different to the
first time?

John 21:11

Luke 5:6

“yet was not the net broken” – The first time, the net broke.

Why wasn’t the net broken this time? What lessons can we learn?

Luke 5:5

This time, the disciples did not doubt. Peter lacked faith the first time. What lessons can we learn?
When we are fishing for men, if we doubt God or lack faith, our nets may break. We may be able to
catch some, but at the same time we will not be as successful as if we had fully trusted the Lord and

http://breachrepairers.webs.com/ 658
moved forward with complete faith.

What did Jesus say to Peter in the morning, after the miraculous catch, surrounded with
other disciples, eating fish and bread and in the front of the fire of coals? Why 3 times?

Feed my sheep. In the beginning Peter was called to catch fish, but now he is called to feed His
sheep. It is one thing to catch, but it is another thing to “feed”. NOTE: Jesus starts with feed my
lambs first. Why 3 times? To replace the 3 times that Peter had denied Christ.

21:22 Jesus saith unto him, If I will that he tarry till I come, what is that to thee? follow thou me.

21:23 Then went this saying abroad among the brethren, that that disciple should not die: yet Jesus said
not unto him, He shall not die; but, If I will that he tarry till I come, what is that to thee?

21:24 This is the disciple which testifieth of these things, and wrote these things: and we know that his
testimony is true.

Now, what event did Peter recall every time he saw a fire of coals?

Luke 24:22-24

The way Jesus restored Peter and proved his repentance in front of all the other disciples.

Why do you think Jesus asked Peter to feed sheep? Remember, what event was Peter
reminded of when Jesus said those words?

Mark 6:34

Jesus reminded Peter of the feeding of the 5,000. What important lesson was taught in feeding of
the 5,000? Feeding the sheep is not just about preaching and healing. Its also about taking care of
their physical needs.

What was the essential ingredient in order for Peter to feed Christ’s lambs? Clue: what did
Jesus keep asking Peter? Jesus asked Peter 3 times whether he loved Him. Love is the essential
ingredient for us to feed Christ’s lambs.

What memories was Jesus bringing to Peter in this encounter? What was Jesus trying to do
for Peter? Peter’s first call when Jesus called him to be a fisher of men. Peter’s denial of Christ.
Jesus’ feeding of the 5,000.

Summary – Jesus was transitioning Peter from being just a fisher of men to a shepherd. Before He
did that, as part of forgiving Peter, Jesus helped to change his guilty memories. Therefore, God’s
forgiveness is not just about cleansing us from our sins, but also changing our guilty, discouraging
memories to good memories.

Application questions: Do you have your “fire” experience with Jesus? Why don’t you have fire for
Jesus? Have you experienced the love of God personally as Peter did? Do you have memories that
you need replaced?

21:25 And there are also many other things which Jesus did, the which, if they should be written every
one, I suppose that even the world itself could not contain the books that should be written. Amen.

http://breachrepairers.webs.com/ 659

Você também pode gostar